]> code.communitydata.science - stats_class_2019.git/blobdiff - problem_sets/week_06/ps6-worked-solution.html
updates and corrections
[stats_class_2019.git] / problem_sets / week_06 / ps6-worked-solution.html
index 50075ed3b0d9cc26eb3259bf3cb63f2877e3f3a8..a0c9e4805bc5dd40f7e9c249e26e2f83819e49f4 100644 (file)
 
 <meta name="date" content="2019-04-11" />
 
-<title>Week 6 Worked Examples</title>
+<title>Week 6 problem set: Worked solutions</title>
 
-<script>/*! jQuery v1.11.3 | (c) 2005, 2015 jQuery Foundation, Inc. | jquery.org/license */
-!function(a,b){"object"==typeof module&&"object"==typeof module.exports?module.exports=a.document?b(a,!0):function(a){if(!a.document)throw new Error("jQuery requires a window with a document");return b(a)}:b(a)}("undefined"!=typeof window?window:this,function(a,b){var c=[],d=c.slice,e=c.concat,f=c.push,g=c.indexOf,h={},i=h.toString,j=h.hasOwnProperty,k={},l="1.11.3",m=function(a,b){return new m.fn.init(a,b)},n=/^[\s\uFEFF\xA0]+|[\s\uFEFF\xA0]+$/g,o=/^-ms-/,p=/-([\da-z])/gi,q=function(a,b){return b.toUpperCase()};m.fn=m.prototype={jquery:l,constructor:m,selector:"",length:0,toArray:function(){return d.call(this)},get:function(a){return null!=a?0>a?this[a+this.length]:this[a]:d.call(this)},pushStack:function(a){var b=m.merge(this.constructor(),a);return b.prevObject=this,b.context=this.context,b},each:function(a,b){return m.each(this,a,b)},map:function(a){return this.pushStack(m.map(this,function(b,c){return a.call(b,c,b)}))},slice:function(){return this.pushStack(d.apply(this,arguments))},first:function(){return this.eq(0)},last:function(){return this.eq(-1)},eq:function(a){var b=this.length,c=+a+(0>a?b:0);return this.pushStack(c>=0&&b>c?[this[c]]:[])},end:function(){return this.prevObject||this.constructor(null)},push:f,sort:c.sort,splice:c.splice},m.extend=m.fn.extend=function(){var a,b,c,d,e,f,g=arguments[0]||{},h=1,i=arguments.length,j=!1;for("boolean"==typeof g&&(j=g,g=arguments[h]||{},h++),"object"==typeof g||m.isFunction(g)||(g={}),h===i&&(g=this,h--);i>h;h++)if(null!=(e=arguments[h]))for(d in e)a=g[d],c=e[d],g!==c&&(j&&c&&(m.isPlainObject(c)||(b=m.isArray(c)))?(b?(b=!1,f=a&&m.isArray(a)?a:[]):f=a&&m.isPlainObject(a)?a:{},g[d]=m.extend(j,f,c)):void 0!==c&&(g[d]=c));return g},m.extend({expando:"jQuery"+(l+Math.random()).replace(/\D/g,""),isReady:!0,error:function(a){throw new Error(a)},noop:function(){},isFunction:function(a){return"function"===m.type(a)},isArray:Array.isArray||function(a){return"array"===m.type(a)},isWindow:function(a){return null!=a&&a==a.window},isNumeric:function(a){return!m.isArray(a)&&a-parseFloat(a)+1>=0},isEmptyObject:function(a){var b;for(b in a)return!1;return!0},isPlainObject:function(a){var b;if(!a||"object"!==m.type(a)||a.nodeType||m.isWindow(a))return!1;try{if(a.constructor&&!j.call(a,"constructor")&&!j.call(a.constructor.prototype,"isPrototypeOf"))return!1}catch(c){return!1}if(k.ownLast)for(b in a)return j.call(a,b);for(b in a);return void 0===b||j.call(a,b)},type:function(a){return null==a?a+"":"object"==typeof a||"function"==typeof a?h[i.call(a)]||"object":typeof a},globalEval:function(b){b&&m.trim(b)&&(a.execScript||function(b){a.eval.call(a,b)})(b)},camelCase:function(a){return a.replace(o,"ms-").replace(p,q)},nodeName:function(a,b){return a.nodeName&&a.nodeName.toLowerCase()===b.toLowerCase()},each:function(a,b,c){var d,e=0,f=a.length,g=r(a);if(c){if(g){for(;f>e;e++)if(d=b.apply(a[e],c),d===!1)break}else for(e in a)if(d=b.apply(a[e],c),d===!1)break}else if(g){for(;f>e;e++)if(d=b.call(a[e],e,a[e]),d===!1)break}else for(e in a)if(d=b.call(a[e],e,a[e]),d===!1)break;return a},trim:function(a){return null==a?"":(a+"").replace(n,"")},makeArray:function(a,b){var c=b||[];return null!=a&&(r(Object(a))?m.merge(c,"string"==typeof a?[a]:a):f.call(c,a)),c},inArray:function(a,b,c){var d;if(b){if(g)return g.call(b,a,c);for(d=b.length,c=c?0>c?Math.max(0,d+c):c:0;d>c;c++)if(c in b&&b[c]===a)return c}return-1},merge:function(a,b){var c=+b.length,d=0,e=a.length;while(c>d)a[e++]=b[d++];if(c!==c)while(void 0!==b[d])a[e++]=b[d++];return a.length=e,a},grep:function(a,b,c){for(var d,e=[],f=0,g=a.length,h=!c;g>f;f++)d=!b(a[f],f),d!==h&&e.push(a[f]);return e},map:function(a,b,c){var d,f=0,g=a.length,h=r(a),i=[];if(h)for(;g>f;f++)d=b(a[f],f,c),null!=d&&i.push(d);else for(f in a)d=b(a[f],f,c),null!=d&&i.push(d);return e.apply([],i)},guid:1,proxy:function(a,b){var c,e,f;return"string"==typeof b&&(f=a[b],b=a,a=f),m.isFunction(a)?(c=d.call(arguments,2),e=function(){return a.apply(b||this,c.concat(d.call(arguments)))},e.guid=a.guid=a.guid||m.guid++,e):void 0},now:function(){return+new Date},support:k}),m.each("Boolean Number String Function Array Date RegExp Object Error".split(" "),function(a,b){h["[object "+b+"]"]=b.toLowerCase()});function r(a){var b="length"in a&&a.length,c=m.type(a);return"function"===c||m.isWindow(a)?!1:1===a.nodeType&&b?!0:"array"===c||0===b||"number"==typeof b&&b>0&&b-1 in a}var s=function(a){var b,c,d,e,f,g,h,i,j,k,l,m,n,o,p,q,r,s,t,u="sizzle"+1*new Date,v=a.document,w=0,x=0,y=ha(),z=ha(),A=ha(),B=function(a,b){return a===b&&(l=!0),0},C=1<<31,D={}.hasOwnProperty,E=[],F=E.pop,G=E.push,H=E.push,I=E.slice,J=function(a,b){for(var c=0,d=a.length;d>c;c++)if(a[c]===b)return c;return-1},K="checked|selected|async|autofocus|autoplay|controls|defer|disabled|hidden|ismap|loop|multiple|open|readonly|required|scoped",L="[\\x20\\t\\r\\n\\f]",M="(?:\\\\.|[\\w-]|[^\\x00-\\xa0])+",N=M.replace("w","w#"),O="\\["+L+"*("+M+")(?:"+L+"*([*^$|!~]?=)"+L+"*(?:'((?:\\\\.|[^\\\\'])*)'|\"((?:\\\\.|[^\\\\\"])*)\"|("+N+"))|)"+L+"*\\]",P=":("+M+")(?:\\((('((?:\\\\.|[^\\\\'])*)'|\"((?:\\\\.|[^\\\\\"])*)\")|((?:\\\\.|[^\\\\()[\\]]|"+O+")*)|.*)\\)|)",Q=new RegExp(L+"+","g"),R=new RegExp("^"+L+"+|((?:^|[^\\\\])(?:\\\\.)*)"+L+"+$","g"),S=new RegExp("^"+L+"*,"+L+"*"),T=new RegExp("^"+L+"*([>+~]|"+L+")"+L+"*"),U=new RegExp("="+L+"*([^\\]'\"]*?)"+L+"*\\]","g"),V=new RegExp(P),W=new RegExp("^"+N+"$"),X={ID:new RegExp("^#("+M+")"),CLASS:new RegExp("^\\.("+M+")"),TAG:new RegExp("^("+M.replace("w","w*")+")"),ATTR:new RegExp("^"+O),PSEUDO:new RegExp("^"+P),CHILD:new RegExp("^:(only|first|last|nth|nth-last)-(child|of-type)(?:\\("+L+"*(even|odd|(([+-]|)(\\d*)n|)"+L+"*(?:([+-]|)"+L+"*(\\d+)|))"+L+"*\\)|)","i"),bool:new RegExp("^(?:"+K+")$","i"),needsContext:new RegExp("^"+L+"*[>+~]|:(even|odd|eq|gt|lt|nth|first|last)(?:\\("+L+"*((?:-\\d)?\\d*)"+L+"*\\)|)(?=[^-]|$)","i")},Y=/^(?:input|select|textarea|button)$/i,Z=/^h\d$/i,$=/^[^{]+\{\s*\[native \w/,_=/^(?:#([\w-]+)|(\w+)|\.([\w-]+))$/,aa=/[+~]/,ba=/'|\\/g,ca=new RegExp("\\\\([\\da-f]{1,6}"+L+"?|("+L+")|.)","ig"),da=function(a,b,c){var d="0x"+b-65536;return d!==d||c?b:0>d?String.fromCharCode(d+65536):String.fromCharCode(d>>10|55296,1023&d|56320)},ea=function(){m()};try{H.apply(E=I.call(v.childNodes),v.childNodes),E[v.childNodes.length].nodeType}catch(fa){H={apply:E.length?function(a,b){G.apply(a,I.call(b))}:function(a,b){var c=a.length,d=0;while(a[c++]=b[d++]);a.length=c-1}}}function ga(a,b,d,e){var f,h,j,k,l,o,r,s,w,x;if((b?b.ownerDocument||b:v)!==n&&m(b),b=b||n,d=d||[],k=b.nodeType,"string"!=typeof a||!a||1!==k&&9!==k&&11!==k)return d;if(!e&&p){if(11!==k&&(f=_.exec(a)))if(j=f[1]){if(9===k){if(h=b.getElementById(j),!h||!h.parentNode)return d;if(h.id===j)return d.push(h),d}else if(b.ownerDocument&&(h=b.ownerDocument.getElementById(j))&&t(b,h)&&h.id===j)return d.push(h),d}else{if(f[2])return H.apply(d,b.getElementsByTagName(a)),d;if((j=f[3])&&c.getElementsByClassName)return H.apply(d,b.getElementsByClassName(j)),d}if(c.qsa&&(!q||!q.test(a))){if(s=r=u,w=b,x=1!==k&&a,1===k&&"object"!==b.nodeName.toLowerCase()){o=g(a),(r=b.getAttribute("id"))?s=r.replace(ba,"\\$&"):b.setAttribute("id",s),s="[id='"+s+"'] ",l=o.length;while(l--)o[l]=s+ra(o[l]);w=aa.test(a)&&pa(b.parentNode)||b,x=o.join(",")}if(x)try{return H.apply(d,w.querySelectorAll(x)),d}catch(y){}finally{r||b.removeAttribute("id")}}}return i(a.replace(R,"$1"),b,d,e)}function ha(){var a=[];function b(c,e){return a.push(c+" ")>d.cacheLength&&delete b[a.shift()],b[c+" "]=e}return b}function ia(a){return a[u]=!0,a}function ja(a){var b=n.createElement("div");try{return!!a(b)}catch(c){return!1}finally{b.parentNode&&b.parentNode.removeChild(b),b=null}}function ka(a,b){var c=a.split("|"),e=a.length;while(e--)d.attrHandle[c[e]]=b}function la(a,b){var c=b&&a,d=c&&1===a.nodeType&&1===b.nodeType&&(~b.sourceIndex||C)-(~a.sourceIndex||C);if(d)return d;if(c)while(c=c.nextSibling)if(c===b)return-1;return a?1:-1}function ma(a){return function(b){var c=b.nodeName.toLowerCase();return"input"===c&&b.type===a}}function na(a){return function(b){var c=b.nodeName.toLowerCase();return("input"===c||"button"===c)&&b.type===a}}function oa(a){return ia(function(b){return b=+b,ia(function(c,d){var e,f=a([],c.length,b),g=f.length;while(g--)c[e=f[g]]&&(c[e]=!(d[e]=c[e]))})})}function pa(a){return a&&"undefined"!=typeof a.getElementsByTagName&&a}c=ga.support={},f=ga.isXML=function(a){var b=a&&(a.ownerDocument||a).documentElement;return b?"HTML"!==b.nodeName:!1},m=ga.setDocument=function(a){var b,e,g=a?a.ownerDocument||a:v;return g!==n&&9===g.nodeType&&g.documentElement?(n=g,o=g.documentElement,e=g.defaultView,e&&e!==e.top&&(e.addEventListener?e.addEventListener("unload",ea,!1):e.attachEvent&&e.attachEvent("onunload",ea)),p=!f(g),c.attributes=ja(function(a){return a.className="i",!a.getAttribute("className")}),c.getElementsByTagName=ja(function(a){return a.appendChild(g.createComment("")),!a.getElementsByTagName("*").length}),c.getElementsByClassName=$.test(g.getElementsByClassName),c.getById=ja(function(a){return o.appendChild(a).id=u,!g.getElementsByName||!g.getElementsByName(u).length}),c.getById?(d.find.ID=function(a,b){if("undefined"!=typeof b.getElementById&&p){var c=b.getElementById(a);return c&&c.parentNode?[c]:[]}},d.filter.ID=function(a){var b=a.replace(ca,da);return function(a){return a.getAttribute("id")===b}}):(delete d.find.ID,d.filter.ID=function(a){var b=a.replace(ca,da);return function(a){var c="undefined"!=typeof a.getAttributeNode&&a.getAttributeNode("id");return c&&c.value===b}}),d.find.TAG=c.getElementsByTagName?function(a,b){return"undefined"!=typeof b.getElementsByTagName?b.getElementsByTagName(a):c.qsa?b.querySelectorAll(a):void 0}:function(a,b){var c,d=[],e=0,f=b.getElementsByTagName(a);if("*"===a){while(c=f[e++])1===c.nodeType&&d.push(c);return d}return f},d.find.CLASS=c.getElementsByClassName&&function(a,b){return p?b.getElementsByClassName(a):void 0},r=[],q=[],(c.qsa=$.test(g.querySelectorAll))&&(ja(function(a){o.appendChild(a).innerHTML="<a id='"+u+"'></a><select id='"+u+"-\f]' msallowcapture=''><option selected=''></option></select>",a.querySelectorAll("[msallowcapture^='']").length&&q.push("[*^$]="+L+"*(?:''|\"\")"),a.querySelectorAll("[selected]").length||q.push("\\["+L+"*(?:value|"+K+")"),a.querySelectorAll("[id~="+u+"-]").length||q.push("~="),a.querySelectorAll(":checked").length||q.push(":checked"),a.querySelectorAll("a#"+u+"+*").length||q.push(".#.+[+~]")}),ja(function(a){var b=g.createElement("input");b.setAttribute("type","hidden"),a.appendChild(b).setAttribute("name","D"),a.querySelectorAll("[name=d]").length&&q.push("name"+L+"*[*^$|!~]?="),a.querySelectorAll(":enabled").length||q.push(":enabled",":disabled"),a.querySelectorAll("*,:x"),q.push(",.*:")})),(c.matchesSelector=$.test(s=o.matches||o.webkitMatchesSelector||o.mozMatchesSelector||o.oMatchesSelector||o.msMatchesSelector))&&ja(function(a){c.disconnectedMatch=s.call(a,"div"),s.call(a,"[s!='']:x"),r.push("!=",P)}),q=q.length&&new RegExp(q.join("|")),r=r.length&&new RegExp(r.join("|")),b=$.test(o.compareDocumentPosition),t=b||$.test(o.contains)?function(a,b){var c=9===a.nodeType?a.documentElement:a,d=b&&b.parentNode;return a===d||!(!d||1!==d.nodeType||!(c.contains?c.contains(d):a.compareDocumentPosition&&16&a.compareDocumentPosition(d)))}:function(a,b){if(b)while(b=b.parentNode)if(b===a)return!0;return!1},B=b?function(a,b){if(a===b)return l=!0,0;var d=!a.compareDocumentPosition-!b.compareDocumentPosition;return d?d:(d=(a.ownerDocument||a)===(b.ownerDocument||b)?a.compareDocumentPosition(b):1,1&d||!c.sortDetached&&b.compareDocumentPosition(a)===d?a===g||a.ownerDocument===v&&t(v,a)?-1:b===g||b.ownerDocument===v&&t(v,b)?1:k?J(k,a)-J(k,b):0:4&d?-1:1)}:function(a,b){if(a===b)return l=!0,0;var c,d=0,e=a.parentNode,f=b.parentNode,h=[a],i=[b];if(!e||!f)return a===g?-1:b===g?1:e?-1:f?1:k?J(k,a)-J(k,b):0;if(e===f)return la(a,b);c=a;while(c=c.parentNode)h.unshift(c);c=b;while(c=c.parentNode)i.unshift(c);while(h[d]===i[d])d++;return d?la(h[d],i[d]):h[d]===v?-1:i[d]===v?1:0},g):n},ga.matches=function(a,b){return ga(a,null,null,b)},ga.matchesSelector=function(a,b){if((a.ownerDocument||a)!==n&&m(a),b=b.replace(U,"='$1']"),!(!c.matchesSelector||!p||r&&r.test(b)||q&&q.test(b)))try{var d=s.call(a,b);if(d||c.disconnectedMatch||a.document&&11!==a.document.nodeType)return d}catch(e){}return ga(b,n,null,[a]).length>0},ga.contains=function(a,b){return(a.ownerDocument||a)!==n&&m(a),t(a,b)},ga.attr=function(a,b){(a.ownerDocument||a)!==n&&m(a);var e=d.attrHandle[b.toLowerCase()],f=e&&D.call(d.attrHandle,b.toLowerCase())?e(a,b,!p):void 0;return void 0!==f?f:c.attributes||!p?a.getAttribute(b):(f=a.getAttributeNode(b))&&f.specified?f.value:null},ga.error=function(a){throw new Error("Syntax error, unrecognized expression: "+a)},ga.uniqueSort=function(a){var b,d=[],e=0,f=0;if(l=!c.detectDuplicates,k=!c.sortStable&&a.slice(0),a.sort(B),l){while(b=a[f++])b===a[f]&&(e=d.push(f));while(e--)a.splice(d[e],1)}return k=null,a},e=ga.getText=function(a){var b,c="",d=0,f=a.nodeType;if(f){if(1===f||9===f||11===f){if("string"==typeof a.textContent)return a.textContent;for(a=a.firstChild;a;a=a.nextSibling)c+=e(a)}else if(3===f||4===f)return a.nodeValue}else while(b=a[d++])c+=e(b);return c},d=ga.selectors={cacheLength:50,createPseudo:ia,match:X,attrHandle:{},find:{},relative:{">":{dir:"parentNode",first:!0}," ":{dir:"parentNode"},"+":{dir:"previousSibling",first:!0},"~":{dir:"previousSibling"}},preFilter:{ATTR:function(a){return a[1]=a[1].replace(ca,da),a[3]=(a[3]||a[4]||a[5]||"").replace(ca,da),"~="===a[2]&&(a[3]=" "+a[3]+" "),a.slice(0,4)},CHILD:function(a){return a[1]=a[1].toLowerCase(),"nth"===a[1].slice(0,3)?(a[3]||ga.error(a[0]),a[4]=+(a[4]?a[5]+(a[6]||1):2*("even"===a[3]||"odd"===a[3])),a[5]=+(a[7]+a[8]||"odd"===a[3])):a[3]&&ga.error(a[0]),a},PSEUDO:function(a){var b,c=!a[6]&&a[2];return X.CHILD.test(a[0])?null:(a[3]?a[2]=a[4]||a[5]||"":c&&V.test(c)&&(b=g(c,!0))&&(b=c.indexOf(")",c.length-b)-c.length)&&(a[0]=a[0].slice(0,b),a[2]=c.slice(0,b)),a.slice(0,3))}},filter:{TAG:function(a){var b=a.replace(ca,da).toLowerCase();return"*"===a?function(){return!0}:function(a){return a.nodeName&&a.nodeName.toLowerCase()===b}},CLASS:function(a){var b=y[a+" "];return b||(b=new RegExp("(^|"+L+")"+a+"("+L+"|$)"))&&y(a,function(a){return b.test("string"==typeof a.className&&a.className||"undefined"!=typeof a.getAttribute&&a.getAttribute("class")||"")})},ATTR:function(a,b,c){return function(d){var e=ga.attr(d,a);return null==e?"!="===b:b?(e+="","="===b?e===c:"!="===b?e!==c:"^="===b?c&&0===e.indexOf(c):"*="===b?c&&e.indexOf(c)>-1:"$="===b?c&&e.slice(-c.length)===c:"~="===b?(" "+e.replace(Q," ")+" ").indexOf(c)>-1:"|="===b?e===c||e.slice(0,c.length+1)===c+"-":!1):!0}},CHILD:function(a,b,c,d,e){var f="nth"!==a.slice(0,3),g="last"!==a.slice(-4),h="of-type"===b;return 1===d&&0===e?function(a){return!!a.parentNode}:function(b,c,i){var j,k,l,m,n,o,p=f!==g?"nextSibling":"previousSibling",q=b.parentNode,r=h&&b.nodeName.toLowerCase(),s=!i&&!h;if(q){if(f){while(p){l=b;while(l=l[p])if(h?l.nodeName.toLowerCase()===r:1===l.nodeType)return!1;o=p="only"===a&&!o&&"nextSibling"}return!0}if(o=[g?q.firstChild:q.lastChild],g&&s){k=q[u]||(q[u]={}),j=k[a]||[],n=j[0]===w&&j[1],m=j[0]===w&&j[2],l=n&&q.childNodes[n];while(l=++n&&l&&l[p]||(m=n=0)||o.pop())if(1===l.nodeType&&++m&&l===b){k[a]=[w,n,m];break}}else if(s&&(j=(b[u]||(b[u]={}))[a])&&j[0]===w)m=j[1];else while(l=++n&&l&&l[p]||(m=n=0)||o.pop())if((h?l.nodeName.toLowerCase()===r:1===l.nodeType)&&++m&&(s&&((l[u]||(l[u]={}))[a]=[w,m]),l===b))break;return m-=e,m===d||m%d===0&&m/d>=0}}},PSEUDO:function(a,b){var c,e=d.pseudos[a]||d.setFilters[a.toLowerCase()]||ga.error("unsupported pseudo: "+a);return e[u]?e(b):e.length>1?(c=[a,a,"",b],d.setFilters.hasOwnProperty(a.toLowerCase())?ia(function(a,c){var d,f=e(a,b),g=f.length;while(g--)d=J(a,f[g]),a[d]=!(c[d]=f[g])}):function(a){return e(a,0,c)}):e}},pseudos:{not:ia(function(a){var b=[],c=[],d=h(a.replace(R,"$1"));return d[u]?ia(function(a,b,c,e){var f,g=d(a,null,e,[]),h=a.length;while(h--)(f=g[h])&&(a[h]=!(b[h]=f))}):function(a,e,f){return b[0]=a,d(b,null,f,c),b[0]=null,!c.pop()}}),has:ia(function(a){return function(b){return ga(a,b).length>0}}),contains:ia(function(a){return a=a.replace(ca,da),function(b){return(b.textContent||b.innerText||e(b)).indexOf(a)>-1}}),lang:ia(function(a){return W.test(a||"")||ga.error("unsupported lang: "+a),a=a.replace(ca,da).toLowerCase(),function(b){var c;do if(c=p?b.lang:b.getAttribute("xml:lang")||b.getAttribute("lang"))return c=c.toLowerCase(),c===a||0===c.indexOf(a+"-");while((b=b.parentNode)&&1===b.nodeType);return!1}}),target:function(b){var c=a.location&&a.location.hash;return c&&c.slice(1)===b.id},root:function(a){return a===o},focus:function(a){return a===n.activeElement&&(!n.hasFocus||n.hasFocus())&&!!(a.type||a.href||~a.tabIndex)},enabled:function(a){return a.disabled===!1},disabled:function(a){return a.disabled===!0},checked:function(a){var b=a.nodeName.toLowerCase();return"input"===b&&!!a.checked||"option"===b&&!!a.selected},selected:function(a){return a.parentNode&&a.parentNode.selectedIndex,a.selected===!0},empty:function(a){for(a=a.firstChild;a;a=a.nextSibling)if(a.nodeType<6)return!1;return!0},parent:function(a){return!d.pseudos.empty(a)},header:function(a){return Z.test(a.nodeName)},input:function(a){return Y.test(a.nodeName)},button:function(a){var b=a.nodeName.toLowerCase();return"input"===b&&"button"===a.type||"button"===b},text:function(a){var b;return"input"===a.nodeName.toLowerCase()&&"text"===a.type&&(null==(b=a.getAttribute("type"))||"text"===b.toLowerCase())},first:oa(function(){return[0]}),last:oa(function(a,b){return[b-1]}),eq:oa(function(a,b,c){return[0>c?c+b:c]}),even:oa(function(a,b){for(var c=0;b>c;c+=2)a.push(c);return a}),odd:oa(function(a,b){for(var c=1;b>c;c+=2)a.push(c);return a}),lt:oa(function(a,b,c){for(var d=0>c?c+b:c;--d>=0;)a.push(d);return a}),gt:oa(function(a,b,c){for(var d=0>c?c+b:c;++d<b;)a.push(d);return a})}},d.pseudos.nth=d.pseudos.eq;for(b in{radio:!0,checkbox:!0,file:!0,password:!0,image:!0})d.pseudos[b]=ma(b);for(b in{submit:!0,reset:!0})d.pseudos[b]=na(b);function qa(){}qa.prototype=d.filters=d.pseudos,d.setFilters=new qa,g=ga.tokenize=function(a,b){var c,e,f,g,h,i,j,k=z[a+" "];if(k)return b?0:k.slice(0);h=a,i=[],j=d.preFilter;while(h){(!c||(e=S.exec(h)))&&(e&&(h=h.slice(e[0].length)||h),i.push(f=[])),c=!1,(e=T.exec(h))&&(c=e.shift(),f.push({value:c,type:e[0].replace(R," ")}),h=h.slice(c.length));for(g in d.filter)!(e=X[g].exec(h))||j[g]&&!(e=j[g](e))||(c=e.shift(),f.push({value:c,type:g,matches:e}),h=h.slice(c.length));if(!c)break}return b?h.length:h?ga.error(a):z(a,i).slice(0)};function ra(a){for(var b=0,c=a.length,d="";c>b;b++)d+=a[b].value;return d}function sa(a,b,c){var d=b.dir,e=c&&"parentNode"===d,f=x++;return b.first?function(b,c,f){while(b=b[d])if(1===b.nodeType||e)return a(b,c,f)}:function(b,c,g){var h,i,j=[w,f];if(g){while(b=b[d])if((1===b.nodeType||e)&&a(b,c,g))return!0}else while(b=b[d])if(1===b.nodeType||e){if(i=b[u]||(b[u]={}),(h=i[d])&&h[0]===w&&h[1]===f)return j[2]=h[2];if(i[d]=j,j[2]=a(b,c,g))return!0}}}function ta(a){return a.length>1?function(b,c,d){var e=a.length;while(e--)if(!a[e](b,c,d))return!1;return!0}:a[0]}function ua(a,b,c){for(var d=0,e=b.length;e>d;d++)ga(a,b[d],c);return c}function va(a,b,c,d,e){for(var f,g=[],h=0,i=a.length,j=null!=b;i>h;h++)(f=a[h])&&(!c||c(f,d,e))&&(g.push(f),j&&b.push(h));return g}function wa(a,b,c,d,e,f){return d&&!d[u]&&(d=wa(d)),e&&!e[u]&&(e=wa(e,f)),ia(function(f,g,h,i){var j,k,l,m=[],n=[],o=g.length,p=f||ua(b||"*",h.nodeType?[h]:h,[]),q=!a||!f&&b?p:va(p,m,a,h,i),r=c?e||(f?a:o||d)?[]:g:q;if(c&&c(q,r,h,i),d){j=va(r,n),d(j,[],h,i),k=j.length;while(k--)(l=j[k])&&(r[n[k]]=!(q[n[k]]=l))}if(f){if(e||a){if(e){j=[],k=r.length;while(k--)(l=r[k])&&j.push(q[k]=l);e(null,r=[],j,i)}k=r.length;while(k--)(l=r[k])&&(j=e?J(f,l):m[k])>-1&&(f[j]=!(g[j]=l))}}else r=va(r===g?r.splice(o,r.length):r),e?e(null,g,r,i):H.apply(g,r)})}function xa(a){for(var b,c,e,f=a.length,g=d.relative[a[0].type],h=g||d.relative[" "],i=g?1:0,k=sa(function(a){return a===b},h,!0),l=sa(function(a){return J(b,a)>-1},h,!0),m=[function(a,c,d){var e=!g&&(d||c!==j)||((b=c).nodeType?k(a,c,d):l(a,c,d));return b=null,e}];f>i;i++)if(c=d.relative[a[i].type])m=[sa(ta(m),c)];else{if(c=d.filter[a[i].type].apply(null,a[i].matches),c[u]){for(e=++i;f>e;e++)if(d.relative[a[e].type])break;return wa(i>1&&ta(m),i>1&&ra(a.slice(0,i-1).concat({value:" "===a[i-2].type?"*":""})).replace(R,"$1"),c,e>i&&xa(a.slice(i,e)),f>e&&xa(a=a.slice(e)),f>e&&ra(a))}m.push(c)}return ta(m)}function ya(a,b){var c=b.length>0,e=a.length>0,f=function(f,g,h,i,k){var l,m,o,p=0,q="0",r=f&&[],s=[],t=j,u=f||e&&d.find.TAG("*",k),v=w+=null==t?1:Math.random()||.1,x=u.length;for(k&&(j=g!==n&&g);q!==x&&null!=(l=u[q]);q++){if(e&&l){m=0;while(o=a[m++])if(o(l,g,h)){i.push(l);break}k&&(w=v)}c&&((l=!o&&l)&&p--,f&&r.push(l))}if(p+=q,c&&q!==p){m=0;while(o=b[m++])o(r,s,g,h);if(f){if(p>0)while(q--)r[q]||s[q]||(s[q]=F.call(i));s=va(s)}H.apply(i,s),k&&!f&&s.length>0&&p+b.length>1&&ga.uniqueSort(i)}return k&&(w=v,j=t),r};return c?ia(f):f}return h=ga.compile=function(a,b){var c,d=[],e=[],f=A[a+" "];if(!f){b||(b=g(a)),c=b.length;while(c--)f=xa(b[c]),f[u]?d.push(f):e.push(f);f=A(a,ya(e,d)),f.selector=a}return f},i=ga.select=function(a,b,e,f){var i,j,k,l,m,n="function"==typeof a&&a,o=!f&&g(a=n.selector||a);if(e=e||[],1===o.length){if(j=o[0]=o[0].slice(0),j.length>2&&"ID"===(k=j[0]).type&&c.getById&&9===b.nodeType&&p&&d.relative[j[1].type]){if(b=(d.find.ID(k.matches[0].replace(ca,da),b)||[])[0],!b)return e;n&&(b=b.parentNode),a=a.slice(j.shift().value.length)}i=X.needsContext.test(a)?0:j.length;while(i--){if(k=j[i],d.relative[l=k.type])break;if((m=d.find[l])&&(f=m(k.matches[0].replace(ca,da),aa.test(j[0].type)&&pa(b.parentNode)||b))){if(j.splice(i,1),a=f.length&&ra(j),!a)return H.apply(e,f),e;break}}}return(n||h(a,o))(f,b,!p,e,aa.test(a)&&pa(b.parentNode)||b),e},c.sortStable=u.split("").sort(B).join("")===u,c.detectDuplicates=!!l,m(),c.sortDetached=ja(function(a){return 1&a.compareDocumentPosition(n.createElement("div"))}),ja(function(a){return a.innerHTML="<a href='#'></a>","#"===a.firstChild.getAttribute("href")})||ka("type|href|height|width",function(a,b,c){return c?void 0:a.getAttribute(b,"type"===b.toLowerCase()?1:2)}),c.attributes&&ja(function(a){return a.innerHTML="<input/>",a.firstChild.setAttribute("value",""),""===a.firstChild.getAttribute("value")})||ka("value",function(a,b,c){return c||"input"!==a.nodeName.toLowerCase()?void 0:a.defaultValue}),ja(function(a){return null==a.getAttribute("disabled")})||ka(K,function(a,b,c){var d;return c?void 0:a[b]===!0?b.toLowerCase():(d=a.getAttributeNode(b))&&d.specified?d.value:null}),ga}(a);m.find=s,m.expr=s.selectors,m.expr[":"]=m.expr.pseudos,m.unique=s.uniqueSort,m.text=s.getText,m.isXMLDoc=s.isXML,m.contains=s.contains;var t=m.expr.match.needsContext,u=/^<(\w+)\s*\/?>(?:<\/\1>|)$/,v=/^.[^:#\[\.,]*$/;function w(a,b,c){if(m.isFunction(b))return m.grep(a,function(a,d){return!!b.call(a,d,a)!==c});if(b.nodeType)return m.grep(a,function(a){return a===b!==c});if("string"==typeof b){if(v.test(b))return m.filter(b,a,c);b=m.filter(b,a)}return m.grep(a,function(a){return m.inArray(a,b)>=0!==c})}m.filter=function(a,b,c){var d=b[0];return c&&(a=":not("+a+")"),1===b.length&&1===d.nodeType?m.find.matchesSelector(d,a)?[d]:[]:m.find.matches(a,m.grep(b,function(a){return 1===a.nodeType}))},m.fn.extend({find:function(a){var b,c=[],d=this,e=d.length;if("string"!=typeof a)return this.pushStack(m(a).filter(function(){for(b=0;e>b;b++)if(m.contains(d[b],this))return!0}));for(b=0;e>b;b++)m.find(a,d[b],c);return c=this.pushStack(e>1?m.unique(c):c),c.selector=this.selector?this.selector+" "+a:a,c},filter:function(a){return this.pushStack(w(this,a||[],!1))},not:function(a){return this.pushStack(w(this,a||[],!0))},is:function(a){return!!w(this,"string"==typeof a&&t.test(a)?m(a):a||[],!1).length}});var x,y=a.document,z=/^(?:\s*(<[\w\W]+>)[^>]*|#([\w-]*))$/,A=m.fn.init=function(a,b){var c,d;if(!a)return this;if("string"==typeof a){if(c="<"===a.charAt(0)&&">"===a.charAt(a.length-1)&&a.length>=3?[null,a,null]:z.exec(a),!c||!c[1]&&b)return!b||b.jquery?(b||x).find(a):this.constructor(b).find(a);if(c[1]){if(b=b instanceof m?b[0]:b,m.merge(this,m.parseHTML(c[1],b&&b.nodeType?b.ownerDocument||b:y,!0)),u.test(c[1])&&m.isPlainObject(b))for(c in b)m.isFunction(this[c])?this[c](b[c]):this.attr(c,b[c]);return this}if(d=y.getElementById(c[2]),d&&d.parentNode){if(d.id!==c[2])return x.find(a);this.length=1,this[0]=d}return this.context=y,this.selector=a,this}return a.nodeType?(this.context=this[0]=a,this.length=1,this):m.isFunction(a)?"undefined"!=typeof x.ready?x.ready(a):a(m):(void 0!==a.selector&&(this.selector=a.selector,this.context=a.context),m.makeArray(a,this))};A.prototype=m.fn,x=m(y);var B=/^(?:parents|prev(?:Until|All))/,C={children:!0,contents:!0,next:!0,prev:!0};m.extend({dir:function(a,b,c){var d=[],e=a[b];while(e&&9!==e.nodeType&&(void 0===c||1!==e.nodeType||!m(e).is(c)))1===e.nodeType&&d.push(e),e=e[b];return d},sibling:function(a,b){for(var c=[];a;a=a.nextSibling)1===a.nodeType&&a!==b&&c.push(a);return c}}),m.fn.extend({has:function(a){var b,c=m(a,this),d=c.length;return this.filter(function(){for(b=0;d>b;b++)if(m.contains(this,c[b]))return!0})},closest:function(a,b){for(var c,d=0,e=this.length,f=[],g=t.test(a)||"string"!=typeof a?m(a,b||this.context):0;e>d;d++)for(c=this[d];c&&c!==b;c=c.parentNode)if(c.nodeType<11&&(g?g.index(c)>-1:1===c.nodeType&&m.find.matchesSelector(c,a))){f.push(c);break}return this.pushStack(f.length>1?m.unique(f):f)},index:function(a){return a?"string"==typeof a?m.inArray(this[0],m(a)):m.inArray(a.jquery?a[0]:a,this):this[0]&&this[0].parentNode?this.first().prevAll().length:-1},add:function(a,b){return this.pushStack(m.unique(m.merge(this.get(),m(a,b))))},addBack:function(a){return this.add(null==a?this.prevObject:this.prevObject.filter(a))}});function D(a,b){do a=a[b];while(a&&1!==a.nodeType);return a}m.each({parent:function(a){var b=a.parentNode;return b&&11!==b.nodeType?b:null},parents:function(a){return m.dir(a,"parentNode")},parentsUntil:function(a,b,c){return m.dir(a,"parentNode",c)},next:function(a){return D(a,"nextSibling")},prev:function(a){return D(a,"previousSibling")},nextAll:function(a){return m.dir(a,"nextSibling")},prevAll:function(a){return m.dir(a,"previousSibling")},nextUntil:function(a,b,c){return m.dir(a,"nextSibling",c)},prevUntil:function(a,b,c){return m.dir(a,"previousSibling",c)},siblings:function(a){return m.sibling((a.parentNode||{}).firstChild,a)},children:function(a){return m.sibling(a.firstChild)},contents:function(a){return m.nodeName(a,"iframe")?a.contentDocument||a.contentWindow.document:m.merge([],a.childNodes)}},function(a,b){m.fn[a]=function(c,d){var e=m.map(this,b,c);return"Until"!==a.slice(-5)&&(d=c),d&&"string"==typeof d&&(e=m.filter(d,e)),this.length>1&&(C[a]||(e=m.unique(e)),B.test(a)&&(e=e.reverse())),this.pushStack(e)}});var E=/\S+/g,F={};function G(a){var b=F[a]={};return m.each(a.match(E)||[],function(a,c){b[c]=!0}),b}m.Callbacks=function(a){a="string"==typeof a?F[a]||G(a):m.extend({},a);var b,c,d,e,f,g,h=[],i=!a.once&&[],j=function(l){for(c=a.memory&&l,d=!0,f=g||0,g=0,e=h.length,b=!0;h&&e>f;f++)if(h[f].apply(l[0],l[1])===!1&&a.stopOnFalse){c=!1;break}b=!1,h&&(i?i.length&&j(i.shift()):c?h=[]:k.disable())},k={add:function(){if(h){var d=h.length;!function f(b){m.each(b,function(b,c){var d=m.type(c);"function"===d?a.unique&&k.has(c)||h.push(c):c&&c.length&&"string"!==d&&f(c)})}(arguments),b?e=h.length:c&&(g=d,j(c))}return this},remove:function(){return h&&m.each(arguments,function(a,c){var d;while((d=m.inArray(c,h,d))>-1)h.splice(d,1),b&&(e>=d&&e--,f>=d&&f--)}),this},has:function(a){return a?m.inArray(a,h)>-1:!(!h||!h.length)},empty:function(){return h=[],e=0,this},disable:function(){return h=i=c=void 0,this},disabled:function(){return!h},lock:function(){return i=void 0,c||k.disable(),this},locked:function(){return!i},fireWith:function(a,c){return!h||d&&!i||(c=c||[],c=[a,c.slice?c.slice():c],b?i.push(c):j(c)),this},fire:function(){return k.fireWith(this,arguments),this},fired:function(){return!!d}};return k},m.extend({Deferred:function(a){var b=[["resolve","done",m.Callbacks("once memory"),"resolved"],["reject","fail",m.Callbacks("once memory"),"rejected"],["notify","progress",m.Callbacks("memory")]],c="pending",d={state:function(){return c},always:function(){return e.done(arguments).fail(arguments),this},then:function(){var a=arguments;return m.Deferred(function(c){m.each(b,function(b,f){var g=m.isFunction(a[b])&&a[b];e[f[1]](function(){var a=g&&g.apply(this,arguments);a&&m.isFunction(a.promise)?a.promise().done(c.resolve).fail(c.reject).progress(c.notify):c[f[0]+"With"](this===d?c.promise():this,g?[a]:arguments)})}),a=null}).promise()},promise:function(a){return null!=a?m.extend(a,d):d}},e={};return d.pipe=d.then,m.each(b,function(a,f){var g=f[2],h=f[3];d[f[1]]=g.add,h&&g.add(function(){c=h},b[1^a][2].disable,b[2][2].lock),e[f[0]]=function(){return e[f[0]+"With"](this===e?d:this,arguments),this},e[f[0]+"With"]=g.fireWith}),d.promise(e),a&&a.call(e,e),e},when:function(a){var b=0,c=d.call(arguments),e=c.length,f=1!==e||a&&m.isFunction(a.promise)?e:0,g=1===f?a:m.Deferred(),h=function(a,b,c){return function(e){b[a]=this,c[a]=arguments.length>1?d.call(arguments):e,c===i?g.notifyWith(b,c):--f||g.resolveWith(b,c)}},i,j,k;if(e>1)for(i=new Array(e),j=new Array(e),k=new Array(e);e>b;b++)c[b]&&m.isFunction(c[b].promise)?c[b].promise().done(h(b,k,c)).fail(g.reject).progress(h(b,j,i)):--f;return f||g.resolveWith(k,c),g.promise()}});var H;m.fn.ready=function(a){return m.ready.promise().done(a),this},m.extend({isReady:!1,readyWait:1,holdReady:function(a){a?m.readyWait++:m.ready(!0)},ready:function(a){if(a===!0?!--m.readyWait:!m.isReady){if(!y.body)return setTimeout(m.ready);m.isReady=!0,a!==!0&&--m.readyWait>0||(H.resolveWith(y,[m]),m.fn.triggerHandler&&(m(y).triggerHandler("ready"),m(y).off("ready")))}}});function I(){y.addEventListener?(y.removeEventListener("DOMContentLoaded",J,!1),a.removeEventListener("load",J,!1)):(y.detachEvent("onreadystatechange",J),a.detachEvent("onload",J))}function J(){(y.addEventListener||"load"===event.type||"complete"===y.readyState)&&(I(),m.ready())}m.ready.promise=function(b){if(!H)if(H=m.Deferred(),"complete"===y.readyState)setTimeout(m.ready);else if(y.addEventListener)y.addEventListener("DOMContentLoaded",J,!1),a.addEventListener("load",J,!1);else{y.attachEvent("onreadystatechange",J),a.attachEvent("onload",J);var c=!1;try{c=null==a.frameElement&&y.documentElement}catch(d){}c&&c.doScroll&&!function e(){if(!m.isReady){try{c.doScroll("left")}catch(a){return setTimeout(e,50)}I(),m.ready()}}()}return H.promise(b)};var K="undefined",L;for(L in m(k))break;k.ownLast="0"!==L,k.inlineBlockNeedsLayout=!1,m(function(){var a,b,c,d;c=y.getElementsByTagName("body")[0],c&&c.style&&(b=y.createElement("div"),d=y.createElement("div"),d.style.cssText="position:absolute;border:0;width:0;height:0;top:0;left:-9999px",c.appendChild(d).appendChild(b),typeof b.style.zoom!==K&&(b.style.cssText="display:inline;margin:0;border:0;padding:1px;width:1px;zoom:1",k.inlineBlockNeedsLayout=a=3===b.offsetWidth,a&&(c.style.zoom=1)),c.removeChild(d))}),function(){var a=y.createElement("div");if(null==k.deleteExpando){k.deleteExpando=!0;try{delete a.test}catch(b){k.deleteExpando=!1}}a=null}(),m.acceptData=function(a){var b=m.noData[(a.nodeName+" ").toLowerCase()],c=+a.nodeType||1;return 1!==c&&9!==c?!1:!b||b!==!0&&a.getAttribute("classid")===b};var M=/^(?:\{[\w\W]*\}|\[[\w\W]*\])$/,N=/([A-Z])/g;function O(a,b,c){if(void 0===c&&1===a.nodeType){var d="data-"+b.replace(N,"-$1").toLowerCase();if(c=a.getAttribute(d),"string"==typeof c){try{c="true"===c?!0:"false"===c?!1:"null"===c?null:+c+""===c?+c:M.test(c)?m.parseJSON(c):c}catch(e){}m.data(a,b,c)}else c=void 0}return c}function P(a){var b;for(b in a)if(("data"!==b||!m.isEmptyObject(a[b]))&&"toJSON"!==b)return!1;
-
-return!0}function Q(a,b,d,e){if(m.acceptData(a)){var f,g,h=m.expando,i=a.nodeType,j=i?m.cache:a,k=i?a[h]:a[h]&&h;if(k&&j[k]&&(e||j[k].data)||void 0!==d||"string"!=typeof b)return k||(k=i?a[h]=c.pop()||m.guid++:h),j[k]||(j[k]=i?{}:{toJSON:m.noop}),("object"==typeof b||"function"==typeof b)&&(e?j[k]=m.extend(j[k],b):j[k].data=m.extend(j[k].data,b)),g=j[k],e||(g.data||(g.data={}),g=g.data),void 0!==d&&(g[m.camelCase(b)]=d),"string"==typeof b?(f=g[b],null==f&&(f=g[m.camelCase(b)])):f=g,f}}function R(a,b,c){if(m.acceptData(a)){var d,e,f=a.nodeType,g=f?m.cache:a,h=f?a[m.expando]:m.expando;if(g[h]){if(b&&(d=c?g[h]:g[h].data)){m.isArray(b)?b=b.concat(m.map(b,m.camelCase)):b in d?b=[b]:(b=m.camelCase(b),b=b in d?[b]:b.split(" ")),e=b.length;while(e--)delete d[b[e]];if(c?!P(d):!m.isEmptyObject(d))return}(c||(delete g[h].data,P(g[h])))&&(f?m.cleanData([a],!0):k.deleteExpando||g!=g.window?delete g[h]:g[h]=null)}}}m.extend({cache:{},noData:{"applet ":!0,"embed ":!0,"object ":"clsid:D27CDB6E-AE6D-11cf-96B8-444553540000"},hasData:function(a){return a=a.nodeType?m.cache[a[m.expando]]:a[m.expando],!!a&&!P(a)},data:function(a,b,c){return Q(a,b,c)},removeData:function(a,b){return R(a,b)},_data:function(a,b,c){return Q(a,b,c,!0)},_removeData:function(a,b){return R(a,b,!0)}}),m.fn.extend({data:function(a,b){var c,d,e,f=this[0],g=f&&f.attributes;if(void 0===a){if(this.length&&(e=m.data(f),1===f.nodeType&&!m._data(f,"parsedAttrs"))){c=g.length;while(c--)g[c]&&(d=g[c].name,0===d.indexOf("data-")&&(d=m.camelCase(d.slice(5)),O(f,d,e[d])));m._data(f,"parsedAttrs",!0)}return e}return"object"==typeof a?this.each(function(){m.data(this,a)}):arguments.length>1?this.each(function(){m.data(this,a,b)}):f?O(f,a,m.data(f,a)):void 0},removeData:function(a){return this.each(function(){m.removeData(this,a)})}}),m.extend({queue:function(a,b,c){var d;return a?(b=(b||"fx")+"queue",d=m._data(a,b),c&&(!d||m.isArray(c)?d=m._data(a,b,m.makeArray(c)):d.push(c)),d||[]):void 0},dequeue:function(a,b){b=b||"fx";var c=m.queue(a,b),d=c.length,e=c.shift(),f=m._queueHooks(a,b),g=function(){m.dequeue(a,b)};"inprogress"===e&&(e=c.shift(),d--),e&&("fx"===b&&c.unshift("inprogress"),delete f.stop,e.call(a,g,f)),!d&&f&&f.empty.fire()},_queueHooks:function(a,b){var c=b+"queueHooks";return m._data(a,c)||m._data(a,c,{empty:m.Callbacks("once memory").add(function(){m._removeData(a,b+"queue"),m._removeData(a,c)})})}}),m.fn.extend({queue:function(a,b){var c=2;return"string"!=typeof a&&(b=a,a="fx",c--),arguments.length<c?m.queue(this[0],a):void 0===b?this:this.each(function(){var c=m.queue(this,a,b);m._queueHooks(this,a),"fx"===a&&"inprogress"!==c[0]&&m.dequeue(this,a)})},dequeue:function(a){return this.each(function(){m.dequeue(this,a)})},clearQueue:function(a){return this.queue(a||"fx",[])},promise:function(a,b){var c,d=1,e=m.Deferred(),f=this,g=this.length,h=function(){--d||e.resolveWith(f,[f])};"string"!=typeof a&&(b=a,a=void 0),a=a||"fx";while(g--)c=m._data(f[g],a+"queueHooks"),c&&c.empty&&(d++,c.empty.add(h));return h(),e.promise(b)}});var S=/[+-]?(?:\d*\.|)\d+(?:[eE][+-]?\d+|)/.source,T=["Top","Right","Bottom","Left"],U=function(a,b){return a=b||a,"none"===m.css(a,"display")||!m.contains(a.ownerDocument,a)},V=m.access=function(a,b,c,d,e,f,g){var h=0,i=a.length,j=null==c;if("object"===m.type(c)){e=!0;for(h in c)m.access(a,b,h,c[h],!0,f,g)}else if(void 0!==d&&(e=!0,m.isFunction(d)||(g=!0),j&&(g?(b.call(a,d),b=null):(j=b,b=function(a,b,c){return j.call(m(a),c)})),b))for(;i>h;h++)b(a[h],c,g?d:d.call(a[h],h,b(a[h],c)));return e?a:j?b.call(a):i?b(a[0],c):f},W=/^(?:checkbox|radio)$/i;!function(){var a=y.createElement("input"),b=y.createElement("div"),c=y.createDocumentFragment();if(b.innerHTML="  <link/><table></table><a href='/a'>a</a><input type='checkbox'/>",k.leadingWhitespace=3===b.firstChild.nodeType,k.tbody=!b.getElementsByTagName("tbody").length,k.htmlSerialize=!!b.getElementsByTagName("link").length,k.html5Clone="<:nav></:nav>"!==y.createElement("nav").cloneNode(!0).outerHTML,a.type="checkbox",a.checked=!0,c.appendChild(a),k.appendChecked=a.checked,b.innerHTML="<textarea>x</textarea>",k.noCloneChecked=!!b.cloneNode(!0).lastChild.defaultValue,c.appendChild(b),b.innerHTML="<input type='radio' checked='checked' name='t'/>",k.checkClone=b.cloneNode(!0).cloneNode(!0).lastChild.checked,k.noCloneEvent=!0,b.attachEvent&&(b.attachEvent("onclick",function(){k.noCloneEvent=!1}),b.cloneNode(!0).click()),null==k.deleteExpando){k.deleteExpando=!0;try{delete b.test}catch(d){k.deleteExpando=!1}}}(),function(){var b,c,d=y.createElement("div");for(b in{submit:!0,change:!0,focusin:!0})c="on"+b,(k[b+"Bubbles"]=c in a)||(d.setAttribute(c,"t"),k[b+"Bubbles"]=d.attributes[c].expando===!1);d=null}();var X=/^(?:input|select|textarea)$/i,Y=/^key/,Z=/^(?:mouse|pointer|contextmenu)|click/,$=/^(?:focusinfocus|focusoutblur)$/,_=/^([^.]*)(?:\.(.+)|)$/;function aa(){return!0}function ba(){return!1}function ca(){try{return y.activeElement}catch(a){}}m.event={global:{},add:function(a,b,c,d,e){var f,g,h,i,j,k,l,n,o,p,q,r=m._data(a);if(r){c.handler&&(i=c,c=i.handler,e=i.selector),c.guid||(c.guid=m.guid++),(g=r.events)||(g=r.events={}),(k=r.handle)||(k=r.handle=function(a){return typeof m===K||a&&m.event.triggered===a.type?void 0:m.event.dispatch.apply(k.elem,arguments)},k.elem=a),b=(b||"").match(E)||[""],h=b.length;while(h--)f=_.exec(b[h])||[],o=q=f[1],p=(f[2]||"").split(".").sort(),o&&(j=m.event.special[o]||{},o=(e?j.delegateType:j.bindType)||o,j=m.event.special[o]||{},l=m.extend({type:o,origType:q,data:d,handler:c,guid:c.guid,selector:e,needsContext:e&&m.expr.match.needsContext.test(e),namespace:p.join(".")},i),(n=g[o])||(n=g[o]=[],n.delegateCount=0,j.setup&&j.setup.call(a,d,p,k)!==!1||(a.addEventListener?a.addEventListener(o,k,!1):a.attachEvent&&a.attachEvent("on"+o,k))),j.add&&(j.add.call(a,l),l.handler.guid||(l.handler.guid=c.guid)),e?n.splice(n.delegateCount++,0,l):n.push(l),m.event.global[o]=!0);a=null}},remove:function(a,b,c,d,e){var f,g,h,i,j,k,l,n,o,p,q,r=m.hasData(a)&&m._data(a);if(r&&(k=r.events)){b=(b||"").match(E)||[""],j=b.length;while(j--)if(h=_.exec(b[j])||[],o=q=h[1],p=(h[2]||"").split(".").sort(),o){l=m.event.special[o]||{},o=(d?l.delegateType:l.bindType)||o,n=k[o]||[],h=h[2]&&new RegExp("(^|\\.)"+p.join("\\.(?:.*\\.|)")+"(\\.|$)"),i=f=n.length;while(f--)g=n[f],!e&&q!==g.origType||c&&c.guid!==g.guid||h&&!h.test(g.namespace)||d&&d!==g.selector&&("**"!==d||!g.selector)||(n.splice(f,1),g.selector&&n.delegateCount--,l.remove&&l.remove.call(a,g));i&&!n.length&&(l.teardown&&l.teardown.call(a,p,r.handle)!==!1||m.removeEvent(a,o,r.handle),delete k[o])}else for(o in k)m.event.remove(a,o+b[j],c,d,!0);m.isEmptyObject(k)&&(delete r.handle,m._removeData(a,"events"))}},trigger:function(b,c,d,e){var f,g,h,i,k,l,n,o=[d||y],p=j.call(b,"type")?b.type:b,q=j.call(b,"namespace")?b.namespace.split("."):[];if(h=l=d=d||y,3!==d.nodeType&&8!==d.nodeType&&!$.test(p+m.event.triggered)&&(p.indexOf(".")>=0&&(q=p.split("."),p=q.shift(),q.sort()),g=p.indexOf(":")<0&&"on"+p,b=b[m.expando]?b:new m.Event(p,"object"==typeof b&&b),b.isTrigger=e?2:3,b.namespace=q.join("."),b.namespace_re=b.namespace?new RegExp("(^|\\.)"+q.join("\\.(?:.*\\.|)")+"(\\.|$)"):null,b.result=void 0,b.target||(b.target=d),c=null==c?[b]:m.makeArray(c,[b]),k=m.event.special[p]||{},e||!k.trigger||k.trigger.apply(d,c)!==!1)){if(!e&&!k.noBubble&&!m.isWindow(d)){for(i=k.delegateType||p,$.test(i+p)||(h=h.parentNode);h;h=h.parentNode)o.push(h),l=h;l===(d.ownerDocument||y)&&o.push(l.defaultView||l.parentWindow||a)}n=0;while((h=o[n++])&&!b.isPropagationStopped())b.type=n>1?i:k.bindType||p,f=(m._data(h,"events")||{})[b.type]&&m._data(h,"handle"),f&&f.apply(h,c),f=g&&h[g],f&&f.apply&&m.acceptData(h)&&(b.result=f.apply(h,c),b.result===!1&&b.preventDefault());if(b.type=p,!e&&!b.isDefaultPrevented()&&(!k._default||k._default.apply(o.pop(),c)===!1)&&m.acceptData(d)&&g&&d[p]&&!m.isWindow(d)){l=d[g],l&&(d[g]=null),m.event.triggered=p;try{d[p]()}catch(r){}m.event.triggered=void 0,l&&(d[g]=l)}return b.result}},dispatch:function(a){a=m.event.fix(a);var b,c,e,f,g,h=[],i=d.call(arguments),j=(m._data(this,"events")||{})[a.type]||[],k=m.event.special[a.type]||{};if(i[0]=a,a.delegateTarget=this,!k.preDispatch||k.preDispatch.call(this,a)!==!1){h=m.event.handlers.call(this,a,j),b=0;while((f=h[b++])&&!a.isPropagationStopped()){a.currentTarget=f.elem,g=0;while((e=f.handlers[g++])&&!a.isImmediatePropagationStopped())(!a.namespace_re||a.namespace_re.test(e.namespace))&&(a.handleObj=e,a.data=e.data,c=((m.event.special[e.origType]||{}).handle||e.handler).apply(f.elem,i),void 0!==c&&(a.result=c)===!1&&(a.preventDefault(),a.stopPropagation()))}return k.postDispatch&&k.postDispatch.call(this,a),a.result}},handlers:function(a,b){var c,d,e,f,g=[],h=b.delegateCount,i=a.target;if(h&&i.nodeType&&(!a.button||"click"!==a.type))for(;i!=this;i=i.parentNode||this)if(1===i.nodeType&&(i.disabled!==!0||"click"!==a.type)){for(e=[],f=0;h>f;f++)d=b[f],c=d.selector+" ",void 0===e[c]&&(e[c]=d.needsContext?m(c,this).index(i)>=0:m.find(c,this,null,[i]).length),e[c]&&e.push(d);e.length&&g.push({elem:i,handlers:e})}return h<b.length&&g.push({elem:this,handlers:b.slice(h)}),g},fix:function(a){if(a[m.expando])return a;var b,c,d,e=a.type,f=a,g=this.fixHooks[e];g||(this.fixHooks[e]=g=Z.test(e)?this.mouseHooks:Y.test(e)?this.keyHooks:{}),d=g.props?this.props.concat(g.props):this.props,a=new m.Event(f),b=d.length;while(b--)c=d[b],a[c]=f[c];return a.target||(a.target=f.srcElement||y),3===a.target.nodeType&&(a.target=a.target.parentNode),a.metaKey=!!a.metaKey,g.filter?g.filter(a,f):a},props:"altKey bubbles cancelable ctrlKey currentTarget eventPhase metaKey relatedTarget shiftKey target timeStamp view which".split(" "),fixHooks:{},keyHooks:{props:"char charCode key keyCode".split(" "),filter:function(a,b){return null==a.which&&(a.which=null!=b.charCode?b.charCode:b.keyCode),a}},mouseHooks:{props:"button buttons clientX clientY fromElement offsetX offsetY pageX pageY screenX screenY toElement".split(" "),filter:function(a,b){var c,d,e,f=b.button,g=b.fromElement;return null==a.pageX&&null!=b.clientX&&(d=a.target.ownerDocument||y,e=d.documentElement,c=d.body,a.pageX=b.clientX+(e&&e.scrollLeft||c&&c.scrollLeft||0)-(e&&e.clientLeft||c&&c.clientLeft||0),a.pageY=b.clientY+(e&&e.scrollTop||c&&c.scrollTop||0)-(e&&e.clientTop||c&&c.clientTop||0)),!a.relatedTarget&&g&&(a.relatedTarget=g===a.target?b.toElement:g),a.which||void 0===f||(a.which=1&f?1:2&f?3:4&f?2:0),a}},special:{load:{noBubble:!0},focus:{trigger:function(){if(this!==ca()&&this.focus)try{return this.focus(),!1}catch(a){}},delegateType:"focusin"},blur:{trigger:function(){return this===ca()&&this.blur?(this.blur(),!1):void 0},delegateType:"focusout"},click:{trigger:function(){return m.nodeName(this,"input")&&"checkbox"===this.type&&this.click?(this.click(),!1):void 0},_default:function(a){return m.nodeName(a.target,"a")}},beforeunload:{postDispatch:function(a){void 0!==a.result&&a.originalEvent&&(a.originalEvent.returnValue=a.result)}}},simulate:function(a,b,c,d){var e=m.extend(new m.Event,c,{type:a,isSimulated:!0,originalEvent:{}});d?m.event.trigger(e,null,b):m.event.dispatch.call(b,e),e.isDefaultPrevented()&&c.preventDefault()}},m.removeEvent=y.removeEventListener?function(a,b,c){a.removeEventListener&&a.removeEventListener(b,c,!1)}:function(a,b,c){var d="on"+b;a.detachEvent&&(typeof a[d]===K&&(a[d]=null),a.detachEvent(d,c))},m.Event=function(a,b){return this instanceof m.Event?(a&&a.type?(this.originalEvent=a,this.type=a.type,this.isDefaultPrevented=a.defaultPrevented||void 0===a.defaultPrevented&&a.returnValue===!1?aa:ba):this.type=a,b&&m.extend(this,b),this.timeStamp=a&&a.timeStamp||m.now(),void(this[m.expando]=!0)):new m.Event(a,b)},m.Event.prototype={isDefaultPrevented:ba,isPropagationStopped:ba,isImmediatePropagationStopped:ba,preventDefault:function(){var a=this.originalEvent;this.isDefaultPrevented=aa,a&&(a.preventDefault?a.preventDefault():a.returnValue=!1)},stopPropagation:function(){var a=this.originalEvent;this.isPropagationStopped=aa,a&&(a.stopPropagation&&a.stopPropagation(),a.cancelBubble=!0)},stopImmediatePropagation:function(){var a=this.originalEvent;this.isImmediatePropagationStopped=aa,a&&a.stopImmediatePropagation&&a.stopImmediatePropagation(),this.stopPropagation()}},m.each({mouseenter:"mouseover",mouseleave:"mouseout",pointerenter:"pointerover",pointerleave:"pointerout"},function(a,b){m.event.special[a]={delegateType:b,bindType:b,handle:function(a){var c,d=this,e=a.relatedTarget,f=a.handleObj;return(!e||e!==d&&!m.contains(d,e))&&(a.type=f.origType,c=f.handler.apply(this,arguments),a.type=b),c}}}),k.submitBubbles||(m.event.special.submit={setup:function(){return m.nodeName(this,"form")?!1:void m.event.add(this,"click._submit keypress._submit",function(a){var b=a.target,c=m.nodeName(b,"input")||m.nodeName(b,"button")?b.form:void 0;c&&!m._data(c,"submitBubbles")&&(m.event.add(c,"submit._submit",function(a){a._submit_bubble=!0}),m._data(c,"submitBubbles",!0))})},postDispatch:function(a){a._submit_bubble&&(delete a._submit_bubble,this.parentNode&&!a.isTrigger&&m.event.simulate("submit",this.parentNode,a,!0))},teardown:function(){return m.nodeName(this,"form")?!1:void m.event.remove(this,"._submit")}}),k.changeBubbles||(m.event.special.change={setup:function(){return X.test(this.nodeName)?(("checkbox"===this.type||"radio"===this.type)&&(m.event.add(this,"propertychange._change",function(a){"checked"===a.originalEvent.propertyName&&(this._just_changed=!0)}),m.event.add(this,"click._change",function(a){this._just_changed&&!a.isTrigger&&(this._just_changed=!1),m.event.simulate("change",this,a,!0)})),!1):void m.event.add(this,"beforeactivate._change",function(a){var b=a.target;X.test(b.nodeName)&&!m._data(b,"changeBubbles")&&(m.event.add(b,"change._change",function(a){!this.parentNode||a.isSimulated||a.isTrigger||m.event.simulate("change",this.parentNode,a,!0)}),m._data(b,"changeBubbles",!0))})},handle:function(a){var b=a.target;return this!==b||a.isSimulated||a.isTrigger||"radio"!==b.type&&"checkbox"!==b.type?a.handleObj.handler.apply(this,arguments):void 0},teardown:function(){return m.event.remove(this,"._change"),!X.test(this.nodeName)}}),k.focusinBubbles||m.each({focus:"focusin",blur:"focusout"},function(a,b){var c=function(a){m.event.simulate(b,a.target,m.event.fix(a),!0)};m.event.special[b]={setup:function(){var d=this.ownerDocument||this,e=m._data(d,b);e||d.addEventListener(a,c,!0),m._data(d,b,(e||0)+1)},teardown:function(){var d=this.ownerDocument||this,e=m._data(d,b)-1;e?m._data(d,b,e):(d.removeEventListener(a,c,!0),m._removeData(d,b))}}}),m.fn.extend({on:function(a,b,c,d,e){var f,g;if("object"==typeof a){"string"!=typeof b&&(c=c||b,b=void 0);for(f in a)this.on(f,b,c,a[f],e);return this}if(null==c&&null==d?(d=b,c=b=void 0):null==d&&("string"==typeof b?(d=c,c=void 0):(d=c,c=b,b=void 0)),d===!1)d=ba;else if(!d)return this;return 1===e&&(g=d,d=function(a){return m().off(a),g.apply(this,arguments)},d.guid=g.guid||(g.guid=m.guid++)),this.each(function(){m.event.add(this,a,d,c,b)})},one:function(a,b,c,d){return this.on(a,b,c,d,1)},off:function(a,b,c){var d,e;if(a&&a.preventDefault&&a.handleObj)return d=a.handleObj,m(a.delegateTarget).off(d.namespace?d.origType+"."+d.namespace:d.origType,d.selector,d.handler),this;if("object"==typeof a){for(e in a)this.off(e,b,a[e]);return this}return(b===!1||"function"==typeof b)&&(c=b,b=void 0),c===!1&&(c=ba),this.each(function(){m.event.remove(this,a,c,b)})},trigger:function(a,b){return this.each(function(){m.event.trigger(a,b,this)})},triggerHandler:function(a,b){var c=this[0];return c?m.event.trigger(a,b,c,!0):void 0}});function da(a){var b=ea.split("|"),c=a.createDocumentFragment();if(c.createElement)while(b.length)c.createElement(b.pop());return c}var ea="abbr|article|aside|audio|bdi|canvas|data|datalist|details|figcaption|figure|footer|header|hgroup|mark|meter|nav|output|progress|section|summary|time|video",fa=/ jQuery\d+="(?:null|\d+)"/g,ga=new RegExp("<(?:"+ea+")[\\s/>]","i"),ha=/^\s+/,ia=/<(?!area|br|col|embed|hr|img|input|link|meta|param)(([\w:]+)[^>]*)\/>/gi,ja=/<([\w:]+)/,ka=/<tbody/i,la=/<|&#?\w+;/,ma=/<(?:script|style|link)/i,na=/checked\s*(?:[^=]|=\s*.checked.)/i,oa=/^$|\/(?:java|ecma)script/i,pa=/^true\/(.*)/,qa=/^\s*<!(?:\[CDATA\[|--)|(?:\]\]|--)>\s*$/g,ra={option:[1,"<select multiple='multiple'>","</select>"],legend:[1,"<fieldset>","</fieldset>"],area:[1,"<map>","</map>"],param:[1,"<object>","</object>"],thead:[1,"<table>","</table>"],tr:[2,"<table><tbody>","</tbody></table>"],col:[2,"<table><tbody></tbody><colgroup>","</colgroup></table>"],td:[3,"<table><tbody><tr>","</tr></tbody></table>"],_default:k.htmlSerialize?[0,"",""]:[1,"X<div>","</div>"]},sa=da(y),ta=sa.appendChild(y.createElement("div"));ra.optgroup=ra.option,ra.tbody=ra.tfoot=ra.colgroup=ra.caption=ra.thead,ra.th=ra.td;function ua(a,b){var c,d,e=0,f=typeof a.getElementsByTagName!==K?a.getElementsByTagName(b||"*"):typeof a.querySelectorAll!==K?a.querySelectorAll(b||"*"):void 0;if(!f)for(f=[],c=a.childNodes||a;null!=(d=c[e]);e++)!b||m.nodeName(d,b)?f.push(d):m.merge(f,ua(d,b));return void 0===b||b&&m.nodeName(a,b)?m.merge([a],f):f}function va(a){W.test(a.type)&&(a.defaultChecked=a.checked)}function wa(a,b){return m.nodeName(a,"table")&&m.nodeName(11!==b.nodeType?b:b.firstChild,"tr")?a.getElementsByTagName("tbody")[0]||a.appendChild(a.ownerDocument.createElement("tbody")):a}function xa(a){return a.type=(null!==m.find.attr(a,"type"))+"/"+a.type,a}function ya(a){var b=pa.exec(a.type);return b?a.type=b[1]:a.removeAttribute("type"),a}function za(a,b){for(var c,d=0;null!=(c=a[d]);d++)m._data(c,"globalEval",!b||m._data(b[d],"globalEval"))}function Aa(a,b){if(1===b.nodeType&&m.hasData(a)){var c,d,e,f=m._data(a),g=m._data(b,f),h=f.events;if(h){delete g.handle,g.events={};for(c in h)for(d=0,e=h[c].length;e>d;d++)m.event.add(b,c,h[c][d])}g.data&&(g.data=m.extend({},g.data))}}function Ba(a,b){var c,d,e;if(1===b.nodeType){if(c=b.nodeName.toLowerCase(),!k.noCloneEvent&&b[m.expando]){e=m._data(b);for(d in e.events)m.removeEvent(b,d,e.handle);b.removeAttribute(m.expando)}"script"===c&&b.text!==a.text?(xa(b).text=a.text,ya(b)):"object"===c?(b.parentNode&&(b.outerHTML=a.outerHTML),k.html5Clone&&a.innerHTML&&!m.trim(b.innerHTML)&&(b.innerHTML=a.innerHTML)):"input"===c&&W.test(a.type)?(b.defaultChecked=b.checked=a.checked,b.value!==a.value&&(b.value=a.value)):"option"===c?b.defaultSelected=b.selected=a.defaultSelected:("input"===c||"textarea"===c)&&(b.defaultValue=a.defaultValue)}}m.extend({clone:function(a,b,c){var d,e,f,g,h,i=m.contains(a.ownerDocument,a);if(k.html5Clone||m.isXMLDoc(a)||!ga.test("<"+a.nodeName+">")?f=a.cloneNode(!0):(ta.innerHTML=a.outerHTML,ta.removeChild(f=ta.firstChild)),!(k.noCloneEvent&&k.noCloneChecked||1!==a.nodeType&&11!==a.nodeType||m.isXMLDoc(a)))for(d=ua(f),h=ua(a),g=0;null!=(e=h[g]);++g)d[g]&&Ba(e,d[g]);if(b)if(c)for(h=h||ua(a),d=d||ua(f),g=0;null!=(e=h[g]);g++)Aa(e,d[g]);else Aa(a,f);return d=ua(f,"script"),d.length>0&&za(d,!i&&ua(a,"script")),d=h=e=null,f},buildFragment:function(a,b,c,d){for(var e,f,g,h,i,j,l,n=a.length,o=da(b),p=[],q=0;n>q;q++)if(f=a[q],f||0===f)if("object"===m.type(f))m.merge(p,f.nodeType?[f]:f);else if(la.test(f)){h=h||o.appendChild(b.createElement("div")),i=(ja.exec(f)||["",""])[1].toLowerCase(),l=ra[i]||ra._default,h.innerHTML=l[1]+f.replace(ia,"<$1></$2>")+l[2],e=l[0];while(e--)h=h.lastChild;if(!k.leadingWhitespace&&ha.test(f)&&p.push(b.createTextNode(ha.exec(f)[0])),!k.tbody){f="table"!==i||ka.test(f)?"<table>"!==l[1]||ka.test(f)?0:h:h.firstChild,e=f&&f.childNodes.length;while(e--)m.nodeName(j=f.childNodes[e],"tbody")&&!j.childNodes.length&&f.removeChild(j)}m.merge(p,h.childNodes),h.textContent="";while(h.firstChild)h.removeChild(h.firstChild);h=o.lastChild}else p.push(b.createTextNode(f));h&&o.removeChild(h),k.appendChecked||m.grep(ua(p,"input"),va),q=0;while(f=p[q++])if((!d||-1===m.inArray(f,d))&&(g=m.contains(f.ownerDocument,f),h=ua(o.appendChild(f),"script"),g&&za(h),c)){e=0;while(f=h[e++])oa.test(f.type||"")&&c.push(f)}return h=null,o},cleanData:function(a,b){for(var d,e,f,g,h=0,i=m.expando,j=m.cache,l=k.deleteExpando,n=m.event.special;null!=(d=a[h]);h++)if((b||m.acceptData(d))&&(f=d[i],g=f&&j[f])){if(g.events)for(e in g.events)n[e]?m.event.remove(d,e):m.removeEvent(d,e,g.handle);j[f]&&(delete j[f],l?delete d[i]:typeof d.removeAttribute!==K?d.removeAttribute(i):d[i]=null,c.push(f))}}}),m.fn.extend({text:function(a){return V(this,function(a){return void 0===a?m.text(this):this.empty().append((this[0]&&this[0].ownerDocument||y).createTextNode(a))},null,a,arguments.length)},append:function(){return this.domManip(arguments,function(a){if(1===this.nodeType||11===this.nodeType||9===this.nodeType){var b=wa(this,a);b.appendChild(a)}})},prepend:function(){return this.domManip(arguments,function(a){if(1===this.nodeType||11===this.nodeType||9===this.nodeType){var b=wa(this,a);b.insertBefore(a,b.firstChild)}})},before:function(){return this.domManip(arguments,function(a){this.parentNode&&this.parentNode.insertBefore(a,this)})},after:function(){return this.domManip(arguments,function(a){this.parentNode&&this.parentNode.insertBefore(a,this.nextSibling)})},remove:function(a,b){for(var c,d=a?m.filter(a,this):this,e=0;null!=(c=d[e]);e++)b||1!==c.nodeType||m.cleanData(ua(c)),c.parentNode&&(b&&m.contains(c.ownerDocument,c)&&za(ua(c,"script")),c.parentNode.removeChild(c));return this},empty:function(){for(var a,b=0;null!=(a=this[b]);b++){1===a.nodeType&&m.cleanData(ua(a,!1));while(a.firstChild)a.removeChild(a.firstChild);a.options&&m.nodeName(a,"select")&&(a.options.length=0)}return this},clone:function(a,b){return a=null==a?!1:a,b=null==b?a:b,this.map(function(){return m.clone(this,a,b)})},html:function(a){return V(this,function(a){var b=this[0]||{},c=0,d=this.length;if(void 0===a)return 1===b.nodeType?b.innerHTML.replace(fa,""):void 0;if(!("string"!=typeof a||ma.test(a)||!k.htmlSerialize&&ga.test(a)||!k.leadingWhitespace&&ha.test(a)||ra[(ja.exec(a)||["",""])[1].toLowerCase()])){a=a.replace(ia,"<$1></$2>");try{for(;d>c;c++)b=this[c]||{},1===b.nodeType&&(m.cleanData(ua(b,!1)),b.innerHTML=a);b=0}catch(e){}}b&&this.empty().append(a)},null,a,arguments.length)},replaceWith:function(){var a=arguments[0];return this.domManip(arguments,function(b){a=this.parentNode,m.cleanData(ua(this)),a&&a.replaceChild(b,this)}),a&&(a.length||a.nodeType)?this:this.remove()},detach:function(a){return this.remove(a,!0)},domManip:function(a,b){a=e.apply([],a);var c,d,f,g,h,i,j=0,l=this.length,n=this,o=l-1,p=a[0],q=m.isFunction(p);if(q||l>1&&"string"==typeof p&&!k.checkClone&&na.test(p))return this.each(function(c){var d=n.eq(c);q&&(a[0]=p.call(this,c,d.html())),d.domManip(a,b)});if(l&&(i=m.buildFragment(a,this[0].ownerDocument,!1,this),c=i.firstChild,1===i.childNodes.length&&(i=c),c)){for(g=m.map(ua(i,"script"),xa),f=g.length;l>j;j++)d=i,j!==o&&(d=m.clone(d,!0,!0),f&&m.merge(g,ua(d,"script"))),b.call(this[j],d,j);if(f)for(h=g[g.length-1].ownerDocument,m.map(g,ya),j=0;f>j;j++)d=g[j],oa.test(d.type||"")&&!m._data(d,"globalEval")&&m.contains(h,d)&&(d.src?m._evalUrl&&m._evalUrl(d.src):m.globalEval((d.text||d.textContent||d.innerHTML||"").replace(qa,"")));i=c=null}return this}}),m.each({appendTo:"append",prependTo:"prepend",insertBefore:"before",insertAfter:"after",replaceAll:"replaceWith"},function(a,b){m.fn[a]=function(a){for(var c,d=0,e=[],g=m(a),h=g.length-1;h>=d;d++)c=d===h?this:this.clone(!0),m(g[d])[b](c),f.apply(e,c.get());return this.pushStack(e)}});var Ca,Da={};function Ea(b,c){var d,e=m(c.createElement(b)).appendTo(c.body),f=a.getDefaultComputedStyle&&(d=a.getDefaultComputedStyle(e[0]))?d.display:m.css(e[0],"display");return e.detach(),f}function Fa(a){var b=y,c=Da[a];return c||(c=Ea(a,b),"none"!==c&&c||(Ca=(Ca||m("<iframe frameborder='0' width='0' height='0'/>")).appendTo(b.documentElement),b=(Ca[0].contentWindow||Ca[0].contentDocument).document,b.write(),b.close(),c=Ea(a,b),Ca.detach()),Da[a]=c),c}!function(){var a;k.shrinkWrapBlocks=function(){if(null!=a)return a;a=!1;var b,c,d;return c=y.getElementsByTagName("body")[0],c&&c.style?(b=y.createElement("div"),d=y.createElement("div"),d.style.cssText="position:absolute;border:0;width:0;height:0;top:0;left:-9999px",c.appendChild(d).appendChild(b),typeof b.style.zoom!==K&&(b.style.cssText="-webkit-box-sizing:content-box;-moz-box-sizing:content-box;box-sizing:content-box;display:block;margin:0;border:0;padding:1px;width:1px;zoom:1",b.appendChild(y.createElement("div")).style.width="5px",a=3!==b.offsetWidth),c.removeChild(d),a):void 0}}();var Ga=/^margin/,Ha=new RegExp("^("+S+")(?!px)[a-z%]+$","i"),Ia,Ja,Ka=/^(top|right|bottom|left)$/;a.getComputedStyle?(Ia=function(b){return b.ownerDocument.defaultView.opener?b.ownerDocument.defaultView.getComputedStyle(b,null):a.getComputedStyle(b,null)},Ja=function(a,b,c){var d,e,f,g,h=a.style;return c=c||Ia(a),g=c?c.getPropertyValue(b)||c[b]:void 0,c&&(""!==g||m.contains(a.ownerDocument,a)||(g=m.style(a,b)),Ha.test(g)&&Ga.test(b)&&(d=h.width,e=h.minWidth,f=h.maxWidth,h.minWidth=h.maxWidth=h.width=g,g=c.width,h.width=d,h.minWidth=e,h.maxWidth=f)),void 0===g?g:g+""}):y.documentElement.currentStyle&&(Ia=function(a){return a.currentStyle},Ja=function(a,b,c){var d,e,f,g,h=a.style;return c=c||Ia(a),g=c?c[b]:void 0,null==g&&h&&h[b]&&(g=h[b]),Ha.test(g)&&!Ka.test(b)&&(d=h.left,e=a.runtimeStyle,f=e&&e.left,f&&(e.left=a.currentStyle.left),h.left="fontSize"===b?"1em":g,g=h.pixelLeft+"px",h.left=d,f&&(e.left=f)),void 0===g?g:g+""||"auto"});function La(a,b){return{get:function(){var c=a();if(null!=c)return c?void delete this.get:(this.get=b).apply(this,arguments)}}}!function(){var b,c,d,e,f,g,h;if(b=y.createElement("div"),b.innerHTML="  <link/><table></table><a href='/a'>a</a><input type='checkbox'/>",d=b.getElementsByTagName("a")[0],c=d&&d.style){c.cssText="float:left;opacity:.5",k.opacity="0.5"===c.opacity,k.cssFloat=!!c.cssFloat,b.style.backgroundClip="content-box",b.cloneNode(!0).style.backgroundClip="",k.clearCloneStyle="content-box"===b.style.backgroundClip,k.boxSizing=""===c.boxSizing||""===c.MozBoxSizing||""===c.WebkitBoxSizing,m.extend(k,{reliableHiddenOffsets:function(){return null==g&&i(),g},boxSizingReliable:function(){return null==f&&i(),f},pixelPosition:function(){return null==e&&i(),e},reliableMarginRight:function(){return null==h&&i(),h}});function i(){var b,c,d,i;c=y.getElementsByTagName("body")[0],c&&c.style&&(b=y.createElement("div"),d=y.createElement("div"),d.style.cssText="position:absolute;border:0;width:0;height:0;top:0;left:-9999px",c.appendChild(d).appendChild(b),b.style.cssText="-webkit-box-sizing:border-box;-moz-box-sizing:border-box;box-sizing:border-box;display:block;margin-top:1%;top:1%;border:1px;padding:1px;width:4px;position:absolute",e=f=!1,h=!0,a.getComputedStyle&&(e="1%"!==(a.getComputedStyle(b,null)||{}).top,f="4px"===(a.getComputedStyle(b,null)||{width:"4px"}).width,i=b.appendChild(y.createElement("div")),i.style.cssText=b.style.cssText="-webkit-box-sizing:content-box;-moz-box-sizing:content-box;box-sizing:content-box;display:block;margin:0;border:0;padding:0",i.style.marginRight=i.style.width="0",b.style.width="1px",h=!parseFloat((a.getComputedStyle(i,null)||{}).marginRight),b.removeChild(i)),b.innerHTML="<table><tr><td></td><td>t</td></tr></table>",i=b.getElementsByTagName("td"),i[0].style.cssText="margin:0;border:0;padding:0;display:none",g=0===i[0].offsetHeight,g&&(i[0].style.display="",i[1].style.display="none",g=0===i[0].offsetHeight),c.removeChild(d))}}}(),m.swap=function(a,b,c,d){var e,f,g={};for(f in b)g[f]=a.style[f],a.style[f]=b[f];e=c.apply(a,d||[]);for(f in b)a.style[f]=g[f];return e};var Ma=/alpha\([^)]*\)/i,Na=/opacity\s*=\s*([^)]*)/,Oa=/^(none|table(?!-c[ea]).+)/,Pa=new RegExp("^("+S+")(.*)$","i"),Qa=new RegExp("^([+-])=("+S+")","i"),Ra={position:"absolute",visibility:"hidden",display:"block"},Sa={letterSpacing:"0",fontWeight:"400"},Ta=["Webkit","O","Moz","ms"];function Ua(a,b){if(b in a)return b;var c=b.charAt(0).toUpperCase()+b.slice(1),d=b,e=Ta.length;while(e--)if(b=Ta[e]+c,b in a)return b;return d}function Va(a,b){for(var c,d,e,f=[],g=0,h=a.length;h>g;g++)d=a[g],d.style&&(f[g]=m._data(d,"olddisplay"),c=d.style.display,b?(f[g]||"none"!==c||(d.style.display=""),""===d.style.display&&U(d)&&(f[g]=m._data(d,"olddisplay",Fa(d.nodeName)))):(e=U(d),(c&&"none"!==c||!e)&&m._data(d,"olddisplay",e?c:m.css(d,"display"))));for(g=0;h>g;g++)d=a[g],d.style&&(b&&"none"!==d.style.display&&""!==d.style.display||(d.style.display=b?f[g]||"":"none"));return a}function Wa(a,b,c){var d=Pa.exec(b);return d?Math.max(0,d[1]-(c||0))+(d[2]||"px"):b}function Xa(a,b,c,d,e){for(var f=c===(d?"border":"content")?4:"width"===b?1:0,g=0;4>f;f+=2)"margin"===c&&(g+=m.css(a,c+T[f],!0,e)),d?("content"===c&&(g-=m.css(a,"padding"+T[f],!0,e)),"margin"!==c&&(g-=m.css(a,"border"+T[f]+"Width",!0,e))):(g+=m.css(a,"padding"+T[f],!0,e),"padding"!==c&&(g+=m.css(a,"border"+T[f]+"Width",!0,e)));return g}function Ya(a,b,c){var d=!0,e="width"===b?a.offsetWidth:a.offsetHeight,f=Ia(a),g=k.boxSizing&&"border-box"===m.css(a,"boxSizing",!1,f);if(0>=e||null==e){if(e=Ja(a,b,f),(0>e||null==e)&&(e=a.style[b]),Ha.test(e))return e;d=g&&(k.boxSizingReliable()||e===a.style[b]),e=parseFloat(e)||0}return e+Xa(a,b,c||(g?"border":"content"),d,f)+"px"}m.extend({cssHooks:{opacity:{get:function(a,b){if(b){var c=Ja(a,"opacity");return""===c?"1":c}}}},cssNumber:{columnCount:!0,fillOpacity:!0,flexGrow:!0,flexShrink:!0,fontWeight:!0,lineHeight:!0,opacity:!0,order:!0,orphans:!0,widows:!0,zIndex:!0,zoom:!0},cssProps:{"float":k.cssFloat?"cssFloat":"styleFloat"},style:function(a,b,c,d){if(a&&3!==a.nodeType&&8!==a.nodeType&&a.style){var e,f,g,h=m.camelCase(b),i=a.style;if(b=m.cssProps[h]||(m.cssProps[h]=Ua(i,h)),g=m.cssHooks[b]||m.cssHooks[h],void 0===c)return g&&"get"in g&&void 0!==(e=g.get(a,!1,d))?e:i[b];if(f=typeof c,"string"===f&&(e=Qa.exec(c))&&(c=(e[1]+1)*e[2]+parseFloat(m.css(a,b)),f="number"),null!=c&&c===c&&("number"!==f||m.cssNumber[h]||(c+="px"),k.clearCloneStyle||""!==c||0!==b.indexOf("background")||(i[b]="inherit"),!(g&&"set"in g&&void 0===(c=g.set(a,c,d)))))try{i[b]=c}catch(j){}}},css:function(a,b,c,d){var e,f,g,h=m.camelCase(b);return b=m.cssProps[h]||(m.cssProps[h]=Ua(a.style,h)),g=m.cssHooks[b]||m.cssHooks[h],g&&"get"in g&&(f=g.get(a,!0,c)),void 0===f&&(f=Ja(a,b,d)),"normal"===f&&b in Sa&&(f=Sa[b]),""===c||c?(e=parseFloat(f),c===!0||m.isNumeric(e)?e||0:f):f}}),m.each(["height","width"],function(a,b){m.cssHooks[b]={get:function(a,c,d){return c?Oa.test(m.css(a,"display"))&&0===a.offsetWidth?m.swap(a,Ra,function(){return Ya(a,b,d)}):Ya(a,b,d):void 0},set:function(a,c,d){var e=d&&Ia(a);return Wa(a,c,d?Xa(a,b,d,k.boxSizing&&"border-box"===m.css(a,"boxSizing",!1,e),e):0)}}}),k.opacity||(m.cssHooks.opacity={get:function(a,b){return Na.test((b&&a.currentStyle?a.currentStyle.filter:a.style.filter)||"")?.01*parseFloat(RegExp.$1)+"":b?"1":""},set:function(a,b){var c=a.style,d=a.currentStyle,e=m.isNumeric(b)?"alpha(opacity="+100*b+")":"",f=d&&d.filter||c.filter||"";c.zoom=1,(b>=1||""===b)&&""===m.trim(f.replace(Ma,""))&&c.removeAttribute&&(c.removeAttribute("filter"),""===b||d&&!d.filter)||(c.filter=Ma.test(f)?f.replace(Ma,e):f+" "+e)}}),m.cssHooks.marginRight=La(k.reliableMarginRight,function(a,b){return b?m.swap(a,{display:"inline-block"},Ja,[a,"marginRight"]):void 0}),m.each({margin:"",padding:"",border:"Width"},function(a,b){m.cssHooks[a+b]={expand:function(c){for(var d=0,e={},f="string"==typeof c?c.split(" "):[c];4>d;d++)e[a+T[d]+b]=f[d]||f[d-2]||f[0];return e}},Ga.test(a)||(m.cssHooks[a+b].set=Wa)}),m.fn.extend({css:function(a,b){return V(this,function(a,b,c){var d,e,f={},g=0;if(m.isArray(b)){for(d=Ia(a),e=b.length;e>g;g++)f[b[g]]=m.css(a,b[g],!1,d);return f}return void 0!==c?m.style(a,b,c):m.css(a,b)},a,b,arguments.length>1)},show:function(){return Va(this,!0)},hide:function(){return Va(this)},toggle:function(a){return"boolean"==typeof a?a?this.show():this.hide():this.each(function(){U(this)?m(this).show():m(this).hide()})}});function Za(a,b,c,d,e){
-return new Za.prototype.init(a,b,c,d,e)}m.Tween=Za,Za.prototype={constructor:Za,init:function(a,b,c,d,e,f){this.elem=a,this.prop=c,this.easing=e||"swing",this.options=b,this.start=this.now=this.cur(),this.end=d,this.unit=f||(m.cssNumber[c]?"":"px")},cur:function(){var a=Za.propHooks[this.prop];return a&&a.get?a.get(this):Za.propHooks._default.get(this)},run:function(a){var b,c=Za.propHooks[this.prop];return this.options.duration?this.pos=b=m.easing[this.easing](a,this.options.duration*a,0,1,this.options.duration):this.pos=b=a,this.now=(this.end-this.start)*b+this.start,this.options.step&&this.options.step.call(this.elem,this.now,this),c&&c.set?c.set(this):Za.propHooks._default.set(this),this}},Za.prototype.init.prototype=Za.prototype,Za.propHooks={_default:{get:function(a){var b;return null==a.elem[a.prop]||a.elem.style&&null!=a.elem.style[a.prop]?(b=m.css(a.elem,a.prop,""),b&&"auto"!==b?b:0):a.elem[a.prop]},set:function(a){m.fx.step[a.prop]?m.fx.step[a.prop](a):a.elem.style&&(null!=a.elem.style[m.cssProps[a.prop]]||m.cssHooks[a.prop])?m.style(a.elem,a.prop,a.now+a.unit):a.elem[a.prop]=a.now}}},Za.propHooks.scrollTop=Za.propHooks.scrollLeft={set:function(a){a.elem.nodeType&&a.elem.parentNode&&(a.elem[a.prop]=a.now)}},m.easing={linear:function(a){return a},swing:function(a){return.5-Math.cos(a*Math.PI)/2}},m.fx=Za.prototype.init,m.fx.step={};var $a,_a,ab=/^(?:toggle|show|hide)$/,bb=new RegExp("^(?:([+-])=|)("+S+")([a-z%]*)$","i"),cb=/queueHooks$/,db=[ib],eb={"*":[function(a,b){var c=this.createTween(a,b),d=c.cur(),e=bb.exec(b),f=e&&e[3]||(m.cssNumber[a]?"":"px"),g=(m.cssNumber[a]||"px"!==f&&+d)&&bb.exec(m.css(c.elem,a)),h=1,i=20;if(g&&g[3]!==f){f=f||g[3],e=e||[],g=+d||1;do h=h||".5",g/=h,m.style(c.elem,a,g+f);while(h!==(h=c.cur()/d)&&1!==h&&--i)}return e&&(g=c.start=+g||+d||0,c.unit=f,c.end=e[1]?g+(e[1]+1)*e[2]:+e[2]),c}]};function fb(){return setTimeout(function(){$a=void 0}),$a=m.now()}function gb(a,b){var c,d={height:a},e=0;for(b=b?1:0;4>e;e+=2-b)c=T[e],d["margin"+c]=d["padding"+c]=a;return b&&(d.opacity=d.width=a),d}function hb(a,b,c){for(var d,e=(eb[b]||[]).concat(eb["*"]),f=0,g=e.length;g>f;f++)if(d=e[f].call(c,b,a))return d}function ib(a,b,c){var d,e,f,g,h,i,j,l,n=this,o={},p=a.style,q=a.nodeType&&U(a),r=m._data(a,"fxshow");c.queue||(h=m._queueHooks(a,"fx"),null==h.unqueued&&(h.unqueued=0,i=h.empty.fire,h.empty.fire=function(){h.unqueued||i()}),h.unqueued++,n.always(function(){n.always(function(){h.unqueued--,m.queue(a,"fx").length||h.empty.fire()})})),1===a.nodeType&&("height"in b||"width"in b)&&(c.overflow=[p.overflow,p.overflowX,p.overflowY],j=m.css(a,"display"),l="none"===j?m._data(a,"olddisplay")||Fa(a.nodeName):j,"inline"===l&&"none"===m.css(a,"float")&&(k.inlineBlockNeedsLayout&&"inline"!==Fa(a.nodeName)?p.zoom=1:p.display="inline-block")),c.overflow&&(p.overflow="hidden",k.shrinkWrapBlocks()||n.always(function(){p.overflow=c.overflow[0],p.overflowX=c.overflow[1],p.overflowY=c.overflow[2]}));for(d in b)if(e=b[d],ab.exec(e)){if(delete b[d],f=f||"toggle"===e,e===(q?"hide":"show")){if("show"!==e||!r||void 0===r[d])continue;q=!0}o[d]=r&&r[d]||m.style(a,d)}else j=void 0;if(m.isEmptyObject(o))"inline"===("none"===j?Fa(a.nodeName):j)&&(p.display=j);else{r?"hidden"in r&&(q=r.hidden):r=m._data(a,"fxshow",{}),f&&(r.hidden=!q),q?m(a).show():n.done(function(){m(a).hide()}),n.done(function(){var b;m._removeData(a,"fxshow");for(b in o)m.style(a,b,o[b])});for(d in o)g=hb(q?r[d]:0,d,n),d in r||(r[d]=g.start,q&&(g.end=g.start,g.start="width"===d||"height"===d?1:0))}}function jb(a,b){var c,d,e,f,g;for(c in a)if(d=m.camelCase(c),e=b[d],f=a[c],m.isArray(f)&&(e=f[1],f=a[c]=f[0]),c!==d&&(a[d]=f,delete a[c]),g=m.cssHooks[d],g&&"expand"in g){f=g.expand(f),delete a[d];for(c in f)c in a||(a[c]=f[c],b[c]=e)}else b[d]=e}function kb(a,b,c){var d,e,f=0,g=db.length,h=m.Deferred().always(function(){delete i.elem}),i=function(){if(e)return!1;for(var b=$a||fb(),c=Math.max(0,j.startTime+j.duration-b),d=c/j.duration||0,f=1-d,g=0,i=j.tweens.length;i>g;g++)j.tweens[g].run(f);return h.notifyWith(a,[j,f,c]),1>f&&i?c:(h.resolveWith(a,[j]),!1)},j=h.promise({elem:a,props:m.extend({},b),opts:m.extend(!0,{specialEasing:{}},c),originalProperties:b,originalOptions:c,startTime:$a||fb(),duration:c.duration,tweens:[],createTween:function(b,c){var d=m.Tween(a,j.opts,b,c,j.opts.specialEasing[b]||j.opts.easing);return j.tweens.push(d),d},stop:function(b){var c=0,d=b?j.tweens.length:0;if(e)return this;for(e=!0;d>c;c++)j.tweens[c].run(1);return b?h.resolveWith(a,[j,b]):h.rejectWith(a,[j,b]),this}}),k=j.props;for(jb(k,j.opts.specialEasing);g>f;f++)if(d=db[f].call(j,a,k,j.opts))return d;return m.map(k,hb,j),m.isFunction(j.opts.start)&&j.opts.start.call(a,j),m.fx.timer(m.extend(i,{elem:a,anim:j,queue:j.opts.queue})),j.progress(j.opts.progress).done(j.opts.done,j.opts.complete).fail(j.opts.fail).always(j.opts.always)}m.Animation=m.extend(kb,{tweener:function(a,b){m.isFunction(a)?(b=a,a=["*"]):a=a.split(" ");for(var c,d=0,e=a.length;e>d;d++)c=a[d],eb[c]=eb[c]||[],eb[c].unshift(b)},prefilter:function(a,b){b?db.unshift(a):db.push(a)}}),m.speed=function(a,b,c){var d=a&&"object"==typeof a?m.extend({},a):{complete:c||!c&&b||m.isFunction(a)&&a,duration:a,easing:c&&b||b&&!m.isFunction(b)&&b};return d.duration=m.fx.off?0:"number"==typeof d.duration?d.duration:d.duration in m.fx.speeds?m.fx.speeds[d.duration]:m.fx.speeds._default,(null==d.queue||d.queue===!0)&&(d.queue="fx"),d.old=d.complete,d.complete=function(){m.isFunction(d.old)&&d.old.call(this),d.queue&&m.dequeue(this,d.queue)},d},m.fn.extend({fadeTo:function(a,b,c,d){return this.filter(U).css("opacity",0).show().end().animate({opacity:b},a,c,d)},animate:function(a,b,c,d){var e=m.isEmptyObject(a),f=m.speed(b,c,d),g=function(){var b=kb(this,m.extend({},a),f);(e||m._data(this,"finish"))&&b.stop(!0)};return g.finish=g,e||f.queue===!1?this.each(g):this.queue(f.queue,g)},stop:function(a,b,c){var d=function(a){var b=a.stop;delete a.stop,b(c)};return"string"!=typeof a&&(c=b,b=a,a=void 0),b&&a!==!1&&this.queue(a||"fx",[]),this.each(function(){var b=!0,e=null!=a&&a+"queueHooks",f=m.timers,g=m._data(this);if(e)g[e]&&g[e].stop&&d(g[e]);else for(e in g)g[e]&&g[e].stop&&cb.test(e)&&d(g[e]);for(e=f.length;e--;)f[e].elem!==this||null!=a&&f[e].queue!==a||(f[e].anim.stop(c),b=!1,f.splice(e,1));(b||!c)&&m.dequeue(this,a)})},finish:function(a){return a!==!1&&(a=a||"fx"),this.each(function(){var b,c=m._data(this),d=c[a+"queue"],e=c[a+"queueHooks"],f=m.timers,g=d?d.length:0;for(c.finish=!0,m.queue(this,a,[]),e&&e.stop&&e.stop.call(this,!0),b=f.length;b--;)f[b].elem===this&&f[b].queue===a&&(f[b].anim.stop(!0),f.splice(b,1));for(b=0;g>b;b++)d[b]&&d[b].finish&&d[b].finish.call(this);delete c.finish})}}),m.each(["toggle","show","hide"],function(a,b){var c=m.fn[b];m.fn[b]=function(a,d,e){return null==a||"boolean"==typeof a?c.apply(this,arguments):this.animate(gb(b,!0),a,d,e)}}),m.each({slideDown:gb("show"),slideUp:gb("hide"),slideToggle:gb("toggle"),fadeIn:{opacity:"show"},fadeOut:{opacity:"hide"},fadeToggle:{opacity:"toggle"}},function(a,b){m.fn[a]=function(a,c,d){return this.animate(b,a,c,d)}}),m.timers=[],m.fx.tick=function(){var a,b=m.timers,c=0;for($a=m.now();c<b.length;c++)a=b[c],a()||b[c]!==a||b.splice(c--,1);b.length||m.fx.stop(),$a=void 0},m.fx.timer=function(a){m.timers.push(a),a()?m.fx.start():m.timers.pop()},m.fx.interval=13,m.fx.start=function(){_a||(_a=setInterval(m.fx.tick,m.fx.interval))},m.fx.stop=function(){clearInterval(_a),_a=null},m.fx.speeds={slow:600,fast:200,_default:400},m.fn.delay=function(a,b){return a=m.fx?m.fx.speeds[a]||a:a,b=b||"fx",this.queue(b,function(b,c){var d=setTimeout(b,a);c.stop=function(){clearTimeout(d)}})},function(){var a,b,c,d,e;b=y.createElement("div"),b.setAttribute("className","t"),b.innerHTML="  <link/><table></table><a href='/a'>a</a><input type='checkbox'/>",d=b.getElementsByTagName("a")[0],c=y.createElement("select"),e=c.appendChild(y.createElement("option")),a=b.getElementsByTagName("input")[0],d.style.cssText="top:1px",k.getSetAttribute="t"!==b.className,k.style=/top/.test(d.getAttribute("style")),k.hrefNormalized="/a"===d.getAttribute("href"),k.checkOn=!!a.value,k.optSelected=e.selected,k.enctype=!!y.createElement("form").enctype,c.disabled=!0,k.optDisabled=!e.disabled,a=y.createElement("input"),a.setAttribute("value",""),k.input=""===a.getAttribute("value"),a.value="t",a.setAttribute("type","radio"),k.radioValue="t"===a.value}();var lb=/\r/g;m.fn.extend({val:function(a){var b,c,d,e=this[0];{if(arguments.length)return d=m.isFunction(a),this.each(function(c){var e;1===this.nodeType&&(e=d?a.call(this,c,m(this).val()):a,null==e?e="":"number"==typeof e?e+="":m.isArray(e)&&(e=m.map(e,function(a){return null==a?"":a+""})),b=m.valHooks[this.type]||m.valHooks[this.nodeName.toLowerCase()],b&&"set"in b&&void 0!==b.set(this,e,"value")||(this.value=e))});if(e)return b=m.valHooks[e.type]||m.valHooks[e.nodeName.toLowerCase()],b&&"get"in b&&void 0!==(c=b.get(e,"value"))?c:(c=e.value,"string"==typeof c?c.replace(lb,""):null==c?"":c)}}}),m.extend({valHooks:{option:{get:function(a){var b=m.find.attr(a,"value");return null!=b?b:m.trim(m.text(a))}},select:{get:function(a){for(var b,c,d=a.options,e=a.selectedIndex,f="select-one"===a.type||0>e,g=f?null:[],h=f?e+1:d.length,i=0>e?h:f?e:0;h>i;i++)if(c=d[i],!(!c.selected&&i!==e||(k.optDisabled?c.disabled:null!==c.getAttribute("disabled"))||c.parentNode.disabled&&m.nodeName(c.parentNode,"optgroup"))){if(b=m(c).val(),f)return b;g.push(b)}return g},set:function(a,b){var c,d,e=a.options,f=m.makeArray(b),g=e.length;while(g--)if(d=e[g],m.inArray(m.valHooks.option.get(d),f)>=0)try{d.selected=c=!0}catch(h){d.scrollHeight}else d.selected=!1;return c||(a.selectedIndex=-1),e}}}}),m.each(["radio","checkbox"],function(){m.valHooks[this]={set:function(a,b){return m.isArray(b)?a.checked=m.inArray(m(a).val(),b)>=0:void 0}},k.checkOn||(m.valHooks[this].get=function(a){return null===a.getAttribute("value")?"on":a.value})});var mb,nb,ob=m.expr.attrHandle,pb=/^(?:checked|selected)$/i,qb=k.getSetAttribute,rb=k.input;m.fn.extend({attr:function(a,b){return V(this,m.attr,a,b,arguments.length>1)},removeAttr:function(a){return this.each(function(){m.removeAttr(this,a)})}}),m.extend({attr:function(a,b,c){var d,e,f=a.nodeType;if(a&&3!==f&&8!==f&&2!==f)return typeof a.getAttribute===K?m.prop(a,b,c):(1===f&&m.isXMLDoc(a)||(b=b.toLowerCase(),d=m.attrHooks[b]||(m.expr.match.bool.test(b)?nb:mb)),void 0===c?d&&"get"in d&&null!==(e=d.get(a,b))?e:(e=m.find.attr(a,b),null==e?void 0:e):null!==c?d&&"set"in d&&void 0!==(e=d.set(a,c,b))?e:(a.setAttribute(b,c+""),c):void m.removeAttr(a,b))},removeAttr:function(a,b){var c,d,e=0,f=b&&b.match(E);if(f&&1===a.nodeType)while(c=f[e++])d=m.propFix[c]||c,m.expr.match.bool.test(c)?rb&&qb||!pb.test(c)?a[d]=!1:a[m.camelCase("default-"+c)]=a[d]=!1:m.attr(a,c,""),a.removeAttribute(qb?c:d)},attrHooks:{type:{set:function(a,b){if(!k.radioValue&&"radio"===b&&m.nodeName(a,"input")){var c=a.value;return a.setAttribute("type",b),c&&(a.value=c),b}}}}}),nb={set:function(a,b,c){return b===!1?m.removeAttr(a,c):rb&&qb||!pb.test(c)?a.setAttribute(!qb&&m.propFix[c]||c,c):a[m.camelCase("default-"+c)]=a[c]=!0,c}},m.each(m.expr.match.bool.source.match(/\w+/g),function(a,b){var c=ob[b]||m.find.attr;ob[b]=rb&&qb||!pb.test(b)?function(a,b,d){var e,f;return d||(f=ob[b],ob[b]=e,e=null!=c(a,b,d)?b.toLowerCase():null,ob[b]=f),e}:function(a,b,c){return c?void 0:a[m.camelCase("default-"+b)]?b.toLowerCase():null}}),rb&&qb||(m.attrHooks.value={set:function(a,b,c){return m.nodeName(a,"input")?void(a.defaultValue=b):mb&&mb.set(a,b,c)}}),qb||(mb={set:function(a,b,c){var d=a.getAttributeNode(c);return d||a.setAttributeNode(d=a.ownerDocument.createAttribute(c)),d.value=b+="","value"===c||b===a.getAttribute(c)?b:void 0}},ob.id=ob.name=ob.coords=function(a,b,c){var d;return c?void 0:(d=a.getAttributeNode(b))&&""!==d.value?d.value:null},m.valHooks.button={get:function(a,b){var c=a.getAttributeNode(b);return c&&c.specified?c.value:void 0},set:mb.set},m.attrHooks.contenteditable={set:function(a,b,c){mb.set(a,""===b?!1:b,c)}},m.each(["width","height"],function(a,b){m.attrHooks[b]={set:function(a,c){return""===c?(a.setAttribute(b,"auto"),c):void 0}}})),k.style||(m.attrHooks.style={get:function(a){return a.style.cssText||void 0},set:function(a,b){return a.style.cssText=b+""}});var sb=/^(?:input|select|textarea|button|object)$/i,tb=/^(?:a|area)$/i;m.fn.extend({prop:function(a,b){return V(this,m.prop,a,b,arguments.length>1)},removeProp:function(a){return a=m.propFix[a]||a,this.each(function(){try{this[a]=void 0,delete this[a]}catch(b){}})}}),m.extend({propFix:{"for":"htmlFor","class":"className"},prop:function(a,b,c){var d,e,f,g=a.nodeType;if(a&&3!==g&&8!==g&&2!==g)return f=1!==g||!m.isXMLDoc(a),f&&(b=m.propFix[b]||b,e=m.propHooks[b]),void 0!==c?e&&"set"in e&&void 0!==(d=e.set(a,c,b))?d:a[b]=c:e&&"get"in e&&null!==(d=e.get(a,b))?d:a[b]},propHooks:{tabIndex:{get:function(a){var b=m.find.attr(a,"tabindex");return b?parseInt(b,10):sb.test(a.nodeName)||tb.test(a.nodeName)&&a.href?0:-1}}}}),k.hrefNormalized||m.each(["href","src"],function(a,b){m.propHooks[b]={get:function(a){return a.getAttribute(b,4)}}}),k.optSelected||(m.propHooks.selected={get:function(a){var b=a.parentNode;return b&&(b.selectedIndex,b.parentNode&&b.parentNode.selectedIndex),null}}),m.each(["tabIndex","readOnly","maxLength","cellSpacing","cellPadding","rowSpan","colSpan","useMap","frameBorder","contentEditable"],function(){m.propFix[this.toLowerCase()]=this}),k.enctype||(m.propFix.enctype="encoding");var ub=/[\t\r\n\f]/g;m.fn.extend({addClass:function(a){var b,c,d,e,f,g,h=0,i=this.length,j="string"==typeof a&&a;if(m.isFunction(a))return this.each(function(b){m(this).addClass(a.call(this,b,this.className))});if(j)for(b=(a||"").match(E)||[];i>h;h++)if(c=this[h],d=1===c.nodeType&&(c.className?(" "+c.className+" ").replace(ub," "):" ")){f=0;while(e=b[f++])d.indexOf(" "+e+" ")<0&&(d+=e+" ");g=m.trim(d),c.className!==g&&(c.className=g)}return this},removeClass:function(a){var b,c,d,e,f,g,h=0,i=this.length,j=0===arguments.length||"string"==typeof a&&a;if(m.isFunction(a))return this.each(function(b){m(this).removeClass(a.call(this,b,this.className))});if(j)for(b=(a||"").match(E)||[];i>h;h++)if(c=this[h],d=1===c.nodeType&&(c.className?(" "+c.className+" ").replace(ub," "):"")){f=0;while(e=b[f++])while(d.indexOf(" "+e+" ")>=0)d=d.replace(" "+e+" "," ");g=a?m.trim(d):"",c.className!==g&&(c.className=g)}return this},toggleClass:function(a,b){var c=typeof a;return"boolean"==typeof b&&"string"===c?b?this.addClass(a):this.removeClass(a):this.each(m.isFunction(a)?function(c){m(this).toggleClass(a.call(this,c,this.className,b),b)}:function(){if("string"===c){var b,d=0,e=m(this),f=a.match(E)||[];while(b=f[d++])e.hasClass(b)?e.removeClass(b):e.addClass(b)}else(c===K||"boolean"===c)&&(this.className&&m._data(this,"__className__",this.className),this.className=this.className||a===!1?"":m._data(this,"__className__")||"")})},hasClass:function(a){for(var b=" "+a+" ",c=0,d=this.length;d>c;c++)if(1===this[c].nodeType&&(" "+this[c].className+" ").replace(ub," ").indexOf(b)>=0)return!0;return!1}}),m.each("blur focus focusin focusout load resize scroll unload click dblclick mousedown mouseup mousemove mouseover mouseout mouseenter mouseleave change select submit keydown keypress keyup error contextmenu".split(" "),function(a,b){m.fn[b]=function(a,c){return arguments.length>0?this.on(b,null,a,c):this.trigger(b)}}),m.fn.extend({hover:function(a,b){return this.mouseenter(a).mouseleave(b||a)},bind:function(a,b,c){return this.on(a,null,b,c)},unbind:function(a,b){return this.off(a,null,b)},delegate:function(a,b,c,d){return this.on(b,a,c,d)},undelegate:function(a,b,c){return 1===arguments.length?this.off(a,"**"):this.off(b,a||"**",c)}});var vb=m.now(),wb=/\?/,xb=/(,)|(\[|{)|(}|])|"(?:[^"\\\r\n]|\\["\\\/bfnrt]|\\u[\da-fA-F]{4})*"\s*:?|true|false|null|-?(?!0\d)\d+(?:\.\d+|)(?:[eE][+-]?\d+|)/g;m.parseJSON=function(b){if(a.JSON&&a.JSON.parse)return a.JSON.parse(b+"");var c,d=null,e=m.trim(b+"");return e&&!m.trim(e.replace(xb,function(a,b,e,f){return c&&b&&(d=0),0===d?a:(c=e||b,d+=!f-!e,"")}))?Function("return "+e)():m.error("Invalid JSON: "+b)},m.parseXML=function(b){var c,d;if(!b||"string"!=typeof b)return null;try{a.DOMParser?(d=new DOMParser,c=d.parseFromString(b,"text/xml")):(c=new ActiveXObject("Microsoft.XMLDOM"),c.async="false",c.loadXML(b))}catch(e){c=void 0}return c&&c.documentElement&&!c.getElementsByTagName("parsererror").length||m.error("Invalid XML: "+b),c};var yb,zb,Ab=/#.*$/,Bb=/([?&])_=[^&]*/,Cb=/^(.*?):[ \t]*([^\r\n]*)\r?$/gm,Db=/^(?:about|app|app-storage|.+-extension|file|res|widget):$/,Eb=/^(?:GET|HEAD)$/,Fb=/^\/\//,Gb=/^([\w.+-]+:)(?:\/\/(?:[^\/?#]*@|)([^\/?#:]*)(?::(\d+)|)|)/,Hb={},Ib={},Jb="*/".concat("*");try{zb=location.href}catch(Kb){zb=y.createElement("a"),zb.href="",zb=zb.href}yb=Gb.exec(zb.toLowerCase())||[];function Lb(a){return function(b,c){"string"!=typeof b&&(c=b,b="*");var d,e=0,f=b.toLowerCase().match(E)||[];if(m.isFunction(c))while(d=f[e++])"+"===d.charAt(0)?(d=d.slice(1)||"*",(a[d]=a[d]||[]).unshift(c)):(a[d]=a[d]||[]).push(c)}}function Mb(a,b,c,d){var e={},f=a===Ib;function g(h){var i;return e[h]=!0,m.each(a[h]||[],function(a,h){var j=h(b,c,d);return"string"!=typeof j||f||e[j]?f?!(i=j):void 0:(b.dataTypes.unshift(j),g(j),!1)}),i}return g(b.dataTypes[0])||!e["*"]&&g("*")}function Nb(a,b){var c,d,e=m.ajaxSettings.flatOptions||{};for(d in b)void 0!==b[d]&&((e[d]?a:c||(c={}))[d]=b[d]);return c&&m.extend(!0,a,c),a}function Ob(a,b,c){var d,e,f,g,h=a.contents,i=a.dataTypes;while("*"===i[0])i.shift(),void 0===e&&(e=a.mimeType||b.getResponseHeader("Content-Type"));if(e)for(g in h)if(h[g]&&h[g].test(e)){i.unshift(g);break}if(i[0]in c)f=i[0];else{for(g in c){if(!i[0]||a.converters[g+" "+i[0]]){f=g;break}d||(d=g)}f=f||d}return f?(f!==i[0]&&i.unshift(f),c[f]):void 0}function Pb(a,b,c,d){var e,f,g,h,i,j={},k=a.dataTypes.slice();if(k[1])for(g in a.converters)j[g.toLowerCase()]=a.converters[g];f=k.shift();while(f)if(a.responseFields[f]&&(c[a.responseFields[f]]=b),!i&&d&&a.dataFilter&&(b=a.dataFilter(b,a.dataType)),i=f,f=k.shift())if("*"===f)f=i;else if("*"!==i&&i!==f){if(g=j[i+" "+f]||j["* "+f],!g)for(e in j)if(h=e.split(" "),h[1]===f&&(g=j[i+" "+h[0]]||j["* "+h[0]])){g===!0?g=j[e]:j[e]!==!0&&(f=h[0],k.unshift(h[1]));break}if(g!==!0)if(g&&a["throws"])b=g(b);else try{b=g(b)}catch(l){return{state:"parsererror",error:g?l:"No conversion from "+i+" to "+f}}}return{state:"success",data:b}}m.extend({active:0,lastModified:{},etag:{},ajaxSettings:{url:zb,type:"GET",isLocal:Db.test(yb[1]),global:!0,processData:!0,async:!0,contentType:"application/x-www-form-urlencoded; charset=UTF-8",accepts:{"*":Jb,text:"text/plain",html:"text/html",xml:"application/xml, text/xml",json:"application/json, text/javascript"},contents:{xml:/xml/,html:/html/,json:/json/},responseFields:{xml:"responseXML",text:"responseText",json:"responseJSON"},converters:{"* text":String,"text html":!0,"text json":m.parseJSON,"text xml":m.parseXML},flatOptions:{url:!0,context:!0}},ajaxSetup:function(a,b){return b?Nb(Nb(a,m.ajaxSettings),b):Nb(m.ajaxSettings,a)},ajaxPrefilter:Lb(Hb),ajaxTransport:Lb(Ib),ajax:function(a,b){"object"==typeof a&&(b=a,a=void 0),b=b||{};var c,d,e,f,g,h,i,j,k=m.ajaxSetup({},b),l=k.context||k,n=k.context&&(l.nodeType||l.jquery)?m(l):m.event,o=m.Deferred(),p=m.Callbacks("once memory"),q=k.statusCode||{},r={},s={},t=0,u="canceled",v={readyState:0,getResponseHeader:function(a){var b;if(2===t){if(!j){j={};while(b=Cb.exec(f))j[b[1].toLowerCase()]=b[2]}b=j[a.toLowerCase()]}return null==b?null:b},getAllResponseHeaders:function(){return 2===t?f:null},setRequestHeader:function(a,b){var c=a.toLowerCase();return t||(a=s[c]=s[c]||a,r[a]=b),this},overrideMimeType:function(a){return t||(k.mimeType=a),this},statusCode:function(a){var b;if(a)if(2>t)for(b in a)q[b]=[q[b],a[b]];else v.always(a[v.status]);return this},abort:function(a){var b=a||u;return i&&i.abort(b),x(0,b),this}};if(o.promise(v).complete=p.add,v.success=v.done,v.error=v.fail,k.url=((a||k.url||zb)+"").replace(Ab,"").replace(Fb,yb[1]+"//"),k.type=b.method||b.type||k.method||k.type,k.dataTypes=m.trim(k.dataType||"*").toLowerCase().match(E)||[""],null==k.crossDomain&&(c=Gb.exec(k.url.toLowerCase()),k.crossDomain=!(!c||c[1]===yb[1]&&c[2]===yb[2]&&(c[3]||("http:"===c[1]?"80":"443"))===(yb[3]||("http:"===yb[1]?"80":"443")))),k.data&&k.processData&&"string"!=typeof k.data&&(k.data=m.param(k.data,k.traditional)),Mb(Hb,k,b,v),2===t)return v;h=m.event&&k.global,h&&0===m.active++&&m.event.trigger("ajaxStart"),k.type=k.type.toUpperCase(),k.hasContent=!Eb.test(k.type),e=k.url,k.hasContent||(k.data&&(e=k.url+=(wb.test(e)?"&":"?")+k.data,delete k.data),k.cache===!1&&(k.url=Bb.test(e)?e.replace(Bb,"$1_="+vb++):e+(wb.test(e)?"&":"?")+"_="+vb++)),k.ifModified&&(m.lastModified[e]&&v.setRequestHeader("If-Modified-Since",m.lastModified[e]),m.etag[e]&&v.setRequestHeader("If-None-Match",m.etag[e])),(k.data&&k.hasContent&&k.contentType!==!1||b.contentType)&&v.setRequestHeader("Content-Type",k.contentType),v.setRequestHeader("Accept",k.dataTypes[0]&&k.accepts[k.dataTypes[0]]?k.accepts[k.dataTypes[0]]+("*"!==k.dataTypes[0]?", "+Jb+"; q=0.01":""):k.accepts["*"]);for(d in k.headers)v.setRequestHeader(d,k.headers[d]);if(k.beforeSend&&(k.beforeSend.call(l,v,k)===!1||2===t))return v.abort();u="abort";for(d in{success:1,error:1,complete:1})v[d](k[d]);if(i=Mb(Ib,k,b,v)){v.readyState=1,h&&n.trigger("ajaxSend",[v,k]),k.async&&k.timeout>0&&(g=setTimeout(function(){v.abort("timeout")},k.timeout));try{t=1,i.send(r,x)}catch(w){if(!(2>t))throw w;x(-1,w)}}else x(-1,"No Transport");function x(a,b,c,d){var j,r,s,u,w,x=b;2!==t&&(t=2,g&&clearTimeout(g),i=void 0,f=d||"",v.readyState=a>0?4:0,j=a>=200&&300>a||304===a,c&&(u=Ob(k,v,c)),u=Pb(k,u,v,j),j?(k.ifModified&&(w=v.getResponseHeader("Last-Modified"),w&&(m.lastModified[e]=w),w=v.getResponseHeader("etag"),w&&(m.etag[e]=w)),204===a||"HEAD"===k.type?x="nocontent":304===a?x="notmodified":(x=u.state,r=u.data,s=u.error,j=!s)):(s=x,(a||!x)&&(x="error",0>a&&(a=0))),v.status=a,v.statusText=(b||x)+"",j?o.resolveWith(l,[r,x,v]):o.rejectWith(l,[v,x,s]),v.statusCode(q),q=void 0,h&&n.trigger(j?"ajaxSuccess":"ajaxError",[v,k,j?r:s]),p.fireWith(l,[v,x]),h&&(n.trigger("ajaxComplete",[v,k]),--m.active||m.event.trigger("ajaxStop")))}return v},getJSON:function(a,b,c){return m.get(a,b,c,"json")},getScript:function(a,b){return m.get(a,void 0,b,"script")}}),m.each(["get","post"],function(a,b){m[b]=function(a,c,d,e){return m.isFunction(c)&&(e=e||d,d=c,c=void 0),m.ajax({url:a,type:b,dataType:e,data:c,success:d})}}),m._evalUrl=function(a){return m.ajax({url:a,type:"GET",dataType:"script",async:!1,global:!1,"throws":!0})},m.fn.extend({wrapAll:function(a){if(m.isFunction(a))return this.each(function(b){m(this).wrapAll(a.call(this,b))});if(this[0]){var b=m(a,this[0].ownerDocument).eq(0).clone(!0);this[0].parentNode&&b.insertBefore(this[0]),b.map(function(){var a=this;while(a.firstChild&&1===a.firstChild.nodeType)a=a.firstChild;return a}).append(this)}return this},wrapInner:function(a){return this.each(m.isFunction(a)?function(b){m(this).wrapInner(a.call(this,b))}:function(){var b=m(this),c=b.contents();c.length?c.wrapAll(a):b.append(a)})},wrap:function(a){var b=m.isFunction(a);return this.each(function(c){m(this).wrapAll(b?a.call(this,c):a)})},unwrap:function(){return this.parent().each(function(){m.nodeName(this,"body")||m(this).replaceWith(this.childNodes)}).end()}}),m.expr.filters.hidden=function(a){return a.offsetWidth<=0&&a.offsetHeight<=0||!k.reliableHiddenOffsets()&&"none"===(a.style&&a.style.display||m.css(a,"display"))},m.expr.filters.visible=function(a){return!m.expr.filters.hidden(a)};var Qb=/%20/g,Rb=/\[\]$/,Sb=/\r?\n/g,Tb=/^(?:submit|button|image|reset|file)$/i,Ub=/^(?:input|select|textarea|keygen)/i;function Vb(a,b,c,d){var e;if(m.isArray(b))m.each(b,function(b,e){c||Rb.test(a)?d(a,e):Vb(a+"["+("object"==typeof e?b:"")+"]",e,c,d)});else if(c||"object"!==m.type(b))d(a,b);else for(e in b)Vb(a+"["+e+"]",b[e],c,d)}m.param=function(a,b){var c,d=[],e=function(a,b){b=m.isFunction(b)?b():null==b?"":b,d[d.length]=encodeURIComponent(a)+"="+encodeURIComponent(b)};if(void 0===b&&(b=m.ajaxSettings&&m.ajaxSettings.traditional),m.isArray(a)||a.jquery&&!m.isPlainObject(a))m.each(a,function(){e(this.name,this.value)});else for(c in a)Vb(c,a[c],b,e);return d.join("&").replace(Qb,"+")},m.fn.extend({serialize:function(){return m.param(this.serializeArray())},serializeArray:function(){return this.map(function(){var a=m.prop(this,"elements");return a?m.makeArray(a):this}).filter(function(){var a=this.type;return this.name&&!m(this).is(":disabled")&&Ub.test(this.nodeName)&&!Tb.test(a)&&(this.checked||!W.test(a))}).map(function(a,b){var c=m(this).val();return null==c?null:m.isArray(c)?m.map(c,function(a){return{name:b.name,value:a.replace(Sb,"\r\n")}}):{name:b.name,value:c.replace(Sb,"\r\n")}}).get()}}),m.ajaxSettings.xhr=void 0!==a.ActiveXObject?function(){return!this.isLocal&&/^(get|post|head|put|delete|options)$/i.test(this.type)&&Zb()||$b()}:Zb;var Wb=0,Xb={},Yb=m.ajaxSettings.xhr();a.attachEvent&&a.attachEvent("onunload",function(){for(var a in Xb)Xb[a](void 0,!0)}),k.cors=!!Yb&&"withCredentials"in Yb,Yb=k.ajax=!!Yb,Yb&&m.ajaxTransport(function(a){if(!a.crossDomain||k.cors){var b;return{send:function(c,d){var e,f=a.xhr(),g=++Wb;if(f.open(a.type,a.url,a.async,a.username,a.password),a.xhrFields)for(e in a.xhrFields)f[e]=a.xhrFields[e];a.mimeType&&f.overrideMimeType&&f.overrideMimeType(a.mimeType),a.crossDomain||c["X-Requested-With"]||(c["X-Requested-With"]="XMLHttpRequest");for(e in c)void 0!==c[e]&&f.setRequestHeader(e,c[e]+"");f.send(a.hasContent&&a.data||null),b=function(c,e){var h,i,j;if(b&&(e||4===f.readyState))if(delete Xb[g],b=void 0,f.onreadystatechange=m.noop,e)4!==f.readyState&&f.abort();else{j={},h=f.status,"string"==typeof f.responseText&&(j.text=f.responseText);try{i=f.statusText}catch(k){i=""}h||!a.isLocal||a.crossDomain?1223===h&&(h=204):h=j.text?200:404}j&&d(h,i,j,f.getAllResponseHeaders())},a.async?4===f.readyState?setTimeout(b):f.onreadystatechange=Xb[g]=b:b()},abort:function(){b&&b(void 0,!0)}}}});function Zb(){try{return new a.XMLHttpRequest}catch(b){}}function $b(){try{return new a.ActiveXObject("Microsoft.XMLHTTP")}catch(b){}}m.ajaxSetup({accepts:{script:"text/javascript, application/javascript, application/ecmascript, application/x-ecmascript"},contents:{script:/(?:java|ecma)script/},converters:{"text script":function(a){return m.globalEval(a),a}}}),m.ajaxPrefilter("script",function(a){void 0===a.cache&&(a.cache=!1),a.crossDomain&&(a.type="GET",a.global=!1)}),m.ajaxTransport("script",function(a){if(a.crossDomain){var b,c=y.head||m("head")[0]||y.documentElement;return{send:function(d,e){b=y.createElement("script"),b.async=!0,a.scriptCharset&&(b.charset=a.scriptCharset),b.src=a.url,b.onload=b.onreadystatechange=function(a,c){(c||!b.readyState||/loaded|complete/.test(b.readyState))&&(b.onload=b.onreadystatechange=null,b.parentNode&&b.parentNode.removeChild(b),b=null,c||e(200,"success"))},c.insertBefore(b,c.firstChild)},abort:function(){b&&b.onload(void 0,!0)}}}});var _b=[],ac=/(=)\?(?=&|$)|\?\?/;m.ajaxSetup({jsonp:"callback",jsonpCallback:function(){var a=_b.pop()||m.expando+"_"+vb++;return this[a]=!0,a}}),m.ajaxPrefilter("json jsonp",function(b,c,d){var e,f,g,h=b.jsonp!==!1&&(ac.test(b.url)?"url":"string"==typeof b.data&&!(b.contentType||"").indexOf("application/x-www-form-urlencoded")&&ac.test(b.data)&&"data");return h||"jsonp"===b.dataTypes[0]?(e=b.jsonpCallback=m.isFunction(b.jsonpCallback)?b.jsonpCallback():b.jsonpCallback,h?b[h]=b[h].replace(ac,"$1"+e):b.jsonp!==!1&&(b.url+=(wb.test(b.url)?"&":"?")+b.jsonp+"="+e),b.converters["script json"]=function(){return g||m.error(e+" was not called"),g[0]},b.dataTypes[0]="json",f=a[e],a[e]=function(){g=arguments},d.always(function(){a[e]=f,b[e]&&(b.jsonpCallback=c.jsonpCallback,_b.push(e)),g&&m.isFunction(f)&&f(g[0]),g=f=void 0}),"script"):void 0}),m.parseHTML=function(a,b,c){if(!a||"string"!=typeof a)return null;"boolean"==typeof b&&(c=b,b=!1),b=b||y;var d=u.exec(a),e=!c&&[];return d?[b.createElement(d[1])]:(d=m.buildFragment([a],b,e),e&&e.length&&m(e).remove(),m.merge([],d.childNodes))};var bc=m.fn.load;m.fn.load=function(a,b,c){if("string"!=typeof a&&bc)return bc.apply(this,arguments);var d,e,f,g=this,h=a.indexOf(" ");return h>=0&&(d=m.trim(a.slice(h,a.length)),a=a.slice(0,h)),m.isFunction(b)?(c=b,b=void 0):b&&"object"==typeof b&&(f="POST"),g.length>0&&m.ajax({url:a,type:f,dataType:"html",data:b}).done(function(a){e=arguments,g.html(d?m("<div>").append(m.parseHTML(a)).find(d):a)}).complete(c&&function(a,b){g.each(c,e||[a.responseText,b,a])}),this},m.each(["ajaxStart","ajaxStop","ajaxComplete","ajaxError","ajaxSuccess","ajaxSend"],function(a,b){m.fn[b]=function(a){return this.on(b,a)}}),m.expr.filters.animated=function(a){return m.grep(m.timers,function(b){return a===b.elem}).length};var cc=a.document.documentElement;function dc(a){return m.isWindow(a)?a:9===a.nodeType?a.defaultView||a.parentWindow:!1}m.offset={setOffset:function(a,b,c){var d,e,f,g,h,i,j,k=m.css(a,"position"),l=m(a),n={};"static"===k&&(a.style.position="relative"),h=l.offset(),f=m.css(a,"top"),i=m.css(a,"left"),j=("absolute"===k||"fixed"===k)&&m.inArray("auto",[f,i])>-1,j?(d=l.position(),g=d.top,e=d.left):(g=parseFloat(f)||0,e=parseFloat(i)||0),m.isFunction(b)&&(b=b.call(a,c,h)),null!=b.top&&(n.top=b.top-h.top+g),null!=b.left&&(n.left=b.left-h.left+e),"using"in b?b.using.call(a,n):l.css(n)}},m.fn.extend({offset:function(a){if(arguments.length)return void 0===a?this:this.each(function(b){m.offset.setOffset(this,a,b)});var b,c,d={top:0,left:0},e=this[0],f=e&&e.ownerDocument;if(f)return b=f.documentElement,m.contains(b,e)?(typeof e.getBoundingClientRect!==K&&(d=e.getBoundingClientRect()),c=dc(f),{top:d.top+(c.pageYOffset||b.scrollTop)-(b.clientTop||0),left:d.left+(c.pageXOffset||b.scrollLeft)-(b.clientLeft||0)}):d},position:function(){if(this[0]){var a,b,c={top:0,left:0},d=this[0];return"fixed"===m.css(d,"position")?b=d.getBoundingClientRect():(a=this.offsetParent(),b=this.offset(),m.nodeName(a[0],"html")||(c=a.offset()),c.top+=m.css(a[0],"borderTopWidth",!0),c.left+=m.css(a[0],"borderLeftWidth",!0)),{top:b.top-c.top-m.css(d,"marginTop",!0),left:b.left-c.left-m.css(d,"marginLeft",!0)}}},offsetParent:function(){return this.map(function(){var a=this.offsetParent||cc;while(a&&!m.nodeName(a,"html")&&"static"===m.css(a,"position"))a=a.offsetParent;return a||cc})}}),m.each({scrollLeft:"pageXOffset",scrollTop:"pageYOffset"},function(a,b){var c=/Y/.test(b);m.fn[a]=function(d){return V(this,function(a,d,e){var f=dc(a);return void 0===e?f?b in f?f[b]:f.document.documentElement[d]:a[d]:void(f?f.scrollTo(c?m(f).scrollLeft():e,c?e:m(f).scrollTop()):a[d]=e)},a,d,arguments.length,null)}}),m.each(["top","left"],function(a,b){m.cssHooks[b]=La(k.pixelPosition,function(a,c){return c?(c=Ja(a,b),Ha.test(c)?m(a).position()[b]+"px":c):void 0})}),m.each({Height:"height",Width:"width"},function(a,b){m.each({padding:"inner"+a,content:b,"":"outer"+a},function(c,d){m.fn[d]=function(d,e){var f=arguments.length&&(c||"boolean"!=typeof d),g=c||(d===!0||e===!0?"margin":"border");return V(this,function(b,c,d){var e;return m.isWindow(b)?b.document.documentElement["client"+a]:9===b.nodeType?(e=b.documentElement,Math.max(b.body["scroll"+a],e["scroll"+a],b.body["offset"+a],e["offset"+a],e["client"+a])):void 0===d?m.css(b,c,g):m.style(b,c,d,g)},b,f?d:void 0,f,null)}})}),m.fn.size=function(){return this.length},m.fn.andSelf=m.fn.addBack,"function"==typeof define&&define.amd&&define("jquery",[],function(){return m});var ec=a.jQuery,fc=a.$;return m.noConflict=function(b){return a.$===m&&(a.$=fc),b&&a.jQuery===m&&(a.jQuery=ec),m},typeof b===K&&(a.jQuery=a.$=m),m});
-</script>
+<script src="data:application/x-javascript;base64,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"></script>
 <meta name="viewport" content="width=device-width, initial-scale=1" />
-<style type="text/css">html{font-family:sans-serif;-webkit-text-size-adjust:100%;-ms-text-size-adjust:100%}body{margin:0}article,aside,details,figcaption,figure,footer,header,hgroup,main,menu,nav,section,summary{display:block}audio,canvas,progress,video{display:inline-block;vertical-align:baseline}audio:not([controls]){display:none;height:0}[hidden],template{display:none}a{background-color:transparent}a:active,a:hover{outline:0}abbr[title]{border-bottom:1px dotted}b,strong{font-weight:700}dfn{font-style:italic}h1{margin:.67em 0;font-size:2em}mark{color:#000;background:#ff0}small{font-size:80%}sub,sup{position:relative;font-size:75%;line-height:0;vertical-align:baseline}sup{top:-.5em}sub{bottom:-.25em}img{border:0}svg:not(:root){overflow:hidden}figure{margin:1em 40px}hr{height:0;-webkit-box-sizing:content-box;-moz-box-sizing:content-box;box-sizing:content-box}pre{overflow:auto}code,kbd,pre,samp{font-family:monospace,monospace;font-size:1em}button,input,optgroup,select,textarea{margin:0;font:inherit;color:inherit}button{overflow:visible}button,select{text-transform:none}button,html input[type=button],input[type=reset],input[type=submit]{-webkit-appearance:button;cursor:pointer}button[disabled],html input[disabled]{cursor:default}button::-moz-focus-inner,input::-moz-focus-inner{padding:0;border:0}input{line-height:normal}input[type=checkbox],input[type=radio]{-webkit-box-sizing:border-box;-moz-box-sizing:border-box;box-sizing:border-box;padding:0}input[type=number]::-webkit-inner-spin-button,input[type=number]::-webkit-outer-spin-button{height:auto}input[type=search]{-webkit-box-sizing:content-box;-moz-box-sizing:content-box;box-sizing:content-box;-webkit-appearance:textfield}input[type=search]::-webkit-search-cancel-button,input[type=search]::-webkit-search-decoration{-webkit-appearance:none}fieldset{padding:.35em .625em .75em;margin:0 2px;border:1px solid silver}legend{padding:0;border:0}textarea{overflow:auto}optgroup{font-weight:700}table{border-spacing:0;border-collapse:collapse}td,th{padding:0}@media print{*,:after,:before{color:#000!important;text-shadow:none!important;background:0 0!important;-webkit-box-shadow:none!important;box-shadow:none!important}a,a:visited{text-decoration:underline}a[href]:after{content:" (" attr(href) ")"}abbr[title]:after{content:" (" attr(title) ")"}a[href^="javascript:"]:after,a[href^="#"]:after{content:""}blockquote,pre{border:1px solid #999;page-break-inside:avoid}thead{display:table-header-group}img,tr{page-break-inside:avoid}img{max-width:100%!important}h2,h3,p{orphans:3;widows:3}h2,h3{page-break-after:avoid}.navbar{display:none}.btn>.caret,.dropup>.btn>.caret{border-top-color:#000!important}.label{border:1px solid #000}.table{border-collapse:collapse!important}.table td,.table th{background-color:#fff!important}.table-bordered td,.table-bordered th{border:1px solid #ddd!important}}@font-face{font-family:'Glyphicons Halflings';src:url(data:application/vnd.ms-fontobject;base64,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);src:url(data:application/vnd.ms-fontobject;base64,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) format('embedded-opentype'),url(data:application/font-woff;base64,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) format('woff'),url(data:application/x-font-truetype;base64,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) format('truetype'),url(data:image/svg+xml;base64,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) format('svg')}.glyphicon{position:relative;top:1px;display:inline-block;font-family:'Glyphicons Halflings';font-style:normal;font-weight:400;line-height:1;-webkit-font-smoothing:antialiased;-moz-osx-font-smoothing:grayscale}.glyphicon-asterisk:before{content:"\2a"}.glyphicon-plus:before{content:"\2b"}.glyphicon-eur:before,.glyphicon-euro:before{content:"\20ac"}.glyphicon-minus:before{content:"\2212"}.glyphicon-cloud:before{content:"\2601"}.glyphicon-envelope:before{content:"\2709"}.glyphicon-pencil:before{content:"\270f"}.glyphicon-glass:before{content:"\e001"}.glyphicon-music:before{content:"\e002"}.glyphicon-search:before{content:"\e003"}.glyphicon-heart:before{content:"\e005"}.glyphicon-star:before{content:"\e006"}.glyphicon-star-empty:before{content:"\e007"}.glyphicon-user:before{content:"\e008"}.glyphicon-film:before{content:"\e009"}.glyphicon-th-large:before{content:"\e010"}.glyphicon-th:before{content:"\e011"}.glyphicon-th-list:before{content:"\e012"}.glyphicon-ok:before{content:"\e013"}.glyphicon-remove:before{content:"\e014"}.glyphicon-zoom-in:before{content:"\e015"}.glyphicon-zoom-out:before{content:"\e016"}.glyphicon-off:before{content:"\e017"}.glyphicon-signal:before{content:"\e018"}.glyphicon-cog:before{content:"\e019"}.glyphicon-trash:before{content:"\e020"}.glyphicon-home:before{content:"\e021"}.glyphicon-file:before{content:"\e022"}.glyphicon-time:before{content:"\e023"}.glyphicon-road:before{content:"\e024"}.glyphicon-download-alt:before{content:"\e025"}.glyphicon-download:before{content:"\e026"}.glyphicon-upload:before{content:"\e027"}.glyphicon-inbox:before{content:"\e028"}.glyphicon-play-circle:before{content:"\e029"}.glyphicon-repeat:before{content:"\e030"}.glyphicon-refresh:before{content:"\e031"}.glyphicon-list-alt:before{content:"\e032"}.glyphicon-lock:before{content:"\e033"}.glyphicon-flag:before{content:"\e034"}.glyphicon-headphones:before{content:"\e035"}.glyphicon-volume-off:before{content:"\e036"}.glyphicon-volume-down:before{content:"\e037"}.glyphicon-volume-up:before{content:"\e038"}.glyphicon-qrcode:before{content:"\e039"}.glyphicon-barcode:before{content:"\e040"}.glyphicon-tag:before{content:"\e041"}.glyphicon-tags:before{content:"\e042"}.glyphicon-book:before{content:"\e043"}.glyphicon-bookmark:before{content:"\e044"}.glyphicon-print:before{content:"\e045"}.glyphicon-camera:before{content:"\e046"}.glyphicon-font:before{content:"\e047"}.glyphicon-bold:before{content:"\e048"}.glyphicon-italic:before{content:"\e049"}.glyphicon-text-height:before{content:"\e050"}.glyphicon-text-width:before{content:"\e051"}.glyphicon-align-left:before{content:"\e052"}.glyphicon-align-center:before{content:"\e053"}.glyphicon-align-right:before{content:"\e054"}.glyphicon-align-justify:before{content:"\e055"}.glyphicon-list:before{content:"\e056"}.glyphicon-indent-left:before{content:"\e057"}.glyphicon-indent-right:before{content:"\e058"}.glyphicon-facetime-video:before{content:"\e059"}.glyphicon-picture:before{content:"\e060"}.glyphicon-map-marker:before{content:"\e062"}.glyphicon-adjust:before{content:"\e063"}.glyphicon-tint:before{content:"\e064"}.glyphicon-edit:before{content:"\e065"}.glyphicon-share:before{content:"\e066"}.glyphicon-check:before{content:"\e067"}.glyphicon-move:before{content:"\e068"}.glyphicon-step-backward:before{content:"\e069"}.glyphicon-fast-backward:before{content:"\e070"}.glyphicon-backward:before{content:"\e071"}.glyphicon-play:before{content:"\e072"}.glyphicon-pause:before{content:"\e073"}.glyphicon-stop:before{content:"\e074"}.glyphicon-forward:before{content:"\e075"}.glyphicon-fast-forward:before{content:"\e076"}.glyphicon-step-forward:before{content:"\e077"}.glyphicon-eject:before{content:"\e078"}.glyphicon-chevron-left:before{content:"\e079"}.glyphicon-chevron-right:before{content:"\e080"}.glyphicon-plus-sign:before{content:"\e081"}.glyphicon-minus-sign:before{content:"\e082"}.glyphicon-remove-sign:before{content:"\e083"}.glyphicon-ok-sign:before{content:"\e084"}.glyphicon-question-sign:before{content:"\e085"}.glyphicon-info-sign:before{content:"\e086"}.glyphicon-screenshot:before{content:"\e087"}.glyphicon-remove-circle:before{content:"\e088"}.glyphicon-ok-circle:before{content:"\e089"}.glyphicon-ban-circle:before{content:"\e090"}.glyphicon-arrow-left:before{content:"\e091"}.glyphicon-arrow-right:before{content:"\e092"}.glyphicon-arrow-up:before{content:"\e093"}.glyphicon-arrow-down:before{content:"\e094"}.glyphicon-share-alt:before{content:"\e095"}.glyphicon-resize-full:before{content:"\e096"}.glyphicon-resize-small:before{content:"\e097"}.glyphicon-exclamation-sign:before{content:"\e101"}.glyphicon-gift:before{content:"\e102"}.glyphicon-leaf:before{content:"\e103"}.glyphicon-fire:before{content:"\e104"}.glyphicon-eye-open:before{content:"\e105"}.glyphicon-eye-close:before{content:"\e106"}.glyphicon-warning-sign:before{content:"\e107"}.glyphicon-plane:before{content:"\e108"}.glyphicon-calendar:before{content:"\e109"}.glyphicon-random:before{content:"\e110"}.glyphicon-comment:before{content:"\e111"}.glyphicon-magnet:before{content:"\e112"}.glyphicon-chevron-up:before{content:"\e113"}.glyphicon-chevron-down:before{content:"\e114"}.glyphicon-retweet:before{content:"\e115"}.glyphicon-shopping-cart:before{content:"\e116"}.glyphicon-folder-close:before{content:"\e117"}.glyphicon-folder-open:before{content:"\e118"}.glyphicon-resize-vertical:before{content:"\e119"}.glyphicon-resize-horizontal:before{content:"\e120"}.glyphicon-hdd:before{content:"\e121"}.glyphicon-bullhorn:before{content:"\e122"}.glyphicon-bell:before{content:"\e123"}.glyphicon-certificate:before{content:"\e124"}.glyphicon-thumbs-up:before{content:"\e125"}.glyphicon-thumbs-down:before{content:"\e126"}.glyphicon-hand-right:before{content:"\e127"}.glyphicon-hand-left:before{content:"\e128"}.glyphicon-hand-up:before{content:"\e129"}.glyphicon-hand-down:before{content:"\e130"}.glyphicon-circle-arrow-right:before{content:"\e131"}.glyphicon-circle-arrow-left:before{content:"\e132"}.glyphicon-circle-arrow-up:before{content:"\e133"}.glyphicon-circle-arrow-down:before{content:"\e134"}.glyphicon-globe:before{content:"\e135"}.glyphicon-wrench:before{content:"\e136"}.glyphicon-tasks:before{content:"\e137"}.glyphicon-filter:before{content:"\e138"}.glyphicon-briefcase:before{content:"\e139"}.glyphicon-fullscreen:before{content:"\e140"}.glyphicon-dashboard:before{content:"\e141"}.glyphicon-paperclip:before{content:"\e142"}.glyphicon-heart-empty:before{content:"\e143"}.glyphicon-link:before{content:"\e144"}.glyphicon-phone:before{content:"\e145"}.glyphicon-pushpin:before{content:"\e146"}.glyphicon-usd:before{content:"\e148"}.glyphicon-gbp:before{content:"\e149"}.glyphicon-sort:before{content:"\e150"}.glyphicon-sort-by-alphabet:before{content:"\e151"}.glyphicon-sort-by-alphabet-alt:before{content:"\e152"}.glyphicon-sort-by-order:before{content:"\e153"}.glyphicon-sort-by-order-alt:before{content:"\e154"}.glyphicon-sort-by-attributes:before{content:"\e155"}.glyphicon-sort-by-attributes-alt:before{content:"\e156"}.glyphicon-unchecked:before{content:"\e157"}.glyphicon-expand:before{content:"\e158"}.glyphicon-collapse-down:before{content:"\e159"}.glyphicon-collapse-up:before{content:"\e160"}.glyphicon-log-in:before{content:"\e161"}.glyphicon-flash:before{content:"\e162"}.glyphicon-log-out:before{content:"\e163"}.glyphicon-new-window:before{content:"\e164"}.glyphicon-record:before{content:"\e165"}.glyphicon-save:before{content:"\e166"}.glyphicon-open:before{content:"\e167"}.glyphicon-saved:before{content:"\e168"}.glyphicon-import:before{content:"\e169"}.glyphicon-export:before{content:"\e170"}.glyphicon-send:before{content:"\e171"}.glyphicon-floppy-disk:before{content:"\e172"}.glyphicon-floppy-saved:before{content:"\e173"}.glyphicon-floppy-remove:before{content:"\e174"}.glyphicon-floppy-save:before{content:"\e175"}.glyphicon-floppy-open:before{content:"\e176"}.glyphicon-credit-card:before{content:"\e177"}.glyphicon-transfer:before{content:"\e178"}.glyphicon-cutlery:before{content:"\e179"}.glyphicon-header:before{content:"\e180"}.glyphicon-compressed:before{content:"\e181"}.glyphicon-earphone:before{content:"\e182"}.glyphicon-phone-alt:before{content:"\e183"}.glyphicon-tower:before{content:"\e184"}.glyphicon-stats:before{content:"\e185"}.glyphicon-sd-video:before{content:"\e186"}.glyphicon-hd-video:before{content:"\e187"}.glyphicon-subtitles:before{content:"\e188"}.glyphicon-sound-stereo:before{content:"\e189"}.glyphicon-sound-dolby:before{content:"\e190"}.glyphicon-sound-5-1:before{content:"\e191"}.glyphicon-sound-6-1:before{content:"\e192"}.glyphicon-sound-7-1:before{content:"\e193"}.glyphicon-copyright-mark:before{content:"\e194"}.glyphicon-registration-mark:before{content:"\e195"}.glyphicon-cloud-download:before{content:"\e197"}.glyphicon-cloud-upload:before{content:"\e198"}.glyphicon-tree-conifer:before{content:"\e199"}.glyphicon-tree-deciduous:before{content:"\e200"}.glyphicon-cd:before{content:"\e201"}.glyphicon-save-file:before{content:"\e202"}.glyphicon-open-file:before{content:"\e203"}.glyphicon-level-up:before{content:"\e204"}.glyphicon-copy:before{content:"\e205"}.glyphicon-paste:before{content:"\e206"}.glyphicon-alert:before{content:"\e209"}.glyphicon-equalizer:before{content:"\e210"}.glyphicon-king:before{content:"\e211"}.glyphicon-queen:before{content:"\e212"}.glyphicon-pawn:before{content:"\e213"}.glyphicon-bishop:before{content:"\e214"}.glyphicon-knight:before{content:"\e215"}.glyphicon-baby-formula:before{content:"\e216"}.glyphicon-tent:before{content:"\26fa"}.glyphicon-blackboard:before{content:"\e218"}.glyphicon-bed:before{content:"\e219"}.glyphicon-apple:before{content:"\f8ff"}.glyphicon-erase:before{content:"\e221"}.glyphicon-hourglass:before{content:"\231b"}.glyphicon-lamp:before{content:"\e223"}.glyphicon-duplicate:before{content:"\e224"}.glyphicon-piggy-bank:before{content:"\e225"}.glyphicon-scissors:before{content:"\e226"}.glyphicon-bitcoin:before{content:"\e227"}.glyphicon-btc:before{content:"\e227"}.glyphicon-xbt:before{content:"\e227"}.glyphicon-yen:before{content:"\00a5"}.glyphicon-jpy:before{content:"\00a5"}.glyphicon-ruble:before{content:"\20bd"}.glyphicon-rub:before{content:"\20bd"}.glyphicon-scale:before{content:"\e230"}.glyphicon-ice-lolly:before{content:"\e231"}.glyphicon-ice-lolly-tasted:before{content:"\e232"}.glyphicon-education:before{content:"\e233"}.glyphicon-option-horizontal:before{content:"\e234"}.glyphicon-option-vertical:before{content:"\e235"}.glyphicon-menu-hamburger:before{content:"\e236"}.glyphicon-modal-window:before{content:"\e237"}.glyphicon-oil:before{content:"\e238"}.glyphicon-grain:before{content:"\e239"}.glyphicon-sunglasses:before{content:"\e240"}.glyphicon-text-size:before{content:"\e241"}.glyphicon-text-color:before{content:"\e242"}.glyphicon-text-background:before{content:"\e243"}.glyphicon-object-align-top:before{content:"\e244"}.glyphicon-object-align-bottom:before{content:"\e245"}.glyphicon-object-align-horizontal:before{content:"\e246"}.glyphicon-object-align-left:before{content:"\e247"}.glyphicon-object-align-vertical:before{content:"\e248"}.glyphicon-object-align-right:before{content:"\e249"}.glyphicon-triangle-right:before{content:"\e250"}.glyphicon-triangle-left:before{content:"\e251"}.glyphicon-triangle-bottom:before{content:"\e252"}.glyphicon-triangle-top:before{content:"\e253"}.glyphicon-console:before{content:"\e254"}.glyphicon-superscript:before{content:"\e255"}.glyphicon-subscript:before{content:"\e256"}.glyphicon-menu-left:before{content:"\e257"}.glyphicon-menu-right:before{content:"\e258"}.glyphicon-menu-down:before{content:"\e259"}.glyphicon-menu-up:before{content:"\e260"}*{-webkit-box-sizing:border-box;-moz-box-sizing:border-box;box-sizing:border-box}:after,:before{-webkit-box-sizing:border-box;-moz-box-sizing:border-box;box-sizing:border-box}html{font-size:10px;-webkit-tap-highlight-color:rgba(0,0,0,0)}body{font-family:"Helvetica Neue",Helvetica,Arial,sans-serif;font-size:14px;line-height:1.42857143;color:#333;background-color:#fff}button,input,select,textarea{font-family:inherit;font-size:inherit;line-height:inherit}a{color:#337ab7;text-decoration:none}a:focus,a:hover{color:#23527c;text-decoration:underline}a:focus{outline:thin dotted;outline:5px auto -webkit-focus-ring-color;outline-offset:-2px}figure{margin:0}img{vertical-align:middle}.carousel-inner>.item>a>img,.carousel-inner>.item>img,.img-responsive,.thumbnail a>img,.thumbnail>img{display:block;max-width:100%;height:auto}.img-rounded{border-radius:6px}.img-thumbnail{display:inline-block;max-width:100%;height:auto;padding:4px;line-height:1.42857143;background-color:#fff;border:1px solid #ddd;border-radius:4px;-webkit-transition:all .2s ease-in-out;-o-transition:all .2s ease-in-out;transition:all .2s ease-in-out}.img-circle{border-radius:50%}hr{margin-top:20px;margin-bottom:20px;border:0;border-top:1px solid #eee}.sr-only{position:absolute;width:1px;height:1px;padding:0;margin:-1px;overflow:hidden;clip:rect(0,0,0,0);border:0}.sr-only-focusable:active,.sr-only-focusable:focus{position:static;width:auto;height:auto;margin:0;overflow:visible;clip:auto}[role=button]{cursor:pointer}.h1,.h2,.h3,.h4,.h5,.h6,h1,h2,h3,h4,h5,h6{font-family:inherit;font-weight:500;line-height:1.1;color:inherit}.h1 .small,.h1 small,.h2 .small,.h2 small,.h3 .small,.h3 small,.h4 .small,.h4 small,.h5 .small,.h5 small,.h6 .small,.h6 small,h1 .small,h1 small,h2 .small,h2 small,h3 .small,h3 small,h4 .small,h4 small,h5 .small,h5 small,h6 .small,h6 small{font-weight:400;line-height:1;color:#777}.h1,.h2,.h3,h1,h2,h3{margin-top:20px;margin-bottom:10px}.h1 .small,.h1 small,.h2 .small,.h2 small,.h3 .small,.h3 small,h1 .small,h1 small,h2 .small,h2 small,h3 .small,h3 small{font-size:65%}.h4,.h5,.h6,h4,h5,h6{margin-top:10px;margin-bottom:10px}.h4 .small,.h4 small,.h5 .small,.h5 small,.h6 .small,.h6 small,h4 .small,h4 small,h5 .small,h5 small,h6 .small,h6 small{font-size:75%}.h1,h1{font-size:36px}.h2,h2{font-size:30px}.h3,h3{font-size:24px}.h4,h4{font-size:18px}.h5,h5{font-size:14px}.h6,h6{font-size:12px}p{margin:0 0 10px}.lead{margin-bottom:20px;font-size:16px;font-weight:300;line-height:1.4}@media (min-width:768px){.lead{font-size:21px}}.small,small{font-size:85%}.mark,mark{padding:.2em;background-color:#fcf8e3}.text-left{text-align:left}.text-right{text-align:right}.text-center{text-align:center}.text-justify{text-align:justify}.text-nowrap{white-space:nowrap}.text-lowercase{text-transform:lowercase}.text-uppercase{text-transform:uppercase}.text-capitalize{text-transform:capitalize}.text-muted{color:#777}.text-primary{color:#337ab7}a.text-primary:focus,a.text-primary:hover{color:#286090}.text-success{color:#3c763d}a.text-success:focus,a.text-success:hover{color:#2b542c}.text-info{color:#31708f}a.text-info:focus,a.text-info:hover{color:#245269}.text-warning{color:#8a6d3b}a.text-warning:focus,a.text-warning:hover{color:#66512c}.text-danger{color:#a94442}a.text-danger:focus,a.text-danger:hover{color:#843534}.bg-primary{color:#fff;background-color:#337ab7}a.bg-primary:focus,a.bg-primary:hover{background-color:#286090}.bg-success{background-color:#dff0d8}a.bg-success:focus,a.bg-success:hover{background-color:#c1e2b3}.bg-info{background-color:#d9edf7}a.bg-info:focus,a.bg-info:hover{background-color:#afd9ee}.bg-warning{background-color:#fcf8e3}a.bg-warning:focus,a.bg-warning:hover{background-color:#f7ecb5}.bg-danger{background-color:#f2dede}a.bg-danger:focus,a.bg-danger:hover{background-color:#e4b9b9}.page-header{padding-bottom:9px;margin:40px 0 20px;border-bottom:1px solid #eee}ol,ul{margin-top:0;margin-bottom:10px}ol ol,ol ul,ul ol,ul ul{margin-bottom:0}.list-unstyled{padding-left:0;list-style:none}.list-inline{padding-left:0;margin-left:-5px;list-style:none}.list-inline>li{display:inline-block;padding-right:5px;padding-left:5px}dl{margin-top:0;margin-bottom:20px}dd,dt{line-height:1.42857143}dt{font-weight:700}dd{margin-left:0}@media (min-width:768px){.dl-horizontal dt{float:left;width:160px;overflow:hidden;clear:left;text-align:right;text-overflow:ellipsis;white-space:nowrap}.dl-horizontal dd{margin-left:180px}}abbr[data-original-title],abbr[title]{cursor:help;border-bottom:1px dotted #777}.initialism{font-size:90%;text-transform:uppercase}blockquote{padding:10px 20px;margin:0 0 20px;font-size:17.5px;border-left:5px solid #eee}blockquote ol:last-child,blockquote p:last-child,blockquote ul:last-child{margin-bottom:0}blockquote .small,blockquote footer,blockquote small{display:block;font-size:80%;line-height:1.42857143;color:#777}blockquote .small:before,blockquote footer:before,blockquote small:before{content:'\2014 \00A0'}.blockquote-reverse,blockquote.pull-right{padding-right:15px;padding-left:0;text-align:right;border-right:5px solid #eee;border-left:0}.blockquote-reverse .small:before,.blockquote-reverse footer:before,.blockquote-reverse small:before,blockquote.pull-right .small:before,blockquote.pull-right footer:before,blockquote.pull-right small:before{content:''}.blockquote-reverse .small:after,.blockquote-reverse footer:after,.blockquote-reverse small:after,blockquote.pull-right .small:after,blockquote.pull-right footer:after,blockquote.pull-right small:after{content:'\00A0 \2014'}address{margin-bottom:20px;font-style:normal;line-height:1.42857143}code,kbd,pre,samp{font-family:monospace}code{padding:2px 4px;font-size:90%;color:#c7254e;background-color:#f9f2f4;border-radius:4px}kbd{padding:2px 4px;font-size:90%;color:#fff;background-color:#333;border-radius:3px;-webkit-box-shadow:inset 0 -1px 0 rgba(0,0,0,.25);box-shadow:inset 0 -1px 0 rgba(0,0,0,.25)}kbd kbd{padding:0;font-size:100%;font-weight:700;-webkit-box-shadow:none;box-shadow:none}pre{display:block;padding:9.5px;margin:0 0 10px;font-size:13px;line-height:1.42857143;color:#333;word-break:break-all;word-wrap:break-word;background-color:#f5f5f5;border:1px solid #ccc;border-radius:4px}pre code{padding:0;font-size:inherit;color:inherit;white-space:pre-wrap;background-color:transparent;border-radius:0}.pre-scrollable{max-height:340px;overflow-y:scroll}.container{padding-right:15px;padding-left:15px;margin-right:auto;margin-left:auto}@media (min-width:768px){.container{width:750px}}@media (min-width:992px){.container{width:970px}}@media (min-width:1200px){.container{width:1170px}}.container-fluid{padding-right:15px;padding-left:15px;margin-right:auto;margin-left:auto}.row{margin-right:-15px;margin-left:-15px}.col-lg-1,.col-lg-10,.col-lg-11,.col-lg-12,.col-lg-2,.col-lg-3,.col-lg-4,.col-lg-5,.col-lg-6,.col-lg-7,.col-lg-8,.col-lg-9,.col-md-1,.col-md-10,.col-md-11,.col-md-12,.col-md-2,.col-md-3,.col-md-4,.col-md-5,.col-md-6,.col-md-7,.col-md-8,.col-md-9,.col-sm-1,.col-sm-10,.col-sm-11,.col-sm-12,.col-sm-2,.col-sm-3,.col-sm-4,.col-sm-5,.col-sm-6,.col-sm-7,.col-sm-8,.col-sm-9,.col-xs-1,.col-xs-10,.col-xs-11,.col-xs-12,.col-xs-2,.col-xs-3,.col-xs-4,.col-xs-5,.col-xs-6,.col-xs-7,.col-xs-8,.col-xs-9{position:relative;min-height:1px;padding-right:15px;padding-left:15px}.col-xs-1,.col-xs-10,.col-xs-11,.col-xs-12,.col-xs-2,.col-xs-3,.col-xs-4,.col-xs-5,.col-xs-6,.col-xs-7,.col-xs-8,.col-xs-9{float:left}.col-xs-12{width:100%}.col-xs-11{width:91.66666667%}.col-xs-10{width:83.33333333%}.col-xs-9{width:75%}.col-xs-8{width:66.66666667%}.col-xs-7{width:58.33333333%}.col-xs-6{width:50%}.col-xs-5{width:41.66666667%}.col-xs-4{width:33.33333333%}.col-xs-3{width:25%}.col-xs-2{width:16.66666667%}.col-xs-1{width:8.33333333%}.col-xs-pull-12{right:100%}.col-xs-pull-11{right:91.66666667%}.col-xs-pull-10{right:83.33333333%}.col-xs-pull-9{right:75%}.col-xs-pull-8{right:66.66666667%}.col-xs-pull-7{right:58.33333333%}.col-xs-pull-6{right:50%}.col-xs-pull-5{right:41.66666667%}.col-xs-pull-4{right:33.33333333%}.col-xs-pull-3{right:25%}.col-xs-pull-2{right:16.66666667%}.col-xs-pull-1{right:8.33333333%}.col-xs-pull-0{right:auto}.col-xs-push-12{left:100%}.col-xs-push-11{left:91.66666667%}.col-xs-push-10{left:83.33333333%}.col-xs-push-9{left:75%}.col-xs-push-8{left:66.66666667%}.col-xs-push-7{left:58.33333333%}.col-xs-push-6{left:50%}.col-xs-push-5{left:41.66666667%}.col-xs-push-4{left:33.33333333%}.col-xs-push-3{left:25%}.col-xs-push-2{left:16.66666667%}.col-xs-push-1{left:8.33333333%}.col-xs-push-0{left:auto}.col-xs-offset-12{margin-left:100%}.col-xs-offset-11{margin-left:91.66666667%}.col-xs-offset-10{margin-left:83.33333333%}.col-xs-offset-9{margin-left:75%}.col-xs-offset-8{margin-left:66.66666667%}.col-xs-offset-7{margin-left:58.33333333%}.col-xs-offset-6{margin-left:50%}.col-xs-offset-5{margin-left:41.66666667%}.col-xs-offset-4{margin-left:33.33333333%}.col-xs-offset-3{margin-left:25%}.col-xs-offset-2{margin-left:16.66666667%}.col-xs-offset-1{margin-left:8.33333333%}.col-xs-offset-0{margin-left:0}@media (min-width:768px){.col-sm-1,.col-sm-10,.col-sm-11,.col-sm-12,.col-sm-2,.col-sm-3,.col-sm-4,.col-sm-5,.col-sm-6,.col-sm-7,.col-sm-8,.col-sm-9{float:left}.col-sm-12{width:100%}.col-sm-11{width:91.66666667%}.col-sm-10{width:83.33333333%}.col-sm-9{width:75%}.col-sm-8{width:66.66666667%}.col-sm-7{width:58.33333333%}.col-sm-6{width:50%}.col-sm-5{width:41.66666667%}.col-sm-4{width:33.33333333%}.col-sm-3{width:25%}.col-sm-2{width:16.66666667%}.col-sm-1{width:8.33333333%}.col-sm-pull-12{right:100%}.col-sm-pull-11{right:91.66666667%}.col-sm-pull-10{right:83.33333333%}.col-sm-pull-9{right:75%}.col-sm-pull-8{right:66.66666667%}.col-sm-pull-7{right:58.33333333%}.col-sm-pull-6{right:50%}.col-sm-pull-5{right:41.66666667%}.col-sm-pull-4{right:33.33333333%}.col-sm-pull-3{right:25%}.col-sm-pull-2{right:16.66666667%}.col-sm-pull-1{right:8.33333333%}.col-sm-pull-0{right:auto}.col-sm-push-12{left:100%}.col-sm-push-11{left:91.66666667%}.col-sm-push-10{left:83.33333333%}.col-sm-push-9{left:75%}.col-sm-push-8{left:66.66666667%}.col-sm-push-7{left:58.33333333%}.col-sm-push-6{left:50%}.col-sm-push-5{left:41.66666667%}.col-sm-push-4{left:33.33333333%}.col-sm-push-3{left:25%}.col-sm-push-2{left:16.66666667%}.col-sm-push-1{left:8.33333333%}.col-sm-push-0{left:auto}.col-sm-offset-12{margin-left:100%}.col-sm-offset-11{margin-left:91.66666667%}.col-sm-offset-10{margin-left:83.33333333%}.col-sm-offset-9{margin-left:75%}.col-sm-offset-8{margin-left:66.66666667%}.col-sm-offset-7{margin-left:58.33333333%}.col-sm-offset-6{margin-left:50%}.col-sm-offset-5{margin-left:41.66666667%}.col-sm-offset-4{margin-left:33.33333333%}.col-sm-offset-3{margin-left:25%}.col-sm-offset-2{margin-left:16.66666667%}.col-sm-offset-1{margin-left:8.33333333%}.col-sm-offset-0{margin-left:0}}@media (min-width:992px){.col-md-1,.col-md-10,.col-md-11,.col-md-12,.col-md-2,.col-md-3,.col-md-4,.col-md-5,.col-md-6,.col-md-7,.col-md-8,.col-md-9{float:left}.col-md-12{width:100%}.col-md-11{width:91.66666667%}.col-md-10{width:83.33333333%}.col-md-9{width:75%}.col-md-8{width:66.66666667%}.col-md-7{width:58.33333333%}.col-md-6{width:50%}.col-md-5{width:41.66666667%}.col-md-4{width:33.33333333%}.col-md-3{width:25%}.col-md-2{width:16.66666667%}.col-md-1{width:8.33333333%}.col-md-pull-12{right:100%}.col-md-pull-11{right:91.66666667%}.col-md-pull-10{right:83.33333333%}.col-md-pull-9{right:75%}.col-md-pull-8{right:66.66666667%}.col-md-pull-7{right:58.33333333%}.col-md-pull-6{right:50%}.col-md-pull-5{right:41.66666667%}.col-md-pull-4{right:33.33333333%}.col-md-pull-3{right:25%}.col-md-pull-2{right:16.66666667%}.col-md-pull-1{right:8.33333333%}.col-md-pull-0{right:auto}.col-md-push-12{left:100%}.col-md-push-11{left:91.66666667%}.col-md-push-10{left:83.33333333%}.col-md-push-9{left:75%}.col-md-push-8{left:66.66666667%}.col-md-push-7{left:58.33333333%}.col-md-push-6{left:50%}.col-md-push-5{left:41.66666667%}.col-md-push-4{left:33.33333333%}.col-md-push-3{left:25%}.col-md-push-2{left:16.66666667%}.col-md-push-1{left:8.33333333%}.col-md-push-0{left:auto}.col-md-offset-12{margin-left:100%}.col-md-offset-11{margin-left:91.66666667%}.col-md-offset-10{margin-left:83.33333333%}.col-md-offset-9{margin-left:75%}.col-md-offset-8{margin-left:66.66666667%}.col-md-offset-7{margin-left:58.33333333%}.col-md-offset-6{margin-left:50%}.col-md-offset-5{margin-left:41.66666667%}.col-md-offset-4{margin-left:33.33333333%}.col-md-offset-3{margin-left:25%}.col-md-offset-2{margin-left:16.66666667%}.col-md-offset-1{margin-left:8.33333333%}.col-md-offset-0{margin-left:0}}@media (min-width:1200px){.col-lg-1,.col-lg-10,.col-lg-11,.col-lg-12,.col-lg-2,.col-lg-3,.col-lg-4,.col-lg-5,.col-lg-6,.col-lg-7,.col-lg-8,.col-lg-9{float:left}.col-lg-12{width:100%}.col-lg-11{width:91.66666667%}.col-lg-10{width:83.33333333%}.col-lg-9{width:75%}.col-lg-8{width:66.66666667%}.col-lg-7{width:58.33333333%}.col-lg-6{width:50%}.col-lg-5{width:41.66666667%}.col-lg-4{width:33.33333333%}.col-lg-3{width:25%}.col-lg-2{width:16.66666667%}.col-lg-1{width:8.33333333%}.col-lg-pull-12{right:100%}.col-lg-pull-11{right:91.66666667%}.col-lg-pull-10{right:83.33333333%}.col-lg-pull-9{right:75%}.col-lg-pull-8{right:66.66666667%}.col-lg-pull-7{right:58.33333333%}.col-lg-pull-6{right:50%}.col-lg-pull-5{right:41.66666667%}.col-lg-pull-4{right:33.33333333%}.col-lg-pull-3{right:25%}.col-lg-pull-2{right:16.66666667%}.col-lg-pull-1{right:8.33333333%}.col-lg-pull-0{right:auto}.col-lg-push-12{left:100%}.col-lg-push-11{left:91.66666667%}.col-lg-push-10{left:83.33333333%}.col-lg-push-9{left:75%}.col-lg-push-8{left:66.66666667%}.col-lg-push-7{left:58.33333333%}.col-lg-push-6{left:50%}.col-lg-push-5{left:41.66666667%}.col-lg-push-4{left:33.33333333%}.col-lg-push-3{left:25%}.col-lg-push-2{left:16.66666667%}.col-lg-push-1{left:8.33333333%}.col-lg-push-0{left:auto}.col-lg-offset-12{margin-left:100%}.col-lg-offset-11{margin-left:91.66666667%}.col-lg-offset-10{margin-left:83.33333333%}.col-lg-offset-9{margin-left:75%}.col-lg-offset-8{margin-left:66.66666667%}.col-lg-offset-7{margin-left:58.33333333%}.col-lg-offset-6{margin-left:50%}.col-lg-offset-5{margin-left:41.66666667%}.col-lg-offset-4{margin-left:33.33333333%}.col-lg-offset-3{margin-left:25%}.col-lg-offset-2{margin-left:16.66666667%}.col-lg-offset-1{margin-left:8.33333333%}.col-lg-offset-0{margin-left:0}}table{background-color:transparent}caption{padding-top:8px;padding-bottom:8px;color:#777;text-align:left}th{}.table{width:100%;max-width:100%;margin-bottom:20px}.table>tbody>tr>td,.table>tbody>tr>th,.table>tfoot>tr>td,.table>tfoot>tr>th,.table>thead>tr>td,.table>thead>tr>th{padding:8px;line-height:1.42857143;vertical-align:top;border-top:1px solid #ddd}.table>thead>tr>th{vertical-align:bottom;border-bottom:2px solid #ddd}.table>caption+thead>tr:first-child>td,.table>caption+thead>tr:first-child>th,.table>colgroup+thead>tr:first-child>td,.table>colgroup+thead>tr:first-child>th,.table>thead:first-child>tr:first-child>td,.table>thead:first-child>tr:first-child>th{border-top:0}.table>tbody+tbody{border-top:2px solid #ddd}.table .table{background-color:#fff}.table-condensed>tbody>tr>td,.table-condensed>tbody>tr>th,.table-condensed>tfoot>tr>td,.table-condensed>tfoot>tr>th,.table-condensed>thead>tr>td,.table-condensed>thead>tr>th{padding:5px}.table-bordered{border:1px solid #ddd}.table-bordered>tbody>tr>td,.table-bordered>tbody>tr>th,.table-bordered>tfoot>tr>td,.table-bordered>tfoot>tr>th,.table-bordered>thead>tr>td,.table-bordered>thead>tr>th{border:1px solid #ddd}.table-bordered>thead>tr>td,.table-bordered>thead>tr>th{border-bottom-width:2px}.table-striped>tbody>tr:nth-of-type(odd){background-color:#f9f9f9}.table-hover>tbody>tr:hover{background-color:#f5f5f5}table col[class*=col-]{position:static;display:table-column;float:none}table td[class*=col-],table th[class*=col-]{position:static;display:table-cell;float:none}.table>tbody>tr.active>td,.table>tbody>tr.active>th,.table>tbody>tr>td.active,.table>tbody>tr>th.active,.table>tfoot>tr.active>td,.table>tfoot>tr.active>th,.table>tfoot>tr>td.active,.table>tfoot>tr>th.active,.table>thead>tr.active>td,.table>thead>tr.active>th,.table>thead>tr>td.active,.table>thead>tr>th.active{background-color:#f5f5f5}.table-hover>tbody>tr.active:hover>td,.table-hover>tbody>tr.active:hover>th,.table-hover>tbody>tr:hover>.active,.table-hover>tbody>tr>td.active:hover,.table-hover>tbody>tr>th.active:hover{background-color:#e8e8e8}.table>tbody>tr.success>td,.table>tbody>tr.success>th,.table>tbody>tr>td.success,.table>tbody>tr>th.success,.table>tfoot>tr.success>td,.table>tfoot>tr.success>th,.table>tfoot>tr>td.success,.table>tfoot>tr>th.success,.table>thead>tr.success>td,.table>thead>tr.success>th,.table>thead>tr>td.success,.table>thead>tr>th.success{background-color:#dff0d8}.table-hover>tbody>tr.success:hover>td,.table-hover>tbody>tr.success:hover>th,.table-hover>tbody>tr:hover>.success,.table-hover>tbody>tr>td.success:hover,.table-hover>tbody>tr>th.success:hover{background-color:#d0e9c6}.table>tbody>tr.info>td,.table>tbody>tr.info>th,.table>tbody>tr>td.info,.table>tbody>tr>th.info,.table>tfoot>tr.info>td,.table>tfoot>tr.info>th,.table>tfoot>tr>td.info,.table>tfoot>tr>th.info,.table>thead>tr.info>td,.table>thead>tr.info>th,.table>thead>tr>td.info,.table>thead>tr>th.info{background-color:#d9edf7}.table-hover>tbody>tr.info:hover>td,.table-hover>tbody>tr.info:hover>th,.table-hover>tbody>tr:hover>.info,.table-hover>tbody>tr>td.info:hover,.table-hover>tbody>tr>th.info:hover{background-color:#c4e3f3}.table>tbody>tr.warning>td,.table>tbody>tr.warning>th,.table>tbody>tr>td.warning,.table>tbody>tr>th.warning,.table>tfoot>tr.warning>td,.table>tfoot>tr.warning>th,.table>tfoot>tr>td.warning,.table>tfoot>tr>th.warning,.table>thead>tr.warning>td,.table>thead>tr.warning>th,.table>thead>tr>td.warning,.table>thead>tr>th.warning{background-color:#fcf8e3}.table-hover>tbody>tr.warning:hover>td,.table-hover>tbody>tr.warning:hover>th,.table-hover>tbody>tr:hover>.warning,.table-hover>tbody>tr>td.warning:hover,.table-hover>tbody>tr>th.warning:hover{background-color:#faf2cc}.table>tbody>tr.danger>td,.table>tbody>tr.danger>th,.table>tbody>tr>td.danger,.table>tbody>tr>th.danger,.table>tfoot>tr.danger>td,.table>tfoot>tr.danger>th,.table>tfoot>tr>td.danger,.table>tfoot>tr>th.danger,.table>thead>tr.danger>td,.table>thead>tr.danger>th,.table>thead>tr>td.danger,.table>thead>tr>th.danger{background-color:#f2dede}.table-hover>tbody>tr.danger:hover>td,.table-hover>tbody>tr.danger:hover>th,.table-hover>tbody>tr:hover>.danger,.table-hover>tbody>tr>td.danger:hover,.table-hover>tbody>tr>th.danger:hover{background-color:#ebcccc}.table-responsive{min-height:.01%;overflow-x:auto}@media screen and (max-width:767px){.table-responsive{width:100%;margin-bottom:15px;overflow-y:hidden;-ms-overflow-style:-ms-autohiding-scrollbar;border:1px solid #ddd}.table-responsive>.table{margin-bottom:0}.table-responsive>.table>tbody>tr>td,.table-responsive>.table>tbody>tr>th,.table-responsive>.table>tfoot>tr>td,.table-responsive>.table>tfoot>tr>th,.table-responsive>.table>thead>tr>td,.table-responsive>.table>thead>tr>th{white-space:nowrap}.table-responsive>.table-bordered{border:0}.table-responsive>.table-bordered>tbody>tr>td:first-child,.table-responsive>.table-bordered>tbody>tr>th:first-child,.table-responsive>.table-bordered>tfoot>tr>td:first-child,.table-responsive>.table-bordered>tfoot>tr>th:first-child,.table-responsive>.table-bordered>thead>tr>td:first-child,.table-responsive>.table-bordered>thead>tr>th:first-child{border-left:0}.table-responsive>.table-bordered>tbody>tr>td:last-child,.table-responsive>.table-bordered>tbody>tr>th:last-child,.table-responsive>.table-bordered>tfoot>tr>td:last-child,.table-responsive>.table-bordered>tfoot>tr>th:last-child,.table-responsive>.table-bordered>thead>tr>td:last-child,.table-responsive>.table-bordered>thead>tr>th:last-child{border-right:0}.table-responsive>.table-bordered>tbody>tr:last-child>td,.table-responsive>.table-bordered>tbody>tr:last-child>th,.table-responsive>.table-bordered>tfoot>tr:last-child>td,.table-responsive>.table-bordered>tfoot>tr:last-child>th{border-bottom:0}}fieldset{min-width:0;padding:0;margin:0;border:0}legend{display:block;width:100%;padding:0;margin-bottom:20px;font-size:21px;line-height:inherit;color:#333;border:0;border-bottom:1px solid #e5e5e5}label{display:inline-block;max-width:100%;margin-bottom:5px;font-weight:700}input[type=search]{-webkit-box-sizing:border-box;-moz-box-sizing:border-box;box-sizing:border-box}input[type=checkbox],input[type=radio]{margin:4px 0 0;margin-top:1px\9;line-height:normal}input[type=file]{display:block}input[type=range]{display:block;width:100%}select[multiple],select[size]{height:auto}input[type=file]:focus,input[type=checkbox]:focus,input[type=radio]:focus{outline:thin dotted;outline:5px auto -webkit-focus-ring-color;outline-offset:-2px}output{display:block;padding-top:7px;font-size:14px;line-height:1.42857143;color:#555}.form-control{display:block;width:100%;height:34px;padding:6px 12px;font-size:14px;line-height:1.42857143;color:#555;background-color:#fff;background-image:none;border:1px solid #ccc;border-radius:4px;-webkit-box-shadow:inset 0 1px 1px rgba(0,0,0,.075);box-shadow:inset 0 1px 1px rgba(0,0,0,.075);-webkit-transition:border-color ease-in-out .15s,-webkit-box-shadow ease-in-out .15s;-o-transition:border-color ease-in-out .15s,box-shadow ease-in-out .15s;transition:border-color ease-in-out .15s,box-shadow ease-in-out .15s}.form-control:focus{border-color:#66afe9;outline:0;-webkit-box-shadow:inset 0 1px 1px rgba(0,0,0,.075),0 0 8px rgba(102,175,233,.6);box-shadow:inset 0 1px 1px rgba(0,0,0,.075),0 0 8px rgba(102,175,233,.6)}.form-control::-moz-placeholder{color:#999;opacity:1}.form-control:-ms-input-placeholder{color:#999}.form-control::-webkit-input-placeholder{color:#999}.form-control[disabled],.form-control[readonly],fieldset[disabled] .form-control{background-color:#eee;opacity:1}.form-control[disabled],fieldset[disabled] .form-control{cursor:not-allowed}textarea.form-control{height:auto}input[type=search]{-webkit-appearance:none}@media screen and (-webkit-min-device-pixel-ratio:0){input[type=date].form-control,input[type=time].form-control,input[type=datetime-local].form-control,input[type=month].form-control{line-height:34px}.input-group-sm input[type=date],.input-group-sm input[type=time],.input-group-sm input[type=datetime-local],.input-group-sm input[type=month],input[type=date].input-sm,input[type=time].input-sm,input[type=datetime-local].input-sm,input[type=month].input-sm{line-height:30px}.input-group-lg input[type=date],.input-group-lg input[type=time],.input-group-lg input[type=datetime-local],.input-group-lg input[type=month],input[type=date].input-lg,input[type=time].input-lg,input[type=datetime-local].input-lg,input[type=month].input-lg{line-height:46px}}.form-group{margin-bottom:15px}.checkbox,.radio{position:relative;display:block;margin-top:10px;margin-bottom:10px}.checkbox label,.radio label{min-height:20px;padding-left:20px;margin-bottom:0;font-weight:400;cursor:pointer}.checkbox input[type=checkbox],.checkbox-inline input[type=checkbox],.radio input[type=radio],.radio-inline input[type=radio]{position:absolute;margin-top:4px\9;margin-left:-20px}.checkbox+.checkbox,.radio+.radio{margin-top:-5px}.checkbox-inline,.radio-inline{position:relative;display:inline-block;padding-left:20px;margin-bottom:0;font-weight:400;vertical-align:middle;cursor:pointer}.checkbox-inline+.checkbox-inline,.radio-inline+.radio-inline{margin-top:0;margin-left:10px}fieldset[disabled] input[type=checkbox],fieldset[disabled] input[type=radio],input[type=checkbox].disabled,input[type=checkbox][disabled],input[type=radio].disabled,input[type=radio][disabled]{cursor:not-allowed}.checkbox-inline.disabled,.radio-inline.disabled,fieldset[disabled] .checkbox-inline,fieldset[disabled] .radio-inline{cursor:not-allowed}.checkbox.disabled label,.radio.disabled label,fieldset[disabled] .checkbox label,fieldset[disabled] .radio label{cursor:not-allowed}.form-control-static{min-height:34px;padding-top:7px;padding-bottom:7px;margin-bottom:0}.form-control-static.input-lg,.form-control-static.input-sm{padding-right:0;padding-left:0}.input-sm{height:30px;padding:5px 10px;font-size:12px;line-height:1.5;border-radius:3px}select.input-sm{height:30px;line-height:30px}select[multiple].input-sm,textarea.input-sm{height:auto}.form-group-sm .form-control{height:30px;padding:5px 10px;font-size:12px;line-height:1.5;border-radius:3px}.form-group-sm select.form-control{height:30px;line-height:30px}.form-group-sm select[multiple].form-control,.form-group-sm textarea.form-control{height:auto}.form-group-sm .form-control-static{height:30px;min-height:32px;padding:6px 10px;font-size:12px;line-height:1.5}.input-lg{height:46px;padding:10px 16px;font-size:18px;line-height:1.3333333;border-radius:6px}select.input-lg{height:46px;line-height:46px}select[multiple].input-lg,textarea.input-lg{height:auto}.form-group-lg .form-control{height:46px;padding:10px 16px;font-size:18px;line-height:1.3333333;border-radius:6px}.form-group-lg select.form-control{height:46px;line-height:46px}.form-group-lg select[multiple].form-control,.form-group-lg textarea.form-control{height:auto}.form-group-lg .form-control-static{height:46px;min-height:38px;padding:11px 16px;font-size:18px;line-height:1.3333333}.has-feedback{position:relative}.has-feedback .form-control{padding-right:42.5px}.form-control-feedback{position:absolute;top:0;right:0;z-index:2;display:block;width:34px;height:34px;line-height:34px;text-align:center;pointer-events:none}.form-group-lg .form-control+.form-control-feedback,.input-group-lg+.form-control-feedback,.input-lg+.form-control-feedback{width:46px;height:46px;line-height:46px}.form-group-sm .form-control+.form-control-feedback,.input-group-sm+.form-control-feedback,.input-sm+.form-control-feedback{width:30px;height:30px;line-height:30px}.has-success .checkbox,.has-success .checkbox-inline,.has-success .control-label,.has-success .help-block,.has-success .radio,.has-success .radio-inline,.has-success.checkbox label,.has-success.checkbox-inline label,.has-success.radio label,.has-success.radio-inline label{color:#3c763d}.has-success .form-control{border-color:#3c763d;-webkit-box-shadow:inset 0 1px 1px rgba(0,0,0,.075);box-shadow:inset 0 1px 1px rgba(0,0,0,.075)}.has-success .form-control:focus{border-color:#2b542c;-webkit-box-shadow:inset 0 1px 1px rgba(0,0,0,.075),0 0 6px #67b168;box-shadow:inset 0 1px 1px rgba(0,0,0,.075),0 0 6px #67b168}.has-success .input-group-addon{color:#3c763d;background-color:#dff0d8;border-color:#3c763d}.has-success .form-control-feedback{color:#3c763d}.has-warning .checkbox,.has-warning .checkbox-inline,.has-warning .control-label,.has-warning .help-block,.has-warning .radio,.has-warning .radio-inline,.has-warning.checkbox label,.has-warning.checkbox-inline label,.has-warning.radio label,.has-warning.radio-inline label{color:#8a6d3b}.has-warning .form-control{border-color:#8a6d3b;-webkit-box-shadow:inset 0 1px 1px rgba(0,0,0,.075);box-shadow:inset 0 1px 1px rgba(0,0,0,.075)}.has-warning .form-control:focus{border-color:#66512c;-webkit-box-shadow:inset 0 1px 1px rgba(0,0,0,.075),0 0 6px #c0a16b;box-shadow:inset 0 1px 1px rgba(0,0,0,.075),0 0 6px #c0a16b}.has-warning .input-group-addon{color:#8a6d3b;background-color:#fcf8e3;border-color:#8a6d3b}.has-warning .form-control-feedback{color:#8a6d3b}.has-error .checkbox,.has-error .checkbox-inline,.has-error .control-label,.has-error .help-block,.has-error .radio,.has-error .radio-inline,.has-error.checkbox label,.has-error.checkbox-inline label,.has-error.radio label,.has-error.radio-inline label{color:#a94442}.has-error .form-control{border-color:#a94442;-webkit-box-shadow:inset 0 1px 1px rgba(0,0,0,.075);box-shadow:inset 0 1px 1px rgba(0,0,0,.075)}.has-error .form-control:focus{border-color:#843534;-webkit-box-shadow:inset 0 1px 1px rgba(0,0,0,.075),0 0 6px #ce8483;box-shadow:inset 0 1px 1px rgba(0,0,0,.075),0 0 6px #ce8483}.has-error .input-group-addon{color:#a94442;background-color:#f2dede;border-color:#a94442}.has-error .form-control-feedback{color:#a94442}.has-feedback label~.form-control-feedback{top:25px}.has-feedback label.sr-only~.form-control-feedback{top:0}.help-block{display:block;margin-top:5px;margin-bottom:10px;color:#737373}@media (min-width:768px){.form-inline .form-group{display:inline-block;margin-bottom:0;vertical-align:middle}.form-inline .form-control{display:inline-block;width:auto;vertical-align:middle}.form-inline .form-control-static{display:inline-block}.form-inline .input-group{display:inline-table;vertical-align:middle}.form-inline .input-group .form-control,.form-inline .input-group .input-group-addon,.form-inline .input-group .input-group-btn{width:auto}.form-inline .input-group>.form-control{width:100%}.form-inline .control-label{margin-bottom:0;vertical-align:middle}.form-inline .checkbox,.form-inline .radio{display:inline-block;margin-top:0;margin-bottom:0;vertical-align:middle}.form-inline .checkbox label,.form-inline .radio label{padding-left:0}.form-inline .checkbox input[type=checkbox],.form-inline .radio input[type=radio]{position:relative;margin-left:0}.form-inline .has-feedback .form-control-feedback{top:0}}.form-horizontal .checkbox,.form-horizontal .checkbox-inline,.form-horizontal .radio,.form-horizontal .radio-inline{padding-top:7px;margin-top:0;margin-bottom:0}.form-horizontal .checkbox,.form-horizontal .radio{min-height:27px}.form-horizontal .form-group{margin-right:-15px;margin-left:-15px}@media (min-width:768px){.form-horizontal .control-label{padding-top:7px;margin-bottom:0;text-align:right}}.form-horizontal .has-feedback .form-control-feedback{right:15px}@media (min-width:768px){.form-horizontal .form-group-lg .control-label{padding-top:14.33px;font-size:18px}}@media (min-width:768px){.form-horizontal .form-group-sm .control-label{padding-top:6px;font-size:12px}}.btn{display:inline-block;padding:6px 12px;margin-bottom:0;font-size:14px;font-weight:400;line-height:1.42857143;text-align:center;white-space:nowrap;vertical-align:middle;-ms-touch-action:manipulation;touch-action:manipulation;cursor:pointer;-webkit-user-select:none;-moz-user-select:none;-ms-user-select:none;user-select:none;background-image:none;border:1px solid transparent;border-radius:4px}.btn.active.focus,.btn.active:focus,.btn.focus,.btn:active.focus,.btn:active:focus,.btn:focus{outline:thin dotted;outline:5px auto -webkit-focus-ring-color;outline-offset:-2px}.btn.focus,.btn:focus,.btn:hover{color:#333;text-decoration:none}.btn.active,.btn:active{background-image:none;outline:0;-webkit-box-shadow:inset 0 3px 5px rgba(0,0,0,.125);box-shadow:inset 0 3px 5px rgba(0,0,0,.125)}.btn.disabled,.btn[disabled],fieldset[disabled] .btn{cursor:not-allowed;filter:alpha(opacity=65);-webkit-box-shadow:none;box-shadow:none;opacity:.65}a.btn.disabled,fieldset[disabled] a.btn{pointer-events:none}.btn-default{color:#333;background-color:#fff;border-color:#ccc}.btn-default.focus,.btn-default:focus{color:#333;background-color:#e6e6e6;border-color:#8c8c8c}.btn-default:hover{color:#333;background-color:#e6e6e6;border-color:#adadad}.btn-default.active,.btn-default:active,.open>.dropdown-toggle.btn-default{color:#333;background-color:#e6e6e6;border-color:#adadad}.btn-default.active.focus,.btn-default.active:focus,.btn-default.active:hover,.btn-default:active.focus,.btn-default:active:focus,.btn-default:active:hover,.open>.dropdown-toggle.btn-default.focus,.open>.dropdown-toggle.btn-default:focus,.open>.dropdown-toggle.btn-default:hover{color:#333;background-color:#d4d4d4;border-color:#8c8c8c}.btn-default.active,.btn-default:active,.open>.dropdown-toggle.btn-default{background-image:none}.btn-default.disabled,.btn-default.disabled.active,.btn-default.disabled.focus,.btn-default.disabled:active,.btn-default.disabled:focus,.btn-default.disabled:hover,.btn-default[disabled],.btn-default[disabled].active,.btn-default[disabled].focus,.btn-default[disabled]:active,.btn-default[disabled]:focus,.btn-default[disabled]:hover,fieldset[disabled] .btn-default,fieldset[disabled] .btn-default.active,fieldset[disabled] .btn-default.focus,fieldset[disabled] .btn-default:active,fieldset[disabled] .btn-default:focus,fieldset[disabled] .btn-default:hover{background-color:#fff;border-color:#ccc}.btn-default .badge{color:#fff;background-color:#333}.btn-primary{color:#fff;background-color:#337ab7;border-color:#2e6da4}.btn-primary.focus,.btn-primary:focus{color:#fff;background-color:#286090;border-color:#122b40}.btn-primary:hover{color:#fff;background-color:#286090;border-color:#204d74}.btn-primary.active,.btn-primary:active,.open>.dropdown-toggle.btn-primary{color:#fff;background-color:#286090;border-color:#204d74}.btn-primary.active.focus,.btn-primary.active:focus,.btn-primary.active:hover,.btn-primary:active.focus,.btn-primary:active:focus,.btn-primary:active:hover,.open>.dropdown-toggle.btn-primary.focus,.open>.dropdown-toggle.btn-primary:focus,.open>.dropdown-toggle.btn-primary:hover{color:#fff;background-color:#204d74;border-color:#122b40}.btn-primary.active,.btn-primary:active,.open>.dropdown-toggle.btn-primary{background-image:none}.btn-primary.disabled,.btn-primary.disabled.active,.btn-primary.disabled.focus,.btn-primary.disabled:active,.btn-primary.disabled:focus,.btn-primary.disabled:hover,.btn-primary[disabled],.btn-primary[disabled].active,.btn-primary[disabled].focus,.btn-primary[disabled]:active,.btn-primary[disabled]:focus,.btn-primary[disabled]:hover,fieldset[disabled] .btn-primary,fieldset[disabled] .btn-primary.active,fieldset[disabled] .btn-primary.focus,fieldset[disabled] .btn-primary:active,fieldset[disabled] .btn-primary:focus,fieldset[disabled] .btn-primary:hover{background-color:#337ab7;border-color:#2e6da4}.btn-primary .badge{color:#337ab7;background-color:#fff}.btn-success{color:#fff;background-color:#5cb85c;border-color:#4cae4c}.btn-success.focus,.btn-success:focus{color:#fff;background-color:#449d44;border-color:#255625}.btn-success:hover{color:#fff;background-color:#449d44;border-color:#398439}.btn-success.active,.btn-success:active,.open>.dropdown-toggle.btn-success{color:#fff;background-color:#449d44;border-color:#398439}.btn-success.active.focus,.btn-success.active:focus,.btn-success.active:hover,.btn-success:active.focus,.btn-success:active:focus,.btn-success:active:hover,.open>.dropdown-toggle.btn-success.focus,.open>.dropdown-toggle.btn-success:focus,.open>.dropdown-toggle.btn-success:hover{color:#fff;background-color:#398439;border-color:#255625}.btn-success.active,.btn-success:active,.open>.dropdown-toggle.btn-success{background-image:none}.btn-success.disabled,.btn-success.disabled.active,.btn-success.disabled.focus,.btn-success.disabled:active,.btn-success.disabled:focus,.btn-success.disabled:hover,.btn-success[disabled],.btn-success[disabled].active,.btn-success[disabled].focus,.btn-success[disabled]:active,.btn-success[disabled]:focus,.btn-success[disabled]:hover,fieldset[disabled] .btn-success,fieldset[disabled] .btn-success.active,fieldset[disabled] .btn-success.focus,fieldset[disabled] .btn-success:active,fieldset[disabled] .btn-success:focus,fieldset[disabled] .btn-success:hover{background-color:#5cb85c;border-color:#4cae4c}.btn-success .badge{color:#5cb85c;background-color:#fff}.btn-info{color:#fff;background-color:#5bc0de;border-color:#46b8da}.btn-info.focus,.btn-info:focus{color:#fff;background-color:#31b0d5;border-color:#1b6d85}.btn-info:hover{color:#fff;background-color:#31b0d5;border-color:#269abc}.btn-info.active,.btn-info:active,.open>.dropdown-toggle.btn-info{color:#fff;background-color:#31b0d5;border-color:#269abc}.btn-info.active.focus,.btn-info.active:focus,.btn-info.active:hover,.btn-info:active.focus,.btn-info:active:focus,.btn-info:active:hover,.open>.dropdown-toggle.btn-info.focus,.open>.dropdown-toggle.btn-info:focus,.open>.dropdown-toggle.btn-info:hover{color:#fff;background-color:#269abc;border-color:#1b6d85}.btn-info.active,.btn-info:active,.open>.dropdown-toggle.btn-info{background-image:none}.btn-info.disabled,.btn-info.disabled.active,.btn-info.disabled.focus,.btn-info.disabled:active,.btn-info.disabled:focus,.btn-info.disabled:hover,.btn-info[disabled],.btn-info[disabled].active,.btn-info[disabled].focus,.btn-info[disabled]:active,.btn-info[disabled]:focus,.btn-info[disabled]:hover,fieldset[disabled] .btn-info,fieldset[disabled] .btn-info.active,fieldset[disabled] .btn-info.focus,fieldset[disabled] .btn-info:active,fieldset[disabled] .btn-info:focus,fieldset[disabled] .btn-info:hover{background-color:#5bc0de;border-color:#46b8da}.btn-info .badge{color:#5bc0de;background-color:#fff}.btn-warning{color:#fff;background-color:#f0ad4e;border-color:#eea236}.btn-warning.focus,.btn-warning:focus{color:#fff;background-color:#ec971f;border-color:#985f0d}.btn-warning:hover{color:#fff;background-color:#ec971f;border-color:#d58512}.btn-warning.active,.btn-warning:active,.open>.dropdown-toggle.btn-warning{color:#fff;background-color:#ec971f;border-color:#d58512}.btn-warning.active.focus,.btn-warning.active:focus,.btn-warning.active:hover,.btn-warning:active.focus,.btn-warning:active:focus,.btn-warning:active:hover,.open>.dropdown-toggle.btn-warning.focus,.open>.dropdown-toggle.btn-warning:focus,.open>.dropdown-toggle.btn-warning:hover{color:#fff;background-color:#d58512;border-color:#985f0d}.btn-warning.active,.btn-warning:active,.open>.dropdown-toggle.btn-warning{background-image:none}.btn-warning.disabled,.btn-warning.disabled.active,.btn-warning.disabled.focus,.btn-warning.disabled:active,.btn-warning.disabled:focus,.btn-warning.disabled:hover,.btn-warning[disabled],.btn-warning[disabled].active,.btn-warning[disabled].focus,.btn-warning[disabled]:active,.btn-warning[disabled]:focus,.btn-warning[disabled]:hover,fieldset[disabled] .btn-warning,fieldset[disabled] .btn-warning.active,fieldset[disabled] .btn-warning.focus,fieldset[disabled] .btn-warning:active,fieldset[disabled] .btn-warning:focus,fieldset[disabled] .btn-warning:hover{background-color:#f0ad4e;border-color:#eea236}.btn-warning .badge{color:#f0ad4e;background-color:#fff}.btn-danger{color:#fff;background-color:#d9534f;border-color:#d43f3a}.btn-danger.focus,.btn-danger:focus{color:#fff;background-color:#c9302c;border-color:#761c19}.btn-danger:hover{color:#fff;background-color:#c9302c;border-color:#ac2925}.btn-danger.active,.btn-danger:active,.open>.dropdown-toggle.btn-danger{color:#fff;background-color:#c9302c;border-color:#ac2925}.btn-danger.active.focus,.btn-danger.active:focus,.btn-danger.active:hover,.btn-danger:active.focus,.btn-danger:active:focus,.btn-danger:active:hover,.open>.dropdown-toggle.btn-danger.focus,.open>.dropdown-toggle.btn-danger:focus,.open>.dropdown-toggle.btn-danger:hover{color:#fff;background-color:#ac2925;border-color:#761c19}.btn-danger.active,.btn-danger:active,.open>.dropdown-toggle.btn-danger{background-image:none}.btn-danger.disabled,.btn-danger.disabled.active,.btn-danger.disabled.focus,.btn-danger.disabled:active,.btn-danger.disabled:focus,.btn-danger.disabled:hover,.btn-danger[disabled],.btn-danger[disabled].active,.btn-danger[disabled].focus,.btn-danger[disabled]:active,.btn-danger[disabled]:focus,.btn-danger[disabled]:hover,fieldset[disabled] .btn-danger,fieldset[disabled] .btn-danger.active,fieldset[disabled] .btn-danger.focus,fieldset[disabled] .btn-danger:active,fieldset[disabled] .btn-danger:focus,fieldset[disabled] .btn-danger:hover{background-color:#d9534f;border-color:#d43f3a}.btn-danger .badge{color:#d9534f;background-color:#fff}.btn-link{font-weight:400;color:#337ab7;border-radius:0}.btn-link,.btn-link.active,.btn-link:active,.btn-link[disabled],fieldset[disabled] .btn-link{background-color:transparent;-webkit-box-shadow:none;box-shadow:none}.btn-link,.btn-link:active,.btn-link:focus,.btn-link:hover{border-color:transparent}.btn-link:focus,.btn-link:hover{color:#23527c;text-decoration:underline;background-color:transparent}.btn-link[disabled]:focus,.btn-link[disabled]:hover,fieldset[disabled] .btn-link:focus,fieldset[disabled] .btn-link:hover{color:#777;text-decoration:none}.btn-group-lg>.btn,.btn-lg{padding:10px 16px;font-size:18px;line-height:1.3333333;border-radius:6px}.btn-group-sm>.btn,.btn-sm{padding:5px 10px;font-size:12px;line-height:1.5;border-radius:3px}.btn-group-xs>.btn,.btn-xs{padding:1px 5px;font-size:12px;line-height:1.5;border-radius:3px}.btn-block{display:block;width:100%}.btn-block+.btn-block{margin-top:5px}input[type=button].btn-block,input[type=reset].btn-block,input[type=submit].btn-block{width:100%}.fade{opacity:0;-webkit-transition:opacity .15s linear;-o-transition:opacity .15s linear;transition:opacity .15s linear}.fade.in{opacity:1}.collapse{display:none}.collapse.in{display:block}tr.collapse.in{display:table-row}tbody.collapse.in{display:table-row-group}.collapsing{position:relative;height:0;overflow:hidden;-webkit-transition-timing-function:ease;-o-transition-timing-function:ease;transition-timing-function:ease;-webkit-transition-duration:.35s;-o-transition-duration:.35s;transition-duration:.35s;-webkit-transition-property:height,visibility;-o-transition-property:height,visibility;transition-property:height,visibility}.caret{display:inline-block;width:0;height:0;margin-left:2px;vertical-align:middle;border-top:4px dashed;border-top:4px solid\9;border-right:4px solid transparent;border-left:4px solid transparent}.dropdown,.dropup{position:relative}.dropdown-toggle:focus{outline:0}.dropdown-menu{position:absolute;top:100%;left:0;z-index:1000;display:none;float:left;min-width:160px;padding:5px 0;margin:2px 0 0;font-size:14px;text-align:left;list-style:none;background-color:#fff;-webkit-background-clip:padding-box;background-clip:padding-box;border:1px solid #ccc;border:1px solid rgba(0,0,0,.15);border-radius:4px;-webkit-box-shadow:0 6px 12px rgba(0,0,0,.175);box-shadow:0 6px 12px rgba(0,0,0,.175)}.dropdown-menu.pull-right{right:0;left:auto}.dropdown-menu .divider{height:1px;margin:9px 0;overflow:hidden;background-color:#e5e5e5}.dropdown-menu>li>a{display:block;padding:3px 20px;clear:both;font-weight:400;line-height:1.42857143;color:#333;white-space:nowrap}.dropdown-menu>li>a:focus,.dropdown-menu>li>a:hover{color:#262626;text-decoration:none;background-color:#f5f5f5}.dropdown-menu>.active>a,.dropdown-menu>.active>a:focus,.dropdown-menu>.active>a:hover{color:#fff;text-decoration:none;background-color:#337ab7;outline:0}.dropdown-menu>.disabled>a,.dropdown-menu>.disabled>a:focus,.dropdown-menu>.disabled>a:hover{color:#777}.dropdown-menu>.disabled>a:focus,.dropdown-menu>.disabled>a:hover{text-decoration:none;cursor:not-allowed;background-color:transparent;background-image:none;filter:progid:DXImageTransform.Microsoft.gradient(enabled=false)}.open>.dropdown-menu{display:block}.open>a{outline:0}.dropdown-menu-right{right:0;left:auto}.dropdown-menu-left{right:auto;left:0}.dropdown-header{display:block;padding:3px 20px;font-size:12px;line-height:1.42857143;color:#777;white-space:nowrap}.dropdown-backdrop{position:fixed;top:0;right:0;bottom:0;left:0;z-index:990}.pull-right>.dropdown-menu{right:0;left:auto}.dropup .caret,.navbar-fixed-bottom .dropdown .caret{content:"";border-top:0;border-bottom:4px dashed;border-bottom:4px solid\9}.dropup .dropdown-menu,.navbar-fixed-bottom .dropdown .dropdown-menu{top:auto;bottom:100%;margin-bottom:2px}@media (min-width:768px){.navbar-right .dropdown-menu{right:0;left:auto}.navbar-right .dropdown-menu-left{right:auto;left:0}}.btn-group,.btn-group-vertical{position:relative;display:inline-block;vertical-align:middle}.btn-group-vertical>.btn,.btn-group>.btn{position:relative;float:left}.btn-group-vertical>.btn.active,.btn-group-vertical>.btn:active,.btn-group-vertical>.btn:focus,.btn-group-vertical>.btn:hover,.btn-group>.btn.active,.btn-group>.btn:active,.btn-group>.btn:focus,.btn-group>.btn:hover{z-index:2}.btn-group .btn+.btn,.btn-group .btn+.btn-group,.btn-group .btn-group+.btn,.btn-group .btn-group+.btn-group{margin-left:-1px}.btn-toolbar{margin-left:-5px}.btn-toolbar .btn,.btn-toolbar .btn-group,.btn-toolbar .input-group{float:left}.btn-toolbar>.btn,.btn-toolbar>.btn-group,.btn-toolbar>.input-group{margin-left:5px}.btn-group>.btn:not(:first-child):not(:last-child):not(.dropdown-toggle){border-radius:0}.btn-group>.btn:first-child{margin-left:0}.btn-group>.btn:first-child:not(:last-child):not(.dropdown-toggle){border-top-right-radius:0;border-bottom-right-radius:0}.btn-group>.btn:last-child:not(:first-child),.btn-group>.dropdown-toggle:not(:first-child){border-top-left-radius:0;border-bottom-left-radius:0}.btn-group>.btn-group{float:left}.btn-group>.btn-group:not(:first-child):not(:last-child)>.btn{border-radius:0}.btn-group>.btn-group:first-child:not(:last-child)>.btn:last-child,.btn-group>.btn-group:first-child:not(:last-child)>.dropdown-toggle{border-top-right-radius:0;border-bottom-right-radius:0}.btn-group>.btn-group:last-child:not(:first-child)>.btn:first-child{border-top-left-radius:0;border-bottom-left-radius:0}.btn-group .dropdown-toggle:active,.btn-group.open .dropdown-toggle{outline:0}.btn-group>.btn+.dropdown-toggle{padding-right:8px;padding-left:8px}.btn-group>.btn-lg+.dropdown-toggle{padding-right:12px;padding-left:12px}.btn-group.open .dropdown-toggle{-webkit-box-shadow:inset 0 3px 5px rgba(0,0,0,.125);box-shadow:inset 0 3px 5px rgba(0,0,0,.125)}.btn-group.open .dropdown-toggle.btn-link{-webkit-box-shadow:none;box-shadow:none}.btn .caret{margin-left:0}.btn-lg .caret{border-width:5px 5px 0;border-bottom-width:0}.dropup .btn-lg .caret{border-width:0 5px 5px}.btn-group-vertical>.btn,.btn-group-vertical>.btn-group,.btn-group-vertical>.btn-group>.btn{display:block;float:none;width:100%;max-width:100%}.btn-group-vertical>.btn-group>.btn{float:none}.btn-group-vertical>.btn+.btn,.btn-group-vertical>.btn+.btn-group,.btn-group-vertical>.btn-group+.btn,.btn-group-vertical>.btn-group+.btn-group{margin-top:-1px;margin-left:0}.btn-group-vertical>.btn:not(:first-child):not(:last-child){border-radius:0}.btn-group-vertical>.btn:first-child:not(:last-child){border-top-right-radius:4px;border-bottom-right-radius:0;border-bottom-left-radius:0}.btn-group-vertical>.btn:last-child:not(:first-child){border-top-left-radius:0;border-top-right-radius:0;border-bottom-left-radius:4px}.btn-group-vertical>.btn-group:not(:first-child):not(:last-child)>.btn{border-radius:0}.btn-group-vertical>.btn-group:first-child:not(:last-child)>.btn:last-child,.btn-group-vertical>.btn-group:first-child:not(:last-child)>.dropdown-toggle{border-bottom-right-radius:0;border-bottom-left-radius:0}.btn-group-vertical>.btn-group:last-child:not(:first-child)>.btn:first-child{border-top-left-radius:0;border-top-right-radius:0}.btn-group-justified{display:table;width:100%;table-layout:fixed;border-collapse:separate}.btn-group-justified>.btn,.btn-group-justified>.btn-group{display:table-cell;float:none;width:1%}.btn-group-justified>.btn-group .btn{width:100%}.btn-group-justified>.btn-group .dropdown-menu{left:auto}[data-toggle=buttons]>.btn input[type=checkbox],[data-toggle=buttons]>.btn input[type=radio],[data-toggle=buttons]>.btn-group>.btn input[type=checkbox],[data-toggle=buttons]>.btn-group>.btn input[type=radio]{position:absolute;clip:rect(0,0,0,0);pointer-events:none}.input-group{position:relative;display:table;border-collapse:separate}.input-group[class*=col-]{float:none;padding-right:0;padding-left:0}.input-group .form-control{position:relative;z-index:2;float:left;width:100%;margin-bottom:0}.input-group-lg>.form-control,.input-group-lg>.input-group-addon,.input-group-lg>.input-group-btn>.btn{height:46px;padding:10px 16px;font-size:18px;line-height:1.3333333;border-radius:6px}select.input-group-lg>.form-control,select.input-group-lg>.input-group-addon,select.input-group-lg>.input-group-btn>.btn{height:46px;line-height:46px}select[multiple].input-group-lg>.form-control,select[multiple].input-group-lg>.input-group-addon,select[multiple].input-group-lg>.input-group-btn>.btn,textarea.input-group-lg>.form-control,textarea.input-group-lg>.input-group-addon,textarea.input-group-lg>.input-group-btn>.btn{height:auto}.input-group-sm>.form-control,.input-group-sm>.input-group-addon,.input-group-sm>.input-group-btn>.btn{height:30px;padding:5px 10px;font-size:12px;line-height:1.5;border-radius:3px}select.input-group-sm>.form-control,select.input-group-sm>.input-group-addon,select.input-group-sm>.input-group-btn>.btn{height:30px;line-height:30px}select[multiple].input-group-sm>.form-control,select[multiple].input-group-sm>.input-group-addon,select[multiple].input-group-sm>.input-group-btn>.btn,textarea.input-group-sm>.form-control,textarea.input-group-sm>.input-group-addon,textarea.input-group-sm>.input-group-btn>.btn{height:auto}.input-group .form-control,.input-group-addon,.input-group-btn{display:table-cell}.input-group .form-control:not(:first-child):not(:last-child),.input-group-addon:not(:first-child):not(:last-child),.input-group-btn:not(:first-child):not(:last-child){border-radius:0}.input-group-addon,.input-group-btn{width:1%;white-space:nowrap;vertical-align:middle}.input-group-addon{padding:6px 12px;font-size:14px;font-weight:400;line-height:1;color:#555;text-align:center;background-color:#eee;border:1px solid #ccc;border-radius:4px}.input-group-addon.input-sm{padding:5px 10px;font-size:12px;border-radius:3px}.input-group-addon.input-lg{padding:10px 16px;font-size:18px;border-radius:6px}.input-group-addon input[type=checkbox],.input-group-addon input[type=radio]{margin-top:0}.input-group .form-control:first-child,.input-group-addon:first-child,.input-group-btn:first-child>.btn,.input-group-btn:first-child>.btn-group>.btn,.input-group-btn:first-child>.dropdown-toggle,.input-group-btn:last-child>.btn-group:not(:last-child)>.btn,.input-group-btn:last-child>.btn:not(:last-child):not(.dropdown-toggle){border-top-right-radius:0;border-bottom-right-radius:0}.input-group-addon:first-child{border-right:0}.input-group .form-control:last-child,.input-group-addon:last-child,.input-group-btn:first-child>.btn-group:not(:first-child)>.btn,.input-group-btn:first-child>.btn:not(:first-child),.input-group-btn:last-child>.btn,.input-group-btn:last-child>.btn-group>.btn,.input-group-btn:last-child>.dropdown-toggle{border-top-left-radius:0;border-bottom-left-radius:0}.input-group-addon:last-child{border-left:0}.input-group-btn{position:relative;font-size:0;white-space:nowrap}.input-group-btn>.btn{position:relative}.input-group-btn>.btn+.btn{margin-left:-1px}.input-group-btn>.btn:active,.input-group-btn>.btn:focus,.input-group-btn>.btn:hover{z-index:2}.input-group-btn:first-child>.btn,.input-group-btn:first-child>.btn-group{margin-right:-1px}.input-group-btn:last-child>.btn,.input-group-btn:last-child>.btn-group{z-index:2;margin-left:-1px}.nav{padding-left:0;margin-bottom:0;list-style:none}.nav>li{position:relative;display:block}.nav>li>a{position:relative;display:block;padding:10px 15px}.nav>li>a:focus,.nav>li>a:hover{text-decoration:none;background-color:#eee}.nav>li.disabled>a{color:#777}.nav>li.disabled>a:focus,.nav>li.disabled>a:hover{color:#777;text-decoration:none;cursor:not-allowed;background-color:transparent}.nav .open>a,.nav .open>a:focus,.nav .open>a:hover{background-color:#eee;border-color:#337ab7}.nav .nav-divider{height:1px;margin:9px 0;overflow:hidden;background-color:#e5e5e5}.nav>li>a>img{max-width:none}.nav-tabs{border-bottom:1px solid #ddd}.nav-tabs>li{float:left;margin-bottom:-1px}.nav-tabs>li>a{margin-right:2px;line-height:1.42857143;border:1px solid transparent;border-radius:4px 4px 0 0}.nav-tabs>li>a:hover{border-color:#eee #eee #ddd}.nav-tabs>li.active>a,.nav-tabs>li.active>a:focus,.nav-tabs>li.active>a:hover{color:#555;cursor:default;background-color:#fff;border:1px solid #ddd;border-bottom-color:transparent}.nav-tabs.nav-justified{width:100%;border-bottom:0}.nav-tabs.nav-justified>li{float:none}.nav-tabs.nav-justified>li>a{margin-bottom:5px;text-align:center}.nav-tabs.nav-justified>.dropdown .dropdown-menu{top:auto;left:auto}@media (min-width:768px){.nav-tabs.nav-justified>li{display:table-cell;width:1%}.nav-tabs.nav-justified>li>a{margin-bottom:0}}.nav-tabs.nav-justified>li>a{margin-right:0;border-radius:4px}.nav-tabs.nav-justified>.active>a,.nav-tabs.nav-justified>.active>a:focus,.nav-tabs.nav-justified>.active>a:hover{border:1px solid #ddd}@media (min-width:768px){.nav-tabs.nav-justified>li>a{border-bottom:1px solid #ddd;border-radius:4px 4px 0 0}.nav-tabs.nav-justified>.active>a,.nav-tabs.nav-justified>.active>a:focus,.nav-tabs.nav-justified>.active>a:hover{border-bottom-color:#fff}}.nav-pills>li{float:left}.nav-pills>li>a{border-radius:4px}.nav-pills>li+li{margin-left:2px}.nav-pills>li.active>a,.nav-pills>li.active>a:focus,.nav-pills>li.active>a:hover{color:#fff;background-color:#337ab7}.nav-stacked>li{float:none}.nav-stacked>li+li{margin-top:2px;margin-left:0}.nav-justified{width:100%}.nav-justified>li{float:none}.nav-justified>li>a{margin-bottom:5px;text-align:center}.nav-justified>.dropdown .dropdown-menu{top:auto;left:auto}@media (min-width:768px){.nav-justified>li{display:table-cell;width:1%}.nav-justified>li>a{margin-bottom:0}}.nav-tabs-justified{border-bottom:0}.nav-tabs-justified>li>a{margin-right:0;border-radius:4px}.nav-tabs-justified>.active>a,.nav-tabs-justified>.active>a:focus,.nav-tabs-justified>.active>a:hover{border:1px solid #ddd}@media (min-width:768px){.nav-tabs-justified>li>a{border-bottom:1px solid #ddd;border-radius:4px 4px 0 0}.nav-tabs-justified>.active>a,.nav-tabs-justified>.active>a:focus,.nav-tabs-justified>.active>a:hover{border-bottom-color:#fff}}.tab-content>.tab-pane{display:none}.tab-content>.active{display:block}.nav-tabs .dropdown-menu{margin-top:-1px;border-top-left-radius:0;border-top-right-radius:0}.navbar{position:relative;min-height:50px;margin-bottom:20px;border:1px solid transparent}@media (min-width:768px){.navbar{border-radius:4px}}@media (min-width:768px){.navbar-header{float:left}}.navbar-collapse{padding-right:15px;padding-left:15px;overflow-x:visible;-webkit-overflow-scrolling:touch;border-top:1px solid transparent;-webkit-box-shadow:inset 0 1px 0 rgba(255,255,255,.1);box-shadow:inset 0 1px 0 rgba(255,255,255,.1)}.navbar-collapse.in{overflow-y:auto}@media (min-width:768px){.navbar-collapse{width:auto;border-top:0;-webkit-box-shadow:none;box-shadow:none}.navbar-collapse.collapse{display:block!important;height:auto!important;padding-bottom:0;overflow:visible!important}.navbar-collapse.in{overflow-y:visible}.navbar-fixed-bottom .navbar-collapse,.navbar-fixed-top .navbar-collapse,.navbar-static-top .navbar-collapse{padding-right:0;padding-left:0}}.navbar-fixed-bottom .navbar-collapse,.navbar-fixed-top .navbar-collapse{max-height:340px}@media (max-device-width:480px) and (orientation:landscape){.navbar-fixed-bottom .navbar-collapse,.navbar-fixed-top .navbar-collapse{max-height:200px}}.container-fluid>.navbar-collapse,.container-fluid>.navbar-header,.container>.navbar-collapse,.container>.navbar-header{margin-right:-15px;margin-left:-15px}@media (min-width:768px){.container-fluid>.navbar-collapse,.container-fluid>.navbar-header,.container>.navbar-collapse,.container>.navbar-header{margin-right:0;margin-left:0}}.navbar-static-top{z-index:1000;border-width:0 0 1px}@media (min-width:768px){.navbar-static-top{border-radius:0}}.navbar-fixed-bottom,.navbar-fixed-top{position:fixed;right:0;left:0;z-index:1030}@media (min-width:768px){.navbar-fixed-bottom,.navbar-fixed-top{border-radius:0}}.navbar-fixed-top{top:0;border-width:0 0 1px}.navbar-fixed-bottom{bottom:0;margin-bottom:0;border-width:1px 0 0}.navbar-brand{float:left;height:50px;padding:15px 15px;font-size:18px;line-height:20px}.navbar-brand:focus,.navbar-brand:hover{text-decoration:none}.navbar-brand>img{display:block}@media (min-width:768px){.navbar>.container .navbar-brand,.navbar>.container-fluid .navbar-brand{margin-left:-15px}}.navbar-toggle{position:relative;float:right;padding:9px 10px;margin-top:8px;margin-right:15px;margin-bottom:8px;background-color:transparent;background-image:none;border:1px solid transparent;border-radius:4px}.navbar-toggle:focus{outline:0}.navbar-toggle .icon-bar{display:block;width:22px;height:2px;border-radius:1px}.navbar-toggle .icon-bar+.icon-bar{margin-top:4px}@media (min-width:768px){.navbar-toggle{display:none}}.navbar-nav{margin:7.5px -15px}.navbar-nav>li>a{padding-top:10px;padding-bottom:10px;line-height:20px}@media (max-width:767px){.navbar-nav .open .dropdown-menu{position:static;float:none;width:auto;margin-top:0;background-color:transparent;border:0;-webkit-box-shadow:none;box-shadow:none}.navbar-nav .open .dropdown-menu .dropdown-header,.navbar-nav .open .dropdown-menu>li>a{padding:5px 15px 5px 25px}.navbar-nav .open .dropdown-menu>li>a{line-height:20px}.navbar-nav .open .dropdown-menu>li>a:focus,.navbar-nav .open .dropdown-menu>li>a:hover{background-image:none}}@media (min-width:768px){.navbar-nav{float:left;margin:0}.navbar-nav>li{float:left}.navbar-nav>li>a{padding-top:15px;padding-bottom:15px}}.navbar-form{padding:10px 15px;margin-top:8px;margin-right:-15px;margin-bottom:8px;margin-left:-15px;border-top:1px solid transparent;border-bottom:1px solid transparent;-webkit-box-shadow:inset 0 1px 0 rgba(255,255,255,.1),0 1px 0 rgba(255,255,255,.1);box-shadow:inset 0 1px 0 rgba(255,255,255,.1),0 1px 0 rgba(255,255,255,.1)}@media (min-width:768px){.navbar-form .form-group{display:inline-block;margin-bottom:0;vertical-align:middle}.navbar-form .form-control{display:inline-block;width:auto;vertical-align:middle}.navbar-form .form-control-static{display:inline-block}.navbar-form .input-group{display:inline-table;vertical-align:middle}.navbar-form .input-group .form-control,.navbar-form .input-group .input-group-addon,.navbar-form .input-group .input-group-btn{width:auto}.navbar-form .input-group>.form-control{width:100%}.navbar-form .control-label{margin-bottom:0;vertical-align:middle}.navbar-form .checkbox,.navbar-form .radio{display:inline-block;margin-top:0;margin-bottom:0;vertical-align:middle}.navbar-form .checkbox label,.navbar-form .radio label{padding-left:0}.navbar-form .checkbox input[type=checkbox],.navbar-form .radio input[type=radio]{position:relative;margin-left:0}.navbar-form .has-feedback .form-control-feedback{top:0}}@media (max-width:767px){.navbar-form .form-group{margin-bottom:5px}.navbar-form .form-group:last-child{margin-bottom:0}}@media (min-width:768px){.navbar-form{width:auto;padding-top:0;padding-bottom:0;margin-right:0;margin-left:0;border:0;-webkit-box-shadow:none;box-shadow:none}}.navbar-nav>li>.dropdown-menu{margin-top:0;border-top-left-radius:0;border-top-right-radius:0}.navbar-fixed-bottom .navbar-nav>li>.dropdown-menu{margin-bottom:0;border-top-left-radius:4px;border-top-right-radius:4px;border-bottom-right-radius:0;border-bottom-left-radius:0}.navbar-btn{margin-top:8px;margin-bottom:8px}.navbar-btn.btn-sm{margin-top:10px;margin-bottom:10px}.navbar-btn.btn-xs{margin-top:14px;margin-bottom:14px}.navbar-text{margin-top:15px;margin-bottom:15px}@media (min-width:768px){.navbar-text{float:left;margin-right:15px;margin-left:15px}}@media (min-width:768px){.navbar-left{float:left!important}.navbar-right{float:right!important;margin-right:-15px}.navbar-right~.navbar-right{margin-right:0}}.navbar-default{background-color:#f8f8f8;border-color:#e7e7e7}.navbar-default .navbar-brand{color:#777}.navbar-default .navbar-brand:focus,.navbar-default .navbar-brand:hover{color:#5e5e5e;background-color:transparent}.navbar-default .navbar-text{color:#777}.navbar-default .navbar-nav>li>a{color:#777}.navbar-default .navbar-nav>li>a:focus,.navbar-default .navbar-nav>li>a:hover{color:#333;background-color:transparent}.navbar-default .navbar-nav>.active>a,.navbar-default .navbar-nav>.active>a:focus,.navbar-default .navbar-nav>.active>a:hover{color:#555;background-color:#e7e7e7}.navbar-default .navbar-nav>.disabled>a,.navbar-default .navbar-nav>.disabled>a:focus,.navbar-default .navbar-nav>.disabled>a:hover{color:#ccc;background-color:transparent}.navbar-default .navbar-toggle{border-color:#ddd}.navbar-default .navbar-toggle:focus,.navbar-default .navbar-toggle:hover{background-color:#ddd}.navbar-default .navbar-toggle .icon-bar{background-color:#888}.navbar-default .navbar-collapse,.navbar-default .navbar-form{border-color:#e7e7e7}.navbar-default .navbar-nav>.open>a,.navbar-default .navbar-nav>.open>a:focus,.navbar-default .navbar-nav>.open>a:hover{color:#555;background-color:#e7e7e7}@media (max-width:767px){.navbar-default .navbar-nav .open .dropdown-menu>li>a{color:#777}.navbar-default .navbar-nav .open .dropdown-menu>li>a:focus,.navbar-default .navbar-nav .open .dropdown-menu>li>a:hover{color:#333;background-color:transparent}.navbar-default .navbar-nav .open .dropdown-menu>.active>a,.navbar-default .navbar-nav .open .dropdown-menu>.active>a:focus,.navbar-default .navbar-nav .open .dropdown-menu>.active>a:hover{color:#555;background-color:#e7e7e7}.navbar-default .navbar-nav .open .dropdown-menu>.disabled>a,.navbar-default .navbar-nav .open .dropdown-menu>.disabled>a:focus,.navbar-default .navbar-nav .open .dropdown-menu>.disabled>a:hover{color:#ccc;background-color:transparent}}.navbar-default .navbar-link{color:#777}.navbar-default .navbar-link:hover{color:#333}.navbar-default .btn-link{color:#777}.navbar-default .btn-link:focus,.navbar-default .btn-link:hover{color:#333}.navbar-default .btn-link[disabled]:focus,.navbar-default .btn-link[disabled]:hover,fieldset[disabled] .navbar-default .btn-link:focus,fieldset[disabled] .navbar-default .btn-link:hover{color:#ccc}.navbar-inverse{background-color:#222;border-color:#080808}.navbar-inverse .navbar-brand{color:#9d9d9d}.navbar-inverse .navbar-brand:focus,.navbar-inverse .navbar-brand:hover{color:#fff;background-color:transparent}.navbar-inverse .navbar-text{color:#9d9d9d}.navbar-inverse .navbar-nav>li>a{color:#9d9d9d}.navbar-inverse .navbar-nav>li>a:focus,.navbar-inverse .navbar-nav>li>a:hover{color:#fff;background-color:transparent}.navbar-inverse .navbar-nav>.active>a,.navbar-inverse .navbar-nav>.active>a:focus,.navbar-inverse .navbar-nav>.active>a:hover{color:#fff;background-color:#080808}.navbar-inverse .navbar-nav>.disabled>a,.navbar-inverse .navbar-nav>.disabled>a:focus,.navbar-inverse .navbar-nav>.disabled>a:hover{color:#444;background-color:transparent}.navbar-inverse .navbar-toggle{border-color:#333}.navbar-inverse .navbar-toggle:focus,.navbar-inverse .navbar-toggle:hover{background-color:#333}.navbar-inverse .navbar-toggle .icon-bar{background-color:#fff}.navbar-inverse .navbar-collapse,.navbar-inverse .navbar-form{border-color:#101010}.navbar-inverse .navbar-nav>.open>a,.navbar-inverse .navbar-nav>.open>a:focus,.navbar-inverse .navbar-nav>.open>a:hover{color:#fff;background-color:#080808}@media (max-width:767px){.navbar-inverse .navbar-nav .open .dropdown-menu>.dropdown-header{border-color:#080808}.navbar-inverse .navbar-nav .open .dropdown-menu .divider{background-color:#080808}.navbar-inverse .navbar-nav .open .dropdown-menu>li>a{color:#9d9d9d}.navbar-inverse .navbar-nav .open .dropdown-menu>li>a:focus,.navbar-inverse .navbar-nav .open .dropdown-menu>li>a:hover{color:#fff;background-color:transparent}.navbar-inverse .navbar-nav .open .dropdown-menu>.active>a,.navbar-inverse .navbar-nav .open .dropdown-menu>.active>a:focus,.navbar-inverse .navbar-nav .open .dropdown-menu>.active>a:hover{color:#fff;background-color:#080808}.navbar-inverse .navbar-nav .open .dropdown-menu>.disabled>a,.navbar-inverse .navbar-nav .open .dropdown-menu>.disabled>a:focus,.navbar-inverse .navbar-nav .open .dropdown-menu>.disabled>a:hover{color:#444;background-color:transparent}}.navbar-inverse .navbar-link{color:#9d9d9d}.navbar-inverse .navbar-link:hover{color:#fff}.navbar-inverse .btn-link{color:#9d9d9d}.navbar-inverse .btn-link:focus,.navbar-inverse .btn-link:hover{color:#fff}.navbar-inverse .btn-link[disabled]:focus,.navbar-inverse .btn-link[disabled]:hover,fieldset[disabled] .navbar-inverse .btn-link:focus,fieldset[disabled] .navbar-inverse .btn-link:hover{color:#444}.breadcrumb{padding:8px 15px;margin-bottom:20px;list-style:none;background-color:#f5f5f5;border-radius:4px}.breadcrumb>li{display:inline-block}.breadcrumb>li+li:before{padding:0 5px;color:#ccc;content:"/\00a0"}.breadcrumb>.active{color:#777}.pagination{display:inline-block;padding-left:0;margin:20px 0;border-radius:4px}.pagination>li{display:inline}.pagination>li>a,.pagination>li>span{position:relative;float:left;padding:6px 12px;margin-left:-1px;line-height:1.42857143;color:#337ab7;text-decoration:none;background-color:#fff;border:1px solid #ddd}.pagination>li:first-child>a,.pagination>li:first-child>span{margin-left:0;border-top-left-radius:4px;border-bottom-left-radius:4px}.pagination>li:last-child>a,.pagination>li:last-child>span{border-top-right-radius:4px;border-bottom-right-radius:4px}.pagination>li>a:focus,.pagination>li>a:hover,.pagination>li>span:focus,.pagination>li>span:hover{z-index:3;color:#23527c;background-color:#eee;border-color:#ddd}.pagination>.active>a,.pagination>.active>a:focus,.pagination>.active>a:hover,.pagination>.active>span,.pagination>.active>span:focus,.pagination>.active>span:hover{z-index:2;color:#fff;cursor:default;background-color:#337ab7;border-color:#337ab7}.pagination>.disabled>a,.pagination>.disabled>a:focus,.pagination>.disabled>a:hover,.pagination>.disabled>span,.pagination>.disabled>span:focus,.pagination>.disabled>span:hover{color:#777;cursor:not-allowed;background-color:#fff;border-color:#ddd}.pagination-lg>li>a,.pagination-lg>li>span{padding:10px 16px;font-size:18px;line-height:1.3333333}.pagination-lg>li:first-child>a,.pagination-lg>li:first-child>span{border-top-left-radius:6px;border-bottom-left-radius:6px}.pagination-lg>li:last-child>a,.pagination-lg>li:last-child>span{border-top-right-radius:6px;border-bottom-right-radius:6px}.pagination-sm>li>a,.pagination-sm>li>span{padding:5px 10px;font-size:12px;line-height:1.5}.pagination-sm>li:first-child>a,.pagination-sm>li:first-child>span{border-top-left-radius:3px;border-bottom-left-radius:3px}.pagination-sm>li:last-child>a,.pagination-sm>li:last-child>span{border-top-right-radius:3px;border-bottom-right-radius:3px}.pager{padding-left:0;margin:20px 0;text-align:center;list-style:none}.pager li{display:inline}.pager li>a,.pager li>span{display:inline-block;padding:5px 14px;background-color:#fff;border:1px solid #ddd;border-radius:15px}.pager li>a:focus,.pager li>a:hover{text-decoration:none;background-color:#eee}.pager .next>a,.pager .next>span{float:right}.pager .previous>a,.pager .previous>span{float:left}.pager .disabled>a,.pager .disabled>a:focus,.pager .disabled>a:hover,.pager .disabled>span{color:#777;cursor:not-allowed;background-color:#fff}.label{display:inline;padding:.2em .6em .3em;font-size:75%;font-weight:700;line-height:1;color:#fff;text-align:center;white-space:nowrap;vertical-align:baseline;border-radius:.25em}a.label:focus,a.label:hover{color:#fff;text-decoration:none;cursor:pointer}.label:empty{display:none}.btn .label{position:relative;top:-1px}.label-default{background-color:#777}.label-default[href]:focus,.label-default[href]:hover{background-color:#5e5e5e}.label-primary{background-color:#337ab7}.label-primary[href]:focus,.label-primary[href]:hover{background-color:#286090}.label-success{background-color:#5cb85c}.label-success[href]:focus,.label-success[href]:hover{background-color:#449d44}.label-info{background-color:#5bc0de}.label-info[href]:focus,.label-info[href]:hover{background-color:#31b0d5}.label-warning{background-color:#f0ad4e}.label-warning[href]:focus,.label-warning[href]:hover{background-color:#ec971f}.label-danger{background-color:#d9534f}.label-danger[href]:focus,.label-danger[href]:hover{background-color:#c9302c}.badge{display:inline-block;min-width:10px;padding:3px 7px;font-size:12px;font-weight:700;line-height:1;color:#fff;text-align:center;white-space:nowrap;vertical-align:middle;background-color:#777;border-radius:10px}.badge:empty{display:none}.btn .badge{position:relative;top:-1px}.btn-group-xs>.btn .badge,.btn-xs .badge{top:0;padding:1px 5px}a.badge:focus,a.badge:hover{color:#fff;text-decoration:none;cursor:pointer}.list-group-item.active>.badge,.nav-pills>.active>a>.badge{color:#337ab7;background-color:#fff}.list-group-item>.badge{float:right}.list-group-item>.badge+.badge{margin-right:5px}.nav-pills>li>a>.badge{margin-left:3px}.jumbotron{padding-top:30px;padding-bottom:30px;margin-bottom:30px;color:inherit;background-color:#eee}.jumbotron .h1,.jumbotron h1{color:inherit}.jumbotron p{margin-bottom:15px;font-size:21px;font-weight:200}.jumbotron>hr{border-top-color:#d5d5d5}.container .jumbotron,.container-fluid .jumbotron{border-radius:6px}.jumbotron .container{max-width:100%}@media screen and (min-width:768px){.jumbotron{padding-top:48px;padding-bottom:48px}.container .jumbotron,.container-fluid .jumbotron{padding-right:60px;padding-left:60px}.jumbotron .h1,.jumbotron h1{font-size:63px}}.thumbnail{display:block;padding:4px;margin-bottom:20px;line-height:1.42857143;background-color:#fff;border:1px solid #ddd;border-radius:4px;-webkit-transition:border .2s ease-in-out;-o-transition:border .2s ease-in-out;transition:border .2s ease-in-out}.thumbnail a>img,.thumbnail>img{margin-right:auto;margin-left:auto}a.thumbnail.active,a.thumbnail:focus,a.thumbnail:hover{border-color:#337ab7}.thumbnail .caption{padding:9px;color:#333}.alert{padding:15px;margin-bottom:20px;border:1px solid transparent;border-radius:4px}.alert h4{margin-top:0;color:inherit}.alert .alert-link{font-weight:700}.alert>p,.alert>ul{margin-bottom:0}.alert>p+p{margin-top:5px}.alert-dismissable,.alert-dismissible{padding-right:35px}.alert-dismissable .close,.alert-dismissible .close{position:relative;top:-2px;right:-21px;color:inherit}.alert-success{color:#3c763d;background-color:#dff0d8;border-color:#d6e9c6}.alert-success hr{border-top-color:#c9e2b3}.alert-success .alert-link{color:#2b542c}.alert-info{color:#31708f;background-color:#d9edf7;border-color:#bce8f1}.alert-info hr{border-top-color:#a6e1ec}.alert-info .alert-link{color:#245269}.alert-warning{color:#8a6d3b;background-color:#fcf8e3;border-color:#faebcc}.alert-warning hr{border-top-color:#f7e1b5}.alert-warning .alert-link{color:#66512c}.alert-danger{color:#a94442;background-color:#f2dede;border-color:#ebccd1}.alert-danger hr{border-top-color:#e4b9c0}.alert-danger .alert-link{color:#843534}@-webkit-keyframes progress-bar-stripes{from{background-position:40px 0}to{background-position:0 0}}@-o-keyframes progress-bar-stripes{from{background-position:40px 0}to{background-position:0 0}}@keyframes progress-bar-stripes{from{background-position:40px 0}to{background-position:0 0}}.progress{height:20px;margin-bottom:20px;overflow:hidden;background-color:#f5f5f5;border-radius:4px;-webkit-box-shadow:inset 0 1px 2px rgba(0,0,0,.1);box-shadow:inset 0 1px 2px rgba(0,0,0,.1)}.progress-bar{float:left;width:0;height:100%;font-size:12px;line-height:20px;color:#fff;text-align:center;background-color:#337ab7;-webkit-box-shadow:inset 0 -1px 0 rgba(0,0,0,.15);box-shadow:inset 0 -1px 0 rgba(0,0,0,.15);-webkit-transition:width .6s ease;-o-transition:width .6s ease;transition:width .6s ease}.progress-bar-striped,.progress-striped .progress-bar{background-image:-webkit-linear-gradient(45deg,rgba(255,255,255,.15) 25%,transparent 25%,transparent 50%,rgba(255,255,255,.15) 50%,rgba(255,255,255,.15) 75%,transparent 75%,transparent);background-image:-o-linear-gradient(45deg,rgba(255,255,255,.15) 25%,transparent 25%,transparent 50%,rgba(255,255,255,.15) 50%,rgba(255,255,255,.15) 75%,transparent 75%,transparent);background-image:linear-gradient(45deg,rgba(255,255,255,.15) 25%,transparent 25%,transparent 50%,rgba(255,255,255,.15) 50%,rgba(255,255,255,.15) 75%,transparent 75%,transparent);-webkit-background-size:40px 40px;background-size:40px 40px}.progress-bar.active,.progress.active .progress-bar{-webkit-animation:progress-bar-stripes 2s linear infinite;-o-animation:progress-bar-stripes 2s linear infinite;animation:progress-bar-stripes 2s linear infinite}.progress-bar-success{background-color:#5cb85c}.progress-striped .progress-bar-success{background-image:-webkit-linear-gradient(45deg,rgba(255,255,255,.15) 25%,transparent 25%,transparent 50%,rgba(255,255,255,.15) 50%,rgba(255,255,255,.15) 75%,transparent 75%,transparent);background-image:-o-linear-gradient(45deg,rgba(255,255,255,.15) 25%,transparent 25%,transparent 50%,rgba(255,255,255,.15) 50%,rgba(255,255,255,.15) 75%,transparent 75%,transparent);background-image:linear-gradient(45deg,rgba(255,255,255,.15) 25%,transparent 25%,transparent 50%,rgba(255,255,255,.15) 50%,rgba(255,255,255,.15) 75%,transparent 75%,transparent)}.progress-bar-info{background-color:#5bc0de}.progress-striped .progress-bar-info{background-image:-webkit-linear-gradient(45deg,rgba(255,255,255,.15) 25%,transparent 25%,transparent 50%,rgba(255,255,255,.15) 50%,rgba(255,255,255,.15) 75%,transparent 75%,transparent);background-image:-o-linear-gradient(45deg,rgba(255,255,255,.15) 25%,transparent 25%,transparent 50%,rgba(255,255,255,.15) 50%,rgba(255,255,255,.15) 75%,transparent 75%,transparent);background-image:linear-gradient(45deg,rgba(255,255,255,.15) 25%,transparent 25%,transparent 50%,rgba(255,255,255,.15) 50%,rgba(255,255,255,.15) 75%,transparent 75%,transparent)}.progress-bar-warning{background-color:#f0ad4e}.progress-striped .progress-bar-warning{background-image:-webkit-linear-gradient(45deg,rgba(255,255,255,.15) 25%,transparent 25%,transparent 50%,rgba(255,255,255,.15) 50%,rgba(255,255,255,.15) 75%,transparent 75%,transparent);background-image:-o-linear-gradient(45deg,rgba(255,255,255,.15) 25%,transparent 25%,transparent 50%,rgba(255,255,255,.15) 50%,rgba(255,255,255,.15) 75%,transparent 75%,transparent);background-image:linear-gradient(45deg,rgba(255,255,255,.15) 25%,transparent 25%,transparent 50%,rgba(255,255,255,.15) 50%,rgba(255,255,255,.15) 75%,transparent 75%,transparent)}.progress-bar-danger{background-color:#d9534f}.progress-striped .progress-bar-danger{background-image:-webkit-linear-gradient(45deg,rgba(255,255,255,.15) 25%,transparent 25%,transparent 50%,rgba(255,255,255,.15) 50%,rgba(255,255,255,.15) 75%,transparent 75%,transparent);background-image:-o-linear-gradient(45deg,rgba(255,255,255,.15) 25%,transparent 25%,transparent 50%,rgba(255,255,255,.15) 50%,rgba(255,255,255,.15) 75%,transparent 75%,transparent);background-image:linear-gradient(45deg,rgba(255,255,255,.15) 25%,transparent 25%,transparent 50%,rgba(255,255,255,.15) 50%,rgba(255,255,255,.15) 75%,transparent 75%,transparent)}.media{margin-top:15px}.media:first-child{margin-top:0}.media,.media-body{overflow:hidden;zoom:1}.media-body{width:10000px}.media-object{display:block}.media-object.img-thumbnail{max-width:none}.media-right,.media>.pull-right{padding-left:10px}.media-left,.media>.pull-left{padding-right:10px}.media-body,.media-left,.media-right{display:table-cell;vertical-align:top}.media-middle{vertical-align:middle}.media-bottom{vertical-align:bottom}.media-heading{margin-top:0;margin-bottom:5px}.media-list{padding-left:0;list-style:none}.list-group{padding-left:0;margin-bottom:20px}.list-group-item{position:relative;display:block;padding:10px 15px;margin-bottom:-1px;background-color:#fff;border:1px solid #ddd}.list-group-item:first-child{border-top-left-radius:4px;border-top-right-radius:4px}.list-group-item:last-child{margin-bottom:0;border-bottom-right-radius:4px;border-bottom-left-radius:4px}a.list-group-item,button.list-group-item{color:#555}a.list-group-item .list-group-item-heading,button.list-group-item .list-group-item-heading{color:#333}a.list-group-item:focus,a.list-group-item:hover,button.list-group-item:focus,button.list-group-item:hover{color:#555;text-decoration:none;background-color:#f5f5f5}button.list-group-item{width:100%;text-align:left}.list-group-item.disabled,.list-group-item.disabled:focus,.list-group-item.disabled:hover{color:#777;cursor:not-allowed;background-color:#eee}.list-group-item.disabled .list-group-item-heading,.list-group-item.disabled:focus .list-group-item-heading,.list-group-item.disabled:hover .list-group-item-heading{color:inherit}.list-group-item.disabled .list-group-item-text,.list-group-item.disabled:focus .list-group-item-text,.list-group-item.disabled:hover .list-group-item-text{color:#777}.list-group-item.active,.list-group-item.active:focus,.list-group-item.active:hover{z-index:2;color:#fff;background-color:#337ab7;border-color:#337ab7}.list-group-item.active .list-group-item-heading,.list-group-item.active .list-group-item-heading>.small,.list-group-item.active .list-group-item-heading>small,.list-group-item.active:focus .list-group-item-heading,.list-group-item.active:focus .list-group-item-heading>.small,.list-group-item.active:focus .list-group-item-heading>small,.list-group-item.active:hover .list-group-item-heading,.list-group-item.active:hover .list-group-item-heading>.small,.list-group-item.active:hover .list-group-item-heading>small{color:inherit}.list-group-item.active .list-group-item-text,.list-group-item.active:focus .list-group-item-text,.list-group-item.active:hover .list-group-item-text{color:#c7ddef}.list-group-item-success{color:#3c763d;background-color:#dff0d8}a.list-group-item-success,button.list-group-item-success{color:#3c763d}a.list-group-item-success .list-group-item-heading,button.list-group-item-success .list-group-item-heading{color:inherit}a.list-group-item-success:focus,a.list-group-item-success:hover,button.list-group-item-success:focus,button.list-group-item-success:hover{color:#3c763d;background-color:#d0e9c6}a.list-group-item-success.active,a.list-group-item-success.active:focus,a.list-group-item-success.active:hover,button.list-group-item-success.active,button.list-group-item-success.active:focus,button.list-group-item-success.active:hover{color:#fff;background-color:#3c763d;border-color:#3c763d}.list-group-item-info{color:#31708f;background-color:#d9edf7}a.list-group-item-info,button.list-group-item-info{color:#31708f}a.list-group-item-info .list-group-item-heading,button.list-group-item-info .list-group-item-heading{color:inherit}a.list-group-item-info:focus,a.list-group-item-info:hover,button.list-group-item-info:focus,button.list-group-item-info:hover{color:#31708f;background-color:#c4e3f3}a.list-group-item-info.active,a.list-group-item-info.active:focus,a.list-group-item-info.active:hover,button.list-group-item-info.active,button.list-group-item-info.active:focus,button.list-group-item-info.active:hover{color:#fff;background-color:#31708f;border-color:#31708f}.list-group-item-warning{color:#8a6d3b;background-color:#fcf8e3}a.list-group-item-warning,button.list-group-item-warning{color:#8a6d3b}a.list-group-item-warning .list-group-item-heading,button.list-group-item-warning .list-group-item-heading{color:inherit}a.list-group-item-warning:focus,a.list-group-item-warning:hover,button.list-group-item-warning:focus,button.list-group-item-warning:hover{color:#8a6d3b;background-color:#faf2cc}a.list-group-item-warning.active,a.list-group-item-warning.active:focus,a.list-group-item-warning.active:hover,button.list-group-item-warning.active,button.list-group-item-warning.active:focus,button.list-group-item-warning.active:hover{color:#fff;background-color:#8a6d3b;border-color:#8a6d3b}.list-group-item-danger{color:#a94442;background-color:#f2dede}a.list-group-item-danger,button.list-group-item-danger{color:#a94442}a.list-group-item-danger .list-group-item-heading,button.list-group-item-danger .list-group-item-heading{color:inherit}a.list-group-item-danger:focus,a.list-group-item-danger:hover,button.list-group-item-danger:focus,button.list-group-item-danger:hover{color:#a94442;background-color:#ebcccc}a.list-group-item-danger.active,a.list-group-item-danger.active:focus,a.list-group-item-danger.active:hover,button.list-group-item-danger.active,button.list-group-item-danger.active:focus,button.list-group-item-danger.active:hover{color:#fff;background-color:#a94442;border-color:#a94442}.list-group-item-heading{margin-top:0;margin-bottom:5px}.list-group-item-text{margin-bottom:0;line-height:1.3}.panel{margin-bottom:20px;background-color:#fff;border:1px solid transparent;border-radius:4px;-webkit-box-shadow:0 1px 1px rgba(0,0,0,.05);box-shadow:0 1px 1px rgba(0,0,0,.05)}.panel-body{padding:15px}.panel-heading{padding:10px 15px;border-bottom:1px solid transparent;border-top-left-radius:3px;border-top-right-radius:3px}.panel-heading>.dropdown .dropdown-toggle{color:inherit}.panel-title{margin-top:0;margin-bottom:0;font-size:16px;color:inherit}.panel-title>.small,.panel-title>.small>a,.panel-title>a,.panel-title>small,.panel-title>small>a{color:inherit}.panel-footer{padding:10px 15px;background-color:#f5f5f5;border-top:1px solid #ddd;border-bottom-right-radius:3px;border-bottom-left-radius:3px}.panel>.list-group,.panel>.panel-collapse>.list-group{margin-bottom:0}.panel>.list-group .list-group-item,.panel>.panel-collapse>.list-group .list-group-item{border-width:1px 0;border-radius:0}.panel>.list-group:first-child .list-group-item:first-child,.panel>.panel-collapse>.list-group:first-child .list-group-item:first-child{border-top:0;border-top-left-radius:3px;border-top-right-radius:3px}.panel>.list-group:last-child .list-group-item:last-child,.panel>.panel-collapse>.list-group:last-child .list-group-item:last-child{border-bottom:0;border-bottom-right-radius:3px;border-bottom-left-radius:3px}.panel>.panel-heading+.panel-collapse>.list-group .list-group-item:first-child{border-top-left-radius:0;border-top-right-radius:0}.panel-heading+.list-group .list-group-item:first-child{border-top-width:0}.list-group+.panel-footer{border-top-width:0}.panel>.panel-collapse>.table,.panel>.table,.panel>.table-responsive>.table{margin-bottom:0}.panel>.panel-collapse>.table caption,.panel>.table caption,.panel>.table-responsive>.table caption{padding-right:15px;padding-left:15px}.panel>.table-responsive:first-child>.table:first-child,.panel>.table:first-child{border-top-left-radius:3px;border-top-right-radius:3px}.panel>.table-responsive:first-child>.table:first-child>tbody:first-child>tr:first-child,.panel>.table-responsive:first-child>.table:first-child>thead:first-child>tr:first-child,.panel>.table:first-child>tbody:first-child>tr:first-child,.panel>.table:first-child>thead:first-child>tr:first-child{border-top-left-radius:3px;border-top-right-radius:3px}.panel>.table-responsive:first-child>.table:first-child>tbody:first-child>tr:first-child td:first-child,.panel>.table-responsive:first-child>.table:first-child>tbody:first-child>tr:first-child th:first-child,.panel>.table-responsive:first-child>.table:first-child>thead:first-child>tr:first-child td:first-child,.panel>.table-responsive:first-child>.table:first-child>thead:first-child>tr:first-child th:first-child,.panel>.table:first-child>tbody:first-child>tr:first-child td:first-child,.panel>.table:first-child>tbody:first-child>tr:first-child th:first-child,.panel>.table:first-child>thead:first-child>tr:first-child td:first-child,.panel>.table:first-child>thead:first-child>tr:first-child th:first-child{border-top-left-radius:3px}.panel>.table-responsive:first-child>.table:first-child>tbody:first-child>tr:first-child td:last-child,.panel>.table-responsive:first-child>.table:first-child>tbody:first-child>tr:first-child th:last-child,.panel>.table-responsive:first-child>.table:first-child>thead:first-child>tr:first-child td:last-child,.panel>.table-responsive:first-child>.table:first-child>thead:first-child>tr:first-child th:last-child,.panel>.table:first-child>tbody:first-child>tr:first-child td:last-child,.panel>.table:first-child>tbody:first-child>tr:first-child th:last-child,.panel>.table:first-child>thead:first-child>tr:first-child td:last-child,.panel>.table:first-child>thead:first-child>tr:first-child th:last-child{border-top-right-radius:3px}.panel>.table-responsive:last-child>.table:last-child,.panel>.table:last-child{border-bottom-right-radius:3px;border-bottom-left-radius:3px}.panel>.table-responsive:last-child>.table:last-child>tbody:last-child>tr:last-child,.panel>.table-responsive:last-child>.table:last-child>tfoot:last-child>tr:last-child,.panel>.table:last-child>tbody:last-child>tr:last-child,.panel>.table:last-child>tfoot:last-child>tr:last-child{border-bottom-right-radius:3px;border-bottom-left-radius:3px}.panel>.table-responsive:last-child>.table:last-child>tbody:last-child>tr:last-child td:first-child,.panel>.table-responsive:last-child>.table:last-child>tbody:last-child>tr:last-child th:first-child,.panel>.table-responsive:last-child>.table:last-child>tfoot:last-child>tr:last-child td:first-child,.panel>.table-responsive:last-child>.table:last-child>tfoot:last-child>tr:last-child th:first-child,.panel>.table:last-child>tbody:last-child>tr:last-child td:first-child,.panel>.table:last-child>tbody:last-child>tr:last-child th:first-child,.panel>.table:last-child>tfoot:last-child>tr:last-child td:first-child,.panel>.table:last-child>tfoot:last-child>tr:last-child th:first-child{border-bottom-left-radius:3px}.panel>.table-responsive:last-child>.table:last-child>tbody:last-child>tr:last-child td:last-child,.panel>.table-responsive:last-child>.table:last-child>tbody:last-child>tr:last-child th:last-child,.panel>.table-responsive:last-child>.table:last-child>tfoot:last-child>tr:last-child td:last-child,.panel>.table-responsive:last-child>.table:last-child>tfoot:last-child>tr:last-child th:last-child,.panel>.table:last-child>tbody:last-child>tr:last-child td:last-child,.panel>.table:last-child>tbody:last-child>tr:last-child th:last-child,.panel>.table:last-child>tfoot:last-child>tr:last-child td:last-child,.panel>.table:last-child>tfoot:last-child>tr:last-child th:last-child{border-bottom-right-radius:3px}.panel>.panel-body+.table,.panel>.panel-body+.table-responsive,.panel>.table+.panel-body,.panel>.table-responsive+.panel-body{border-top:1px solid #ddd}.panel>.table>tbody:first-child>tr:first-child td,.panel>.table>tbody:first-child>tr:first-child th{border-top:0}.panel>.table-bordered,.panel>.table-responsive>.table-bordered{border:0}.panel>.table-bordered>tbody>tr>td:first-child,.panel>.table-bordered>tbody>tr>th:first-child,.panel>.table-bordered>tfoot>tr>td:first-child,.panel>.table-bordered>tfoot>tr>th:first-child,.panel>.table-bordered>thead>tr>td:first-child,.panel>.table-bordered>thead>tr>th:first-child,.panel>.table-responsive>.table-bordered>tbody>tr>td:first-child,.panel>.table-responsive>.table-bordered>tbody>tr>th:first-child,.panel>.table-responsive>.table-bordered>tfoot>tr>td:first-child,.panel>.table-responsive>.table-bordered>tfoot>tr>th:first-child,.panel>.table-responsive>.table-bordered>thead>tr>td:first-child,.panel>.table-responsive>.table-bordered>thead>tr>th:first-child{border-left:0}.panel>.table-bordered>tbody>tr>td:last-child,.panel>.table-bordered>tbody>tr>th:last-child,.panel>.table-bordered>tfoot>tr>td:last-child,.panel>.table-bordered>tfoot>tr>th:last-child,.panel>.table-bordered>thead>tr>td:last-child,.panel>.table-bordered>thead>tr>th:last-child,.panel>.table-responsive>.table-bordered>tbody>tr>td:last-child,.panel>.table-responsive>.table-bordered>tbody>tr>th:last-child,.panel>.table-responsive>.table-bordered>tfoot>tr>td:last-child,.panel>.table-responsive>.table-bordered>tfoot>tr>th:last-child,.panel>.table-responsive>.table-bordered>thead>tr>td:last-child,.panel>.table-responsive>.table-bordered>thead>tr>th:last-child{border-right:0}.panel>.table-bordered>tbody>tr:first-child>td,.panel>.table-bordered>tbody>tr:first-child>th,.panel>.table-bordered>thead>tr:first-child>td,.panel>.table-bordered>thead>tr:first-child>th,.panel>.table-responsive>.table-bordered>tbody>tr:first-child>td,.panel>.table-responsive>.table-bordered>tbody>tr:first-child>th,.panel>.table-responsive>.table-bordered>thead>tr:first-child>td,.panel>.table-responsive>.table-bordered>thead>tr:first-child>th{border-bottom:0}.panel>.table-bordered>tbody>tr:last-child>td,.panel>.table-bordered>tbody>tr:last-child>th,.panel>.table-bordered>tfoot>tr:last-child>td,.panel>.table-bordered>tfoot>tr:last-child>th,.panel>.table-responsive>.table-bordered>tbody>tr:last-child>td,.panel>.table-responsive>.table-bordered>tbody>tr:last-child>th,.panel>.table-responsive>.table-bordered>tfoot>tr:last-child>td,.panel>.table-responsive>.table-bordered>tfoot>tr:last-child>th{border-bottom:0}.panel>.table-responsive{margin-bottom:0;border:0}.panel-group{margin-bottom:20px}.panel-group .panel{margin-bottom:0;border-radius:4px}.panel-group .panel+.panel{margin-top:5px}.panel-group .panel-heading{border-bottom:0}.panel-group .panel-heading+.panel-collapse>.list-group,.panel-group .panel-heading+.panel-collapse>.panel-body{border-top:1px solid #ddd}.panel-group .panel-footer{border-top:0}.panel-group .panel-footer+.panel-collapse .panel-body{border-bottom:1px solid #ddd}.panel-default{border-color:#ddd}.panel-default>.panel-heading{color:#333;background-color:#f5f5f5;border-color:#ddd}.panel-default>.panel-heading+.panel-collapse>.panel-body{border-top-color:#ddd}.panel-default>.panel-heading .badge{color:#f5f5f5;background-color:#333}.panel-default>.panel-footer+.panel-collapse>.panel-body{border-bottom-color:#ddd}.panel-primary{border-color:#337ab7}.panel-primary>.panel-heading{color:#fff;background-color:#337ab7;border-color:#337ab7}.panel-primary>.panel-heading+.panel-collapse>.panel-body{border-top-color:#337ab7}.panel-primary>.panel-heading .badge{color:#337ab7;background-color:#fff}.panel-primary>.panel-footer+.panel-collapse>.panel-body{border-bottom-color:#337ab7}.panel-success{border-color:#d6e9c6}.panel-success>.panel-heading{color:#3c763d;background-color:#dff0d8;border-color:#d6e9c6}.panel-success>.panel-heading+.panel-collapse>.panel-body{border-top-color:#d6e9c6}.panel-success>.panel-heading .badge{color:#dff0d8;background-color:#3c763d}.panel-success>.panel-footer+.panel-collapse>.panel-body{border-bottom-color:#d6e9c6}.panel-info{border-color:#bce8f1}.panel-info>.panel-heading{color:#31708f;background-color:#d9edf7;border-color:#bce8f1}.panel-info>.panel-heading+.panel-collapse>.panel-body{border-top-color:#bce8f1}.panel-info>.panel-heading .badge{color:#d9edf7;background-color:#31708f}.panel-info>.panel-footer+.panel-collapse>.panel-body{border-bottom-color:#bce8f1}.panel-warning{border-color:#faebcc}.panel-warning>.panel-heading{color:#8a6d3b;background-color:#fcf8e3;border-color:#faebcc}.panel-warning>.panel-heading+.panel-collapse>.panel-body{border-top-color:#faebcc}.panel-warning>.panel-heading .badge{color:#fcf8e3;background-color:#8a6d3b}.panel-warning>.panel-footer+.panel-collapse>.panel-body{border-bottom-color:#faebcc}.panel-danger{border-color:#ebccd1}.panel-danger>.panel-heading{color:#a94442;background-color:#f2dede;border-color:#ebccd1}.panel-danger>.panel-heading+.panel-collapse>.panel-body{border-top-color:#ebccd1}.panel-danger>.panel-heading .badge{color:#f2dede;background-color:#a94442}.panel-danger>.panel-footer+.panel-collapse>.panel-body{border-bottom-color:#ebccd1}.embed-responsive{position:relative;display:block;height:0;padding:0;overflow:hidden}.embed-responsive .embed-responsive-item,.embed-responsive embed,.embed-responsive iframe,.embed-responsive object,.embed-responsive video{position:absolute;top:0;bottom:0;left:0;width:100%;height:100%;border:0}.embed-responsive-16by9{padding-bottom:56.25%}.embed-responsive-4by3{padding-bottom:75%}.well{min-height:20px;padding:19px;margin-bottom:20px;background-color:#f5f5f5;border:1px solid #e3e3e3;border-radius:4px;-webkit-box-shadow:inset 0 1px 1px rgba(0,0,0,.05);box-shadow:inset 0 1px 1px rgba(0,0,0,.05)}.well blockquote{border-color:#ddd;border-color:rgba(0,0,0,.15)}.well-lg{padding:24px;border-radius:6px}.well-sm{padding:9px;border-radius:3px}.close{float:right;font-size:21px;font-weight:700;line-height:1;color:#000;text-shadow:0 1px 0 #fff;filter:alpha(opacity=20);opacity:.2}.close:focus,.close:hover{color:#000;text-decoration:none;cursor:pointer;filter:alpha(opacity=50);opacity:.5}button.close{-webkit-appearance:none;padding:0;cursor:pointer;background:0 0;border:0}.modal-open{overflow:hidden}.modal{position:fixed;top:0;right:0;bottom:0;left:0;z-index:1050;display:none;overflow:hidden;-webkit-overflow-scrolling:touch;outline:0}.modal.fade .modal-dialog{-webkit-transition:-webkit-transform .3s ease-out;-o-transition:-o-transform .3s ease-out;transition:transform .3s ease-out;-webkit-transform:translate(0,-25%);-ms-transform:translate(0,-25%);-o-transform:translate(0,-25%);transform:translate(0,-25%)}.modal.in .modal-dialog{-webkit-transform:translate(0,0);-ms-transform:translate(0,0);-o-transform:translate(0,0);transform:translate(0,0)}.modal-open .modal{overflow-x:hidden;overflow-y:auto}.modal-dialog{position:relative;width:auto;margin:10px}.modal-content{position:relative;background-color:#fff;-webkit-background-clip:padding-box;background-clip:padding-box;border:1px solid #999;border:1px solid rgba(0,0,0,.2);border-radius:6px;outline:0;-webkit-box-shadow:0 3px 9px rgba(0,0,0,.5);box-shadow:0 3px 9px rgba(0,0,0,.5)}.modal-backdrop{position:fixed;top:0;right:0;bottom:0;left:0;z-index:1040;background-color:#000}.modal-backdrop.fade{filter:alpha(opacity=0);opacity:0}.modal-backdrop.in{filter:alpha(opacity=50);opacity:.5}.modal-header{min-height:16.43px;padding:15px;border-bottom:1px solid #e5e5e5}.modal-header .close{margin-top:-2px}.modal-title{margin:0;line-height:1.42857143}.modal-body{position:relative;padding:15px}.modal-footer{padding:15px;text-align:right;border-top:1px solid #e5e5e5}.modal-footer .btn+.btn{margin-bottom:0;margin-left:5px}.modal-footer .btn-group .btn+.btn{margin-left:-1px}.modal-footer .btn-block+.btn-block{margin-left:0}.modal-scrollbar-measure{position:absolute;top:-9999px;width:50px;height:50px;overflow:scroll}@media (min-width:768px){.modal-dialog{width:600px;margin:30px auto}.modal-content{-webkit-box-shadow:0 5px 15px rgba(0,0,0,.5);box-shadow:0 5px 15px rgba(0,0,0,.5)}.modal-sm{width:300px}}@media (min-width:992px){.modal-lg{width:900px}}.tooltip{position:absolute;z-index:1070;display:block;font-family:"Helvetica Neue",Helvetica,Arial,sans-serif;font-size:12px;font-style:normal;font-weight:400;line-height:1.42857143;text-align:left;text-align:start;text-decoration:none;text-shadow:none;text-transform:none;letter-spacing:normal;word-break:normal;word-spacing:normal;word-wrap:normal;white-space:normal;filter:alpha(opacity=0);opacity:0;line-break:auto}.tooltip.in{filter:alpha(opacity=90);opacity:.9}.tooltip.top{padding:5px 0;margin-top:-3px}.tooltip.right{padding:0 5px;margin-left:3px}.tooltip.bottom{padding:5px 0;margin-top:3px}.tooltip.left{padding:0 5px;margin-left:-3px}.tooltip-inner{max-width:200px;padding:3px 8px;color:#fff;text-align:center;background-color:#000;border-radius:4px}.tooltip-arrow{position:absolute;width:0;height:0;border-color:transparent;border-style:solid}.tooltip.top .tooltip-arrow{bottom:0;left:50%;margin-left:-5px;border-width:5px 5px 0;border-top-color:#000}.tooltip.top-left .tooltip-arrow{right:5px;bottom:0;margin-bottom:-5px;border-width:5px 5px 0;border-top-color:#000}.tooltip.top-right .tooltip-arrow{bottom:0;left:5px;margin-bottom:-5px;border-width:5px 5px 0;border-top-color:#000}.tooltip.right .tooltip-arrow{top:50%;left:0;margin-top:-5px;border-width:5px 5px 5px 0;border-right-color:#000}.tooltip.left .tooltip-arrow{top:50%;right:0;margin-top:-5px;border-width:5px 0 5px 5px;border-left-color:#000}.tooltip.bottom .tooltip-arrow{top:0;left:50%;margin-left:-5px;border-width:0 5px 5px;border-bottom-color:#000}.tooltip.bottom-left .tooltip-arrow{top:0;right:5px;margin-top:-5px;border-width:0 5px 5px;border-bottom-color:#000}.tooltip.bottom-right .tooltip-arrow{top:0;left:5px;margin-top:-5px;border-width:0 5px 5px;border-bottom-color:#000}.popover{position:absolute;top:0;left:0;z-index:1060;display:none;max-width:276px;padding:1px;font-family:"Helvetica Neue",Helvetica,Arial,sans-serif;font-size:14px;font-style:normal;font-weight:400;line-height:1.42857143;text-align:left;text-align:start;text-decoration:none;text-shadow:none;text-transform:none;letter-spacing:normal;word-break:normal;word-spacing:normal;word-wrap:normal;white-space:normal;background-color:#fff;-webkit-background-clip:padding-box;background-clip:padding-box;border:1px solid #ccc;border:1px solid rgba(0,0,0,.2);border-radius:6px;-webkit-box-shadow:0 5px 10px rgba(0,0,0,.2);box-shadow:0 5px 10px rgba(0,0,0,.2);line-break:auto}.popover.top{margin-top:-10px}.popover.right{margin-left:10px}.popover.bottom{margin-top:10px}.popover.left{margin-left:-10px}.popover-title{padding:8px 14px;margin:0;font-size:14px;background-color:#f7f7f7;border-bottom:1px solid #ebebeb;border-radius:5px 5px 0 0}.popover-content{padding:9px 14px}.popover>.arrow,.popover>.arrow:after{position:absolute;display:block;width:0;height:0;border-color:transparent;border-style:solid}.popover>.arrow{border-width:11px}.popover>.arrow:after{content:"";border-width:10px}.popover.top>.arrow{bottom:-11px;left:50%;margin-left:-11px;border-top-color:#999;border-top-color:rgba(0,0,0,.25);border-bottom-width:0}.popover.top>.arrow:after{bottom:1px;margin-left:-10px;content:" ";border-top-color:#fff;border-bottom-width:0}.popover.right>.arrow{top:50%;left:-11px;margin-top:-11px;border-right-color:#999;border-right-color:rgba(0,0,0,.25);border-left-width:0}.popover.right>.arrow:after{bottom:-10px;left:1px;content:" ";border-right-color:#fff;border-left-width:0}.popover.bottom>.arrow{top:-11px;left:50%;margin-left:-11px;border-top-width:0;border-bottom-color:#999;border-bottom-color:rgba(0,0,0,.25)}.popover.bottom>.arrow:after{top:1px;margin-left:-10px;content:" ";border-top-width:0;border-bottom-color:#fff}.popover.left>.arrow{top:50%;right:-11px;margin-top:-11px;border-right-width:0;border-left-color:#999;border-left-color:rgba(0,0,0,.25)}.popover.left>.arrow:after{right:1px;bottom:-10px;content:" ";border-right-width:0;border-left-color:#fff}.carousel{position:relative}.carousel-inner{position:relative;width:100%;overflow:hidden}.carousel-inner>.item{position:relative;display:none;-webkit-transition:.6s ease-in-out left;-o-transition:.6s ease-in-out left;transition:.6s ease-in-out left}.carousel-inner>.item>a>img,.carousel-inner>.item>img{line-height:1}@media all and (transform-3d),(-webkit-transform-3d){.carousel-inner>.item{-webkit-transition:-webkit-transform .6s ease-in-out;-o-transition:-o-transform .6s ease-in-out;transition:transform .6s ease-in-out;-webkit-backface-visibility:hidden;backface-visibility:hidden;-webkit-perspective:1000px;perspective:1000px}.carousel-inner>.item.active.right,.carousel-inner>.item.next{left:0;-webkit-transform:translate3d(100%,0,0);transform:translate3d(100%,0,0)}.carousel-inner>.item.active.left,.carousel-inner>.item.prev{left:0;-webkit-transform:translate3d(-100%,0,0);transform:translate3d(-100%,0,0)}.carousel-inner>.item.active,.carousel-inner>.item.next.left,.carousel-inner>.item.prev.right{left:0;-webkit-transform:translate3d(0,0,0);transform:translate3d(0,0,0)}}.carousel-inner>.active,.carousel-inner>.next,.carousel-inner>.prev{display:block}.carousel-inner>.active{left:0}.carousel-inner>.next,.carousel-inner>.prev{position:absolute;top:0;width:100%}.carousel-inner>.next{left:100%}.carousel-inner>.prev{left:-100%}.carousel-inner>.next.left,.carousel-inner>.prev.right{left:0}.carousel-inner>.active.left{left:-100%}.carousel-inner>.active.right{left:100%}.carousel-control{position:absolute;top:0;bottom:0;left:0;width:15%;font-size:20px;color:#fff;text-align:center;text-shadow:0 1px 2px rgba(0,0,0,.6);filter:alpha(opacity=50);opacity:.5}.carousel-control.left{background-image:-webkit-linear-gradient(left,rgba(0,0,0,.5) 0,rgba(0,0,0,.0001) 100%);background-image:-o-linear-gradient(left,rgba(0,0,0,.5) 0,rgba(0,0,0,.0001) 100%);background-image:-webkit-gradient(linear,left top,right top,from(rgba(0,0,0,.5)),to(rgba(0,0,0,.0001)));background-image:linear-gradient(to right,rgba(0,0,0,.5) 0,rgba(0,0,0,.0001) 100%);filter:progid:DXImageTransform.Microsoft.gradient(startColorstr='#80000000', endColorstr='#00000000', GradientType=1);background-repeat:repeat-x}.carousel-control.right{right:0;left:auto;background-image:-webkit-linear-gradient(left,rgba(0,0,0,.0001) 0,rgba(0,0,0,.5) 100%);background-image:-o-linear-gradient(left,rgba(0,0,0,.0001) 0,rgba(0,0,0,.5) 100%);background-image:-webkit-gradient(linear,left top,right top,from(rgba(0,0,0,.0001)),to(rgba(0,0,0,.5)));background-image:linear-gradient(to right,rgba(0,0,0,.0001) 0,rgba(0,0,0,.5) 100%);filter:progid:DXImageTransform.Microsoft.gradient(startColorstr='#00000000', endColorstr='#80000000', GradientType=1);background-repeat:repeat-x}.carousel-control:focus,.carousel-control:hover{color:#fff;text-decoration:none;filter:alpha(opacity=90);outline:0;opacity:.9}.carousel-control .glyphicon-chevron-left,.carousel-control .glyphicon-chevron-right,.carousel-control .icon-next,.carousel-control .icon-prev{position:absolute;top:50%;z-index:5;display:inline-block;margin-top:-10px}.carousel-control .glyphicon-chevron-left,.carousel-control .icon-prev{left:50%;margin-left:-10px}.carousel-control .glyphicon-chevron-right,.carousel-control .icon-next{right:50%;margin-right:-10px}.carousel-control .icon-next,.carousel-control .icon-prev{width:20px;height:20px;font-family:serif;line-height:1}.carousel-control .icon-prev:before{content:'\2039'}.carousel-control .icon-next:before{content:'\203a'}.carousel-indicators{position:absolute;bottom:10px;left:50%;z-index:15;width:60%;padding-left:0;margin-left:-30%;text-align:center;list-style:none}.carousel-indicators li{display:inline-block;width:10px;height:10px;margin:1px;text-indent:-999px;cursor:pointer;background-color:#000\9;background-color:rgba(0,0,0,0);border:1px solid #fff;border-radius:10px}.carousel-indicators .active{width:12px;height:12px;margin:0;background-color:#fff}.carousel-caption{position:absolute;right:15%;bottom:20px;left:15%;z-index:10;padding-top:20px;padding-bottom:20px;color:#fff;text-align:center;text-shadow:0 1px 2px rgba(0,0,0,.6)}.carousel-caption .btn{text-shadow:none}@media screen and (min-width:768px){.carousel-control .glyphicon-chevron-left,.carousel-control .glyphicon-chevron-right,.carousel-control .icon-next,.carousel-control .icon-prev{width:30px;height:30px;margin-top:-15px;font-size:30px}.carousel-control .glyphicon-chevron-left,.carousel-control .icon-prev{margin-left:-15px}.carousel-control .glyphicon-chevron-right,.carousel-control .icon-next{margin-right:-15px}.carousel-caption{right:20%;left:20%;padding-bottom:30px}.carousel-indicators{bottom:20px}}.btn-group-vertical>.btn-group:after,.btn-group-vertical>.btn-group:before,.btn-toolbar:after,.btn-toolbar:before,.clearfix:after,.clearfix:before,.container-fluid:after,.container-fluid:before,.container:after,.container:before,.dl-horizontal dd:after,.dl-horizontal dd:before,.form-horizontal .form-group:after,.form-horizontal .form-group:before,.modal-footer:after,.modal-footer:before,.nav:after,.nav:before,.navbar-collapse:after,.navbar-collapse:before,.navbar-header:after,.navbar-header:before,.navbar:after,.navbar:before,.pager:after,.pager:before,.panel-body:after,.panel-body:before,.row:after,.row:before{display:table;content:" "}.btn-group-vertical>.btn-group:after,.btn-toolbar:after,.clearfix:after,.container-fluid:after,.container:after,.dl-horizontal dd:after,.form-horizontal .form-group:after,.modal-footer:after,.nav:after,.navbar-collapse:after,.navbar-header:after,.navbar:after,.pager:after,.panel-body:after,.row:after{clear:both}.center-block{display:block;margin-right:auto;margin-left:auto}.pull-right{float:right!important}.pull-left{float:left!important}.hide{display:none!important}.show{display:block!important}.invisible{visibility:hidden}.text-hide{font:0/0 a;color:transparent;text-shadow:none;background-color:transparent;border:0}.hidden{display:none!important}.affix{position:fixed}@-ms-viewport{width:device-width}.visible-lg,.visible-md,.visible-sm,.visible-xs{display:none!important}.visible-lg-block,.visible-lg-inline,.visible-lg-inline-block,.visible-md-block,.visible-md-inline,.visible-md-inline-block,.visible-sm-block,.visible-sm-inline,.visible-sm-inline-block,.visible-xs-block,.visible-xs-inline,.visible-xs-inline-block{display:none!important}@media (max-width:767px){.visible-xs{display:block!important}table.visible-xs{display:table!important}tr.visible-xs{display:table-row!important}td.visible-xs,th.visible-xs{display:table-cell!important}}@media (max-width:767px){.visible-xs-block{display:block!important}}@media (max-width:767px){.visible-xs-inline{display:inline!important}}@media (max-width:767px){.visible-xs-inline-block{display:inline-block!important}}@media (min-width:768px) and (max-width:991px){.visible-sm{display:block!important}table.visible-sm{display:table!important}tr.visible-sm{display:table-row!important}td.visible-sm,th.visible-sm{display:table-cell!important}}@media (min-width:768px) and (max-width:991px){.visible-sm-block{display:block!important}}@media (min-width:768px) and (max-width:991px){.visible-sm-inline{display:inline!important}}@media (min-width:768px) and (max-width:991px){.visible-sm-inline-block{display:inline-block!important}}@media (min-width:992px) and (max-width:1199px){.visible-md{display:block!important}table.visible-md{display:table!important}tr.visible-md{display:table-row!important}td.visible-md,th.visible-md{display:table-cell!important}}@media (min-width:992px) and (max-width:1199px){.visible-md-block{display:block!important}}@media (min-width:992px) and (max-width:1199px){.visible-md-inline{display:inline!important}}@media (min-width:992px) and (max-width:1199px){.visible-md-inline-block{display:inline-block!important}}@media (min-width:1200px){.visible-lg{display:block!important}table.visible-lg{display:table!important}tr.visible-lg{display:table-row!important}td.visible-lg,th.visible-lg{display:table-cell!important}}@media (min-width:1200px){.visible-lg-block{display:block!important}}@media (min-width:1200px){.visible-lg-inline{display:inline!important}}@media (min-width:1200px){.visible-lg-inline-block{display:inline-block!important}}@media (max-width:767px){.hidden-xs{display:none!important}}@media (min-width:768px) and (max-width:991px){.hidden-sm{display:none!important}}@media (min-width:992px) and (max-width:1199px){.hidden-md{display:none!important}}@media (min-width:1200px){.hidden-lg{display:none!important}}.visible-print{display:none!important}@media print{.visible-print{display:block!important}table.visible-print{display:table!important}tr.visible-print{display:table-row!important}td.visible-print,th.visible-print{display:table-cell!important}}.visible-print-block{display:none!important}@media print{.visible-print-block{display:block!important}}.visible-print-inline{display:none!important}@media print{.visible-print-inline{display:inline!important}}.visible-print-inline-block{display:none!important}@media print{.visible-print-inline-block{display:inline-block!important}}@media print{.hidden-print{display:none!important}}
-</style>
-<script>/*!
- * Bootstrap v3.3.5 (http://getbootstrap.com)
- * Copyright 2011-2015 Twitter, Inc.
- * Licensed under the MIT license
- */
-if("undefined"==typeof jQuery)throw new Error("Bootstrap's JavaScript requires jQuery");+function(a){"use strict";var b=a.fn.jquery.split(" ")[0].split(".");if(b[0]<2&&b[1]<9||1==b[0]&&9==b[1]&&b[2]<1)throw new Error("Bootstrap's JavaScript requires jQuery version 1.9.1 or higher")}(jQuery),+function(a){"use strict";function b(){var a=document.createElement("bootstrap"),b={WebkitTransition:"webkitTransitionEnd",MozTransition:"transitionend",OTransition:"oTransitionEnd otransitionend",transition:"transitionend"};for(var c in b)if(void 0!==a.style[c])return{end:b[c]};return!1}a.fn.emulateTransitionEnd=function(b){var c=!1,d=this;a(this).one("bsTransitionEnd",function(){c=!0});var e=function(){c||a(d).trigger(a.support.transition.end)};return setTimeout(e,b),this},a(function(){a.support.transition=b(),a.support.transition&&(a.event.special.bsTransitionEnd={bindType:a.support.transition.end,delegateType:a.support.transition.end,handle:function(b){return a(b.target).is(this)?b.handleObj.handler.apply(this,arguments):void 0}})})}(jQuery),+function(a){"use strict";function b(b){return this.each(function(){var c=a(this),e=c.data("bs.alert");e||c.data("bs.alert",e=new d(this)),"string"==typeof b&&e[b].call(c)})}var c='[data-dismiss="alert"]',d=function(b){a(b).on("click",c,this.close)};d.VERSION="3.3.5",d.TRANSITION_DURATION=150,d.prototype.close=function(b){function c(){g.detach().trigger("closed.bs.alert").remove()}var e=a(this),f=e.attr("data-target");f||(f=e.attr("href"),f=f&&f.replace(/.*(?=#[^\s]*$)/,""));var g=a(f);b&&b.preventDefault(),g.length||(g=e.closest(".alert")),g.trigger(b=a.Event("close.bs.alert")),b.isDefaultPrevented()||(g.removeClass("in"),a.support.transition&&g.hasClass("fade")?g.one("bsTransitionEnd",c).emulateTransitionEnd(d.TRANSITION_DURATION):c())};var e=a.fn.alert;a.fn.alert=b,a.fn.alert.Constructor=d,a.fn.alert.noConflict=function(){return a.fn.alert=e,this},a(document).on("click.bs.alert.data-api",c,d.prototype.close)}(jQuery),+function(a){"use strict";function b(b){return this.each(function(){var d=a(this),e=d.data("bs.button"),f="object"==typeof b&&b;e||d.data("bs.button",e=new c(this,f)),"toggle"==b?e.toggle():b&&e.setState(b)})}var c=function(b,d){this.$element=a(b),this.options=a.extend({},c.DEFAULTS,d),this.isLoading=!1};c.VERSION="3.3.5",c.DEFAULTS={loadingText:"loading..."},c.prototype.setState=function(b){var c="disabled",d=this.$element,e=d.is("input")?"val":"html",f=d.data();b+="Text",null==f.resetText&&d.data("resetText",d[e]()),setTimeout(a.proxy(function(){d[e](null==f[b]?this.options[b]:f[b]),"loadingText"==b?(this.isLoading=!0,d.addClass(c).attr(c,c)):this.isLoading&&(this.isLoading=!1,d.removeClass(c).removeAttr(c))},this),0)},c.prototype.toggle=function(){var a=!0,b=this.$element.closest('[data-toggle="buttons"]');if(b.length){var c=this.$element.find("input");"radio"==c.prop("type")?(c.prop("checked")&&(a=!1),b.find(".active").removeClass("active"),this.$element.addClass("active")):"checkbox"==c.prop("type")&&(c.prop("checked")!==this.$element.hasClass("active")&&(a=!1),this.$element.toggleClass("active")),c.prop("checked",this.$element.hasClass("active")),a&&c.trigger("change")}else this.$element.attr("aria-pressed",!this.$element.hasClass("active")),this.$element.toggleClass("active")};var d=a.fn.button;a.fn.button=b,a.fn.button.Constructor=c,a.fn.button.noConflict=function(){return a.fn.button=d,this},a(document).on("click.bs.button.data-api",'[data-toggle^="button"]',function(c){var d=a(c.target);d.hasClass("btn")||(d=d.closest(".btn")),b.call(d,"toggle"),a(c.target).is('input[type="radio"]')||a(c.target).is('input[type="checkbox"]')||c.preventDefault()}).on("focus.bs.button.data-api blur.bs.button.data-api",'[data-toggle^="button"]',function(b){a(b.target).closest(".btn").toggleClass("focus",/^focus(in)?$/.test(b.type))})}(jQuery),+function(a){"use strict";function b(b){return this.each(function(){var d=a(this),e=d.data("bs.carousel"),f=a.extend({},c.DEFAULTS,d.data(),"object"==typeof b&&b),g="string"==typeof b?b:f.slide;e||d.data("bs.carousel",e=new c(this,f)),"number"==typeof b?e.to(b):g?e[g]():f.interval&&e.pause().cycle()})}var c=function(b,c){this.$element=a(b),this.$indicators=this.$element.find(".carousel-indicators"),this.options=c,this.paused=null,this.sliding=null,this.interval=null,this.$active=null,this.$items=null,this.options.keyboard&&this.$element.on("keydown.bs.carousel",a.proxy(this.keydown,this)),"hover"==this.options.pause&&!("ontouchstart"in document.documentElement)&&this.$element.on("mouseenter.bs.carousel",a.proxy(this.pause,this)).on("mouseleave.bs.carousel",a.proxy(this.cycle,this))};c.VERSION="3.3.5",c.TRANSITION_DURATION=600,c.DEFAULTS={interval:5e3,pause:"hover",wrap:!0,keyboard:!0},c.prototype.keydown=function(a){if(!/input|textarea/i.test(a.target.tagName)){switch(a.which){case 37:this.prev();break;case 39:this.next();break;default:return}a.preventDefault()}},c.prototype.cycle=function(b){return b||(this.paused=!1),this.interval&&clearInterval(this.interval),this.options.interval&&!this.paused&&(this.interval=setInterval(a.proxy(this.next,this),this.options.interval)),this},c.prototype.getItemIndex=function(a){return this.$items=a.parent().children(".item"),this.$items.index(a||this.$active)},c.prototype.getItemForDirection=function(a,b){var c=this.getItemIndex(b),d="prev"==a&&0===c||"next"==a&&c==this.$items.length-1;if(d&&!this.options.wrap)return b;var e="prev"==a?-1:1,f=(c+e)%this.$items.length;return this.$items.eq(f)},c.prototype.to=function(a){var b=this,c=this.getItemIndex(this.$active=this.$element.find(".item.active"));return a>this.$items.length-1||0>a?void 0:this.sliding?this.$element.one("slid.bs.carousel",function(){b.to(a)}):c==a?this.pause().cycle():this.slide(a>c?"next":"prev",this.$items.eq(a))},c.prototype.pause=function(b){return b||(this.paused=!0),this.$element.find(".next, .prev").length&&a.support.transition&&(this.$element.trigger(a.support.transition.end),this.cycle(!0)),this.interval=clearInterval(this.interval),this},c.prototype.next=function(){return this.sliding?void 0:this.slide("next")},c.prototype.prev=function(){return this.sliding?void 0:this.slide("prev")},c.prototype.slide=function(b,d){var e=this.$element.find(".item.active"),f=d||this.getItemForDirection(b,e),g=this.interval,h="next"==b?"left":"right",i=this;if(f.hasClass("active"))return this.sliding=!1;var j=f[0],k=a.Event("slide.bs.carousel",{relatedTarget:j,direction:h});if(this.$element.trigger(k),!k.isDefaultPrevented()){if(this.sliding=!0,g&&this.pause(),this.$indicators.length){this.$indicators.find(".active").removeClass("active");var l=a(this.$indicators.children()[this.getItemIndex(f)]);l&&l.addClass("active")}var m=a.Event("slid.bs.carousel",{relatedTarget:j,direction:h});return a.support.transition&&this.$element.hasClass("slide")?(f.addClass(b),f[0].offsetWidth,e.addClass(h),f.addClass(h),e.one("bsTransitionEnd",function(){f.removeClass([b,h].join(" ")).addClass("active"),e.removeClass(["active",h].join(" ")),i.sliding=!1,setTimeout(function(){i.$element.trigger(m)},0)}).emulateTransitionEnd(c.TRANSITION_DURATION)):(e.removeClass("active"),f.addClass("active"),this.sliding=!1,this.$element.trigger(m)),g&&this.cycle(),this}};var d=a.fn.carousel;a.fn.carousel=b,a.fn.carousel.Constructor=c,a.fn.carousel.noConflict=function(){return a.fn.carousel=d,this};var e=function(c){var d,e=a(this),f=a(e.attr("data-target")||(d=e.attr("href"))&&d.replace(/.*(?=#[^\s]+$)/,""));if(f.hasClass("carousel")){var g=a.extend({},f.data(),e.data()),h=e.attr("data-slide-to");h&&(g.interval=!1),b.call(f,g),h&&f.data("bs.carousel").to(h),c.preventDefault()}};a(document).on("click.bs.carousel.data-api","[data-slide]",e).on("click.bs.carousel.data-api","[data-slide-to]",e),a(window).on("load",function(){a('[data-ride="carousel"]').each(function(){var c=a(this);b.call(c,c.data())})})}(jQuery),+function(a){"use strict";function b(b){var c,d=b.attr("data-target")||(c=b.attr("href"))&&c.replace(/.*(?=#[^\s]+$)/,"");return a(d)}function c(b){return this.each(function(){var c=a(this),e=c.data("bs.collapse"),f=a.extend({},d.DEFAULTS,c.data(),"object"==typeof b&&b);!e&&f.toggle&&/show|hide/.test(b)&&(f.toggle=!1),e||c.data("bs.collapse",e=new d(this,f)),"string"==typeof b&&e[b]()})}var d=function(b,c){this.$element=a(b),this.options=a.extend({},d.DEFAULTS,c),this.$trigger=a('[data-toggle="collapse"][href="#'+b.id+'"],[data-toggle="collapse"][data-target="#'+b.id+'"]'),this.transitioning=null,this.options.parent?this.$parent=this.getParent():this.addAriaAndCollapsedClass(this.$element,this.$trigger),this.options.toggle&&this.toggle()};d.VERSION="3.3.5",d.TRANSITION_DURATION=350,d.DEFAULTS={toggle:!0},d.prototype.dimension=function(){var a=this.$element.hasClass("width");return a?"width":"height"},d.prototype.show=function(){if(!this.transitioning&&!this.$element.hasClass("in")){var b,e=this.$parent&&this.$parent.children(".panel").children(".in, .collapsing");if(!(e&&e.length&&(b=e.data("bs.collapse"),b&&b.transitioning))){var f=a.Event("show.bs.collapse");if(this.$element.trigger(f),!f.isDefaultPrevented()){e&&e.length&&(c.call(e,"hide"),b||e.data("bs.collapse",null));var g=this.dimension();this.$element.removeClass("collapse").addClass("collapsing")[g](0).attr("aria-expanded",!0),this.$trigger.removeClass("collapsed").attr("aria-expanded",!0),this.transitioning=1;var h=function(){this.$element.removeClass("collapsing").addClass("collapse in")[g](""),this.transitioning=0,this.$element.trigger("shown.bs.collapse")};if(!a.support.transition)return h.call(this);var i=a.camelCase(["scroll",g].join("-"));this.$element.one("bsTransitionEnd",a.proxy(h,this)).emulateTransitionEnd(d.TRANSITION_DURATION)[g](this.$element[0][i])}}}},d.prototype.hide=function(){if(!this.transitioning&&this.$element.hasClass("in")){var b=a.Event("hide.bs.collapse");if(this.$element.trigger(b),!b.isDefaultPrevented()){var c=this.dimension();this.$element[c](this.$element[c]())[0].offsetHeight,this.$element.addClass("collapsing").removeClass("collapse in").attr("aria-expanded",!1),this.$trigger.addClass("collapsed").attr("aria-expanded",!1),this.transitioning=1;var e=function(){this.transitioning=0,this.$element.removeClass("collapsing").addClass("collapse").trigger("hidden.bs.collapse")};return a.support.transition?void this.$element[c](0).one("bsTransitionEnd",a.proxy(e,this)).emulateTransitionEnd(d.TRANSITION_DURATION):e.call(this)}}},d.prototype.toggle=function(){this[this.$element.hasClass("in")?"hide":"show"]()},d.prototype.getParent=function(){return a(this.options.parent).find('[data-toggle="collapse"][data-parent="'+this.options.parent+'"]').each(a.proxy(function(c,d){var e=a(d);this.addAriaAndCollapsedClass(b(e),e)},this)).end()},d.prototype.addAriaAndCollapsedClass=function(a,b){var c=a.hasClass("in");a.attr("aria-expanded",c),b.toggleClass("collapsed",!c).attr("aria-expanded",c)};var e=a.fn.collapse;a.fn.collapse=c,a.fn.collapse.Constructor=d,a.fn.collapse.noConflict=function(){return a.fn.collapse=e,this},a(document).on("click.bs.collapse.data-api",'[data-toggle="collapse"]',function(d){var e=a(this);e.attr("data-target")||d.preventDefault();var f=b(e),g=f.data("bs.collapse"),h=g?"toggle":e.data();c.call(f,h)})}(jQuery),+function(a){"use strict";function b(b){var c=b.attr("data-target");c||(c=b.attr("href"),c=c&&/#[A-Za-z]/.test(c)&&c.replace(/.*(?=#[^\s]*$)/,""));var d=c&&a(c);return d&&d.length?d:b.parent()}function c(c){c&&3===c.which||(a(e).remove(),a(f).each(function(){var d=a(this),e=b(d),f={relatedTarget:this};e.hasClass("open")&&(c&&"click"==c.type&&/input|textarea/i.test(c.target.tagName)&&a.contains(e[0],c.target)||(e.trigger(c=a.Event("hide.bs.dropdown",f)),c.isDefaultPrevented()||(d.attr("aria-expanded","false"),e.removeClass("open").trigger("hidden.bs.dropdown",f))))}))}function d(b){return this.each(function(){var c=a(this),d=c.data("bs.dropdown");d||c.data("bs.dropdown",d=new g(this)),"string"==typeof b&&d[b].call(c)})}var e=".dropdown-backdrop",f='[data-toggle="dropdown"]',g=function(b){a(b).on("click.bs.dropdown",this.toggle)};g.VERSION="3.3.5",g.prototype.toggle=function(d){var e=a(this);if(!e.is(".disabled, :disabled")){var f=b(e),g=f.hasClass("open");if(c(),!g){"ontouchstart"in document.documentElement&&!f.closest(".navbar-nav").length&&a(document.createElement("div")).addClass("dropdown-backdrop").insertAfter(a(this)).on("click",c);var h={relatedTarget:this};if(f.trigger(d=a.Event("show.bs.dropdown",h)),d.isDefaultPrevented())return;e.trigger("focus").attr("aria-expanded","true"),f.toggleClass("open").trigger("shown.bs.dropdown",h)}return!1}},g.prototype.keydown=function(c){if(/(38|40|27|32)/.test(c.which)&&!/input|textarea/i.test(c.target.tagName)){var d=a(this);if(c.preventDefault(),c.stopPropagation(),!d.is(".disabled, :disabled")){var e=b(d),g=e.hasClass("open");if(!g&&27!=c.which||g&&27==c.which)return 27==c.which&&e.find(f).trigger("focus"),d.trigger("click");var h=" li:not(.disabled):visible a",i=e.find(".dropdown-menu"+h);if(i.length){var j=i.index(c.target);38==c.which&&j>0&&j--,40==c.which&&j<i.length-1&&j++,~j||(j=0),i.eq(j).trigger("focus")}}}};var h=a.fn.dropdown;a.fn.dropdown=d,a.fn.dropdown.Constructor=g,a.fn.dropdown.noConflict=function(){return a.fn.dropdown=h,this},a(document).on("click.bs.dropdown.data-api",c).on("click.bs.dropdown.data-api",".dropdown form",function(a){a.stopPropagation()}).on("click.bs.dropdown.data-api",f,g.prototype.toggle).on("keydown.bs.dropdown.data-api",f,g.prototype.keydown).on("keydown.bs.dropdown.data-api",".dropdown-menu",g.prototype.keydown)}(jQuery),+function(a){"use strict";function b(b,d){return this.each(function(){var e=a(this),f=e.data("bs.modal"),g=a.extend({},c.DEFAULTS,e.data(),"object"==typeof b&&b);f||e.data("bs.modal",f=new c(this,g)),"string"==typeof b?f[b](d):g.show&&f.show(d)})}var c=function(b,c){this.options=c,this.$body=a(document.body),this.$element=a(b),this.$dialog=this.$element.find(".modal-dialog"),this.$backdrop=null,this.isShown=null,this.originalBodyPad=null,this.scrollbarWidth=0,this.ignoreBackdropClick=!1,this.options.remote&&this.$element.find(".modal-content").load(this.options.remote,a.proxy(function(){this.$element.trigger("loaded.bs.modal")},this))};c.VERSION="3.3.5",c.TRANSITION_DURATION=300,c.BACKDROP_TRANSITION_DURATION=150,c.DEFAULTS={backdrop:!0,keyboard:!0,show:!0},c.prototype.toggle=function(a){return this.isShown?this.hide():this.show(a)},c.prototype.show=function(b){var d=this,e=a.Event("show.bs.modal",{relatedTarget:b});this.$element.trigger(e),this.isShown||e.isDefaultPrevented()||(this.isShown=!0,this.checkScrollbar(),this.setScrollbar(),this.$body.addClass("modal-open"),this.escape(),this.resize(),this.$element.on("click.dismiss.bs.modal",'[data-dismiss="modal"]',a.proxy(this.hide,this)),this.$dialog.on("mousedown.dismiss.bs.modal",function(){d.$element.one("mouseup.dismiss.bs.modal",function(b){a(b.target).is(d.$element)&&(d.ignoreBackdropClick=!0)})}),this.backdrop(function(){var e=a.support.transition&&d.$element.hasClass("fade");d.$element.parent().length||d.$element.appendTo(d.$body),d.$element.show().scrollTop(0),d.adjustDialog(),e&&d.$element[0].offsetWidth,d.$element.addClass("in"),d.enforceFocus();var f=a.Event("shown.bs.modal",{relatedTarget:b});e?d.$dialog.one("bsTransitionEnd",function(){d.$element.trigger("focus").trigger(f)}).emulateTransitionEnd(c.TRANSITION_DURATION):d.$element.trigger("focus").trigger(f)}))},c.prototype.hide=function(b){b&&b.preventDefault(),b=a.Event("hide.bs.modal"),this.$element.trigger(b),this.isShown&&!b.isDefaultPrevented()&&(this.isShown=!1,this.escape(),this.resize(),a(document).off("focusin.bs.modal"),this.$element.removeClass("in").off("click.dismiss.bs.modal").off("mouseup.dismiss.bs.modal"),this.$dialog.off("mousedown.dismiss.bs.modal"),a.support.transition&&this.$element.hasClass("fade")?this.$element.one("bsTransitionEnd",a.proxy(this.hideModal,this)).emulateTransitionEnd(c.TRANSITION_DURATION):this.hideModal())},c.prototype.enforceFocus=function(){a(document).off("focusin.bs.modal").on("focusin.bs.modal",a.proxy(function(a){this.$element[0]===a.target||this.$element.has(a.target).length||this.$element.trigger("focus")},this))},c.prototype.escape=function(){this.isShown&&this.options.keyboard?this.$element.on("keydown.dismiss.bs.modal",a.proxy(function(a){27==a.which&&this.hide()},this)):this.isShown||this.$element.off("keydown.dismiss.bs.modal")},c.prototype.resize=function(){this.isShown?a(window).on("resize.bs.modal",a.proxy(this.handleUpdate,this)):a(window).off("resize.bs.modal")},c.prototype.hideModal=function(){var a=this;this.$element.hide(),this.backdrop(function(){a.$body.removeClass("modal-open"),a.resetAdjustments(),a.resetScrollbar(),a.$element.trigger("hidden.bs.modal")})},c.prototype.removeBackdrop=function(){this.$backdrop&&this.$backdrop.remove(),this.$backdrop=null},c.prototype.backdrop=function(b){var d=this,e=this.$element.hasClass("fade")?"fade":"";if(this.isShown&&this.options.backdrop){var f=a.support.transition&&e;if(this.$backdrop=a(document.createElement("div")).addClass("modal-backdrop "+e).appendTo(this.$body),this.$element.on("click.dismiss.bs.modal",a.proxy(function(a){return this.ignoreBackdropClick?void(this.ignoreBackdropClick=!1):void(a.target===a.currentTarget&&("static"==this.options.backdrop?this.$element[0].focus():this.hide()))},this)),f&&this.$backdrop[0].offsetWidth,this.$backdrop.addClass("in"),!b)return;f?this.$backdrop.one("bsTransitionEnd",b).emulateTransitionEnd(c.BACKDROP_TRANSITION_DURATION):b()}else if(!this.isShown&&this.$backdrop){this.$backdrop.removeClass("in");var g=function(){d.removeBackdrop(),b&&b()};a.support.transition&&this.$element.hasClass("fade")?this.$backdrop.one("bsTransitionEnd",g).emulateTransitionEnd(c.BACKDROP_TRANSITION_DURATION):g()}else b&&b()},c.prototype.handleUpdate=function(){this.adjustDialog()},c.prototype.adjustDialog=function(){var a=this.$element[0].scrollHeight>document.documentElement.clientHeight;this.$element.css({paddingLeft:!this.bodyIsOverflowing&&a?this.scrollbarWidth:"",paddingRight:this.bodyIsOverflowing&&!a?this.scrollbarWidth:""})},c.prototype.resetAdjustments=function(){this.$element.css({paddingLeft:"",paddingRight:""})},c.prototype.checkScrollbar=function(){var a=window.innerWidth;if(!a){var b=document.documentElement.getBoundingClientRect();a=b.right-Math.abs(b.left)}this.bodyIsOverflowing=document.body.clientWidth<a,this.scrollbarWidth=this.measureScrollbar()},c.prototype.setScrollbar=function(){var a=parseInt(this.$body.css("padding-right")||0,10);this.originalBodyPad=document.body.style.paddingRight||"",this.bodyIsOverflowing&&this.$body.css("padding-right",a+this.scrollbarWidth)},c.prototype.resetScrollbar=function(){this.$body.css("padding-right",this.originalBodyPad)},c.prototype.measureScrollbar=function(){var a=document.createElement("div");a.className="modal-scrollbar-measure",this.$body.append(a);var b=a.offsetWidth-a.clientWidth;return this.$body[0].removeChild(a),b};var d=a.fn.modal;a.fn.modal=b,a.fn.modal.Constructor=c,a.fn.modal.noConflict=function(){return a.fn.modal=d,this},a(document).on("click.bs.modal.data-api",'[data-toggle="modal"]',function(c){var d=a(this),e=d.attr("href"),f=a(d.attr("data-target")||e&&e.replace(/.*(?=#[^\s]+$)/,"")),g=f.data("bs.modal")?"toggle":a.extend({remote:!/#/.test(e)&&e},f.data(),d.data());d.is("a")&&c.preventDefault(),f.one("show.bs.modal",function(a){a.isDefaultPrevented()||f.one("hidden.bs.modal",function(){d.is(":visible")&&d.trigger("focus")})}),b.call(f,g,this)})}(jQuery),+function(a){"use strict";function b(b){return this.each(function(){var d=a(this),e=d.data("bs.tooltip"),f="object"==typeof b&&b;(e||!/destroy|hide/.test(b))&&(e||d.data("bs.tooltip",e=new c(this,f)),"string"==typeof b&&e[b]())})}var c=function(a,b){this.type=null,this.options=null,this.enabled=null,this.timeout=null,this.hoverState=null,this.$element=null,this.inState=null,this.init("tooltip",a,b)};c.VERSION="3.3.5",c.TRANSITION_DURATION=150,c.DEFAULTS={animation:!0,placement:"top",selector:!1,template:'<div class="tooltip" role="tooltip"><div class="tooltip-arrow"></div><div class="tooltip-inner"></div></div>',trigger:"hover focus",title:"",delay:0,html:!1,container:!1,viewport:{selector:"body",padding:0}},c.prototype.init=function(b,c,d){if(this.enabled=!0,this.type=b,this.$element=a(c),this.options=this.getOptions(d),this.$viewport=this.options.viewport&&a(a.isFunction(this.options.viewport)?this.options.viewport.call(this,this.$element):this.options.viewport.selector||this.options.viewport),this.inState={click:!1,hover:!1,focus:!1},this.$element[0]instanceof document.constructor&&!this.options.selector)throw new Error("`selector` option must be specified when initializing "+this.type+" on the window.document object!");for(var e=this.options.trigger.split(" "),f=e.length;f--;){var g=e[f];if("click"==g)this.$element.on("click."+this.type,this.options.selector,a.proxy(this.toggle,this));else if("manual"!=g){var h="hover"==g?"mouseenter":"focusin",i="hover"==g?"mouseleave":"focusout";this.$element.on(h+"."+this.type,this.options.selector,a.proxy(this.enter,this)),this.$element.on(i+"."+this.type,this.options.selector,a.proxy(this.leave,this))}}this.options.selector?this._options=a.extend({},this.options,{trigger:"manual",selector:""}):this.fixTitle()},c.prototype.getDefaults=function(){return c.DEFAULTS},c.prototype.getOptions=function(b){return b=a.extend({},this.getDefaults(),this.$element.data(),b),b.delay&&"number"==typeof b.delay&&(b.delay={show:b.delay,hide:b.delay}),b},c.prototype.getDelegateOptions=function(){var b={},c=this.getDefaults();return this._options&&a.each(this._options,function(a,d){c[a]!=d&&(b[a]=d)}),b},c.prototype.enter=function(b){var c=b instanceof this.constructor?b:a(b.currentTarget).data("bs."+this.type);return c||(c=new this.constructor(b.currentTarget,this.getDelegateOptions()),a(b.currentTarget).data("bs."+this.type,c)),b instanceof a.Event&&(c.inState["focusin"==b.type?"focus":"hover"]=!0),c.tip().hasClass("in")||"in"==c.hoverState?void(c.hoverState="in"):(clearTimeout(c.timeout),c.hoverState="in",c.options.delay&&c.options.delay.show?void(c.timeout=setTimeout(function(){"in"==c.hoverState&&c.show()},c.options.delay.show)):c.show())},c.prototype.isInStateTrue=function(){for(var a in this.inState)if(this.inState[a])return!0;return!1},c.prototype.leave=function(b){var c=b instanceof this.constructor?b:a(b.currentTarget).data("bs."+this.type);return c||(c=new this.constructor(b.currentTarget,this.getDelegateOptions()),a(b.currentTarget).data("bs."+this.type,c)),b instanceof a.Event&&(c.inState["focusout"==b.type?"focus":"hover"]=!1),c.isInStateTrue()?void 0:(clearTimeout(c.timeout),c.hoverState="out",c.options.delay&&c.options.delay.hide?void(c.timeout=setTimeout(function(){"out"==c.hoverState&&c.hide()},c.options.delay.hide)):c.hide())},c.prototype.show=function(){var b=a.Event("show.bs."+this.type);if(this.hasContent()&&this.enabled){this.$element.trigger(b);var d=a.contains(this.$element[0].ownerDocument.documentElement,this.$element[0]);if(b.isDefaultPrevented()||!d)return;var e=this,f=this.tip(),g=this.getUID(this.type);this.setContent(),f.attr("id",g),this.$element.attr("aria-describedby",g),this.options.animation&&f.addClass("fade");var h="function"==typeof this.options.placement?this.options.placement.call(this,f[0],this.$element[0]):this.options.placement,i=/\s?auto?\s?/i,j=i.test(h);j&&(h=h.replace(i,"")||"top"),f.detach().css({top:0,left:0,display:"block"}).addClass(h).data("bs."+this.type,this),this.options.container?f.appendTo(this.options.container):f.insertAfter(this.$element),this.$element.trigger("inserted.bs."+this.type);var k=this.getPosition(),l=f[0].offsetWidth,m=f[0].offsetHeight;if(j){var n=h,o=this.getPosition(this.$viewport);h="bottom"==h&&k.bottom+m>o.bottom?"top":"top"==h&&k.top-m<o.top?"bottom":"right"==h&&k.right+l>o.width?"left":"left"==h&&k.left-l<o.left?"right":h,f.removeClass(n).addClass(h)}var p=this.getCalculatedOffset(h,k,l,m);this.applyPlacement(p,h);var q=function(){var a=e.hoverState;e.$element.trigger("shown.bs."+e.type),e.hoverState=null,"out"==a&&e.leave(e)};a.support.transition&&this.$tip.hasClass("fade")?f.one("bsTransitionEnd",q).emulateTransitionEnd(c.TRANSITION_DURATION):q()}},c.prototype.applyPlacement=function(b,c){var d=this.tip(),e=d[0].offsetWidth,f=d[0].offsetHeight,g=parseInt(d.css("margin-top"),10),h=parseInt(d.css("margin-left"),10);isNaN(g)&&(g=0),isNaN(h)&&(h=0),b.top+=g,b.left+=h,a.offset.setOffset(d[0],a.extend({using:function(a){d.css({top:Math.round(a.top),left:Math.round(a.left)})}},b),0),d.addClass("in");var i=d[0].offsetWidth,j=d[0].offsetHeight;"top"==c&&j!=f&&(b.top=b.top+f-j);var k=this.getViewportAdjustedDelta(c,b,i,j);k.left?b.left+=k.left:b.top+=k.top;var l=/top|bottom/.test(c),m=l?2*k.left-e+i:2*k.top-f+j,n=l?"offsetWidth":"offsetHeight";d.offset(b),this.replaceArrow(m,d[0][n],l)},c.prototype.replaceArrow=function(a,b,c){this.arrow().css(c?"left":"top",50*(1-a/b)+"%").css(c?"top":"left","")},c.prototype.setContent=function(){var a=this.tip(),b=this.getTitle();a.find(".tooltip-inner")[this.options.html?"html":"text"](b),a.removeClass("fade in top bottom left right")},c.prototype.hide=function(b){function d(){"in"!=e.hoverState&&f.detach(),e.$element.removeAttr("aria-describedby").trigger("hidden.bs."+e.type),b&&b()}var e=this,f=a(this.$tip),g=a.Event("hide.bs."+this.type);return this.$element.trigger(g),g.isDefaultPrevented()?void 0:(f.removeClass("in"),a.support.transition&&f.hasClass("fade")?f.one("bsTransitionEnd",d).emulateTransitionEnd(c.TRANSITION_DURATION):d(),this.hoverState=null,this)},c.prototype.fixTitle=function(){var a=this.$element;(a.attr("title")||"string"!=typeof a.attr("data-original-title"))&&a.attr("data-original-title",a.attr("title")||"").attr("title","")},c.prototype.hasContent=function(){return this.getTitle()},c.prototype.getPosition=function(b){b=b||this.$element;var c=b[0],d="BODY"==c.tagName,e=c.getBoundingClientRect();null==e.width&&(e=a.extend({},e,{width:e.right-e.left,height:e.bottom-e.top}));var f=d?{top:0,left:0}:b.offset(),g={scroll:d?document.documentElement.scrollTop||document.body.scrollTop:b.scrollTop()},h=d?{width:a(window).width(),height:a(window).height()}:null;return a.extend({},e,g,h,f)},c.prototype.getCalculatedOffset=function(a,b,c,d){return"bottom"==a?{top:b.top+b.height,left:b.left+b.width/2-c/2}:"top"==a?{top:b.top-d,left:b.left+b.width/2-c/2}:"left"==a?{top:b.top+b.height/2-d/2,left:b.left-c}:{top:b.top+b.height/2-d/2,left:b.left+b.width}},c.prototype.getViewportAdjustedDelta=function(a,b,c,d){var e={top:0,left:0};if(!this.$viewport)return e;var f=this.options.viewport&&this.options.viewport.padding||0,g=this.getPosition(this.$viewport);if(/right|left/.test(a)){var h=b.top-f-g.scroll,i=b.top+f-g.scroll+d;h<g.top?e.top=g.top-h:i>g.top+g.height&&(e.top=g.top+g.height-i)}else{var j=b.left-f,k=b.left+f+c;j<g.left?e.left=g.left-j:k>g.right&&(e.left=g.left+g.width-k)}return e},c.prototype.getTitle=function(){var a,b=this.$element,c=this.options;return a=b.attr("data-original-title")||("function"==typeof c.title?c.title.call(b[0]):c.title)},c.prototype.getUID=function(a){do a+=~~(1e6*Math.random());while(document.getElementById(a));return a},c.prototype.tip=function(){if(!this.$tip&&(this.$tip=a(this.options.template),1!=this.$tip.length))throw new Error(this.type+" `template` option must consist of exactly 1 top-level element!");return this.$tip},c.prototype.arrow=function(){return this.$arrow=this.$arrow||this.tip().find(".tooltip-arrow")},c.prototype.enable=function(){this.enabled=!0},c.prototype.disable=function(){this.enabled=!1},c.prototype.toggleEnabled=function(){this.enabled=!this.enabled},c.prototype.toggle=function(b){var c=this;b&&(c=a(b.currentTarget).data("bs."+this.type),c||(c=new this.constructor(b.currentTarget,this.getDelegateOptions()),a(b.currentTarget).data("bs."+this.type,c))),b?(c.inState.click=!c.inState.click,c.isInStateTrue()?c.enter(c):c.leave(c)):c.tip().hasClass("in")?c.leave(c):c.enter(c)},c.prototype.destroy=function(){var a=this;clearTimeout(this.timeout),this.hide(function(){a.$element.off("."+a.type).removeData("bs."+a.type),a.$tip&&a.$tip.detach(),a.$tip=null,a.$arrow=null,a.$viewport=null})};var d=a.fn.tooltip;a.fn.tooltip=b,a.fn.tooltip.Constructor=c,a.fn.tooltip.noConflict=function(){return a.fn.tooltip=d,this}}(jQuery),+function(a){"use strict";function b(b){return this.each(function(){var d=a(this),e=d.data("bs.popover"),f="object"==typeof b&&b;(e||!/destroy|hide/.test(b))&&(e||d.data("bs.popover",e=new c(this,f)),"string"==typeof b&&e[b]())})}var c=function(a,b){this.init("popover",a,b)};if(!a.fn.tooltip)throw new Error("Popover requires tooltip.js");c.VERSION="3.3.5",c.DEFAULTS=a.extend({},a.fn.tooltip.Constructor.DEFAULTS,{placement:"right",trigger:"click",content:"",template:'<div class="popover" role="tooltip"><div class="arrow"></div><h3 class="popover-title"></h3><div class="popover-content"></div></div>'}),c.prototype=a.extend({},a.fn.tooltip.Constructor.prototype),c.prototype.constructor=c,c.prototype.getDefaults=function(){return c.DEFAULTS},c.prototype.setContent=function(){var a=this.tip(),b=this.getTitle(),c=this.getContent();a.find(".popover-title")[this.options.html?"html":"text"](b),a.find(".popover-content").children().detach().end()[this.options.html?"string"==typeof c?"html":"append":"text"](c),a.removeClass("fade top bottom left right in"),a.find(".popover-title").html()||a.find(".popover-title").hide()},c.prototype.hasContent=function(){return this.getTitle()||this.getContent()},c.prototype.getContent=function(){var a=this.$element,b=this.options;return a.attr("data-content")||("function"==typeof b.content?b.content.call(a[0]):b.content)},c.prototype.arrow=function(){return this.$arrow=this.$arrow||this.tip().find(".arrow")};var d=a.fn.popover;a.fn.popover=b,a.fn.popover.Constructor=c,a.fn.popover.noConflict=function(){return a.fn.popover=d,this}}(jQuery),+function(a){"use strict";function b(c,d){this.$body=a(document.body),this.$scrollElement=a(a(c).is(document.body)?window:c),this.options=a.extend({},b.DEFAULTS,d),this.selector=(this.options.target||"")+" .nav li > a",this.offsets=[],this.targets=[],this.activeTarget=null,this.scrollHeight=0,this.$scrollElement.on("scroll.bs.scrollspy",a.proxy(this.process,this)),this.refresh(),this.process()}function c(c){return this.each(function(){var d=a(this),e=d.data("bs.scrollspy"),f="object"==typeof c&&c;e||d.data("bs.scrollspy",e=new b(this,f)),"string"==typeof c&&e[c]()})}b.VERSION="3.3.5",b.DEFAULTS={offset:10},b.prototype.getScrollHeight=function(){return this.$scrollElement[0].scrollHeight||Math.max(this.$body[0].scrollHeight,document.documentElement.scrollHeight)},b.prototype.refresh=function(){var b=this,c="offset",d=0;this.offsets=[],this.targets=[],this.scrollHeight=this.getScrollHeight(),a.isWindow(this.$scrollElement[0])||(c="position",d=this.$scrollElement.scrollTop()),this.$body.find(this.selector).map(function(){var b=a(this),e=b.data("target")||b.attr("href"),f=/^#./.test(e)&&a(e);return f&&f.length&&f.is(":visible")&&[[f[c]().top+d,e]]||null}).sort(function(a,b){return a[0]-b[0]}).each(function(){b.offsets.push(this[0]),b.targets.push(this[1])})},b.prototype.process=function(){var a,b=this.$scrollElement.scrollTop()+this.options.offset,c=this.getScrollHeight(),d=this.options.offset+c-this.$scrollElement.height(),e=this.offsets,f=this.targets,g=this.activeTarget;if(this.scrollHeight!=c&&this.refresh(),b>=d)return g!=(a=f[f.length-1])&&this.activate(a);if(g&&b<e[0])return this.activeTarget=null,this.clear();for(a=e.length;a--;)g!=f[a]&&b>=e[a]&&(void 0===e[a+1]||b<e[a+1])&&this.activate(f[a])},b.prototype.activate=function(b){this.activeTarget=b,this.clear();var c=this.selector+'[data-target="'+b+'"],'+this.selector+'[href="'+b+'"]',d=a(c).parents("li").addClass("active");d.parent(".dropdown-menu").length&&(d=d.closest("li.dropdown").addClass("active")),
-d.trigger("activate.bs.scrollspy")},b.prototype.clear=function(){a(this.selector).parentsUntil(this.options.target,".active").removeClass("active")};var d=a.fn.scrollspy;a.fn.scrollspy=c,a.fn.scrollspy.Constructor=b,a.fn.scrollspy.noConflict=function(){return a.fn.scrollspy=d,this},a(window).on("load.bs.scrollspy.data-api",function(){a('[data-spy="scroll"]').each(function(){var b=a(this);c.call(b,b.data())})})}(jQuery),+function(a){"use strict";function b(b){return this.each(function(){var d=a(this),e=d.data("bs.tab");e||d.data("bs.tab",e=new c(this)),"string"==typeof b&&e[b]()})}var c=function(b){this.element=a(b)};c.VERSION="3.3.5",c.TRANSITION_DURATION=150,c.prototype.show=function(){var b=this.element,c=b.closest("ul:not(.dropdown-menu)"),d=b.data("target");if(d||(d=b.attr("href"),d=d&&d.replace(/.*(?=#[^\s]*$)/,"")),!b.parent("li").hasClass("active")){var e=c.find(".active:last a"),f=a.Event("hide.bs.tab",{relatedTarget:b[0]}),g=a.Event("show.bs.tab",{relatedTarget:e[0]});if(e.trigger(f),b.trigger(g),!g.isDefaultPrevented()&&!f.isDefaultPrevented()){var h=a(d);this.activate(b.closest("li"),c),this.activate(h,h.parent(),function(){e.trigger({type:"hidden.bs.tab",relatedTarget:b[0]}),b.trigger({type:"shown.bs.tab",relatedTarget:e[0]})})}}},c.prototype.activate=function(b,d,e){function f(){g.removeClass("active").find("> .dropdown-menu > .active").removeClass("active").end().find('[data-toggle="tab"]').attr("aria-expanded",!1),b.addClass("active").find('[data-toggle="tab"]').attr("aria-expanded",!0),h?(b[0].offsetWidth,b.addClass("in")):b.removeClass("fade"),b.parent(".dropdown-menu").length&&b.closest("li.dropdown").addClass("active").end().find('[data-toggle="tab"]').attr("aria-expanded",!0),e&&e()}var g=d.find("> .active"),h=e&&a.support.transition&&(g.length&&g.hasClass("fade")||!!d.find("> .fade").length);g.length&&h?g.one("bsTransitionEnd",f).emulateTransitionEnd(c.TRANSITION_DURATION):f(),g.removeClass("in")};var d=a.fn.tab;a.fn.tab=b,a.fn.tab.Constructor=c,a.fn.tab.noConflict=function(){return a.fn.tab=d,this};var e=function(c){c.preventDefault(),b.call(a(this),"show")};a(document).on("click.bs.tab.data-api",'[data-toggle="tab"]',e).on("click.bs.tab.data-api",'[data-toggle="pill"]',e)}(jQuery),+function(a){"use strict";function b(b){return this.each(function(){var d=a(this),e=d.data("bs.affix"),f="object"==typeof b&&b;e||d.data("bs.affix",e=new c(this,f)),"string"==typeof b&&e[b]()})}var c=function(b,d){this.options=a.extend({},c.DEFAULTS,d),this.$target=a(this.options.target).on("scroll.bs.affix.data-api",a.proxy(this.checkPosition,this)).on("click.bs.affix.data-api",a.proxy(this.checkPositionWithEventLoop,this)),this.$element=a(b),this.affixed=null,this.unpin=null,this.pinnedOffset=null,this.checkPosition()};c.VERSION="3.3.5",c.RESET="affix affix-top affix-bottom",c.DEFAULTS={offset:0,target:window},c.prototype.getState=function(a,b,c,d){var e=this.$target.scrollTop(),f=this.$element.offset(),g=this.$target.height();if(null!=c&&"top"==this.affixed)return c>e?"top":!1;if("bottom"==this.affixed)return null!=c?e+this.unpin<=f.top?!1:"bottom":a-d>=e+g?!1:"bottom";var h=null==this.affixed,i=h?e:f.top,j=h?g:b;return null!=c&&c>=e?"top":null!=d&&i+j>=a-d?"bottom":!1},c.prototype.getPinnedOffset=function(){if(this.pinnedOffset)return this.pinnedOffset;this.$element.removeClass(c.RESET).addClass("affix");var a=this.$target.scrollTop(),b=this.$element.offset();return this.pinnedOffset=b.top-a},c.prototype.checkPositionWithEventLoop=function(){setTimeout(a.proxy(this.checkPosition,this),1)},c.prototype.checkPosition=function(){if(this.$element.is(":visible")){var b=this.$element.height(),d=this.options.offset,e=d.top,f=d.bottom,g=Math.max(a(document).height(),a(document.body).height());"object"!=typeof d&&(f=e=d),"function"==typeof e&&(e=d.top(this.$element)),"function"==typeof f&&(f=d.bottom(this.$element));var h=this.getState(g,b,e,f);if(this.affixed!=h){null!=this.unpin&&this.$element.css("top","");var i="affix"+(h?"-"+h:""),j=a.Event(i+".bs.affix");if(this.$element.trigger(j),j.isDefaultPrevented())return;this.affixed=h,this.unpin="bottom"==h?this.getPinnedOffset():null,this.$element.removeClass(c.RESET).addClass(i).trigger(i.replace("affix","affixed")+".bs.affix")}"bottom"==h&&this.$element.offset({top:g-b-f})}};var d=a.fn.affix;a.fn.affix=b,a.fn.affix.Constructor=c,a.fn.affix.noConflict=function(){return a.fn.affix=d,this},a(window).on("load",function(){a('[data-spy="affix"]').each(function(){var c=a(this),d=c.data();d.offset=d.offset||{},null!=d.offsetBottom&&(d.offset.bottom=d.offsetBottom),null!=d.offsetTop&&(d.offset.top=d.offsetTop),b.call(c,d)})})}(jQuery);</script>
-<script>/**
-* @preserve HTML5 Shiv 3.7.2 | @afarkas @jdalton @jon_neal @rem | MIT/GPL2 Licensed
-*/
-// Only run this code in IE 8
-if (!!window.navigator.userAgent.match("MSIE 8")) {
-!function(a,b){function c(a,b){var c=a.createElement("p"),d=a.getElementsByTagName("head")[0]||a.documentElement;return c.innerHTML="x<style>"+b+"</style>",d.insertBefore(c.lastChild,d.firstChild)}function d(){var a=t.elements;return"string"==typeof a?a.split(" "):a}function e(a,b){var c=t.elements;"string"!=typeof c&&(c=c.join(" ")),"string"!=typeof a&&(a=a.join(" ")),t.elements=c+" "+a,j(b)}function f(a){var b=s[a[q]];return b||(b={},r++,a[q]=r,s[r]=b),b}function g(a,c,d){if(c||(c=b),l)return c.createElement(a);d||(d=f(c));var e;return e=d.cache[a]?d.cache[a].cloneNode():p.test(a)?(d.cache[a]=d.createElem(a)).cloneNode():d.createElem(a),!e.canHaveChildren||o.test(a)||e.tagUrn?e:d.frag.appendChild(e)}function h(a,c){if(a||(a=b),l)return a.createDocumentFragment();c=c||f(a);for(var e=c.frag.cloneNode(),g=0,h=d(),i=h.length;i>g;g++)e.createElement(h[g]);return e}function i(a,b){b.cache||(b.cache={},b.createElem=a.createElement,b.createFrag=a.createDocumentFragment,b.frag=b.createFrag()),a.createElement=function(c){return t.shivMethods?g(c,a,b):b.createElem(c)},a.createDocumentFragment=Function("h,f","return function(){var n=f.cloneNode(),c=n.createElement;h.shivMethods&&("+d().join().replace(/[\w\-:]+/g,function(a){return b.createElem(a),b.frag.createElement(a),'c("'+a+'")'})+");return n}")(t,b.frag)}function j(a){a||(a=b);var d=f(a);return!t.shivCSS||k||d.hasCSS||(d.hasCSS=!!c(a,"article,aside,dialog,figcaption,figure,footer,header,hgroup,main,nav,section{display:block}mark{background:#FF0;color:#000}template{display:none}")),l||i(a,d),a}var k,l,m="3.7.2",n=a.html5||{},o=/^<|^(?:button|map|select|textarea|object|iframe|option|optgroup)$/i,p=/^(?:a|b|code|div|fieldset|h1|h2|h3|h4|h5|h6|i|label|li|ol|p|q|span|strong|style|table|tbody|td|th|tr|ul)$/i,q="_html5shiv",r=0,s={};!function(){try{var a=b.createElement("a");a.innerHTML="<xyz></xyz>",k="hidden"in a,l=1==a.childNodes.length||function(){b.createElement("a");var a=b.createDocumentFragment();return"undefined"==typeof a.cloneNode||"undefined"==typeof a.createDocumentFragment||"undefined"==typeof a.createElement}()}catch(c){k=!0,l=!0}}();var t={elements:n.elements||"abbr article aside audio bdi canvas data datalist details dialog figcaption figure footer header hgroup main mark meter nav output picture progress section summary template time video",version:m,shivCSS:n.shivCSS!==!1,supportsUnknownElements:l,shivMethods:n.shivMethods!==!1,type:"default",shivDocument:j,createElement:g,createDocumentFragment:h,addElements:e};a.html5=t,j(b)}(this,document);
-};
-</script>
-<script>/*! Respond.js v1.4.2: min/max-width media query polyfill * Copyright 2013 Scott Jehl
- * Licensed under https://github.com/scottjehl/Respond/blob/master/LICENSE-MIT
- *  */
-
-// Only run this code in IE 8
-if (!!window.navigator.userAgent.match("MSIE 8")) {
-!function(a){"use strict";a.matchMedia=a.matchMedia||function(a){var b,c=a.documentElement,d=c.firstElementChild||c.firstChild,e=a.createElement("body"),f=a.createElement("div");return f.id="mq-test-1",f.style.cssText="position:absolute;top:-100em",e.style.background="none",e.appendChild(f),function(a){return f.innerHTML='&shy;<style media="'+a+'"> #mq-test-1 { width: 42px; }</style>',c.insertBefore(e,d),b=42===f.offsetWidth,c.removeChild(e),{matches:b,media:a}}}(a.document)}(this),function(a){"use strict";function b(){u(!0)}var c={};a.respond=c,c.update=function(){};var d=[],e=function(){var b=!1;try{b=new a.XMLHttpRequest}catch(c){b=new a.ActiveXObject("Microsoft.XMLHTTP")}return function(){return b}}(),f=function(a,b){var c=e();c&&(c.open("GET",a,!0),c.onreadystatechange=function(){4!==c.readyState||200!==c.status&&304!==c.status||b(c.responseText)},4!==c.readyState&&c.send(null))};if(c.ajax=f,c.queue=d,c.regex={media:/@media[^\{]+\{([^\{\}]*\{[^\}\{]*\})+/gi,keyframes:/@(?:\-(?:o|moz|webkit)\-)?keyframes[^\{]+\{(?:[^\{\}]*\{[^\}\{]*\})+[^\}]*\}/gi,urls:/(url\()['"]?([^\/\)'"][^:\)'"]+)['"]?(\))/g,findStyles:/@media *([^\{]+)\{([\S\s]+?)$/,only:/(only\s+)?([a-zA-Z]+)\s?/,minw:/\([\s]*min\-width\s*:[\s]*([\s]*[0-9\.]+)(px|em)[\s]*\)/,maxw:/\([\s]*max\-width\s*:[\s]*([\s]*[0-9\.]+)(px|em)[\s]*\)/},c.mediaQueriesSupported=a.matchMedia&&null!==a.matchMedia("only all")&&a.matchMedia("only all").matches,!c.mediaQueriesSupported){var g,h,i,j=a.document,k=j.documentElement,l=[],m=[],n=[],o={},p=30,q=j.getElementsByTagName("head")[0]||k,r=j.getElementsByTagName("base")[0],s=q.getElementsByTagName("link"),t=function(){var a,b=j.createElement("div"),c=j.body,d=k.style.fontSize,e=c&&c.style.fontSize,f=!1;return b.style.cssText="position:absolute;font-size:1em;width:1em",c||(c=f=j.createElement("body"),c.style.background="none"),k.style.fontSize="100%",c.style.fontSize="100%",c.appendChild(b),f&&k.insertBefore(c,k.firstChild),a=b.offsetWidth,f?k.removeChild(c):c.removeChild(b),k.style.fontSize=d,e&&(c.style.fontSize=e),a=i=parseFloat(a)},u=function(b){var c="clientWidth",d=k[c],e="CSS1Compat"===j.compatMode&&d||j.body[c]||d,f={},o=s[s.length-1],r=(new Date).getTime();if(b&&g&&p>r-g)return a.clearTimeout(h),h=a.setTimeout(u,p),void 0;g=r;for(var v in l)if(l.hasOwnProperty(v)){var w=l[v],x=w.minw,y=w.maxw,z=null===x,A=null===y,B="em";x&&(x=parseFloat(x)*(x.indexOf(B)>-1?i||t():1)),y&&(y=parseFloat(y)*(y.indexOf(B)>-1?i||t():1)),w.hasquery&&(z&&A||!(z||e>=x)||!(A||y>=e))||(f[w.media]||(f[w.media]=[]),f[w.media].push(m[w.rules]))}for(var C in n)n.hasOwnProperty(C)&&n[C]&&n[C].parentNode===q&&q.removeChild(n[C]);n.length=0;for(var D in f)if(f.hasOwnProperty(D)){var E=j.createElement("style"),F=f[D].join("\n");E.type="text/css",E.media=D,q.insertBefore(E,o.nextSibling),E.styleSheet?E.styleSheet.cssText=F:E.appendChild(j.createTextNode(F)),n.push(E)}},v=function(a,b,d){var e=a.replace(c.regex.keyframes,"").match(c.regex.media),f=e&&e.length||0;b=b.substring(0,b.lastIndexOf("/"));var g=function(a){return a.replace(c.regex.urls,"$1"+b+"$2$3")},h=!f&&d;b.length&&(b+="/"),h&&(f=1);for(var i=0;f>i;i++){var j,k,n,o;h?(j=d,m.push(g(a))):(j=e[i].match(c.regex.findStyles)&&RegExp.$1,m.push(RegExp.$2&&g(RegExp.$2))),n=j.split(","),o=n.length;for(var p=0;o>p;p++)k=n[p],l.push({media:k.split("(")[0].match(c.regex.only)&&RegExp.$2||"all",rules:m.length-1,hasquery:k.indexOf("(")>-1,minw:k.match(c.regex.minw)&&parseFloat(RegExp.$1)+(RegExp.$2||""),maxw:k.match(c.regex.maxw)&&parseFloat(RegExp.$1)+(RegExp.$2||"")})}u()},w=function(){if(d.length){var b=d.shift();f(b.href,function(c){v(c,b.href,b.media),o[b.href]=!0,a.setTimeout(function(){w()},0)})}},x=function(){for(var b=0;b<s.length;b++){var c=s[b],e=c.href,f=c.media,g=c.rel&&"stylesheet"===c.rel.toLowerCase();e&&g&&!o[e]&&(c.styleSheet&&c.styleSheet.rawCssText?(v(c.styleSheet.rawCssText,e,f),o[e]=!0):(!/^([a-zA-Z:]*\/\/)/.test(e)&&!r||e.replace(RegExp.$1,"").split("/")[0]===a.location.host)&&("//"===e.substring(0,2)&&(e=a.location.protocol+e),d.push({href:e,media:f})))}w()};x(),c.update=x,c.getEmValue=t,a.addEventListener?a.addEventListener("resize",b,!1):a.attachEvent&&a.attachEvent("onresize",b)}}(this);
-};
-</script>
-<script>
-
-/**
- * jQuery Plugin: Sticky Tabs
- *
- * @author Aidan Lister <aidan@php.net>
- * adapted by Ruben Arslan to activate parent tabs too
- * http://www.aidanlister.com/2014/03/persisting-the-tab-state-in-bootstrap/
- */
-(function($) {
-  "use strict";
-  $.fn.rmarkdownStickyTabs = function() {
-    var context = this;
-    // Show the tab corresponding with the hash in the URL, or the first tab
-    var showStuffFromHash = function() {
-      var hash = window.location.hash;
-      var selector = hash ? 'a[href="' + hash + '"]' : 'li.active > a';
-      var $selector = $(selector, context);
-      if($selector.data('toggle') === "tab") {
-        $selector.tab('show');
-        // walk up the ancestors of this element, show any hidden tabs
-        $selector.parents('.section.tabset').each(function(i, elm) {
-          var link = $('a[href="#' + $(elm).attr('id') + '"]');
-          if(link.data('toggle') === "tab") {
-            link.tab("show");
-          }
-        });
-      }
-    };
-
-
-    // Set the correct tab when the page loads
-    showStuffFromHash(context);
-
-    // Set the correct tab when a user uses their back/forward button
-    $(window).on('hashchange', function() {
-      showStuffFromHash(context);
-    });
-
-    // Change the URL when tabs are clicked
-    $('a', context).on('click', function(e) {
-      history.pushState(null, null, this.href);
-      showStuffFromHash(context);
-    });
-
-    return this;
-  };
-}(jQuery));
-
-window.buildTabsets = function(tocID) {
-
-  // build a tabset from a section div with the .tabset class
-  function buildTabset(tabset) {
-
-    // check for fade and pills options
-    var fade = tabset.hasClass("tabset-fade");
-    var pills = tabset.hasClass("tabset-pills");
-    var navClass = pills ? "nav-pills" : "nav-tabs";
-
-    // determine the heading level of the tabset and tabs
-    var match = tabset.attr('class').match(/level(\d) /);
-    if (match === null)
-      return;
-    var tabsetLevel = Number(match[1]);
-    var tabLevel = tabsetLevel + 1;
-
-    // find all subheadings immediately below
-    var tabs = tabset.find("div.section.level" + tabLevel);
-    if (!tabs.length)
-      return;
-
-    // create tablist and tab-content elements
-    var tabList = $('<ul class="nav ' + navClass + '" role="tablist"></ul>');
-    $(tabs[0]).before(tabList);
-    var tabContent = $('<div class="tab-content"></div>');
-    $(tabs[0]).before(tabContent);
-
-    // build the tabset
-    var activeTab = 0;
-    tabs.each(function(i) {
-
-      // get the tab div
-      var tab = $(tabs[i]);
-
-      // get the id then sanitize it for use with bootstrap tabs
-      var id = tab.attr('id');
-
-      // see if this is marked as the active tab
-      if (tab.hasClass('active'))
-        activeTab = i;
-
-      // remove any table of contents entries associated with
-      // this ID (since we'll be removing the heading element)
-      $("div#" + tocID + " li a[href='#" + id + "']").parent().remove();
-
-      // sanitize the id for use with bootstrap tabs
-      id = id.replace(/[.\/?&!#<>]/g, '').replace(/\s/g, '_');
-      tab.attr('id', id);
-
-      // get the heading element within it, grab it's text, then remove it
-      var heading = tab.find('h' + tabLevel + ':first');
-      var headingText = heading.html();
-      heading.remove();
-
-      // build and append the tab list item
-      var a = $('<a role="tab" data-toggle="tab">' + headingText + '</a>');
-      a.attr('href', '#' + id);
-      a.attr('aria-controls', id);
-      var li = $('<li role="presentation"></li>');
-      li.append(a);
-      tabList.append(li);
-
-      // set it's attributes
-      tab.attr('role', 'tabpanel');
-      tab.addClass('tab-pane');
-      tab.addClass('tabbed-pane');
-      if (fade)
-        tab.addClass('fade');
-
-      // move it into the tab content div
-      tab.detach().appendTo(tabContent);
-    });
-
-    // set active tab
-    $(tabList.children('li')[activeTab]).addClass('active');
-    var active = $(tabContent.children('div.section')[activeTab]);
-    active.addClass('active');
-    if (fade)
-      active.addClass('in');
-
-    if (tabset.hasClass("tabset-sticky"))
-      tabset.rmarkdownStickyTabs();
-  }
-
-  // convert section divs with the .tabset class to tabsets
-  var tabsets = $("div.section.tabset");
-  tabsets.each(function(i) {
-    buildTabset($(tabsets[i]));
-  });
-};
-
-</script>
-<style type="text/css">.hljs-literal {
-color: #990073;
-}
-.hljs-number {
-color: #099;
-}
-.hljs-comment {
-color: #998;
-font-style: italic;
-}
-.hljs-keyword {
-color: #900;
-font-weight: bold;
-}
-.hljs-string {
-color: #d14;
-}
-</style>
-<script src="data:application/javascript;base64,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"></script>
+<link href="data:text/css;charset=utf-8,html%7Bfont%2Dfamily%3Asans%2Dserif%3B%2Dwebkit%2Dtext%2Dsize%2Dadjust%3A100%25%3B%2Dms%2Dtext%2Dsize%2Dadjust%3A100%25%7Dbody%7Bmargin%3A0%7Darticle%2Caside%2Cdetails%2Cfigcaption%2Cfigure%2Cfooter%2Cheader%2Chgroup%2Cmain%2Cmenu%2Cnav%2Csection%2Csummary%7Bdisplay%3Ablock%7Daudio%2Ccanvas%2Cprogress%2Cvideo%7Bdisplay%3Ainline%2Dblock%3Bvertical%2Dalign%3Abaseline%7Daudio%3Anot%28%5Bcontrols%5D%29%7Bdisplay%3Anone%3Bheight%3A0%7D%5Bhidden%5D%2Ctemplate%7Bdisplay%3Anone%7Da%7Bbackground%2Dcolor%3Atransparent%7Da%3Aactive%2Ca%3Ahover%7Boutline%3A0%7Dabbr%5Btitle%5D%7Bborder%2Dbottom%3A1px%20dotted%7Db%2Cstrong%7Bfont%2Dweight%3A700%7Ddfn%7Bfont%2Dstyle%3Aitalic%7Dh1%7Bmargin%3A%2E67em%200%3Bfont%2Dsize%3A2em%7Dmark%7Bcolor%3A%23000%3Bbackground%3A%23ff0%7Dsmall%7Bfont%2Dsize%3A80%25%7Dsub%2Csup%7Bposition%3Arelative%3Bfont%2Dsize%3A75%25%3Bline%2Dheight%3A0%3Bvertical%2Dalign%3Abaseline%7Dsup%7Btop%3A%2D%2E5em%7Dsub%7Bbottom%3A%2D%2E25em%7Dimg%7Bborder%3A0%7Dsvg%3Anot%28%3Aroot%29%7Boverflow%3Ahidden%7Dfigure%7Bmargin%3A1em%2040px%7Dhr%7Bheight%3A0%3B%2Dwebkit%2Dbox%2Dsizing%3Acontent%2Dbox%3B%2Dmoz%2Dbox%2Dsizing%3Acontent%2Dbox%3Bbox%2Dsizing%3Acontent%2Dbox%7Dpre%7Boverflow%3Aauto%7Dcode%2Ckbd%2Cpre%2Csamp%7Bfont%2Dfamily%3Amonospace%2Cmonospace%3Bfont%2Dsize%3A1em%7Dbutton%2Cinput%2Coptgroup%2Cselect%2Ctextarea%7Bmargin%3A0%3Bfont%3Ainherit%3Bcolor%3Ainherit%7Dbutton%7Boverflow%3Avisible%7Dbutton%2Cselect%7Btext%2Dtransform%3Anone%7Dbutton%2Chtml%20input%5Btype%3Dbutton%5D%2Cinput%5Btype%3Dreset%5D%2Cinput%5Btype%3Dsubmit%5D%7B%2Dwebkit%2Dappearance%3Abutton%3Bcursor%3Apointer%7Dbutton%5Bdisabled%5D%2Chtml%20input%5Bdisabled%5D%7Bcursor%3Adefault%7Dbutton%3A%3A%2Dmoz%2Dfocus%2Dinner%2Cinput%3A%3A%2Dmoz%2Dfocus%2Dinner%7Bpadding%3A0%3Bborder%3A0%7Dinput%7Bline%2Dheight%3Anormal%7Dinput%5Btype%3Dcheckbox%5D%2Cinput%5Btype%3Dradio%5D%7B%2Dwebkit%2Dbox%2Dsizing%3Aborder%2Dbox%3B%2Dmoz%2Dbox%2Dsizing%3Aborder%2Dbox%3Bbox%2Dsizing%3Aborder%2Dbox%3Bpadding%3A0%7Dinput%5Btype%3Dnumber%5D%3A%3A%2Dwebkit%2Dinner%2Dspin%2Dbutton%2Cinput%5Btype%3Dnumber%5D%3A%3A%2Dwebkit%2Douter%2Dspin%2Dbutton%7Bheight%3Aauto%7Dinput%5Btype%3Dsearch%5D%7B%2Dwebkit%2Dbox%2Dsizing%3Acontent%2Dbox%3B%2Dmoz%2Dbox%2Dsizing%3Acontent%2Dbox%3Bbox%2Dsizing%3Acontent%2Dbox%3B%2Dwebkit%2Dappearance%3Atextfield%7Dinput%5Btype%3Dsearch%5D%3A%3A%2Dwebkit%2Dsearch%2Dcancel%2Dbutton%2Cinput%5Btype%3Dsearch%5D%3A%3A%2Dwebkit%2Dsearch%2Ddecoration%7B%2Dwebkit%2Dappearance%3Anone%7Dfieldset%7Bpadding%3A%2E35em%20%2E625em%20%2E75em%3Bmargin%3A0%202px%3Bborder%3A1px%20solid%20silver%7Dlegend%7Bpadding%3A0%3Bborder%3A0%7Dtextarea%7Boverflow%3Aauto%7Doptgroup%7Bfont%2Dweight%3A700%7Dtable%7Bborder%2Dspacing%3A0%3Bborder%2Dcollapse%3Acollapse%7Dtd%2Cth%7Bpadding%3A0%7D%40media%20print%7B%2A%2C%3Aafter%2C%3Abefore%7Bcolor%3A%23000%21important%3Btext%2Dshadow%3Anone%21important%3Bbackground%3A0%200%21important%3B%2Dwebkit%2Dbox%2Dshadow%3Anone%21important%3Bbox%2Dshadow%3Anone%21important%7Da%2Ca%3Avisited%7Btext%2Ddecoration%3Aunderline%7Da%5Bhref%5D%3Aafter%7Bcontent%3A%22%20%28%22%20attr%28href%29%20%22%29%22%7Dabbr%5Btitle%5D%3Aafter%7Bcontent%3A%22%20%28%22%20attr%28title%29%20%22%29%22%7Da%5Bhref%5E%3D%22javascript%3A%22%5D%3Aafter%2Ca%5Bhref%5E%3D%22%23%22%5D%3Aafter%7Bcontent%3A%22%22%7Dblockquote%2Cpre%7Bborder%3A1px%20solid%20%23999%3Bpage%2Dbreak%2Dinside%3Aavoid%7Dthead%7Bdisplay%3Atable%2Dheader%2Dgroup%7Dimg%2Ctr%7Bpage%2Dbreak%2Dinside%3Aavoid%7Dimg%7Bmax%2Dwidth%3A100%25%21important%7Dh2%2Ch3%2Cp%7Borphans%3A3%3Bwidows%3A3%7Dh2%2Ch3%7Bpage%2Dbreak%2Dafter%3Aavoid%7D%2Enavbar%7Bdisplay%3Anone%7D%2Ebtn%3E%2Ecaret%2C%2Edropup%3E%2Ebtn%3E%2Ecaret%7Bborder%2Dtop%2Dcolor%3A%23000%21important%7D%2Elabel%7Bborder%3A1px%20solid%20%23000%7D%2Etable%7Bborder%2Dcollapse%3Acollapse%21important%7D%2Etable%20td%2C%2Etable%20th%7Bbackground%2Dcolor%3A%23fff%21important%7D%2Etable%2Dbordered%20td%2C%2Etable%2Dbordered%20th%7Bborder%3A1px%20solid%20%23ddd%21important%7D%7D%40font%2Dface%7Bfont%2Dfamily%3A%27Glyphicons%20Halflings%27%3Bsrc%3Aurl%28data%3Aapplication%2Fvnd%2Ems%2Dfontobject%3Bbase64%2Cn04AAEFNAAACAAIABAAAAAAABQAAAAAAAAABAJABAAAEAExQAAAAAAAAAAIAAAAAAAAAAAEAAAAAAAAAJxJ%2FLAAAAAAAAAAAAAAAAAAAAAAAACgARwBMAFkAUABIAEkAQwBPAE4AUwAgAEgAYQBsAGYAbABpAG4AZwBzAAAADgBSAGUAZwB1AGwAYQByAAAAeABWAGUAcgBzAGkAbwBuACAAMQAuADAAMAA5ADsAUABTACAAMAAwADEALgAwADAAOQA7AGgAbwB0AGMAbwBuAHYAIAAxAC4AMAAuADcAMAA7AG0AYQBrAGUAbwB0AGYALgBsAGkAYgAyAC4ANQAuADUAOAAzADIAOQAAADgARwBMAFkAUABIAEkAQwBPAE4AUwAgAEgAYQBsAGYAbABpAG4AZwBzACAAUgBlAGcAdQBsAGEAcgAAAAAAQlNHUAAAAAAAAAAAAAAAAAAAAAADAKncAE0TAE0ZAEbuFM3pjM%2FSEdmjKHUbyow8ATBE40IvWA3vTu8LiABDQ%2BpexwUMcm1SMnNryctQSiI1K5ZnbOlXKmnVV5YvRe6RnNMFNCOs1KNVpn6yZhCJkRtVRNzEufeIq7HgSrcx4S8h%2Fv4vnrrKc6oCNxmSk2uKlZQHBii6iKFoH0746ThvkO1kJHlxjrkxs%2BLWORaDQBEtiYJIR5IB9Bi1UyL4Rmr0BNigNkMzlKQmnofBHviqVzUxwdMb3NdCn69hy%2BpRYVKGVS%2F1tnsqv4LL7wCCPZZAZPT4aCShHjHJVNuXbmMrY5LeQaGnvAkXlVrJgKRAUdFjrWEah9XebPeQMj7KS7DIBAFt8ycgC5PLGUOHSE3ErGZCiViNLL5ZARfywnCoZaKQCu6NuFX42AEeKtKUGnr%2FCm2Cy8tpFhBPMW5Fxi4Qm4TkDWh4IWFDClhU2hRWosUWqcKLlgyXB%2BlSHaWaHiWlBAR8SeSgSPCQxdVQgzUixWKSTrIQEbU94viDctkvX%2BVSjJuUmV8L4CXShI11esnp0pjWNZIyxKHS4wVQ2ime1P4RnhvGw0aDN1OLAXGERsB7buFpFGGBAre4QEQR0HOIO5oYH305G%2BKspT%2FFupEGGafCCwxSe6ZUa%2B073rXHnNdVXE6eWvibUS27XtRzkH838mYLMBmYysZTM0EM3A1fbpCBYFccN1B%2FEnCYu%2FTgCGmr7bMh8GfYL%2BBfcLvB0gRagC09w9elfldaIy%2FhNCBLRgBgtCC7jAF63wLSMAfbfAlEggYU0bUA7ACCJmTDpEmJtI78w4%2FBO7dN7JR7J7ZvbYaUbaILSQsRBiF3HGk5fEg6p9unwLvn98r%2BvnsV%2B372uf1xBLq4qU%2F45fTuqaAP%2BpssmCCCTF0mhEow8ZXZOS8D7Q85JsxZ%2BAzok7B7O%2Ff6J8AzYBySZQB%2FQHYUSA%2BEeQhEWiS6AIQzgcsDiER4MjgMBAWDV4AgQ3g1eBgIdweCQmCjJEMkJ%2BPKRWyFHHmg1Wi%2F6xzUgA0LREoKJChwnQa9B%2B5RQZRB3IlBlkAnxyQNaANwHMowzlYSMCBgnbpzvqpl0iTJNCQidDI9ZrSYNIRBhHtUa5YHMHxyGEik9hDE0AKj72AbTCaxtHPUaKZdAZSnQTyjGqGLsmBStCejApUhg4uBMU6mATujEl%2BKdDPbI6Ag4vLr%2BhjY6lbjBeoLKnZl0UZgRX8gTySOeynZVz1wOq7e1hFGYIq%2BMhrGxDLak0PrwYzSXtcuyhXEhwOYofiW%2BEcI%2Fjw8P6IY6ed%2BetAbuqKp5QIapT77LnAe505lMuqL79a0ut4rWexzFttsOsLDy7zvtQzcq3U1qabe7tB0wHWVXji%2BzDbo8x8HyIRUbXnwUcklFv51fvTymiV%2BMXLSmGH9d9%2BaXpD5X6lao41anWGig7IwIdnoBY2ht%2FpO9mClLo4NdXHAsefqWUKlXJkbqPOFhMoR4aiA1BXqhRNbB2Xwi%2B7u%2FjpAoOpKJ0UX24EsrzMfHXViakCNcKjBxuQX8BO0ZqjJ3xXzf%2B61t2VXOSgJ8xu65QKgtN6FibPmPYsXbJRHHqbgATcSZxBqGiDiU4NNNsYBsKD0MIP%2FOfKnlk%2FLkaid%2FO2NbKeuQrwOB2Gq3YHyr6ALgzym5wIBnsdC1ZkoBFZSQXChZvlesPqvK2c5oHHT3Q65jYpNxnQcGF0EHbvYqoFw60WNlXIHQF2HQB7zD6lWjZ9rVqUKBXUT6hrkZOle0RFYII0V5ZYGl1JAP0Ud1fZZMvSomBzJ710j4Me8mjQDwEre5Uv2wQfk1ifDwb5ksuJQQ3xt423lbuQjvoIQByQrNDh1JxGFkOdlJvu%2FgFtuW0wR4cgd%2BZKesSV7QkNE2kw6AV4hoIuC02LGmTomyf8PiO6CZzOTLTPQ%2BHW06H%2Btx%2BbQ8LmDYg1pTFrp2oJXgkZTyeRJZM0C8aE2LpFrNVDuhARsN543%2FFV6klQ6Tv1OoZGXLv0igKrl%2FCmJxRmX7JJbJ998VSIPQRyDBICzl4JJlYHbdql30NvYcOuZ7a10uWRrgoieOdgIm4rlq6vNOQBuqESLbXG5lzdJGHw2m0sDYmODXbYGTfSTGRKpssTO95fothJCjUGQgEL4yKoGAF%2F0SrpUDNn8CBgBcSDQByAeNkCXp4S4Ro2Xh4OeaGRgR66PVOsU8bc6TR5%2FxTcn4IVMLOkXSWiXxkZQCbvKfmoAvQaKjO3EDKwkwqHChCDEM5loQRPd5ACBki1TjF772oaQhQbQ5C0lcWXPFOzrfsDGUXGrpxasbG4iab6eByaQkQfm0VFlP0ZsDkvvqCL6QXMUwCjdMx1ZOyKhTJ7a1GWAdOUcJ8RSejxNVyGs31OKMyRyBVoZFjqIkmKlLQ5eHMeEL4MkUf23cQ%2F1SgRCJ1dk4UdBT7OoyuNgLs0oCd8RnrEIb6QdMxT2QjD4zMrJkfgx5aDMcA4orsTtKCqWb%2FVeyceqa5OGSmB28YwH4rFbkQaLoUN8OQQYnD3w2eXpI4ScQfbCUZiJ4yMOIKLyyTc7BQ4uXUw6Ee6%2FxM%2B4Y67ngNBknxIPwuppgIhFcwJyr6EIj%2BLzNj%2FmfR2vhhRlx0BILZoAYruF0caWQ7YxO66UmeguDREAFHYuC7HJviRgVO6ruJH59h%2FC%2FPkgSle8xNzZJULLWq9JMDTE2fjGE146a1Us6PZDGYle6ldWRqn%2FpdpgHKNGrGIdkRK%2BKPETT9nKT6kLyDI8xd9A1FgWmXWRAIHwZ37WyZHOVyCadJEmMVz0MadMjDrPho%2BEIochkVC2xgGiwwsQ6DMv2P7UXqT4x7CdcYGId2BJQQa85EQKmCmwcRejQ9Bm4oATENFPkxPXILHpMPUyWTI5rjNOsIlmEeMbcOCEqInpXACYQ9DDxmFo9vcmsDblcMtg4tqBerNngkIKaFJmrQAPnq1dEzsMXcwjcHdfdCibcAxxA%2Bq%2Fj9m3LM%2FO7WJka4tSidVCjsvo2lQ%2F2ewyoYyXwAYyr2PlRoR5MpgVmSUIrM3PQxXPbgjBOaDQFIyFMJvx3Pc5RSYj12ySVF9fwFPQu2e2KWVoL9q3Ayv3IzpGHUdvdPdrNUdicjsTQ2ISy7QU3DrEytIjvbzJnAkmANXjAFERA0MUoPF3%2F5KFmW14bBNOhwircYgMqoDpUMcDtCmBE82QM2YtdjVLB4kBuKho%2FbcwQdeboqfQartuU3CsCf%2BcXkgYAqp%2F0Ee3RorAZt0AvvOCSI4JICIlGlsV0bsSid%2FNIEALAAzb6HAgyWHBps6xAOwkJIGcB82CxRQq4sJf3FzA70A%2BTRqcqjEMETCoez3mkPcpnoALs0ugJY8kQwrC%2BJE5ik3w9rzrvDRjAQnqgEVvdGrNwlanR0SOKWzxOJOvLJhcd8Cl4AshACUkv9czdMkJCVQSQhp6kp7StAlpVRpK0t0SW6LHeBJnE2QchB5Ccu8kxRghZXGIgZIiSj7gEKMJDClcnX6hgoqJMwiQDigIXg3ioFLCgDgjPtYHYpsF5EiA4kcnN18MZtOrY866dEQAb0FB34OGKHGZQjwW%2FWDHA60cYFaI%2FPjpzquUqdaYGcIq%2BmLez3WLFFCtNBN2QJcrlcoELgiPku5R5dSlJFaCEqEZle1AQzAKC%2B1SotMcBNyQUFuRHRF6OlimSBgjZeTBCwLyc6A%2BP%2FoFRchXTz5ADknYJHxzrJ5pGuIKRQISU6WyKTBBjD8WozmVYWIsto1AS5rxzKlvJu4E%2FvwOiKxRtCWsDM%2BeTHUrmwrCK5BIfMzGkD%2B0Fk5LzBs0jMYXktNDblB06LMNJ09U8pzSLmo14MS0OMjcdrZ31pyQqxJJpRImlSvfYAK8inkYU52QY2FPEVsjoWewpwhRp5yAuNpkqhdb7ku9Seefl2D0B8SMTFD90xi4CSOwwZy9IKkpMtI3FmFUg3%2FkFutpQGNc3pCR7gvC4sgwbupDu3DyEN%2BW6YGLNM21jpB49irxy9BSlHrVDlnihGKHwPrbVFtc%2Bh1rVQKZduxIyojccZIIcOCmhEnC7UkY68WXKQgLi2JCDQkQWJRQuk60hZp0D3rtCTINSeY9Ej2kIKYfGxwOs4j9qMM7fYZiipzgcf7TamnehqdhsiMiCawXnz4xAbyCkLAx5EGbo3Ax1u3dUIKnTxIaxwQTHehPl3V491H0%2BbC5zgpGz7Io%2BmjdhKlPJ01EeMpM7UsRJMi1nGjmJg35i6bQBAAxjO%2FENJubU2mg3ONySEoWklCwdABETcs7ck3jgiuU9pcKKpbgn%2B3YlzV1FzIkB6pmEDOSSyDfPPlQskznctFji0kpgZjW5RZe6x9kYT4KJcXg0bNiCyif%2BpZACCyRMmYsfiKmN9tSO65F0R2OO6ytlEhY5Sj6uRKfFxw0ijJaAx%2Fk3QgnAFSq27%2F2i4GEBA%2BUvTJKK%2F9eISNvG46Em5RZfjTYLdeD8kdXHyrwId%2FDQZUaMCY4gGbke2C8vfjgV%2FY9kkRQOJIn%2FxM9INZSpiBnqX0Q9GlQPpPKAyO5y%2BW5NMPSRdBCUlmuxl40ZfMCnf2Cp044uI9WLFtCi4YVxKjuRCOBWIb4XbIsGdbo4qtMQnNOQz4XDSui7W%2FN6l54qOynCqD3DpWQ%2BmpD7C40D8BZEWGJX3tlAaZBMj1yjvDYKwCJBa201u6nBKE5UE%2B7QSEhCwrXfbRZylAaAkplhBWX50dumrElePyNMRYUrC99UmcSSNgImhFhDI4BXjMtiqkgizUGCrZ8iwFxU6fQ8GEHCFdLewwxYWxgScAYMdMLmcZR6b7rZl95eQVDGVoUKcRMM1ixXQtXNkBETZkVVPg8LoSrdetHzkuM7DjZRHP02tCxA1fmkXKF3VzfN1pc1cv%2F8lbTIkkYpqKM9VOhp65ktYk%2BQ46myFWBapDfyWUCnsnI00QTBQmuFjMZTcd0V2NQ768Fhpby04k2IzNR1wKabuGJqYWwSly6ocMFGTeeI%2BejsWDYgEvr66QgqdcIbFYDNgsm0x9UHY6SCd5%2B7tpsLpKdvhahIDyYmEJQCqMqtCF6UlrE5GXRmbu%2Bvtm3BFSxI6ND6UxIE7GsGMgWqghXxSnaRJuGFveTcK5ZVSPJyjUxe1dKgI6kNF7EZhIZs8y8FVqwEfbM0Xk2ltORVDKZZM40SD3qQoQe0orJEKwPfZwm3YPqwixhUMOndis6MhbmfvLBKjC8sKKIZKbJk8L11oNkCQzCgvjhyyEiQSuJcgCQSG4Mocfgc0Hkwcjal1UNgP0CBPikYqBIk9tONv4kLtBswH07vUCjEaHiFGlLf8MgXKzSgjp2HolRRccAOh0ILHz9qlGgIFkwAnzHJRjWFhlA7ROwINyB5HFj59PRZHFor6voq7l23EPNRwdWhgawqbivLSjRA4htEYUFkjESu67icTg5S0aW1sOkCiIysfJ9UnIWevOOLGpepcBxy1wEhd2WI3AZg7sr9WBmHWyasxMcvY%2FiOmsLtHSWNUWEGk9hScMPShasUA1AcHOtRZlqMeQ0OzYS9vQvYUjOLrzP07BUAFikcJNMi7gIxEw4pL1G54TcmmmoAQ5s7TGWErJZ2Io4yQ0ljRYhL8H5e62oDtLF8aDpnIvZ5R3GWJyAugdiiJW9hQAVTsnCBHhwu7rkBlBX6r3b7ejEY0k5GGeyKv66v%2B6dg7mcJTrWHbtMywbedYqCQ0FPwoytmSWsL8WTtChZCKKzEF7vP6De4x2BJkkniMgSdWhbeBSLtJZR9CTHetK1xb34AYIJ37OegYIoPVbXgJ%2FqDQK%2BbfCtxQRVKQu77WzOoM6SGL7MaZwCGJVk46aImai9fmam%2BWpHG%2B0BtQPWUgZ7RIAlPq6lkECUhZQ2gqWkMYKcYMYaIc4gYCDFHYa2d1nzp3%2BJ1eCBay8IYZ0wQRKGAqvCuZ%2FUgbQPyllosq%2BXtfKIZOzmeJqRazpmmoP%2F76YfkjzV2NlXTDSBYB04SVlNQsFTbGPk1t%2FI4Jktu0XSgifO2ozFOiwd%2F0SssJDn0dn4xqk4GDTTKX73%2FwQyBLdqgJ%2BWx6AQaba3BA9CKEzjtQYIfAsiYamapq80LAamYjinlKXUkxdpIDk0puXUEYzSalfRibAeDAKpNiqQ0FTwoxuGYzRnisyTotdVTclis1LHRQCy%2FqqL8oUaQzWRxilq5Mi0IJGtMY02cGLD69vGjkj3p6pGePKI8bkBv5evq8SjjyU04vJR2cQXQwSJyoinDsUJHCQ50jrFTT7yRdbdYQMB3MYCb6uBzJ9ewhXYPAIZSXfeEQBZZ3GPN3Nbhh%2FwkvAJLXnQMdi5NYYZ5GHE400GS5rXkOZSQsdZgIbzRnF9ueLnsfQ47wHAsirITnTlkCcuWWIUhJSbpM3wWhXNHvt2xUsKKMpdBSbJnBMcihkoDqAd1Zml%2FR4yrzow1Q2A5G%2Bkzo%2FRhRxQS2lCSDRV8LlYLBOOoo1bF4jwJAwKMK1tWLHlu9i0j4Ig8qVm6wE1DxXwAwQwsaBWUg2pOOol2dHxyt6npwJEdLDDVYyRc2D0HbcbLUJQj8gPevQBUBOUHXPrsAPBERICpnYESeu2OHotpXQxRGlCCtLdIsu23MhZVEoJg8Qumj%2FUMMc34IBqTKLDTp76WzL%2FdMjCxK7MjhiGjeYAC%2Fkj%2FjY%2FRde7hpSM1xChrog6yZ7OWTuD56xBJnGFE%2BpT2ElSyCnJcwVzCjkqeNLfMEJqKW0G7OFIp0G%2B9mh50I9o8k1tpCY0xYqFNIALgIfc2me4n1bmJnRZ89oepgLPT0NTMLNZsvSCZAc3TXaNB07vail36%2FdBySis4m9%2FDR8izaLJW6bWCkVgm5T%2Bius3ZXq4xI%2BGnbveLbdRwF2mNtsrE0JjYc1AXknCOrLSu7Te%2Fr4dPYMCl5qtiHNTn%2BTPbh1jCBHH%2BdMJNhwNgs3nT%2BOhQoQ0vYif56BMG6WowAcHR3DjQolxLzyVekHj00PBAaW7IIAF1EF%2BuRIWyXjQMAs2chdpaKPNaB%2BkSezYt0%2BCA04sOg5vx8Fr7Ofa9sUv87h7SLAUFSzbetCCZ9pmyLt6l6%2FTzoA1%2FZBG9bIUVHLAbi%2FkdBFgYGyGwRQGBpkqCEg2ah9UD6EedEcEL3j4y0BQQCiExEnocA3SZboh%2Bepgd3YsOkHskZwPuQ5OoyA0fTA5AXrHcUOQF%2BzkJHIA7PwCDk1gGVmGUZSSoPhNf%2BTklauz98QofOlCIQ%2FtCD4dosHYPqtPCXB3agggQQIqQJsSkB%2Bqn0rkQ1toJjON%2FOtCIB9RYv3PqRA4C4U68ZMlZn6BdgEvi2ziU%2BTQ6NIw3ej%2BAtDwMGEZk7e2IjxUWKdAxyaw9OCwSmeADTPPleyk6UhGDNXQb%2B%2BW6Uk4q6F7%2Frg6WVTo82IoCxSIsFDrav4EPHphD3u4hR53WKVvYZUwNCCeM4PMBWzK%2BEfIthZOkuAwPo5C5jgoZgn6dUdvx5rIDmd58cXXdKNfw3l%2BwM2UjgrDJeQHhbD7HW2QDoZMCujgIUkk5Fg8VCsdyjOtnGRx8wgKRPZN5dR0zPUyfGZFVihbFRniXZFOZGKPnEQzU3AnD1KfR6weHW2XS6KbPJxUkOTZsAB9vTVp3Le1F8q5l%2BDMcLiIq78jxAImD2pGFw0VHfRatScGlK6SMu8leTmhUSMy8Uhdd6xBiH3Gdman4tjQGLboJfqz6fL2WKHTmrfsKZRYX6BTDjDldKMosaSTLdQS7oDisJNqAUhw1PfTlnacCO8vl8706Km1FROgLDmudzxg%2BEWTiArtHgLsRrAXYWdB0NmToNCJdKm0KWycZQqb%2BMw76Qy29iQ5up%2FX7oyw8QZ75kP5F6iJAJz6KCmqxz8fEa%2FxnsMYcIO%2FvEkGRuMckhr4rIeLrKaXnmIzlNLxbFspOphkcnJdnz%2FChp%2FVlpj2P7jJQmQRwGnltkTV5dbF9fE3%2FfxoSqTROgq9wFUlbuYzYcasE0ouzBo%2BdDCDzxKAfhbAZYxQiHrLzV2iVexnDX%2FQnT1fsT%2Fxuhu1ui5qIytgbGmRoQkeQooO8eJNNZsf0iALur8QxZFH0nCMnjerYQqG1pIfjyVZWxhVRznmmfLG00BcBWJE6hzQWRyFknuJnXuk8A5FRDCulwrWASSNoBtR%2BCtGdkPwYN2o7DOw%2FVGlCZPusRBFXODQdUM5zeHDIVuAJBLqbO%2Ff9Qua%2BpDqEPk230Sob9lEZ8BHiCorjVghuI0lI4JDgHGRDD%2FprQ84B1pVGkIpVUAHCG%2Biz3Bn3qm2AVrYcYWhock4jso5%2BJ7HfHVj4WMIQdGctq3psBCVVzupQOEioBGA2Bk%2BUILT7%2BVoX5mdxxA5fS42gISQVi%2FHTzrgMxu0fY6hE1ocUwwbsbWcezrY2n6S8%2F6cxXkOH4prpmPuFoikTzY7T85C4T2XYlbxLglSv2uLCgFv8Quk%2FwdesUdWPeHYIH0R729JIisN9Apdd4eB10aqwXrPt%2BSu9mA8k8n1sjMwnfsfF2j3jMUzXepSHmZ%2FBfqXvzgUNQQWOXO8YEuFBh4QTYCkOAPxywpYu1VxiDyJmKVcmJPGWk%2Fgc3Pov02StyYDahwmzw3E1gYC9wkupyWfDqDSUMpCTH5e5N8B%2F%2FlHiMuIkTNw4USHrJU67bjXGqNav6PBuQSoqTxc8avHoGmvqNtXzIaoyMIQIiiUHIM64cXieouplhNYln7qgc4wBVAYR104kO%2BCvKqsg4yIUlFNThVUAKZxZt1XA34h3TCUUiXVkZ0w8Hh2R0Z5L0b4LZvPd%2Fp1gi%2F07h8qfwHrByuSxglc9cI4QIg2oqvC%2Fqm0i7tjPLTgDhoWTAKDO2ONW5oe%2B%2FeKB9vZB8K6C25yCZ9RFVMnb6NRdRjyVK57CHHSkJBfnM2%2Fj4ODUwRkqrtBBCrDsDpt8jhZdXoy%2F1BCqw3sSGhgGGy0a5Jw6BP%2FTExoCmNFYjZl248A0osgPyGEmRA%2BfAsqPVaNAfytu0vuQJ7rk3J4kTDTR2AlCHJ5cls26opZM4w3jMULh2YXKpcqGBtuleAlOZnaZGbD6DHzMd6i2oFeJ8z9XYmalg1Szd%2FocZDc1C7Y6vcALJz2lYnTXiWEr2wawtoR4g3jvWUU2Ngjd1cewtFzEvM1NiHZPeLlIXFbBPawxNgMwwAlyNSuGF3zizVeOoC9bag1qRAQKQE%2FEZBWC2J8mnXAN2aTBboZ7HewnObE8CwROudZHmUM5oZ%2FUgd%2FJZQK8lvAm43uDRAbyW8gZ%2BZGq0EVerVGUKUSm%2FIdn8AQHdR4m7bue88WBwft9mSCeMOt1ncBwziOmJYI2ZR7ewNMPiCugmSsE4EyQ%2BQATJG6qORMGd4snEzc6B4shPIo4G1T7PgSm8PY5eUkPdF8JZ0VBtadbHXoJgnEhZQaODPj2gpODKJY5Yp4DOsLBFxWbvXN755KWylJm%2BoOd4zEL9Hpubuy2gyyfxh8oEfFutnYWdfB8PdESLWYvSqbElP9qo3u6KTmkhoacDauMNNjj0oy40DFV7Ql0aZj77xfGl7TJNHnIwgqOkenruYYNo6h724%2BzUQ7%2BvkCpZB%2BpGA562hYQiDxHVWOq0oDQl%2FQsoiY%2BcuI7iWq%2FZIBtHcXJ7kks%2Bh2fCNUPA82BzjnqktNts%2BRLdk1VSu%2BtqEn7QZCCsvEqk6FkfiOYkrsw092J8jsfIuEKypNjLxrKA9kiA19mxBD2suxQKCzwXGws7kEJvlhUiV9tArLIdZW0IORcxEzdzKmjtFhsjKy%2F44XYXdI5noQoRcvjZ1RMPACRqYg2V1%2BOwOepcOknRLLFdYgTkT5UApt%2FJhLM3jeFYprZV%2BZow2g8fP%2BU68hkKFWJj2yBbKqsrp25xkZX1DAjUw52IMYWaOhab8Kp05VrdNftqwRrymWF4OQSjbdfzmRZirK8FMJELEgER2PHjEAN9pGfLhCUiTJFbd5LBkOBMaxLr%2FA1SY9dXFz4RjzoU9ExfJCmx%2FI9FKEGT3n2cmzl2X42L3Jh%2BAbQq6sA%2BSs1kitoa4TAYgKHaoybHUDJ51oETdeI%2F9ThSmjWGkyLi5QAGWhL0BG1UsTyRGRJOldKBrYJeB8ljLJHfATWTEQBXBDnQexOHTB%2BUn44zExFE4vLytcu5NwpWrUxO%2F0ZICUGM7hGABXym0V6ZvDST0E370St9MIWQOTWngeoQHUTdCJUP04spMBMS8LSker9cReVQkULFDIZDFPrhTzBl6sed9wcZQTbL%2BBDqMyaN3RJPh%2Fanbx%2BIv%2BqgQdAa3M9Z5JmvYlh4qop%2BHo1F1W5gbOE9YKLgAnWytXElU4G8GtW47lhgFE6gaSs%2Bgs37sFvi0PPVvA5dnCBgILTwoKd%2F%2BDoL9F6inlM7H4rOTzD79KJgKlZO%2FZgt22UsKhrAaXU5ZcLrAglTVKJEmNJvORGN1vqrcfSMizfpsgbIe9zno%2BgBoKVXgIL%2FVI8dB1O5o%2FR3Suez%2FgD7M781ShjKpIIORM%2FnxG%2BjjhhgPwsn2IoXsPGPqYHXA63zJ07M2GPEykQwJBYLK808qYxuIew4frk52nhCsnCYmXiR6CuapvE1IwRB4%2FQftDbEn%2BAucIr1oxrLabRj9q4ae0%2BfXkHnteAJwXRbVkR0mctVSwEbqhJiMSZUp9DNbEDMmjX22m3ABpkrPQQTP3S1sib5pD2VRKRd%2BeNAjLYyT0hGrdjWJZy24OYXRoWQAIhGBZRxuBFMjjZQhpgrWo8SiFYbojcHO8V5DyscJpLTHyx9Fimassyo5U6WNtquUMYgccaHY5amgR3PQzq3ToNM5ABnoB9kuxsebqmYZm0R9qxJbFXCQ1UPyFIbxoUraTJFDpCk0Wk9GaYJKz%2F6oHwEP0Q14lMtlddQsOAU9zlYdMVHiT7RQP3XCmWYDcHCGbVRHGnHuwzScA0BaSBOGkz3lM8CArjrBsyEoV6Ys4qgDK3ykQQPZ3hCRGNXQTNNXbEb6tDiTDLKOyMzRhCFT%2BmAUmiYbV3YQVqFVp9dorv%2BTsLeCykS2b5yyu8AV7IS9cxcL8z4Kfwp%2BxJyYLv1OsxQCZwTB4a8BZ%2F5EdxTBJthApqyfd9u3ifr%2FWILTqq5VqgwMT9SOxbSGWLQJUUWCVi4k9tho9nEsbUh7U6NUsLmkYFXOhZ0kmamaJLRNJzSj%2Fqn4Mso6zb6iLLBXoaZ6AqeWCjHQm2lztnejYYM2eubnpBdKVLORZhudH3JF1waBJKA9%2BW8EhMj3Kzf0L4vi4k6RoHh3Z5YgmSZmk6ns4fjScjAoL8GoOECgqgYEBYUGFVO4FUv4%2FYtowhEmTs0vrvlD%2FCrisnoBNDAcUi%2FteY7OctFlmARQzjOItrrlKuPO6E2Ox93L4O%2F4DcgV%2FdZ7qR3VBwVQxP1GCieA4RIpweYJ5FoYrHxqRBdJjnqbsikA2Ictbb8vE1GYIo9dacK0REgDX4smy6GAkxlH1yCGGsk%2BtgiDhNKuKu3yNrMdxafmKTF632F8Vx4BNK57GvlFisrkjN9WDAtjsWA0ENT2e2nETUb%2Fn7qwhvGnrHuf5bX6Vh%2Fn3xffU3PeHdR%2BFA92i6ufT3AlyAREoNDh6chiMWTvjKjHDeRhOa9YkOQRq1vQXEMppAQVwHCuIcV2g5rBn6GmZZpTR7vnSD6ZmhdSl176gqKTXu5E%2BYbfL0adwNtHP7dT7t7b46DVZIkzaRJOM%2BS6KcrzYVg%2BT3wSRFRQashjfU18NutrKa%2F7PXbtuJvpIjbgPeqd%2BpjmRw6YKpnANFSQcpzTZgpSNJ6J7uiagAbir%2F8tNXJ%2FOsOnRh6iuIexxrmkIneAgz8QoLmiaJ8sLQrELVK2yn3wOHp57BAZJhDZjTBzyoRAuuZ4eoxHruY1pSb7qq79cIeAdOwin4GdgMeIMHeG%2BFZWYaiUQQyC5b50zKjYw97dFjAeY2I4Bnl105Iku1y0lMA1ZHolLx19uZnRdILcXKlZGQx%2FGdEqSsMRU1BIrFqRcV1qQOOHyxOLXEGcbRtAEsuAC2V4K3p5mFJ22IDWaEkk9ttf5Izb2LkD1MnrSwztXmmD%2FQi%2FEmVEFBfiKGmftsPwVaIoZanlKndMZsIBOskFYpDOq3QUs9aSbAAtL5Dbokus2G4%2FasthNMK5UQKCOhU97oaOYNGsTah%2BjfCKsZnTRn5TbhFX8ghg8CBYt%2FBjeYYYUrtUZ5jVij%2Fop7V5SsbA4mYTOwZ46hqdpbB6Qvq3AS2HHNkC15pTDIcDNGsMPXaBidXYPHc6PJAkRh29Vx8KcgX46LoUQBhRM%2B3SW6Opll%2FwgxxsPgKJKzr5QCmwkUxNbeg6Wj34SUnEzOemSuvS2OetRCO8Tyy%2BQbSKVJcqkia%2BGvDefFwMOmgnD7h81TUtMn%2BmRpyJJ349HhAnoWFTejhpYTL9G8N2nVg1qkXBeoS9Nw2fB27t7trm7d%2FQK7Cr4uoCeOQ7%2F8JfKT77KiDzLImESHw%2F0wf73QeHu74hxv7uihi4fTX%2BXEwAyQG3264dwv17aJ5N335Vt9sdrAXhPOAv8JFvzqyYXwfx8WYJaef1gMl98JRFyl5Mv5Uo%2FoVH5ww5OzLFsiTPDns7fS6EURSSWd%2F92BxMYQ8sBaH%2Bj%2BwthQPdVgDGpTfi%2BJQIWMD8xKqULliRH01rTeyF8x8q%2FGBEEEBrAJMPf25UQwi0b8tmqRXY7kIvNkzrkvRWLnxoGYEJsz8u4oOyMp8cHyaybb1HdMCaLApUE%2B%2F7xLIZGP6H9xuSEXp1zLIdjk5nBaMuV%2FyTDRRP8Y2ww5RO6d2D94o%2B6ucWIqUAvgHIHXhZsmDhjVLczmZ3ca0Cb3PpKwt2UtHVQ0BgFJsqqTsnzZPlKahRUkEu4qmkJt%2Bkqdae76ViWe3STan69yaF9%2BfESD2lcQshLHWVu4ovItXxO69bqC5p1nZLvI8NdQB9s9UNaJGlQ5mG947ipdDA0eTIw%2FA1zEdjWquIsQXXGIVEH0thC5M%2BW9pZe7IhAVnPJkYCCXN5a32HjN6nsvokEqRS44tGIs7s2LVTvcrHAF%2BRVmI8L4HUYk4x%2B67AxSMJKqCg8zrGOgvK9kNMdDrNiUtSWuHFpC8%2Fp5qIQrEo%2FH%2B1l%2F0cAwQ2nKmpWxKcMIuHY44Y6DlkpO48tRuUGBWT0FyHwSKO72Ud%2BtJUfdaZ4CWNijzZtlRa8%2BCkmO%2FEwHYfPZFU%2FhzjFWH7vnzHRMo%2BaF9u8qHSAiEkA2HjoNQPEwHsDKOt6hOoK3Ce%2F%2B%2F9boMWDa44I6FrQhdgS7OnNaSzwxWKZMcyHi6LN4WC6sSj0qm2PSOGBTvDs%2FGWJS6SwEN%2FULwpb4LQo9fYjUfSXRwZkynUazlSpvX9e%2BG2zor8l%2BYaMxSEomDdLHGcD6YVQPegTaA74H8%2BV4WvJkFUrjMLGLlvSZQWvi8%2FQA7yzQ8GPno%2F%2F5SJHRP%2FOqKObPCo81s%2F%2B6WgLqykYpGAgQZhVDEBPXWgU%2FWzFZjKUhSFInufPRiMAUULC6T11yL45ZrRoB4DzOyJShKXaAJIBS9wzLYIoCEcJKQW8GVCx4fihqJ6mshBUXSw3wWVj3grrHQlGNGhIDNNzsxQ3M%2BGWn6ASobIWC%2BLbYOC6UpahVO13Zs2zOzZC8z7FmA05JhUGyBsF4tsG0drcggIFzgg%2Fkpf3%2BCnAXKiMgIE8Jk%2FMhpkc8DUJEUzDSnWlQFme3d0sHZDrg7LavtsEX3cHwjCYA17pMTfx8Ajw9hHscN67hyo%2BRJQ4458RmPywXykkVcW688oVUrQhahpPRvTWPnuI0B%2BSkQu7dCyvLRyFYlC1LG1gRCIvn3rwQeINzZQC2KXq31FaR9UmVV2QeGVqBHjmE%2BVMd3b1fhCynD0pQNhCG6%2FWCDbKPyE7NRQzL3BzQAJ0g09aUzcQA6mUp9iZFK6Sbp%2FYbHjo%2B%2B7%2FWj8S4YNa%2BZdqAw1hDrKWFXv9%2BzaXpf8ZTDSbiqsxnwN%2FCzK5tPkOr4tRh2kY3Bn9JtalbIOI4b3F7F1vPQMfoDcdxMS8CW9m%2FNCW%2FHILTUVWQIPiD0j1A6bo8vsv6P1hCESl2abrSJWDrq5sSzUpwoxaCU9FtJyYH4QFMxDBpkkBR6kn0LMPO%2B5EJ7Z6bCiRoPedRZ%2FP0SSdii7ZnPAtVwwHUidcdyspwncz5uq6vvm4IEDbJVLUFCn%2FLvIHfooUBTkFO130FC7CmmcrKdgDJcid9mvVzsDSibOoXtIf9k6ABle3PmIxejodc4aob0QKS432srrCMndbfD454q52V01G4q913mC5HOsTzWF4h2No1av1VbcUgWAqyoZl%2B11PoFYnNv2HwAODeNRkHj%2B8SF1fcvVBu6MrehHAZK1Gm69ICcTKizykHgGFx7QdowTVAsYEF2tVc0Z6wLryz2FI1sc5By2znJAAmINndoJiB4sfPdPrTC8RnkW7KRCwxC6YvXg5ahMlQuMpoCSXjOlBy0Kij%2BbsCYPbGp8BdCBiLmLSAkEQRaieWo1SYvZIKJGj9Ur%2FeWHjiB7SOVdqMAVmpBvfRiebsFjger7DC%2B8kRFGtNrTrnnGD2GAJb8rQCWkUPYHhwXsjNBSkE6lGWUj5QNhK0DMNM2l%2BkXRZ0KLZaGsFSIdQz%2FHXDxf3%2FTE30%2BDgBKWGWdxElyLccJfEpjsnszECNoDGZpdwdRgCixeg9L4EPhH%2BRptvRMVRaahu4cySjS3P5wxAUCPkmn%2BrhyASpmiTaiDeggaIxYBmtLZDDhiWIJaBgzfCsAGUF1Q1SFZYyXDt9skCaxJsxK2Ms65dmdp5WAZyxik%2FzbrTQk5KmgxCg%2Ff45L0jywebOWUYFJQAJia7XzCV0x89rpp%2Ff3AVWhSPyTanqmik2SkD8A3Ml4NhIGLAjBXtPShwKYfi2eXtrDuKLk4QlSyTw1ftXgwqA2jUuopDl%2B5tfUWZNwBpEPXghzbBggYCw%2Fdhy0ntds2yeHCDKkF%2FYxQjNIL%2FF%2F37jLPHCKBO9ibwYCmuxImIo0ijV2Wbg3kSN2psoe8IsABv3RNFaF9uMyCtCYtqcD%2BqNOhwMlfARQUdJ2tUX%2BMNJqOwIciWalZsmEjt07tfa8ma4cji9sqz%2BQ9hWfmMoKEbIHPOQORbhQRHIsrTYlnVTNvcq1imqmmPDdVDkJgRcTgB8Sb6epCQVmFZe%2BjGDiNJQLWnfx%2BdrTKYjm0G8yH0ZAGMWzEJhUEQ4Maimgf%2Fbkvo8PLVBsZl152y5S8%2BHRDfZIMCbYZ1WDp4yrdchOJw8k6R%2B%2F2pHmydK4NIK2PHdFPHtoLmHxRDwLFb7eB%2BM4zNZcB9NrAgjVyzLM7xyYSY13ykWfIEEd2n5%2FiYp3ZdrCf7fL%2Ben%2BsIJu2W7E30MrAgZBD1rAAbZHPgeAMtKCg3NpSpYQUDWJu9bT3V7tOKv%2BNRiJc8JAKqqgCA%2FPNRBR7ChpiEulyQApMK1AyqcWnpSOmYh6yLiWkGJ2mklCSPIqN7UypWj3dGi5MvsHQ87MrB4VFgypJaFriaHivwcHIpmyi5LhNqtem4q0n8awM19Qk8BOS0EsqGscuuydYsIGsbT5GHnERUiMpKJl4ON7qjB4fEqlGN%2FhCky89232UQCiaeWpDYCJINXjT6xl4Gc7DxRCtgV0i1ma4RgWLsNtnEBRQFqZggCLiuyEydmFd7WlogpkCw5G1x4ft2psm3KAREwVwr1Gzl6RT7FDAqpVal34ewVm3VH4qn5mjGj%2BbYL1NgfLNeXDwtmYSpwzbruDKpTjOdgiIHDVQSb5%2FzBgSMbHLkxWWgghIh9QTFSDILixVwg0Eg1puooBiHAt7DzwJ7m8i8%2Fi%2BjHvKf0QDnnHVkVTIqMvIQImOrzCJwhSR7qYB5gSwL6aWL9hERHCZc4G2%2BJrpgHNB8eCCmcIWIQ6rSdyPCyftXkDlErUkHafHRlkOIjxGbAktz75bnh50dU7YHk%2BMz7wwstg6RFZb%2BTZuSOx1qqP5C66c0mptQmzIC2dlpte7vZrauAMm%2F7RfBYkGtXWGiaWTtwvAQiq2oD4YixPLXE2khB2FRaNRDTk%2B9sZ6K74Ia9VntCpN4BhJGJMT4Z5c5FhSepRCRWmBXqx%2BwhVZC4me4saDs2iNqXMuCl6iAZflH8fscC1sTsy4PHeC%2BXYuqMBMUun5YezKbRKmEPwuK%2BCLzijPEQgfhahQswBBLfg%2FGBgBiI4QwAqzJkkyYAWtjzSg2ILgMAgqxYfwERRo3zruBL9WOryUArSD8sQOcD7fvIODJxKFS615KFPsb68USBEPPj1orNzFY2xoTtNBVTyzBhPbhFH0PI5AtlJBl2aSgNPYzxYLw7XTDBDinmVoENwiGzmngrMo8OmnRP0Z0i0Zrln9DDFcnmOoBZjABaQIbPOJYZGqX%2BRCMlDDbElcjaROLDoualmUIQ88Kekk3iM4OQrADcxi3rJguS4MOIBIgKgXrjd1WkbCdqxJk%2F4efRIFsavZA7KvvJQqp3Iid5Z0NFc5aiMRzGN3vrpBzaMy4JYde3wr96PjN90AYOIbyp6T4zj8LoE66OGcX1Ef4Z3KoWLAUF4BTg7ug%2FAbkG5UNQXAMkQezujSHeir2uTThgd3gpyzDrbnEdDRH2W7U6PeRvBX1ZFMP5RM%2BZu6UUZZD8hDPHldVWntTCNk7To8IeOW9yn2wx0gmurwqC60AOde4r3ETi5pVMSDK8wxhoGAoEX9NLWHIR33VbrbMveii2jAJlrxwytTHbWNu8Y4N8vCCyZjAX%2FpcsfwXbLze2%2BD%2Bu33OGBoJyAAL3jn3RuEcdp5If8O%2Ba4NKWvxOTyDltG0IWoHhwVGe7dKkCWFT%2B%2Btm%2BhaBCikRUUMrMhYKZJKYoVuv%2FbsJzO8DwfVIInQq3g3BYypiz8baogH3r3GwqCwFtZnz4xMjAVOYnyOi5HWbFA8n0qz1OjSpHWFzpQOpvkNETZBGpxN8ybhtqV%2FDMUxd9uFZmBfKXMCn%2FSqkWJyKPnT6lq%2B4zBZni6fYRByJn6OK%2BOgPBGRAJluwGSk4wxjOOzyce%2FPKODwRlsgrVkdcsEiYrqYdXo0Er2GXi2GQZd0tNJT6c9pK1EEJG1zgDJBoTVuCXGAU8BKTvCO%2FcEQ1Wjk3Zzuy90JX4m3O5IlxVFhYkSUwuQB2up7jhvkm%2BbddRQu5F9s0XftGEJ9JSuSk%2BZachCbdU45fEqbugzTIUokwoAKvpUQF%2FCvLbWW5BNQFqFkJg2f30E%2F48StNe5QwBg8zz3YAJ82FZoXBxXSv4QDooDo79NixyglO9AembuBcx5Re3CwOKTHebOPhkmFC7wNaWtoBhFuV4AkEuJ0J%2B1pT0tLkvFVZaNzfhs%2FKd3%2BA9YsImlO4XK4vpCo%2FelHQi%2F9gkFg07xxnuXLt21unCIpDV%2BbbRxb7FC6nWYTsMFF8%2B1LUg4JFjVt3vqbuhHmDKbgQ4e%2BRGizRiO8ky05LQGMdL2IKLSNar0kNG7lHJMaXr5mLdG3nykgj6vB%2FKVijd1ARWkFEf3yiUw1v%2FWaQivVUpIDdSNrrKbjO5NPnxz6qTTGgYg03HgPhDrCFyYZTi3XQw3HXCva39mpLNFtz8AiEhxAJHpWX13gCTAwgm9YTvMeiqetdNQv6IU0hH0G%2BZManTqDLPjyrOse7WiiwOJCG%2BJ0pZYULhN8NILulmYYvmVcV2MjAfA39sGKqGdjpiPo86fecg65UPyXDIAOyOkCx5NQsLeD4gGVjTVDwOHWkbbBW0GeNjDkcSOn2Nq4cEssP54t9D749A7M1AIOBl0Fi0sSO5v3P7LCBrM6ZwFY6kp2FX6AcbGUdybnfChHPyu6WlRZ2Fwv9YM0RMI7kISRgR8HpQSJJOyTfXj%2F6gQKuihPtiUtlCQVPohUgzfezTg8o1b3n9pNZeco1QucaoXe40Fa5JYhqdTspFmxGtW9h5ezLFZs3j%2FN46f%2BS2rjYNC2JySXrnSAFhvAkz9a5L3pza8eYKHNoPrvBRESpxYPJdKVUxBE39nJ1chrAFpy4MMkf0qKgYALctGg1DQI1kIymyeS2AJNT4X240d3IFQb%2F0jQbaHJ2YRK8A%2Bls6WMhWmpCXYG5jqapGs5%2FeOJErxi2%2F2KWVHiPellTgh%2FfNl%2F2KYPKb7DUcAg%2BmCOPQFCiU9Mq%2FWLcU1xxC8aLePFZZlE%2BPCLzf7ey46INWRw2kcXySR9FDgByXzfxiNKwDFbUSMMhALPFSedyjEVM5442GZ4hTrsAEvZxIieSHGSgkwFh%2FnFNdrrFD4tBH4Il7fW6ur4J8Xaz7RW9jgtuPEXQsYk7gcMs2neu3zJwTyUerHKSh1iTBkj2YJh1SSOZL5pLuQbFFAvyO4k1Hxg2h99MTC6cTUkbONQIAnEfGsGkNFWRbuRyyaEZInM5pij73EA9rPIUfU4XoqQpHT9THZkW%2BoKFLvpyvTBMM69tN1Ydwv1LIEhHsC%2BueVG%2Bw%2BkyCPsvV3erRikcscHjZCkccx6VrBkBRusTDDd8847GA7p2Ucy0y0HdSRN6YIBciYa4vuXcAZbQAuSEmzw%2BH%2FAuOx%2BaH%2BtBL88H57D0MsqyiZxhOEQkF%2F8DR1d2hSPMj%2FsNOa5rxcUnBgH8ictv2J%2Bcb4BA4v3MCShdZ2vtK30vAwkobnEWh7rsSyhmos3WC93Gn9C4nnAd%2FPjMMtQfyDNZsOPd6XcAsnBE%2FmRHtHEyJMzJfZFLE9OvQa0i9kUmToJ0ZxknTgdl%2FXPV8xoh0K7wNHHsnBdvFH3sv52lU7UFteseLG%2FVanIvcwycVA7%2BBE1Ulyb20BvwUWZcMTKhaCcmY3ROpvonVMV4N7yBXTL7IDtHzQ4CCcqF66LjF3xUqgErKzolLyCG6Kb7irP%2FMVTCCwGRxfrPGpMMGvPLgJ881PHMNMIO09T5ig7AzZTX%2F5PLlwnJLDAPfuHynSGhV4tPqR3gJ4kg4c06c%2FF1AcjGytKm2Yb5jwMotF7vro4YDLWlnMIpmPg36NgAZsGA0W1spfLSue4xxat0Gdwd0lqDBOgIaMANykwwDKejt5YaNtJYIkrSgu0KjIg0pznY0SCd1qlC6R19g97UrWDoYJGlrvCE05J%2F5wkjpkre727p5PTRX5FGrSBIfJqhJE%2FIS876PaHFkx9pGTH3oaY3jJRvLX9Iy3Edoar7cFvJqyUlOhAEiOSAyYgVEGkzHdug%2BoRHIEOXAExMiTSKU9A6nmRC8mp8iYhwWdP2U%2F5EkFAdPrZw03YA3gSyNUtMZeh7dDCu8pF5x0VORCTgKp07ehy7NZqKTpIC4UJJ89lnboyAfy5OyXzXtuDRbtAFjZRSyGFTpFrXwkpjSLIQIG3N0Vj4BtzK3wdlkBJrO18MNsgseR4BysJilI0wI6ZahLhBFA0XBmV8d4LUzEcNVb0xbLjLTETYN8OEVqNxkt10W614dd1FlFFVTIgB7%2FBQQp1sWlNolpIu4ekxUTBV7NmxOFKEBmmN%2BnA7pvF78%2FRII5ZHA09OAiE%2F66MF6HQ%2BqVEJCHxwymukkNvzqHEh52dULPbVasfQMgTDyBZzx4007YiKdBuUauQOt27Gmy8ISclPmEUCIcuLbkb1mzQSqIa3iE0PJh7UMYQbkpe%2BhXjTJKdldyt2mVPwywoODGJtBV1lJTgMsuSQBlDMwhEKIfrvsxGQjHPCEfNfMAY2oxvyKcKPUbQySkKG6tj9AQyEW3Q5rpaDJ5Sns9ScLKeizPRbvWYAw4bXkrZdmB7CQopCH8NAmqbuciZChHN8lVGaDbCnmddnqO1PQ4ieMYfcSiBE5zzMz%2BJV%2F4eyzrzTEShvqSGzgWimkNxLvUj86iAwcZuIkqdB0VaIB7wncLRmzHkiUQpPBIXbDDLHBlq7vp9xwuC9AiNkIptAYlG7Biyuk8ILdynuUM1cHWJgeB%2BK3wBP%2FineogxkvBNNQ4AkW0hvpBOQGFfeptF2YTR75MexYDUy7Q%2F9uocGsx41O4IZhViw%2F2FvAEuGO5g2kyXBUijAggWM08bRhXg5ijgMwDJy40QeY%2FcQpUDZiIzmvskQpO5G1zyGZA8WByjIQU4jRoFJt56behxtHUUE%2Fom7Rj2psYXGmq3llVOCgGYKNMo4pzwntITtapDqjvQtqpjaJwjHmDzSVGLxMt12gEXAdLi%2FcaHSM3FPRGRf7dB7YC%2BcD2ho6oL2zGDCkjlf%2FDFoQVl8GS%2F56wur3rdV6ggtzZW60MRB3g%2BU1W8o8cvqIpMkctiGVMzXUFI7FacFLrgtdz4mTEr4aRAaQ2AFQaNeG7GX0yOJgMRYFziXdJf24kg%2FgBQIZMG%2FYcPEllRTVNoDYR6oSJ8wQNLuihfw81UpiKPm714bZX1KYjcXJdfclCUOOpvTxr9AAJevTY4HK%2FG7F3mUc3GOAKqh60zM0v34v%2BELyhJZqhkaMA8UMMOU90f8RKEJFj7EqepBVwsRiLbwMo1J2zrE2UYJnsgIAscDmjPjnzI8a719Wxp757wqmSJBjXowhc46QN4RwKIxqEE6E5218OeK7RfcpGjWG1jD7qND%2B%2FGTk6M56Ig4yMsU6LUW1EWE%2BfIYycVV1thldSlbP6ltdC01y3KUfkobkt2q01YYMmxpKRvh1Z48uNKzP%2FIoRIZ%2FF6buOymSnW8gICitpJjKWBscSb9JJKaWkvEkqinAJ2kowKoqkqZftRqfRQlLtKoqvTRDi2vg%2FRrPD%2Fd3a09J8JhGZlEkOM6znTsoMCsuvTmywxTCDhw5dd0GJOHCMPbsj3QLkTE3MInsZsimDQ3HkvthT7U9VA4s6G07sID0FW4SHJmRGwCl%2BMu4xf0ezqeXD2PtPDnwMPo86sbwDV%2B9PWcgFcARUVYm3hrFQrHcgMElFGbSM2A1zUYA3baWfheJp2AINmTJLuoyYD%2FOwA4a6V0ChBN97E8YtDBerUECv0u0TlxR5yhJCXvJxgyM73Bb6pyq0jTFJDZ4p1Am1SA6sh8nADd1hAcGBMfq4d%2FUfwnmBqe0Jun1n1LzrgKuZMAnxA3NtCN7Klf4BH%2B14B7ibBmgt0TGUafVzI4uKlpF7v8NmgNjg90D6QE3tbx8AjSAC%2BOA1YJvclyPKgT27QpIEgVYpbPYGBsnyCNrGz9XUsCHkW1QAHgL2STZk12QGqmvAB0NFteERkvBIH7INDsNW9KKaAYyDMdBEMzJiWaJHZALqDxQDWRntumSDPcplyFiI1oDpT8wbwe01AHhW6%2BvAUUBoGhY3CT2tgwehdPqU%2F4Q7ZLYvhRl%2FogOvR9O2%2BwkkPKW5vCTjD2fHRYXONCoIl4Jh1bZY0ZE1O94mMGn%2FdFSWBWzQ%2FVYk%2BGezi46RgiDv3EshoTmMSlioUK6MQEN8qeyK6FRninyX8ZPeUWjjbMJChn0n%2FyJvrq5bh5UcCAcBYSafTFg7p0jDgrXo2QWLb3WpSOET%2FHh4oSadBTvyDo10IufLzxiMLAnbZ1vcUmj3w7BQuIXjEZXifwukVxrGa9j%2BDXfpi12m1RbzYLg9J2wFergEwOxFyD0%2FJstNK06ZN2XdZSGWxcJODpQHOq4iKqjqkJUmPu1VczL5xTGUfCgLEYyNBCCbMBFT%2FcUP6pE%2FmujnHsSDeWxMbhrNilS5MyYR0nJyzanWXBeVcEQrRIhQeJA6Xt4f2eQESNeLwmC10WJVHqwx8SSyrtAAjpGjidcj1E2FYN0LObUcFQhafUKTiGmHWRHGsFCB%2BHEXgrzJEB5bp0QiF8ZHh11nFX8AboTD0PS4O1LqF8XBks2MpjsQnwKHF6HgaKCVLJtcr0XjqFMRGfKv8tmmykhLRzu%2BvqQ02%2BKpJBjaLt9ye1Ab%2BBbEBhy4EVdIJDrL2naV0o4wU8YZ2Lq04FG1mWCKC%2BUwkXOoAjneU%2FxHplMQo2cXUlrVNqJYczgYlaOEczVCs%2FOCgkyvLmTmdaBJc1iBLuKwmr6qtRnhowngsDxhzKFAi02tf8bmET8BO27ovJKF1plJwm3b0JpMh38%2BxsrXXg7U74QUM8ZCIMOpXujHntKdaRtsgyEZl5MClMVMMMZkZLNxH9%2Bb8fH6%2Bb8Lev30A9TuEVj9CqAdmwAAHBPbfOBFEATAPZ2CS0OH1Pj%2F0Q7PFUcC8hDrxESWdfgFRm%2B7vvWbkEppHB4T%2F1ApWnlTIqQwjcPl0VgS1yHSmD0OdsCVST8CQVwuiew1Y%2Bg3QGFjNMzwRB2DSsAk26cmA8lp2wIU4p93AUBiUHFGOxOajAqD7Gm6NezNDjYzwLOaSXRBYcWipTSONHjUDXCY4mMI8XoVCR%2FRrs%2FJLKXgEx%2BqkmeDlFOD1%2FyTQNDClRuiUyKYCllfMiQiyFkmuTz2vLsBNyRW%2Bxz%2B5FElFxWB28VjYIGZ0Yd%2B5wIjkcoMaggxswbT0pCmckRAErbRlIlcOGdBo4djTNO8FAgQ%2BlT6vPS60BwTRSUAM3ddkEAZiwtEyArrkiDRnS7LJ%2B2hwbzd2YDQagSgACpsovmjil5wfPuXq3GuH0CyE7FK3M4FgRaFoIkaodORrPx1%2BJpI9psyNYIFuJogZa0%2F1AhOWdlHQxdAgbwacsHqPZo8u%2FngAH2GmaTdhYnBfSDbBfh8CHq6Bx5bttP2%2BRdM%2BMAaYaZ0Y%2FADkbNCZuAyAVQa2OcXOeICmDn9Q%2FeFkDeFQg5MgHEDXq%2FtVjj%2Bjtd26nhaaolWxs1ixSUgOBwrDhRIGOLyOVk2%2FBc0UxvseQCO2pQ2i%2BKrfhu%2FWeBovNb5dJxQtJRUDv2mCwYVpNl2efQM9xQHnK0JwLYt%2FU0Wf%2BphiA4uw8G91slC832pmOTCAoZXohg1fewCZqLBhkOUBofBWpMPsqg7XEXgPfAlDo2U5WXjtFdS87PIqClCK5nW6adCeXPkUiTGx0emOIDQqw1yFYGHEVx20xKjJVYe0O8iLmnQr3FA9nSIQilUKtJ4ZAdcTm7%2BExseJauyqo30hs%2B1qSW211A1SFAOUgDlCGq7eTIcMAeyZkV1SQJ4j%2Fe1Smbq4HcjqgFbLAGLyKxlMDMgZavK5NAYH19Olz3la%2FQCTiVelFnU6O%2FGCvykqS%2FwZJDhKN9gBtSOp%2F1SP5VRgJcoVj%2Bkmf2wBgv4gjrgARBWiURYx8xENV3bEVUAAWWD3dYDKAIWk5opaCFCMR5ZjJExiCAw7gYiSZ2rkyTce4eNMY3lfGn%2B8p6%2BvBckGlKEXnA6Eota69OxDO9oOsJoy28BXOR0UoXNRaJD5ceKdlWMJlOFzDdZNpc05tkMGQtqeNF2lttZqNco1VtwXgRstLSQ6tSPChgqtGV5h2DcDReIQadaNRR6AsAYKL5gSFsCJMgfsaZ7DpKh8mg8Wz8V7H%2BgDnLuMxaWEIUPevIbClgap4dqmVWSrPgVYCzAoZHIa5z2Ocx1D%2FGvDOEqMOKLrMefWIbSWHZ6jbgA8qVBhYNHpx0P%2BjAgN5TB3haSifDcApp6yymEi6Ij%2FGsEpDYUgcHATJUYDUAmC1SCkJ4cuZXSAP2DEpQsGUjQmKJfJOvlC2x%2FpChkOyLW7KEoMYc5FDC4v2FGqSoRWiLsbPCiyg1U5yiHZVm1XLkHMMZL11%2Fyxyw0UnGig3MFdZklN5FI%2FqiT65T%2BjOXOdO7XbgWurOAZR6Cv9uu1cm5LjkXX4xi6mWn5r5NjBS0gTliHhMZI2WNqSiSphEtiCAwnafS11JhseDGHYQ5%2BbqWiAYiAv6Jsf79%2FVUs4cIl%2Bn6%2BWOjcgB%2F2l5TreoAV2717JzZbQIR0W1cl%2FdEqCy5kJ3ZSIHuU0vBoHooEpiHeQWVkkkOqRX27eD1FWw4BfO9CJDdKoSogQi3hAAwsPRFrN5RbX7bqLdBJ9JYMohWrgJKHSjVl1sy2xAG0E3sNyO0oCbSGOxCNBRRXTXenYKuwAoDLfnDcQaCwehUOIDiHAu5m5hMpKeKM4sIo3vxACakIxKoH2YWF2QM84e6F5C5hJU4g8uxuFOlAYnqtwxmHyNEawLW%2FPhoawJDrGAP0JYWHgAVUByo%2FbGdiv2T2EMg8gsS14%2FrAdzlOYazFE7w4OzxeKiWdm3nSOnQRRKXSlVo8HEAbBfyJMKqoq%2BSCcTSx5NDtbFwNlh8VhjGGDu7JG5%2FTAGAvniQSSUog0pNzTim8Owc6QTuSKSTXlQqwV3eiEnklS3LeSXYPXGK2VgeZBqNcHG6tZHvA3vTINhV0ELuQdp3t1y9%2BogD8Kk%2FW7QoRN1UWPqM4%2BxdygkFDPLoTaumKReKiLWoPHOfY54m3qPx4c%2B4pgY3MRKKbljG8w4wvz8pxk3AqKsy4GMAkAtmRjRMsCxbb4Q2Ds0Ia9ci8cMT6DmsJG00XaHCIS%2Bo3F8YVVeikw13w%2BOEDaCYYhC0ZE54kA4jpjruBr5STWeqQG6M74HHL6TZ3lXrd99ZX%2B%2B7LhNatQaZosuxEf5yRA15S9gPeHskBIq3Gcw81AGb9%2FO53DYi%2F5CsQ51EmEh8Rkg4vOciClpy4d04eYsfr6fyQkBmtD%2BP8sNh6e%2BXYHJXT%2FlkXxT4KXU5F2sGxYyzfniMMQkb9OjDN2C8tRRgTyL7GwozH14PrEUZc6oz05Emne3Ts5EG7WolDmU8OB1LDG3VrpQxp%2BpT0KYV5dGtknU64JhabdqcVQbGZiAxQAnvN1u70y1AnmvOSPgLI6uB4AuDGhmAu3ATkJSw7OtS%2F2ToPjqkaq62%2F7WFG8advGlRRqxB9diP07JrXowKR9tpRa%2BjGJ91zxNTT1h8I2PcSfoUPtd7NejVoH03EUcqSBuFZPkMZhegHyo2ZAITovmm3zAIdGFWxoNNORiMRShgwdYwFzkPw5PA4a5MIIQpmq%2Bnsp3YMuXt%2FGkXxLx%2FP6%2BZJS0lFyz4MunC3eWSGE8xlCQrKvhKUPXr0hjpAN9ZK4PfEDrPMfMbGNWcHDzjA7ngMxTPnT7GMHar%2BgMQQ3NwHCv4zH4BIMYvzsdiERi6gebRmerTsVwZJTRsL8dkZgxgRxmpbgRcud%2BYlCIRpPwHShlUSwuipZnx9QCsEWziVazdDeKSYU5CF7UVPAhLer3CgJOQXl%2Fzh575R5rsrmRnKAzq4POFdgbYBuEviM4%2BLVC15ssLNFghbTtHWerS1hDt5s4qkLUha%2FqpZXhWh1C6lTQAqCNQnaDjS7UGFBC6wTu8yFnKJnExCnAs3Ok9yj5KpfZESQ4lTy5pTGTnkAUpxI%2ByjEldJfSo4y0QhG4i4IwkRFGcjWY8%2BEzgYYJUK7BXQksLxAww%2FYYWBMhJILB9e8ePEJ4OP7z%2B4%2FwOQDl64iOYDp26DaONPxpKtBxq%2FaTzRGarm3VkPYTLJKx6Z%2FMw2YbBGseJhPMwhhNswrIkyvV2BYzrvZbxLpKwcWJhYmFtVZ%2BlPEq91FzVp1HlQY1bZVLqeNR9SAUn6n0E28k%2FUuGkNpP1DBI5ch%2FEehZfjUQ9aE41NhETExoPT2gGQz0IhWJbEOvTQ4wgcXCHHFBhewYUiFHuhRSAUVmEHeCRQHQkXGFwkAgyzREJCVN7TRnTon36Zw3tPhx4EALwNdwDv%2BJ41YSP4B2CQqz0EFgARZ4ESgBHQgROwAVn9GTI%2BHYexTUevLUeta4%2FDqKrbMVS%2BYqb8hUwYCrlgKtmAq1YCrFgKrd4qpXiqZcKn1oqdWipjYKpWwVPVYqW6xUpVipKqFR3QKjagVEtAqHpxUMTitsnFaJOKx2cVhswq35RVpyiq9lFVNIKnOQVMkgqtYxVNxiqQjFS7GKlSIVIsQqPIhUWwioigFQ%2B%2BKkN8VHr49HDw9Ebo9EDo9DTo9Crg9BDg9%2FWx7gWx7YWwlobYrOGxWPNisAaAHEyALpkAVDIAeWAArsABVXACYuAD5cAF6wAKFQAQqgAbVAAsoAAlQAUaYAfkwAvogBWQACOgAD9AAHSAAKT4GUdMiOvFngBTwCn2AZ7Dv6B6k%2F90B8%2ByRnkV144AIBoAMTQATGgAjNAA4YABgwABZgB%2FmQCwyAVlwCguASlwCEuAQFwB4uAMlwBYuAJlQAUVAAhUD2KgdpUDaJgaRMDFJgX5MC1JgWJEAokQCWRAHxEAWkQBMRADpEAMkQAYROAEecC484DRpwBDTnwNOdw05tjTmiNOYwtswhYFwLA7BYG4LA2BYGOLAwRYFuLAsxYFQJAohIEyJAMwkAwiQC0JAJgkAeiQBkJAFokAPCQA0JABwcD4Dgc4cDdDgaYcDIDgYgUC6CgWgUClCgUYUAVBQBOFAEYMALgwAgDA9QYAdIn8AZzeBB2L5EcWrenUT1KXienEsuJJ7x5U8XlTjc1NVzUyXFTGb1LlpUtWlTDIjqwE4LsagowoCi2gJLKAkpoBgJQNpAIhNqaEoneI6kiiqQ6Go%2Fn6j0cS%2Ba2gEU8gIHJ%2BBwfgZX4GL%2BBd%2FgW34FZ%2BBS%2FgUH4FN6BTegTvoEv6BJegRnYEF2A79gOvYDl2BdEjCkqkGtwXp0LNToIskOTXzh%2FF062yJ7AAAAEDAWAAABWhJ%2BKPEIJgBFxMVP7w2QJBGHASQnOBKXKFIdUK4igKA9IEaYJg%29%3Bsrc%3Aurl%28data%3Aapplication%2Fvnd%2Ems%2Dfontobject%3Bbase64%2Cn04AAEFNAAACAAIABAAAAAAABQAAAAAAAAABAJABAAAEAExQAAAAAAAAAAIAAAAAAAAAAAEAAAAAAAAAJxJ%2FLAAAAAAAAAAAAAAAAAAAAAAAACgARwBMAFkAUABIAEkAQwBPAE4AUwAgAEgAYQBsAGYAbABpAG4AZwBzAAAADgBSAGUAZwB1AGwAYQByAAAAeABWAGUAcgBzAGkAbwBuACAAMQAuADAAMAA5ADsAUABTACAAMAAwADEALgAwADAAOQA7AGgAbwB0AGMAbwBuAHYAIAAxAC4AMAAuADcAMAA7AG0AYQBrAGUAbwB0AGYALgBsAGkAYgAyAC4ANQAuADUAOAAzADIAOQAAADgARwBMAFkAUABIAEkAQwBPAE4AUwAgAEgAYQBsAGYAbABpAG4AZwBzACAAUgBlAGcAdQBsAGEAcgAAAAAAQlNHUAAAAAAAAAAAAAAAAAAAAAADAKncAE0TAE0ZAEbuFM3pjM%2FSEdmjKHUbyow8ATBE40IvWA3vTu8LiABDQ%2BpexwUMcm1SMnNryctQSiI1K5ZnbOlXKmnVV5YvRe6RnNMFNCOs1KNVpn6yZhCJkRtVRNzEufeIq7HgSrcx4S8h%2Fv4vnrrKc6oCNxmSk2uKlZQHBii6iKFoH0746ThvkO1kJHlxjrkxs%2BLWORaDQBEtiYJIR5IB9Bi1UyL4Rmr0BNigNkMzlKQmnofBHviqVzUxwdMb3NdCn69hy%2BpRYVKGVS%2F1tnsqv4LL7wCCPZZAZPT4aCShHjHJVNuXbmMrY5LeQaGnvAkXlVrJgKRAUdFjrWEah9XebPeQMj7KS7DIBAFt8ycgC5PLGUOHSE3ErGZCiViNLL5ZARfywnCoZaKQCu6NuFX42AEeKtKUGnr%2FCm2Cy8tpFhBPMW5Fxi4Qm4TkDWh4IWFDClhU2hRWosUWqcKLlgyXB%2BlSHaWaHiWlBAR8SeSgSPCQxdVQgzUixWKSTrIQEbU94viDctkvX%2BVSjJuUmV8L4CXShI11esnp0pjWNZIyxKHS4wVQ2ime1P4RnhvGw0aDN1OLAXGERsB7buFpFGGBAre4QEQR0HOIO5oYH305G%2BKspT%2FFupEGGafCCwxSe6ZUa%2B073rXHnNdVXE6eWvibUS27XtRzkH838mYLMBmYysZTM0EM3A1fbpCBYFccN1B%2FEnCYu%2FTgCGmr7bMh8GfYL%2BBfcLvB0gRagC09w9elfldaIy%2FhNCBLRgBgtCC7jAF63wLSMAfbfAlEggYU0bUA7ACCJmTDpEmJtI78w4%2FBO7dN7JR7J7ZvbYaUbaILSQsRBiF3HGk5fEg6p9unwLvn98r%2BvnsV%2B372uf1xBLq4qU%2F45fTuqaAP%2BpssmCCCTF0mhEow8ZXZOS8D7Q85JsxZ%2BAzok7B7O%2Ff6J8AzYBySZQB%2FQHYUSA%2BEeQhEWiS6AIQzgcsDiER4MjgMBAWDV4AgQ3g1eBgIdweCQmCjJEMkJ%2BPKRWyFHHmg1Wi%2F6xzUgA0LREoKJChwnQa9B%2B5RQZRB3IlBlkAnxyQNaANwHMowzlYSMCBgnbpzvqpl0iTJNCQidDI9ZrSYNIRBhHtUa5YHMHxyGEik9hDE0AKj72AbTCaxtHPUaKZdAZSnQTyjGqGLsmBStCejApUhg4uBMU6mATujEl%2BKdDPbI6Ag4vLr%2BhjY6lbjBeoLKnZl0UZgRX8gTySOeynZVz1wOq7e1hFGYIq%2BMhrGxDLak0PrwYzSXtcuyhXEhwOYofiW%2BEcI%2Fjw8P6IY6ed%2BetAbuqKp5QIapT77LnAe505lMuqL79a0ut4rWexzFttsOsLDy7zvtQzcq3U1qabe7tB0wHWVXji%2BzDbo8x8HyIRUbXnwUcklFv51fvTymiV%2BMXLSmGH9d9%2BaXpD5X6lao41anWGig7IwIdnoBY2ht%2FpO9mClLo4NdXHAsefqWUKlXJkbqPOFhMoR4aiA1BXqhRNbB2Xwi%2B7u%2FjpAoOpKJ0UX24EsrzMfHXViakCNcKjBxuQX8BO0ZqjJ3xXzf%2B61t2VXOSgJ8xu65QKgtN6FibPmPYsXbJRHHqbgATcSZxBqGiDiU4NNNsYBsKD0MIP%2FOfKnlk%2FLkaid%2FO2NbKeuQrwOB2Gq3YHyr6ALgzym5wIBnsdC1ZkoBFZSQXChZvlesPqvK2c5oHHT3Q65jYpNxnQcGF0EHbvYqoFw60WNlXIHQF2HQB7zD6lWjZ9rVqUKBXUT6hrkZOle0RFYII0V5ZYGl1JAP0Ud1fZZMvSomBzJ710j4Me8mjQDwEre5Uv2wQfk1ifDwb5ksuJQQ3xt423lbuQjvoIQByQrNDh1JxGFkOdlJvu%2FgFtuW0wR4cgd%2BZKesSV7QkNE2kw6AV4hoIuC02LGmTomyf8PiO6CZzOTLTPQ%2BHW06H%2Btx%2BbQ8LmDYg1pTFrp2oJXgkZTyeRJZM0C8aE2LpFrNVDuhARsN543%2FFV6klQ6Tv1OoZGXLv0igKrl%2FCmJxRmX7JJbJ998VSIPQRyDBICzl4JJlYHbdql30NvYcOuZ7a10uWRrgoieOdgIm4rlq6vNOQBuqESLbXG5lzdJGHw2m0sDYmODXbYGTfSTGRKpssTO95fothJCjUGQgEL4yKoGAF%2F0SrpUDNn8CBgBcSDQByAeNkCXp4S4Ro2Xh4OeaGRgR66PVOsU8bc6TR5%2FxTcn4IVMLOkXSWiXxkZQCbvKfmoAvQaKjO3EDKwkwqHChCDEM5loQRPd5ACBki1TjF772oaQhQbQ5C0lcWXPFOzrfsDGUXGrpxasbG4iab6eByaQkQfm0VFlP0ZsDkvvqCL6QXMUwCjdMx1ZOyKhTJ7a1GWAdOUcJ8RSejxNVyGs31OKMyRyBVoZFjqIkmKlLQ5eHMeEL4MkUf23cQ%2F1SgRCJ1dk4UdBT7OoyuNgLs0oCd8RnrEIb6QdMxT2QjD4zMrJkfgx5aDMcA4orsTtKCqWb%2FVeyceqa5OGSmB28YwH4rFbkQaLoUN8OQQYnD3w2eXpI4ScQfbCUZiJ4yMOIKLyyTc7BQ4uXUw6Ee6%2FxM%2B4Y67ngNBknxIPwuppgIhFcwJyr6EIj%2BLzNj%2FmfR2vhhRlx0BILZoAYruF0caWQ7YxO66UmeguDREAFHYuC7HJviRgVO6ruJH59h%2FC%2FPkgSle8xNzZJULLWq9JMDTE2fjGE146a1Us6PZDGYle6ldWRqn%2FpdpgHKNGrGIdkRK%2BKPETT9nKT6kLyDI8xd9A1FgWmXWRAIHwZ37WyZHOVyCadJEmMVz0MadMjDrPho%2BEIochkVC2xgGiwwsQ6DMv2P7UXqT4x7CdcYGId2BJQQa85EQKmCmwcRejQ9Bm4oATENFPkxPXILHpMPUyWTI5rjNOsIlmEeMbcOCEqInpXACYQ9DDxmFo9vcmsDblcMtg4tqBerNngkIKaFJmrQAPnq1dEzsMXcwjcHdfdCibcAxxA%2Bq%2Fj9m3LM%2FO7WJka4tSidVCjsvo2lQ%2F2ewyoYyXwAYyr2PlRoR5MpgVmSUIrM3PQxXPbgjBOaDQFIyFMJvx3Pc5RSYj12ySVF9fwFPQu2e2KWVoL9q3Ayv3IzpGHUdvdPdrNUdicjsTQ2ISy7QU3DrEytIjvbzJnAkmANXjAFERA0MUoPF3%2F5KFmW14bBNOhwircYgMqoDpUMcDtCmBE82QM2YtdjVLB4kBuKho%2FbcwQdeboqfQartuU3CsCf%2BcXkgYAqp%2F0Ee3RorAZt0AvvOCSI4JICIlGlsV0bsSid%2FNIEALAAzb6HAgyWHBps6xAOwkJIGcB82CxRQq4sJf3FzA70A%2BTRqcqjEMETCoez3mkPcpnoALs0ugJY8kQwrC%2BJE5ik3w9rzrvDRjAQnqgEVvdGrNwlanR0SOKWzxOJOvLJhcd8Cl4AshACUkv9czdMkJCVQSQhp6kp7StAlpVRpK0t0SW6LHeBJnE2QchB5Ccu8kxRghZXGIgZIiSj7gEKMJDClcnX6hgoqJMwiQDigIXg3ioFLCgDgjPtYHYpsF5EiA4kcnN18MZtOrY866dEQAb0FB34OGKHGZQjwW%2FWDHA60cYFaI%2FPjpzquUqdaYGcIq%2BmLez3WLFFCtNBN2QJcrlcoELgiPku5R5dSlJFaCEqEZle1AQzAKC%2B1SotMcBNyQUFuRHRF6OlimSBgjZeTBCwLyc6A%2BP%2FoFRchXTz5ADknYJHxzrJ5pGuIKRQISU6WyKTBBjD8WozmVYWIsto1AS5rxzKlvJu4E%2FvwOiKxRtCWsDM%2BeTHUrmwrCK5BIfMzGkD%2B0Fk5LzBs0jMYXktNDblB06LMNJ09U8pzSLmo14MS0OMjcdrZ31pyQqxJJpRImlSvfYAK8inkYU52QY2FPEVsjoWewpwhRp5yAuNpkqhdb7ku9Seefl2D0B8SMTFD90xi4CSOwwZy9IKkpMtI3FmFUg3%2FkFutpQGNc3pCR7gvC4sgwbupDu3DyEN%2BW6YGLNM21jpB49irxy9BSlHrVDlnihGKHwPrbVFtc%2Bh1rVQKZduxIyojccZIIcOCmhEnC7UkY68WXKQgLi2JCDQkQWJRQuk60hZp0D3rtCTINSeY9Ej2kIKYfGxwOs4j9qMM7fYZiipzgcf7TamnehqdhsiMiCawXnz4xAbyCkLAx5EGbo3Ax1u3dUIKnTxIaxwQTHehPl3V491H0%2BbC5zgpGz7Io%2BmjdhKlPJ01EeMpM7UsRJMi1nGjmJg35i6bQBAAxjO%2FENJubU2mg3ONySEoWklCwdABETcs7ck3jgiuU9pcKKpbgn%2B3YlzV1FzIkB6pmEDOSSyDfPPlQskznctFji0kpgZjW5RZe6x9kYT4KJcXg0bNiCyif%2BpZACCyRMmYsfiKmN9tSO65F0R2OO6ytlEhY5Sj6uRKfFxw0ijJaAx%2Fk3QgnAFSq27%2F2i4GEBA%2BUvTJKK%2F9eISNvG46Em5RZfjTYLdeD8kdXHyrwId%2FDQZUaMCY4gGbke2C8vfjgV%2FY9kkRQOJIn%2FxM9INZSpiBnqX0Q9GlQPpPKAyO5y%2BW5NMPSRdBCUlmuxl40ZfMCnf2Cp044uI9WLFtCi4YVxKjuRCOBWIb4XbIsGdbo4qtMQnNOQz4XDSui7W%2FN6l54qOynCqD3DpWQ%2BmpD7C40D8BZEWGJX3tlAaZBMj1yjvDYKwCJBa201u6nBKE5UE%2B7QSEhCwrXfbRZylAaAkplhBWX50dumrElePyNMRYUrC99UmcSSNgImhFhDI4BXjMtiqkgizUGCrZ8iwFxU6fQ8GEHCFdLewwxYWxgScAYMdMLmcZR6b7rZl95eQVDGVoUKcRMM1ixXQtXNkBETZkVVPg8LoSrdetHzkuM7DjZRHP02tCxA1fmkXKF3VzfN1pc1cv%2F8lbTIkkYpqKM9VOhp65ktYk%2BQ46myFWBapDfyWUCnsnI00QTBQmuFjMZTcd0V2NQ768Fhpby04k2IzNR1wKabuGJqYWwSly6ocMFGTeeI%2BejsWDYgEvr66QgqdcIbFYDNgsm0x9UHY6SCd5%2B7tpsLpKdvhahIDyYmEJQCqMqtCF6UlrE5GXRmbu%2Bvtm3BFSxI6ND6UxIE7GsGMgWqghXxSnaRJuGFveTcK5ZVSPJyjUxe1dKgI6kNF7EZhIZs8y8FVqwEfbM0Xk2ltORVDKZZM40SD3qQoQe0orJEKwPfZwm3YPqwixhUMOndis6MhbmfvLBKjC8sKKIZKbJk8L11oNkCQzCgvjhyyEiQSuJcgCQSG4Mocfgc0Hkwcjal1UNgP0CBPikYqBIk9tONv4kLtBswH07vUCjEaHiFGlLf8MgXKzSgjp2HolRRccAOh0ILHz9qlGgIFkwAnzHJRjWFhlA7ROwINyB5HFj59PRZHFor6voq7l23EPNRwdWhgawqbivLSjRA4htEYUFkjESu67icTg5S0aW1sOkCiIysfJ9UnIWevOOLGpepcBxy1wEhd2WI3AZg7sr9WBmHWyasxMcvY%2FiOmsLtHSWNUWEGk9hScMPShasUA1AcHOtRZlqMeQ0OzYS9vQvYUjOLrzP07BUAFikcJNMi7gIxEw4pL1G54TcmmmoAQ5s7TGWErJZ2Io4yQ0ljRYhL8H5e62oDtLF8aDpnIvZ5R3GWJyAugdiiJW9hQAVTsnCBHhwu7rkBlBX6r3b7ejEY0k5GGeyKv66v%2B6dg7mcJTrWHbtMywbedYqCQ0FPwoytmSWsL8WTtChZCKKzEF7vP6De4x2BJkkniMgSdWhbeBSLtJZR9CTHetK1xb34AYIJ37OegYIoPVbXgJ%2FqDQK%2BbfCtxQRVKQu77WzOoM6SGL7MaZwCGJVk46aImai9fmam%2BWpHG%2B0BtQPWUgZ7RIAlPq6lkECUhZQ2gqWkMYKcYMYaIc4gYCDFHYa2d1nzp3%2BJ1eCBay8IYZ0wQRKGAqvCuZ%2FUgbQPyllosq%2BXtfKIZOzmeJqRazpmmoP%2F76YfkjzV2NlXTDSBYB04SVlNQsFTbGPk1t%2FI4Jktu0XSgifO2ozFOiwd%2F0SssJDn0dn4xqk4GDTTKX73%2FwQyBLdqgJ%2BWx6AQaba3BA9CKEzjtQYIfAsiYamapq80LAamYjinlKXUkxdpIDk0puXUEYzSalfRibAeDAKpNiqQ0FTwoxuGYzRnisyTotdVTclis1LHRQCy%2FqqL8oUaQzWRxilq5Mi0IJGtMY02cGLD69vGjkj3p6pGePKI8bkBv5evq8SjjyU04vJR2cQXQwSJyoinDsUJHCQ50jrFTT7yRdbdYQMB3MYCb6uBzJ9ewhXYPAIZSXfeEQBZZ3GPN3Nbhh%2FwkvAJLXnQMdi5NYYZ5GHE400GS5rXkOZSQsdZgIbzRnF9ueLnsfQ47wHAsirITnTlkCcuWWIUhJSbpM3wWhXNHvt2xUsKKMpdBSbJnBMcihkoDqAd1Zml%2FR4yrzow1Q2A5G%2Bkzo%2FRhRxQS2lCSDRV8LlYLBOOoo1bF4jwJAwKMK1tWLHlu9i0j4Ig8qVm6wE1DxXwAwQwsaBWUg2pOOol2dHxyt6npwJEdLDDVYyRc2D0HbcbLUJQj8gPevQBUBOUHXPrsAPBERICpnYESeu2OHotpXQxRGlCCtLdIsu23MhZVEoJg8Qumj%2FUMMc34IBqTKLDTp76WzL%2FdMjCxK7MjhiGjeYAC%2Fkj%2FjY%2FRde7hpSM1xChrog6yZ7OWTuD56xBJnGFE%2BpT2ElSyCnJcwVzCjkqeNLfMEJqKW0G7OFIp0G%2B9mh50I9o8k1tpCY0xYqFNIALgIfc2me4n1bmJnRZ89oepgLPT0NTMLNZsvSCZAc3TXaNB07vail36%2FdBySis4m9%2FDR8izaLJW6bWCkVgm5T%2Bius3ZXq4xI%2BGnbveLbdRwF2mNtsrE0JjYc1AXknCOrLSu7Te%2Fr4dPYMCl5qtiHNTn%2BTPbh1jCBHH%2BdMJNhwNgs3nT%2BOhQoQ0vYif56BMG6WowAcHR3DjQolxLzyVekHj00PBAaW7IIAF1EF%2BuRIWyXjQMAs2chdpaKPNaB%2BkSezYt0%2BCA04sOg5vx8Fr7Ofa9sUv87h7SLAUFSzbetCCZ9pmyLt6l6%2FTzoA1%2FZBG9bIUVHLAbi%2FkdBFgYGyGwRQGBpkqCEg2ah9UD6EedEcEL3j4y0BQQCiExEnocA3SZboh%2Bepgd3YsOkHskZwPuQ5OoyA0fTA5AXrHcUOQF%2BzkJHIA7PwCDk1gGVmGUZSSoPhNf%2BTklauz98QofOlCIQ%2FtCD4dosHYPqtPCXB3agggQQIqQJsSkB%2Bqn0rkQ1toJjON%2FOtCIB9RYv3PqRA4C4U68ZMlZn6BdgEvi2ziU%2BTQ6NIw3ej%2BAtDwMGEZk7e2IjxUWKdAxyaw9OCwSmeADTPPleyk6UhGDNXQb%2B%2BW6Uk4q6F7%2Frg6WVTo82IoCxSIsFDrav4EPHphD3u4hR53WKVvYZUwNCCeM4PMBWzK%2BEfIthZOkuAwPo5C5jgoZgn6dUdvx5rIDmd58cXXdKNfw3l%2BwM2UjgrDJeQHhbD7HW2QDoZMCujgIUkk5Fg8VCsdyjOtnGRx8wgKRPZN5dR0zPUyfGZFVihbFRniXZFOZGKPnEQzU3AnD1KfR6weHW2XS6KbPJxUkOTZsAB9vTVp3Le1F8q5l%2BDMcLiIq78jxAImD2pGFw0VHfRatScGlK6SMu8leTmhUSMy8Uhdd6xBiH3Gdman4tjQGLboJfqz6fL2WKHTmrfsKZRYX6BTDjDldKMosaSTLdQS7oDisJNqAUhw1PfTlnacCO8vl8706Km1FROgLDmudzxg%2BEWTiArtHgLsRrAXYWdB0NmToNCJdKm0KWycZQqb%2BMw76Qy29iQ5up%2FX7oyw8QZ75kP5F6iJAJz6KCmqxz8fEa%2FxnsMYcIO%2FvEkGRuMckhr4rIeLrKaXnmIzlNLxbFspOphkcnJdnz%2FChp%2FVlpj2P7jJQmQRwGnltkTV5dbF9fE3%2FfxoSqTROgq9wFUlbuYzYcasE0ouzBo%2BdDCDzxKAfhbAZYxQiHrLzV2iVexnDX%2FQnT1fsT%2Fxuhu1ui5qIytgbGmRoQkeQooO8eJNNZsf0iALur8QxZFH0nCMnjerYQqG1pIfjyVZWxhVRznmmfLG00BcBWJE6hzQWRyFknuJnXuk8A5FRDCulwrWASSNoBtR%2BCtGdkPwYN2o7DOw%2FVGlCZPusRBFXODQdUM5zeHDIVuAJBLqbO%2Ff9Qua%2BpDqEPk230Sob9lEZ8BHiCorjVghuI0lI4JDgHGRDD%2FprQ84B1pVGkIpVUAHCG%2Biz3Bn3qm2AVrYcYWhock4jso5%2BJ7HfHVj4WMIQdGctq3psBCVVzupQOEioBGA2Bk%2BUILT7%2BVoX5mdxxA5fS42gISQVi%2FHTzrgMxu0fY6hE1ocUwwbsbWcezrY2n6S8%2F6cxXkOH4prpmPuFoikTzY7T85C4T2XYlbxLglSv2uLCgFv8Quk%2FwdesUdWPeHYIH0R729JIisN9Apdd4eB10aqwXrPt%2BSu9mA8k8n1sjMwnfsfF2j3jMUzXepSHmZ%2FBfqXvzgUNQQWOXO8YEuFBh4QTYCkOAPxywpYu1VxiDyJmKVcmJPGWk%2Fgc3Pov02StyYDahwmzw3E1gYC9wkupyWfDqDSUMpCTH5e5N8B%2F%2FlHiMuIkTNw4USHrJU67bjXGqNav6PBuQSoqTxc8avHoGmvqNtXzIaoyMIQIiiUHIM64cXieouplhNYln7qgc4wBVAYR104kO%2BCvKqsg4yIUlFNThVUAKZxZt1XA34h3TCUUiXVkZ0w8Hh2R0Z5L0b4LZvPd%2Fp1gi%2F07h8qfwHrByuSxglc9cI4QIg2oqvC%2Fqm0i7tjPLTgDhoWTAKDO2ONW5oe%2B%2FeKB9vZB8K6C25yCZ9RFVMnb6NRdRjyVK57CHHSkJBfnM2%2Fj4ODUwRkqrtBBCrDsDpt8jhZdXoy%2F1BCqw3sSGhgGGy0a5Jw6BP%2FTExoCmNFYjZl248A0osgPyGEmRA%2BfAsqPVaNAfytu0vuQJ7rk3J4kTDTR2AlCHJ5cls26opZM4w3jMULh2YXKpcqGBtuleAlOZnaZGbD6DHzMd6i2oFeJ8z9XYmalg1Szd%2FocZDc1C7Y6vcALJz2lYnTXiWEr2wawtoR4g3jvWUU2Ngjd1cewtFzEvM1NiHZPeLlIXFbBPawxNgMwwAlyNSuGF3zizVeOoC9bag1qRAQKQE%2FEZBWC2J8mnXAN2aTBboZ7HewnObE8CwROudZHmUM5oZ%2FUgd%2FJZQK8lvAm43uDRAbyW8gZ%2BZGq0EVerVGUKUSm%2FIdn8AQHdR4m7bue88WBwft9mSCeMOt1ncBwziOmJYI2ZR7ewNMPiCugmSsE4EyQ%2BQATJG6qORMGd4snEzc6B4shPIo4G1T7PgSm8PY5eUkPdF8JZ0VBtadbHXoJgnEhZQaODPj2gpODKJY5Yp4DOsLBFxWbvXN755KWylJm%2BoOd4zEL9Hpubuy2gyyfxh8oEfFutnYWdfB8PdESLWYvSqbElP9qo3u6KTmkhoacDauMNNjj0oy40DFV7Ql0aZj77xfGl7TJNHnIwgqOkenruYYNo6h724%2BzUQ7%2BvkCpZB%2BpGA562hYQiDxHVWOq0oDQl%2FQsoiY%2BcuI7iWq%2FZIBtHcXJ7kks%2Bh2fCNUPA82BzjnqktNts%2BRLdk1VSu%2BtqEn7QZCCsvEqk6FkfiOYkrsw092J8jsfIuEKypNjLxrKA9kiA19mxBD2suxQKCzwXGws7kEJvlhUiV9tArLIdZW0IORcxEzdzKmjtFhsjKy%2F44XYXdI5noQoRcvjZ1RMPACRqYg2V1%2BOwOepcOknRLLFdYgTkT5UApt%2FJhLM3jeFYprZV%2BZow2g8fP%2BU68hkKFWJj2yBbKqsrp25xkZX1DAjUw52IMYWaOhab8Kp05VrdNftqwRrymWF4OQSjbdfzmRZirK8FMJELEgER2PHjEAN9pGfLhCUiTJFbd5LBkOBMaxLr%2FA1SY9dXFz4RjzoU9ExfJCmx%2FI9FKEGT3n2cmzl2X42L3Jh%2BAbQq6sA%2BSs1kitoa4TAYgKHaoybHUDJ51oETdeI%2F9ThSmjWGkyLi5QAGWhL0BG1UsTyRGRJOldKBrYJeB8ljLJHfATWTEQBXBDnQexOHTB%2BUn44zExFE4vLytcu5NwpWrUxO%2F0ZICUGM7hGABXym0V6ZvDST0E370St9MIWQOTWngeoQHUTdCJUP04spMBMS8LSker9cReVQkULFDIZDFPrhTzBl6sed9wcZQTbL%2BBDqMyaN3RJPh%2Fanbx%2BIv%2BqgQdAa3M9Z5JmvYlh4qop%2BHo1F1W5gbOE9YKLgAnWytXElU4G8GtW47lhgFE6gaSs%2Bgs37sFvi0PPVvA5dnCBgILTwoKd%2F%2BDoL9F6inlM7H4rOTzD79KJgKlZO%2FZgt22UsKhrAaXU5ZcLrAglTVKJEmNJvORGN1vqrcfSMizfpsgbIe9zno%2BgBoKVXgIL%2FVI8dB1O5o%2FR3Suez%2FgD7M781ShjKpIIORM%2FnxG%2BjjhhgPwsn2IoXsPGPqYHXA63zJ07M2GPEykQwJBYLK808qYxuIew4frk52nhCsnCYmXiR6CuapvE1IwRB4%2FQftDbEn%2BAucIr1oxrLabRj9q4ae0%2BfXkHnteAJwXRbVkR0mctVSwEbqhJiMSZUp9DNbEDMmjX22m3ABpkrPQQTP3S1sib5pD2VRKRd%2BeNAjLYyT0hGrdjWJZy24OYXRoWQAIhGBZRxuBFMjjZQhpgrWo8SiFYbojcHO8V5DyscJpLTHyx9Fimassyo5U6WNtquUMYgccaHY5amgR3PQzq3ToNM5ABnoB9kuxsebqmYZm0R9qxJbFXCQ1UPyFIbxoUraTJFDpCk0Wk9GaYJKz%2F6oHwEP0Q14lMtlddQsOAU9zlYdMVHiT7RQP3XCmWYDcHCGbVRHGnHuwzScA0BaSBOGkz3lM8CArjrBsyEoV6Ys4qgDK3ykQQPZ3hCRGNXQTNNXbEb6tDiTDLKOyMzRhCFT%2BmAUmiYbV3YQVqFVp9dorv%2BTsLeCykS2b5yyu8AV7IS9cxcL8z4Kfwp%2BxJyYLv1OsxQCZwTB4a8BZ%2F5EdxTBJthApqyfd9u3ifr%2FWILTqq5VqgwMT9SOxbSGWLQJUUWCVi4k9tho9nEsbUh7U6NUsLmkYFXOhZ0kmamaJLRNJzSj%2Fqn4Mso6zb6iLLBXoaZ6AqeWCjHQm2lztnejYYM2eubnpBdKVLORZhudH3JF1waBJKA9%2BW8EhMj3Kzf0L4vi4k6RoHh3Z5YgmSZmk6ns4fjScjAoL8GoOECgqgYEBYUGFVO4FUv4%2FYtowhEmTs0vrvlD%2FCrisnoBNDAcUi%2FteY7OctFlmARQzjOItrrlKuPO6E2Ox93L4O%2F4DcgV%2FdZ7qR3VBwVQxP1GCieA4RIpweYJ5FoYrHxqRBdJjnqbsikA2Ictbb8vE1GYIo9dacK0REgDX4smy6GAkxlH1yCGGsk%2BtgiDhNKuKu3yNrMdxafmKTF632F8Vx4BNK57GvlFisrkjN9WDAtjsWA0ENT2e2nETUb%2Fn7qwhvGnrHuf5bX6Vh%2Fn3xffU3PeHdR%2BFA92i6ufT3AlyAREoNDh6chiMWTvjKjHDeRhOa9YkOQRq1vQXEMppAQVwHCuIcV2g5rBn6GmZZpTR7vnSD6ZmhdSl176gqKTXu5E%2BYbfL0adwNtHP7dT7t7b46DVZIkzaRJOM%2BS6KcrzYVg%2BT3wSRFRQashjfU18NutrKa%2F7PXbtuJvpIjbgPeqd%2BpjmRw6YKpnANFSQcpzTZgpSNJ6J7uiagAbir%2F8tNXJ%2FOsOnRh6iuIexxrmkIneAgz8QoLmiaJ8sLQrELVK2yn3wOHp57BAZJhDZjTBzyoRAuuZ4eoxHruY1pSb7qq79cIeAdOwin4GdgMeIMHeG%2BFZWYaiUQQyC5b50zKjYw97dFjAeY2I4Bnl105Iku1y0lMA1ZHolLx19uZnRdILcXKlZGQx%2FGdEqSsMRU1BIrFqRcV1qQOOHyxOLXEGcbRtAEsuAC2V4K3p5mFJ22IDWaEkk9ttf5Izb2LkD1MnrSwztXmmD%2FQi%2FEmVEFBfiKGmftsPwVaIoZanlKndMZsIBOskFYpDOq3QUs9aSbAAtL5Dbokus2G4%2FasthNMK5UQKCOhU97oaOYNGsTah%2BjfCKsZnTRn5TbhFX8ghg8CBYt%2FBjeYYYUrtUZ5jVij%2Fop7V5SsbA4mYTOwZ46hqdpbB6Qvq3AS2HHNkC15pTDIcDNGsMPXaBidXYPHc6PJAkRh29Vx8KcgX46LoUQBhRM%2B3SW6Opll%2FwgxxsPgKJKzr5QCmwkUxNbeg6Wj34SUnEzOemSuvS2OetRCO8Tyy%2BQbSKVJcqkia%2BGvDefFwMOmgnD7h81TUtMn%2BmRpyJJ349HhAnoWFTejhpYTL9G8N2nVg1qkXBeoS9Nw2fB27t7trm7d%2FQK7Cr4uoCeOQ7%2F8JfKT77KiDzLImESHw%2F0wf73QeHu74hxv7uihi4fTX%2BXEwAyQG3264dwv17aJ5N335Vt9sdrAXhPOAv8JFvzqyYXwfx8WYJaef1gMl98JRFyl5Mv5Uo%2FoVH5ww5OzLFsiTPDns7fS6EURSSWd%2F92BxMYQ8sBaH%2Bj%2BwthQPdVgDGpTfi%2BJQIWMD8xKqULliRH01rTeyF8x8q%2FGBEEEBrAJMPf25UQwi0b8tmqRXY7kIvNkzrkvRWLnxoGYEJsz8u4oOyMp8cHyaybb1HdMCaLApUE%2B%2F7xLIZGP6H9xuSEXp1zLIdjk5nBaMuV%2FyTDRRP8Y2ww5RO6d2D94o%2B6ucWIqUAvgHIHXhZsmDhjVLczmZ3ca0Cb3PpKwt2UtHVQ0BgFJsqqTsnzZPlKahRUkEu4qmkJt%2Bkqdae76ViWe3STan69yaF9%2BfESD2lcQshLHWVu4ovItXxO69bqC5p1nZLvI8NdQB9s9UNaJGlQ5mG947ipdDA0eTIw%2FA1zEdjWquIsQXXGIVEH0thC5M%2BW9pZe7IhAVnPJkYCCXN5a32HjN6nsvokEqRS44tGIs7s2LVTvcrHAF%2BRVmI8L4HUYk4x%2B67AxSMJKqCg8zrGOgvK9kNMdDrNiUtSWuHFpC8%2Fp5qIQrEo%2FH%2B1l%2F0cAwQ2nKmpWxKcMIuHY44Y6DlkpO48tRuUGBWT0FyHwSKO72Ud%2BtJUfdaZ4CWNijzZtlRa8%2BCkmO%2FEwHYfPZFU%2FhzjFWH7vnzHRMo%2BaF9u8qHSAiEkA2HjoNQPEwHsDKOt6hOoK3Ce%2F%2B%2F9boMWDa44I6FrQhdgS7OnNaSzwxWKZMcyHi6LN4WC6sSj0qm2PSOGBTvDs%2FGWJS6SwEN%2FULwpb4LQo9fYjUfSXRwZkynUazlSpvX9e%2BG2zor8l%2BYaMxSEomDdLHGcD6YVQPegTaA74H8%2BV4WvJkFUrjMLGLlvSZQWvi8%2FQA7yzQ8GPno%2F%2F5SJHRP%2FOqKObPCo81s%2F%2B6WgLqykYpGAgQZhVDEBPXWgU%2FWzFZjKUhSFInufPRiMAUULC6T11yL45ZrRoB4DzOyJShKXaAJIBS9wzLYIoCEcJKQW8GVCx4fihqJ6mshBUXSw3wWVj3grrHQlGNGhIDNNzsxQ3M%2BGWn6ASobIWC%2BLbYOC6UpahVO13Zs2zOzZC8z7FmA05JhUGyBsF4tsG0drcggIFzgg%2Fkpf3%2BCnAXKiMgIE8Jk%2FMhpkc8DUJEUzDSnWlQFme3d0sHZDrg7LavtsEX3cHwjCYA17pMTfx8Ajw9hHscN67hyo%2BRJQ4458RmPywXykkVcW688oVUrQhahpPRvTWPnuI0B%2BSkQu7dCyvLRyFYlC1LG1gRCIvn3rwQeINzZQC2KXq31FaR9UmVV2QeGVqBHjmE%2BVMd3b1fhCynD0pQNhCG6%2FWCDbKPyE7NRQzL3BzQAJ0g09aUzcQA6mUp9iZFK6Sbp%2FYbHjo%2B%2B7%2FWj8S4YNa%2BZdqAw1hDrKWFXv9%2BzaXpf8ZTDSbiqsxnwN%2FCzK5tPkOr4tRh2kY3Bn9JtalbIOI4b3F7F1vPQMfoDcdxMS8CW9m%2FNCW%2FHILTUVWQIPiD0j1A6bo8vsv6P1hCESl2abrSJWDrq5sSzUpwoxaCU9FtJyYH4QFMxDBpkkBR6kn0LMPO%2B5EJ7Z6bCiRoPedRZ%2FP0SSdii7ZnPAtVwwHUidcdyspwncz5uq6vvm4IEDbJVLUFCn%2FLvIHfooUBTkFO130FC7CmmcrKdgDJcid9mvVzsDSibOoXtIf9k6ABle3PmIxejodc4aob0QKS432srrCMndbfD454q52V01G4q913mC5HOsTzWF4h2No1av1VbcUgWAqyoZl%2B11PoFYnNv2HwAODeNRkHj%2B8SF1fcvVBu6MrehHAZK1Gm69ICcTKizykHgGFx7QdowTVAsYEF2tVc0Z6wLryz2FI1sc5By2znJAAmINndoJiB4sfPdPrTC8RnkW7KRCwxC6YvXg5ahMlQuMpoCSXjOlBy0Kij%2BbsCYPbGp8BdCBiLmLSAkEQRaieWo1SYvZIKJGj9Ur%2FeWHjiB7SOVdqMAVmpBvfRiebsFjger7DC%2B8kRFGtNrTrnnGD2GAJb8rQCWkUPYHhwXsjNBSkE6lGWUj5QNhK0DMNM2l%2BkXRZ0KLZaGsFSIdQz%2FHXDxf3%2FTE30%2BDgBKWGWdxElyLccJfEpjsnszECNoDGZpdwdRgCixeg9L4EPhH%2BRptvRMVRaahu4cySjS3P5wxAUCPkmn%2BrhyASpmiTaiDeggaIxYBmtLZDDhiWIJaBgzfCsAGUF1Q1SFZYyXDt9skCaxJsxK2Ms65dmdp5WAZyxik%2FzbrTQk5KmgxCg%2Ff45L0jywebOWUYFJQAJia7XzCV0x89rpp%2Ff3AVWhSPyTanqmik2SkD8A3Ml4NhIGLAjBXtPShwKYfi2eXtrDuKLk4QlSyTw1ftXgwqA2jUuopDl%2B5tfUWZNwBpEPXghzbBggYCw%2Fdhy0ntds2yeHCDKkF%2FYxQjNIL%2FF%2F37jLPHCKBO9ibwYCmuxImIo0ijV2Wbg3kSN2psoe8IsABv3RNFaF9uMyCtCYtqcD%2BqNOhwMlfARQUdJ2tUX%2BMNJqOwIciWalZsmEjt07tfa8ma4cji9sqz%2BQ9hWfmMoKEbIHPOQORbhQRHIsrTYlnVTNvcq1imqmmPDdVDkJgRcTgB8Sb6epCQVmFZe%2BjGDiNJQLWnfx%2BdrTKYjm0G8yH0ZAGMWzEJhUEQ4Maimgf%2Fbkvo8PLVBsZl152y5S8%2BHRDfZIMCbYZ1WDp4yrdchOJw8k6R%2B%2F2pHmydK4NIK2PHdFPHtoLmHxRDwLFb7eB%2BM4zNZcB9NrAgjVyzLM7xyYSY13ykWfIEEd2n5%2FiYp3ZdrCf7fL%2Ben%2BsIJu2W7E30MrAgZBD1rAAbZHPgeAMtKCg3NpSpYQUDWJu9bT3V7tOKv%2BNRiJc8JAKqqgCA%2FPNRBR7ChpiEulyQApMK1AyqcWnpSOmYh6yLiWkGJ2mklCSPIqN7UypWj3dGi5MvsHQ87MrB4VFgypJaFriaHivwcHIpmyi5LhNqtem4q0n8awM19Qk8BOS0EsqGscuuydYsIGsbT5GHnERUiMpKJl4ON7qjB4fEqlGN%2FhCky89232UQCiaeWpDYCJINXjT6xl4Gc7DxRCtgV0i1ma4RgWLsNtnEBRQFqZggCLiuyEydmFd7WlogpkCw5G1x4ft2psm3KAREwVwr1Gzl6RT7FDAqpVal34ewVm3VH4qn5mjGj%2BbYL1NgfLNeXDwtmYSpwzbruDKpTjOdgiIHDVQSb5%2FzBgSMbHLkxWWgghIh9QTFSDILixVwg0Eg1puooBiHAt7DzwJ7m8i8%2Fi%2BjHvKf0QDnnHVkVTIqMvIQImOrzCJwhSR7qYB5gSwL6aWL9hERHCZc4G2%2BJrpgHNB8eCCmcIWIQ6rSdyPCyftXkDlErUkHafHRlkOIjxGbAktz75bnh50dU7YHk%2BMz7wwstg6RFZb%2BTZuSOx1qqP5C66c0mptQmzIC2dlpte7vZrauAMm%2F7RfBYkGtXWGiaWTtwvAQiq2oD4YixPLXE2khB2FRaNRDTk%2B9sZ6K74Ia9VntCpN4BhJGJMT4Z5c5FhSepRCRWmBXqx%2BwhVZC4me4saDs2iNqXMuCl6iAZflH8fscC1sTsy4PHeC%2BXYuqMBMUun5YezKbRKmEPwuK%2BCLzijPEQgfhahQswBBLfg%2FGBgBiI4QwAqzJkkyYAWtjzSg2ILgMAgqxYfwERRo3zruBL9WOryUArSD8sQOcD7fvIODJxKFS615KFPsb68USBEPPj1orNzFY2xoTtNBVTyzBhPbhFH0PI5AtlJBl2aSgNPYzxYLw7XTDBDinmVoENwiGzmngrMo8OmnRP0Z0i0Zrln9DDFcnmOoBZjABaQIbPOJYZGqX%2BRCMlDDbElcjaROLDoualmUIQ88Kekk3iM4OQrADcxi3rJguS4MOIBIgKgXrjd1WkbCdqxJk%2F4efRIFsavZA7KvvJQqp3Iid5Z0NFc5aiMRzGN3vrpBzaMy4JYde3wr96PjN90AYOIbyp6T4zj8LoE66OGcX1Ef4Z3KoWLAUF4BTg7ug%2FAbkG5UNQXAMkQezujSHeir2uTThgd3gpyzDrbnEdDRH2W7U6PeRvBX1ZFMP5RM%2BZu6UUZZD8hDPHldVWntTCNk7To8IeOW9yn2wx0gmurwqC60AOde4r3ETi5pVMSDK8wxhoGAoEX9NLWHIR33VbrbMveii2jAJlrxwytTHbWNu8Y4N8vCCyZjAX%2FpcsfwXbLze2%2BD%2Bu33OGBoJyAAL3jn3RuEcdp5If8O%2Ba4NKWvxOTyDltG0IWoHhwVGe7dKkCWFT%2B%2Btm%2BhaBCikRUUMrMhYKZJKYoVuv%2FbsJzO8DwfVIInQq3g3BYypiz8baogH3r3GwqCwFtZnz4xMjAVOYnyOi5HWbFA8n0qz1OjSpHWFzpQOpvkNETZBGpxN8ybhtqV%2FDMUxd9uFZmBfKXMCn%2FSqkWJyKPnT6lq%2B4zBZni6fYRByJn6OK%2BOgPBGRAJluwGSk4wxjOOzyce%2FPKODwRlsgrVkdcsEiYrqYdXo0Er2GXi2GQZd0tNJT6c9pK1EEJG1zgDJBoTVuCXGAU8BKTvCO%2FcEQ1Wjk3Zzuy90JX4m3O5IlxVFhYkSUwuQB2up7jhvkm%2BbddRQu5F9s0XftGEJ9JSuSk%2BZachCbdU45fEqbugzTIUokwoAKvpUQF%2FCvLbWW5BNQFqFkJg2f30E%2F48StNe5QwBg8zz3YAJ82FZoXBxXSv4QDooDo79NixyglO9AembuBcx5Re3CwOKTHebOPhkmFC7wNaWtoBhFuV4AkEuJ0J%2B1pT0tLkvFVZaNzfhs%2FKd3%2BA9YsImlO4XK4vpCo%2FelHQi%2F9gkFg07xxnuXLt21unCIpDV%2BbbRxb7FC6nWYTsMFF8%2B1LUg4JFjVt3vqbuhHmDKbgQ4e%2BRGizRiO8ky05LQGMdL2IKLSNar0kNG7lHJMaXr5mLdG3nykgj6vB%2FKVijd1ARWkFEf3yiUw1v%2FWaQivVUpIDdSNrrKbjO5NPnxz6qTTGgYg03HgPhDrCFyYZTi3XQw3HXCva39mpLNFtz8AiEhxAJHpWX13gCTAwgm9YTvMeiqetdNQv6IU0hH0G%2BZManTqDLPjyrOse7WiiwOJCG%2BJ0pZYULhN8NILulmYYvmVcV2MjAfA39sGKqGdjpiPo86fecg65UPyXDIAOyOkCx5NQsLeD4gGVjTVDwOHWkbbBW0GeNjDkcSOn2Nq4cEssP54t9D749A7M1AIOBl0Fi0sSO5v3P7LCBrM6ZwFY6kp2FX6AcbGUdybnfChHPyu6WlRZ2Fwv9YM0RMI7kISRgR8HpQSJJOyTfXj%2F6gQKuihPtiUtlCQVPohUgzfezTg8o1b3n9pNZeco1QucaoXe40Fa5JYhqdTspFmxGtW9h5ezLFZs3j%2FN46f%2BS2rjYNC2JySXrnSAFhvAkz9a5L3pza8eYKHNoPrvBRESpxYPJdKVUxBE39nJ1chrAFpy4MMkf0qKgYALctGg1DQI1kIymyeS2AJNT4X240d3IFQb%2F0jQbaHJ2YRK8A%2Bls6WMhWmpCXYG5jqapGs5%2FeOJErxi2%2F2KWVHiPellTgh%2FfNl%2F2KYPKb7DUcAg%2BmCOPQFCiU9Mq%2FWLcU1xxC8aLePFZZlE%2BPCLzf7ey46INWRw2kcXySR9FDgByXzfxiNKwDFbUSMMhALPFSedyjEVM5442GZ4hTrsAEvZxIieSHGSgkwFh%2FnFNdrrFD4tBH4Il7fW6ur4J8Xaz7RW9jgtuPEXQsYk7gcMs2neu3zJwTyUerHKSh1iTBkj2YJh1SSOZL5pLuQbFFAvyO4k1Hxg2h99MTC6cTUkbONQIAnEfGsGkNFWRbuRyyaEZInM5pij73EA9rPIUfU4XoqQpHT9THZkW%2BoKFLvpyvTBMM69tN1Ydwv1LIEhHsC%2BueVG%2Bw%2BkyCPsvV3erRikcscHjZCkccx6VrBkBRusTDDd8847GA7p2Ucy0y0HdSRN6YIBciYa4vuXcAZbQAuSEmzw%2BH%2FAuOx%2BaH%2BtBL88H57D0MsqyiZxhOEQkF%2F8DR1d2hSPMj%2FsNOa5rxcUnBgH8ictv2J%2Bcb4BA4v3MCShdZ2vtK30vAwkobnEWh7rsSyhmos3WC93Gn9C4nnAd%2FPjMMtQfyDNZsOPd6XcAsnBE%2FmRHtHEyJMzJfZFLE9OvQa0i9kUmToJ0ZxknTgdl%2FXPV8xoh0K7wNHHsnBdvFH3sv52lU7UFteseLG%2FVanIvcwycVA7%2BBE1Ulyb20BvwUWZcMTKhaCcmY3ROpvonVMV4N7yBXTL7IDtHzQ4CCcqF66LjF3xUqgErKzolLyCG6Kb7irP%2FMVTCCwGRxfrPGpMMGvPLgJ881PHMNMIO09T5ig7AzZTX%2F5PLlwnJLDAPfuHynSGhV4tPqR3gJ4kg4c06c%2FF1AcjGytKm2Yb5jwMotF7vro4YDLWlnMIpmPg36NgAZsGA0W1spfLSue4xxat0Gdwd0lqDBOgIaMANykwwDKejt5YaNtJYIkrSgu0KjIg0pznY0SCd1qlC6R19g97UrWDoYJGlrvCE05J%2F5wkjpkre727p5PTRX5FGrSBIfJqhJE%2FIS876PaHFkx9pGTH3oaY3jJRvLX9Iy3Edoar7cFvJqyUlOhAEiOSAyYgVEGkzHdug%2BoRHIEOXAExMiTSKU9A6nmRC8mp8iYhwWdP2U%2F5EkFAdPrZw03YA3gSyNUtMZeh7dDCu8pF5x0VORCTgKp07ehy7NZqKTpIC4UJJ89lnboyAfy5OyXzXtuDRbtAFjZRSyGFTpFrXwkpjSLIQIG3N0Vj4BtzK3wdlkBJrO18MNsgseR4BysJilI0wI6ZahLhBFA0XBmV8d4LUzEcNVb0xbLjLTETYN8OEVqNxkt10W614dd1FlFFVTIgB7%2FBQQp1sWlNolpIu4ekxUTBV7NmxOFKEBmmN%2BnA7pvF78%2FRII5ZHA09OAiE%2F66MF6HQ%2BqVEJCHxwymukkNvzqHEh52dULPbVasfQMgTDyBZzx4007YiKdBuUauQOt27Gmy8ISclPmEUCIcuLbkb1mzQSqIa3iE0PJh7UMYQbkpe%2BhXjTJKdldyt2mVPwywoODGJtBV1lJTgMsuSQBlDMwhEKIfrvsxGQjHPCEfNfMAY2oxvyKcKPUbQySkKG6tj9AQyEW3Q5rpaDJ5Sns9ScLKeizPRbvWYAw4bXkrZdmB7CQopCH8NAmqbuciZChHN8lVGaDbCnmddnqO1PQ4ieMYfcSiBE5zzMz%2BJV%2F4eyzrzTEShvqSGzgWimkNxLvUj86iAwcZuIkqdB0VaIB7wncLRmzHkiUQpPBIXbDDLHBlq7vp9xwuC9AiNkIptAYlG7Biyuk8ILdynuUM1cHWJgeB%2BK3wBP%2FineogxkvBNNQ4AkW0hvpBOQGFfeptF2YTR75MexYDUy7Q%2F9uocGsx41O4IZhViw%2F2FvAEuGO5g2kyXBUijAggWM08bRhXg5ijgMwDJy40QeY%2FcQpUDZiIzmvskQpO5G1zyGZA8WByjIQU4jRoFJt56behxtHUUE%2Fom7Rj2psYXGmq3llVOCgGYKNMo4pzwntITtapDqjvQtqpjaJwjHmDzSVGLxMt12gEXAdLi%2FcaHSM3FPRGRf7dB7YC%2BcD2ho6oL2zGDCkjlf%2FDFoQVl8GS%2F56wur3rdV6ggtzZW60MRB3g%2BU1W8o8cvqIpMkctiGVMzXUFI7FacFLrgtdz4mTEr4aRAaQ2AFQaNeG7GX0yOJgMRYFziXdJf24kg%2FgBQIZMG%2FYcPEllRTVNoDYR6oSJ8wQNLuihfw81UpiKPm714bZX1KYjcXJdfclCUOOpvTxr9AAJevTY4HK%2FG7F3mUc3GOAKqh60zM0v34v%2BELyhJZqhkaMA8UMMOU90f8RKEJFj7EqepBVwsRiLbwMo1J2zrE2UYJnsgIAscDmjPjnzI8a719Wxp757wqmSJBjXowhc46QN4RwKIxqEE6E5218OeK7RfcpGjWG1jD7qND%2B%2FGTk6M56Ig4yMsU6LUW1EWE%2BfIYycVV1thldSlbP6ltdC01y3KUfkobkt2q01YYMmxpKRvh1Z48uNKzP%2FIoRIZ%2FF6buOymSnW8gICitpJjKWBscSb9JJKaWkvEkqinAJ2kowKoqkqZftRqfRQlLtKoqvTRDi2vg%2FRrPD%2Fd3a09J8JhGZlEkOM6znTsoMCsuvTmywxTCDhw5dd0GJOHCMPbsj3QLkTE3MInsZsimDQ3HkvthT7U9VA4s6G07sID0FW4SHJmRGwCl%2BMu4xf0ezqeXD2PtPDnwMPo86sbwDV%2B9PWcgFcARUVYm3hrFQrHcgMElFGbSM2A1zUYA3baWfheJp2AINmTJLuoyYD%2FOwA4a6V0ChBN97E8YtDBerUECv0u0TlxR5yhJCXvJxgyM73Bb6pyq0jTFJDZ4p1Am1SA6sh8nADd1hAcGBMfq4d%2FUfwnmBqe0Jun1n1LzrgKuZMAnxA3NtCN7Klf4BH%2B14B7ibBmgt0TGUafVzI4uKlpF7v8NmgNjg90D6QE3tbx8AjSAC%2BOA1YJvclyPKgT27QpIEgVYpbPYGBsnyCNrGz9XUsCHkW1QAHgL2STZk12QGqmvAB0NFteERkvBIH7INDsNW9KKaAYyDMdBEMzJiWaJHZALqDxQDWRntumSDPcplyFiI1oDpT8wbwe01AHhW6%2BvAUUBoGhY3CT2tgwehdPqU%2F4Q7ZLYvhRl%2FogOvR9O2%2BwkkPKW5vCTjD2fHRYXONCoIl4Jh1bZY0ZE1O94mMGn%2FdFSWBWzQ%2FVYk%2BGezi46RgiDv3EshoTmMSlioUK6MQEN8qeyK6FRninyX8ZPeUWjjbMJChn0n%2FyJvrq5bh5UcCAcBYSafTFg7p0jDgrXo2QWLb3WpSOET%2FHh4oSadBTvyDo10IufLzxiMLAnbZ1vcUmj3w7BQuIXjEZXifwukVxrGa9j%2BDXfpi12m1RbzYLg9J2wFergEwOxFyD0%2FJstNK06ZN2XdZSGWxcJODpQHOq4iKqjqkJUmPu1VczL5xTGUfCgLEYyNBCCbMBFT%2FcUP6pE%2FmujnHsSDeWxMbhrNilS5MyYR0nJyzanWXBeVcEQrRIhQeJA6Xt4f2eQESNeLwmC10WJVHqwx8SSyrtAAjpGjidcj1E2FYN0LObUcFQhafUKTiGmHWRHGsFCB%2BHEXgrzJEB5bp0QiF8ZHh11nFX8AboTD0PS4O1LqF8XBks2MpjsQnwKHF6HgaKCVLJtcr0XjqFMRGfKv8tmmykhLRzu%2BvqQ02%2BKpJBjaLt9ye1Ab%2BBbEBhy4EVdIJDrL2naV0o4wU8YZ2Lq04FG1mWCKC%2BUwkXOoAjneU%2FxHplMQo2cXUlrVNqJYczgYlaOEczVCs%2FOCgkyvLmTmdaBJc1iBLuKwmr6qtRnhowngsDxhzKFAi02tf8bmET8BO27ovJKF1plJwm3b0JpMh38%2BxsrXXg7U74QUM8ZCIMOpXujHntKdaRtsgyEZl5MClMVMMMZkZLNxH9%2Bb8fH6%2Bb8Lev30A9TuEVj9CqAdmwAAHBPbfOBFEATAPZ2CS0OH1Pj%2F0Q7PFUcC8hDrxESWdfgFRm%2B7vvWbkEppHB4T%2F1ApWnlTIqQwjcPl0VgS1yHSmD0OdsCVST8CQVwuiew1Y%2Bg3QGFjNMzwRB2DSsAk26cmA8lp2wIU4p93AUBiUHFGOxOajAqD7Gm6NezNDjYzwLOaSXRBYcWipTSONHjUDXCY4mMI8XoVCR%2FRrs%2FJLKXgEx%2BqkmeDlFOD1%2FyTQNDClRuiUyKYCllfMiQiyFkmuTz2vLsBNyRW%2Bxz%2B5FElFxWB28VjYIGZ0Yd%2B5wIjkcoMaggxswbT0pCmckRAErbRlIlcOGdBo4djTNO8FAgQ%2BlT6vPS60BwTRSUAM3ddkEAZiwtEyArrkiDRnS7LJ%2B2hwbzd2YDQagSgACpsovmjil5wfPuXq3GuH0CyE7FK3M4FgRaFoIkaodORrPx1%2BJpI9psyNYIFuJogZa0%2F1AhOWdlHQxdAgbwacsHqPZo8u%2FngAH2GmaTdhYnBfSDbBfh8CHq6Bx5bttP2%2BRdM%2BMAaYaZ0Y%2FADkbNCZuAyAVQa2OcXOeICmDn9Q%2FeFkDeFQg5MgHEDXq%2FtVjj%2Bjtd26nhaaolWxs1ixSUgOBwrDhRIGOLyOVk2%2FBc0UxvseQCO2pQ2i%2BKrfhu%2FWeBovNb5dJxQtJRUDv2mCwYVpNl2efQM9xQHnK0JwLYt%2FU0Wf%2BphiA4uw8G91slC832pmOTCAoZXohg1fewCZqLBhkOUBofBWpMPsqg7XEXgPfAlDo2U5WXjtFdS87PIqClCK5nW6adCeXPkUiTGx0emOIDQqw1yFYGHEVx20xKjJVYe0O8iLmnQr3FA9nSIQilUKtJ4ZAdcTm7%2BExseJauyqo30hs%2B1qSW211A1SFAOUgDlCGq7eTIcMAeyZkV1SQJ4j%2Fe1Smbq4HcjqgFbLAGLyKxlMDMgZavK5NAYH19Olz3la%2FQCTiVelFnU6O%2FGCvykqS%2FwZJDhKN9gBtSOp%2F1SP5VRgJcoVj%2Bkmf2wBgv4gjrgARBWiURYx8xENV3bEVUAAWWD3dYDKAIWk5opaCFCMR5ZjJExiCAw7gYiSZ2rkyTce4eNMY3lfGn%2B8p6%2BvBckGlKEXnA6Eota69OxDO9oOsJoy28BXOR0UoXNRaJD5ceKdlWMJlOFzDdZNpc05tkMGQtqeNF2lttZqNco1VtwXgRstLSQ6tSPChgqtGV5h2DcDReIQadaNRR6AsAYKL5gSFsCJMgfsaZ7DpKh8mg8Wz8V7H%2BgDnLuMxaWEIUPevIbClgap4dqmVWSrPgVYCzAoZHIa5z2Ocx1D%2FGvDOEqMOKLrMefWIbSWHZ6jbgA8qVBhYNHpx0P%2BjAgN5TB3haSifDcApp6yymEi6Ij%2FGsEpDYUgcHATJUYDUAmC1SCkJ4cuZXSAP2DEpQsGUjQmKJfJOvlC2x%2FpChkOyLW7KEoMYc5FDC4v2FGqSoRWiLsbPCiyg1U5yiHZVm1XLkHMMZL11%2Fyxyw0UnGig3MFdZklN5FI%2FqiT65T%2BjOXOdO7XbgWurOAZR6Cv9uu1cm5LjkXX4xi6mWn5r5NjBS0gTliHhMZI2WNqSiSphEtiCAwnafS11JhseDGHYQ5%2BbqWiAYiAv6Jsf79%2FVUs4cIl%2Bn6%2BWOjcgB%2F2l5TreoAV2717JzZbQIR0W1cl%2FdEqCy5kJ3ZSIHuU0vBoHooEpiHeQWVkkkOqRX27eD1FWw4BfO9CJDdKoSogQi3hAAwsPRFrN5RbX7bqLdBJ9JYMohWrgJKHSjVl1sy2xAG0E3sNyO0oCbSGOxCNBRRXTXenYKuwAoDLfnDcQaCwehUOIDiHAu5m5hMpKeKM4sIo3vxACakIxKoH2YWF2QM84e6F5C5hJU4g8uxuFOlAYnqtwxmHyNEawLW%2FPhoawJDrGAP0JYWHgAVUByo%2FbGdiv2T2EMg8gsS14%2FrAdzlOYazFE7w4OzxeKiWdm3nSOnQRRKXSlVo8HEAbBfyJMKqoq%2BSCcTSx5NDtbFwNlh8VhjGGDu7JG5%2FTAGAvniQSSUog0pNzTim8Owc6QTuSKSTXlQqwV3eiEnklS3LeSXYPXGK2VgeZBqNcHG6tZHvA3vTINhV0ELuQdp3t1y9%2BogD8Kk%2FW7QoRN1UWPqM4%2BxdygkFDPLoTaumKReKiLWoPHOfY54m3qPx4c%2B4pgY3MRKKbljG8w4wvz8pxk3AqKsy4GMAkAtmRjRMsCxbb4Q2Ds0Ia9ci8cMT6DmsJG00XaHCIS%2Bo3F8YVVeikw13w%2BOEDaCYYhC0ZE54kA4jpjruBr5STWeqQG6M74HHL6TZ3lXrd99ZX%2B%2B7LhNatQaZosuxEf5yRA15S9gPeHskBIq3Gcw81AGb9%2FO53DYi%2F5CsQ51EmEh8Rkg4vOciClpy4d04eYsfr6fyQkBmtD%2BP8sNh6e%2BXYHJXT%2FlkXxT4KXU5F2sGxYyzfniMMQkb9OjDN2C8tRRgTyL7GwozH14PrEUZc6oz05Emne3Ts5EG7WolDmU8OB1LDG3VrpQxp%2BpT0KYV5dGtknU64JhabdqcVQbGZiAxQAnvN1u70y1AnmvOSPgLI6uB4AuDGhmAu3ATkJSw7OtS%2F2ToPjqkaq62%2F7WFG8advGlRRqxB9diP07JrXowKR9tpRa%2BjGJ91zxNTT1h8I2PcSfoUPtd7NejVoH03EUcqSBuFZPkMZhegHyo2ZAITovmm3zAIdGFWxoNNORiMRShgwdYwFzkPw5PA4a5MIIQpmq%2Bnsp3YMuXt%2FGkXxLx%2FP6%2BZJS0lFyz4MunC3eWSGE8xlCQrKvhKUPXr0hjpAN9ZK4PfEDrPMfMbGNWcHDzjA7ngMxTPnT7GMHar%2BgMQQ3NwHCv4zH4BIMYvzsdiERi6gebRmerTsVwZJTRsL8dkZgxgRxmpbgRcud%2BYlCIRpPwHShlUSwuipZnx9QCsEWziVazdDeKSYU5CF7UVPAhLer3CgJOQXl%2Fzh575R5rsrmRnKAzq4POFdgbYBuEviM4%2BLVC15ssLNFghbTtHWerS1hDt5s4qkLUha%2FqpZXhWh1C6lTQAqCNQnaDjS7UGFBC6wTu8yFnKJnExCnAs3Ok9yj5KpfZESQ4lTy5pTGTnkAUpxI%2ByjEldJfSo4y0QhG4i4IwkRFGcjWY8%2BEzgYYJUK7BXQksLxAww%2FYYWBMhJILB9e8ePEJ4OP7z%2B4%2FwOQDl64iOYDp26DaONPxpKtBxq%2FaTzRGarm3VkPYTLJKx6Z%2FMw2YbBGseJhPMwhhNswrIkyvV2BYzrvZbxLpKwcWJhYmFtVZ%2BlPEq91FzVp1HlQY1bZVLqeNR9SAUn6n0E28k%2FUuGkNpP1DBI5ch%2FEehZfjUQ9aE41NhETExoPT2gGQz0IhWJbEOvTQ4wgcXCHHFBhewYUiFHuhRSAUVmEHeCRQHQkXGFwkAgyzREJCVN7TRnTon36Zw3tPhx4EALwNdwDv%2BJ41YSP4B2CQqz0EFgARZ4ESgBHQgROwAVn9GTI%2BHYexTUevLUeta4%2FDqKrbMVS%2BYqb8hUwYCrlgKtmAq1YCrFgKrd4qpXiqZcKn1oqdWipjYKpWwVPVYqW6xUpVipKqFR3QKjagVEtAqHpxUMTitsnFaJOKx2cVhswq35RVpyiq9lFVNIKnOQVMkgqtYxVNxiqQjFS7GKlSIVIsQqPIhUWwioigFQ%2B%2BKkN8VHr49HDw9Ebo9EDo9DTo9Crg9BDg9%2FWx7gWx7YWwlobYrOGxWPNisAaAHEyALpkAVDIAeWAArsABVXACYuAD5cAF6wAKFQAQqgAbVAAsoAAlQAUaYAfkwAvogBWQACOgAD9AAHSAAKT4GUdMiOvFngBTwCn2AZ7Dv6B6k%2F90B8%2ByRnkV144AIBoAMTQATGgAjNAA4YABgwABZgB%2FmQCwyAVlwCguASlwCEuAQFwB4uAMlwBYuAJlQAUVAAhUD2KgdpUDaJgaRMDFJgX5MC1JgWJEAokQCWRAHxEAWkQBMRADpEAMkQAYROAEecC484DRpwBDTnwNOdw05tjTmiNOYwtswhYFwLA7BYG4LA2BYGOLAwRYFuLAsxYFQJAohIEyJAMwkAwiQC0JAJgkAeiQBkJAFokAPCQA0JABwcD4Dgc4cDdDgaYcDIDgYgUC6CgWgUClCgUYUAVBQBOFAEYMALgwAgDA9QYAdIn8AZzeBB2L5EcWrenUT1KXienEsuJJ7x5U8XlTjc1NVzUyXFTGb1LlpUtWlTDIjqwE4LsagowoCi2gJLKAkpoBgJQNpAIhNqaEoneI6kiiqQ6Go%2Fn6j0cS%2Ba2gEU8gIHJ%2BBwfgZX4GL%2BBd%2FgW34FZ%2BBS%2FgUH4FN6BTegTvoEv6BJegRnYEF2A79gOvYDl2BdEjCkqkGtwXp0LNToIskOTXzh%2FF062yJ7AAAAEDAWAAABWhJ%2BKPEIJgBFxMVP7w2QJBGHASQnOBKXKFIdUK4igKA9IEaYJg%29%20format%28%27embedded%2Dopentype%27%29%2Curl%28data%3Aapplication%2Ffont%2Dwoff%3Bbase64%2Cd09GRgABAAAAAFuAAA8AAAAAsVwAAQAAAAAAAAAAAAAAAAAAAAAAAAAAAABGRlRNAAABWAAAABwAAAAcbSqX3EdERUYAAAF0AAAAHwAAACABRAAET1MvMgAAAZQAAABFAAAAYGe5a4ljbWFwAAAB3AAAAsAAAAZy2q3jgWN2dCAAAAScAAAABAAAAAQAKAL4Z2FzcAAABKAAAAAIAAAACP%2F%2FAANnbHlmAAAEqAAATRcAAJSkfV3Cb2hlYWQAAFHAAAAANAAAADYFTS%2FYaGhlYQAAUfQAAAAcAAAAJApEBBFobXR4AABSEAAAAU8AAAN00scgYGxvY2EAAFNgAAACJwAAAjBv%2B5XObWF4cAAAVYgAAAAgAAAAIAFqANhuYW1lAABVqAAAAZ4AAAOisyygm3Bvc3QAAFdIAAAELQAACtG6o%2BU1d2ViZgAAW3gAAAAGAAAABsMYVFAAAAABAAAAAMw9os8AAAAA0HaBdQAAAADQdnOXeNpjYGRgYOADYgkGEGBiYGRgZBQDkixgHgMABUgASgB42mNgZulmnMDAysDCzMN0gYGBIQpCMy5hMGLaAeQDpRCACYkd6h3ux%2BDAoPD%2FP%2FOB%2FwJAdSIM1UBhRiQlCgyMADGWCwwAAAB42u2UP2hTQRzHf5ekaVPExv6JjW3fvTQ0sa3QLA5xylBLgyBx0gzSWEUaXbIoBBQyCQGHLqXUqYNdtIIgIg5FHJxEtwqtpbnfaV1E1KFaSvX5vVwGEbW6OPngk8%2FvvXfv7pt3v4SImojIDw6BViKxRgIVBaZwVdSv%2BxvXA%2BIuzqcog2cOkkvDNE8Lbqs74k64i%2B5Sf3u8Z2AnIRLbyVCyTflVSEXVoEqrrMqrgiqqsqqqWQ5xlAc5zWOc5TwXucxVnuE5HdQhHdFRHdNJndZZndeFLc%2FzsKJLQ%2FWV6BcrCdWkwspVKZVROaw0qUqqoqZZcJhdTnGGxznHBS5xhad5VhNWCuturBTXKZ3RObuS98pb9c57k6ql9rp2v1as5deb1r6s9q1GV2IrHSt73T631424YXzjgPwqt%2BRn%2BVG%2BlRvyirwsS%2FKCPCfPytPypDwhj8mjctRZd9acF86y89x55jxxHjkPnXstXfbt%2FpNjj%2FnwXW%2BcHa6%2FSYvZ7yEwbDYazDcIgoUGzY3h2HtqgUcs1AFPWKgTXrRQF7xkoQhRf7uF9hPFeyzUTTSwY6EoUUJY6AC8bSGMS4Ys1Au3WaiPSGGsMtkdGH2rzJgYHAaYjxIwQqtB1CnYkEZ9BM6ALOpROAfyqI%2FDBQudgidBETXuqRIooz4DV0AV9UV4GsyivkTEyMMmw1UYGdhkuAYjA5sMGMvIwCbDDRgZeAz1TXgcmDy3YeRhk%2BcOjCxsMjyAkYFNhscwMrDJ8BQ2886gXoaRhedQvyTSkDZ7uA6HLLQBI5vGntAbGHugTc53cMxC7%2BE4SKL%2BACOzNpk3YWTWJid%2BiRo5NXIKM3fBItAPW55FdJLY3FeHBDr90606JCIU9Jk%2BMs3%2FY%2F8L8jUq3y79bJ%2F0%2F%2BROoP4v9v%2F4%2Fmj%2Bi7HBXUd0%2FelU6IHfHt8Aj9EPGAAoAvgAAAAB%2F%2F8AAnjaxb0JfBvVtTA%2BdxaN1hltI1m2ZVuSJVneLVlSHCdy9oTEWchqtrBEJRAgCYEsQNhC2EsbWmpI2dqkQBoSYgKlpaQthVL0yusrpW77aEubfq%2Fly%2BujvJampSTW5Dvnzmi1E%2Bjr%2F%2F3%2BXmbu3Llz77nnbuece865DMu0MAy5jGtiOEZkOp8lTNeUwyLP%2FDH%2BrEH41ZTDHAtB5lkOowWMPiwayNiUwwTjE46AI5xwhFrINPXYn%2F7ENY0dbWHfZAiTZbL8ID%2FInAd5xz2NpIH4STpDGonHIJNE3OP1KG4ISaSNeBuITAyRLgIxoiEUhFAnmUpEiXSRSGqAQEw0kuyFUIb0k2gnGSApyBFi0il2SI5YLGb5MdFjXCey4mNHzQ7WwLGEdZiPPgYR64we8THZHAt%2BwnT84D%2Fx8YTpGPgheKH4CMEDVF9xBOIeP3EbQgGH29BGgpGkIxCMTCW9qUTA0Zsir%2BQUP1mt%2BP2KusevwIO6Bx%2FIaj8%2FOD5O0VNrZW2EsqZBWbO1skRiEKE0DdlKKaSVO5VAuRpqk8VQJAqY7ydxaK44YJvrO2EWjOoDBoFYzQbDNkON%2BUbiKoRkywMWWf1j4bEY2iIY1AeMgvmEz%2FkVo9v4FSc%2FaMZMrFbjl4zWLL0%2BY5FlyzNlEVYDudJohg8gPUP7kcB%2Fmn%2BG6cd%2B5PV4Q72dXCgocWJADBgUuDTwiXiGSyZo14HOEQ2lE6k0XDIEusexDzZOMXwt1Dutz%2BtqmxTvlskNWXXUQIbhaurum9GrePqm9Yaeabjkiqf%2BbUvzDOvb2Y1E%2BEX2DnemcTP%2FzLcuu7xjQXdAtjR0Lo5n4%2FHs%2FGtntMlysHt%2B29NXbH6se%2F%2FWbFcyu%2Br28H0MwzI30DYeYTLMXIA2EG8QlHpAsyS0EfEToR0a3utIxFPJ3kiIHCCrZ66b0e2xEmL1dM9YN%2FMwS5p01N5jMX%2FBLKt%2F1R83l0LyC29M6%2BiYxo%2FUNg%2FEF7c2WyyW5tYl8WnhWg2%2FhyySbD5UhnDyS7OcU0dnrFw%2BDfGdI7v4QfYIIzOMq9hFtY55gmvC7jZ2FK7sEdrn6IXBuucYhjsGdQ8z0yEbWkkczjjsE5hNAIZrPx2zOLZDmKNXcXtg7EMqidAEEWg%2BSJCBBNwxvxJfc%2FbZa%2BKKf%2BxoKZybnq5vaqpPTye7CiF%2BZFjxZ8%2F7Qij0hfOG%2FcowPA1rT1l4ymWnrKmxxqfErTVrpgwPlz1kC%2BOy8NMDz6c%2BIO38K%2Fx0xkPnLW8Kx6qGAoQdL%2BTD9V9rb%2B%2Fctn%2F%2Ftrxz8dUrZrD%2Fzk%2FferF0cNt1BzctmX2FZPXt%2FjnFCQNz4Ah%2FiKllGiCMs1w5Lkg0kiEwj6VTXCDKsX9rMpnvIj9pcDecXAIXMnqn2dTUbN6w0XQ9ue6FV%2FnnXCH7S3lPWGltVcLsH75ub3ab7A8M28caNrIeOr3o5Q0yFsYL80xaa0EY%2FUEczV7icUMY5pnelAkmUAXmHYjvFWFGxuqlSaow3OM%2B%2FiYY7%2Fl%2FhVELF4EjRqNR%2FbvRbOY%2BDUGzGR%2FOh3EqmE%2FugIQQguGt%2FeMYz%2F%2BL0cimjeZfQDI3phXMbMQsqH%2BCjwVz%2Fhf4idHovgVmB8gLvjbicDcC%2FNypP536E%2F9N%2FpuMibExdohBmNwyiaZdJGoigos7GpF222xrfnZhML%2F7Z%2BylaqP63Hr%2Bm7bdUkQ6%2F2cXqdfmvwixY%2Bs2ksXFeXcE%2BiX0Z%2BIow76DBNgjJ7TOdUK18iPsPflfQD%2BDPsZG2Aj9VmKMMJ4fYRrhIaxhTDR0Elh2vA6h%2FAE6xUb29mj3sjmL72petXjejPy%2Boel60M99tFduCI59N3221xe7apOvxs6aHs7vab1IqY2tv7q2xsHeHGml%2FcV06u%2F8S%2FxTjJ%2BJYc0bWEX0ukW6YmIbGkJRMdjJ9mYIH5QIdJF4hvRGyK7cC7ctImQRcUET99fGXOoft35GYLMQu%2Bg2smnkgZUrH8AL%2F9Si217IssJ916nv14ZrJrvdxLkQvrvtBcjgPC0NXOicO8Qf4mcxPqh3hgUw3DDfdvLJXngg7N3dN2zbPJSaed3OfZnMU7dvmznp3C3bruO%2BNmue0LFsy7S%2B6265%2BfCKFYdvvuW6vmlblnUI8xCXp37CrOZv4B9gauDBlYp7adcUXB5DNCwYImlXOJJKkAdvExXxVvKEYnCo%2B3eIskP9qrrfIYs71CccBjfXRC52udTHHdaP1A1ui%2FVvH1otbrLrpNXBsGX5B89QghDyimlvNB2KfkxZ5C9%2Fem3%2Bd1%2Bd%2F%2FIfFp2%2B2Oxn%2Fs%2B9n%2F79p39S3s8idN6g0yZObwJOgKUpNB3GyU0Ls0PbRzIRq4lcarLKOJBkLRzJQD4j2090XrbA7DW8K3jNF5hlGS5e4V2D17zgss4T20egOJte5iD0bReM9yjTxnQxCRj3c5kFzGJmGbNKmwGw39IJDJcXJZGMkaAB4jyJAKw0jt5IAuIE%2BA%2BU3cVAZZrq9zhDyBrU8oosuxcGNTzCKJfla7JjNVmuSb%2F%2BtuzN2H%2BX4vlB%2BPpdfMXXmuVsNiub1T34SFbjYw5itEvVi0K0Nt9pNJUMI7SLGRhf2xipfCYf8z5OdlGKayOucFeVPeS%2Fdbo3lBrbSMmwUiQN5%2Fed7g0Ds1s17IuZC5kNzM3MZ6EWCa0DtekdJfAxz%2BR%2FOX28sND7yRMTBcf%2B%2Bs8mQCQWHya4qBv%2FufeMoWyslPA9DtMxUknxkH%2FyfTnm2CMYzs%2BCq3r7PxY%2FMXomrvTEsRpfEGHa%2BWN8E1AHjElb7d06ddA7oK%2F%2B5Mdsv9EtPms0jv0Z5kf1FqPxWdFtfFr0kHfgDX0Y%2B5PRSG7RUj0tQr7rmfX8DH4G5W28kKeJLtmQsQkuwMP1pk16EV4sl7vrMJATfyUWo%2FGwEco4rh4XFQgaiUX9qxZHrMQqKnz%2Fc2d8b9TysYrAuXpP%2FRf%2FGr8b1qwwc5a%2BeuLa6S6sneNXToG2XrEJi4R5SGs8Sq2S3d97bsfCRaTdaLwKClRHt37mkudvXbjwVrLhuYeGhh56bvfQkHpk2CwvwClqgWwuBfndC3c8dwmstj81KkagcUgbfPY8Zje0W%2F82VPWJHmSq6pP8hPWpotc%2FEexDOK3qU%2BwngPhOCiO9MJRm8TJefjelrzoKnG2Bn%2B1NCUmPE4gHFmBN9jrTigRIpsACrc9Gstg58ULkp9467%2BGf%2FeFnD5%2F31lNrt2967dhrm7bzI%2BVT5m%2BfzKhvf2MzpICEm79Bopkn07lt1762adNr127LwVqQLdJ5%2BlpQDcvHPQtVY5knhYrK6q8%2FJsiP6EuhGZdFdaNszjvpqvc%2BPI0CdjN0AXsFOC3ZfALDJwr4q2Xq%2BGF%2BGNbsxUg5NLLIEXi8otcDQcUts0D8eQ1iVDRAMBTsYiNdRIxE09EIBJO9A2xqgERTaW86BUFn0OD2xFO97FAgFhF6OoQ7prYt4XwSeUgQHiJyDbeke9IdQntciLQ1FlJMaYcUNvZBg%2BFB1ubjlnRNvl3o6IEU2w7fdNPhm%2Fhh%2BFLysUu6%2B%2BDLHkOkrSHYEjH0tEPe7WdD3uyDgvAgK%2Fm4szFFR7ch0toUgBTdWHr7EpaWru6%2B6dmbbnqWEbV2EtxAsXiZAPTtGPSbHsotI2leoM8TePEqgSQprs7AGFf8kuOkPdZPXGb55POAW1d%2FjLST9v5YflasP6v%2FCO7%2BGNAPC2BMZWmsOjp2NNbfHwMCJD%2BLPVL%2BD%2FOYlWEEI%2F9jpPddOFkB5d1GSuKZYggmCCd7JUxD7EXAzxyirYnNDLdDZoFdx14kivkvGc3579Jm36reTTvDgBnaO6vzyQ6chQmlsMoIkIQ2%2BbBDWBud1Va4pcCn8CPqxlh%2FfgtG8IPaPH8C5wk6%2FnZDv69jurV5QhtwE0x2iqOsj9Mx8B9%2F0EaUdiPfOYYDCi%2Fq9jhWRuupMDEU0%2BCtX0sDFxv07T%2FK5niBPqN9%2BtQjgEc31NGCXFeMcCEuQBIc%2FBK4CO78u7EPYvl3yaEfK3vcb6qP1R2tI7vUjVDDUdKubsSrNjYKY1qBEa2P50SJoaXiksIoLiCwnxS6EBuBde87botNfdEWwYvF%2FR0%2Fu5yCqhGeEOR2ynSeyXjt6ka7neyye8kryBSWE52y%2BRBgogrXPZ8E1yIHoHIFUM%2BAbJhE7lbMtt8ApL%2BxmZW7PwbjAO0fAVoXQOuiSP%2FksIVdFZ0aulsamKUzwPZ%2FNYDMJRBPCxsBqLzqHyneXF6Ej9HlIFo7%2Bpg%2BjUb3unRmGpstGkm6etOuDBGA5wCMefp1gTHcdZlvPBXlOslvYTp1cd8UjYLVd%2FJ5awNrIOKLnIt9MD9qdrKrWCvA6ALm3QV9VrsPm60Q7%2BRHJHP%2B2hqfugo%2FMvI2H%2Fmqr4b9tFnKSRY1Y5Ek80Nm%2FWIhr1ikKnxGz9TWXrokf9xwujfvcOTtNTWnxd0F37Y2W79tteBqZ4G5qLCuomw%2BnSr28QESCRVLTyYKILGJOPfcnaIFOsewhRdvv%2BrWa%2FWih0vlbX6Zb75T5C0qNKVFvH1QL%2FvazSWgC2s6oWXXIuUxQelKiJbowuJDQViatLmLijg9CQBMg8WiPgiw3LEeYRmm5f%2BXdnvkDnxLLjMLxtvX74C3OlwPQqx4xwIdpPx38LrlDphiyWUWHWKAzzxurS%2FxTo%2BP5wGFak62ap1PVFFN4v%2Fy%2BxuR39WnIO7lsWfwgVsK17wxrs9K8ltIKuhkw7f%2F6dhK6gQokFKhWX3urrjk%2FrnI0pgfpGMeuQIUaEM7%2BGF5q2iMkCaMQwxxOzcvU0eXbsnS9XknXvP7Gtw5dwPXlFu2ecvSHEZgNDsU6x%2FGdXBYXyOQjzZReSedeEPY6nEv9gJR4oBQJtFO6Kd0fwC6BO4LNHDeBujB6dSNcUQC9zIv2LnAzGk99bUDrdFY%2B9yGFQtEo0GQPNv6vS2drj4%2B1jHbv3aJSMUWP%2BQTZrmbNTjU8wyG%2FiXNNpskybLcJ3CiTF5Ir%2BJYzmJwE0mSVhlxbtbmvweB3ulB6Til5UuUZydpgiFVeobhU0WaBqpJ198d%2B%2FXeNRTZ9%2F1OPfG7%2B2hwzd5W3D%2BhmyjsRcUg%2F%2BCavb%2B%2BVh2ls3L7zT%2FetOnHNxeerv313vzLVqPai4nJv%2BK1FC6040%2F4udw7sAb3laSg0XCkAAs0npBO6VJabS4Elk%2FU%2BD4gTXW%2Bj0wnrMlqNamq4tMIYB87tE10i0FR3LZNhJsb7%2FR561btmes8YBCRkhYNByRtKd55mqTas9FYhJnbRGHuOh3M4QTdgQSqmgRxuzGdSvZGcbMxNQGk5C3ebLjoXIOFM4l%2BWKHmLTJwRv9E8GWJ6dYvf%2FFmEyEGr%2Bgyrr1p5zrgkz0Cw2j94Hv8Jdx7dIVegBSNtgsqGsRQEYiIBoXwD0LNvQ5d7s5Z00QzwNhqZA0b%2BtMG1tQq5nd84uq8R0zPvX35G8uRaze4jcOHzz0w1%2BQ2BIRvf6J6Kgatnrbiem%2BCFvAxfkrndzD9MFPP1GWTUHclpASUkCNAQkpCCcCgDSUDAhDZ%2BCuEkgn8J7i9nMA7pA4lISappxILKfAeSAbIcSDuN2bJcfZILqeO5rLs0MnngSHYRdrHjmaz7JEsEPw51ZqDJDmUIOZIe34WaQeegNsJn1qz8AIpT3yCjyEih%2FxELkuJ0lEMYTLVCiWpo5oYMleMH6USyYJcD%2BuOe%2BkWKpn1Qns34iyYDjkSLvgnZXcgVQNeqINXr48m3iS7cjm8tedyY0f1QvTnHHdsrKby%2F%2BSSbPY8%2FNH6vpl%2FEsq3Ae4ZU1HC44KFiI9o7CEgab%2FRqHbj7s5KAg06s39ZP%2FzxI%2FmVuF%2FTbTSy%2B3Fb8If9%2Fcv7%2Bwt91yy8RfP1QXtW5RzQn7qIiZyuFM5QfJ5E9uVnqT85TanFx0lkP3ukBAMprvsRyi%2FC8NAJL1xbIIirSvnSj4O5netb4JxmNANHPssHAcHMHsFRgEug816gDBeMbdfiuRcghqYcm0%2BXxx%2F5IAEtN3fqFF3LzAXqwoT0PN0OVTNqxo8sxMkd5Ig6k79Zk7VxxX6gMLOZFQgvpW2RrMW1D0BDihaXQ9wVRoBxPLfpknmkeMtoB%2FqM9cRc9IqmMD2XUmdZ7GSRKPUZvChf8BoykriM2MnKYbOHX8R7cLdNCxSFFVQqoYswnlWtlFS2mNkhswVpZiQW1J%2FUKFfipHGlUkM6UKBhMz1istELIHJLMSctu3ugzfaVSOjKvUgc%2FTHK4Sdg2Wscz69leKIkkrwuuWiOe9yGYKQXRumkC3qbRcMwrvhjNXgdZk3RxAUEhuSPvn3nnd%2B%2BU%2F3vlVOmrJzCD8JLxV1OHRjrZifbcFDOuRNTGqdgQm1tSNJ2OcQ04YiEXuxtII1ECSQRoQGYioEsgCfchB4ghAtw7FfJre4WZ9hkVi9MtjuWqtdNDlpMrfEG9fOT6q21okg%2Be4As38MfGquNt7oUws6Ysarj1%2FefE%2Byst86YUVNvDdts3Pv5c8m%2FaP0C%2Bf8%2FQb%2BIMnGq09BgwN01oIOAnAdagI8mBSrqk1gxTDUBOtk2ousEtBH2z4Ir2d3f6k8PXXVlt2qN9RODxRuoJT%2Fv27wm09jRYVc%2Fe%2B%2Biyx2tyzJb%2Fn3J0htXP87eSsQaf2Ly0s6Zmxela88REy1cf4273mI3iXNJ7KxrZibOm9xm6rl4fqy%2Ft27smU8tOfdW2ucBzg2UfmOIVyLIl3kpYlwphDISTXJXsctmiDtN7fNV6zelgxwnWxsVr83Aj%2FS5ki1jL%2Fa0GC6%2B2L6Um%2BaoddlNFuj%2BbJ8mH%2FiaLh8I0%2FU51NspIEfq0dohwyFXKgm4NggwQ4rRhCOUFtxxo8XnitT4cnGfT93IS8FaT85XE3H5LMY4zIEPL1hw443wz%2B1UmhTJyJGxZzw%2BwsKkKZgUiVtKOKMEb2AKHTv61FNc01PQFwKnvsZ%2F9pPA4RKTASWahmh%2B8MxwzHxKy74IRn5LGRjsPUUwTu64UYNY38caqd7HKucZ%2FtHnODtENw%2F2UfHRMaq1UUPDJQ0OKkWCeet5fYOhII1VRz8%2B%2FElg5j4Gxur3J8o2PJ4rg%2B2d08T%2FfwEzSVbyZ9XPro95T477lRKqUSRXQnauHNsISAl27oWi6Fv9z48JMv8r%2FaMMj8onCP%2FDuDZOuN%2BGPPr%2F%2Bp7bx%2B7JlbYdppcNhzKU%2F1Px5aiaGDn%2Fs1iGMaBcleKUo%2Fv9rcxkZj7DBEKOfrayytXNLYiUdBY%2BpleQXdnscKlQcpzuWluxsieeyuXIK6SdxozitWyGOV3vOHHjguyCQ6fpIYy2JwvrQEF%2FQa9Pdf%2FQqOSqCiE%2FEE1%2FXIVKTc2tzWbHnimrEd%2BVyz311Ml3P0GVTj7PD5aDnsvCvH36alEaPMePcMegXs7x8igTu4B9v7G9vTHvhCu%2FkzIdx%2BBxC0ay9zRSvoS0F2lIxI%2BX7klU63I40gLQ3w5ep5na%2BSFnba3z5D64zv%2BQtM4n4ffG3tq4aNHGRfxgrXPMim%2B5487abL7xhdseIRn1KDl%2B7aINixdv0OD%2BJSPwKf5%2BxoP6aiTeQIDVlIhMcL1H5R9PYXvprs3fv2bO7MOplCmweuiq2JRZ1zz%2B9a%2Fv2PH1Hfz9236w%2BZrPXvWfAxlj4NLLHpq3c%2FPQ3uvmvbrjG7fe%2Bo2y%2FcLdtE6VUlXi0ASb1VLUBVSUWSU4HdvAraTyS8xzM8NxvxFkXV6pUVRiJwcgC5zEeht4rwcp7ki0k41G0qlQhG1Vzlq8alEmnFi58caB5Q9vn988MLhqyVlHvLEWjtQFeupdiocF%2FtkkOGPW2ibWaBTkeZ%2FdvPWazXfOnnvL6jkRXpi85sFzZt%2B55ZptW3bl1cCCHZPD06MhySha7UFzjcjbp8fOecFCirzAG%2FyVjBX6OFIaadSjQq1nNhyIe8tVbaaSdHlXIWKacMeuZA1uxS95zILhyrxAdsXTL6m7kNQlx2P9uZf2qhufePFFbpI6%2FOU0WcP99RrCsrwseVot5mtytpf6Y0gm9sdeyKnPQ7onyK4nXlR%2Frg7H95M1upzu89DH6pgUcikoiihJ6NJKmRxV1x%2BMJiOA3YwhDRQrWU0u%2F0rvq0VYXnyCwsLeTJYBq3dAtJDavuzyoVpzZ99Z0%2Ba0uoiFH%2FxcqgDR7rUFeOrUn6Cywb8ZeNMbhLV5ugP9l0zv9UN5b5mFkjzxUcpPJCn3V402pRxtJd2GrnLdhtVk9ZSZh9W91fCSH5B7ofxPiWL%2Bj3D%2FuwhBRdyAyozeZwvQzs79soi%2BBKSnafLviZCcfrpBpLyimfLfTyJtbyruIQKD01tUwJyKEo%2FybaxkSNFUMdMkhQoJyRBQFhnUkDQSXhTM%2B3NmY0EDM7ffLIjqWEGt8lCO6mLia3PukFnghosJD5p5SIho%2FVDkzQfLE%2BIrYoJXkD19pdP7OwG%2FvoIUtagiWiZ4PAFTHHlTVhRZ7dYmPar%2BNJ%2B8JhmR6DFK5DV1foHoLNO%2FpHrvZfmWZ15RQlwvoVDKhCWNK3CCch9lfFBuAqUgpFSShmNaPj%2Bi5%2B%2BWZfKeViJfW5HnUakVL4UCNVkA4%2BETfIqx4B5xSaP2L1yn0zn2ltPn4%2BOqZGmwwEVCaCSqG53ldtL1oLGAhdMLd09MpCCF6tD6ZnAZBY9hDaYsP0jzZ0j5ZjKsF4i1UmLuhbJMCnYJPt5VwFNvmZawXjEvLJqIH8STonZjq7BZ8gKgR20C9MDFqJAX1H64QW2NEup6qgzLP8cvppL%2FNNTOBTCJABOHeWoXzLhw4Wuy7gaBtjKr9kgKq8ZlRYBS32Lpxc8vIhpNDTfyNXWybMJbn2RyQ5EmWc2QF9wmSZ0KYCE%2BcPuYO6b15Uotj2Kd4MItLS7gtFbkTdrFND6pvEZqv5Yv7jXAus7Pg7avo7KDot50NX3CPkP%2BKps8J9%2F3mGQIteY%2FLGPC%2BL7872SPR2br5fy8MtKBMHedGuM28%2FMZmPJMrGgi3Gb1S%2BSi1%2FL%2FzrZwO9XH1ce%2Fz7ZQ1WSoY%2F%2BpMb5FT4ua0Wm%2BJf%2F298nFmChEQ%2BTi71est4mq9VYI6RsymoRJKYidElT2FGnDTZvqtfhGAFTbeqEw68GqtfmbVa%2F1IFO1%2FjdWr%2F8BDRRtQh9XNjubEm4aWVpVonpTGR7PVGc%2BKJNoBIWF7kYi4gUV3r1U6723i6TxUl3n3%2FtM27aZfKb7THiHW9VzFSwHJ05VfK6Ar7kaB0XgPPE0BSkSFKsBUpaLihEWoA9wBt8qirh2VSOkZwXEwyrxZ5jyt2rJmSo9gX7cg6jsEUGJU9z9xJPOEM3uQQxKgkh35DNATnVyrmJ3mbCNyIB%2Fyox4wH1bg2DwN7q9kov4pFqny8oSm3RQbGgJ1QQTs6ZMLilOVYJ9v6Wha3HcJ9jddsXp9YhGUXLXt%2FqMDnvLpPNTXfNa60z5%2FyjXQOMq%2BlNmwh5egpYrdfZQZV9rI47xlRkuyTjpzsmCBSWNkAXVoK8sgYWqQJWbo1RLo6QH0YW6pxqfCnRgkd%2BRiFjUQUQ7poIaYoakgXxwFd9BuuI38H1xBxXSFb%2FpBDIKQFn7YB3dB36l7sG1FLaKiBdp1KxLvfswap%2F30lnVESgNnvjbUoT6w9N%2BXoio0qcYOIM%2Bheg940YimsucQVvli9NEcft2UZwGQwLuilj1fFr1i3NP94X%2BPE7Hpvtj6lBJfJ4R6NvWiaL6MgzWHxiN66DExa%2BdAdAbMYX6HVF8A%2B7rjEZIXAVbDe7PVI9rmN69JOLV1DOSvRPxWNPZBZf%2FNf%2BNy65BhYxxxV%2B77XJ2wfQ389%2FIQPgajXbwMsuAz%2F0IaQcXJavKbRqR2IqyZruXjVC2%2Bhdee%2F5vdnYOedpmVtR3NGXldxSzDSIiBVpkGb9by89UpEPKrSLZmyFDzMab%2FwXl2CNe7s%2FqCtTvWgG5kpBmCBlSzDS%2Fr8N4uwBwohRW63JTS1y32f0TQsPfXVGEHQrV8%2FNCfiOUVirYcBbIeA2%2BiF68rQIo3B%2FS628vYESr79ehzS7Q9LEL9UXmik9XVHb1yBO3Ngvt5935%2Bk1efkV51mzzrM0LL3%2F20avnwMeKuWyOUZg2TasSqZ%2BKcZQiOn1Iu2Vh497ALUVZiCKt%2Fgh6IvTIj1ZLRjWAkpHKOKovNwp00eqPROiAbiNEKieXwMLcXhVJ1%2FuzmLP4tfxaHR59cBdJVG1kTAgl9ze9QKUEQ946Hkb%2BokJ5JRDyf54Axur1D%2BWS49cLr0tTPEu7UmXrxcSr3XNvumv4yXzInXKH4F7Tc7p17Zt%2Bt%2FqW2%2B93k063X7VW6lALxTY7i1nBXMxcxmzQbabxz%2BtJo%2BwijYaIGMNS8AoSMgAPt84DdHOoMPfjXhF%2BkuH1tZvuFQrRCN07xGcXRX9MYxYchDe5BcHj%2BZ4i%2B42WyPc8Xofi7bbZJN5nJLJ5qr6IqRtzqNlM17SpFsnkEyTWoABEjz4JXOQvzWYuwdnV5LNGOwTM5v9r4RpQ8ZXsYodks3o31JBlzbYtNotisnm22MxiwGFXam5oN1n0TA%2FhRvshvTSDwHff4nNzRo9Dum6PaJbMXzDz%2Bx%2BFkj4L4bFNBb1asqsgH7Dyh4DvbkPtf5yMDKzEwyoaESMSNS9P9gJVA3%2FRTlwoMwZvxECFWxIPNw9gi01nOHjP32esZTtmXHnxvZd8ZtakqQ7ekajbXetpNa6ocTVxJtY%2BuSe69OLz77zh5bDR3xjZMzUz6fxrz1nqrZGcHQHfPVefN%2BfiK86LeXj%2BSc5lPKy%2Bk%2FvCUI%2FDaLFYCWHr6nbXuILTIsb5imNKY%2FrCm28fSMxPhkN1XbNMNZGuqwOBhtTSxWuTk6bw0ZaG86b1hKddePOKuBvmiguYBn4T%2FyOqOyGRBt7bKUI1GjioBC8aUKwF7Q319UgcmtFGIzCJGBqwQij0ynDsfdFGc3TS3BlNfJ25xmzniMkpXXTPvCaD3ZaZvyzjmZdudBostmhb0ORZNN2sJBeed1HXkrUsywueQH%2BL0eCPxmsa5ZpgRJSDZ11yDv%2Bjmbd86vxZfc1WcZJ3UkMq1BOOOVtvu%2F%2BpB%2Ben186d3GTwWAw2jheaJs09%2F%2BLNfZft37DALyrNj1wABMuUKbODyTVnT%2FKYbJ3Tpq8IrNh92dkxOj5P%2FYpZx4%2FycyiVcDYdn4JbEoKdQi9054iBKsygLW46FRGxAb0NPNCm8BSNCPjoKcj6EAus4SuP3rB%2BcV99%2FeTF6294dA8%2BTK6v74MHVpYNRt%2FI30e8QGTOOdfGWzzxcy%2B87a7bLjw37rHw1nPzp0KyyRSeZO%2BQQhInt3dYgvycjrPOv%2BT8s1rptaP84VeywdWX2T4ysr0%2F7TLIs6%2Bx9zib56ye1dM9e%2FXsZmePY3NDs9zlnNVt4%2BWgHJbbz3Livg4P9WWgviOMm4kCRT6I8vw0NbUUEnFvOuFKoxQW1gTsvFirsF5pb7qTUCx4i7VmtToveaDxvK9uOaedVvPRpVOnNz0Q6bry7uiSdQ8t7Vy4JQKVS%2BXPplV2ts4bvCwZu%2BKzgITtxepaPRzWdpv74muvv6RO0SorX6cu%2FdqKn%2FXWnrtp%2FZragz13DUCl5myiFW2Ycvb0PtsXnU%2Btx8pvLFbUspLX68mdegwmOif%2FNPDONajTGoUh6tU56HBJCTBASVvNUB5VIiKpc9kd7kludodSFz7xQbiOmMk5dOYk56gzL6uaf7N8a6MQOHm0ae6snZpFDfuT3%2FjdYzjzwkXXIVHoXNuCfQslQZqBZjTsoHMqrkE4jaYdgkGz2ATOgB3cPkSukD01DnV3ttb1wx%2B6arPqbkcNAHoFPzKUUQ%2BqL0k97pjbZv1I%2FegC9zTFbrrlFpNdmea%2BgIgfWW3wqkcis8ky5FAcRd1If5nNZrl2FFpungc8wpoCl1BpQV%2FScS%2BzjlASyUTVv%2FAJ46gkJI4bHX4lTnloctxPZE1ckS3%2BjG2fKIjkQFyzuo8jvYQG1OrGvJPSTu%2FnSp9PHNTl4z5hK%2F8gtXVKF6gEKiglgcKiRlCESsQCV5QIlKWKpr34lt%2FwkSx%2FJCmP5%2FcBKQfl%2F5gd%2BrOS%2F%2Bp91%2F%2BYCg5CXK2W4M9fu%2B%2F6xxX%2BvnelVuldIDCG0VQTpU9Dw4pRfei%2B6zWx0MLie0gPbyrkmRU7OwT16JGeyXLHqOLqAfVN1GPlBzWtFNzj0TRTCjogtP1NjIvu5habN5Aoa1k66wGpqriVetJgiGdwDZtKhnN0y4n9sXYnsqGmZfDSR15%2B5NLBlhoDaedEm7sxmpqRija6ZEEg2EAnTiAC8IrmFbGz1q08P9PSkjl%2F5bqzYqT9hMmptEXDgTqP3Wiye%2BsD4Wir4jCeoHbbp5hRfpB7BakUIppIlPCD30dR1GtslDz8OsqbXmejFC%2Fv8wu5X2myq7SJ8Avzv9DFUJySf5uNvq4%2BTi7W9D%2FOZrLChdwxmPNiBRqVjnpK%2FaGxRCDspVYKAW9AN1JANoo8wP4BJUlGqdgw6m1qPQ2QW3%2BOfU5%2FieLS%2FNuKpDU3uf8bcAXyBal5jMR2NEAbPAZt0K3hvxHBEDlUxfIGcD%2BN2gNSNx36nfqlAYow0puatNpRz0e4W2oahKzQHsjf2c16ad%2F3t2KTtPobnX6D8C8pd0MDP%2BKx7wnXqGGlLQcvikMErm6TmfsuxJXbSAxqNjOogJLQBLiKEHAE%2BJGTS3JoEhTrz8%2FCB%2B5YlupJ58aOat8Kv4JvregxwcU5Cp8GFAFm1FyOfto6GS2m1NGTS6CPNKkbsTdCBlnN9onMho55BX8IJZtEQ35lk%2BhtwN5A0V3RCPoD%2FyXAcv6pAtbZczRUA64JmcUf4q7Q89ZHLeJVZ5D1Ps%2Ft%2B0iCT3AHVtZC7JDCXfR7OSb%2FXja5H3zQbZL1B%2BULX1BMTEk3AseSpmnKEK4T9ekMIidUCRQFfcbj7z8gNLvzF7mbhQN8h6ZbRset%2BnQWdS%2FZX3k7WpS8P9sfo0iGS64wV516pOhjI6TZ2dApgI5%2BLhxywYoWxKUrykKJsIoDsR4mSrCTg0egMPnLW%2F3Q5Nn8BZEuzqEI7HK3n0%2BzFmuO3TtWQ5WJoG9YqCD6Gc32SxnbnVPfsxvrFXK2dILl7bLthDp6glhcsfp4bYvbSmj%2FmQ94uBTw0E73x2jbNRCvC6VL6GCFDwU7eWQDcC5FY5s0slieRDwtAbRsbLXbaXAuu14e2OJw1dc6jQ3ZdY8v7rv2%2FBWZLqvFWVvvcmwZkK9f5jS4muO9yR5res4kfkRxhV03L1RfPOiPtYi8pd7jNEsOpyTwxpaY%2FyCZu%2FAmd5Or9uS3DYaeqVOhH7gZN%2F8I%2Fwi1fEuLXvyNivibjuKvN%2B1Nc01HF%2F3h%2Bef%2FsOhox8MPd5SFucPjorQwXT%2BytA8EmA5mamHNFDVhBI5pjZbQpugBNkO8MvRub8KVDKST1Wag7D3xlin1ZF7LFP%2F79nbvCXFOY%2BPUjrT7%2FotsPXXZ4exdPzuhZuL5LUXVAn7k7PbhG89uz3b41X01gbjP1xwlu5rrvvf9%2Bpbs6E%2FVu7Nk642%2FPYRaAiUBdrmO6CDTBLPQFA1ur0uXoBR1INDMkypKpoTqnSMx5GiEdTEaSHLs0Alvu%2F19%2F5QW9Rv1U1ridT22i%2B53pzumbs%2BXFFXYC%2B%2BCGsTj5JUT%2FGCgRt3n78i2n71FHG4%2Fu6X%2B%2B9%2Braya7os3ZbDmgWfXun44e%2Bu2NZKuGZ0HiF8M4TlMPR%2BEU6rPKRJ8wOU2RFUFLex3egEsz3YqEAq0cqhAAW19dBZIlVzR61tuIdTnpXH7l%2BuXrbjPUyep%2B8cl6aXKWhPHpDcXl9KiTWDNr4mBQc8Tq%2BNzK%2FOKSbsfl79o9G20R%2BbrBXYvUg0rLHhtrc4TN81TTOWSZ0gL1ZVlOYH2ery%2F7XVUjFMbzYpg7UswcqJPQwBd0LKLabJ8IaCr2otcjSkIrGwootKECaUd4XH1%2BSdazRrfddkBU98t1htvWrbjqSqjaCguxrffM%2F5zDCpBALUycmajhd%2BR6ww4SWafuZ5eU%2BtPid4lgd3gt%2Bb%2FY9rQoZNmiXYPXyRHbRs8zX%2Ff4WIFjWZJtUdSD55AP3xtXH%2BZipC0EqdBGDA4CoYEU6gRLGPU11QhkLTBiEYPiqOeQgwTCl9aok1Qr5pFf71qEeNxjy%2F8F0GoqYPv75Yh9j3x4DuJ%2BuEzHRpAq2lMqb%2BqfTdiq6kGtzfOWsv0c7lSeMXDHBDe1MT%2BLUgx0Pg%2Fp87u2UicdIvqQi8DkxhcUwUXCedMpb4NQjwY3npTmgsURJavLwCRyEcN2HfWsDVGfv%2Fu9ZUWUx%2BPYFueUKwaNvbtu%2BXps3eVWbN1GcgVrdMnWJ7WmJz9SD66EBidag0NF1Ukep0t5A7sFCWdhzvYwHv6L%2FBehXuHqfaBwBEU7hfVLcXvS4VQv%2BT%2FvaSIl7cbeMc7ekv9i8S3e1L5xxpvMGcu1EYPbKyCiijjGXcDKckm43PqU2qNWlXusZMiqF82cuVzolUHN9NNR0HZPxFPV9V0wLtvq%2Bk4DqOwVWDlzuQLVdqFiP08cRX7aRlBVfR8cb55bWe5LExnlcsDp1vAP8Q9BucPMk1Ulh4GnN0SAdxcNHv3q9ohx1Ati4S%2FtkWjIDe3hQdkUGrGRaFBiUdiTSkI41UkMuuQHP%2BEaSQYlPQTFWJF03BNPpTu5KFAdkWgDukzsZKMG0Q1TAQQglScOaP%2FdsZ8%2BfP75D%2F9Uu5Gs3FY%2F2SxPld0DHOciXI9gqjcEidXjE%2B3BLosy0OcX3T7O5g65ROGyzQ2BZs7WbZVnO5ydLe32hMwTQ4wnnKXW6XW5LAa7oaXOIHoUl0FgLQLH2by8wSTWeAx2Y5PDazK3BqZbeJZwXGPaYhX87ZNszoDdaRxotXO1nNlpdvAPFWHDm8PqEE0sZxDEqGzxisFNnuCWetPcGrObN0p23tTZwMuRVodSV8%2BLTrOV3eRvzjQZiSjaLYS1WEJe0kNsJlZu9LFun7%2B%2BwW4gRDRbaxw2nrOGm%2BxOj9cmtbp9ZqeTM1m8UXfQQCSTVSQox6pvtjot%2FFpHvIUjJovFEoYvHYV9C5Y%2FxN9OfcalvII37UEhTbTg%2FAQIaPb4Vz6j5u8%2FaViycMod%2FfkDcpu8QZbZoeBi%2FvbzP3XPsZvOubMtaPHkD9jt6%2BU2O7vqU%2F9C9SMvgrXpQNG%2FE0oJxun%2BCiElUa0IKQSUwERxOntKSV7ekcuh9VBZBBo3VUcB58ofKBHCwLyf9qFosz9Ibf8dGqwaBMjRig4SGOZ2UkWI7UiO9OfUPdxOYFApUZyfpY7mgEc5rtNGGk2H1lPhAk1Hp%2FVAMqQEHEUfEYkkUQq1JMdzsX7kklRrTrUi1wMcDjmu1YYfATj7Y%2BpGpPEBXuoQIj8rR9mgCl4C9yqmF7xnVWxGVniNqtpVmXBvQ6iwni5YQ8a1jYrXtc2J13HvgkvqWxuva1sbr%2BP2S5ceKGyBwDv2DbrToe1u6BkAJV7xnVLUaq0sJB8pFqcUIPi3yuwxi4JuLr%2BP30f3OkPQ72aO0xYo3%2FEsmO3QO5qEF8S0qQH0UsKXv0brnl9%2B8M7jF174%2BDsfvPOl1au%2FRL5%2F9DsbNnwHL2pHR1NTRxMZhJtHktOOxLxErPF6YlLvpC9YP73x%2B4ofw%2B3xVdrHcDE0dQQCmCRgvt9b35xINDf1CDcRSfJ%2BpYl%2BSf8YcurfmXP5F%2Fkj6J82jNsrkWiEuhVlgFfyNkB3S5MUzLhoNiwSCYcxQ7Ui4J0Xh7fmqRbaPa1tzujxkBRlsEHy0%2FOM4pYLPb7g9O6BQJN6l9zQ0OGyCaZz0vMTbHOzXfQ7a2tsterTcqxeInODoemdktw%2B1SbVhKwtW9ffe8VKadK0OVuC3bWzyKm5LeddsWTeorWyY9IMtUFutdu5g%2BRn533qkocdvLs2HmhU75br%2FMmWtD8zA3OP2t1ea636jEzqYxJZGAwFiDEd61oTsrRuW3%2F3pYNi3bS%2BRd%2BGjOfVpAPNd6y64Gsz1GaZleWIPoYL%2Fv9mTeQBENVEguiF1aC4YeXxFETw6QyPfn0m9g8IrMFAvKM1EI11DARnbqibHk%2FIojy5rSdgCyZi06y8sS024PeuO4MfwQ5Y9yKRZCqyYaF30vzeHlmUprR21tR0t0yz8KZY66zWuGvxVQB%2F36kP%2BK38t2Hu6NQ9SFJfw0AdpqPEK2qTMpf2VCqJwqPoJezTL824b8akoL%2Bx03nhh%2BoNo5e77psxg9Q5LzebIKD%2BfsY34f2MtB9fk9v5b8PT6tYrgv4kRPwd0q9z3gdJSJ0653KjCYPwCaR5aUY63eW48O%2Fkdo33yxX9wCiMv2QTrk8eGSI6Ag6moG9t2P%2FF7GRNlDjl0gw7pJ5aOXXqyqn8SENnXBmbSwUYLyqJjv3UmY1nKr4t80no0faXsaIEiF%2FBRaIBnItSce4OUif7W6Vm9T9H1X9Vj71BEm%2BRdmIJQST%2FZfVdudUvh9S%2FqqNvqT98g9SQ3lHibZY0mRVHooyDN%2FFHmTgzjdozKw28NwQ0hwN6BCoPKaEk3YtKwNhwRLXuk076CGoZNXDQcRwZvreTZY9EZi%2Bd0s4%2Bztv8iei04JQl6ZbDD2eHV7X4uHuFVfPrOmcs6m6Kr7hssr%2B1VZFcEZ%2FPdJkn1hOs8SXS%2FNFFgqt94PIZzZ3tdaL6Q5vo6piSzdy737pwsX1VyxUrF15iJ4uNkq%2Brbyg1Z%2BO8VsNC1UmcvORPRfxtPrfRwL2p%2FoA1eZp6Z%2FaGffoewaXcA%2FxBlKlQLfhQL%2FoPgBGP3qsA7IQS8qDVNswHKRSheDUvA3Q7MZoRcJMxlEygujn1QdyzfPfq3dEp%2FbXh5e5YXW2Ngfvza0ZF6UgFL%2FE0fTq4LBlvTE2qb%2FKuuzYSXVnjTfM1osvqMHVbm9950quIZlbqaL6YP7jk3kUtA0GnX2nvq53f3WoSsvEdDRnULgo2fN7lNZJgI8%2FVWi33c3bBZnGY05%2Bdm%2B3qc7fNmj4YGKLj2nfqFP%2Bg7jdDlxEV5XsJQZP6hYrS1l0VQr4c69Xueixp90gnZPmE5OF22j%2BSYEWHlZ0K%2FHgsh%2FZtsbh6h2DNRlvv6jJh9XaJaHCZDiUDKNTMkvb8vsqCyf3ZNdSmO0fa0Y4baJTtpbKzuVzeeSI7fCKr2Z0WypapnXJ4gnoWy3PoUIlIQ1TXdqhQJIXp9Wx5fYdpeWh2TY5D%2BYVyKd0jw3iumwi%2FBC3cEy4o83QlZnW79MrCgCjbhWXBlRZVVZZv4rIKpXC01HFlHdHLoeWVl6UVc%2FJ5uGm6CViW5mulYMk%2BHqNYr0AyUPivLg2oMs2MPqtuhHyRyiwvNJej1Br%2BfcLyoAyu8D9B7bgmzUqfFobF5nKnK4%2Bt8MPJkI%2FxHUNWk117jugWF%2BxazTAALQn6%2BUE9lhoI5ApGA%2FiuJOsrlNP28SVVuBVajXmircLel46w2bJS1Q0Ft0KDuikDFL%2F3pYrid1Q4FvofwRIo4R9h2ftSwc6jHAMqLcCql8YPHtlzGoByNXYN6v8hXnRaOhUvx0sVLCexwupGDR4NOYC7PePa5keIPACnuAdD7dEadRuTIiS6Lb7uskb381My5yjzF8lGCjBRqdwrWJCagfB3yCy7XT1i92hbcZ5Ci1FJkgYMDf6n%2BjspIsHFjJrTOdzSMuOa9DbDcj%2FnH9N9bIoGVgzHPWIQuFuYtaMRaq8eCKI0gEF6lPOZjBz3EEvaaxwSUT9U%2F8JbJZPJJLBLolH1La%2FRbF9AbC8JJjv%2FmMnssKjLRBJyqj9QXxNko0Ux%2FX79epfiXkm6fmKwF%2Fen1HLc6LxloXWKvGa5rVCVL83VuiPcDEX%2FK5pTXOxHfx6HHB0t2FI0qI2rCZFTrvPWU67zVuS%2FkTsLnc7IKhFg30e4FOkqNSfH5PtkmUy6Cpiv%2F36k2sbqCeCFNa%2BURpoY0sZoYmCgCr3qgZz6s8I0gP1bYiR%2BD79H56NOz0EVWCTy2%2FfffvSCCx59W7uRV9995eqrX8GLesOXNm360iZ%2BT%2FEl3uZqL%2BFyzSZ8XxpTiI%2FG0nkT4zznFZ0t4ipMz5v4q9ssqbdKUZt6u82knPCrt6PZwsnn0XySVnyPR1ZXAn72yx48bWJsu7apnI3Hy8bygUK5Js32qcytapqgmn95uexccj205vGgJ%2BeuOeG2SORmKZr%2FqKzcx9SFctMJdwMUFZDJITs7dnOp1EKZCxg304Cevyfya%2BvlKqv6aXK1qIj3imL%2BL6hL%2ByvUlFfE0VKZ7E8gBY3M%2F8VoJCFgizH1W6VyC76nH6b7jiibYVxUmVIEspry%2FLgZIlCeP11Z4zs%2FAwvVwtGFEut5S1JY4lfyT0N%2FevOLo%2BrUEgjcqc9IkGpQbv3iW7Co5b%2BKgjvpzYdH85PLcc4X21ouwEGl%2FS4qnUAvoSlXUUhR1eKr2VWFTB%2BGMl6FsiQsVD1R3urlAAIoSn7JQkmiVVCHSpCwDH%2FqPepXQ0Db77CJOAImohB%2BRPWr31ev5g%2FkE%2BzTa4lbvZo8xdWPffQu9yJTPCNB66s%2BzXoJt%2F0L6hSoCuBIoK8fnBGG87OoRckJpLqyWe4YbpGi50g0%2B3I3UD85Oa0fzubfoXxPLbW3FDWzigmyJeM0tQkax7PqTy80%2BUxfUHPlBZIRVNQ%2Bv0xRm8REKPoLmNr0%2BUo48v9GFbXPKylqQ2IKm00QddgyWGMROCTxdLB9nCY8P7j2DjlsV%2F%2Bmfr0C0r%2FNkeXbbpPlOTBBwT0mVz1zx9S%2FwJecBF9Wgv3p032iP2v4VSgfgW2G%2BHUEdEXU6iq4CtpLJfIN9XQG8dwa1VoO8XC2SrPDDyCOQptXgbcPvlAgBfxBoGwftQKeKFrNTASPt3pGGqDt%2FQRasn2kri%2BH6L80MJRsmVYJrAKyDItpJUy3%2F15WYIJqcJ9Q5N%2FLFJ4c3dc1URpWl9hW6mu50MUIelg4ucTPf15zs5DFo1c0VSp1tKB9jkwIyuM45kb%2BIP8gHed%2B6jO3v0KbIknzLy636E8KPTdCuUpB0wLo9JKnAO6pv0vS31EtBha%2FfJemkgLVVnd8KCk4qBTpQ5m7FbifBKrPJcq0pZAFVG%2FXbOFz%2BTcq2MLrcmV28Nmi%2FOHskh82bau0k8eWCaPijQPWQ5lUvslwVCfHkXBMIehqUgtDNLeauH1huvZTbYmw%2BluPjyWoNGEuxRLR7LK5fSyXFUyK7PURQv2v8D3XOt2NJ6liBbmPGOsakw1kbeOs%2B31Wm5qpH%2BiJWSzqdPr2O7zc2TmtnrzCig6bBd%2FvgQmzOlz0STWIlmZEQfupogOZFHUZ7EkUnMn0RrpIMqAgHRJAOjIJ3yGw1I%2FMAp9q9S3Q%2FclADNm1wEeO%2Bxbwg5OIYHZLY3ehG5lJk2xhco%2B6JWybpEVz2wrR6hZyD0QXZbeDVB%2BonmlimpkWprdAs4WEZDSQppsDlcdCBJJESIYFuAtUnC4GIF2C3Uu2Kv7L1bdz6FxtqxpG4TqQOqOUNAJ2HLvPWA2GgDy4O4vaDrtyl6P%2B1fAll%2BSyFcQ28GHqh7fvvf37udylf0fNwhzgz87Y%2Bcf5x9GnF6ygHu18sAbipWeF0YPBgp2GaKeQduxxdEr3SgbH1kvH7tvqSLhedomOvZyts2dw8acu3dY%2Ff%2BucuMtCuP%2Fe4zC4XnH3OLZ8ZuxTWxy8dJfU5dhDeKPSlJy5pn%2F%2B7u3XrJhmr9C5CuleGflGQocKnlAUaRKp0BAHV0ZwUt9VCqk6zYOgRIuMfePJzdmBdpPJ7%2F6B23%2Bf%2Bsp9NMDZevovvfYHG5dGPISQq1DojqNckchVrCcCYz%2FQ0hI0m3NKDRfkgsrnamo%2Bp0CAq1FyvC3a3Nak%2Fs5VX282x9Ufy3E39VAx6o7LpCvO2wK%2Bch9jNqpJCutcIOooKnYWtDK8gTRVYygRQfwgzKM5%2BjP2jOZdx3r32Py7rQUPOzAnoRs95NvRAR0qLGU11Taqu1bUYSzMcWjMEir067JQQHfIrLBHsrgv00%2FWavd8HRLMEEYFSW3HCSNQehnrHztKqHcDyo4VfZ6gPKCR%2BgufwA8GegxUEo4A%2Bgd0BASHiH6jYMLIsUdQJTs%2FC641KN4oCHWolCMLlMfIdtWKScjx7SM5LD9HnfmhrGI0S139UWfUnxgOXdJFW%2BAMcGjKr6eHAttHF5sUoeArYKDcxMSYcKA%2FxUDhPiEOEAPafSIUFArN0r24ynI91EPARDXvIDYyvqZaWeroBOUABQA%2FE%2BDXC7PWafDLQY2oiwpUEyj4RQtVlUp1GrM7In2p2A7VuiOW6otMiGOo5Mrp05ejVuTy6dNX%2Fk%2F7mybZQ0nUmfrbx3U4KueDnlHm5wdh8FFeKnoaKKh%2FTK18StOPhwG9Xo5mqXAxvw%2F79YQwwDR%2BnAKQQ4izVXioB84qcppWB7IqjU45z4CE17OvF1Dw%2BoTFqxtz8dxwtogBnF9MjIl%2Fin%2BK8s3hM9laIn0TiCbTAXL0T798bPXqx36p3chrv0O%2BGC9Xaj48Ecv8U8UEeBvUEsDlTepiU5OvlpeNGvpnKF0RvUooWhIjnx6GeBapXCQYTw9DNg6%2FOC3gZjp76oNTj9Kz6Jqobxb9NDqc08vcKReOpcsQV2K8InXFaXW3aI6Ofr1k48rp7CX7rx%2Bv1UKPsfvzQU0Kc83i2VdILmd2%2FyX55zT9luN2%2BCu4nKfwPcK%2FCvDVU%2BpHh8%2BLaldIf1fA5h3ndT6Fln9%2FW%2F9Ce1vndfvJtnPVO2xhm3qbafHVCN1X363UXHq9xuVD8OSD29Z8pZ5cZrern9cAdGW%2Fuib%2Fud%2BVK0L9a42r6C90kL8KzxwLQw9NkIQJL0ASU8M%2BVG0KsUdgdvpgP%2F6NqqP0%2FgHZFUfGEijZLHpiIgvV5%2FBltrj8Qd7XQd5p4P%2B7tJo30NMO6VGBwahSPMYiaaBYoLY6uEnciyhhh1Z%2FvvacG%2Frjpsvnpzs0B1Id6fmX8119l88XnOxe%2FuGrzzHcdu7UtY3%2B2vmXN5zUyj3ZcPl8p1sZSs6%2FnGXtwrV7Ka0XZdz83fwjjINpZWYw85lL8BRK4nGyIir2RiOsEyipuEcIakpGjWgBjLiHWOgj0Yi34gW1kKPxHt2Na5q%2Blwg1RdRSpFDNzosb44YJXnAfoEOpZW%2F%2F6u1lhYA6leevezbI26zNHO811M2dc5HFxpk4i1jPC0s21%2FBWW5DnPQbn2X1WK43%2FaM2n18DfSoybbNHijFpamzXI31eRibGUOxSu%2FlT96YZlq1Yt20DaSBuG6knw2eusHs5EPBfNmVvHKdaQzcDfz9ZsXmLDWGXy2U5OsYSsIn8CS12jQIyD12KKqZrLPy7mSPdICmd6WGHG8NDZkkHuE4h9TU8FpmUO%2FVjC%2FEinToFyoNDz2p9XD6g78WgQdPG7Z3R0T%2FZ5dTM9lsL8Ktek7szl2L%2BgQwGgwkZHc2g5Su7NvVqwGy2Ua4KSXUwt1X4PaM5paaEu6jQ5zVFyNabxvUksVt2T%2F4VeamYPlLtffdQsk%2B2sUTY%2FzDXl%2F05W53%2FBz9UK3p7LjapZ2ZxOm%2BUlZXrL3HHGqO8%2BwVroDaCTTnTxitMxmiAAYQzVJQH%2Bnj3oIHnPaN6Zq6sNSLjBl8tKgVr2mj%2F9CWi9dnKca8rBQBsd5R1tzVlgrl5pbnPw6kZclCr2CHxMnHohLz%2B3KRQokzALyeIKFU1TNCiayJdoHvDYe7K6mZLm8S3uJ9dojuaJ62%2FqN%2FtjQxnSnhnKPw%2BLNrLi8ZKyJ3x1YhiI1aNAtP6NzCGzYv3DmaGh%2FLvQZnt0evgIhTFV0kE%2FPYxAnOHhCQUZdCWY5JWJwMzlAGl1mpNbDU7yyGnhRMILsYhH3VRAijrPcBU8%2FCj1Y9NY6cnGVW0CjTLaz7E3epvaT%2FLtTV72Rs%2B0WVVmd0dz%2FMGTI5F0OsIviaqDlbbO5X6xT3PeXbXHRtf%2Fz%2Bfdka%2BeKPr8KF7IF4vBsT9MFPuPJMBTBMq9hQxXelQ%2Bbewnf18ap4Ib%2BmSMrtDU5zqlD8QANa5MBGh%2FOwOvSDfcV2d66mfEWsbGWmIz6nsyZDWQSmqmxDneYyvjHPmRXHZxeueyRGLZzvRioKnGto9nIPkibAJA16adcOZRQr1iAP3bUyBR7T4RgAWTKxhkCYFwshq%2B7iV9r0whk50cmRcTg4fy5x4OmmNkHndIA2%2BYuMbmE9dwGYB4KFTsvnDE6Ah47r%2FfE3AYI%2BoXADpkdlENcZ8OZEEf8FFGZNxMs6ZLpG3SUFLL7Q2kcFU%2FA%2FJsw%2BvWDa%2F7emewLaoeibaF1B9qUNnuqWK3%2BUfXYVL1v%2FomD15xxeDkPnXTOKSVcCbDGtOu0YQNpGAP7U1HU58UrqGu8xIbHtkQ3LVhb7Dx46ET3Ffcm1q0YcOizNmf3bC3VjWfAcpSv3MyTlgJ23FHQgmgvk%2Bgk8pL0mcCDOn08MDAQlf%2B%2FSlTZ1z12fnqntOhbOTL9%2FZdevbAPN%2Byby1f%2FuUtC%2Fixm8ZBo59LTXEW060hGrTDplNprWd58fwB%2Fb%2FE27BdS%2Fs7U%2BrGVCeQ46nzaw9QccnmZerGZZs3Yw9aVHt%2BKh6HN4ti6lxIhT%2FwahnZtWwzlY9QHQ2c79C%2BdxzvVDKy8GqKWQERO9YAKbpsDUTLdWV5dE8PVPjvj9pqw7ah%2FPFVtkit7aj6G5xY9mfJrCz1j1e0BcnPol4UjtrCdbahIVtd2HaURujnFJR8CuOuUUfhrGhgKKgjCYNSvCc1WKlEp8wHUaAYynFNyzZn%2B2MnYv36dbMDBTonl%2FT%2Fma5IKAyEGz%2B4eRnVtaX6tss2o34u8mWorFtuFgm4A6qK%2Fyp%2FgLEBVat5WnPDdKA574ubuFJ%2FIUfZ%2FY2Nt6mN%2BZNNTSTaeI56gKwkXerTe9DDHUw8%2FH35FY3nNN7GGuBKWhrV9ep%2B0k1WjNWVaHkW1yA%2BQHWNu8rtBw2a5YXuE40rs7%2FGA%2Bj09V3hA98yRnFPOGr8ltGlsFdD%2F7tRce3LH6Trcneuiy7K7J3khKu%2B3qUaXPWaX7T6%2FKfj9BX2eZq2XAcZT79u1ClJzUtHUqfqSMWBcZS43Ena0cUGLgpkKxB1QM%2B0Fxz10wgg6r5rltnFpH05pepUq3Y2HfYqeKRntmUFNz%2BXmcOs1H31U6cC6RTVLfCg7RNBF1UF2%2FwBgu0fFQtPEU1sSg3VcNsR7dWq3af87tUFn1l3ltXpaJxpNvtcZkH2WmMst3JqRpxUH%2BWC0E1qOGtP66s1MYv%2BVLu8%2FXFXvV%2FZbunYYBeVN64ls0ur6NzpV9xzlmQwB5qC4Tq70WC0tk8dWJXeHvkD0h9zJOM0vD86%2F1NJMaIAolctvlByferCsqOKDKceOfUu1PsmoFCamV5mCrMUOCi6V6FJosMF22AcrKJgQDVhfYh6tepp%2FlYgvnCEAbJQ1L0rOpajEmRcasMiPfxhgGoVo4rwreQpV6fUJHH2e8fa1s2c13Apl1b89a58ozdoap2sjgLN9uISl7P1DrulyeIkt0zr6JjWocoPOZsaXPb6jtqBblsgsaRre2xHi4nELm0MhG1%2Bx1SXwLpFi53b%2BaHRYo%2FIrbZtuWAKu5cSEXfybnnmUCaXGTpQr0xK2O2WWY76f%2BnAjNVf7nCZHU5XqIkTnpt6VtvsFlPXg1031g%2FVRdpkkyVpD7jnmax88QwDvg%2F66NnMRdRXTcGTmQc3cuINwN5IQqi0yzb%2BYFVHuVqI5s4ADfg5oE4ybDLd28mFSFmYvRoomsWXEdLU2Wl3GJy93ZNb%2Fd5gqmNaqJZSO1l6PVRy0nZIj%2F45EetjLguh1rLqR%2BSK0hO6NrsqcNX8zoUdjQYDJ7tb4os6%2Bi%2BY0qpY2AWlnLRDWdGFTfGY1gV0zNAtJ7pdo24se0D88AwLY%2FgZmE9iuP4V5v7CSR%2FRThaHLh%2BUeBkXwU6BC7lGOevK65udTv%2BtS%2FPfW7qj3ljTcj3b9OkbV85t8xsMj7Ddj7DGpthZKwKPvso%2Fc%2F1K9aLE12fMWLV1y1D9ua8lyJdWXr%2FbG%2BnoCFutf%2FmLILe39ITUV4igr3876fpX5g2zeB52sWnIL4fXHlgeUzOx5QfIvJQyrKQE9wHUqVq%2BPEaOrz0wVvNbJZVSfsuMzxN4l9PkedFzw9V5Dj%2BnzpgoT4ZxCxJfC5RWLc74YVHxKlExCYt0JAOMatREhHBSCAtSfod6x6Ls8HCWECLwXZ9nd5Dz1T24JUdWs6fU3%2B%2BfcnT49Qe%2BkBs%2BwdsMZgPXMp3U5S958snPP%2FEE7bvkOPCuTUDTUQ%2FUzirLhML9yPahoe1D5Fj5jWsaoveyP00PehdUAHk%2FseDVWsvDWXXXsyn%2F4wfpXc2V3%2FQxli3jl%2F5hj%2F83avSCfpTNxOEKLmTjxOEKuxgNlsQn0xgct724mhynupNW1Ph6o3RYS3%2F%2B2TJrzLlkFz%2Bip3qCHKf6eqW02QJLjBYuuj4sobhCWqa%2FYHGEHpcnumuWSOhxeaL7sOakNR6vvmo%2BYcfFA8UFXEPZf9UjyudIOyNwx%2Fi90DdsujS%2FFX2UAwvWSVK4NxaMhAGw3oowp%2Fuc8CTi7D2rBgZWwb%2F60faR7SPsEbjkXy4G0XaqhXPwe2cePjxjxuHD6ssQuR1fq6PF0E%2Bo2t1nePTn8TUmxz%2FA3crMoCc7egESuoTHYc7mYdg6etORoOhR7BBGD%2BqJopELrl4S6cJNRtEAsLP%2FOdvnJq0Wo0GolY2Et9VFB2Kf%2B4bZvVyxfOMz3WdFfSIryj6DwWghre7aQbdiDrkTL3A3vNDuDpk93HqXwam%2BbWmUJZfNn5ozKV5Pmmq8PF%2FjVY%2B2Tlk2M2RzSXKjmbQ4RZcQavEYrN%2F9rlXwtIQqzxQNMzPPfHYLvuPoO9TbT8bpGw5CQPGd%2BSyX%2FCyf0Vxjd2R9NmsunnXYa8xGHzn%2BsSfM5J0y0DZEXWWxkXjcR75KBLNLHi7XvX2G8VOrf4Ykg0AMdBESIpo7MgAfyakA6rkqpI6UjNs0px7cMV%2BD5BF49Tez1VGnYmq0WIijp985m4Sn2gJR9b07riPPFo97OYbUZbxJCpot7H%2FlpZBicglCPN7WOfJkcHqc3ElWqvvz%2F1E6bIQrG%2Btz6WkM1SM9FBTR7FSs8KyBBytSmNEoquJNFN5EQyTiCrnKDx1h58yxCepPHU5nxGoxEQeeOZi2m80DxNxncVhr6BmEfUarxejw%2BWSiHhWk19bSY7aKR5MsteblJpfTLtjimBouXsm3d3djjYM%2BwEW0El9dM%2FueVRWIsXwe43R7SgbVZqrnqoJ1X%2FkuF7pcgf8duv4q6vayV5U9zMV91GxO59UUjW8rHV6u799WzKMT7umRCXbYUKM%2BfoaCcwgaoqZUtmodV3p%2BX7akb4dnU9B9La38RPFUG2SCC90tVA4XwEFhyOpZZrUCsgWYHsczLFBBVGNtstoN1bw0Z%2BO4fYIbvZVt4EUcJEKOhHeincWqONw%2Bq6w5Go%2BWGOSR7LhKV%2BKBqbBPpfUvOf9QqkpDyVhBeyyZQGMsdA5FBUqvFMtUyGq9vjnsAJU4UcrxldP1CCaofyDkSAifoP5QwWx%2BSyUGxp75BzGAvtG7uQ38LehlyEQMeh0TeE6Bm7tYdXqdkt0uOb3kfYlNwmOdDyacOq%2FqlFo1v%2BPTmTi3E%2FglC9W11b34A22zmLzvb231Q0L2Bgg60OTW4YdstO%2BYOJnO38TtpH7zy9ymokWyA79qlVSn38HtpFlImFnhu3b4boNWXklOXV0Iwo7lQ1hrZyPFcwtjwFP7iEKSHSSJw509kh8kj6pr%2BH1jR7km9vcvqN9657vffefkv%2BfKxge1X%2B7RdjYUPIESN7gTvRkB%2FRMYtEkaVkdHApmdBPpnKmz0n1xSWFOyVIuLrinZwpoCRe6kyiVZoHX088F%2BUX4%2BWKS4iBTP0IWxGtZgOdMaV4KTayqHQF%2FVihBwTbgDXTCmKoOBJeNhwJMzEVjtjIFLuU38fPR7hqNG1JS7g%2FqRCuy3vmQ3W9Vu8qbVbP%2BSzazGRJH83MzP90Ck2m31mMjP8TiLn5uwD2Ugr2PFvPQjB5BnSJvQxGQZZEB%2BLopqzGzDbMmbkAPkZVJjeO5FzOSBKCgJze2ZS4Gemc9twrwY6u9H61iUQTcRvtdT9RW3tRxAWwFs2tcuJRnI6xjmBdWjbgFNRHMHiF1uHYBfUR%2Fut5Ug2jXAaT96%2B9RH%2FFToRwIzGbKmVJ1AZQnoabSB1yyIg7ByAridHApPMjyw0OiV6RjSbCuzwLAvFizBliWJua1tsuAgvNPbmljYbpt8lkWam7b3XZiOiKJskMOtmfScnsbPW208knwjuXrXK4Q1iKIgNyYXXDVT9C2Ye%2F78GQ5BEEXfFdde2RwauOysdJNL5AzCy84ard%2FnGAVN8alecnFdgu5Gbd5DJTL%2BhHZK0vApVy3OfU8XTSJg1TlssivsPYUlIqvn66PzrVTymCc4wgF6SDNR0pDf%2B9Gp%2BVnsUH5WtpHYsuhOaey8zdwLN47V8MTbm78g687%2BP3cx6tcAeNpjYGRgYGBk8s0%2FzBIfz2%2FzlUGeZQNQhOFCWfF0GP0%2F8P8c1jusIkAuBwMTSBQAYwQM6HjaY2BkYGAV%2Bd8KJgP%2FXWG9wwAUQQGLAYqPBl942n1TvUoDQRCe1VM8kWARjNrZGIurBAsRBIuA2vkAFsJiKTYW4guIjT5ARMgTxCLoA1hcb5OgDyGHrY7f7M65e8fpLF%2B%2B2W%2FnZ2eTmGfaIJi5I0qGDlZZcD51QzTTJirZPAI9JIwVA%2BwT8L5nOdMaV0AuMJ%2BicRHq8of6LSD18fzq8ds7xjpwBnQiSI9V5QVl6NwPvgM15NXn%2FAtWZyj3W0HjEXitOc%2FdIdbetPdFTZ%2BP6t%2BX7xU0%2Fk6GJtOe1%2FB3arN0%2Fpmz1J4UZc%2BD6ExwjD7vioeGd5HvhvU%2BR%2BDZcGZ6YBPNfAi0G97iBPwFXqph2cW8%2BD7kjMfwtinHb6kLb6Wygk3cZytSEoptGrlScdHtLPeri1JKueACMZfU1ViJG1Sq5E43dIt7SZZFl1zuRhb%2FGOs44xFVDbrJzB5tYs35OmaXTrEmkv0DajnMWQB42mNgYNCCwk0MLxheMPrhgUuY2JiUmOqY2pjWMD1hdmPOY%2B5hPsLCwWLEksSyiOUOawzrLrYiti%2FsCuxJ7Kc45DiSOPZxmnG2cG7jvMelweXDNYXrEbcBdxf3KR4OngheLd443g18fHwZfFv4NfiX8T8TEBIIEZggsEpQS7BMcJsQl5CFUI3QAWEp4RLhCyJaIldEbURXiJ4RYxEzE0sQ2yD2TzxIfJkEk4SeRJbENIkNEg8k%2FklqSGZITpE8InlL8p2UmVSG1A6pb9Jx0ltkjGSmyDySlZF1kc2RnSK7R%2FaZnJ5cmdwB%2BST5SwpuCvsUjRTLFHcoOShNU9qhzKespGyhXKV8SPmBCpOKgUqcyjSVR6omqgmqe9RE1OrUnqkHqO9R%2F6FholGgsUZzgeYZLTUtL60WbS7tKh0OnQydXTpvdGV0O3S%2F6Gnopekt0ruhz6fvpl%2Bnv0n%2Fh4GdQYvBJUMhwwTDdYYvjFSM4oxmGd0zVjK2M84w3mYiYZJgssLkkqmO6TzTF2Z2ZjVmd8ylzP3MJ5lfsRCwcLJoszhhyWXpZdlhecZKxirHapbVPesF1ndsJGwCbBbZ%2FLA1sn1jZ2XXY3fFXsM%2Bz36V%2FS8HD4cGh2OOTI51ThJOK5zeOUs4OzmXOS9wPuUi4JLgss7lm2uU6zY3NrcSty1u39zN3Mvct7l%2F8xDzMPLw88jyaPM44ynkaeEZ59niucqLyUvPKwgAn3OqOQAAAQAAARcApwARAAAAAAACAAAAAQABAAAAQAAuAAAAAHjarZK9TgJBEMf%2Fd6CRaAyRhMLqCgsbL4ciglTGRPEjSiSKlnLycXJ86CEniU%2FhM9jYWPgIFkYfwd6nsDD%2Bd1mBIIUx3mZnfzs3MzszuwDCeIYG8UUwQxmAFgxxPeeuyxrmcaNYxzTuFAewi0fFQSTxqXgM11pC8TgS2oPiCUS1d8Uh8ofiSczpYcVT5LjiCPlY8Qui%2BncOr7D02y6%2FBTCrP%2Fm%2Bb5bdTrPi2I26Z9qNGtbRQBMdXMJBGRW0YOCecxEWYoiTCvxrYBunqHPdoX2bLOyrMKlZg8thDETw5K7Itci1TXlGy0124QRZZLDFU%2FexhxztMozlosTpMH6ZPge0L%2BOKGnFKjJ4WRwppHPL0PP3SI2P9jLQwFOu3GRhDfkeyDo%2F%2FG7IHgzllZQxLdquvrdCyBVvat3seJlYo06gxapUxhU2JWnFygR03sSxnEkvcpf5Y5eibGq315TDp7fKWm8zbUVl71Aqq%2FZtNnlkWmLnQtno9ycvXYbA6W2pF3aKfCayyC0Ja7Fr%2FPW70%2FHO4YM0OKxFvzf0C1MyPjwAAeNpt1VWUU2cYRuHsgxenQt1d8%2F3JOUnqAyR1d%2FcCLQVKO22pu7tQd3d3d3d3d3cXmGzumrWy3pWLs%2FNdPDMpZaWu1783l1Lpf14MnfzO6FbqVupfGkD30iR60JNe9KYP09CXfvRnAAMZxGCGMG3pW6ZjemZgKDMyEzMzC7MyG7MzB3MyF3MzD%2FMyH%2FOzAAuyEAuzCIuyGIuzBGWCRIUqOQU16jRYkqVYmmVYluVYng6GMZwRNGmxAiuyEiuzCquyGquzBmuyFmuzDuuyHuuzARuyERuzCZuyGZuzBVuyFVuzDduyHdszklGMZgd2ZAw7MZZxjGdnJrALu9LJbuzOHkxkT%2FZib%2FZhX%2FZjfw7gQA7iYA7hUA7jcI7gSI7iaI7hWI7jeE7gRE7iZE5hEqdyGqdzBmdyFmdzDudyHudzARdyERdzCZdyGZdzBVdyFVdzDddyHddzAzdyEzdzC7dyG7dzB3dyF3dzD%2FdyH%2FfzAA%2FyEA%2FzCI%2FyGI%2FzBE%2FyFE%2FzDM%2FyHM%2FzAi%2FyEi%2FzCq%2FyGq%2FzBm%2FyFm%2FzDu%2FyHu%2FzAR%2FyER%2FzCZ%2FyGZ%2FzBV%2FyFV%2FzDd%2FyHd%2FzAz%2FyEz%2FzC7%2FyG7%2FzB3%2FyF3%2FzD%2F9mpYwsy7pl3bMeWc%2BsV9Y765NNk%2FXN%2BmX9swHZwGxQNjgb0nPkmInjR0V7Uq%2FOsaPL5Y7ylE3l8tQNN7kVt%2BrmbuHW3LrbcDvam1rtzVvdm50TxrU%2FDBvRtZUY1rV5a3jXFn550Wo%2FXDNWK3dFmh7X9LimxzU9qulRTY9qelTTo5rlKLt2wk7YiaprL%2ByFvbAX9pK9ZC%2FZS%2FaSvWQv2Uv2kr1kr2KvYq9ir2KvYq9ir2KvYq9ir2Kvaq9qr2qvaq9qr2qvaq9qr2qvai%2B3l9vL7eX2cnu5vdxebi%2B3l9sr7BV2CjuFncJOYaewU9gp7NTs1LyrZq9mr2avZq9mr2avZq9mr26vbq9ur26vbq9ur26vbq9ur26vYa9hr2GvYa9hr2GvYa%2FR7oXuQ%2Feh%2B2j%2FUU7e3C3cqc%2FV3fYdof%2FQf%2Bg%2F9B%2F6D%2F2H%2FkP%2Fof%2FQf%2Bg%2F9B%2F6D%2F2H%2FkP%2Fof%2FQf%2Bg%2F9B%2F6D%2F2H%2FkP%2Fof%2FQf%2Bg%2F9B%2F6D%2F2H%2FkP%2Fof%2FQf%2Bg%2F9B%2F6D92H7kP3ofvQfeg%2BdB%2B6D92H7kP3ofvQfRT29B%2F6D%2F2H%2FkP%2Fof%2FQf%2Bg%2F9B%2F6D%2F2H%2FkP%2Fof%2FQf%2Bg%2F9B%2F6D%2F2H%2FkP%2Fof%2FQf%2Bg%2F9B%2F6D%2F2H%2FkP%2Fof%2FQf%2Bg%2F9B%2F6j6nuG3Ya7U5q%2F0hN3nCTW3Grbu4Wrs%2FrP%2Bk%2F6T%2FpP%2Bk%2F6T%2FpP%2Bk%2B6T7pPek86TzpPOk86TzpOuk66TrpOuk66TrpOlWmPu%2F36zrpOuk66TrpOuk66TrpOvl%2FPek76TvpO%2Bk76TvpO%2Bk76TvpO%2Bk76TvpO7V9t%2BqtVs%2FOaOURU6bo6PgPt6rZbwAAAAABVFDDFwAA%29%20format%28%27woff%27%29%2Curl%28data%3Aapplication%2Fx%2Dfont%2Dtruetype%3Bbase64%2CAAEAAAAPAIAAAwBwRkZUTW0ql9wAAAD8AAAAHEdERUYBRAAEAAABGAAAACBPUy8yZ7lriQAAATgAAABgY21hcNqt44EAAAGYAAAGcmN2dCAAKAL4AAAIDAAAAARnYXNw%2F%2F8AAwAACBAAAAAIZ2x5Zn1dwm8AAAgYAACUpGhlYWQFTS%2FYAACcvAAAADZoaGVhCkQEEQAAnPQAAAAkaG10eNLHIGAAAJ0YAAADdGxvY2Fv%2B5XOAACgjAAAAjBtYXhwAWoA2AAAorwAAAAgbmFtZbMsoJsAAKLcAAADonBvc3S6o%2BU1AACmgAAACtF3ZWJmwxhUUAAAsVQAAAAGAAAAAQAAAADMPaLPAAAAANB2gXUAAAAA0HZzlwABAAAADgAAABgAAAAAAAIAAQABARYAAQAEAAAAAgAAAAMEiwGQAAUABAMMAtAAAABaAwwC0AAAAaQAMgK4AAAAAAUAAAAAAAAAAAAAAAIAAAAAAAAAAAAAAFVLV04AQAAg%2F%2F8DwP8QAAAFFAB7AAAAAQAAAAAAAAAAAAAAIAABAAAABQAAAAMAAAAsAAAACgAAAdwAAQAAAAAEaAADAAEAAAAsAAMACgAAAdwABAGwAAAAaABAAAUAKAAgACsAoAClIAogLyBfIKwgvSISIxsl%2FCYBJvonCScP4APgCeAZ4CngOeBJ4FngYOBp4HngieCX4QnhGeEp4TnhRuFJ4VnhaeF54YnhleGZ4gbiCeIW4hniIeIn4jniSeJZ4mD4%2F%2F%2F%2FAAAAIAAqAKAApSAAIC8gXyCsIL0iEiMbJfwmASb6JwknD%2BAB4AXgEOAg4DDgQOBQ4GDgYuBw4IDgkOEB4RDhIOEw4UDhSOFQ4WDhcOGA4ZDhl%2BIA4gniEOIY4iHiI%2BIw4kDiUOJg%2BP%2F%2F%2F%2F%2Fj%2F9r%2FZv9i4Ajf5N%2B132nfWd4F3P3aHdoZ2SHZE9kOIB0gHCAWIBAgCiAEH%2F4f%2BB%2F3H%2FEf6x%2FlH3wfdh9wH2ofZB9jH10fVx9RH0sfRR9EHt4e3B7WHtUezh7NHsUevx65HrMIFQABAAAAAAAAAAAAAAAAAAAAAAAAAAAAAAAAAAAAAAAAAAAAAAAAAAAAAAAAAAAAAAAAAAAAAAAAAAAAAAAAAAAAAAAAAAAAAAAAAAAAAAAAAAAAAAAAAAAAAAAAAAAAAAAAAAAAAAAAAAAADAAAAAACjAAAAAAAAAA1AAAAIAAAACAAAAADAAAAKgAAACsAAAAEAAAAoAAAAKAAAAAGAAAApQAAAKUAAAAHAAAgAAAAIAoAAAAIAAAgLwAAIC8AAAATAAAgXwAAIF8AAAAUAAAgrAAAIKwAAAAVAAAgvQAAIL0AAAAWAAAiEgAAIhIAAAAXAAAjGwAAIxsAAAAYAAAl%2FAAAJfwAAAAZAAAmAQAAJgEAAAAaAAAm%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%2FAADiUAAA4lkAAAEJAADiYAAA4mAAAAETAAD4%2FwAA%2BP8AAAEUAAH1EQAB9REAAAEVAAH2qgAB9qoAAAEWAAYCCgAAAAABAAABAAAAAAAAAAAAAAAAAAAAAQACAAAAAAAAAAIAAAAAAAAAAAAAAAAAAAAAAAAAAAAAAAAAAAAAAAAAAQAAAAAAAwAAAAAAAAAAAAAAAAAAAAAAAAAEAAUAAAAAAAAAAAAAAAAAAAAAAAAAAAAAAAAAAAAAAAAAAAAAAAAAAAAAAAAAAAAAAAAAAAAAAAAAAAAAAAAAAAAAAAAAAAAAAAAAAAAAAAAAAAAAAAAAAAAAAAAAAAAAAAAAAAAAAAAAAAAAAAAAAAAAAAAAAAAAAAAAAAAAAAAAAAAAAAAAAAAAAAAAAAAAAAAAAAAAAAAAAAAAAAAAAAAAAAAAAAAAAAAAAAAAAAAAAAAAAAAAAAAAAAAAAAAAAAAAAAAAAAAAAAAAAAAAAAAAAAAAAAAAAAAAAAAAAAAAAAAAAAAAAAAAAAAAAAAAAAAAAAAAAAAAAAAAAAAAAAAAAAAAAAAAAAAAAAAAAAAAAAAHAAAAAAAAAAAAAAAAAAAAAAAAAAAAAAAAAAAAAAAAAAAAAAAAAAAAAAAAAAYAAAAAAAAAAAAAAAAAAAAAAAAAAAAAAAAAAAAAAAAAAAAVAAAAAAAAAAAAAAAAAAAAAAAAAAAAAAAAAAAAAAAAAAAAAAAAAAAAAAEUAAAAAAAAAAAAAAAAAAAAAAAAAAAAAAAAAAAAAAAAAAAAKAL4AAAAAf%2F%2FAAIAAgAoAAABaAMgAAMABwAusQEALzyyBwQA7TKxBgXcPLIDAgDtMgCxAwAvPLIFBADtMrIHBgH8PLIBAgDtMjMRIRElMxEjKAFA%2Fujw8AMg%2FOAoAtAAAQBkAGQETARMAFsAAAEyFh8BHgEdATc%2BAR8BFgYPATMyFhcWFRQGDwEOASsBFx4BDwEGJi8BFRQGBwYjIiYvAS4BPQEHDgEvASY2PwEjIiYnJjU0Nj8BPgE7AScuAT8BNhYfATU0Njc2AlgPJgsLCg%2BeBxYIagcCB57gChECBgMCAQIRCuCeBwIHaggWB54PCikiDyYLCwoPngcWCGoHAgee4AoRAgYDAgECEQrgngcCB2oIFgeeDwopBEwDAgECEQrgngcCB2oIFgeeDwopIg8mCwsKD54HFghqBwIHnuAKEQIGAwIBAhEK4J4HAgdqCBYHng8KKSIPJgsLCg%2BeBxYIagcCB57gChECBgAAAAABAAAAAARMBEwAIwAAATMyFhURITIWHQEUBiMhERQGKwEiJjURISImPQE0NjMhETQ2AcLIFR0BXhUdHRX%2Boh0VyBUd%2FqIVHR0VAV4dBEwdFf6iHRXIFR3%2BohUdHRUBXh0VyBUdAV4VHQAAAAABAHAAAARABEwARQAAATMyFgcBBgchMhYPAQ4BKwEVITIWDwEOASsBFRQGKwEiJj0BISImPwE%2BATsBNSEiJj8BPgE7ASYnASY2OwEyHwEWMj8BNgM5%2BgoFCP6UBgUBDAoGBngGGAp9ARMKBgZ4BhgKfQ8LlAsP%2Fu0KBgZ4BhgKff7tCgYGeAYYCnYFBv6UCAUK%2BhkSpAgUCKQSBEwKCP6UBgwMCKAIDGQMCKAIDK4LDw8LrgwIoAgMZAwIoAgMDAYBbAgKEqQICKQSAAABAGQABQSMBK4AOwAAATIXFhcjNC4DIyIOAwchByEGFSEHIR4EMzI%2BAzUzBgcGIyInLgEnIzczNjcjNzM%2BATc2AujycDwGtSM0QDkXEys4MjAPAXtk%2FtQGAZZk%2FtQJMDlCNBUWOUA0I64eYmunznYkQgzZZHABBdpkhhQ%2BH3UErr1oaS1LMCEPCx4uTzJkMjJkSnRCKw8PIjBKK6trdZ4wqndkLzVkV4UljQAAAgB7AAAETASwAD4ARwAAASEyHgUVHAEVFA4FKwEHITIWDwEOASsBFRQGKwEiJj0BISImPwE%2BATsBNSEiJj8BPgE7ARE0NhcRMzI2NTQmIwGsAV5DakIwFgwBAQwWMEJqQ7ICASAKBgZ4BhgKigsKlQoP%2FvUKBgZ4BhgKdf71CgYGeAYYCnUPtstALS1ABLAaJD8yTyokCwsLJCpQMkAlGmQMCKAIDK8LDg8KrwwIoAgMZAwIoAgMAdsKD8j%2B1EJWVEAAAAEAyAGQBEwCvAAPAAATITIWHQEUBiMhIiY9ATQ2%2BgMgFR0dFfzgFR0dArwdFcgVHR0VyBUdAAAAAgDIAAAD6ASwACUAQQAAARUUBisBFRQGBx4BHQEzMhYdASE1NDY7ATU0NjcuAT0BIyImPQEXFRQWFx4BFAYHDgEdASE1NCYnLgE0Njc%2BAT0BA%2BgdFTJjUVFjMhUd%2FOAdFTJjUVFjMhUdyEE3HCAgHDdBAZBBNxwgIBw3QQSwlhUdZFuVIyOVW5YdFZaWFR2WW5UjI5VbZB0VlshkPGMYDDI8MgwYYzyWljxjGAwyPDIMGGM8ZAAAAAEAAAAAAAAAAAAAAAAxAAAB%2F%2FIBLATCBEEAFgAAATIWFzYzMhYVFAYjISImNTQ2NyY1NDYB9261LCwueKqqeP0ST3FVQgLYBEF3YQ6teHmtclBFaw4MGZnXAAAAAgAAAGQEsASvABoAHgAAAB4BDwEBMzIWHQEhNTQ2OwEBJyY%2BARYfATc2AyEnAwL2IAkKiAHTHhQe%2B1AeFB4B1IcKCSAkCm9wCXoBebbDBLMTIxC7%2FRYlFSoqFSUC6rcQJBQJEJSWEPwecAIWAAAAAAQAAABkBLAETAALABcAIwA3AAATITIWBwEGIicBJjYXARYUBwEGJjURNDYJATYWFREUBicBJjQHARYGIyEiJjcBNjIfARYyPwE2MhkEfgoFCP3MCBQI%2FcwIBQMBCAgI%2FvgICgoDjAEICAoKCP74CFwBbAgFCvuCCgUIAWwIFAikCBQIpAgUBEwKCP3JCAgCNwgK2v74CBQI%2FvgIBQoCJgoF%2FvABCAgFCv3aCgUIAQgIFID%2BlAgKCggBbAgIpAgIpAgAAAAD%2F%2FD%2F8AS6BLoACQANABAAAAAyHwEWFA8BJzcTAScJAQUTA%2BAmDpkNDWPWXyL9mdYCZv4f%2FrNuBLoNmQ4mDlzWYP50%2FZrWAmb8anABTwAAAAEAAAAABLAEsAAPAAABETMyFh0BITU0NjsBEQEhArz6FR384B0V%2Bv4MBLACiv3aHRUyMhUdAiYCJgAAAAEADgAIBEwEnAAfAAABJTYWFREUBgcGLgE2NzYXEQURFAYHBi4BNjc2FxE0NgFwAoUnMFNGT4gkV09IQv2oWEFPiCRXT0hCHQP5ow8eIvzBN1EXGSltchkYEAIJm%2F2iKmAVGilucRoYEQJ%2FJioAAAACAAn%2F%2BAS7BKcAHQApAAAAMh4CFQcXFAcBFgYPAQYiJwEGIycHIi4CND4BBCIOARQeATI%2BATQmAZDItoNOAQFOARMXARY7GikT%2Fu13jgUCZLaDTk6DAXKwlFZWlLCUVlYEp06DtmQCBY15%2Fu4aJRg6FBQBEk0BAU6Dtsi2g1tWlLCUVlaUsJQAAQBkAFgErwREABkAAAE%2BAh4CFRQOAwcuBDU0PgIeAQKJMHt4dVg2Q3mEqD4%2Bp4V4Qzhadnh5A7VESAUtU3ZAOXmAf7JVVbJ%2FgHk5QHZTLQVIAAAAAf%2FTAF4EewSUABgAAAETNjIXEyEyFgcFExYGJyUFBiY3EyUmNjMBl4MHFQeBAaUVBhH%2BqoIHDxH%2Bqf6qEQ8Hgv6lEQYUAyABYRMT%2Fp8RDPn%2BbxQLDPb3DAsUAZD7DBEAAv%2FTAF4EewSUABgAIgAAARM2MhcTITIWBwUTFgYnJQUGJjcTJSY2MwUjFwc3Fyc3IycBl4MHFQeBAaUVBhH%2BqoIHDxH%2Bqf6qEQ8Hgv6lEQYUAfPwxUrBw0rA6k4DIAFhExP%2BnxEM%2Bf5vFAsM9vcMCxQBkPsMEWSO4ouM5YzTAAABAAAAAASwBLAAJgAAATIWHQEUBiMVFBYXBR4BHQEUBiMhIiY9ATQ2NyU%2BAT0BIiY9ATQ2Alh8sD4mDAkBZgkMDwr7ggoPDAkBZgkMJj6wBLCwfPouaEsKFwbmBRcKXQoPDwpdChcF5gYXCktoLvp8sAAAAA0AAAAABLAETAAPABMAIwAnACsALwAzADcARwBLAE8AUwBXAAATITIWFREUBiMhIiY1ETQ2FxUzNSkBIgYVERQWMyEyNjURNCYzFTM1BRUzNSEVMzUFFTM1IRUzNQchIgYVERQWMyEyNjURNCYFFTM1IRUzNQUVMzUhFTM1GQR%2BCg8PCvuCCg8PVWQCo%2F3aCg8PCgImCg8Pc2T8GGQDIGT8GGQDIGTh%2FdoKDw8KAiYKDw%2F872QDIGT8GGQDIGQETA8K%2B%2BYKDw8KBBoKD2RkZA8K%2FqIKDw8KAV4KD2RkyGRkZGTIZGRkZGQPCv6iCg8PCgFeCg9kZGRkZMhkZGRkAAAEAAAAAARMBEwADwAfAC8APwAAEyEyFhURFAYjISImNRE0NikBMhYVERQGIyEiJjURNDYBITIWFREUBiMhIiY1ETQ2KQEyFhURFAYjISImNRE0NjIBkBUdHRX%2BcBUdHQJtAZAVHR0V%2FnAVHR39vQGQFR0dFf5wFR0dAm0BkBUdHRX%2BcBUdHQRMHRX%2BcBUdHRUBkBUdHRX%2BcBUdHRUBkBUd%2FagdFf5wFR0dFQGQFR0dFf5wFR0dFQGQFR0AAAkAAAAABEwETAAPAB8ALwA%2FAE8AXwBvAH8AjwAAEzMyFh0BFAYrASImPQE0NiEzMhYdARQGKwEiJj0BNDYhMzIWHQEUBisBIiY9ATQ2ATMyFh0BFAYrASImPQE0NiEzMhYdARQGKwEiJj0BNDYhMzIWHQEUBisBIiY9ATQ2ATMyFh0BFAYrASImPQE0NiEzMhYdARQGKwEiJj0BNDYhMzIWHQEUBisBIiY9ATQ2MsgVHR0VyBUdHQGlyBUdHRXIFR0dAaXIFR0dFcgVHR389cgVHR0VyBUdHQGlyBUdHRXIFR0dAaXIFR0dFcgVHR389cgVHR0VyBUdHQGlyBUdHRXIFR0dAaXIFR0dFcgVHR0ETB0VyBUdHRXIFR0dFcgVHR0VyBUdHRXIFR0dFcgVHf5wHRXIFR0dFcgVHR0VyBUdHRXIFR0dFcgVHR0VyBUd%2FnAdFcgVHR0VyBUdHRXIFR0dFcgVHR0VyBUdHRXIFR0ABgAAAAAEsARMAA8AHwAvAD8ATwBfAAATMzIWHQEUBisBIiY9ATQ2KQEyFh0BFAYjISImPQE0NgEzMhYdARQGKwEiJj0BNDYpATIWHQEUBiMhIiY9ATQ2ATMyFh0BFAYrASImPQE0NikBMhYdARQGIyEiJj0BNDYyyBUdHRXIFR0dAaUCvBUdHRX9RBUdHf6FyBUdHRXIFR0dAaUCvBUdHRX9RBUdHf6FyBUdHRXIFR0dAaUCvBUdHRX9RBUdHQRMHRXIFR0dFcgVHR0VyBUdHRXIFR3%2BcB0VyBUdHRXIFR0dFcgVHR0VyBUd%2FnAdFcgVHR0VyBUdHRXIFR0dFcgVHQAAAAABACYALAToBCAAFwAACQE2Mh8BFhQHAQYiJwEmND8BNjIfARYyAdECOwgUB7EICPzxBxUH%2FoAICLEHFAirBxYB3QI7CAixBxQI%2FPAICAGACBQHsQgIqwcAAQBuAG4EQgRCACMAAAEXFhQHCQEWFA8BBiInCQEGIi8BJjQ3CQEmND8BNjIXCQE2MgOIsggI%2FvUBCwgIsggVB%2F70%2FvQHFQiyCAgBC%2F71CAiyCBUHAQwBDAcVBDuzCBUH%2FvT%2B9AcVCLIICAEL%2FvUICLIIFQcBDAEMBxUIsggI%2FvUBDAcAAwAX%2F%2BsExQSZABkAJQBJAAAAMh4CFRQHARYUDwEGIicBBiMiLgI0PgEEIg4BFB4BMj4BNCYFMzIWHQEzMhYdARQGKwEVFAYrASImPQEjIiY9ATQ2OwE1NDYBmcSzgk1OASwICG0HFQj%2B1HeOYrSBTU2BAW%2BzmFhYmLOZWFj%2BvJYKD0sKDw8KSw8KlgoPSwoPDwpLDwSZTYKzYo15%2FtUIFQhsCAgBK01NgbTEs4JNWJmzmFhYmLOZIw8KSw8KlgoPSwoPDwpLDwqWCg9LCg8AAAMAF%2F%2FrBMUEmQAZACUANQAAADIeAhUUBwEWFA8BBiInAQYjIi4CND4BBCIOARQeATI%2BATQmBSEyFh0BFAYjISImPQE0NgGZxLOCTU4BLAgIbQcVCP7Ud45itIFNTYEBb7OYWFiYs5lYWP5YAV4KDw8K%2FqIKDw8EmU2Cs2KNef7VCBUIbAgIAStNTYG0xLOCTViZs5hYWJizmYcPCpYKDw8KlgoPAAAAAAIAFwAXBJkEsAAPAC0AAAEzMhYVERQGKwEiJjURNDYFNRYSFRQOAiIuAjU0EjcVDgEVFB4BMj4BNTQmAiZkFR0dFWQVHR0BD6fSW5vW6tabW9KnZ3xyxejFcnwEsB0V%2FnAVHR0VAZAVHeGmPv7ZuHXWm1tbm9Z1uAEnPqY3yHh0xXJyxXR4yAAEAGQAAASwBLAADwAfAC8APwAAATMyFhURFAYrASImNRE0NgEzMhYVERQGKwEiJjURNDYBMzIWFREUBisBIiY1ETQ2BTMyFh0BFAYrASImPQE0NgQBlgoPDwqWCg8P%2Ft6WCg8PCpYKDw%2F%2B3pYKDw8KlgoPD%2F7elgoPDwqWCg8PBLAPCvuCCg8PCgR%2BCg%2F%2BcA8K%2FRIKDw8KAu4KD%2F7UDwr%2BPgoPDwoBwgoPyA8K%2BgoPDwr6Cg8AAAAAAgAaABsElgSWAEcATwAAATIfAhYfATcWFwcXFh8CFhUUDwIGDwEXBgcnBwYPAgYjIi8CJi8BByYnNycmLwImNTQ%2FAjY%2FASc2Nxc3Nj8CNhIiBhQWMjY0AlghKSYFMS0Fhj0rUAMZDgGYBQWYAQ8YA1AwOIYFLDIFJisfISkmBTEtBYY8LFADGQ0ClwYGlwINGQNQLzqFBS0xBSYreLJ%2BfrJ%2BBJYFmAEOGQJQMDmGBSwxBiYrHiIoJgYxLAWGPSxRAxkOApcFBZcCDhkDUTA5hgUtMAYmKiAhKCYGMC0Fhj0sUAIZDgGYBf6ZfrF%2BfrEABwBkAAAEsAUUABMAFwAhACUAKQAtADEAAAEhMhYdASEyFh0BITU0NjMhNTQ2FxUhNQERFAYjISImNREXETMRMxEzETMRMxEzETMRAfQBLCk7ARMKD%2Fu0DwoBEzspASwBLDsp%2FUQpO2RkZGRkZGRkBRQ7KWQPCktLCg9kKTtkZGT%2B1PzgKTs7KQMgZP1EArz9RAK8%2FUQCvP1EArwAAQAMAAAFCATRAB8AABMBNjIXARYGKwERFAYrASImNREhERQGKwEiJjURIyImEgJsCBUHAmAIBQqvDwr6Cg%2F%2B1A8K%2BgoPrwoFAmoCYAcH%2FaAICv3BCg8PCgF3%2FokKDw8KAj8KAAIAZAAAA%2BgEsAARABcAAAERFBYzIREUBiMhIiY1ETQ2MwEjIiY9AQJYOykBLB0V%2FOAVHR0VA1L6FR0EsP5wKTv9dhUdHRUETBUd%2FnAdFfoAAwAXABcEmQSZAA8AGwAwAAAAMh4CFA4CIi4CND4BBCIOARQeATI%2BATQmBTMyFhURMzIWHQEUBisBIiY1ETQ2AePq1ptbW5vW6tabW1ubAb%2FoxXJyxejFcnL%2BfDIKD68KDw8K%2BgoPDwSZW5vW6tabW1ub1urWmztyxejFcnLF6MUNDwr%2B7Q8KMgoPDwoBXgoPAAAAAAL%2FnAAABRQEsAALAA8AACkBAyMDIQEzAzMDMwEDMwMFFP3mKfIp%2FeYBr9EVohTQ%2Fp4b4BsBkP5wBLD%2B1AEs%2FnD%2B1AEsAAAAAAIAZAAABLAEsAAVAC8AAAEzMhYVETMyFgcBBiInASY2OwERNDYBMzIWFREUBiMhIiY1ETQ2OwEyFh0BITU0NgImyBUdvxQLDf65DSYN%2FrkNCxS%2FHQJUMgoPDwr75goPDwoyCg8DhA8EsB0V%2Fj4XEP5wEBABkBAXAcIVHfzgDwr%2BogoPDwoBXgoPDwqvrwoPAAMAFwAXBJkEmQAPABsAMQAAADIeAhQOAiIuAjQ%2BAQQiDgEUHgEyPgE0JgUzMhYVETMyFgcDBiInAyY2OwERNDYB4%2BrWm1tbm9bq1ptbW5sBv%2BjFcnLF6MVycv58lgoPiRUKDd8NJg3fDQoViQ8EmVub1urWm1tbm9bq1ps7csXoxXJyxejFDQ8K%2Fu0XEP7tEBABExAXARMKDwAAAAMAFwAXBJkEmQAPABsAMQAAADIeAhQOAiIuAjQ%2BAQQiDgEUHgEyPgE0JiUTFgYrAREUBisBIiY1ESMiJjcTNjIB4%2BrWm1tbm9bq1ptbW5sBv%2BjFcnLF6MVycv7n3w0KFYkPCpYKD4kVCg3fDSYEmVub1urWm1tbm9bq1ps7csXoxXJyxejFAf7tEBf%2B7QoPDwoBExcQARMQAAAAAAIAAAAABLAEsAAZADkAABMhMhYXExYVERQGBwYjISImJyY1EzQ3Ez4BBSEiBgcDBhY7ATIWHwEeATsBMjY%2FAT4BOwEyNicDLgHhAu4KEwO6BwgFDBn7tAweAgYBB7kDEwKX%2FdQKEgJXAgwKlgoTAiYCEwr6ChMCJgITCpYKDAJXAhIEsA4K%2FXQYGf5XDB4CBggEDRkBqRkYAowKDsgOC%2F4%2BCw4OCpgKDg4KmAoODgsBwgsOAAMAFwAXBJkEmQAPABsAJwAAADIeAhQOAiIuAjQ%2BAQQiDgEUHgEyPgE0JgUXFhQPAQYmNRE0NgHj6tabW1ub1urWm1tbmwG%2F6MVycsXoxXJy%2Fov9ERH9EBgYBJlbm9bq1ptbW5vW6tabO3LF6MVycsXoxV2%2BDCQMvgwLFQGQFQsAAQAXABcEmQSwACgAAAE3NhYVERQGIyEiJj8BJiMiDgEUHgEyPgE1MxQOAiIuAjQ%2BAjMyA7OHBwsPCv6WCwQHhW2BdMVycsXoxXKWW5vW6tabW1ub1nXABCSHBwQL%2FpYKDwsHhUxyxejFcnLFdHXWm1tbm9bq1ptbAAAAAAIAFwABBJkEsAAaADUAAAE3NhYVERQGIyEiJj8BJiMiDgEVIzQ%2BAjMyEzMUDgIjIicHBiY1ETQ2MyEyFg8BFjMyPgEDs4cHCw8L%2FpcLBAeGboF0xXKWW5vWdcDrllub1nXAnIYHCw8LAWgKBQiFboJ0xXIEJIcHBAv%2BlwsPCweGS3LFdHXWm1v9v3XWm1t2hggFCgFoCw8LB4VMcsUAAAAKAGQAAASwBLAADwAfAC8APwBPAF8AbwB%2FAI8AnwAAEyEyFhURFAYjISImNRE0NgUhIgYVERQWMyEyNjURNCYFMzIWHQEUBisBIiY9ATQ2MyEyFh0BFAYjISImPQE0NgczMhYdARQGKwEiJj0BNDYzITIWHQEUBiMhIiY9ATQ2BzMyFh0BFAYrASImPQE0NjMhMhYdARQGIyEiJj0BNDYHMzIWHQEUBisBIiY9ATQ2MyEyFh0BFAYjISImPQE0Nn0EGgoPDwr75goPDwPA%2FK4KDw8KA1IKDw%2F9CDIKDw8KMgoPD9IBwgoPDwr%2BPgoPD74yCg8PCjIKDw%2FSAcIKDw8K%2Fj4KDw%2B%2BMgoPDwoyCg8P0gHCCg8PCv4%2BCg8PvjIKDw8KMgoPD9IBwgoPDwr%2BPgoPDwSwDwr7ggoPDwoEfgoPyA8K%2FK4KDw8KA1IKD2QPCjIKDw8KMgoPDwoyCg8PCjIKD8gPCjIKDw8KMgoPDwoyCg8PCjIKD8gPCjIKDw8KMgoPDwoyCg8PCjIKD8gPCjIKDw8KMgoPDwoyCg8PCjIKDwAAAAACAAAAAARMBLAAGQAjAAABNTQmIyEiBh0BIyIGFREUFjMhMjY1ETQmIyE1NDY7ATIWHQEDhHVT%2FtRSdmQpOzspA4QpOzsp%2FageFMgUHgMgyFN1dlLIOyn9qCk7OykCWCk7lhUdHRWWAAIAZAAABEwETAAJADcAABMzMhYVESMRNDYFMhcWFREUBw4DIyIuAScuAiMiBwYjIicmNRE%2BATc2HgMXHgIzMjc2fTIKD2QPA8AEBRADIUNAMRwaPyonKSxHHlVLBwgGBQ4WeDsXKC4TOQQpLUUdZ1AHBEwPCvvNBDMKDzACBhH%2BWwYGO1AkDQ0ODg8PDzkFAwcPAbY3VwMCAwsGFAEODg5XCAAAAwAAAAAEsASXACEAMQBBAAAAMh4CFREUBisBIiY1ETQuASAOARURFAYrASImNRE0PgEDMzIWFREUBisBIiY1ETQ2ITMyFhURFAYrASImNRE0NgHk6N6jYw8KMgoPjeT%2B%2BuSNDwoyCg9joyqgCAwMCKAIDAwCYKAIDAwIoAgMDASXY6PedP7UCg8PCgEsf9FyctF%2F%2FtQKDw8KASx03qP9wAwI%2FjQIDAwIAcwIDAwI%2FjQIDAwIAcwIDAAAAAACAAAA0wRHA90AFQA5AAABJTYWFREUBiclJisBIiY1ETQ2OwEyBTc2Mh8BFhQPARcWFA8BBiIvAQcGIi8BJjQ%2FAScmND8BNjIXAUEBAgkMDAn%2B%2FhUZ%2BgoPDwr6GQJYeAcUByIHB3h4BwciBxQHeHgHFAciBwd3dwcHIgcUBwMurAYHCv0SCgcGrA4PCgFeCg%2BEeAcHIgcUB3h4BxQHIgcHd3cHByIHFAd4eAcUByIICAAAAAACAAAA0wNyA90AFQAvAAABJTYWFREUBiclJisBIiY1ETQ2OwEyJTMWFxYVFAcGDwEiLwEuATc2NTQnJjY%2FATYBQQECCQwMCf7%2BFRn6Cg8PCvoZAdIECgZgWgYLAwkHHQcDBkhOBgMIHQcDLqwGBwr9EgoHBqwODwoBXgoPZAEJgaGafwkBAQYXBxMIZ36EaggUBxYFAAAAAAMAAADEBGID7AAbADEASwAAATMWFxYVFAYHBgcjIi8BLgE3NjU0JicmNj8BNgUlNhYVERQGJyUmKwEiJjURNDY7ATIlMxYXFhUUBwYPASIvAS4BNzY1NCcmNj8BNgPHAwsGh0RABwoDCQcqCAIGbzs3BgIJKgf9ggECCQwMCf7%2BFRn6Cg8PCvoZAdIECgZgWgYLAwkHHQcDBkhOBgMIHQcD7AEJs9lpy1QJAQYiBhQIlrJarEcJFAYhBb6sBgcK%2FRIKBwasDg8KAV4KD2QBCYGhmn8JAQEGFwcTCGd%2BhGoIFQYWBQAAAAANAAAAAASwBLAACQAVABkAHQAhACUALQA7AD8AQwBHAEsATwAAATMVIxUhFSMRIQEjFTMVIREjESM1IQURIREhESERBSM1MwUjNTMBMxEhETM1MwEzFSMVIzUjNTM1IzUhBREhEQcjNTMFIzUzASM1MwUhNSEB9GRk%2FnBkAfQCvMjI%2FtTIZAJY%2B7QBLAGQASz84GRkArxkZP1EyP4MyGQB9MhkyGRkyAEs%2FUQBLGRkZAOEZGT%2BDGRkAfT%2B1AEsA4RkZGQCWP4MZMgBLAEsyGT%2B1AEs%2FtQBLMhkZGT%2BDP4MAfRk%2FtRkZGRkyGTI%2FtQBLMhkZGT%2B1GRkZAAAAAAJAAAAAASwBLAAAwAHAAsADwATABcAGwAfACMAADcjETMTIxEzASMRMxMjETMBIxEzASE1IRcjNTMXIzUzBSM1M2RkZMhkZAGQyMjIZGQBLMjI%2FOD%2B1AEsyGRkyGRkASzIyMgD6PwYA%2Bj8GAPo%2FBgD6PwYA%2Bj7UGRkW1tbW1sAAAIAAAAKBKYEsAANABUAAAkBFhQHAQYiJwETNDYzBCYiBhQWMjYB9AKqCAj%2BMAgUCP1WAQ8KAUM7Uzs7UzsEsP1WCBQI%2FjAICAKqAdsKD807O1Q7OwAAAAADAAAACgXSBLAADQAZACEAAAkBFhQHAQYiJwETNDYzIQEWFAcBBiIvAQkBBCYiBhQWMjYB9AKqCAj%2BMAgUCP1WAQ8KAwYCqggI%2FjAIFAg4Aaj9RP7TO1M7O1M7BLD9VggUCP4wCAgCqgHbCg%2F9VggUCP4wCAg4AaoCvM07O1Q7OwAAAAABAGQAAASwBLAAJgAAASEyFREUDwEGJjURNCYjISIPAQYWMyEyFhURFAYjISImNRE0PwE2ASwDOUsSQAgKDwr9RBkSQAgFCgK8Cg8PCvyuCg8SixIEsEv8fBkSQAgFCgO2Cg8SQAgKDwr8SgoPDwoDzxkSixIAAAABAMj%2F%2FwRMBLAACgAAEyEyFhURCQERNDb6AyAVHf4%2B%2Fj4dBLAdFfuCAbz%2BQwR%2FFR0AAAAAAwAAAAAEsASwABUARQBVAAABISIGBwMGHwEeATMhMjY%2FATYnAy4BASMiBg8BDgEjISImLwEuASsBIgYVERQWOwEyNj0BNDYzITIWHQEUFjsBMjY1ETQmASEiBg8BBhYzITI2LwEuAQM2%2FkQLEAFOBw45BhcKAcIKFwY%2BDgdTARABVpYKFgROBBYK%2FdoKFgROBBYKlgoPDwqWCg8PCgLuCg8PCpYKDw%2F%2Bsf4MChMCJgILCgJYCgsCJgITBLAPCv7TGBVsCQwMCWwVGAEtCg%2F%2BcA0JnAkNDQmcCQ0PCv12Cg8PCpYKDw8KlgoPDwoCigoP%2FagOCpgKDg4KmAoOAAAAAAQAAABkBLAETAAdACEAKQAxAAABMzIeAh8BMzIWFREUBiMhIiY1ETQ2OwE%2BBAEVMzUEIgYUFjI2NCQyFhQGIiY0AfTIOF00JAcGlik7Oyn8GCk7OymWAgknM10ByGT%2Bz76Hh76H%2Fu9WPDxWPARMKTs7FRQ7Kf2oKTs7KQJYKTsIG0U1K%2F7UZGRGh76Hh74IPFY8PFYAAAAAAgA1AAAEsASvACAAIwAACQEWFx4BHwEVITUyNi8BIQYHBh4CMxUhNTY3PgE%2FAQEDIQMCqQGBFCgSJQkK%2Fl81LBFS%2Fnk6IgsJKjIe%2FpM4HAwaBwcBj6wBVKIEr%2FwaMioTFQECQkJXLd6RWSIuHAxCQhgcDCUNDQPu%2FVoByQAAAAADAGQAAAPwBLAAJwAyADsAAAEeBhUUDgMjITU%2BATURNC4EJzUFMh4CFRQOAgclMzI2NTQuAisBETMyNjU0JisBAvEFEzUwOyodN1htbDD%2BDCk7AQYLFyEaAdc5dWM%2BHy0tEP6Pi05pESpTPnbYUFJ9Xp8CgQEHGB0zOlIuQ3VONxpZBzMoAzsYFBwLEAkHRwEpSXNDM1s6KwkxYUopOzQb%2FK5lUFqBAAABAMgAAANvBLAAGQAAARcOAQcDBhYXFSE1NjcTNjQuBCcmJzUDbQJTQgeECSxK%2Fgy6Dq0DAw8MHxUXDQYEsDkTNSj8uTEoBmFhEFIDQBEaExAJCwYHAwI5AAAAAAL%2FtQAABRQEsAAlAC8AAAEjNC4FKwERFBYfARUhNTI%2BAzURIyIOBRUjESEFIxEzByczESM3BRQyCAsZEyYYGcgyGRn%2BcAQOIhoWyBkYJhMZCwgyA%2Bj7m0tLfX1LS30DhBUgFQ4IAwH8rhYZAQJkZAEFCRUOA1IBAwgOFSAVASzI%2FOCnpwMgpwACACH%2FtQSPBLAAJQAvAAABIzQuBSsBERQWHwEVITUyPgM1ESMiDgUVIxEhEwc1IRUnNxUhNQRMMggLGRMmGBnIMhkZ%2FnAEDiIaFsgZGCYTGQsIMgPoQ6f84KenAyADhBUgFQ4IAwH9dhYZAQJkZAEFCRUOAooBAwgOFSAVASz7gn1LS319S0sABAAAAAAEsARMAA8AHwAvAD8AABMhMhYdARQGIyEiJj0BNDYTITIWHQEUBiMhIiY9ATQ2EyEyFh0BFAYjISImPQE0NhMhMhYdARQGIyEiJj0BNDYyAlgVHR0V%2FagVHR0VA%2BgVHR0V%2FBgVHR0VAyAVHR0V%2FOAVHR0VBEwVHR0V%2B7QVHR0ETB0VZBUdHRVkFR3%2B1B0VZBUdHRVkFR3%2B1B0VZBUdHRVkFR3%2B1B0VZBUdHRVkFR0ABAAAAAAEsARMAA8AHwAvAD8AABMhMhYdARQGIyEiJj0BNDYDITIWHQEUBiMhIiY9ATQ2EyEyFh0BFAYjISImPQE0NgMhMhYdARQGIyEiJj0BNDb6ArwVHR0V%2FUQVHR2zBEwVHR0V%2B7QVHR3dArwVHR0V%2FUQVHR2zBEwVHR0V%2B7QVHR0ETB0VZBUdHRVkFR3%2B1B0VZBUdHRVkFR3%2B1B0VZBUdHRVkFR3%2B1B0VZBUdHRVkFR0ABAAAAAAEsARMAA8AHwAvAD8AAAE1NDYzITIWHQEUBiMhIiYBNTQ2MyEyFh0BFAYjISImEzU0NjMhMhYdARQGIyEiJgE1NDYzITIWHQEUBiMhIiYB9B0VAlgVHR0V%2FagVHf5wHRUD6BUdHRX8GBUdyB0VAyAVHR0V%2FOAVHf7UHRUETBUdHRX7tBUdA7ZkFR0dFWQVHR3%2B6WQVHR0VZBUdHf7pZBUdHRVkFR0d%2FulkFR0dFWQVHR0AAAQAAAAABLAETAAPAB8ALwA%2FAAATITIWHQEUBiMhIiY9ATQ2EyEyFh0BFAYjISImPQE0NhMhMhYdARQGIyEiJj0BNDYTITIWHQEUBiMhIiY9ATQ2MgRMFR0dFfu0FR0dFQRMFR0dFfu0FR0dFQRMFR0dFfu0FR0dFQRMFR0dFfu0FR0dBEwdFWQVHR0VZBUd%2FtQdFWQVHR0VZBUd%2FtQdFWQVHR0VZBUd%2FtQdFWQVHR0VZBUdAAgAAAAABLAETAAPAB8ALwA%2FAE8AXwBvAH8AABMzMhYdARQGKwEiJj0BNDYpATIWHQEUBiMhIiY9ATQ2ATMyFh0BFAYrASImPQE0NikBMhYdARQGIyEiJj0BNDYBMzIWHQEUBisBIiY9ATQ2KQEyFh0BFAYjISImPQE0NgEzMhYdARQGKwEiJj0BNDYpATIWHQEUBiMhIiY9ATQ2MmQVHR0VZBUdHQFBAyAVHR0V%2FOAVHR3%2B6WQVHR0VZBUdHQFBAyAVHR0V%2FOAVHR3%2B6WQVHR0VZBUdHQFBAyAVHR0V%2FOAVHR3%2B6WQVHR0VZBUdHQFBAyAVHR0V%2FOAVHR0ETB0VZBUdHRVkFR0dFWQVHR0VZBUd%2FtQdFWQVHR0VZBUdHRVkFR0dFWQVHf7UHRVkFR0dFWQVHR0VZBUdHRVkFR3%2B1B0VZBUdHRVkFR0dFWQVHR0VZBUdAAAG%2F5wAAASwBEwAAwATACMAKgA6AEoAACEjETsCMhYdARQGKwEiJj0BNDYTITIWHQEUBiMhIiY9ATQ2BQc1IzUzNQUhMhYdARQGIyEiJj0BNDYTITIWHQEUBiMhIiY9ATQ2AZBkZJZkFR0dFWQVHR0VAfQVHR0V%2FgwVHR3%2B%2BqfIyAHCASwVHR0V%2FtQVHR0VAlgVHR0V%2FagVHR0ETB0VZBUdHRVkFR3%2B1B0VZBUdHRVkFR36fUtkS68dFWQVHR0VZBUd%2FtQdFWQVHR0VZBUdAAAABgAAAAAFFARMAA8AEwAjACoAOgBKAAATMzIWHQEUBisBIiY9ATQ2ASMRMwEhMhYdARQGIyEiJj0BNDYFMxUjFSc3BSEyFh0BFAYjISImPQE0NhMhMhYdARQGIyEiJj0BNDYyZBUdHRVkFR0dA2dkZPyuAfQVHR0V%2FgwVHR0EL8jIp6f75gEsFR0dFf7UFR0dFQJYFR0dFf2oFR0dBEwdFWQVHR0VZBUd%2B7QETP7UHRVkFR0dFWQVHchkS319rx0VZBUdHRVkFR3%2B1B0VZBUdHRVkFR0AAAAAAgAAAMgEsAPoAA8AEgAAEyEyFhURFAYjISImNRE0NgkCSwLuHywsH%2F0SHywsBIT%2B1AEsA%2BgsH%2F12HywsHwKKHyz9RAEsASwAAwAAAAAEsARMAA8AFwAfAAATITIWFREUBiMhIiY1ETQ2FxE3BScBExEEMhYUBiImNCwEWBIaGhL7qBIaGkr3ASpKASXs%2FNJwTk5wTgRMGhL8DBIaGhID9BIaZP0ftoOcAT7%2B4AH0dE5vT09vAAAAAAIA2wAFBDYEkQAWAB4AAAEyHgEVFAcOAQ8BLgQnJjU0PgIWIgYUFjI2NAKIdcZzRkWyNjYJIV5YbSk8RHOft7eCgreCBJF4ynVzj23pPz4IIWZomEiEdVijeUjDgriBgbgAAAACABcAFwSZBJkADwAXAAAAMh4CFA4CIi4CND4BAREiDgEUHgEB4%2BrWm1tbm9bq1ptbW5sBS3TFcnLFBJlbm9bq1ptbW5vW6tab%2FG8DVnLF6MVyAAACAHUAAwPfBQ8AGgA1AAABHgYVFA4DBy4DNTQ%2BBQMOAhceBBcWNj8BNiYnLgInJjc2IyYCKhVJT1dOPiUzVnB9P1SbfEokP0xXUEm8FykoAwEbITEcExUWAgYCCQkFEikMGiACCAgFD0iPdXdzdYdFR4BeRiYEBTpjl1lFh3ZzeHaQ%2Ff4hS4I6JUEnIw4IBwwQIgoYBwQQQSlZtgsBAAAAAwAAAAAEywRsAAwAKgAvAAABNz4CHgEXHgEPAiUhMhcHISIGFREUFjMhMjY9ATcRFAYjISImNRE0NgkBBzcBA%2BhsAgYUFR0OFgoFBmz9BQGQMje7%2FpApOzspAfQpO8i7o%2F5wpbm5Azj%2BlqE3AWMD9XMBAgIEDw4WKgsKc8gNuzsp%2FgwpOzsptsj%2BtKW5uaUBkKW5%2Ftf%2BljKqAWMAAgAAAAAEkwRMABsANgAAASEGByMiBhURFBYzITI2NTcVFAYjISImNRE0NgUBFhQHAQYmJzUmDgMHPgY3NT4BAV4BaaQ0wyk7OykB9Ck7yLml%2FnClubkCfwFTCAj%2BrAcLARo5ZFRYGgouOUlARioTAQsETJI2Oyn%2BDCk7OymZZ6W5uaUBkKW5G%2F7TBxUH%2Fs4GBAnLAQINFjAhO2JBNB0UBwHSCgUAAAAAAgAAAAAEnQRMAB0ANQAAASEyFwchIgYVERQWMyEyNj0BNxUUBiMhIiY1ETQ2CQE2Mh8BFhQHAQYiLwEmND8BNjIfARYyAV4BXjxDsv6jKTs7KQH0KTvIuaX%2BcKW5uQHKAYsHFQdlBwf97QcVB%2FgHB2UHFQdvCBQETBexOyn%2BDCk7OylFyNulubmlAZCluf4zAYsHB2UHFQf97AcH%2BAcVB2UHB28HAAAAAQAKAAoEpgSmADsAAAkBNjIXARYGKwEVMzU0NhcBFhQHAQYmPQEjFTMyFgcBBiInASY2OwE1IxUUBicBJjQ3ATYWHQEzNSMiJgE%2BAQgIFAgBBAcFCqrICggBCAgI%2FvgICsiqCgUH%2FvwIFAj%2B%2BAgFCq%2FICgj%2B%2BAgIAQgICsivCgUDlgEICAj%2B%2BAgKyK0KBAf%2B%2FAcVB%2F73BwQKrcgKCP74CAgBCAgKyK0KBAcBCQcVBwEEBwQKrcgKAAEAyAAAA4QETAAZAAATMzIWFREBNhYVERQGJwERFAYrASImNRE0NvpkFR0B0A8VFQ%2F%2BMB0VZBUdHQRMHRX%2BSgHFDggV%2FBgVCA4Bxf5KFR0dFQPoFR0AAAABAAAAAASwBEwAIwAAEzMyFhURATYWFREBNhYVERQGJwERFAYnAREUBisBIiY1ETQ2MmQVHQHQDxUB0A8VFQ%2F%2BMBUP%2FjAdFWQVHR0ETB0V%2FkoBxQ4IFf5KAcUOCBX8GBUIDgHF%2FkoVCA4Bxf5KFR0dFQPoFR0AAAABAJ0AGQSwBDMAFQAAAREUBicBERQGJwEmNDcBNhYVEQE2FgSwFQ%2F%2BMBUP%2FhQPDwHsDxUB0A8VBBr8GBUIDgHF%2FkoVCA4B4A4qDgHgDggV%2FkoBxQ4IAAAAAQDIABYEMwQ2AAsAABMBFhQHAQYmNRE0NvMDLhIS%2FNISGRkEMv4OCx4L%2Fg4LDhUD6BUOAAIAyABkA4QD6AAPAB8AABMzMhYVERQGKwEiJjURNDYhMzIWFREUBisBIiY1ETQ2%2BsgVHR0VyBUdHQGlyBUdHRXIFR0dA%2BgdFfzgFR0dFQMgFR0dFfzgFR0dFQMgFR0AAAEAyABkBEwD6AAPAAABERQGIyEiJjURNDYzITIWBEwdFfzgFR0dFQMgFR0DtvzgFR0dFQMgFR0dAAAAAAEAAAAZBBMEMwAVAAABETQ2FwEWFAcBBiY1EQEGJjURNDYXAfQVDwHsDw%2F%2BFA8V%2FjAPFRUPAmQBthUIDv4gDioO%2FiAOCBUBtv47DggVA%2BgVCA4AAAH%2F%2FgACBLMETwAjAAABNzIWFRMUBiMHIiY1AwEGJjUDAQYmNQM0NhcBAzQ2FwEDNDYEGGQUHgUdFWQVHQL%2BMQ4VAv4yDxUFFQ8B0gIVDwHSAh0ETgEdFfwYFR0BHRUBtf46DwkVAbX%2BOQ4JFAPoFQkP%2Fj4BthQJDv49AbYVHQAAAQEsAAAD6ARMABkAAAEzMhYVERQGKwEiJjURAQYmNRE0NhcBETQ2A1JkFR0dFWQVHf4wDxUVDwHQHQRMHRX8GBUdHRUBtv47DggVA%2BgVCA7%2BOwG2FR0AAAIAZADIBLAESAALABsAAAkBFgYjISImNwE2MgEhMhYdARQGIyEiJj0BNDYCrgH1DwkW%2B%2B4WCQ8B9Q8q%2FfcD6BUdHRX8GBUdHQQ5%2FeQPFhYPAhwP%2FUgdFWQVHR0VZBUdAAEAiP%2F8A3UESgAFAAAJAgcJAQN1%2FqABYMX92AIoA4T%2Bn%2F6fxgIoAiYAAAAAAQE7%2F%2FwEKARKAAUAAAkBJwkBNwQo%2FdnGAWH%2Bn8YCI%2F3ZxgFhAWHGAAIAFwAXBJkEmQAPADMAAAAyHgIUDgIiLgI0PgEFIyIGHQEjIgYdARQWOwEVFBY7ATI2PQEzMjY9ATQmKwE1NCYB4%2BrWm1tbm9bq1ptbW5sBfWQVHZYVHR0Vlh0VZBUdlhUdHRWWHQSZW5vW6tabW1ub1urWm7odFZYdFWQVHZYVHR0Vlh0VZBUdlhUdAAAAAAIAFwAXBJkEmQAPAB8AAAAyHgIUDgIiLgI0PgEBISIGHQEUFjMhMjY9ATQmAePq1ptbW5vW6tabW1ubAkX%2BDBUdHRUB9BUdHQSZW5vW6tabW1ub1urWm%2F5%2BHRVkFR0dFWQVHQACABcAFwSZBJkADwAzAAAAMh4CFA4CIi4CND4BBCIPAScmIg8BBhQfAQcGFB8BFjI%2FARcWMj8BNjQvATc2NC8BAePq1ptbW5vW6tabW1ubAeUZCXh4CRkJjQkJeHgJCY0JGQl4eAkZCY0JCXh4CQmNBJlbm9bq1ptbW5vW6tabrQl4eAkJjQkZCXh4CRkJjQkJeHgJCY0JGQl4eAkZCY0AAgAXABcEmQSZAA8AJAAAADIeAhQOAiIuAjQ%2BAQEnJiIPAQYUHwEWMjcBNjQvASYiBwHj6tabW1ub1urWm1tbmwEVVAcVCIsHB%2FIHFQcBdwcHiwcVBwSZW5vW6tabW1ub1urWm%2F4xVQcHiwgUCPEICAF3BxUIiwcHAAAAAAMAFwAXBJkEmQAPADsASwAAADIeAhQOAiIuAjQ%2BAQUiDgMVFDsBFjc%2BATMyFhUUBgciDgUHBhY7ATI%2BAzU0LgMTIyIGHQEUFjsBMjY9ATQmAePq1ptbW5vW6tabW1ubAT8dPEIyIRSDHgUGHR8UFw4TARkOGhITDAIBDQ6tBx4oIxgiM0Q8OpYKDw8KlgoPDwSZW5vW6tabW1ub1urWm5ELHi9PMhkFEBQQFRIXFgcIBw4UHCoZCBEQKDhcNi9IKhsJ%2FeMPCpYKDw8KlgoPAAADABcAFwSZBJkADwAfAD4AAAAyHgIUDgIiLgI0PgEFIyIGHQEUFjsBMjY9ATQmAyMiBh0BFBY7ARUjIgYdARQWMyEyNj0BNCYrARE0JgHj6tabW1ub1urWm1tbmwGWlgoPDwqWCg8PCvoKDw8KS0sKDw8KAV4KDw8KSw8EmVub1urWm1tbm9bq1ptWDwqWCg8PCpYKD%2F7UDwoyCg%2FIDwoyCg8PCjIKDwETCg8AAgAAAAAEsASwAC8AXwAAATMyFh0BHgEXMzIWHQEUBisBDgEHFRQGKwEiJj0BLgEnIyImPQE0NjsBPgE3NTQ2ExUUBisBIiY9AQ4BBzMyFh0BFAYrAR4BFzU0NjsBMhYdAT4BNyMiJj0BNDY7AS4BAg2WCg9nlxvCCg8PCsIbl2cPCpYKD2eXG8IKDw8KwhuXZw%2B5DwqWCg9EZheoCg8PCqgXZkQPCpYKD0RmF6gKDw8KqBdmBLAPCsIbl2cPCpYKD2eXG8IKDw8KwhuXZw8KlgoPZ5cbwgoP%2Fs2oCg8PCqgXZkQPCpYKD0RmF6gKDw8KqBdmRA8KlgoPRGYAAwAXABcEmQSZAA8AGwA%2FAAAAMh4CFA4CIi4CND4BBCIOARQeATI%2BATQmBxcWFA8BFxYUDwEGIi8BBwYiLwEmND8BJyY0PwE2Mh8BNzYyAePq1ptbW5vW6tabW1ubAb%2FoxXJyxejFcnKaQAcHfHwHB0AHFQd8fAcVB0AHB3x8BwdABxUHfHwHFQSZW5vW6tabW1ub1urWmztyxejFcnLF6MVaQAcVB3x8BxUHQAcHfHwHB0AHFQd8fAcVB0AHB3x8BwAAAAMAFwAXBJkEmQAPABsAMAAAADIeAhQOAiIuAjQ%2BAQQiDgEUHgEyPgE0JgcXFhQHAQYiLwEmND8BNjIfATc2MgHj6tabW1ub1urWm1tbmwG%2F6MVycsXoxXJyg2oHB%2F7ACBQIyggIagcVB0%2FFBxUEmVub1urWm1tbm9bq1ps7csXoxXJyxejFfWoHFQf%2BvwcHywcVB2oICE%2FFBwAAAAMAFwAXBJkEmQAPABgAIQAAADIeAhQOAiIuAjQ%2BAQUiDgEVFBcBJhcBFjMyPgE1NAHj6tabW1ub1urWm1tbmwFLdMVyQQJLafX9uGhzdMVyBJlbm9bq1ptbW5vW6tabO3LFdHhpAktB0P24PnLFdHMAAAAAAQAXAFMEsAP5ABUAABMBNhYVESEyFh0BFAYjIREUBicBJjQnAgoQFwImFR0dFf3aFxD99hACRgGrDQoV%2Ft0dFcgVHf7dFQoNAasNJgAAAAABAAAAUwSZA%2FkAFQAACQEWFAcBBiY1ESEiJj0BNDYzIRE0NgJ%2FAgoQEP32EBf92hUdHRUCJhcD8f5VDSYN%2FlUNChUBIx0VyBUdASMVCgAAAAEAtwAABF0EmQAVAAAJARYGIyERFAYrASImNREhIiY3ATYyAqoBqw0KFf7dHRXIFR3%2B3RUKDQGrDSYEif32EBf92hUdHRUCJhcQAgoQAAAAAQC3ABcEXQSwABUAAAEzMhYVESEyFgcBBiInASY2MyERNDYCJsgVHQEjFQoN%2FlUNJg3%2BVQ0KFQEjHQSwHRX92hcQ%2FfYQEAIKEBcCJhUdAAABAAAAtwSZBF0AFwAACQEWFAcBBiY1EQ4DBz4ENxE0NgJ%2FAgoQEP32EBdesKWBJAUsW4fHfhcEVf5VDSYN%2FlUNChUBIwIkRHVNabGdcUYHAQYVCgACAAAAAASwBLAAFQArAAABITIWFREUBi8BBwYiLwEmND8BJyY2ASEiJjURNDYfATc2Mh8BFhQPARcWBgNSASwVHRUOXvkIFAhqBwf5Xg4I%2FiH%2B1BUdFQ5e%2BQgUCGoHB%2FleDggEsB0V%2FtQVCA5e%2BQcHaggUCPleDhX7UB0VASwVCA5e%2BQcHaggUCPleDhUAAAACAEkASQRnBGcAFQArAAABFxYUDwEXFgYjISImNRE0Nh8BNzYyASEyFhURFAYvAQcGIi8BJjQ%2FAScmNgP2agcH%2BV4OCBX%2B1BUdFQ5e%2BQgU%2FQwBLBUdFQ5e%2BQgUCGoHB%2FleDggEYGoIFAj5Xg4VHRUBLBUIDl75B%2F3xHRX%2B1BUIDl75BwdqCBQI%2BV4OFQAAAAADABcAFwSZBJkADwAfAC8AAAAyHgIUDgIiLgI0PgEFIyIGFxMeATsBMjY3EzYmAyMiBh0BFBY7ATI2PQE0JgHj6tabW1ub1urWm1tbmwGz0BQYBDoEIxQ2FCMEOgQYMZYKDw8KlgoPDwSZW5vW6tabW1ub1urWm7odFP7SFB0dFAEuFB3%2BDA8KlgoPDwqWCg8AAAAABQAAAAAEsASwAEkAVQBhAGgAbwAAATIWHwEWHwEWFxY3Nj8BNjc2MzIWHwEWHwIeATsBMhYdARQGKwEiBh0BIREjESE1NCYrASImPQE0NjsBMjY1ND8BNjc%2BBAUHBhY7ATI2LwEuAQUnJgYPAQYWOwEyNhMhIiY1ESkBERQGIyERAQQJFAUFFhbEFQ8dCAsmxBYXERUXMA0NDgQZCAEPCj0KDw8KMgoP%2FnDI%2FnAPCjIKDw8KPQsOCRkFDgIGFRYfAp2mBwQK2woKAzMDEP41sQgQAzMDCgrnCwMe%2FokKDwGQAlgPCv6JBLAEAgIKDXYNCxUJDRZ2DQoHIREQFRh7LAkLDwoyCg8PCq8BLP7UrwoPDwoyCg8GBQQwgBkUAwgWEQ55ogcKDgqVCgSqnQcECo8KDgr8cg8KAXf%2BiQoPAZAAAAAAAgAAAAwErwSmACsASQAAATYWFQYCDgQuAScmByYOAQ8BBiY1NDc%2BATc%2BAScuAT4BNz4GFyYGBw4BDwEOBAcOARY2Nz4CNz4DNz4BBI0IGgItQmxhi2KORDg9EQQRMxuZGhYqCFUYEyADCQIQOjEnUmFch3vAJQgdHyaiPT44XHRZUhcYDhItIRmKcVtGYWtbKRYEBKYDEwiy%2Ft3IlVgxEQgLCwwBAQIbG5kYEyJAJghKFRE8Hzdff4U%2FM0o1JSMbL0QJGCYvcSEhHjZST2c1ODwEJygeW0AxJUBff1UyFAABAF0AHgRyBM8ATwAAAQ4BHgQXLgc%2BATceAwYHDgQHBicmNzY3PgQuAScWDgMmJy4BJyY%2BBDcGHgM3PgEuAicmPgMCjScfCic4R0IgBBsKGAoQAwEJEg5gikggBhANPkpTPhZINx8SBgsNJysiCRZOQQoVNU1bYC9QZwICBAUWITsoCAYdJzIYHw8YIiYHDyJJYlkEz0OAZVxEOSQMBzgXOB42IzElKRIqg5Gnl0o3Z0c6IAYWCwYNAwQFIDhHXGF1OWiqb0sdBxUknF0XNTQ8PEUiNWNROBYJDS5AQVUhVZloUSkAAAAAA%2F%2FcAGoE1ARGABsAPwBRAAAAMh4FFA4FIi4FND4EBSYGFxYVFAYiJjU0NzYmBwYHDgEXHgQyPgM3NiYnJgUHDgEXFhcWNj8BNiYnJicuAQIGpJ17bk85HBw6T257naKde25POhwcOU9uewIPDwYIGbD4sBcIBw5GWg0ECxYyWl%2BDiINfWjIWCwQMWv3%2FIw8JCSU4EC0OIw4DDywtCyIERi1JXGJcSSpJXGJcSS0tSVxiXEkqSVxiXEncDwYTOT58sLB8OzcTBg9FcxAxEiRGXkQxMEVeRSQSMRF1HiQPLxJEMA0EDyIPJQ8sSRIEAAAABP%2FcAAAE1ASwABQAJwA7AEwAACEjNy4ENTQ%2BBTMyFzczEzceARUUDgMHNz4BNzYmJyYlBgcOARceBBc3LgE1NDc2JhcHDgEXFhcWNj8CJyYnLgECUJQfW6l2WSwcOU9ue51SPUEglCYvbIknUGqYUi5NdiYLBAw2%2FVFGWg0ECxIqSExoNSlrjxcIB3wjDwkJJTgQLQ4MFgMsLQsieBRhdHpiGxVJXGJcSS0Pef5StVXWNBpacm5jGq0xiD8SMRFGckVzEDESHjxRQTkNmhKnbjs3EwZwJA8vEkQwDQQPC1YELEkSBAAAAAP%2FngAABRIEqwALABgAKAAAJwE2FhcBFgYjISImJSE1NDY7ATIWHQEhAQczMhYPAQ4BKwEiJi8BJjZaAoIUOBQCghUbJfryJRsBCgFZDwqWCg8BWf5DaNAUGAQ6BCMUNhQjBDoEGGQEKh8FIfvgIEdEhEsKDw8KSwLT3x0U%2FBQdHRT8FB0AAAABAGQAFQSwBLAAKAAAADIWFREBHgEdARQGJyURFh0BFAYvAQcGJj0BNDcRBQYmPQE0NjcBETQCTHxYAWsPFhgR%2FplkGhPNzRMaZP6ZERgWDwFrBLBYPv6t%2FrsOMRQpFA0M%2Bf75XRRAFRAJgIAJEBVAFF0BB%2FkMDRQpFDEOAUUBUz4AAAARAAAAAARMBLAAHQAnACsALwAzADcAOwA%2FAEMARwBLAE8AUwBXAFsAXwBjAAABMzIWHQEzMhYdASE1NDY7ATU0NjsBMhYdASE1NDYBERQGIyEiJjURFxUzNTMVMzUzFTM1MxUzNTMVMzUFFTM1MxUzNTMVMzUzFTM1MxUzNQUVMzUzFTM1MxUzNTMVMzUzFTM1A1JkFR0yFR37tB0VMh0VZBUdAfQdAQ8dFfwYFR1kZGRkZGRkZGRk%2FHxkZGRkZGRkZGT8fGRkZGRkZGRkZASwHRUyHRWWlhUdMhUdHRUyMhUd%2FnD9EhUdHRUC7shkZGRkZGRkZGRkyGRkZGRkZGRkZGTIZGRkZGRkZGRkZAAAAAMAAAAZBXcElwAZACUANwAAARcWFA8BBiY9ASMBISImPQE0NjsBATM1NDYBBycjIiY9ATQ2MyEBFxYUDwEGJj0BIyc3FzM1NDYEb%2FkPD%2FkOFZ%2F9qP7dFR0dFdECWPEV%2FamNetEVHR0VASMDGvkPD%2FkOFfG1jXqfFQSN5g4qDuYOCBWW%2FagdFWQVHQJYlhUI%2FpiNeh0VZBUd%2Fk3mDioO5g4IFZa1jXqWFQgAAAABAAAAAASwBEwAEgAAEyEyFhURFAYjIQERIyImNRE0NmQD6Ck7Oyn9rP7QZCk7OwRMOyn9qCk7%2FtQBLDspAlgpOwAAAAMAZAAABEwEsAAJABMAPwAAEzMyFh0BITU0NiEzMhYdASE1NDYBERQOBSIuBTURIRUUFRwBHgYyPgYmNTQ9AZbIFR3%2B1B0C0cgVHf7UHQEPBhgoTGacwJxmTCgYBgEsAwcNFB8nNkI2Jx8TDwUFAQSwHRX6%2BhUdHRX6%2BhUd%2FnD%2B1ClJalZcPigoPlxWakkpASz6CRIVKyclIRsWEAgJEBccISUnKhURCPoAAAAB%2F%2F8A1ARMA8IABQAAAQcJAScBBEzG%2Fp%2F%2Bn8UCJwGbxwFh%2Fp%2FHAicAAQAAAO4ETQPcAAUAAAkCNwkBBE392v3ZxgFhAWEDFf3ZAifH%2Fp8BYQAAAAAC%2F1EAZAVfA%2BgAFAApAAABITIWFREzMhYPAQYiLwEmNjsBESElFxYGKwERIRchIiY1ESMiJj8BNjIBlALqFR2WFQgO5g4qDuYOCBWW%2FoP%2BHOYOCBWWAYHX%2FRIVHZYVCA7mDioD6B0V%2FdkVDvkPD%2FkOFQGRuPkOFf5wyB0VAiYVDvkPAAABAAYAAASeBLAAMAAAEzMyFh8BITIWBwMOASMhFyEyFhQGKwEVFAYiJj0BIRUUBiImPQEjIiYvAQMjIiY0NjheERwEJgOAGB4FZAUsIf2HMAIXFR0dFTIdKh3%2B1B0qHR8SHQYFyTYUHh4EsBYQoiUY%2FiUVK8gdKh0yFR0dFTIyFR0dFTIUCQoDwR0qHQAAAAACAAAAAASwBEwACwAPAAABFSE1MzQ2MyEyFhUFIREhBLD7UMg7KQEsKTv9RASw%2B1AD6GRkKTs7Kcj84AACAAAAAAXcBEwADAAQAAATAxEzNDYzITIWFSEVBQEhAcjIyDspASwqOgH0ASz%2B1PtQASwDIP5wAlgpOzspyGT9RAK8AAEBRQAAA2sErwAbAAABFxYGKwERMzIWDwEGIi8BJjY7AREjIiY%2FATYyAnvmDggVlpYVCA7mDioO5g4IFZaWFQgO5g4qBKD5DhX9pxUO%2BQ8P%2BQ4VAlkVDvkPAAAAAQABAUQErwNrABsAAAEXFhQPAQYmPQEhFRQGLwEmND8BNhYdASE1NDYDqPkODvkPFf2oFQ%2F5Dg75DxUCWBUDYOUPKQ%2FlDwkUl5cUCQ%2FlDykP5Q8JFZWVFQkAAAAEAAAAAASwBLAACQAZAB0AIQAAAQMuASMhIgYHAwUhIgYdARQWMyEyNj0BNCYFNTMVMzUzFQSRrAUkFP1gFCQFrAQt%2FBgpOzspA%2BgpOzv%2Bq2RkZAGQAtwXLSgV%2FR1kOylkKTs7KWQpO8hkZGRkAAAAA%2F%2BcAGQEsARMAAsAIwAxAAAAMhYVERQGIiY1ETQDJSMTFgYjIisBIiYnAj0BNDU0PgE7ASUBFSIuAz0BND4CNwRpKh0dKh1k%2FV0mLwMRFQUCVBQdBDcCCwzIAqP8GAQOIhoWFR0dCwRMHRX8rhUdHRUDUhX8mcj%2B7BAIHBUBUQ76AgQQDw36%2FtT6AQsTKRwyGigUDAEAAAACAEoAAARmBLAALAA1AAABMzIWDwEeARcTFzMyFhQGBw4EIyIuBC8BLgE0NjsBNxM%2BATcnJjYDFjMyNw4BIiYCKV4UEgYSU3oPP3YRExwaEggeZGqfTzl0XFU%2BLwwLEhocExF2Pw96UxIGEyQyNDUxDDdGOASwFRMlE39N%2FrmtHSkoBwQLHBYSCg4REg4FBAgoKR2tAUdNfhQgExr7vgYGMT09AAEAFAAUBJwEnAAXAAABNwcXBxcHFycHJwcnBzcnNyc3Jxc3FzcDIOBO6rS06k7gLZubLeBO6rS06k7gLZubA7JO4C2bmy3gTuq0tOpO4C2bmy3gTuq0tAADAAAAZASwBLAAIQAtAD0AAAEzMhYdAQchMhYdARQHAw4BKwEiJi8BIyImNRE0PwI%2BARcPAREzFzMTNSE3NQEzMhYVERQGKwEiJjURNDYCijIoPBwBSCg8He4QLBf6B0YfHz0tNxSRYA0xG2SWZIjW%2Bv4%2BMv12ZBUdHRVkFR0dBLBRLJZ9USxkLR3%2BqBghMhkZJCcBkCQbxMYcKGTU1f6JZAF3feGv%2FtQdFf4MFR0dFQH0FR0AAAAAAwAAAAAEsARMACAAMAA8AAABMzIWFxMWHQEUBiMhFh0BFAYrASImLwImNRE0NjsBNgUzMhYVERQGKwEiJjURNDYhByMRHwEzNSchNQMCWPoXLBDuHTwo%2FrgcPCgyGzENYJEUNy09fP3pZBUdHRVkFR0dAl%2BIZJZkMjIBwvoETCEY%2FqgdLWQsUXYHlixRKBzGxBskAZAnJGRkHRX%2BDBUdHRUB9BUdZP6J1dSv4X0BdwADAAAAZAUOBE8AGwA3AEcAAAElNh8BHgEPASEyFhQGKwEDDgEjISImNRE0NjcXERchEz4BOwEyNiYjISoDLgQnJj8BJwUzMhYVERQGKwEiJjURNDYBZAFrHxZuDQEMVAEuVGxuVGqDBhsP%2FqoHphwOOmQBJYMGGw%2FLFRMSFv44AgoCCQMHAwUDAQwRklb9T2QVHR0VZBUdHQNp5hAWcA0mD3lMkE7%2BrRUoog0CDRElCkj%2BCVkBUxUoMjIBAgIDBQIZFrdT5B0V%2FgwVHR0VAfQVHQAAAAP%2FnABkBLAETwAdADYARgAAAQUeBBURFAYjISImJwMjIiY0NjMhJyY2PwE2BxcWBw4FKgIjIRUzMhYXEyE3ESUFMzIWFREUBisBIiY1ETQ2AdsBbgIIFBANrAf%2Bqg8bBoNqVW1sVAEuVQsBDW4WSpIRDAIDBQMHAwkDCgH%2BJd0PHAaCASZq%2FqoCUGQVHR0VZBUdHQRP5gEFEBEXC%2F3zDaIoFQFTTpBMeQ8mDXAWrrcWGQIFAwICAWQoFf6tWQH37OQdFf4MFR0dFQH0FR0AAAADAGEAAARMBQ4AGwA3AEcAAAAyFh0BBR4BFREUBiMhIiYvAQMmPwE%2BAR8BETQXNTQmBhURHAMOBAcGLwEHEyE3ESUuAQMhMhYdARQGIyEiJj0BNDYB3pBOAVMVKKIN%2FfMRJQoJ5hAWcA0mD3nGMjIBAgIDBQIZFrdT7AH3Wf6tFSiWAfQVHR0V%2FgwVHR0FDm5UaoMGGw%2F%2BqgemHA4OAWsfFm4NAQxUAS5U1ssVExIW%2FjgCCgIJAwcDBQMBDBGSVv6tZAElgwYb%2FQsdFWQVHR0VZBUdAAP%2F%2FQAGA%2BgFFAAPAC0ASQAAASEyNj0BNCYjISIGHQEUFgEVFAYiJjURBwYmLwEmNxM%2BBDMhMhYVERQGBwEDFzc2Fx4FHAIVERQWNj0BNDY3JREnAV4B9BUdHRX%2BDBUdHQEPTpBMeQ8mDXAWEOYBBRARFwsCDQ2iKBX9iexTtxYZAgUDAgIBMjIoFQFTWQRMHRVkFR0dFWQVHfzmalRubFQBLlQMAQ1uFh8BawIIEw8Mpgf%2Bqg8bBgHP%2Fq1WkhEMAQMFAwcDCQIKAv44FhITFcsPGwaDASVkAAIAFgAWBJoEmgAPACUAAAAyHgIUDgIiLgI0PgEBJSYGHQEhIgYdARQWMyEVFBY3JTY0AeLs1ptbW5vW7NabW1ubAob%2B7RAX%2Fu0KDw8KARMXEAETEASaW5vW7NabW1ub1uzWm%2F453w0KFYkPCpYKD4kVCg3fDSYAAAIAFgAWBJoEmgAPACUAAAAyHgIUDgIiLgI0PgENAQYUFwUWNj0BITI2PQE0JiMhNTQmAeLs1ptbW5vW7NabW1ubASX%2B7RAQARMQFwETCg8PCv7tFwSaW5vW7NabW1ub1uzWm%2BjfDSYN3w0KFYkPCpYKD4kVCgAAAAIAFgAWBJoEmgAPACUAAAAyHgIUDgIiLgI0PgEBAyYiBwMGFjsBERQWOwEyNjURMzI2AeLs1ptbW5vW7NabW1ubAkvfDSYN3w0KFYkPCpYKD4kVCgSaW5vW7NabW1ub1uzWm%2F5AARMQEP7tEBf%2B7QoPDwoBExcAAAIAFgAWBJoEmgAPACUAAAAyHgIUDgIiLgI0PgEFIyIGFREjIgYXExYyNxM2JisBETQmAeLs1ptbW5vW7NabW1ubAZeWCg%2BJFQoN3w0mDd8NChWJDwSaW5vW7NabW1ub1uzWm7sPCv7tFxD%2B7RAQARMQFwETCg8AAAMAGAAYBJgEmAAPAJYApgAAADIeAhQOAiIuAjQ%2BASUOAwcGJgcOAQcGFgcOAQcGFgcUFgcyHgEXHgIXHgI3Fg4BFx4CFxQGFBcWNz4CNy4BJy4BJyIOAgcGJyY2NS4BJzYuAQYHBicmNzY3HgIXHgMfAT4CJyY%2BATc%2BAzcmNzIWMjY3LgMnND4CJiceAT8BNi4CJwYHFB4BFS4CJz4BNxYyPgEB5OjVm1xcm9Xo1ZtcXJsBZA8rHDoKDz0PFD8DAxMBAzEFCRwGIgEMFhkHECIvCxU%2FOR0HFBkDDRQjEwcFaHUeISQDDTAMD0UREi4oLBAzDwQBBikEAQMLGhIXExMLBhAGKBsGBxYVEwYFAgsFAwMNFwQGCQcYFgYQCCARFwkKKiFBCwQCAQMDHzcLDAUdLDgNEiEQEgg%2FKhADGgMKEgoRBJhcm9Xo1ZtcXJvV6NWbEQwRBwkCAwYFBycPCxcHInIWInYcCUcYChQECA4QBAkuHgQPJioRFRscBAcSCgwCch0kPiAIAQcHEAsBAgsLIxcBMQENCQIPHxkCFBkdHB4QBgEBBwoMGBENBAMMJSAQEhYXDQ4qFBkKEhIDCQsXJxQiBgEOCQwHAQ0DBAUcJAwSCwRnETIoAwEJCwsLJQcKDBEAAAAAAQAAAAIErwSFABYAAAE2FwUXNxYGBw4BJwEGIi8BJjQ3ASY2AvSkjv79kfsGUE08hjv9rA8rD28PDwJYIk8EhVxliuh%2BWYcrIgsW%2FawQEG4PKxACV2XJAAYAAABgBLAErAAPABMAIwAnADcAOwAAEyEyFh0BFAYjISImPQE0NgUjFTMFITIWHQEUBiMhIiY9ATQ2BSEVIQUhMhYdARQGIyEiJj0BNDYFIRUhZAPoKTs7KfwYKTs7BBHIyPwYA%2BgpOzsp%2FBgpOzsEEf4MAfT8GAPoKTs7KfwYKTs7BBH%2B1AEsBKw7KWQpOzspZCk7ZGTIOylkKTs7KWQpO2RkyDspZCk7OylkKTtkZAAAAAIAZAAABEwEsAALABEAABMhMhYUBiMhIiY0NgERBxEBIZYDhBUdHRX8fBUdHQI7yP6iA4QEsB0qHR0qHf1E%2FtTIAfQB9AAAAAMAAABkBLAEsAAXABsAJQAAATMyFh0BITIWFREhNSMVIRE0NjMhNTQ2FxUzNQEVFAYjISImPQEB9MgpOwEsKTv%2BDMj%2BDDspASw7KcgB9Dsp%2FBgpOwSwOylkOyn%2BcGRkAZApO2QpO2RkZP1EyCk7OynIAAAABAAAAAAEsASwABUAKwBBAFcAABMhMhYPARcWFA8BBiIvAQcGJjURNDYpATIWFREUBi8BBwYiLwEmND8BJyY2ARcWFA8BFxYGIyEiJjURNDYfATc2MgU3NhYVERQGIyEiJj8BJyY0PwE2MhcyASwVCA5exwcHaggUCMdeDhUdAzUBLBUdFQ5exwgUCGoHB8deDgj%2BL2oHB8deDggV%2FtQVHRUOXscIFALLXg4VHRX%2B1BUIDl7HBwdqCBQIBLAVDl7HCBQIagcHx14OCBUBLBUdHRX%2B1BUIDl7HBwdqCBQIx14OFf0maggUCMdeDhUdFQEsFQgOXscHzl4OCBX%2B1BUdFQ5exwgUCGoHBwAAAAYAAAAABKgEqAAPABsAIwA7AEMASwAAADIeAhQOAiIuAjQ%2BAQQiDgEUHgEyPgE0JiQyFhQGIiY0JDIWFAYjIicHFhUUBiImNTQ2PwImNTQEMhYUBiImNCQyFhQGIiY0Advy3Z9fX5%2Fd8t2gXl6gAcbgv29vv%2BC%2Fb2%2F%2BLS0gIC0gAUwtICAWDg83ETNIMykfegEJ%2FoctICAtIAIdLSAgLSAEqF%2Bf3fLdoF5eoN3y3Z9Xb7%2Fgv29vv%2BC%2FBiAtISEtICAtIQqRFxwkMzMkIDEFfgEODhekIC0gIC0gIC0gIC0AAf%2FYAFoEuQS8AFsAACUBNjc2JicmIyIOAwcABw4EFx4BMzI3ATYnLgEjIgcGBwEOASY0NwA3PgEzMhceARcWBgcOBgcGIyImJyY2NwE2NzYzMhceARcWBgcBDgEnLgECIgHVWwgHdl8WGSJBMD8hIP6IDx4eLRMNBQlZN0ozAiQkEAcdEhoYDRr%2Bqw8pHA4BRyIjQS4ODyw9DQ4YIwwod26La1YOOEBGdiIwGkQB%2F0coW2tQSE5nDxE4Qv4eDyoQEAOtAdZbZWKbEQQUGjIhH%2F6JDxsdNSg3HT5CMwIkJCcQFBcMGv6uDwEcKQ4BTSIjIQEINykvYyMLKnhuiWZMBxtAOU6%2BRAH%2FSBg3ISSGV121Qv4kDwIPDyYAAAACAGQAWASvBEQAGQBEAAABPgIeAhUUDgMHLgQ1ND4CHgEFIg4DIi4DIyIGFRQeAhcWFx4EMj4DNzY3PgQ1NCYCiTB7eHVYNkN5hKg%2BPqeFeEM4WnZ4eQEjIT8yLSohJyktPyJDbxtBMjMPBw86KzEhDSIzKUAMBAgrKT8dF2oDtURIBS1TdkA5eYB%2FslVVsn%2BAeTlAdlMtBUgtJjY1JiY1NiZvTRc4SjQxDwcOPCouGBgwKEALBAkpKkQqMhNPbQACADn%2F8gR3BL4AFwAuAAAAMh8BFhUUBg8BJi8BNycBFwcvASY0NwEDNxYfARYUBwEGIi8BJjQ%2FARYfAQcXAQKru0KNQjgiHR8uEl%2F3%2FnvUaRONQkIBGxJpCgmNQkL%2B5UK6Qo1CQjcdLhJf9wGFBL5CjUJeKmsiHTUuEl%2F4%2FnvUahKNQrpCARv%2BRmkICY1CukL%2B5UJCjUK7Qjc3LxFf%2BAGFAAAAAAMAyAAAA%2BgEsAARABUAHQAAADIeAhURFAYjISImNRE0PgEHESERACIGFBYyNjQCBqqaZDo7Kf2oKTs8Zj4CWP7%2FVj09Vj0EsB4uMhX8Ryk7OykDuRUzLar9RAK8%2FRY9Vj09VgABAAAAAASwBLAAFgAACQEWFAYiLwEBEScBBRMBJyEBJyY0NjIDhgEbDx0qDiT%2B6dT%2BzP7oywEz0gEsAQsjDx0qBKH%2B5g8qHQ8j%2FvX%2B1NL%2BzcsBGAE01AEXJA4qHQAAAAADAScAEQQJBOAAMgBAAEsAAAEVHgQXIy4DJxEXHgQVFAYHFSM1JicuASczHgEXEScuBDU0PgI3NRkBDgMVFB4DFxYXET4ENC4CArwmRVI8LAKfBA0dMydAIjxQNyiym2SWVygZA4sFV0obLkJOMCAyVWg6HSoqFQ4TJhkZCWgWKTEiGBkzNwTgTgUTLD9pQiQuLBsH%2Fs0NBxMtPGQ%2Bi6oMTU8QVyhrVk1iEAFPCA4ZLzlYNkZwSCoGTf4SARIEDh02Jh0rGRQIBgPQ%2FsoCCRYgNEM0JRkAAAABAGQAZgOUBK0ASgAAATIeARUjNC4CIyIGBwYVFB4BFxYXMxUjFgYHBgc%2BATM2FjMyNxcOAyMiLgEHDgEPASc%2BBTc%2BAScjNTMmJy4CPgE3NgIxVJlemSc8OxolVBQpGxoYBgPxxQgVFS02ImIWIIwiUzUyHzY4HCAXanQmJ1YYFzcEGAcTDBEJMAwk3aYXFQcKAg4tJGEErVCLTig%2FIhIdFSw5GkowKgkFZDKCHj4yCg8BIh6TExcIASIfBAMaDAuRAxAFDQsRCjePR2QvORQrREFMIVgAAAACABn%2F%2FwSXBLAADwAfAAABMzIWDwEGIi8BJjY7AREzBRcWBisBESMRIyImPwE2MgGQlhUIDuYOKg7mDggVlsgCF%2BYOCBWWyJYVCA7mDioBLBYO%2Bg8P%2Bg4WA4QQ%2BQ4V%2FHwDhBUO%2BQ8AAAQAGf%2F%2FA%2BgEsAAHABcAGwAlAAABIzUjFSMRIQEzMhYPAQYiLwEmNjsBETMFFTM1EwczFSE1NyM1IQPoZGRkASz9qJYVCA7mDioO5g4IFZbIAZFkY8jI%2FtTIyAEsArxkZAH0%2FHwWDvoPD%2FoOFgOEZMjI%2FRL6ZJb6ZAAAAAAEABn%2F%2FwPoBLAADwAZACEAJQAAATMyFg8BBiIvASY2OwERMwUHMxUhNTcjNSERIzUjFSMRIQcVMzUBkJYVCA7mDioO5g4IFZbIAljIyP7UyMgBLGRkZAEsx2QBLBYO%2Bg8P%2Bg4WA4SW%2BmSW%2BmT7UGRkAfRkyMgAAAAEABn%2F%2FwRMBLAADwAVABsAHwAAATMyFg8BBiIvASY2OwERMwEjESM1MxMjNSMRIQcVMzUBkJYVCA7mDioO5g4IFZbIAlhkZMhkZMgBLMdkASwWDvoPD%2FoOFgOE%2FgwBkGT7UGQBkGTIyAAAAAAEABn%2F%2FwRMBLAADwAVABkAHwAAATMyFg8BBiIvASY2OwERMwEjNSMRIQcVMzUDIxEjNTMBkJYVCA7mDioO5g4IFZbIArxkyAEsx2QBZGTIASwWDvoPD%2FoOFgOE%2FgxkAZBkyMj7tAGQZAAAAAAFABn%2F%2FwSwBLAADwATABcAGwAfAAABMzIWDwEGIi8BJjY7AREzBSM1MxMhNSETITUhEyE1IQGQlhUIDuYOKg7mDggVlsgB9MjIZP7UASxk%2FnABkGT%2BDAH0ASwWDvoPD%2FoOFgOEyMj%2BDMj%2BDMj%2BDMgABQAZ%2F%2F8EsASwAA8AEwAXABsAHwAAATMyFg8BBiIvASY2OwERMwUhNSEDITUhAyE1IQMjNTMBkJYVCA7mDioO5g4IFZbIAyD%2BDAH0ZP5wAZBk%2FtQBLGTIyAEsFg76Dw%2F6DhYDhMjI%2FgzI%2FgzI%2FgzIAAIAAAAABEwETAAPAB8AAAEhMhYVERQGIyEiJjURNDYFISIGFREUFjMhMjY1ETQmAV4BkKK8u6P%2BcKW5uQJn%2FgwpOzspAfQpOzsETLuj%2FnClubmlAZClucg7Kf4MKTs7KQH0KTsAAAAAAwAAAAAETARMAA8AHwArAAABITIWFREUBiMhIiY1ETQ2BSEiBhURFBYzITI2NRE0JgUXFhQPAQYmNRE0NgFeAZClubml%2FnCju7wCZP4MKTs7KQH0KTs7%2Fm%2F9ERH9EBgYBEy5pf5wpbm5pQGQo7vIOyn%2BDCk7OykB9Ck7gr4MJAy%2BDAsVAZAVCwAAAAADAAAAAARMBEwADwAfACsAAAEhMhYVERQGIyEiJjURNDYFISIGFREUFjMhMjY1ETQmBSEyFg8BBiIvASY2AV4BkKO7uaX%2BcKW5uQJn%2FgwpOzspAfQpOzv%2BFQGQFQsMvgwkDL4MCwRMvKL%2BcKW5uaUBkKO7yDsp%2FgwpOzspAfQpO8gYEP0REf0QGAAAAAMAAAAABEwETAAPAB8AKwAAASEyFhURFAYjISImNRE0NgUhIgYVERQWMyEyNjURNCYFFxYGIyEiJj8BNjIBXgGQpbm5pf5wo7u5Amf%2BDCk7OykB9Ck7O%2F77vgwLFf5wFQsMvgwkBEy5pf5wo7u8ogGQpbnIOyn%2BDCk7OykB9Ck7z%2F0QGBgQ%2FREAAAAAAgAAAAAFFARMAB8ANQAAASEyFhURFAYjISImPQE0NjMhMjY1ETQmIyEiJj0BNDYHARYUBwEGJj0BIyImPQE0NjsBNTQ2AiYBkKW5uaX%2BcBUdHRUBwik7Oyn%2BPhUdHb8BRBAQ%2FrwQFvoVHR0V%2BhYETLml%2FnCluR0VZBUdOykB9Ck7HRVkFR3p%2FuQOJg7%2B5A4KFZYdFcgVHZYVCgAAAQDZAAID1wSeACMAAAEXFgcGAgclMhYHIggBBwYrAScmNz4BPwEhIicmNzYANjc2MwMZCQgDA5gCASwYEQ4B%2Fvf%2B8wQMDgkJCQUCUCcn%2FtIXCAoQSwENuwUJEASeCQoRC%2F5TBwEjEv7K%2FsUFDwgLFQnlbm4TFRRWAS%2FTBhAAAAACAAAAAAT%2BBEwAHwA1AAABITIWHQEUBiMhIgYVERQWMyEyFh0BFAYjISImNRE0NgUBFhQHAQYmPQEjIiY9ATQ2OwE1NDYBXgGQFR0dFf4%2BKTs7KQHCFR0dFf5wpbm5AvEBRBAQ%2FrwQFvoVHR0V%2BhYETB0VZBUdOyn%2BDCk7HRVkFR25pQGQpbnp%2FuQOJg7%2B5A4KFZYdFcgVHZYVCgACAAAAAASwBLAAFQAxAAABITIWFREUBi8BAQYiLwEmNDcBJyY2ASMiBhURFBYzITI2PQE3ERQGIyEiJjURNDYzIQLuAZAVHRUObf7IDykPjQ8PAThtDgj%2B75wpOzspAfQpO8i7o%2F5wpbm5pQEsBLAdFf5wFQgObf7IDw%2BNDykPAThtDhX%2B1Dsp%2FgwpOzsplMj%2B1qW5uaUBkKW5AAADAA4ADgSiBKIADwAbACMAAAAyHgIUDgIiLgI0PgEEIg4BFB4BMj4BNCYEMhYUBiImNAHh7tmdXV2d2e7ZnV1dnQHD5sJxccLmwnFx%2FnugcnKgcgSiXZ3Z7tmdXV2d2e7ZnUdxwubCcXHC5sJzcqBycqAAAAMAAAAABEwEsAAVAB8AIwAAATMyFhURMzIWBwEGIicBJjY7ARE0NgEhMhYdASE1NDYFFTM1AcLIFR31FAoO%2FoEOJw3%2BhQ0JFfod%2FoUD6BUd%2B7QdA2dkBLAdFf6iFg%2F%2BVg8PAaoPFgFeFR38fB0V%2BvoVHWQyMgAAAAMAAAAABEwErAAVAB8AIwAACQEWBisBFRQGKwEiJj0BIyImNwE%2BAQEhMhYdASE1NDYFFTM1AkcBeg4KFfQiFsgUGPoUCw4Bfw4n%2FfkD6BUd%2B7QdA2dkBJ7%2BTQ8g%2BhQeHRX6IQ8BrxAC%2FH8dFfr6FR1kMjIAAwAAAAAETARLABQAHgAiAAAJATYyHwEWFAcBBiInASY0PwE2MhcDITIWHQEhNTQ2BRUzNQGMAXEHFQeLBwf98wcVB%2F7cBweLCBUH1APoFR37tB0DZ2QC0wFxBweLCBUH%2FfMICAEjCBQIiwcH%2FdIdFfr6FR1kMjIABAAAAAAETASbAAkAGQAjACcAABM3NjIfAQcnJjQFNzYWFQMOASMFIiY%2FASc3ASEyFh0BITU0NgUVMzWHjg4qDk3UTQ4CFtIOFQIBHRX9qxUIDtCa1P49A%2BgVHfu0HQNnZAP%2Fjg4OTdRMDyqa0g4IFf2pFB4BFQ7Qm9T9Oh0V%2BvoVHWQyMgAAAAQAAAAABEwEsAAPABkAIwAnAAABBR4BFRMUBi8BByc3JyY2EwcGIi8BJjQ%2FAQEhMhYdASE1NDYFFTM1AV4CVxQeARUO0JvUm9IOCMNMDyoOjg4OTf76A%2BgVHfu0HQNnZASwAgEdFf2rFQgO0JrUmtIOFf1QTQ4Ojg4qDk3%2BWB0V%2BvoVHWQyMgACAAT%2F7ASwBK8ABQAIAAAlCQERIQkBFQEEsP4d%2Fsb%2BcQSs%2FTMCq2cBFP5xAacDHPz55gO5AAAAAAIAAABkBEwEsAAVABkAAAERFAYrAREhESMiJjURNDY7AREhETMHIzUzBEwdFZb9RJYVHR0V%2BgH0ZMhkZAPo%2FK4VHQGQ%2FnAdFQPoFB7%2B1AEsyMgAAAMAAABFBN0EsAAWABoALwAAAQcBJyYiDwEhESMiJjURNDY7AREhETMHIzUzARcWFAcBBiIvASY0PwE2Mh8BATYyBEwC%2FtVfCRkJlf7IlhUdHRX6AfRkyGRkAbBqBwf%2BXAgUCMoICGoHFQdPASkHFQPolf7VXwkJk%2F5wHRUD6BQe%2FtQBLMjI%2Fc5qBxUH%2FlsHB8sHFQdqCAhPASkHAAMAAAANBQcEsAAWABoAPgAAAREHJy4BBwEhESMiJjURNDY7AREhETMHIzUzARcWFA8BFxYUDwEGIi8BBwYiLwEmND8BJyY0PwE2Mh8BNzYyBExnhg8lEP72%2FreWFR0dFfoB9GTIZGQB9kYPD4ODDw9GDykPg4MPKQ9GDw%2BDgw8PRg8pD4ODDykD6P7zZ4YPAw7%2B9v5wHRUD6BQe%2FtQBLMjI%2FYxGDykPg4MPKQ9GDw%2BDgw8PRg8pD4ODDykPRg8Pg4MPAAADAAAAFQSXBLAAFQAZAC8AAAERISIGHQEhESMiJjURNDY7AREhETMHIzUzEzMyFh0BMzIWDwEGIi8BJjY7ATU0NgRM%2FqIVHf4MlhUdHRX6AfRkyGRklmQVHZYVCA7mDioO5g4IFZYdA%2Bj%2B1B0Vlv5wHRUD6BQe%2FtQBLMjI%2FagdFfoVDuYODuYOFfoVHQAAAAADAAAAAASXBLAAFQAZAC8AAAERJyYiBwEhESMiJjURNDY7AREhETMHIzUzExcWBisBFRQGKwEiJj0BIyImPwE2MgRMpQ4qDv75%2Fm6WFR0dFfoB9GTIZGTr5g4IFZYdFWQVHZYVCA7mDioD6P5wpQ8P%2Fvf%2BcB0VA%2BgUHv7UASzIyP2F5Q8V%2BhQeHhT6FQ%2FlDwADAAAAyASwBEwACQATABcAABMhMhYdASE1NDYBERQGIyEiJjURExUhNTIETBUd%2B1AdBJMdFfu0FR1kAZAETB0VlpYVHf7U%2FdoVHR0VAib%2B1MjIAAAGAAMAfQStBJcADwAZAB0ALQAxADsAAAEXFhQPAQYmPQEhNSE1NDYBIyImPQE0NjsBFyM1MwE3NhYdASEVIRUUBi8BJjQFIzU7AjIWHQEUBisBA6f4Dg74DhX%2BcAGQFf0vMhUdHRUyyGRk%2FoL3DhUBkP5wFQ73DwOBZGRkMxQdHRQzBI3mDioO5g4IFZbIlhUI%2FoUdFWQVHcjI%2FcvmDggVlsiWFQgO5g4qecgdFWQVHQAAAAACAGQAAASwBLAAFgBRAAABJTYWFREUBisBIiY1ES4ENRE0NiUyFh8BERQOAg8BERQGKwEiJjURLgQ1ETQ%2BAzMyFh8BETMRPAE%2BAjMyFh8BETMRND4DA14BFBklHRXIFR0EDiIaFiX%2B4RYZAgEVHR0LCh0VyBUdBA4iGhYBBwoTDRQZAgNkBQkVDxcZAQFkAQUJFQQxdBIUH%2FuuFR0dFQGNAQgbHzUeAWcfRJEZDA3%2BPhw%2FMSkLC%2F5BFR0dFQG%2FBA8uLkAcAcICBxENCxkMDf6iAV4CBxENCxkMDf6iAV4CBxENCwABAGQAAASwBEwAMwAAARUiDgMVERQWHwEVITUyNjURIREUFjMVITUyPgM1ETQmLwE1IRUiBhURIRE0JiM1BLAEDiIaFjIZGf5wSxn%2BDBlL%2FnAEDiIaFjIZGQGQSxkB9BlLBEw4AQUKFA78iBYZAQI4OA0lAYr%2BdiUNODgBBQoUDgN4FhkBAjg4DSX%2BdgGKJQ04AAAABgAAAAAETARMAAwAHAAgACQAKAA0AAABITIWHQEjBTUnITchBSEyFhURFAYjISImNRE0NhcVITUBBTUlBRUhNQUVFAYjIQchJyE3MwKjAXcVHWn%2B2cj%2BcGQBd%2F4lASwpOzsp%2FtQpOzspASwCvP5wAZD8GAEsArwdFf6JZP6JZAGQyGkD6B0VlmJiyGTIOyn%2BDCk7OykB9Ck7ZMjI%2FveFo4XGyMhm%2BBUdZGTIAAEAEAAQBJ8EnwAmAAATNzYWHwEWBg8BHgEXNz4BHwEeAQ8BBiIuBicuBTcRohEuDosOBhF3ZvyNdxEzE8ATBxGjAw0uMUxPZWZ4O0p3RjITCwED76IRBhPCFDERdo78ZXYRBA6IDi8RogEECBUgNUNjO0qZfHNVQBAAAAACAAAAAASwBEwAIwBBAAAAMh4EHwEVFAYvAS4BPQEmIAcVFAYPAQYmPQE%2BBRIyHgIfARUBHgEdARQGIyEiJj0BNDY3ATU0PgIB%2FLimdWQ%2FLAkJHRTKFB2N%2FsKNHRTKFB0DDTE7ZnTKcFImFgEBAW0OFR0V%2B7QVHRUOAW0CFiYETBUhKCgiCgrIFRgDIgMiFZIYGJIVIgMiAxgVyAQNJyQrIP7kExwcCgoy%2FtEPMhTUFR0dFdQUMg8BLzIEDSEZAAADAAAAAASwBLAADQAdACcAAAEHIScRMxUzNTMVMzUzASEyFhQGKwEXITcjIiY0NgMhMhYdASE1NDYETMj9qMjIyMjIyPyuArwVHR0VDIn8SokMFR0dswRMFR37UB0CvMjIAfTIyMjI%2FOAdKh1kZB0qHf7UHRUyMhUdAAAAAwBkAAAEsARMAAkAEwAdAAABIyIGFREhETQmASMiBhURIRE0JgEhETQ2OwEyFhUCvGQpOwEsOwFnZCk7ASw7%2FRv%2B1DspZCk7BEw7KfwYA%2BgpO%2F7UOyn9RAK8KTv84AGQKTs7KQAAAAAF%2F5wAAASwBEwADwATAB8AJQApAAATITIWFREUBiMhIiY1ETQ2FxEhEQUjFTMRITUzNSMRIQURByMRMwcRMxHIArx8sLB8%2FUR8sLAYA4T%2BDMjI%2FtTIyAEsAZBkyMhkZARMsHz%2BDHywsHwB9HywyP1EArzIZP7UZGQBLGT%2B1GQB9GT%2B1AEsAAAABf%2BcAAAEsARMAA8AEwAfACUAKQAAEyEyFhURFAYjISImNRE0NhcRIREBIzUjFSMRMxUzNTMFEQcjETMHETMRyAK8fLCwfP1EfLCwGAOE%2FgxkZGRkZGQBkGTIyGRkBEywfP4MfLCwfAH0fLDI%2FUQCvP2oyMgB9MjIZP7UZAH0ZP7UASwABP%2BcAAAEsARMAA8AEwAbACMAABMhMhYVERQGIyEiJjURNDYXESERBSMRMxUhESEFIxEzFSERIcgCvHywsHz9RHywsBgDhP4MyMj%2B1AEsAZDIyP7UASwETLB8%2Fgx8sLB8AfR8sMj9RAK8yP7UZAH0ZP7UZAH0AAAABP%2BcAAAEsARMAA8AEwAWABkAABMhMhYVERQGIyEiJjURNDYXESERAS0BDQERyAK8fLCwfP1EfLCwGAOE%2Fgz%2B1AEsAZD%2B1ARMsHz%2BDHywsHwB9HywyP1EArz%2BDJaWlpYBLAAAAAX%2FnAAABLAETAAPABMAFwAgACkAABMhMhYVERQGIyEiJjURNDYXESERAyERIQcjIgYVFBY7AQERMzI2NTQmI8gCvHywsHz9RHywsBgDhGT9RAK8ZIImOTYpgv4Mgik2OSYETLB8%2Fgx8sLB8AfR8sMj9RAK8%2FagB9GRWQUFUASz%2B1FRBQVYAAAAF%2F5wAAASwBEwADwATAB8AJQApAAATITIWFREUBiMhIiY1ETQ2FxEhEQUjFTMRITUzNSMRIQEjESM1MwMjNTPIArx8sLB8%2FUR8sLAYA4T%2BDMjI%2FtTIyAEsAZBkZMjIZGQETLB8%2Fgx8sLB8AfR8sMj9RAK8yGT%2B1GRkASz%2BDAGQZP4MZAAG%2F5wAAASwBEwADwATABkAHwAjACcAABMhMhYVERQGIyEiJjURNDYXESERBTMRIREzASMRIzUzBRUzNQEjNTPIArx8sLB8%2FUR8sLAYA4T9RMj%2B1GQCWGRkyP2oZAEsZGQETLB8%2Fgx8sLB8AfR8sMj9RAK8yP5wAfT%2BDAGQZMjIyP7UZAAF%2F5wAAASwBEwADwATABwAIgAmAAATITIWFREUBiMhIiY1ETQ2FxEhEQEHIzU3NSM1IQEjESM1MwMjNTPIArx8sLB8%2FUR8sLAYA4T%2BDMdkx8gBLAGQZGTIx2RkBEywfP4MfLCwfAH0fLDI%2FUQCvP5wyDLIlmT%2BDAGQZP4MZAAAAAMACQAJBKcEpwAPABsAJQAAADIeAhQOAiIuAjQ%2BAQQiDgEUHgEyPgE0JgchFSEVISc1NyEB4PDbnl5entvw255eXp4BxeTCcXHC5MJxcWz%2B1AEs%2FtRkZAEsBKdentvw255eXp7b8NueTHHC5MJxccLkwtDIZGTIZAAAAAAEAAkACQSnBKcADwAbACcAKwAAADIeAhQOAiIuAjQ%2BAQQiDgEUHgEyPgE0JgcVBxcVIycjFSMRIQcVMzUB4PDbnl5entvw255eXp4BxeTCcXHC5MJxcWwyZGRklmQBLMjIBKdentvw255eXp7b8NueTHHC5MJxccLkwtBkMmQyZGQBkGRkZAAAAv%2Fy%2F50EwgRBACAANgAAATIWFzYzMhYUBisBNTQmIyEiBh0BIyImNTQ2NyY1ND4BEzMyFhURMzIWDwEGIi8BJjY7ARE0NgH3brUsLC54qqp4gB0V%2FtQVHd5QcFZBAmKqepYKD4kVCg3fDSYN3w0KFYkPBEF3YQ6t8a36FR0dFfpzT0VrDhMSZKpi%2FbMPCv7tFxD0EBD0EBcBEwoPAAAAAAL%2F8v%2BcBMMEQQAcADMAAAEyFhc2MzIWFxQGBwEmIgcBIyImNTQ2NyY1ND4BExcWBisBERQGKwEiJjURIyImNzY3NjIB9m62LCsueaoBeFr%2Bhg0lDf6DCU9xVkECYqnm3w0KFYkPCpYKD4kVCg3HGBMZBEF3YQ%2BteGOkHAFoEBD%2Bk3NPRWsOExNkqWP9kuQQF%2F7tCg8PCgETFxDMGBMAAAABAGQAAARMBG0AGAAAJTUhATMBMwkBMwEzASEVIyIGHQEhNTQmIwK8AZD%2B8qr%2B8qr%2B1P7Uqv7yqv7yAZAyFR0BkB0VZGQBLAEsAU3%2Bs%2F7U%2FtRkHRUyMhUdAAAAAAEAeQAABDcEmwAvAAABMhYXHgEVFAYHFhUUBiMiJxUyFh0BITU0NjM1BiMiJjU0Ny4BNTQ2MzIXNCY1NDYCWF6TGll7OzIJaUo3LRUd%2FtQdFS03SmkELzlpSgUSAqMEm3FZBoNaPWcfHRpKaR77HRUyMhUd%2Bx5pShIUFVg1SmkCAhAFdKMAAAAGACcAFASJBJwAEQAqAEIASgBiAHsAAAEWEgIHDgEiJicmAhI3PgEyFgUiBw4BBwYWHwEWMzI3Njc2Nz4BLwEmJyYXIgcOAQcGFh8BFjMyNz4BNz4BLwEmJyYWJiIGFBYyNjciBw4BBw4BHwEWFxYzMjc%2BATc2Ji8BJhciBwYHBgcOAR8BFhcWMzI3PgE3NiYvASYD8m9PT29T2dzZU29PT29T2dzZ%2Fj0EBHmxIgQNDCQDBBcGG0dGYAsNAwkDCwccBAVQdRgEDA0iBAQWBhJROQwMAwkDCwf5Y4xjY4xjVhYGElE6CwwDCQMLBwgEBVB1GAQNDCIEjRcGG0dGYAsNAwkDCwcIBAR5sSIEDQwkAwPyb%2F7V%2FtVvU1dXU28BKwErb1NXVxwBIrF5DBYDCQEWYEZHGwMVDCMNBgSRAhh1UA0WAwkBFTpREgMVCyMMBwT6Y2OMY2MVFTpREQQVCyMMBwQCGHVQDRYDCQEkFmBGRxsDFQwjDQYEASKxeQwWAwkBAAAABQBkAAAD6ASwAAwADwAWABwAIgAAASERIzUhFSERNDYzIQEjNQMzByczNTMDISImNREFFRQGKwECvAEstP6s%2FoQPCgI%2FASzIZKLU1KJktP51Cg8DhA8KwwMg%2FoTIyALzCg%2F%2B1Mj84NTUyP4MDwoBi8jDCg8AAAAABQBkAAAD6ASwAAkADAATABoAIQAAASERCQERNDYzIQEjNRMjFSM1IzcDISImPQEpARUUBisBNQK8ASz%2Bov3aDwoCPwEsyD6iZKLUqv6dCg8BfAIIDwqbAyD9%2BAFe%2FdoERwoP%2FtTI%2FHzIyNT%2BZA8KNzcKD1AAAAAAAwAAAAAEsAP0AAgAGQAfAAABIxUzFyERIzcFMzIeAhUhFSEDETM0PgIBMwMhASEEiqJkZP7UotT9EsgbGiEOASz9qMhkDiEaAnPw8PzgASwB9AMgyGQBLNTUBBErJGT%2BogHCJCsRBP5w%2FnAB9AAAAAMAAAAABEwETAAZADIAOQAAATMyFh0BMzIWHQEUBiMhIiY9ATQ2OwE1NDYFNTIWFREUBiMhIic3ARE0NjMVFBYzITI2AQc1IzUzNQKKZBUdMhUdHRX%2B1BUdHRUyHQFzKTs7Kf2oARP2%2Fro7KVg%2BASw%2BWP201MjIBEwdFTIdFWQVHR0VZBUdMhUd%2BpY7KfzgKTsE9gFGAUQpO5Y%2BWFj95tSiZKIAAwBkAAAEvARMABkANgA9AAABMzIWHQEzMhYdARQGIyEiJj0BNDY7ATU0NgU1MhYVESMRMxQOAiMhIiY1ETQ2MxUUFjMhMjYBBzUjNTM1AcJkFR0yFR0dFf7UFR0dFTIdAXMpO8jIDiEaG%2F2oKTs7KVg%2BASw%2BWAGc1MjIBEwdFTIdFWQVHR0VZBUdMhUd%2BpY7Kf4M%2FtQkKxEEOykDICk7lj5YWP3m1KJkogAAAAP%2FogAABRYE1AALABsAHwAACQEWBiMhIiY3ATYyEyMiBhcTHgE7ATI2NxM2JgMVMzUCkgJ9FyAs%2BwQsIBcCfRZARNAUGAQ6BCMUNhQjBDoEGODIBK37sCY3NyYEUCf%2BTB0U%2FtIUHR0UAS4UHf4MZGQAAAAACQAAAAAETARMAA8AHwAvAD8ATwBfAG8AfwCPAAABMzIWHQEUBisBIiY9ATQ2EzMyFh0BFAYrASImPQE0NiEzMhYdARQGKwEiJj0BNDYBMzIWHQEUBisBIiY9ATQ2ITMyFh0BFAYrASImPQE0NiEzMhYdARQGKwEiJj0BNDYBMzIWHQEUBisBIiY9ATQ2ITMyFh0BFAYrASImPQE0NiEzMhYdARQGKwEiJj0BNDYBqfoKDw8K%2BgoPDwr6Cg8PCvoKDw8BmvoKDw8K%2BgoPD%2Fzq%2BgoPDwr6Cg8PAZr6Cg8PCvoKDw8BmvoKDw8K%2BgoPD%2Fzq%2BgoPDwr6Cg8PAZr6Cg8PCvoKDw8BmvoKDw8K%2BgoPDwRMDwqWCg8PCpYKD%2F7UDwqWCg8PCpYKDw8KlgoPDwqWCg%2F%2B1A8KlgoPDwqWCg8PCpYKDw8KlgoPDwqWCg8PCpYKD%2F7UDwqWCg8PCpYKDw8KlgoPDwqWCg8PCpYKDw8KlgoPAAAAAwAAAAAEsAUUABkAKQAzAAABMxUjFSEyFg8BBgchJi8BJjYzITUjNTM1MwEhMhYUBisBFyE3IyImNDYDITIWHQEhNTQ2ArxkZAFePjEcQiko%2FPwoKUIcMT4BXmRkyP4%2BArwVHR0VDIn8SooNFR0dswRMFR37UB0EsMhkTzeEUzMzU4Q3T2TIZPx8HSodZGQdKh3%2B1B0VMjIVHQAABAAAAAAEsAUUAAUAGQArADUAAAAyFhUjNAchFhUUByEyFg8BIScmNjMhJjU0AyEyFhQGKwEVBSElNSMiJjQ2AyEyFh0BITU0NgIwUDnCPAE6EgMBSCkHIq%2F9WrIiCikBSAOvArwVHR0VlgET%2FEoBE5YVHR2zBEwVHftQHQUUOykpjSUmCBEhFpGRFiERCCb%2BlR0qHcjIyMgdKh39qB0VMjIVHQAEAAAAAASwBJ0ABwAUACQALgAAADIWFAYiJjQTMzIWFRQXITY1NDYzASEyFhQGKwEXITcjIiY0NgMhMhYdASE1NDYCDZZqapZqty4iKyf%2BvCcrI%2F7NArwVHR0VDYr8SokMFR0dswRMFR37UB0EnWqWamqW%2Fus5Okxra0w6Of5yHSodZGQdKh3%2B1B0VMjIVHQAEAAAAAASwBRQADwAcACwANgAAATIeARUUBiImNTQ3FzcnNhMzMhYVFBchNjU0NjMBITIWFAYrARchNyMiJjQ2AyEyFh0BITU0NgJYL1szb5xvIpBvoyIfLiIrJ%2F68Jysj%2Fs0CvBUdHRUNivxKiQwVHR2zBEwVHftQHQUUa4s2Tm9vTj5Rj2%2BjGv4KOTpMa2tMOjn%2Bch0qHWRkHSod%2FtQdFTIyFR0AAAADAAAAAASwBRIAEgAiACwAAAEFFSEUHgMXIS4BNTQ%2BAjcBITIWFAYrARchNyMiJjQ2AyEyFh0BITU0NgJYASz%2B1CU%2FP00T%2Fe48PUJtj0r%2BogK8FR0dFQ2K%2FEqJDBUdHbMETBUd%2B1AdBLChizlmUT9IGVO9VFShdksE%2FH4dKh1kZB0qHf7UHRUyMhUdAAIAyAAAA%2BgFFAAPACkAAAAyFh0BHgEdASE1NDY3NTQDITIWFyMVMxUjFTMVIxUzFAYjISImNRE0NgIvUjsuNv5wNi5kAZA2XBqsyMjIyMh1U%2F5wU3V1BRQ7KU4aXDYyMjZcGk4p%2Fkc2LmRkZGRkU3V1UwGQU3UAAAMAZP%2F%2FBEwETAAPAC8AMwAAEyEyFhURFAYjISImNRE0NgMhMhYdARQGIyEXFhQGIi8BIQcGIiY0PwEhIiY9ATQ2BQchJ5YDhBUdHRX8fBUdHQQDtgoPDwr%2B5eANGiUNWP30Vw0mGg3g%2Ft8KDw8BqmQBRGQETB0V%2FgwVHR0VAfQVHf1EDwoyCg%2FgDSUbDVhYDRslDeAPCjIKD2RkZAAAAAAEAAAAAASwBEwAGQAjAC0ANwAAEyEyFh0BIzQmKwEiBhUjNCYrASIGFSM1NDYDITIWFREhETQ2ExUUBisBIiY9ASEVFAYrASImPQHIAyBTdWQ7KfopO2Q7KfopO2R1EQPoKTv7UDvxHRVkFR0D6B0VZBUdBEx1U8gpOzspKTs7KchTdf4MOyn%2B1AEsKTv%2BDDIVHR0VMjIVHR0VMgADAAEAAASpBKwADQARABsAAAkBFhQPASEBJjQ3ATYyCQMDITIWHQEhNTQ2AeACqh8fg%2F4f%2FfsgIAEnH1n%2BrAFWAS%2F%2Bq6IDIBUd%2FHwdBI39VR9ZH4MCBh9ZHwEoH%2F5u%2FqoBMAFV%2FBsdFTIyFR0AAAAAAgCPAAAEIQSwABcALwAAAQMuASMhIgYHAwYWMyEVFBYyNj0BMzI2AyE1NDY7ATU0NjsBETMRMzIWHQEzMhYVBCG9CCcV%2FnAVJwi9CBMVAnEdKh19FROo%2Fa0dFTIdFTDILxUdMhUdAocB%2BhMcHBP%2BBhMclhUdHRWWHP2MMhUdMhUdASz%2B1B0VMh0VAAAEAAAAAASwBLAADQAQAB8AIgAAASERFAYjIREBNTQ2MyEBIzUBIREUBiMhIiY1ETQ2MyEBIzUDhAEsDwr%2Bif7UDwoBdwEsyP2oASwPCv12Cg8PCgF3ASzIAyD9wQoPAk8BLFQKD%2F7UyP4M%2FcEKDw8KA7YKD%2F7UyAAC%2F5wAZAUUBEcARgBWAAABMzIeAhcWFxY2NzYnJjc%2BARYXFgcOASsBDgEPAQ4BKwEiJj8BBisBIicHDgErASImPwEmLwEuAT0BNDY7ATY3JyY2OwE2BSMiBh0BFBY7ATI2PQE0JgHkw0uOakkMEhEfQwoKGRMKBQ8XDCkCA1Y9Pgc4HCcDIhVkFRgDDDEqwxgpCwMiFWQVGAMaVCyfExwdFXwLLW8QBxXLdAFF%2BgoPDwr6Cg8PBEdBa4pJDgYKISAiJRsQCAYIDCw9P1c3fCbqFB0dFEYOCEAUHR0UnUplNQcmFTIVHVdPXw4TZV8PCjIKDw8KMgoPAAb%2FnP%2FmBRQEfgAJACQANAA8AFIAYgAAASU2Fh8BFgYPASUzMhYfASEyFh0BFAYHBQYmJyYjISImPQE0NhcjIgYdARQ7ATI2NTQmJyYEIgYUFjI2NAE3PgEeARceAT8BFxYGDwEGJi8BJjYlBwYfAR4BPwE2Jy4BJy4BAoEBpxMuDiAOAxCL%2FCtqQ0geZgM3FR0cE%2F0fFyIJKjr%2B1D5YWLlQExIqhhALIAsSAYBALS1ALf4PmBIgHhMQHC0aPzANITNQL3wpgigJASlmHyElDR0RPRMFAhQHCxADhPcICxAmDyoNeMgiNtQdFTIVJgeEBBQPQ1g%2ByD5YrBwVODMQEAtEERzJLUAtLUD%2B24ITChESEyMgAwWzPUkrRSgJL5cvfRxYGyYrDwkLNRAhFEgJDAQAAAAAAwBkAAAEOQSwAFEAYABvAAABMzIWHQEeARcWDgIPATIeBRUUDgUjFRQGKwEiJj0BIxUUBisBIiY9ASMiJj0BNDY7AREjIiY9ATQ2OwE1NDY7ATIWHQEzNTQ2AxUhMj4CNTc0LgMjARUhMj4CNTc0LgMjAnGWCg9PaAEBIC4uEBEGEjQwOiodFyI2LUAjGg8KlgoPZA8KlgoPrwoPDwpLSwoPDwqvDwqWCg9kD9cBBxwpEwsBAQsTKRz%2B%2BQFrHCkTCwEBCxMpHASwDwptIW1KLk0tHwYGAw8UKDJOLTtdPCoVCwJLCg8PCktLCg8PCksPCpYKDwJYDwqWCg9LCg8PCktLCg%2F%2B1MgVHR0LCgQOIhoW%2FnDIFR0dCwoEDiIaFgAAAwAEAAIEsASuABcAKQAsAAATITIWFREUBg8BDgEjISImJy4CNRE0NgQiDgQPARchNy4FAyMT1AMMVnokEhIdgVL9xFKCHAgYKHoCIIx9VkcrHQYGnAIwnAIIIClJVSGdwwSuelb%2BYDO3QkJXd3ZYHFrFMwGgVnqZFyYtLSUMDPPzBQ8sKDEj%2FsIBBQACAMgAAAOEBRQADwAZAAABMzIWFREUBiMhIiY1ETQ2ARUUBisBIiY9AQHblmesVCn%2BPilUrAFINhWWFTYFFKxn%2FgwpVFQpAfRnrPwY4RU2NhXhAAACAMgAAAOEBRQADwAZAAABMxQWMxEUBiMhIiY1ETQ2ARUUBisBIiY9AQHbYLOWVCn%2BPilUrAFINhWWFTYFFJaz%2FkIpVFQpAfRnrPwY4RU2NhXhAAACAAAAFAUOBBoAFAAaAAAJASUHFRcVJwc1NzU0Jj4CPwEnCQEFJTUFJQUO%2FYL%2Bhk5klpZkAQEBBQQvkwKCAVz%2Bov6iAV4BXgL%2F%2FuWqPOCWx5SVyJb6BA0GCgYDKEEBG%2F1ipqaTpaUAAAMAZAH0BLADIAAHAA8AFwAAEjIWFAYiJjQkMhYUBiImNCQyFhQGIiY0vHxYWHxYAeh8WFh8WAHofFhYfFgDIFh8WFh8WFh8WFh8WFh8WFh8AAAAAAMBkAAAArwETAAHAA8AFwAAADIWFAYiJjQSMhYUBiImNBIyFhQGIiY0Aeh8WFh8WFh8WFh8WFh8WFh8WARMWHxYWHz%2ByFh8WFh8%2FshYfFhYfAAAAAMAZABkBEwETAAPAB8ALwAAEyEyFh0BFAYjISImPQE0NhMhMhYdARQGIyEiJj0BNDYTITIWHQEUBiMhIiY9ATQ2fQO2Cg8PCvxKCg8PCgO2Cg8PCvxKCg8PCgO2Cg8PCvxKCg8PBEwPCpYKDw8KlgoP%2FnAPCpYKDw8KlgoP%2FnAPCpYKDw8KlgoPAAAABAAAAAAEsASwAA8AHwAvADMAAAEhMhYVERQGIyEiJjURNDYFISIGFREUFjMhMjY1ETQmBSEyFhURFAYjISImNRE0NhcVITUBXgH0ory7o%2F4Mpbm5Asv9qCk7OykCWCk7O%2F2xAfQVHR0V%2FgwVHR1HAZAEsLuj%2FgylubmlAfSlucg7Kf2oKTs7KQJYKTtkHRX%2B1BUdHRUBLBUdZMjIAAAAAAEAZABkBLAETAA7AAATITIWFAYrARUzMhYUBisBFTMyFhQGKwEVMzIWFAYjISImNDY7ATUjIiY0NjsBNSMiJjQ2OwE1IyImNDaWA%2BgVHR0VMjIVHR0VMjIVHR0VMjIVHR0V%2FBgVHR0VMjIVHR0VMjIVHR0VMjIVHR0ETB0qHcgdKh3IHSodyB0qHR0qHcgdKh3IHSodyB0qHQAAAAYBLAAFA%2BgEowAHAA0AEwAZAB8AKgAAAR4BBgcuATYBMhYVIiYlFAYjNDYBMhYVIiYlFAYjNDYDFRQGIiY9ARYzMgKKVz8%2FV1c%2FP%2F75fLB8sAK8sHyw%2FcB8sHywArywfLCwHSodKAMRBKNDsrJCQrKy%2FsCwfLB8fLB8sP7UsHywfHywfLD%2B05AVHR0VjgQAAAH%2FtQDIBJQDgQBCAAABNzYXAR4BBw4BKwEyFRQOBCsBIhE0NyYiBxYVECsBIi4DNTQzIyImJyY2NwE2HwEeAQ4BLwEHIScHBi4BNgLpRRkUASoLCAYFGg8IAQQNGyc%2FKZK4ChRUFQu4jjBJJxkHAgcPGQYGCAsBKhQaTBQVCiMUM7YDe7YsFCMKFgNuEwYS%2FtkLHw8OEw0dNkY4MhwBIBgXBAQYF%2F7gKjxTQyMNEw4PHwoBKBIHEwUjKBYGDMHBDAUWKCMAAAAAAgAAAAAEsASwACUAQwAAASM0LgUrAREUFh8BFSE1Mj4DNREjIg4FFSMRIQEjNC4DKwERFBYXMxUjNTI1ESMiDgMVIzUhBLAyCAsZEyYYGcgyGRn%2BcAQOIhoWyBkYJhMZCwgyA%2Bj9RBkIChgQEWQZDQzIMmQREBgKCBkB9AOEFSAVDggDAfyuFhkBAmRkAQUJFQ4DUgEDCA4VIBUBLP0SDxMKBQH%2BVwsNATIyGQGpAQUKEw%2BWAAAAAAMAAAAABEwErgAdACAAMAAAATUiJy4BLwEBIwEGBw4BDwEVITUiJj8BIRcWBiMVARsBARUUBiMhIiY9ATQ2MyEyFgPoGR4OFgUE%2Ft9F%2FtQSFQkfCwsBETE7EkUBJT0NISf%2B7IZ5AbEdFfwYFR0dFQPoFR0BLDIgDiIKCwLr%2FQ4jFQkTBQUyMisusKYiQTIBhwFW%2Fqr942QVHR0VZBUdHQADAAAAAASwBLAADwBHAEoAABMhMhYVERQGIyEiJjURNDYFIyIHAQYHBgcGHQEUFjMhMjY9ATQmIyInJj8BIRcWBwYjIgYdARQWMyEyNj0BNCYnIicmJyMBJhMjEzIETBUdHRX7tBUdHQJGRg0F%2FtUREhImDAsJAREIDAwINxAKCj8BCjkLEQwYCAwMCAE5CAwLCBEZGQ8B%2FuAFDsVnBLAdFfu0FR0dFQRMFR1SDP0PIBMSEAUNMggMDAgyCAwXDhmjmR8YEQwIMggMDAgyBwwBGRskAuwM%2FgUBCAAABAAAAAAEsASwAAMAEwAjACcAAAEhNSEFITIWFREUBiMhIiY1ETQ2KQEyFhURFAYjISImNRE0NhcRIREEsPtQBLD7ggGQFR0dFf5wFR0dAm0BkBUdHRX%2BcBUdHUcBLARMZMgdFfx8FR0dFQOEFR0dFf5wFR0dFQGQFR1k%2FtQBLAAEAAAAAASwBLAADwAfACMAJwAAEyEyFhURFAYjISImNRE0NgEhMhYVERQGIyEiJjURNDYXESEREyE1ITIBkBUdHRX%2BcBUdHQJtAZAVHR0V%2FnAVHR1HASzI%2B1AEsASwHRX8fBUdHRUDhBUd%2FgwdFf5wFR0dFQGQFR1k%2FtQBLP2oZAAAAAACAAAAZASwA%2BgAJwArAAATITIWFREzNTQ2MyEyFh0BMxUjFRQGIyEiJj0BIxEUBiMhIiY1ETQ2AREhETIBkBUdZB0VAZAVHWRkHRX%2BcBUdZB0V%2FnAVHR0CnwEsA%2BgdFf6ilhUdHRWWZJYVHR0Vlv6iFR0dFQMgFR3%2B1P7UASwAAAQAAAAABLAEsAADABMAFwAnAAAzIxEzFyEyFhURFAYjISImNRE0NhcRIREBITIWFREUBiMhIiY1ETQ2ZGRklgGQFR0dFf5wFR0dRwEs%2FqIDhBUdHRX8fBUdHQSwZB0V%2FnAVHR0VAZAVHWT%2B1AEs%2FgwdFf5wFR0dFQGQFR0AAAAAAgBkAAAETASwACcAKwAAATMyFhURFAYrARUhMhYVERQGIyEiJjURNDYzITUjIiY1ETQ2OwE1MwcRIRECWJYVHR0VlgHCFR0dFfx8FR0dFQFelhUdHRWWZMgBLARMHRX%2BcBUdZB0V%2FnAVHR0VAZAVHWQdFQGQFR1kyP7UASwAAAAEAAAAAASwBLAAAwATABcAJwAAISMRMwUhMhYVERQGIyEiJjURNDYXESERASEyFhURFAYjISImNRE0NgSwZGT9dgGQFR0dFf5wFR0dRwEs%2FK4DhBUdHRX8fBUdHQSwZB0V%2FnAVHR0VAZAVHWT%2B1AEs%2FgwdFf5wFR0dFQGQFR0AAAEBLAAwA28EgAAPAAAJAQYjIiY1ETQ2MzIXARYUA2H%2BEhcSDhAQDhIXAe4OAjX%2BEhcbGQPoGRsX%2FhIOKgAAAAABAUEAMgOEBH4ACwAACQE2FhURFAYnASY0AU8B7h0qKh3%2BEg4CewHuHREp%2FBgpER0B7g4qAAAAAAEAMgFBBH4DhAALAAATITIWBwEGIicBJjZkA%2BgpER3%2BEg4qDv4SHREDhCod%2FhIODgHuHSoAAAAAAQAyASwEfgNvAAsAAAkBFgYjISImNwE2MgJ7Ae4dESn8GCkRHQHuDioDYf4SHSoqHQHuDgAAAAACAAgAAASwBCgABgAKAAABFQE1LQE1ASE1IQK8%2FUwBnf5jBKj84AMgAuW2%2Fr3dwcHd%2B9jIAAAAAAIAAABkBLAEsAALADEAAAEjFTMVIREzNSM1IQEzND4FOwERFAYPARUhNSIuAzURMzIeBRUzESEEsMjI%2FtTIyAEs%2B1AyCAsZEyYYGWQyGRkBkAQOIhoWZBkYJhMZCwgy%2FOADhGRkASxkZP4MFSAVDggDAf3aFhkBAmRkAQUJFQ4CJgEDCA4VIBUBLAAAAgAAAAAETAPoACUAMQAAASM0LgUrAREUFh8BFSE1Mj4DNREjIg4FFSMRIQEjFTMVIREzNSM1IQMgMggLGRMmGBlkMhkZ%2FnAEDiIaFmQZGCYTGQsIMgMgASzIyP7UyMgBLAK8FSAVDggDAf3aFhkCAWRkAQUJFQ4CJgEDCA4VIBUBLPzgZGQBLGRkAAABAMgAZgNyBEoAEgAAATMyFgcJARYGKwEiJwEmNDcBNgK9oBAKDP4wAdAMChCgDQr%2BKQcHAdcKBEoWDP4w%2FjAMFgkB1wgUCAHXCQAAAQE%2BAGYD6ARKABIAAAEzMhcBFhQHAQYrASImNwkBJjYBU6ANCgHXBwf%2BKQoNoBAKDAHQ%2FjAMCgRKCf4pCBQI%2FikJFgwB0AHQDBYAAAEAZgDIBEoDcgASAAAAFh0BFAcBBiInASY9ATQ2FwkBBDQWCf4pCBQI%2FikJFgwB0AHQA3cKEKANCv4pBwcB1woNoBAKDP4wAdAAAAABAGYBPgRKA%2BgAEgAACQEWHQEUBicJAQYmPQE0NwE2MgJqAdcJFgz%2BMP4wDBYJAdcIFAPh%2FikKDaAQCgwB0P4wDAoQoA0KAdcHAAAAAgDZ%2F%2FkEPQSwAAUAOgAAARQGIzQ2BTMyFh8BNjc%2BAh4EBgcOBgcGIiYjIgYiJy4DLwEuAT4EHgEXJyY2A%2BiwfLD%2BVmQVJgdPBQsiKFAzRyorDwURAQQSFyozTSwNOkkLDkc3EDlfNyYHBw8GDyUqPjdGMR%2BTDA0EsHywfLDIHBPCAQIGBwcFDx81S21DBxlLR1xKQhEFBQcHGWt0bCQjP2hJNyATBwMGBcASGAAAAAACAMgAFQOEBLAAFgAaAAATITIWFREUBisBEQcGJjURIyImNRE0NhcVITX6AlgVHR0Vlv8TGpYVHR2rASwEsB0V%2FnAVHf4MsgkQFQKKHRUBkBUdZGRkAAAAAgDIABkETASwAA4AEgAAEyEyFhURBRElIREjETQ2ARU3NfoC7ic9%2FUQCWP1EZB8BDWQEsFEs%2FFt1A7Z9%2FBgEARc0%2FV1kFGQAAQAAAAECTW%2FDBF9fDzz1AB8EsAAAAADQdnOXAAAAANB2c5f%2FUf%2BcBdwFFAAAAAgAAgAAAAAAAAABAAAFFP%2BFAAAFFP9R%2FtQF3AABAAAAAAAAAAAAAAAAAAAAowG4ACgAAAAAAZAAAASwAAAEsABkBLAAAASwAAAEsABwAooAAAUUAAACigAABRQAAAGxAAABRQAAANgAAADYAAAAogAAAQQAAABIAAABBAAAAUUAAASwAGQEsAB7BLAAyASwAMgB9AAABLD%2F8gSwAAAEsAAABLD%2F8ASwAAAEsAAOBLAACQSwAGQEsP%2FTBLD%2F0wSwAAAEsAAABLAAAASwAAAEsAAABLAAJgSwAG4EsAAXBLAAFwSwABcEsABkBLAAGgSwAGQEsAAMBLAAZASwABcEsP%2BcBLAAZASwABcEsAAXBLAAAASwABcEsAAXBLAAFwSwAGQEsAAABLAAZASwAAAEsAAABLAAAASwAAAEsAAABLAAAASwAAAEsAAABLAAZASwAMgEsAAABLAAAASwADUEsABkBLAAyASw%2F7UEsAAhBLAAAASwAAAEsAAABLAAAASwAAAEsP%2BcBLAAAASwAAAEsAAABLAA2wSwABcEsAB1BLAAAASwAAAEsAAABLAACgSwAMgEsAAABLAAnQSwAMgEsADIBLAAyASwAAAEsP%2F%2BBLABLASwAGQEsACIBLABOwSwABcEsAAXBLAAFwSwABcEsAAXBLAAFwSwAAAEsAAXBLAAFwSwABcEsAAXBLAAAASwALcEsAC3BLAAAASwAAAEsABJBLAAFwSwAAAEsAAABLAAXQSw%2F9wEsP%2FcBLD%2FnwSwAGQEsAAABLAAAASwAAAEsABkBLD%2F%2FwSwAAAEsP9RBLAABgSwAAAEsAAABLABRQSwAAEEsAAABLD%2FnASwAEoEsAAUBLAAAASwAAAEsAAABLD%2FnASwAGEEsP%2F9BLAAFgSwABYEsAAWBLAAFgSwABgEsAAABMQAAASwAGQAAAAAAAD%2F2ABkADkAyAAAAScAZAAZABkAGQAZABkAGQAZAAAAAAAAAAAAAADZAAAAAAAOAAAAAAAAAAAAAAAEAAAAAAAAAAAAAAAAAAMAZABkAAAAEAAAAAAAZP%2Bc%2F5z%2FnP%2Bc%2F5z%2FnP%2Bc%2F5wACQAJ%2F%2FL%2F8gBkAHkAJwBkAGQAAAAAAGT%2FogAAAAAAAAAAAAAAAADIAGQAAAABAI8AAP%2Bc%2F5wAZAAEAMgAyAAAAGQBkABkAAAAZAEs%2F7UAAAAAAAAAAAAAAAAAAABkAAABLAFBADIAMgAIAAAAAADIAT4AZgBmANkAyADIAAAAKgAqACoAKgCyAOgA6AFOAU4BTgFOAU4BTgFOAU4BTgFOAU4BTgFOAU4BpAIGAiICfgKGAqwC5ANGA24DjAPEBAgEMgRiBKIE3AVcBboGcgb0ByAHYgfKCB4IYgi%2BCTYJhAm2Cd4KKApMCpQK4gswC4oLygwIDFgNKg1eDbAODg5oDrQPKA%2BmD%2BYQEhBUEJAQqhEqEXYRthIKEjgSfBLAExoTdBPQFCoU1BU8FagVzBYEFjYWYBawFv4XUhemGAIYLhhqGJYYsBjgGP4ZKBloGZQZxBnaGe4aNhpoGrga9hteG7QcMhyUHOIdHB1EHWwdlB28HeYeLh52HsAfYh%2FSIEYgviEyIXYhuCJAIpYiuCMOIyIjOCN6I8Ij4CQCJDAkXiSWJOIlNCVgJbwmFCZ%2BJuYnUCe8J%2FgoNChwKKwpoCnMKiYqSiqEKworeiwILGgsuizsLRwtiC30LiguZi6iLtgvDi9GL34vsi%2F4MD4whDDSMRIxYDGuMegyJDJeMpoy3jMiMz4zaDO2NBg0YDSoNNI1LDWeNeg2PjZ8Ntw3GjdON5I31DgQOEI4hjjIOQo5SjmIOcw6HDpsOpo63jugO9w8GDxQPKI8%2BD0yPew%2BOj6MPtQ%2FKD9uP6o%2F%2BkBIQIBAxkECQX5CGEKoQu5DGENCQ3ZDoEPKRBBEYESuRPZFWkW2RgZGdEa0RvZHNkd2R7ZH9kgWSDJITkhqSIZIzEkSSThJXkmESapKAkouSlIAAQAAARcApwARAAAAAAACAAAAAQABAAAAQAAuAAAAAAAAABAAxgABAAAAAAATABIAAAADAAEECQAAAGoAEgADAAEECQABACgAfAADAAEECQACAA4ApAADAAEECQADAEwAsgADAAEECQAEADgA%2FgADAAEECQAFAHgBNgADAAEECQAGADYBrgADAAEECQAIABYB5AADAAEECQAJABYB%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%2FtQAyAAAAAAAAAAAAAAAAAAAAAAAAAAABFwAAAQIBAwADAA0ADgEEAJYBBQEGAQcBCAEJAQoBCwEMAQ0BDgEPARABEQESARMA7wEUARUBFgEXARgBGQEaARsBHAEdAR4BHwEgASEBIgEjASQBJQEmAScBKAEpASoBKwEsAS0BLgEvATABMQEyATMBNAE1ATYBNwE4ATkBOgE7ATwBPQE%2BAT8BQAFBAUIBQwFEAUUBRgFHAUgBSQFKAUsBTAFNAU4BTwFQAVEBUgFTAVQBVQFWAVcBWAFZAVoBWwFcAV0BXgFfAWABYQFiAWMBZAFlAWYBZwFoAWkBagFrAWwBbQFuAW8BcAFxAXIBcwF0AXUBdgF3AXgBeQF6AXsBfAF9AX4BfwGAAYEBggGDAYQBhQGGAYcBiAGJAYoBiwGMAY0BjgGPAZABkQGSAZMBlAGVAZYBlwGYAZkBmgGbAZwBnQGeAZ8BoAGhAaIBowGkAaUBpgGnAagBqQGqAasBrAGtAa4BrwGwAbEBsgGzAbQBtQG2AbcBuAG5AboBuwG8Ab0BvgG%2FAcABwQHCAcMBxAHFAcYBxwHIAckBygHLAcwBzQHOAc8B0AHRAdIB0wHUAdUB1gHXAdgB2QHaAdsB3AHdAd4B3wHgAeEB4gHjAeQB5QHmAecB6AHpAeoB6wHsAe0B7gHvAfAB8QHyAfMB9AH1AfYB9wH4AfkB%2BgH7AfwB%2FQH%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%3D%29%20format%28%27truetype%27%29%2Curl%28data%3Aimage%2Fsvg%2Bxml%3Bbase64%2CPD94bWwgdmVyc2lvbj0iMS4wIiBzdGFuZGFsb25lPSJubyI%2FPgo8IURPQ1RZUEUgc3ZnIFBVQkxJQyAiLS8vVzNDLy9EVEQgU1ZHIDEuMS8vRU4iICJodHRwOi8vd3d3LnczLm9yZy9HcmFwaGljcy9TVkcvMS4xL0RURC9zdmcxMS5kdGQiID4KPHN2ZyB4bWxucz0iaHR0cDovL3d3dy53My5vcmcvMjAwMC9zdmciPgo8bWV0YWRhdGE%2BPC9tZXRhZGF0YT4KPGRlZnM%2BCjxmb250IGlkPSJnbHlwaGljb25zX2hhbGZsaW5nc3JlZ3VsYXIiIGhvcml6LWFkdi14PSIxMjAwIiA%2BCjxmb250LWZhY2UgdW5pdHMtcGVyLWVtPSIxMjAwIiBhc2NlbnQ9Ijk2MCIgZGVzY2VudD0iLTI0MCIgLz4KPG1pc3NpbmctZ2x5cGggaG9yaXotYWR2LXg9IjUwMCIgLz4KPGdseXBoIGhvcml6LWFkdi14PSIwIiAvPgo8Z2x5cGggaG9yaXotYWR2LXg9IjQwMCIgLz4KPGdseXBoIHVuaWNvZGU9IiAiIC8%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%2BCjxnbHlwaCB1bmljb2RlPSImI3gyMDAzOyIgaG9yaXotYWR2LXg9IjEzMDAiIC8%2BCjxnbHlwaCB1bmljb2RlPSImI3gyMDA0OyIgaG9yaXotYWR2LXg9IjQzMyIgLz4KPGdseXBoIHVuaWNvZGU9IiYjeDIwMDU7IiBob3Jpei1hZHYteD0iMzI1IiAvPgo8Z2x5cGggdW5pY29kZT0iJiN4MjAwNjsiIGhvcml6LWFkdi14PSIyMTYiIC8%2BCjxnbHlwaCB1bmljb2RlPSImI3gyMDA3OyIgaG9yaXotYWR2LXg9IjIxNiIgLz4KPGdseXBoIHVuaWNvZGU9IiYjeDIwMDg7IiBob3Jpei1hZHYteD0iMTYyIiAvPgo8Z2x5cGggdW5pY29kZT0iJiN4MjAwOTsiIGhvcml6LWFkdi14PSIyNjAiIC8%2BCjxnbHlwaCB1bmljb2RlPSImI3gyMDBhOyIgaG9yaXotYWR2LXg9IjcyIiAvPgo8Z2x5cGggdW5pY29kZT0iJiN4MjAyZjsiIGhvcml6LWFkdi14PSIyNjAiIC8%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%2BCjxnbHlwaCB1bmljb2RlPSImI3gyMGJkOyIgZD0iTTQyOCAxMjAwaDM1MHE2NyAwIDEyMCAtMTN0ODYgLTMxdDU3IC00OS41dDM1IC01Ni41dDE3IC02NC41dDYuNSAtNjAuNXQwLjUgLTU3di0xNi41di0xNi41cTAgLTM2IC0wLjUgLTU3dC02LjUgLTYxdC0xNyAtNjV0LTM1IC01N3QtNTcgLTUwLjV0LTg2IC0zMS41dC0xMjAgLTEzaC0xNzhsLTIgLTEwMGgyODhxMTAgMCAxMyAtNnQtMyAtMTRsLTEyMCAtMTYwcS02IC04IC0xOCAtMTR0LTIyIC02aC0xMzh2LTE3NXEwIC0xMSAtNS41IC0xOCB0LTE1LjUgLTdoLTE0OXEtMTAgMCAtMTcuNSA3LjV0LTcuNSAxNy41djE3NWgtMjY3cS0xMCAwIC0xMyA2dDMgMTRsMTIwIDE2MHE2IDggMTggMTR0MjIgNmgxMTd2MTAwaC0yNjdxLTEwIDAgLTEzIDZ0MyAxNGwxMjAgMTYwcTYgOCAxOCAxNHQyMiA2aDExN3Y0NzVxMCAxMCA3LjUgMTcuNXQxNy41IDcuNXpNNjAwIDEwMDB2LTMwMGgyMDNxNjQgMCA4Ni41IDMzdDIyLjUgMTE5cTAgODQgLTIyLjUgMTE2dC04Ni41IDMyaC0yMDN6IiAvPgo8Z2x5cGggdW5pY29kZT0iJiN4MjIxMjsiIGQ9Ik0yNTAgNzAwaDgwMHEyMSAwIDM1LjUgLTE0LjV0MTQuNSAtMzUuNXYtMjAwcTAgLTIxIC0xNC41IC0zNS41dC0zNS41IC0xNC41aC04MDBxLTIxIDAgLTM1LjUgMTQuNXQtMTQuNSAzNS41djIwMHEwIDIxIDE0LjUgMzUuNXQzNS41IDE0LjV6IiAvPgo8Z2x5cGggdW5pY29kZT0iJiN4MjMxYjsiIGQ9Ik0xMDAwIDEyMDB2LTE1MHEwIC0yMSAtMTQuNSAtMzUuNXQtMzUuNSAtMTQuNWgtNTB2LTEwMHEwIC05MSAtNDkuNSAtMTY1LjV0LTEzMC41IC0xMDkuNXE4MSAtMzUgMTMwLjUgLTEwOS41dDQ5LjUgLTE2NS41di0xNTBoNTBxMjEgMCAzNS41IC0xNC41dDE0LjUgLTM1LjV2LTE1MGgtODAwdjE1MHEwIDIxIDE0LjUgMzUuNXQzNS41IDE0LjVoNTB2MTUwcTAgOTEgNDkuNSAxNjUuNXQxMzAuNSAxMDkuNXEtODEgMzUgLTEzMC41IDEwOS41IHQtNDkuNSAxNjUuNXYxMDBoLTUwcS0yMSAwIC0zNS41IDE0LjV0LTE0LjUgMzUuNXYxNTBoODAwek00MDAgMTAwMHYtMTAwcTAgLTYwIDMyLjUgLTEwOS41dDg3LjUgLTczLjVxMjggLTEyIDQ0IC0zN3QxNiAtNTV0LTE2IC01NXQtNDQgLTM3cS01NSAtMjQgLTg3LjUgLTczLjV0LTMyLjUgLTEwOS41di0xNTBoNDAwdjE1MHEwIDYwIC0zMi41IDEwOS41dC04Ny41IDczLjVxLTI4IDEyIC00NCAzN3QtMTYgNTV0MTYgNTV0NDQgMzcgcTU1IDI0IDg3LjUgNzMuNXQzMi41IDEwOS41djEwMGgtNDAweiIgLz4KPGdseXBoIHVuaWNvZGU9IiYjeDI1ZmM7IiBob3Jpei1hZHYteD0iNTAwIiBkPSJNMCAweiIgLz4KPGdseXBoIHVuaWNvZGU9IiYjeDI2MDE7IiBkPSJNNTAzIDEwODlxMTEwIDAgMjAwLjUgLTU5LjV0MTM0LjUgLTE1Ni41cTQ0IDE0IDkwIDE0cTEyMCAwIDIwNSAtODYuNXQ4NSAtMjA2LjVxMCAtMTIxIC04NSAtMjA3LjV0LTIwNSAtODYuNWgtNzUwcS03OSAwIC0xMzUuNSA1N3QtNTYuNSAxMzdxMCA2OSA0Mi41IDEyMi41dDEwOC41IDY3LjVxLTIgMTIgLTIgMzdxMCAxNTMgMTA4IDI2MC41dDI2MCAxMDcuNXoiIC8%2BCjxnbHlwaCB1bmljb2RlPSImI3gyNmZhOyIgZD0iTTc3NCAxMTkzLjVxMTYgLTkuNSAyMC41IC0yN3QtNS41IC0zMy41bC0xMzYgLTE4N2w0NjcgLTc0NmgzMHEyMCAwIDM1IC0xOC41dDE1IC0zOS41di00MmgtMTIwMHY0MnEwIDIxIDE1IDM5LjV0MzUgMTguNWgzMGw0NjggNzQ2bC0xMzUgMTgzcS0xMCAxNiAtNS41IDM0dDIwLjUgMjh0MzQgNS41dDI4IC0yMC41bDExMSAtMTQ4bDExMiAxNTBxOSAxNiAyNyAyMC41dDM0IC01ek02MDAgMjAwaDM3N2wtMTgyIDExMmwtMTk1IDUzNHYtNjQ2eiAiIC8%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%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%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%2BCjxnbHlwaCB1bmljb2RlPSImI3hlMDI0OyIgZD0iTTEzMDAgMGgtNTM4bC00MSA0MDBoLTI0MmwtNDEgLTQwMGgtNTM4bDQzMSAxMjAwaDIwOWwtMjEgLTMwMGgxNjJsLTIwIDMwMGgyMDh6TTUxNSA4MDBsLTI3IC0zMDBoMjI0bC0yNyAzMDBoLTE3MHoiIC8%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%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%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%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%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%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%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%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%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%2BCjxnbHlwaCB1bmljb2RlPSImI3hlMDQyOyIgZD0iTTUwMCAxMjAwbDY4MiAtNjgycTggLTggOCAtMTh0LTggLTE4bC00NjQgLTQ2NHEtOCAtOCAtMTggLTh0LTE4IDhsLTY4MiA2ODJsMSA0NzVxMCAxMCA3LjUgMTcuNXQxNy41IDcuNWg0NzR6TTgwMCAxMjAwbDY4MiAtNjgycTggLTggOCAtMTh0LTggLTE4bC00NjQgLTQ2NHEtOCAtOCAtMTggLTh0LTE4IDhsLTU2IDU2bDQyNCA0MjZsLTcwMCA3MDBoMTUwek0zMTkuNSAxMDI0LjVxLTI5LjUgMjkuNSAtNzEgMjkuNXQtNzEgLTI5LjUgdC0yOS41IC03MS41dDI5LjUgLTcxLjV0NzEgLTI5LjV0NzEgMjkuNXQyOS41IDcxLjV0LTI5LjUgNzEuNXoiIC8%2BCjxnbHlwaCB1bmljb2RlPSImI3hlMDQzOyIgZD0iTTMwMCAxMjAwaDgyNXE3NSAwIDc1IC03NXYtOTAwcTAgLTI1IC0xOCAtNDNsLTY0IC02NHEtOCAtOCAtMTMgLTUuNXQtNSAxMi41djk1MHEwIDEwIC03LjUgMTcuNXQtMTcuNSA3LjVoLTcwMHEtMjUgMCAtNDMgLTE4bC02NCAtNjRxLTggLTggLTUuNSAtMTN0MTIuNSAtNWg3MDBxMTAgMCAxNy41IC03LjV0Ny41IC0xNy41di05NTBxMCAtMTAgLTcuNSAtMTcuNXQtMTcuNSAtNy41aC04NTBxLTEwIDAgLTE3LjUgNy41dC03LjUgMTcuNXY5NzUgcTAgMjUgMTggNDNsMTM5IDEzOXExOCAxOCA0MyAxOHoiIC8%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%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%2BCjxnbHlwaCB1bmljb2RlPSImI3hlMDQ5OyIgZD0iTTg3NyAxMjAwbDIgLTU3cS04MyAtMTkgLTExNiAtNDUuNXQtNDAgLTY2LjVsLTEzMiAtODM5cS05IC00OSAxMyAtNjl0OTYgLTI2di05N2gtNTAwdjk3cTE4NiAxNiAyMDAgOThsMTczIDgzMnEzIDE3IDMgMzB0LTEuNSAyMi41dC05IDE3LjV0LTEzLjUgMTIuNXQtMjEuNSAxMHQtMjYgOC41dC0zMy41IDEwcS0xMyAzIC0xOSA1djU3aDQyNXoiIC8%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%2BCjxnbHlwaCB1bmljb2RlPSImI3hlMDU0OyIgZD0iTTUwMCA5NTB2MTAwcTAgMjEgMTQuNSAzNS41dDM1LjUgMTQuNWg2MDBxMjEgMCAzNS41IC0xNC41dDE0LjUgLTM1LjV2LTEwMHEwIC0yMSAtMTQuNSAtMzUuNXQtMzUuNSAtMTQuNWgtNjAwcS0yMSAwIC0zNS41IDE0LjV0LTE0LjUgMzUuNXpNMTAwIDY1MHYxMDBxMCAyMSAxNC41IDM1LjV0MzUuNSAxNC41aDEwMDBxMjEgMCAzNS41IC0xNC41dDE0LjUgLTM1LjV2LTEwMHEwIC0yMSAtMTQuNSAtMzUuNXQtMzUuNSAtMTQuNWgtMTAwMCBxLTIxIDAgLTM1LjUgMTQuNXQtMTQuNSAzNS41ek0zMDAgMzUwdjEwMHEwIDIxIDE0LjUgMzUuNXQzNS41IDE0LjVoODAwcTIxIDAgMzUuNSAtMTQuNXQxNC41IC0zNS41di0xMDBxMCAtMjEgLTE0LjUgLTM1LjV0LTM1LjUgLTE0LjVoLTgwMHEtMjEgMCAtMzUuNSAxNC41dC0xNC41IDM1LjV6TTAgNTB2MTAwcTAgMjEgMTQuNSAzNS41dDM1LjUgMTQuNWgxMTAwcTIxIDAgMzUuNSAtMTQuNXQxNC41IC0zNS41di0xMDAgcTAgLTIxIC0xNC41IC0zNS41dC0zNS41IC0xNC41aC0xMTAwcS0yMSAwIC0zNS41IDE0LjV0LTE0LjUgMzUuNXoiIC8%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%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%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%2BCjxnbHlwaCB1bmljb2RlPSImI3hlMDY5OyIgZD0iTTI1MCAxMTAwaDEwMHEyMSAwIDM1LjUgLTE0LjV0MTQuNSAtMzUuNXYtNDM4bDQ2NCA0NTNxMTUgMTQgMjUuNSAxMHQxMC41IC0yNXYtMTAwMHEwIC0yMSAtMTAuNSAtMjV0LTI1LjUgMTBsLTQ2NCA0NTN2LTQzOHEwIC0yMSAtMTQuNSAtMzUuNXQtMzUuNSAtMTQuNWgtMTAwcS0yMSAwIC0zNS41IDE0LjV0LTE0LjUgMzUuNXYxMDAwcTAgMjEgMTQuNSAzNS41dDM1LjUgMTQuNXoiIC8%2BCjxnbHlwaCB1bmljb2RlPSImI3hlMDcwOyIgZD0iTTUwIDExMDBoMTAwcTIxIDAgMzUuNSAtMTQuNXQxNC41IC0zNS41di00MzhsNDY0IDQ1M3ExNSAxNCAyNS41IDEwdDEwLjUgLTI1di00MzhsNDY0IDQ1M3ExNSAxNCAyNS41IDEwdDEwLjUgLTI1di0xMDAwcTAgLTIxIC0xMC41IC0yNXQtMjUuNSAxMGwtNDY0IDQ1M3YtNDM4cTAgLTIxIC0xMC41IC0yNXQtMjUuNSAxMGwtNDY0IDQ1M3YtNDM4cTAgLTIxIC0xNC41IC0zNS41dC0zNS41IC0xNC41aC0xMDBxLTIxIDAgLTM1LjUgMTQuNSB0LTE0LjUgMzUuNXYxMDAwcTAgMjEgMTQuNSAzNS41dDM1LjUgMTQuNXoiIC8%2BCjxnbHlwaCB1bmljb2RlPSImI3hlMDcxOyIgZD0iTTEyMDAgMTA1MHYtMTAwMHEwIC0yMSAtMTAuNSAtMjV0LTI1LjUgMTBsLTQ2NCA0NTN2LTQzOHEwIC0yMSAtMTAuNSAtMjV0LTI1LjUgMTBsLTQ5MiA0ODBxLTE1IDE0IC0xNSAzNXQxNSAzNWw0OTIgNDgwcTE1IDE0IDI1LjUgMTB0MTAuNSAtMjV2LTQzOGw0NjQgNDUzcTE1IDE0IDI1LjUgMTB0MTAuNSAtMjV6IiAvPgo8Z2x5cGggdW5pY29kZT0iJiN4ZTA3MjsiIGQ9Ik0yNDMgMTA3NGw4MTQgLTQ5OHExOCAtMTEgMTggLTI2dC0xOCAtMjZsLTgxNCAtNDk4cS0xOCAtMTEgLTMwLjUgLTR0LTEyLjUgMjh2MTAwMHEwIDIxIDEyLjUgMjh0MzAuNSAtNHoiIC8%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%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%2BCjxnbHlwaCB1bmljb2RlPSImI3hlMDc5OyIgZD0iTTg4NSA5MDBsLTM1MiAtMzUzbDM1MiAtMzUzbC0xOTcgLTE5OGwtNTUyIDU1Mmw1NTIgNTUweiIgLz4KPGdseXBoIHVuaWNvZGU9IiYjeGUwODA7IiBkPSJNMTA2NCA1NDdsLTU1MSAtNTUxbC0xOTggMTk4bDM1MyAzNTNsLTM1MyAzNTNsMTk4IDE5OHoiIC8%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%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%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%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%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%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%2BCjxnbHlwaCB1bmljb2RlPSImI3hlMDk0OyIgZD0iTTU1MCAxMjAwaDIwMHEyMSAwIDM1LjUgLTE0LjV0MTQuNSAtMzUuNXYtNTUwaDI5MXEyMSAwIDI2IC0xMS41dC04IC0yNy41bC00MjcgLTUyMnEtMTMgLTE2IC0zMiAtMTZ0LTMyIDE2bC00MjcgNTIycS0xMyAxNiAtOCAyNy41dDI2IDExLjVoMjkxdjU1MHEwIDIxIDE0LjUgMzUuNXQzNS41IDE0LjV6IiAvPgo8Z2x5cGggdW5pY29kZT0iJiN4ZTA5NTsiIGQ9Ik02MzkgMTEwOWw1MjIgLTQyN3ExNiAtMTMgMTYgLTMydC0xNiAtMzJsLTUyMiAtNDI3cS0xNiAtMTMgLTI3LjUgLTh0LTExLjUgMjZ2MjkxcS05NCAtMiAtMTgyIC0yMHQtMTcwLjUgLTUydC0xNDcgLTkyLjV0LTEwMC41IC0xMzUuNXE1IDEwNSAyNyAxOTMuNXQ2Ny41IDE2N3QxMTMgMTM1dDE2NyA5MS41dDIyNS41IDQydjI2MnEwIDIxIDExLjUgMjZ0MjcuNSAtOHoiIC8%2BCjxnbHlwaCB1bmljb2RlPSImI3hlMDk2OyIgZD0iTTg1MCAxMjAwaDMwMHEyMSAwIDM1LjUgLTE0LjV0MTQuNSAtMzUuNXYtMzAwcTAgLTIxIC0xMC41IC0yNXQtMjQuNSAxMGwtOTQgOTRsLTI0OSAtMjQ5cS04IC03IC0xOCAtN3QtMTggN2wtMTA2IDEwNnEtNyA4IC03IDE4dDcgMThsMjQ5IDI0OWwtOTQgOTRxLTE0IDE0IC0xMCAyNC41dDI1IDEwLjV6TTM1MCAwaC0zMDBxLTIxIDAgLTM1LjUgMTQuNXQtMTQuNSAzNS41djMwMHEwIDIxIDEwLjUgMjV0MjQuNSAtMTBsOTQgLTk0bDI0OSAyNDkgcTggNyAxOCA3dDE4IC03bDEwNiAtMTA2cTcgLTggNyAtMTh0LTcgLTE4bC0yNDkgLTI0OWw5NCAtOTRxMTQgLTE0IDEwIC0yNC41dC0yNSAtMTAuNXoiIC8%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%2BCjxnbHlwaCB1bmljb2RlPSImI3hlMTAxOyIgZD0iTTYwMCAxMTc3cTExNyAwIDIyNCAtNDUuNXQxODQuNSAtMTIzdDEyMyAtMTg0LjV0NDUuNSAtMjI0dC00NS41IC0yMjR0LTEyMyAtMTg0LjV0LTE4NC41IC0xMjN0LTIyNCAtNDUuNXQtMjI0IDQ1LjV0LTE4NC41IDEyM3QtMTIzIDE4NC41dC00NS41IDIyNHQ0NS41IDIyNHQxMjMgMTg0LjV0MTg0LjUgMTIzdDIyNCA0NS41ek03MDQgOTAwaC0yMDhxLTIwIDAgLTMyIC0xNC41dC04IC0zNC41bDU4IC0zMDJxNCAtMjAgMjEuNSAtMzQuNSB0MzcuNSAtMTQuNWg1NHEyMCAwIDM3LjUgMTQuNXQyMS41IDM0LjVsNTggMzAycTQgMjAgLTggMzQuNXQtMzIgMTQuNXpNNjc1IDQwMGgtMTUwcS0xMCAwIC0xNy41IC03LjV0LTcuNSAtMTcuNXYtMTUwcTAgLTEwIDcuNSAtMTcuNXQxNy41IC03LjVoMTUwcTEwIDAgMTcuNSA3LjV0Ny41IDE3LjV2MTUwcTAgMTAgLTcuNSAxNy41dC0xNy41IDcuNXoiIC8%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%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%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%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%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%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%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%2BCjxnbHlwaCB1bmljb2RlPSImI3hlMTI2OyIgZD0iTTYwMCAxMTAwaDI1MHEyMyAwIDQ1IC0xNi41dDM4IC00MC41bDIzOCAtMzQ0cTI5IC0yOSAyOSAtNzR2LTEwMHEwIC00NCAtMzAgLTg0LjV0LTcwIC00MC41aC0zMjhxMjggLTExOCAyOCAtMTI1di0xNTBxMCAtNDQgLTMwIC04NC41dC03MCAtNDAuNWgtNTBxLTI3IDAgLTUxLjUgMjB0LTM3LjUgNDhsLTk2IDE5OGwtMTQ1IDE5NnEtMjAgMjcgLTIwIDYzdjQwMHEwIDM5IDI3LjUgNTd0NzIuNSAxOGg2MXExMjQgMTAwIDEzOSAxMDB6IE01MCAxMDAwaDEwMHEyMSAwIDM1LjUgLTE0LjV0MTQuNSAtMzUuNXYtNTAwcTAgLTIxIC0xNC41IC0zNS41dC0zNS41IC0xNC41aC0xMDBxLTIxIDAgLTM1LjUgMTQuNXQtMTQuNSAzNS41djUwMHEwIDIxIDE0LjUgMzUuNXQzNS41IDE0LjV6TTYzNiAxMDAwbC0xMzYgLTEwMGgtMTAwdi0zNzVsMTUwIC0yMTNsMTAwIC0yMTJoNTB2MTc1bC01MCAyMjVoNDUwdjEyNWwtMjUwIDM3NWgtMjE0eiIgLz4KPGdseXBoIHVuaWNvZGU9IiYjeGUxMjc7IiBkPSJNMzU2IDg3M2wzNjMgMjMwcTMxIDE2IDUzIC02bDExMCAtMTEycTEzIC0xMyAxMy41IC0zMnQtMTEuNSAtMzRsLTg0IC0xMjFoMzAycTg0IDAgMTM4IC0zOHQ1NCAtMTEwdC01NSAtMTExdC0xMzkgLTM5aC0xMDZsLTEzMSAtMzM5cS02IC0yMSAtMTkuNSAtNDF0LTI4LjUgLTIwaC0zNDJxLTcgMCAtOTAgODF0LTgzIDk0djUyNXEwIDE3IDE0IDM1LjV0MjggMjguNXpNNDAwIDc5MnYtNTAzbDEwMCAtODloMjkzbDEzMSAzMzkgcTYgMjEgMTkuNSA0MXQyOC41IDIwaDIwM3EyMSAwIDMwLjUgMjV0MC41IDUwdC0zMSAyNWgtNDU2aC03aC02aC01LjV0LTYgMC41dC01IDEuNXQtNSAydC00IDIuNXQtNCA0dC0yLjUgNC41cS0xMiAyNSA1IDQ3bDE0NiAxODNsLTg2IDgzek01MCA4MDBoMTAwcTIxIDAgMzUuNSAtMTQuNXQxNC41IC0zNS41di01MDBxMCAtMjEgLTE0LjUgLTM1LjV0LTM1LjUgLTE0LjVoLTEwMHEtMjEgMCAtMzUuNSAxNC41dC0xNC41IDM1LjV2NTAwIHEwIDIxIDE0LjUgMzUuNXQzNS41IDE0LjV6IiAvPgo8Z2x5cGggdW5pY29kZT0iJiN4ZTEyODsiIGQ9Ik00NzUgMTEwM2wzNjYgLTIzMHEyIC0xIDYgLTMuNXQxNCAtMTAuNXQxOCAtMTYuNXQxNC41IC0yMHQ2LjUgLTIyLjV2LTUyNXEwIC0xMyAtODYgLTk0dC05MyAtODFoLTM0MnEtMTUgMCAtMjguNSAyMHQtMTkuNSA0MWwtMTMxIDMzOWgtMTA2cS04NSAwIC0xMzkuNSAzOXQtNTQuNSAxMTF0NTQgMTEwdDEzOCAzOGgzMDJsLTg1IDEyMXEtMTEgMTUgLTEwLjUgMzR0MTMuNSAzMmwxMTAgMTEycTIyIDIyIDUzIDZ6TTM3MCA5NDVsMTQ2IC0xODMgcTE3IC0yMiA1IC00N3EtMiAtMiAtMy41IC00LjV0LTQgLTR0LTQgLTIuNXQtNSAtMnQtNSAtMS41dC02IC0wLjVoLTZoLTYuNWgtNmgtNDc1di0xMDBoMjIxcTE1IDAgMjkgLTIwdDIwIC00MWwxMzAgLTMzOWgyOTRsMTA2IDg5djUwM2wtMzQyIDIzNnpNMTA1MCA4MDBoMTAwcTIxIDAgMzUuNSAtMTQuNXQxNC41IC0zNS41di01MDBxMCAtMjEgLTE0LjUgLTM1LjV0LTM1LjUgLTE0LjVoLTEwMHEtMjEgMCAtMzUuNSAxNC41dC0xNC41IDM1LjUgdjUwMHEwIDIxIDE0LjUgMzUuNXQzNS41IDE0LjV6IiAvPgo8Z2x5cGggdW5pY29kZT0iJiN4ZTEyOTsiIGQ9Ik01NTAgMTI5NHE3MiAwIDExMSAtNTV0MzkgLTEzOXYtMTA2bDMzOSAtMTMxcTIxIC02IDQxIC0xOS41dDIwIC0yOC41di0zNDJxMCAtNyAtODEgLTkwdC05NCAtODNoLTUyNXEtMTcgMCAtMzUuNSAxNHQtMjguNSAyOGwtOSAxNGwtMjMwIDM2M3EtMTYgMzEgNiA1M2wxMTIgMTEwcTEzIDEzIDMyIDEzLjV0MzQgLTExLjVsMTIxIC04NHYzMDJxMCA4NCAzOCAxMzh0MTEwIDU0ek02MDAgOTcydjIwM3EwIDIxIC0yNSAzMC41dC01MCAwLjUgdC0yNSAtMzF2LTQ1NnYtN3YtNnYtNS41dC0wLjUgLTZ0LTEuNSAtNXQtMiAtNXQtMi41IC00dC00IC00dC00LjUgLTIuNXEtMjUgLTEyIC00NyA1bC0xODMgMTQ2bC04MyAtODZsMjM2IC0zMzloNTAzbDg5IDEwMHYyOTNsLTMzOSAxMzFxLTIxIDYgLTQxIDE5LjV0LTIwIDI4LjV6TTQ1MCAyMDBoNTAwcTIxIDAgMzUuNSAtMTQuNXQxNC41IC0zNS41di0xMDBxMCAtMjEgLTE0LjUgLTM1LjV0LTM1LjUgLTE0LjVoLTUwMCBxLTIxIDAgLTM1LjUgMTQuNXQtMTQuNSAzNS41djEwMHEwIDIxIDE0LjUgMzUuNXQzNS41IDE0LjV6IiAvPgo8Z2x5cGggdW5pY29kZT0iJiN4ZTEzMDsiIGQ9Ik0zNTAgMTEwMGg1MDBxMjEgMCAzNS41IDE0LjV0MTQuNSAzNS41djEwMHEwIDIxIC0xNC41IDM1LjV0LTM1LjUgMTQuNWgtNTAwcS0yMSAwIC0zNS41IC0xNC41dC0xNC41IC0zNS41di0xMDBxMCAtMjEgMTQuNSAtMzUuNXQzNS41IC0xNC41ek02MDAgMzA2di0xMDZxMCAtODQgLTM5IC0xMzl0LTExMSAtNTV0LTExMCA1NHQtMzggMTM4djMwMmwtMTIxIC04NHEtMTUgLTEyIC0zNCAtMTEuNXQtMzIgMTMuNWwtMTEyIDExMCBxLTIyIDIyIC02IDUzbDIzMCAzNjNxMSAyIDMuNSA2dDEwLjUgMTMuNXQxNi41IDE3dDIwIDEzLjV0MjIuNSA2aDUyNXExMyAwIDk0IC04M3Q4MSAtOTB2LTM0MnEwIC0xNSAtMjAgLTI4LjV0LTQxIC0xOS41ek0zMDggOTAwbC0yMzYgLTMzOWw4MyAtODZsMTgzIDE0NnEyMiAxNyA0NyA1cTIgLTEgNC41IC0yLjV0NCAtNHQyLjUgLTR0MiAtNXQxLjUgLTV0MC41IC02di01LjV2LTZ2LTd2LTQ1NnEwIC0yMiAyNSAtMzF0NTAgMC41dDI1IDMwLjUgdjIwM3EwIDE1IDIwIDI4LjV0NDEgMTkuNWwzMzkgMTMxdjI5M2wtODkgMTAwaC01MDN6IiAvPgo8Z2x5cGggdW5pY29kZT0iJiN4ZTEzMTsiIGQ9Ik02MDAgMTE3OHExMTggMCAyMjUgLTQ1LjV0MTg0LjUgLTEyM3QxMjMgLTE4NC41dDQ1LjUgLTIyNXQtNDUuNSAtMjI1dC0xMjMgLTE4NC41dC0xODQuNSAtMTIzdC0yMjUgLTQ1LjV0LTIyNSA0NS41dC0xODQuNSAxMjN0LTEyMyAxODQuNXQtNDUuNSAyMjV0NDUuNSAyMjV0MTIzIDE4NC41dDE4NC41IDEyM3QyMjUgNDUuNXpNOTE0IDYzMmwtMjc1IDIyM3EtMTYgMTMgLTI3LjUgOHQtMTEuNSAtMjZ2LTEzN2gtMjc1IHEtMTAgMCAtMTcuNSAtNy41dC03LjUgLTE3LjV2LTE1MHEwIC0xMCA3LjUgLTE3LjV0MTcuNSAtNy41aDI3NXYtMTM3cTAgLTIxIDExLjUgLTI2dDI3LjUgOGwyNzUgMjIzcTE2IDEzIDE2IDMydC0xNiAzMnoiIC8%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%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%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%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%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%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%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%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%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%2BCjxnbHlwaCB1bmljb2RlPSImI3hlMTYyOyIgZD0iTTc5MyAxMTgybDkgLTlxOCAtMTAgNSAtMjdxLTMgLTExIC03OSAtMjI1LjV0LTc4IC0yMjEuNWwzMDAgMXEyNCAwIDMyLjUgLTE3LjV0LTUuNSAtMzUuNXEtMSAwIC0xMzMuNSAtMTU1dC0yNjcgLTMxMi41dC0xMzguNSAtMTYyLjVxLTEyIC0xNSAtMjYgLTE1aC05bC05IDhxLTkgMTEgLTQgMzJxMiA5IDQyIDEyMy41dDc5IDIyNC41bDM5IDExMGgtMzAycS0yMyAwIC0zMSAxOXEtMTAgMjEgNiA0MXE3NSA4NiAyMDkuNSAyMzcuNSB0MjI4IDI1N3Q5OC41IDExMS41cTkgMTYgMjUgMTZoOXoiIC8%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%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%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%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%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%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%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%2BCjxnbHlwaCB1bmljb2RlPSImI3hlMTkwOyIgZD0iTTIwMCAxMTAwaDcwMHExMjQgMCAyMTIgLTg4dDg4IC0yMTJ2LTUwMHEwIC0xMjQgLTg4IC0yMTJ0LTIxMiAtODhoLTcwMHEtMTI0IDAgLTIxMiA4OHQtODggMjEydjUwMHEwIDEyNCA4OCAyMTJ0MjEyIDg4ek0xMDAgOTAwdi03MDBoOTAwdjcwMGgtOTAwek05MDAgMzAwaC03MDB2NTAwaDcwMHYtNTAwek04MDAgNzAwaC0xMzBxLTM4IDAgLTY2LjUgLTQzdC0yOC41IC0xMDh0MjcgLTEwN3Q2OCAtNDJoMTMwdjMwMHpNMzAwIDcwMHYtMzAwIGgxMzBxNDEgMCA2OCA0MnQyNyAxMDd0LTI4LjUgMTA4dC02Ni41IDQzaC0xMzB6IiAvPgo8Z2x5cGggdW5pY29kZT0iJiN4ZTE5MTsiIGQ9Ik0yMDAgMTEwMGg3MDBxMTI0IDAgMjEyIC04OHQ4OCAtMjEydi01MDBxMCAtMTI0IC04OCAtMjEydC0yMTIgLTg4aC03MDBxLTEyNCAwIC0yMTIgODh0LTg4IDIxMnY1MDBxMCAxMjQgODggMjEydDIxMiA4OHpNMTAwIDkwMHYtNzAwaDkwMHY3MDBoLTkwMHpNNTAwIDcwMGgtMjAwdi0xMDBoMjAwdi0zMDBoLTMwMHYxMDBoMjAwdjEwMGgtMjAwdjMwMGgzMDB2LTEwMHpNOTAwIDMwMGgtMTAwdjQwMGgtMTAwdjEwMGgyMDB2LTUwMHogTTcwMCAzMDBoLTEwMHYxMDBoMTAwdi0xMDB6IiAvPgo8Z2x5cGggdW5pY29kZT0iJiN4ZTE5MjsiIGQ9Ik0yMDAgMTEwMGg3MDBxMTI0IDAgMjEyIC04OHQ4OCAtMjEydi01MDBxMCAtMTI0IC04OCAtMjEydC0yMTIgLTg4aC03MDBxLTEyNCAwIC0yMTIgODh0LTg4IDIxMnY1MDBxMCAxMjQgODggMjEydDIxMiA4OHpNMTAwIDkwMHYtNzAwaDkwMHY3MDBoLTkwMHpNMzAwIDcwMGgyMDB2LTQwMGgtMzAwdjUwMGgxMDB2LTEwMHpNOTAwIDMwMGgtMTAwdjQwMGgtMTAwdjEwMGgyMDB2LTUwMHpNMzAwIDYwMHYtMjAwaDEwMHYyMDBoLTEwMHogTTcwMCAzMDBoLTEwMHYxMDBoMTAwdi0xMDB6IiAvPgo8Z2x5cGggdW5pY29kZT0iJiN4ZTE5MzsiIGQ9Ik0yMDAgMTEwMGg3MDBxMTI0IDAgMjEyIC04OHQ4OCAtMjEydi01MDBxMCAtMTI0IC04OCAtMjEydC0yMTIgLTg4aC03MDBxLTEyNCAwIC0yMTIgODh0LTg4IDIxMnY1MDBxMCAxMjQgODggMjEydDIxMiA4OHpNMTAwIDkwMHYtNzAwaDkwMHY3MDBoLTkwMHpNNTAwIDUwMGwtMTk5IC0yMDBoLTEwMHY1MGwxOTkgMjAwdjE1MGgtMjAwdjEwMGgzMDB2LTMwMHpNOTAwIDMwMGgtMTAwdjQwMGgtMTAwdjEwMGgyMDB2LTUwMHpNNzAxIDMwMGgtMTAwIHYxMDBoMTAwdi0xMDB6IiAvPgo8Z2x5cGggdW5pY29kZT0iJiN4ZTE5NDsiIGQ9Ik02MDAgMTE5MXExMjAgMCAyMjkuNSAtNDd0MTg4LjUgLTEyNnQxMjYgLTE4OC41dDQ3IC0yMjkuNXQtNDcgLTIyOS41dC0xMjYgLTE4OC41dC0xODguNSAtMTI2dC0yMjkuNSAtNDd0LTIyOS41IDQ3dC0xODguNSAxMjZ0LTEyNiAxODguNXQtNDcgMjI5LjV0NDcgMjI5LjV0MTI2IDE4OC41dDE4OC41IDEyNnQyMjkuNSA0N3pNNjAwIDEwMjFxLTExNCAwIC0yMTEgLTU2LjV0LTE1My41IC0xNTMuNXQtNTYuNSAtMjExdDU2LjUgLTIxMSB0MTUzLjUgLTE1My41dDIxMSAtNTYuNXQyMTEgNTYuNXQxNTMuNSAxNTMuNXQ1Ni41IDIxMXQtNTYuNSAyMTF0LTE1My41IDE1My41dC0yMTEgNTYuNXpNODAwIDcwMGgtMzAwdi0yMDBoMzAwdi0xMDBoLTMwMGwtMTAwIDEwMHYyMDBsMTAwIDEwMGgzMDB2LTEwMHoiIC8%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%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%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%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%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%2BCjxnbHlwaCB1bmljb2RlPSImI3hlMjIxOyIgZD0iTTQ4MCAxMTY1bDY4MiAtNjgzcTMxIC0zMSAzMSAtNzUuNXQtMzEgLTc1LjVsLTEzMSAtMTMxaC00ODFsLTUxNyA1MThxLTMyIDMxIC0zMiA3NS41dDMyIDc1LjVsMjk1IDI5NnEzMSAzMSA3NS41IDMxdDc2LjUgLTMxek0xMDggNzk0bDM0MiAtMzQybDMwMyAzMDRsLTM0MSAzNDF6TTI1MCAxMDBoODAwcTIxIDAgMzUuNSAtMTQuNXQxNC41IC0zNS41di01MGgtOTAwdjUwcTAgMjEgMTQuNSAzNS41dDM1LjUgMTQuNXoiIC8%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%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%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%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%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%2BCjxnbHlwaCB1bmljb2RlPSImI3hlMjQ0OyIgZD0iTTEyMDAgMTEwMGgtMTIwMHYxMDBoMTIwMHYtMTAwek01MCAxMDAwaDQwMHEyMSAwIDM1LjUgLTE0LjV0MTQuNSAtMzUuNXYtOTAwcTAgLTIxIC0xNC41IC0zNS41dC0zNS41IC0xNC41aC00MDBxLTIxIDAgLTM1LjUgMTQuNXQtMTQuNSAzNS41djkwMHEwIDIxIDE0LjUgMzUuNXQzNS41IDE0LjV6TTY1MCAxMDAwaDQwMHEyMSAwIDM1LjUgLTE0LjV0MTQuNSAtMzUuNXYtNDAwcTAgLTIxIC0xNC41IC0zNS41dC0zNS41IC0xNC41aC00MDAgcS0yMSAwIC0zNS41IDE0LjV0LTE0LjUgMzUuNXY0MDBxMCAyMSAxNC41IDM1LjV0MzUuNSAxNC41ek03MDAgOTAwdi0zMDBoMzAwdjMwMGgtMzAweiIgLz4KPGdseXBoIHVuaWNvZGU9IiYjeGUyNDU7IiBkPSJNNTAgMTIwMGg0MDBxMjEgMCAzNS41IC0xNC41dDE0LjUgLTM1LjV2LTkwMHEwIC0yMSAtMTQuNSAtMzUuNXQtMzUuNSAtMTQuNWgtNDAwcS0yMSAwIC0zNS41IDE0LjV0LTE0LjUgMzUuNXY5MDBxMCAyMSAxNC41IDM1LjV0MzUuNSAxNC41ek02NTAgNzAwaDQwMHEyMSAwIDM1LjUgLTE0LjV0MTQuNSAtMzUuNXYtNDAwcTAgLTIxIC0xNC41IC0zNS41dC0zNS41IC0xNC41aC00MDBxLTIxIDAgLTM1LjUgMTQuNXQtMTQuNSAzNS41djQwMCBxMCAyMSAxNC41IDM1LjV0MzUuNSAxNC41ek03MDAgNjAwdi0zMDBoMzAwdjMwMGgtMzAwek0xMjAwIDBoLTEyMDB2MTAwaDEyMDB2LTEwMHoiIC8%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%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%2BCjxnbHlwaCB1bmljb2RlPSImI3hlMjUwOyIgZD0iTTg2NSA1NjVsLTQ5NCAtNDk0cS0yMyAtMjMgLTQxIC0yM3EtMTQgMCAtMjIgMTMuNXQtOCAzOC41djEwMDBxMCAyNSA4IDM4LjV0MjIgMTMuNXExOCAwIDQxIC0yM2w0OTQgLTQ5NHExNCAtMTQgMTQgLTM1dC0xNCAtMzV6IiAvPgo8Z2x5cGggdW5pY29kZT0iJiN4ZTI1MTsiIGQ9Ik0zMzUgNjM1bDQ5NCA0OTRxMjkgMjkgNTAgMjAuNXQyMSAtNDkuNXYtMTAwMHEwIC00MSAtMjEgLTQ5LjV0LTUwIDIwLjVsLTQ5NCA0OTRxLTE0IDE0IC0xNCAzNXQxNCAzNXoiIC8%2BCjxnbHlwaCB1bmljb2RlPSImI3hlMjUyOyIgZD0iTTEwMCA5MDBoMTAwMHE0MSAwIDQ5LjUgLTIxdC0yMC41IC01MGwtNDk0IC00OTRxLTE0IC0xNCAtMzUgLTE0dC0zNSAxNGwtNDk0IDQ5NHEtMjkgMjkgLTIwLjUgNTB0NDkuNSAyMXoiIC8%2BCjxnbHlwaCB1bmljb2RlPSImI3hlMjUzOyIgZD0iTTYzNSA4NjVsNDk0IC00OTRxMjkgLTI5IDIwLjUgLTUwdC00OS41IC0yMWgtMTAwMHEtNDEgMCAtNDkuNSAyMXQyMC41IDUwbDQ5NCA0OTRxMTQgMTQgMzUgMTR0MzUgLTE0eiIgLz4KPGdseXBoIHVuaWNvZGU9IiYjeGUyNTQ7IiBkPSJNNzAwIDc0MXYtMTgybC02OTIgLTMyM3YyMjFsNDEzIDE5M2wtNDEzIDE5M3YyMjF6TTEyMDAgMGgtODAwdjIwMGg4MDB2LTIwMHoiIC8%2BCjxnbHlwaCB1bmljb2RlPSImI3hlMjU1OyIgZD0iTTEyMDAgOTAwaC0yMDB2LTEwMGgyMDB2LTEwMGgtMzAwdjMwMGgyMDB2MTAwaC0yMDB2MTAwaDMwMHYtMzAwek0wIDcwMGg1MHEwIDIxIDQgMzd0OS41IDI2LjV0MTggMTcuNXQyMiAxMXQyOC41IDUuNXQzMSAydDM3IDAuNWgxMDB2LTU1MHEwIC0yMiAtMjUgLTM0LjV0LTUwIC0xMy41bC0yNSAtMnYtMTAwaDQwMHYxMDBxLTQgMCAtMTEgMC41dC0yNCAzdC0zMCA3dC0yNCAxNXQtMTEgMjQuNXY1NTBoMTAwcTI1IDAgMzcgLTAuNXQzMSAtMiB0MjguNSAtNS41dDIyIC0xMXQxOCAtMTcuNXQ5LjUgLTI2LjV0NCAtMzdoNTB2MzAwaC04MDB2LTMwMHoiIC8%2BCjxnbHlwaCB1bmljb2RlPSImI3hlMjU2OyIgZD0iTTgwMCA3MDBoLTUwcTAgMjEgLTQgMzd0LTkuNSAyNi41dC0xOCAxNy41dC0yMiAxMXQtMjguNSA1LjV0LTMxIDJ0LTM3IDAuNWgtMTAwdi01NTBxMCAtMjIgMjUgLTM0LjV0NTAgLTE0LjVsMjUgLTF2LTEwMGgtNDAwdjEwMHE0IDAgMTEgMC41dDI0IDN0MzAgN3QyNCAxNXQxMSAyNC41djU1MGgtMTAwcS0yNSAwIC0zNyAtMC41dC0zMSAtMnQtMjguNSAtNS41dC0yMiAtMTF0LTE4IC0xNy41dC05LjUgLTI2LjV0LTQgLTM3aC01MHYzMDAgaDgwMHYtMzAwek0xMTAwIDIwMGgtMjAwdi0xMDBoMjAwdi0xMDBoLTMwMHYzMDBoMjAwdjEwMGgtMjAwdjEwMGgzMDB2LTMwMHoiIC8%2BCjxnbHlwaCB1bmljb2RlPSImI3hlMjU3OyIgZD0iTTcwMSAxMDk4aDE2MHExNiAwIDIxIC0xMXQtNyAtMjNsLTQ2NCAtNDY0bDQ2NCAtNDY0cTEyIC0xMiA3IC0yM3QtMjEgLTExaC0xNjBxLTEzIDAgLTIzIDlsLTQ3MSA0NzFxLTcgOCAtNyAxOHQ3IDE4bDQ3MSA0NzFxMTAgOSAyMyA5eiIgLz4KPGdseXBoIHVuaWNvZGU9IiYjeGUyNTg7IiBkPSJNMzM5IDEwOThoMTYwcTEzIDAgMjMgLTlsNDcxIC00NzFxNyAtOCA3IC0xOHQtNyAtMThsLTQ3MSAtNDcxcS0xMCAtOSAtMjMgLTloLTE2MHEtMTYgMCAtMjEgMTF0NyAyM2w0NjQgNDY0bC00NjQgNDY0cS0xMiAxMiAtNyAyM3QyMSAxMXoiIC8%2BCjxnbHlwaCB1bmljb2RlPSImI3hlMjU5OyIgZD0iTTEwODcgODgycTExIC01IDExIC0yMXYtMTYwcTAgLTEzIC05IC0yM2wtNDcxIC00NzFxLTggLTcgLTE4IC03dC0xOCA3bC00NzEgNDcxcS05IDEwIC05IDIzdjE2MHEwIDE2IDExIDIxdDIzIC03bDQ2NCAtNDY0bDQ2NCA0NjRxMTIgMTIgMjMgN3oiIC8%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%2BCjwvZGVmcz48L3N2Zz4g%29%20format%28%27svg%27%29%7D%2Eglyphicon%7Bposition%3Arelative%3Btop%3A1px%3Bdisplay%3Ainline%2Dblock%3Bfont%2Dfamily%3A%27Glyphicons%20Halflings%27%3Bfont%2Dstyle%3Anormal%3Bfont%2Dweight%3A400%3Bline%2Dheight%3A1%3B%2Dwebkit%2Dfont%2Dsmoothing%3Aantialiased%3B%2Dmoz%2Dosx%2Dfont%2Dsmoothing%3Agrayscale%7D%2Eglyphicon%2Dasterisk%3Abefore%7Bcontent%3A%22%5C2a%22%7D%2Eglyphicon%2Dplus%3Abefore%7Bcontent%3A%22%5C2b%22%7D%2Eglyphicon%2Deur%3Abefore%2C%2Eglyphicon%2Deuro%3Abefore%7Bcontent%3A%22%5C20ac%22%7D%2Eglyphicon%2Dminus%3Abefore%7Bcontent%3A%22%5C2212%22%7D%2Eglyphicon%2Dcloud%3Abefore%7Bcontent%3A%22%5C2601%22%7D%2Eglyphicon%2Denvelope%3Abefore%7Bcontent%3A%22%5C2709%22%7D%2Eglyphicon%2Dpencil%3Abefore%7Bcontent%3A%22%5C270f%22%7D%2Eglyphicon%2Dglass%3Abefore%7Bcontent%3A%22%5Ce001%22%7D%2Eglyphicon%2Dmusic%3Abefore%7Bcontent%3A%22%5Ce002%22%7D%2Eglyphicon%2Dsearch%3Abefore%7Bcontent%3A%22%5Ce003%22%7D%2Eglyphicon%2Dheart%3Abefore%7Bcontent%3A%22%5Ce005%22%7D%2Eglyphicon%2Dstar%3Abefore%7Bcontent%3A%22%5Ce006%22%7D%2Eglyphicon%2Dstar%2Dempty%3Abefore%7Bcontent%3A%22%5Ce007%22%7D%2Eglyphicon%2Duser%3Abefore%7Bcontent%3A%22%5Ce008%22%7D%2Eglyphicon%2Dfilm%3Abefore%7Bcontent%3A%22%5Ce009%22%7D%2Eglyphicon%2Dth%2Dlarge%3Abefore%7Bcontent%3A%22%5Ce010%22%7D%2Eglyphicon%2Dth%3Abefore%7Bcontent%3A%22%5Ce011%22%7D%2Eglyphicon%2Dth%2Dlist%3Abefore%7Bcontent%3A%22%5Ce012%22%7D%2Eglyphicon%2Dok%3Abefore%7Bcontent%3A%22%5Ce013%22%7D%2Eglyphicon%2Dremove%3Abefore%7Bcontent%3A%22%5Ce014%22%7D%2Eglyphicon%2Dzoom%2Din%3Abefore%7Bcontent%3A%22%5Ce015%22%7D%2Eglyphicon%2Dzoom%2Dout%3Abefore%7Bcontent%3A%22%5Ce016%22%7D%2Eglyphicon%2Doff%3Abefore%7Bcontent%3A%22%5Ce017%22%7D%2Eglyphicon%2Dsignal%3Abefore%7Bcontent%3A%22%5Ce018%22%7D%2Eglyphicon%2Dcog%3Abefore%7Bcontent%3A%22%5Ce019%22%7D%2Eglyphicon%2Dtrash%3Abefore%7Bcontent%3A%22%5Ce020%22%7D%2Eglyphicon%2Dhome%3Abefore%7Bcontent%3A%22%5Ce021%22%7D%2Eglyphicon%2Dfile%3Abefore%7Bcontent%3A%22%5Ce022%22%7D%2Eglyphicon%2Dtime%3Abefore%7Bcontent%3A%22%5Ce023%22%7D%2Eglyphicon%2Droad%3Abefore%7Bcontent%3A%22%5Ce024%22%7D%2Eglyphicon%2Ddownload%2Dalt%3Abefore%7Bcontent%3A%22%5Ce025%22%7D%2Eglyphicon%2Ddownload%3Abefore%7Bcontent%3A%22%5Ce026%22%7D%2Eglyphicon%2Dupload%3Abefore%7Bcontent%3A%22%5Ce027%22%7D%2Eglyphicon%2Dinbox%3Abefore%7Bcontent%3A%22%5Ce028%22%7D%2Eglyphicon%2Dplay%2Dcircle%3Abefore%7Bcontent%3A%22%5Ce029%22%7D%2Eglyphicon%2Drepeat%3Abefore%7Bcontent%3A%22%5Ce030%22%7D%2Eglyphicon%2Drefresh%3Abefore%7Bcontent%3A%22%5Ce031%22%7D%2Eglyphicon%2Dlist%2Dalt%3Abefore%7Bcontent%3A%22%5Ce032%22%7D%2Eglyphicon%2Dlock%3Abefore%7Bcontent%3A%22%5Ce033%22%7D%2Eglyphicon%2Dflag%3Abefore%7Bcontent%3A%22%5Ce034%22%7D%2Eglyphicon%2Dheadphones%3Abefore%7Bcontent%3A%22%5Ce035%22%7D%2Eglyphicon%2Dvolume%2Doff%3Abefore%7Bcontent%3A%22%5Ce036%22%7D%2Eglyphicon%2Dvolume%2Ddown%3Abefore%7Bcontent%3A%22%5Ce037%22%7D%2Eglyphicon%2Dvolume%2Dup%3Abefore%7Bcontent%3A%22%5Ce038%22%7D%2Eglyphicon%2Dqrcode%3Abefore%7Bcontent%3A%22%5Ce039%22%7D%2Eglyphicon%2Dbarcode%3Abefore%7Bcontent%3A%22%5Ce040%22%7D%2Eglyphicon%2Dtag%3Abefore%7Bcontent%3A%22%5Ce041%22%7D%2Eglyphicon%2Dtags%3Abefore%7Bcontent%3A%22%5Ce042%22%7D%2Eglyphicon%2Dbook%3Abefore%7Bcontent%3A%22%5Ce043%22%7D%2Eglyphicon%2Dbookmark%3Abefore%7Bcontent%3A%22%5Ce044%22%7D%2Eglyphicon%2Dprint%3Abefore%7Bcontent%3A%22%5Ce045%22%7D%2Eglyphicon%2Dcamera%3Abefore%7Bcontent%3A%22%5Ce046%22%7D%2Eglyphicon%2Dfont%3Abefore%7Bcontent%3A%22%5Ce047%22%7D%2Eglyphicon%2Dbold%3Abefore%7Bcontent%3A%22%5Ce048%22%7D%2Eglyphicon%2Ditalic%3Abefore%7Bcontent%3A%22%5Ce049%22%7D%2Eglyphicon%2Dtext%2Dheight%3Abefore%7Bcontent%3A%22%5Ce050%22%7D%2Eglyphicon%2Dtext%2Dwidth%3Abefore%7Bcontent%3A%22%5Ce051%22%7D%2Eglyphicon%2Dalign%2Dleft%3Abefore%7Bcontent%3A%22%5Ce052%22%7D%2Eglyphicon%2Dalign%2Dcenter%3Abefore%7Bcontent%3A%22%5Ce053%22%7D%2Eglyphicon%2Dalign%2Dright%3Abefore%7Bcontent%3A%22%5Ce054%22%7D%2Eglyphicon%2Dalign%2Djustify%3Abefore%7Bcontent%3A%22%5Ce055%22%7D%2Eglyphicon%2Dlist%3Abefore%7Bcontent%3A%22%5Ce056%22%7D%2Eglyphicon%2Dindent%2Dleft%3Abefore%7Bcontent%3A%22%5Ce057%22%7D%2Eglyphicon%2Dindent%2Dright%3Abefore%7Bcontent%3A%22%5Ce058%22%7D%2Eglyphicon%2Dfacetime%2Dvideo%3Abefore%7Bcontent%3A%22%5Ce059%22%7D%2Eglyphicon%2Dpicture%3Abefore%7Bcontent%3A%22%5Ce060%22%7D%2Eglyphicon%2Dmap%2Dmarker%3Abefore%7Bcontent%3A%22%5Ce062%22%7D%2Eglyphicon%2Dadjust%3Abefore%7Bcontent%3A%22%5Ce063%22%7D%2Eglyphicon%2Dtint%3Abefore%7Bcontent%3A%22%5Ce064%22%7D%2Eglyphicon%2Dedit%3Abefore%7Bcontent%3A%22%5Ce065%22%7D%2Eglyphicon%2Dshare%3Abefore%7Bcontent%3A%22%5Ce066%22%7D%2Eglyphicon%2Dcheck%3Abefore%7Bcontent%3A%22%5Ce067%22%7D%2Eglyphicon%2Dmove%3Abefore%7Bcontent%3A%22%5Ce068%22%7D%2Eglyphicon%2Dstep%2Dbackward%3Abefore%7Bcontent%3A%22%5Ce069%22%7D%2Eglyphicon%2Dfast%2Dbackward%3Abefore%7Bcontent%3A%22%5Ce070%22%7D%2Eglyphicon%2Dbackward%3Abefore%7Bcontent%3A%22%5Ce071%22%7D%2Eglyphicon%2Dplay%3Abefore%7Bcontent%3A%22%5Ce072%22%7D%2Eglyphicon%2Dpause%3Abefore%7Bcontent%3A%22%5Ce073%22%7D%2Eglyphicon%2Dstop%3Abefore%7Bcontent%3A%22%5Ce074%22%7D%2Eglyphicon%2Dforward%3Abefore%7Bcontent%3A%22%5Ce075%22%7D%2Eglyphicon%2Dfast%2Dforward%3Abefore%7Bcontent%3A%22%5Ce076%22%7D%2Eglyphicon%2Dstep%2Dforward%3Abefore%7Bcontent%3A%22%5Ce077%22%7D%2Eglyphicon%2Deject%3Abefore%7Bcontent%3A%22%5Ce078%22%7D%2Eglyphicon%2Dchevron%2Dleft%3Abefore%7Bcontent%3A%22%5Ce079%22%7D%2Eglyphicon%2Dchevron%2Dright%3Abefore%7Bcontent%3A%22%5Ce080%22%7D%2Eglyphicon%2Dplus%2Dsign%3Abefore%7Bcontent%3A%22%5Ce081%22%7D%2Eglyphicon%2Dminus%2Dsign%3Abefore%7Bcontent%3A%22%5Ce082%22%7D%2Eglyphicon%2Dremove%2Dsign%3Abefore%7Bcontent%3A%22%5Ce083%22%7D%2Eglyphicon%2Dok%2Dsign%3Abefore%7Bcontent%3A%22%5Ce084%22%7D%2Eglyphicon%2Dquestion%2Dsign%3Abefore%7Bcontent%3A%22%5Ce085%22%7D%2Eglyphicon%2Dinfo%2Dsign%3Abefore%7Bcontent%3A%22%5Ce086%22%7D%2Eglyphicon%2Dscreenshot%3Abefore%7Bcontent%3A%22%5Ce087%22%7D%2Eglyphicon%2Dremove%2Dcircle%3Abefore%7Bcontent%3A%22%5Ce088%22%7D%2Eglyphicon%2Dok%2Dcircle%3Abefore%7Bcontent%3A%22%5Ce089%22%7D%2Eglyphicon%2Dban%2Dcircle%3Abefore%7Bcontent%3A%22%5Ce090%22%7D%2Eglyphicon%2Darrow%2Dleft%3Abefore%7Bcontent%3A%22%5Ce091%22%7D%2Eglyphicon%2Darrow%2Dright%3Abefore%7Bcontent%3A%22%5Ce092%22%7D%2Eglyphicon%2Darrow%2Dup%3Abefore%7Bcontent%3A%22%5Ce093%22%7D%2Eglyphicon%2Darrow%2Ddown%3Abefore%7Bcontent%3A%22%5Ce094%22%7D%2Eglyphicon%2Dshare%2Dalt%3Abefore%7Bcontent%3A%22%5Ce095%22%7D%2Eglyphicon%2Dresize%2Dfull%3Abefore%7Bcontent%3A%22%5Ce096%22%7D%2Eglyphicon%2Dresize%2Dsmall%3Abefore%7Bcontent%3A%22%5Ce097%22%7D%2Eglyphicon%2Dexclamation%2Dsign%3Abefore%7Bcontent%3A%22%5Ce101%22%7D%2Eglyphicon%2Dgift%3Abefore%7Bcontent%3A%22%5Ce102%22%7D%2Eglyphicon%2Dleaf%3Abefore%7Bcontent%3A%22%5Ce103%22%7D%2Eglyphicon%2Dfire%3Abefore%7Bcontent%3A%22%5Ce104%22%7D%2Eglyphicon%2Deye%2Dopen%3Abefore%7Bcontent%3A%22%5Ce105%22%7D%2Eglyphicon%2Deye%2Dclose%3Abefore%7Bcontent%3A%22%5Ce106%22%7D%2Eglyphicon%2Dwarning%2Dsign%3Abefore%7Bcontent%3A%22%5Ce107%22%7D%2Eglyphicon%2Dplane%3Abefore%7Bcontent%3A%22%5Ce108%22%7D%2Eglyphicon%2Dcalendar%3Abefore%7Bcontent%3A%22%5Ce109%22%7D%2Eglyphicon%2Drandom%3Abefore%7Bcontent%3A%22%5Ce110%22%7D%2Eglyphicon%2Dcomment%3Abefore%7Bcontent%3A%22%5Ce111%22%7D%2Eglyphicon%2Dmagnet%3Abefore%7Bcontent%3A%22%5Ce112%22%7D%2Eglyphicon%2Dchevron%2Dup%3Abefore%7Bcontent%3A%22%5Ce113%22%7D%2Eglyphicon%2Dchevron%2Ddown%3Abefore%7Bcontent%3A%22%5Ce114%22%7D%2Eglyphicon%2Dretweet%3Abefore%7Bcontent%3A%22%5Ce115%22%7D%2Eglyphicon%2Dshopping%2Dcart%3Abefore%7Bcontent%3A%22%5Ce116%22%7D%2Eglyphicon%2Dfolder%2Dclose%3Abefore%7Bcontent%3A%22%5Ce117%22%7D%2Eglyphicon%2Dfolder%2Dopen%3Abefore%7Bcontent%3A%22%5Ce118%22%7D%2Eglyphicon%2Dresize%2Dvertical%3Abefore%7Bcontent%3A%22%5Ce119%22%7D%2Eglyphicon%2Dresize%2Dhorizontal%3Abefore%7Bcontent%3A%22%5Ce120%22%7D%2Eglyphicon%2Dhdd%3Abefore%7Bcontent%3A%22%5Ce121%22%7D%2Eglyphicon%2Dbullhorn%3Abefore%7Bcontent%3A%22%5Ce122%22%7D%2Eglyphicon%2Dbell%3Abefore%7Bcontent%3A%22%5Ce123%22%7D%2Eglyphicon%2Dcertificate%3Abefore%7Bcontent%3A%22%5Ce124%22%7D%2Eglyphicon%2Dthumbs%2Dup%3Abefore%7Bcontent%3A%22%5Ce125%22%7D%2Eglyphicon%2Dthumbs%2Ddown%3Abefore%7Bcontent%3A%22%5Ce126%22%7D%2Eglyphicon%2Dhand%2Dright%3Abefore%7Bcontent%3A%22%5Ce127%22%7D%2Eglyphicon%2Dhand%2Dleft%3Abefore%7Bcontent%3A%22%5Ce128%22%7D%2Eglyphicon%2Dhand%2Dup%3Abefore%7Bcontent%3A%22%5Ce129%22%7D%2Eglyphicon%2Dhand%2Ddown%3Abefore%7Bcontent%3A%22%5Ce130%22%7D%2Eglyphicon%2Dcircle%2Darrow%2Dright%3Abefore%7Bcontent%3A%22%5Ce131%22%7D%2Eglyphicon%2Dcircle%2Darrow%2Dleft%3Abefore%7Bcontent%3A%22%5Ce132%22%7D%2Eglyphicon%2Dcircle%2Darrow%2Dup%3Abefore%7Bcontent%3A%22%5Ce133%22%7D%2Eglyphicon%2Dcircle%2Darrow%2Ddown%3Abefore%7Bcontent%3A%22%5Ce134%22%7D%2Eglyphicon%2Dglobe%3Abefore%7Bcontent%3A%22%5Ce135%22%7D%2Eglyphicon%2Dwrench%3Abefore%7Bcontent%3A%22%5Ce136%22%7D%2Eglyphicon%2Dtasks%3Abefore%7Bcontent%3A%22%5Ce137%22%7D%2Eglyphicon%2Dfilter%3Abefore%7Bcontent%3A%22%5Ce138%22%7D%2Eglyphicon%2Dbriefcase%3Abefore%7Bcontent%3A%22%5Ce139%22%7D%2Eglyphicon%2Dfullscreen%3Abefore%7Bcontent%3A%22%5Ce140%22%7D%2Eglyphicon%2Ddashboard%3Abefore%7Bcontent%3A%22%5Ce141%22%7D%2Eglyphicon%2Dpaperclip%3Abefore%7Bcontent%3A%22%5Ce142%22%7D%2Eglyphicon%2Dheart%2Dempty%3Abefore%7Bcontent%3A%22%5Ce143%22%7D%2Eglyphicon%2Dlink%3Abefore%7Bcontent%3A%22%5Ce144%22%7D%2Eglyphicon%2Dphone%3Abefore%7Bcontent%3A%22%5Ce145%22%7D%2Eglyphicon%2Dpushpin%3Abefore%7Bcontent%3A%22%5Ce146%22%7D%2Eglyphicon%2Dusd%3Abefore%7Bcontent%3A%22%5Ce148%22%7D%2Eglyphicon%2Dgbp%3Abefore%7Bcontent%3A%22%5Ce149%22%7D%2Eglyphicon%2Dsort%3Abefore%7Bcontent%3A%22%5Ce150%22%7D%2Eglyphicon%2Dsort%2Dby%2Dalphabet%3Abefore%7Bcontent%3A%22%5Ce151%22%7D%2Eglyphicon%2Dsort%2Dby%2Dalphabet%2Dalt%3Abefore%7Bcontent%3A%22%5Ce152%22%7D%2Eglyphicon%2Dsort%2Dby%2Dorder%3Abefore%7Bcontent%3A%22%5Ce153%22%7D%2Eglyphicon%2Dsort%2Dby%2Dorder%2Dalt%3Abefore%7Bcontent%3A%22%5Ce154%22%7D%2Eglyphicon%2Dsort%2Dby%2Dattributes%3Abefore%7Bcontent%3A%22%5Ce155%22%7D%2Eglyphicon%2Dsort%2Dby%2Dattributes%2Dalt%3Abefore%7Bcontent%3A%22%5Ce156%22%7D%2Eglyphicon%2Dunchecked%3Abefore%7Bcontent%3A%22%5Ce157%22%7D%2Eglyphicon%2Dexpand%3Abefore%7Bcontent%3A%22%5Ce158%22%7D%2Eglyphicon%2Dcollapse%2Ddown%3Abefore%7Bcontent%3A%22%5Ce159%22%7D%2Eglyphicon%2Dcollapse%2Dup%3Abefore%7Bcontent%3A%22%5Ce160%22%7D%2Eglyphicon%2Dlog%2Din%3Abefore%7Bcontent%3A%22%5Ce161%22%7D%2Eglyphicon%2Dflash%3Abefore%7Bcontent%3A%22%5Ce162%22%7D%2Eglyphicon%2Dlog%2Dout%3Abefore%7Bcontent%3A%22%5Ce163%22%7D%2Eglyphicon%2Dnew%2Dwindow%3Abefore%7Bcontent%3A%22%5Ce164%22%7D%2Eglyphicon%2Drecord%3Abefore%7Bcontent%3A%22%5Ce165%22%7D%2Eglyphicon%2Dsave%3Abefore%7Bcontent%3A%22%5Ce166%22%7D%2Eglyphicon%2Dopen%3Abefore%7Bcontent%3A%22%5Ce167%22%7D%2Eglyphicon%2Dsaved%3Abefore%7Bcontent%3A%22%5Ce168%22%7D%2Eglyphicon%2Dimport%3Abefore%7Bcontent%3A%22%5Ce169%22%7D%2Eglyphicon%2Dexport%3Abefore%7Bcontent%3A%22%5Ce170%22%7D%2Eglyphicon%2Dsend%3Abefore%7Bcontent%3A%22%5Ce171%22%7D%2Eglyphicon%2Dfloppy%2Ddisk%3Abefore%7Bcontent%3A%22%5Ce172%22%7D%2Eglyphicon%2Dfloppy%2Dsaved%3Abefore%7Bcontent%3A%22%5Ce173%22%7D%2Eglyphicon%2Dfloppy%2Dremove%3Abefore%7Bcontent%3A%22%5Ce174%22%7D%2Eglyphicon%2Dfloppy%2Dsave%3Abefore%7Bcontent%3A%22%5Ce175%22%7D%2Eglyphicon%2Dfloppy%2Dopen%3Abefore%7Bcontent%3A%22%5Ce176%22%7D%2Eglyphicon%2Dcredit%2Dcard%3Abefore%7Bcontent%3A%22%5Ce177%22%7D%2Eglyphicon%2Dtransfer%3Abefore%7Bcontent%3A%22%5Ce178%22%7D%2Eglyphicon%2Dcutlery%3Abefore%7Bcontent%3A%22%5Ce179%22%7D%2Eglyphicon%2Dheader%3Abefore%7Bcontent%3A%22%5Ce180%22%7D%2Eglyphicon%2Dcompressed%3Abefore%7Bcontent%3A%22%5Ce181%22%7D%2Eglyphicon%2Dearphone%3Abefore%7Bcontent%3A%22%5Ce182%22%7D%2Eglyphicon%2Dphone%2Dalt%3Abefore%7Bcontent%3A%22%5Ce183%22%7D%2Eglyphicon%2Dtower%3Abefore%7Bcontent%3A%22%5Ce184%22%7D%2Eglyphicon%2Dstats%3Abefore%7Bcontent%3A%22%5Ce185%22%7D%2Eglyphicon%2Dsd%2Dvideo%3Abefore%7Bcontent%3A%22%5Ce186%22%7D%2Eglyphicon%2Dhd%2Dvideo%3Abefore%7Bcontent%3A%22%5Ce187%22%7D%2Eglyphicon%2Dsubtitles%3Abefore%7Bcontent%3A%22%5Ce188%22%7D%2Eglyphicon%2Dsound%2Dstereo%3Abefore%7Bcontent%3A%22%5Ce189%22%7D%2Eglyphicon%2Dsound%2Ddolby%3Abefore%7Bcontent%3A%22%5Ce190%22%7D%2Eglyphicon%2Dsound%2D5%2D1%3Abefore%7Bcontent%3A%22%5Ce191%22%7D%2Eglyphicon%2Dsound%2D6%2D1%3Abefore%7Bcontent%3A%22%5Ce192%22%7D%2Eglyphicon%2Dsound%2D7%2D1%3Abefore%7Bcontent%3A%22%5Ce193%22%7D%2Eglyphicon%2Dcopyright%2Dmark%3Abefore%7Bcontent%3A%22%5Ce194%22%7D%2Eglyphicon%2Dregistration%2Dmark%3Abefore%7Bcontent%3A%22%5Ce195%22%7D%2Eglyphicon%2Dcloud%2Ddownload%3Abefore%7Bcontent%3A%22%5Ce197%22%7D%2Eglyphicon%2Dcloud%2Dupload%3Abefore%7Bcontent%3A%22%5Ce198%22%7D%2Eglyphicon%2Dtree%2Dconifer%3Abefore%7Bcontent%3A%22%5Ce199%22%7D%2Eglyphicon%2Dtree%2Ddeciduous%3Abefore%7Bcontent%3A%22%5Ce200%22%7D%2Eglyphicon%2Dcd%3Abefore%7Bcontent%3A%22%5Ce201%22%7D%2Eglyphicon%2Dsave%2Dfile%3Abefore%7Bcontent%3A%22%5Ce202%22%7D%2Eglyphicon%2Dopen%2Dfile%3Abefore%7Bcontent%3A%22%5Ce203%22%7D%2Eglyphicon%2Dlevel%2Dup%3Abefore%7Bcontent%3A%22%5Ce204%22%7D%2Eglyphicon%2Dcopy%3Abefore%7Bcontent%3A%22%5Ce205%22%7D%2Eglyphicon%2Dpaste%3Abefore%7Bcontent%3A%22%5Ce206%22%7D%2Eglyphicon%2Dalert%3Abefore%7Bcontent%3A%22%5Ce209%22%7D%2Eglyphicon%2Dequalizer%3Abefore%7Bcontent%3A%22%5Ce210%22%7D%2Eglyphicon%2Dking%3Abefore%7Bcontent%3A%22%5Ce211%22%7D%2Eglyphicon%2Dqueen%3Abefore%7Bcontent%3A%22%5Ce212%22%7D%2Eglyphicon%2Dpawn%3Abefore%7Bcontent%3A%22%5Ce213%22%7D%2Eglyphicon%2Dbishop%3Abefore%7Bcontent%3A%22%5Ce214%22%7D%2Eglyphicon%2Dknight%3Abefore%7Bcontent%3A%22%5Ce215%22%7D%2Eglyphicon%2Dbaby%2Dformula%3Abefore%7Bcontent%3A%22%5Ce216%22%7D%2Eglyphicon%2Dtent%3Abefore%7Bcontent%3A%22%5C26fa%22%7D%2Eglyphicon%2Dblackboard%3Abefore%7Bcontent%3A%22%5Ce218%22%7D%2Eglyphicon%2Dbed%3Abefore%7Bcontent%3A%22%5Ce219%22%7D%2Eglyphicon%2Dapple%3Abefore%7Bcontent%3A%22%5Cf8ff%22%7D%2Eglyphicon%2Derase%3Abefore%7Bcontent%3A%22%5Ce221%22%7D%2Eglyphicon%2Dhourglass%3Abefore%7Bcontent%3A%22%5C231b%22%7D%2Eglyphicon%2Dlamp%3Abefore%7Bcontent%3A%22%5Ce223%22%7D%2Eglyphicon%2Dduplicate%3Abefore%7Bcontent%3A%22%5Ce224%22%7D%2Eglyphicon%2Dpiggy%2Dbank%3Abefore%7Bcontent%3A%22%5Ce225%22%7D%2Eglyphicon%2Dscissors%3Abefore%7Bcontent%3A%22%5Ce226%22%7D%2Eglyphicon%2Dbitcoin%3Abefore%7Bcontent%3A%22%5Ce227%22%7D%2Eglyphicon%2Dbtc%3Abefore%7Bcontent%3A%22%5Ce227%22%7D%2Eglyphicon%2Dxbt%3Abefore%7Bcontent%3A%22%5Ce227%22%7D%2Eglyphicon%2Dyen%3Abefore%7Bcontent%3A%22%5C00a5%22%7D%2Eglyphicon%2Djpy%3Abefore%7Bcontent%3A%22%5C00a5%22%7D%2Eglyphicon%2Druble%3Abefore%7Bcontent%3A%22%5C20bd%22%7D%2Eglyphicon%2Drub%3Abefore%7Bcontent%3A%22%5C20bd%22%7D%2Eglyphicon%2Dscale%3Abefore%7Bcontent%3A%22%5Ce230%22%7D%2Eglyphicon%2Dice%2Dlolly%3Abefore%7Bcontent%3A%22%5Ce231%22%7D%2Eglyphicon%2Dice%2Dlolly%2Dtasted%3Abefore%7Bcontent%3A%22%5Ce232%22%7D%2Eglyphicon%2Deducation%3Abefore%7Bcontent%3A%22%5Ce233%22%7D%2Eglyphicon%2Doption%2Dhorizontal%3Abefore%7Bcontent%3A%22%5Ce234%22%7D%2Eglyphicon%2Doption%2Dvertical%3Abefore%7Bcontent%3A%22%5Ce235%22%7D%2Eglyphicon%2Dmenu%2Dhamburger%3Abefore%7Bcontent%3A%22%5Ce236%22%7D%2Eglyphicon%2Dmodal%2Dwindow%3Abefore%7Bcontent%3A%22%5Ce237%22%7D%2Eglyphicon%2Doil%3Abefore%7Bcontent%3A%22%5Ce238%22%7D%2Eglyphicon%2Dgrain%3Abefore%7Bcontent%3A%22%5Ce239%22%7D%2Eglyphicon%2Dsunglasses%3Abefore%7Bcontent%3A%22%5Ce240%22%7D%2Eglyphicon%2Dtext%2Dsize%3Abefore%7Bcontent%3A%22%5Ce241%22%7D%2Eglyphicon%2Dtext%2Dcolor%3Abefore%7Bcontent%3A%22%5Ce242%22%7D%2Eglyphicon%2Dtext%2Dbackground%3Abefore%7Bcontent%3A%22%5Ce243%22%7D%2Eglyphicon%2Dobject%2Dalign%2Dtop%3Abefore%7Bcontent%3A%22%5Ce244%22%7D%2Eglyphicon%2Dobject%2Dalign%2Dbottom%3Abefore%7Bcontent%3A%22%5Ce245%22%7D%2Eglyphicon%2Dobject%2Dalign%2Dhorizontal%3Abefore%7Bcontent%3A%22%5Ce246%22%7D%2Eglyphicon%2Dobject%2Dalign%2Dleft%3Abefore%7Bcontent%3A%22%5Ce247%22%7D%2Eglyphicon%2Dobject%2Dalign%2Dvertical%3Abefore%7Bcontent%3A%22%5Ce248%22%7D%2Eglyphicon%2Dobject%2Dalign%2Dright%3Abefore%7Bcontent%3A%22%5Ce249%22%7D%2Eglyphicon%2Dtriangle%2Dright%3Abefore%7Bcontent%3A%22%5Ce250%22%7D%2Eglyphicon%2Dtriangle%2Dleft%3Abefore%7Bcontent%3A%22%5Ce251%22%7D%2Eglyphicon%2Dtriangle%2Dbottom%3Abefore%7Bcontent%3A%22%5Ce252%22%7D%2Eglyphicon%2Dtriangle%2Dtop%3Abefore%7Bcontent%3A%22%5Ce253%22%7D%2Eglyphicon%2Dconsole%3Abefore%7Bcontent%3A%22%5Ce254%22%7D%2Eglyphicon%2Dsuperscript%3Abefore%7Bcontent%3A%22%5Ce255%22%7D%2Eglyphicon%2Dsubscript%3Abefore%7Bcontent%3A%22%5Ce256%22%7D%2Eglyphicon%2Dmenu%2Dleft%3Abefore%7Bcontent%3A%22%5Ce257%22%7D%2Eglyphicon%2Dmenu%2Dright%3Abefore%7Bcontent%3A%22%5Ce258%22%7D%2Eglyphicon%2Dmenu%2Ddown%3Abefore%7Bcontent%3A%22%5Ce259%22%7D%2Eglyphicon%2Dmenu%2Dup%3Abefore%7Bcontent%3A%22%5Ce260%22%7D%2A%7B%2Dwebkit%2Dbox%2Dsizing%3Aborder%2Dbox%3B%2Dmoz%2Dbox%2Dsizing%3Aborder%2Dbox%3Bbox%2Dsizing%3Aborder%2Dbox%7D%3Aafter%2C%3Abefore%7B%2Dwebkit%2Dbox%2Dsizing%3Aborder%2Dbox%3B%2Dmoz%2Dbox%2Dsizing%3Aborder%2Dbox%3Bbox%2Dsizing%3Aborder%2Dbox%7Dhtml%7Bfont%2Dsize%3A10px%3B%2Dwebkit%2Dtap%2Dhighlight%2Dcolor%3Argba%280%2C0%2C0%2C0%29%7Dbody%7Bfont%2Dfamily%3A%22Helvetica%20Neue%22%2CHelvetica%2CArial%2Csans%2Dserif%3Bfont%2Dsize%3A14px%3Bline%2Dheight%3A1%2E42857143%3Bcolor%3A%23333%3Bbackground%2Dcolor%3A%23fff%7Dbutton%2Cinput%2Cselect%2Ctextarea%7Bfont%2Dfamily%3Ainherit%3Bfont%2Dsize%3Ainherit%3Bline%2Dheight%3Ainherit%7Da%7Bcolor%3A%23337ab7%3Btext%2Ddecoration%3Anone%7Da%3Afocus%2Ca%3Ahover%7Bcolor%3A%2323527c%3Btext%2Ddecoration%3Aunderline%7Da%3Afocus%7Boutline%3Athin%20dotted%3Boutline%3A5px%20auto%20%2Dwebkit%2Dfocus%2Dring%2Dcolor%3Boutline%2Doffset%3A%2D2px%7Dfigure%7Bmargin%3A0%7Dimg%7Bvertical%2Dalign%3Amiddle%7D%2Ecarousel%2Dinner%3E%2Eitem%3Ea%3Eimg%2C%2Ecarousel%2Dinner%3E%2Eitem%3Eimg%2C%2Eimg%2Dresponsive%2C%2Ethumbnail%20a%3Eimg%2C%2Ethumbnail%3Eimg%7Bdisplay%3Ablock%3Bmax%2Dwidth%3A100%25%3Bheight%3Aauto%7D%2Eimg%2Drounded%7Bborder%2Dradius%3A6px%7D%2Eimg%2Dthumbnail%7Bdisplay%3Ainline%2Dblock%3Bmax%2Dwidth%3A100%25%3Bheight%3Aauto%3Bpadding%3A4px%3Bline%2Dheight%3A1%2E42857143%3Bbackground%2Dcolor%3A%23fff%3Bborder%3A1px%20solid%20%23ddd%3Bborder%2Dradius%3A4px%3B%2Dwebkit%2Dtransition%3Aall%20%2E2s%20ease%2Din%2Dout%3B%2Do%2Dtransition%3Aall%20%2E2s%20ease%2Din%2Dout%3Btransition%3Aall%20%2E2s%20ease%2Din%2Dout%7D%2Eimg%2Dcircle%7Bborder%2Dradius%3A50%25%7Dhr%7Bmargin%2Dtop%3A20px%3Bmargin%2Dbottom%3A20px%3Bborder%3A0%3Bborder%2Dtop%3A1px%20solid%20%23eee%7D%2Esr%2Donly%7Bposition%3Aabsolute%3Bwidth%3A1px%3Bheight%3A1px%3Bpadding%3A0%3Bmargin%3A%2D1px%3Boverflow%3Ahidden%3Bclip%3Arect%280%2C0%2C0%2C0%29%3Bborder%3A0%7D%2Esr%2Donly%2Dfocusable%3Aactive%2C%2Esr%2Donly%2Dfocusable%3Afocus%7Bposition%3Astatic%3Bwidth%3Aauto%3Bheight%3Aauto%3Bmargin%3A0%3Boverflow%3Avisible%3Bclip%3Aauto%7D%5Brole%3Dbutton%5D%7Bcursor%3Apointer%7D%2Eh1%2C%2Eh2%2C%2Eh3%2C%2Eh4%2C%2Eh5%2C%2Eh6%2Ch1%2Ch2%2Ch3%2Ch4%2Ch5%2Ch6%7Bfont%2Dfamily%3Ainherit%3Bfont%2Dweight%3A500%3Bline%2Dheight%3A1%2E1%3Bcolor%3Ainherit%7D%2Eh1%20%2Esmall%2C%2Eh1%20small%2C%2Eh2%20%2Esmall%2C%2Eh2%20small%2C%2Eh3%20%2Esmall%2C%2Eh3%20small%2C%2Eh4%20%2Esmall%2C%2Eh4%20small%2C%2Eh5%20%2Esmall%2C%2Eh5%20small%2C%2Eh6%20%2Esmall%2C%2Eh6%20small%2Ch1%20%2Esmall%2Ch1%20small%2Ch2%20%2Esmall%2Ch2%20small%2Ch3%20%2Esmall%2Ch3%20small%2Ch4%20%2Esmall%2Ch4%20small%2Ch5%20%2Esmall%2Ch5%20small%2Ch6%20%2Esmall%2Ch6%20small%7Bfont%2Dweight%3A400%3Bline%2Dheight%3A1%3Bcolor%3A%23777%7D%2Eh1%2C%2Eh2%2C%2Eh3%2Ch1%2Ch2%2Ch3%7Bmargin%2Dtop%3A20px%3Bmargin%2Dbottom%3A10px%7D%2Eh1%20%2Esmall%2C%2Eh1%20small%2C%2Eh2%20%2Esmall%2C%2Eh2%20small%2C%2Eh3%20%2Esmall%2C%2Eh3%20small%2Ch1%20%2Esmall%2Ch1%20small%2Ch2%20%2Esmall%2Ch2%20small%2Ch3%20%2Esmall%2Ch3%20small%7Bfont%2Dsize%3A65%25%7D%2Eh4%2C%2Eh5%2C%2Eh6%2Ch4%2Ch5%2Ch6%7Bmargin%2Dtop%3A10px%3Bmargin%2Dbottom%3A10px%7D%2Eh4%20%2Esmall%2C%2Eh4%20small%2C%2Eh5%20%2Esmall%2C%2Eh5%20small%2C%2Eh6%20%2Esmall%2C%2Eh6%20small%2Ch4%20%2Esmall%2Ch4%20small%2Ch5%20%2Esmall%2Ch5%20small%2Ch6%20%2Esmall%2Ch6%20small%7Bfont%2Dsize%3A75%25%7D%2Eh1%2Ch1%7Bfont%2Dsize%3A36px%7D%2Eh2%2Ch2%7Bfont%2Dsize%3A30px%7D%2Eh3%2Ch3%7Bfont%2Dsize%3A24px%7D%2Eh4%2Ch4%7Bfont%2Dsize%3A18px%7D%2Eh5%2Ch5%7Bfont%2Dsize%3A14px%7D%2Eh6%2Ch6%7Bfont%2Dsize%3A12px%7Dp%7Bmargin%3A0%200%2010px%7D%2Elead%7Bmargin%2Dbottom%3A20px%3Bfont%2Dsize%3A16px%3Bfont%2Dweight%3A300%3Bline%2Dheight%3A1%2E4%7D%40media%20%28min%2Dwidth%3A768px%29%7B%2Elead%7Bfont%2Dsize%3A21px%7D%7D%2Esmall%2Csmall%7Bfont%2Dsize%3A85%25%7D%2Emark%2Cmark%7Bpadding%3A%2E2em%3Bbackground%2Dcolor%3A%23fcf8e3%7D%2Etext%2Dleft%7Btext%2Dalign%3Aleft%7D%2Etext%2Dright%7Btext%2Dalign%3Aright%7D%2Etext%2Dcenter%7Btext%2Dalign%3Acenter%7D%2Etext%2Djustify%7Btext%2Dalign%3Ajustify%7D%2Etext%2Dnowrap%7Bwhite%2Dspace%3Anowrap%7D%2Etext%2Dlowercase%7Btext%2Dtransform%3Alowercase%7D%2Etext%2Duppercase%7Btext%2Dtransform%3Auppercase%7D%2Etext%2Dcapitalize%7Btext%2Dtransform%3Acapitalize%7D%2Etext%2Dmuted%7Bcolor%3A%23777%7D%2Etext%2Dprimary%7Bcolor%3A%23337ab7%7Da%2Etext%2Dprimary%3Afocus%2Ca%2Etext%2Dprimary%3Ahover%7Bcolor%3A%23286090%7D%2Etext%2Dsuccess%7Bcolor%3A%233c763d%7Da%2Etext%2Dsuccess%3Afocus%2Ca%2Etext%2Dsuccess%3Ahover%7Bcolor%3A%232b542c%7D%2Etext%2Dinfo%7Bcolor%3A%2331708f%7Da%2Etext%2Dinfo%3Afocus%2Ca%2Etext%2Dinfo%3Ahover%7Bcolor%3A%23245269%7D%2Etext%2Dwarning%7Bcolor%3A%238a6d3b%7Da%2Etext%2Dwarning%3Afocus%2Ca%2Etext%2Dwarning%3Ahover%7Bcolor%3A%2366512c%7D%2Etext%2Ddanger%7Bcolor%3A%23a94442%7Da%2Etext%2Ddanger%3Afocus%2Ca%2Etext%2Ddanger%3Ahover%7Bcolor%3A%23843534%7D%2Ebg%2Dprimary%7Bcolor%3A%23fff%3Bbackground%2Dcolor%3A%23337ab7%7Da%2Ebg%2Dprimary%3Afocus%2Ca%2Ebg%2Dprimary%3Ahover%7Bbackground%2Dcolor%3A%23286090%7D%2Ebg%2Dsuccess%7Bbackground%2Dcolor%3A%23dff0d8%7Da%2Ebg%2Dsuccess%3Afocus%2Ca%2Ebg%2Dsuccess%3Ahover%7Bbackground%2Dcolor%3A%23c1e2b3%7D%2Ebg%2Dinfo%7Bbackground%2Dcolor%3A%23d9edf7%7Da%2Ebg%2Dinfo%3Afocus%2Ca%2Ebg%2Dinfo%3Ahover%7Bbackground%2Dcolor%3A%23afd9ee%7D%2Ebg%2Dwarning%7Bbackground%2Dcolor%3A%23fcf8e3%7Da%2Ebg%2Dwarning%3Afocus%2Ca%2Ebg%2Dwarning%3Ahover%7Bbackground%2Dcolor%3A%23f7ecb5%7D%2Ebg%2Ddanger%7Bbackground%2Dcolor%3A%23f2dede%7Da%2Ebg%2Ddanger%3Afocus%2Ca%2Ebg%2Ddanger%3Ahover%7Bbackground%2Dcolor%3A%23e4b9b9%7D%2Epage%2Dheader%7Bpadding%2Dbottom%3A9px%3Bmargin%3A40px%200%2020px%3Bborder%2Dbottom%3A1px%20solid%20%23eee%7Dol%2Cul%7Bmargin%2Dtop%3A0%3Bmargin%2Dbottom%3A10px%7Dol%20ol%2Col%20ul%2Cul%20ol%2Cul%20ul%7Bmargin%2Dbottom%3A0%7D%2Elist%2Dunstyled%7Bpadding%2Dleft%3A0%3Blist%2Dstyle%3Anone%7D%2Elist%2Dinline%7Bpadding%2Dleft%3A0%3Bmargin%2Dleft%3A%2D5px%3Blist%2Dstyle%3Anone%7D%2Elist%2Dinline%3Eli%7Bdisplay%3Ainline%2Dblock%3Bpadding%2Dright%3A5px%3Bpadding%2Dleft%3A5px%7Ddl%7Bmargin%2Dtop%3A0%3Bmargin%2Dbottom%3A20px%7Ddd%2Cdt%7Bline%2Dheight%3A1%2E42857143%7Ddt%7Bfont%2Dweight%3A700%7Ddd%7Bmargin%2Dleft%3A0%7D%40media%20%28min%2Dwidth%3A768px%29%7B%2Edl%2Dhorizontal%20dt%7Bfloat%3Aleft%3Bwidth%3A160px%3Boverflow%3Ahidden%3Bclear%3Aleft%3Btext%2Dalign%3Aright%3Btext%2Doverflow%3Aellipsis%3Bwhite%2Dspace%3Anowrap%7D%2Edl%2Dhorizontal%20dd%7Bmargin%2Dleft%3A180px%7D%7Dabbr%5Bdata%2Doriginal%2Dtitle%5D%2Cabbr%5Btitle%5D%7Bcursor%3Ahelp%3Bborder%2Dbottom%3A1px%20dotted%20%23777%7D%2Einitialism%7Bfont%2Dsize%3A90%25%3Btext%2Dtransform%3Auppercase%7Dblockquote%7Bpadding%3A10px%2020px%3Bmargin%3A0%200%2020px%3Bfont%2Dsize%3A17%2E5px%3Bborder%2Dleft%3A5px%20solid%20%23eee%7Dblockquote%20ol%3Alast%2Dchild%2Cblockquote%20p%3Alast%2Dchild%2Cblockquote%20ul%3Alast%2Dchild%7Bmargin%2Dbottom%3A0%7Dblockquote%20%2Esmall%2Cblockquote%20footer%2Cblockquote%20small%7Bdisplay%3Ablock%3Bfont%2Dsize%3A80%25%3Bline%2Dheight%3A1%2E42857143%3Bcolor%3A%23777%7Dblockquote%20%2Esmall%3Abefore%2Cblockquote%20footer%3Abefore%2Cblockquote%20small%3Abefore%7Bcontent%3A%27%5C2014%20%5C00A0%27%7D%2Eblockquote%2Dreverse%2Cblockquote%2Epull%2Dright%7Bpadding%2Dright%3A15px%3Bpadding%2Dleft%3A0%3Btext%2Dalign%3Aright%3Bborder%2Dright%3A5px%20solid%20%23eee%3Bborder%2Dleft%3A0%7D%2Eblockquote%2Dreverse%20%2Esmall%3Abefore%2C%2Eblockquote%2Dreverse%20footer%3Abefore%2C%2Eblockquote%2Dreverse%20small%3Abefore%2Cblockquote%2Epull%2Dright%20%2Esmall%3Abefore%2Cblockquote%2Epull%2Dright%20footer%3Abefore%2Cblockquote%2Epull%2Dright%20small%3Abefore%7Bcontent%3A%27%27%7D%2Eblockquote%2Dreverse%20%2Esmall%3Aafter%2C%2Eblockquote%2Dreverse%20footer%3Aafter%2C%2Eblockquote%2Dreverse%20small%3Aafter%2Cblockquote%2Epull%2Dright%20%2Esmall%3Aafter%2Cblockquote%2Epull%2Dright%20footer%3Aafter%2Cblockquote%2Epull%2Dright%20small%3Aafter%7Bcontent%3A%27%5C00A0%20%5C2014%27%7Daddress%7Bmargin%2Dbottom%3A20px%3Bfont%2Dstyle%3Anormal%3Bline%2Dheight%3A1%2E42857143%7Dcode%2Ckbd%2Cpre%2Csamp%7Bfont%2Dfamily%3Amonospace%7Dcode%7Bpadding%3A2px%204px%3Bfont%2Dsize%3A90%25%3Bcolor%3A%23c7254e%3Bbackground%2Dcolor%3A%23f9f2f4%3Bborder%2Dradius%3A4px%7Dkbd%7Bpadding%3A2px%204px%3Bfont%2Dsize%3A90%25%3Bcolor%3A%23fff%3Bbackground%2Dcolor%3A%23333%3Bborder%2Dradius%3A3px%3B%2Dwebkit%2Dbox%2Dshadow%3Ainset%200%20%2D1px%200%20rgba%280%2C0%2C0%2C%2E25%29%3Bbox%2Dshadow%3Ainset%200%20%2D1px%200%20rgba%280%2C0%2C0%2C%2E25%29%7Dkbd%20kbd%7Bpadding%3A0%3Bfont%2Dsize%3A100%25%3Bfont%2Dweight%3A700%3B%2Dwebkit%2Dbox%2Dshadow%3Anone%3Bbox%2Dshadow%3Anone%7Dpre%7Bdisplay%3Ablock%3Bpadding%3A9%2E5px%3Bmargin%3A0%200%2010px%3Bfont%2Dsize%3A13px%3Bline%2Dheight%3A1%2E42857143%3Bcolor%3A%23333%3Bword%2Dbreak%3Abreak%2Dall%3Bword%2Dwrap%3Abreak%2Dword%3Bbackground%2Dcolor%3A%23f5f5f5%3Bborder%3A1px%20solid%20%23ccc%3Bborder%2Dradius%3A4px%7Dpre%20code%7Bpadding%3A0%3Bfont%2Dsize%3Ainherit%3Bcolor%3Ainherit%3Bwhite%2Dspace%3Apre%2Dwrap%3Bbackground%2Dcolor%3Atransparent%3Bborder%2Dradius%3A0%7D%2Epre%2Dscrollable%7Bmax%2Dheight%3A340px%3Boverflow%2Dy%3Ascroll%7D%2Econtainer%7Bpadding%2Dright%3A15px%3Bpadding%2Dleft%3A15px%3Bmargin%2Dright%3Aauto%3Bmargin%2Dleft%3Aauto%7D%40media%20%28min%2Dwidth%3A768px%29%7B%2Econtainer%7Bwidth%3A750px%7D%7D%40media%20%28min%2Dwidth%3A992px%29%7B%2Econtainer%7Bwidth%3A970px%7D%7D%40media%20%28min%2Dwidth%3A1200px%29%7B%2Econtainer%7Bwidth%3A1170px%7D%7D%2Econtainer%2Dfluid%7Bpadding%2Dright%3A15px%3Bpadding%2Dleft%3A15px%3Bmargin%2Dright%3Aauto%3Bmargin%2Dleft%3Aauto%7D%2Erow%7Bmargin%2Dright%3A%2D15px%3Bmargin%2Dleft%3A%2D15px%7D%2Ecol%2Dlg%2D1%2C%2Ecol%2Dlg%2D10%2C%2Ecol%2Dlg%2D11%2C%2Ecol%2Dlg%2D12%2C%2Ecol%2Dlg%2D2%2C%2Ecol%2Dlg%2D3%2C%2Ecol%2Dlg%2D4%2C%2Ecol%2Dlg%2D5%2C%2Ecol%2Dlg%2D6%2C%2Ecol%2Dlg%2D7%2C%2Ecol%2Dlg%2D8%2C%2Ecol%2Dlg%2D9%2C%2Ecol%2Dmd%2D1%2C%2Ecol%2Dmd%2D10%2C%2Ecol%2Dmd%2D11%2C%2Ecol%2Dmd%2D12%2C%2Ecol%2Dmd%2D2%2C%2Ecol%2Dmd%2D3%2C%2Ecol%2Dmd%2D4%2C%2Ecol%2Dmd%2D5%2C%2Ecol%2Dmd%2D6%2C%2Ecol%2Dmd%2D7%2C%2Ecol%2Dmd%2D8%2C%2Ecol%2Dmd%2D9%2C%2Ecol%2Dsm%2D1%2C%2Ecol%2Dsm%2D10%2C%2Ecol%2Dsm%2D11%2C%2Ecol%2Dsm%2D12%2C%2Ecol%2Dsm%2D2%2C%2Ecol%2Dsm%2D3%2C%2Ecol%2Dsm%2D4%2C%2Ecol%2Dsm%2D5%2C%2Ecol%2Dsm%2D6%2C%2Ecol%2Dsm%2D7%2C%2Ecol%2Dsm%2D8%2C%2Ecol%2Dsm%2D9%2C%2Ecol%2Dxs%2D1%2C%2Ecol%2Dxs%2D10%2C%2Ecol%2Dxs%2D11%2C%2Ecol%2Dxs%2D12%2C%2Ecol%2Dxs%2D2%2C%2Ecol%2Dxs%2D3%2C%2Ecol%2Dxs%2D4%2C%2Ecol%2Dxs%2D5%2C%2Ecol%2Dxs%2D6%2C%2Ecol%2Dxs%2D7%2C%2Ecol%2Dxs%2D8%2C%2Ecol%2Dxs%2D9%7Bposition%3Arelative%3Bmin%2Dheight%3A1px%3Bpadding%2Dright%3A15px%3Bpadding%2Dleft%3A15px%7D%2Ecol%2Dxs%2D1%2C%2Ecol%2Dxs%2D10%2C%2Ecol%2Dxs%2D11%2C%2Ecol%2Dxs%2D12%2C%2Ecol%2Dxs%2D2%2C%2Ecol%2Dxs%2D3%2C%2Ecol%2Dxs%2D4%2C%2Ecol%2Dxs%2D5%2C%2Ecol%2Dxs%2D6%2C%2Ecol%2Dxs%2D7%2C%2Ecol%2Dxs%2D8%2C%2Ecol%2Dxs%2D9%7Bfloat%3Aleft%7D%2Ecol%2Dxs%2D12%7Bwidth%3A100%25%7D%2Ecol%2Dxs%2D11%7Bwidth%3A91%2E66666667%25%7D%2Ecol%2Dxs%2D10%7Bwidth%3A83%2E33333333%25%7D%2Ecol%2Dxs%2D9%7Bwidth%3A75%25%7D%2Ecol%2Dxs%2D8%7Bwidth%3A66%2E66666667%25%7D%2Ecol%2Dxs%2D7%7Bwidth%3A58%2E33333333%25%7D%2Ecol%2Dxs%2D6%7Bwidth%3A50%25%7D%2Ecol%2Dxs%2D5%7Bwidth%3A41%2E66666667%25%7D%2Ecol%2Dxs%2D4%7Bwidth%3A33%2E33333333%25%7D%2Ecol%2Dxs%2D3%7Bwidth%3A25%25%7D%2Ecol%2Dxs%2D2%7Bwidth%3A16%2E66666667%25%7D%2Ecol%2Dxs%2D1%7Bwidth%3A8%2E33333333%25%7D%2Ecol%2Dxs%2Dpull%2D12%7Bright%3A100%25%7D%2Ecol%2Dxs%2Dpull%2D11%7Bright%3A91%2E66666667%25%7D%2Ecol%2Dxs%2Dpull%2D10%7Bright%3A83%2E33333333%25%7D%2Ecol%2Dxs%2Dpull%2D9%7Bright%3A75%25%7D%2Ecol%2Dxs%2Dpull%2D8%7Bright%3A66%2E66666667%25%7D%2Ecol%2Dxs%2Dpull%2D7%7Bright%3A58%2E33333333%25%7D%2Ecol%2Dxs%2Dpull%2D6%7Bright%3A50%25%7D%2Ecol%2Dxs%2Dpull%2D5%7Bright%3A41%2E66666667%25%7D%2Ecol%2Dxs%2Dpull%2D4%7Bright%3A33%2E33333333%25%7D%2Ecol%2Dxs%2Dpull%2D3%7Bright%3A25%25%7D%2Ecol%2Dxs%2Dpull%2D2%7Bright%3A16%2E66666667%25%7D%2Ecol%2Dxs%2Dpull%2D1%7Bright%3A8%2E33333333%25%7D%2Ecol%2Dxs%2Dpull%2D0%7Bright%3Aauto%7D%2Ecol%2Dxs%2Dpush%2D12%7Bleft%3A100%25%7D%2Ecol%2Dxs%2Dpush%2D11%7Bleft%3A91%2E66666667%25%7D%2Ecol%2Dxs%2Dpush%2D10%7Bleft%3A83%2E33333333%25%7D%2Ecol%2Dxs%2Dpush%2D9%7Bleft%3A75%25%7D%2Ecol%2Dxs%2Dpush%2D8%7Bleft%3A66%2E66666667%25%7D%2Ecol%2Dxs%2Dpush%2D7%7Bleft%3A58%2E33333333%25%7D%2Ecol%2Dxs%2Dpush%2D6%7Bleft%3A50%25%7D%2Ecol%2Dxs%2Dpush%2D5%7Bleft%3A41%2E66666667%25%7D%2Ecol%2Dxs%2Dpush%2D4%7Bleft%3A33%2E33333333%25%7D%2Ecol%2Dxs%2Dpush%2D3%7Bleft%3A25%25%7D%2Ecol%2Dxs%2Dpush%2D2%7Bleft%3A16%2E66666667%25%7D%2Ecol%2Dxs%2Dpush%2D1%7Bleft%3A8%2E33333333%25%7D%2Ecol%2Dxs%2Dpush%2D0%7Bleft%3Aauto%7D%2Ecol%2Dxs%2Doffset%2D12%7Bmargin%2Dleft%3A100%25%7D%2Ecol%2Dxs%2Doffset%2D11%7Bmargin%2Dleft%3A91%2E66666667%25%7D%2Ecol%2Dxs%2Doffset%2D10%7Bmargin%2Dleft%3A83%2E33333333%25%7D%2Ecol%2Dxs%2Doffset%2D9%7Bmargin%2Dleft%3A75%25%7D%2Ecol%2Dxs%2Doffset%2D8%7Bmargin%2Dleft%3A66%2E66666667%25%7D%2Ecol%2Dxs%2Doffset%2D7%7Bmargin%2Dleft%3A58%2E33333333%25%7D%2Ecol%2Dxs%2Doffset%2D6%7Bmargin%2Dleft%3A50%25%7D%2Ecol%2Dxs%2Doffset%2D5%7Bmargin%2Dleft%3A41%2E66666667%25%7D%2Ecol%2Dxs%2Doffset%2D4%7Bmargin%2Dleft%3A33%2E33333333%25%7D%2Ecol%2Dxs%2Doffset%2D3%7Bmargin%2Dleft%3A25%25%7D%2Ecol%2Dxs%2Doffset%2D2%7Bmargin%2Dleft%3A16%2E66666667%25%7D%2Ecol%2Dxs%2Doffset%2D1%7Bmargin%2Dleft%3A8%2E33333333%25%7D%2Ecol%2Dxs%2Doffset%2D0%7Bmargin%2Dleft%3A0%7D%40media%20%28min%2Dwidth%3A768px%29%7B%2Ecol%2Dsm%2D1%2C%2Ecol%2Dsm%2D10%2C%2Ecol%2Dsm%2D11%2C%2Ecol%2Dsm%2D12%2C%2Ecol%2Dsm%2D2%2C%2Ecol%2Dsm%2D3%2C%2Ecol%2Dsm%2D4%2C%2Ecol%2Dsm%2D5%2C%2Ecol%2Dsm%2D6%2C%2Ecol%2Dsm%2D7%2C%2Ecol%2Dsm%2D8%2C%2Ecol%2Dsm%2D9%7Bfloat%3Aleft%7D%2Ecol%2Dsm%2D12%7Bwidth%3A100%25%7D%2Ecol%2Dsm%2D11%7Bwidth%3A91%2E66666667%25%7D%2Ecol%2Dsm%2D10%7Bwidth%3A83%2E33333333%25%7D%2Ecol%2Dsm%2D9%7Bwidth%3A75%25%7D%2Ecol%2Dsm%2D8%7Bwidth%3A66%2E66666667%25%7D%2Ecol%2Dsm%2D7%7Bwidth%3A58%2E33333333%25%7D%2Ecol%2Dsm%2D6%7Bwidth%3A50%25%7D%2Ecol%2Dsm%2D5%7Bwidth%3A41%2E66666667%25%7D%2Ecol%2Dsm%2D4%7Bwidth%3A33%2E33333333%25%7D%2Ecol%2Dsm%2D3%7Bwidth%3A25%25%7D%2Ecol%2Dsm%2D2%7Bwidth%3A16%2E66666667%25%7D%2Ecol%2Dsm%2D1%7Bwidth%3A8%2E33333333%25%7D%2Ecol%2Dsm%2Dpull%2D12%7Bright%3A100%25%7D%2Ecol%2Dsm%2Dpull%2D11%7Bright%3A91%2E66666667%25%7D%2Ecol%2Dsm%2Dpull%2D10%7Bright%3A83%2E33333333%25%7D%2Ecol%2Dsm%2Dpull%2D9%7Bright%3A75%25%7D%2Ecol%2Dsm%2Dpull%2D8%7Bright%3A66%2E66666667%25%7D%2Ecol%2Dsm%2Dpull%2D7%7Bright%3A58%2E33333333%25%7D%2Ecol%2Dsm%2Dpull%2D6%7Bright%3A50%25%7D%2Ecol%2Dsm%2Dpull%2D5%7Bright%3A41%2E66666667%25%7D%2Ecol%2Dsm%2Dpull%2D4%7Bright%3A33%2E33333333%25%7D%2Ecol%2Dsm%2Dpull%2D3%7Bright%3A25%25%7D%2Ecol%2Dsm%2Dpull%2D2%7Bright%3A16%2E66666667%25%7D%2Ecol%2Dsm%2Dpull%2D1%7Bright%3A8%2E33333333%25%7D%2Ecol%2Dsm%2Dpull%2D0%7Bright%3Aauto%7D%2Ecol%2Dsm%2Dpush%2D12%7Bleft%3A100%25%7D%2Ecol%2Dsm%2Dpush%2D11%7Bleft%3A91%2E66666667%25%7D%2Ecol%2Dsm%2Dpush%2D10%7Bleft%3A83%2E33333333%25%7D%2Ecol%2Dsm%2Dpush%2D9%7Bleft%3A75%25%7D%2Ecol%2Dsm%2Dpush%2D8%7Bleft%3A66%2E66666667%25%7D%2Ecol%2Dsm%2Dpush%2D7%7Bleft%3A58%2E33333333%25%7D%2Ecol%2Dsm%2Dpush%2D6%7Bleft%3A50%25%7D%2Ecol%2Dsm%2Dpush%2D5%7Bleft%3A41%2E66666667%25%7D%2Ecol%2Dsm%2Dpush%2D4%7Bleft%3A33%2E33333333%25%7D%2Ecol%2Dsm%2Dpush%2D3%7Bleft%3A25%25%7D%2Ecol%2Dsm%2Dpush%2D2%7Bleft%3A16%2E66666667%25%7D%2Ecol%2Dsm%2Dpush%2D1%7Bleft%3A8%2E33333333%25%7D%2Ecol%2Dsm%2Dpush%2D0%7Bleft%3Aauto%7D%2Ecol%2Dsm%2Doffset%2D12%7Bmargin%2Dleft%3A100%25%7D%2Ecol%2Dsm%2Doffset%2D11%7Bmargin%2Dleft%3A91%2E66666667%25%7D%2Ecol%2Dsm%2Doffset%2D10%7Bmargin%2Dleft%3A83%2E33333333%25%7D%2Ecol%2Dsm%2Doffset%2D9%7Bmargin%2Dleft%3A75%25%7D%2Ecol%2Dsm%2Doffset%2D8%7Bmargin%2Dleft%3A66%2E66666667%25%7D%2Ecol%2Dsm%2Doffset%2D7%7Bmargin%2Dleft%3A58%2E33333333%25%7D%2Ecol%2Dsm%2Doffset%2D6%7Bmargin%2Dleft%3A50%25%7D%2Ecol%2Dsm%2Doffset%2D5%7Bmargin%2Dleft%3A41%2E66666667%25%7D%2Ecol%2Dsm%2Doffset%2D4%7Bmargin%2Dleft%3A33%2E33333333%25%7D%2Ecol%2Dsm%2Doffset%2D3%7Bmargin%2Dleft%3A25%25%7D%2Ecol%2Dsm%2Doffset%2D2%7Bmargin%2Dleft%3A16%2E66666667%25%7D%2Ecol%2Dsm%2Doffset%2D1%7Bmargin%2Dleft%3A8%2E33333333%25%7D%2Ecol%2Dsm%2Doffset%2D0%7Bmargin%2Dleft%3A0%7D%7D%40media%20%28min%2Dwidth%3A992px%29%7B%2Ecol%2Dmd%2D1%2C%2Ecol%2Dmd%2D10%2C%2Ecol%2Dmd%2D11%2C%2Ecol%2Dmd%2D12%2C%2Ecol%2Dmd%2D2%2C%2Ecol%2Dmd%2D3%2C%2Ecol%2Dmd%2D4%2C%2Ecol%2Dmd%2D5%2C%2Ecol%2Dmd%2D6%2C%2Ecol%2Dmd%2D7%2C%2Ecol%2Dmd%2D8%2C%2Ecol%2Dmd%2D9%7Bfloat%3Aleft%7D%2Ecol%2Dmd%2D12%7Bwidth%3A100%25%7D%2Ecol%2Dmd%2D11%7Bwidth%3A91%2E66666667%25%7D%2Ecol%2Dmd%2D10%7Bwidth%3A83%2E33333333%25%7D%2Ecol%2Dmd%2D9%7Bwidth%3A75%25%7D%2Ecol%2Dmd%2D8%7Bwidth%3A66%2E66666667%25%7D%2Ecol%2Dmd%2D7%7Bwidth%3A58%2E33333333%25%7D%2Ecol%2Dmd%2D6%7Bwidth%3A50%25%7D%2Ecol%2Dmd%2D5%7Bwidth%3A41%2E66666667%25%7D%2Ecol%2Dmd%2D4%7Bwidth%3A33%2E33333333%25%7D%2Ecol%2Dmd%2D3%7Bwidth%3A25%25%7D%2Ecol%2Dmd%2D2%7Bwidth%3A16%2E66666667%25%7D%2Ecol%2Dmd%2D1%7Bwidth%3A8%2E33333333%25%7D%2Ecol%2Dmd%2Dpull%2D12%7Bright%3A100%25%7D%2Ecol%2Dmd%2Dpull%2D11%7Bright%3A91%2E66666667%25%7D%2Ecol%2Dmd%2Dpull%2D10%7Bright%3A83%2E33333333%25%7D%2Ecol%2Dmd%2Dpull%2D9%7Bright%3A75%25%7D%2Ecol%2Dmd%2Dpull%2D8%7Bright%3A66%2E66666667%25%7D%2Ecol%2Dmd%2Dpull%2D7%7Bright%3A58%2E33333333%25%7D%2Ecol%2Dmd%2Dpull%2D6%7Bright%3A50%25%7D%2Ecol%2Dmd%2Dpull%2D5%7Bright%3A41%2E66666667%25%7D%2Ecol%2Dmd%2Dpull%2D4%7Bright%3A33%2E33333333%25%7D%2Ecol%2Dmd%2Dpull%2D3%7Bright%3A25%25%7D%2Ecol%2Dmd%2Dpull%2D2%7Bright%3A16%2E66666667%25%7D%2Ecol%2Dmd%2Dpull%2D1%7Bright%3A8%2E33333333%25%7D%2Ecol%2Dmd%2Dpull%2D0%7Bright%3Aauto%7D%2Ecol%2Dmd%2Dpush%2D12%7Bleft%3A100%25%7D%2Ecol%2Dmd%2Dpush%2D11%7Bleft%3A91%2E66666667%25%7D%2Ecol%2Dmd%2Dpush%2D10%7Bleft%3A83%2E33333333%25%7D%2Ecol%2Dmd%2Dpush%2D9%7Bleft%3A75%25%7D%2Ecol%2Dmd%2Dpush%2D8%7Bleft%3A66%2E66666667%25%7D%2Ecol%2Dmd%2Dpush%2D7%7Bleft%3A58%2E33333333%25%7D%2Ecol%2Dmd%2Dpush%2D6%7Bleft%3A50%25%7D%2Ecol%2Dmd%2Dpush%2D5%7Bleft%3A41%2E66666667%25%7D%2Ecol%2Dmd%2Dpush%2D4%7Bleft%3A33%2E33333333%25%7D%2Ecol%2Dmd%2Dpush%2D3%7Bleft%3A25%25%7D%2Ecol%2Dmd%2Dpush%2D2%7Bleft%3A16%2E66666667%25%7D%2Ecol%2Dmd%2Dpush%2D1%7Bleft%3A8%2E33333333%25%7D%2Ecol%2Dmd%2Dpush%2D0%7Bleft%3Aauto%7D%2Ecol%2Dmd%2Doffset%2D12%7Bmargin%2Dleft%3A100%25%7D%2Ecol%2Dmd%2Doffset%2D11%7Bmargin%2Dleft%3A91%2E66666667%25%7D%2Ecol%2Dmd%2Doffset%2D10%7Bmargin%2Dleft%3A83%2E33333333%25%7D%2Ecol%2Dmd%2Doffset%2D9%7Bmargin%2Dleft%3A75%25%7D%2Ecol%2Dmd%2Doffset%2D8%7Bmargin%2Dleft%3A66%2E66666667%25%7D%2Ecol%2Dmd%2Doffset%2D7%7Bmargin%2Dleft%3A58%2E33333333%25%7D%2Ecol%2Dmd%2Doffset%2D6%7Bmargin%2Dleft%3A50%25%7D%2Ecol%2Dmd%2Doffset%2D5%7Bmargin%2Dleft%3A41%2E66666667%25%7D%2Ecol%2Dmd%2Doffset%2D4%7Bmargin%2Dleft%3A33%2E33333333%25%7D%2Ecol%2Dmd%2Doffset%2D3%7Bmargin%2Dleft%3A25%25%7D%2Ecol%2Dmd%2Doffset%2D2%7Bmargin%2Dleft%3A16%2E66666667%25%7D%2Ecol%2Dmd%2Doffset%2D1%7Bmargin%2Dleft%3A8%2E33333333%25%7D%2Ecol%2Dmd%2Doffset%2D0%7Bmargin%2Dleft%3A0%7D%7D%40media%20%28min%2Dwidth%3A1200px%29%7B%2Ecol%2Dlg%2D1%2C%2Ecol%2Dlg%2D10%2C%2Ecol%2Dlg%2D11%2C%2Ecol%2Dlg%2D12%2C%2Ecol%2Dlg%2D2%2C%2Ecol%2Dlg%2D3%2C%2Ecol%2Dlg%2D4%2C%2Ecol%2Dlg%2D5%2C%2Ecol%2Dlg%2D6%2C%2Ecol%2Dlg%2D7%2C%2Ecol%2Dlg%2D8%2C%2Ecol%2Dlg%2D9%7Bfloat%3Aleft%7D%2Ecol%2Dlg%2D12%7Bwidth%3A100%25%7D%2Ecol%2Dlg%2D11%7Bwidth%3A91%2E66666667%25%7D%2Ecol%2Dlg%2D10%7Bwidth%3A83%2E33333333%25%7D%2Ecol%2Dlg%2D9%7Bwidth%3A75%25%7D%2Ecol%2Dlg%2D8%7Bwidth%3A66%2E66666667%25%7D%2Ecol%2Dlg%2D7%7Bwidth%3A58%2E33333333%25%7D%2Ecol%2Dlg%2D6%7Bwidth%3A50%25%7D%2Ecol%2Dlg%2D5%7Bwidth%3A41%2E66666667%25%7D%2Ecol%2Dlg%2D4%7Bwidth%3A33%2E33333333%25%7D%2Ecol%2Dlg%2D3%7Bwidth%3A25%25%7D%2Ecol%2Dlg%2D2%7Bwidth%3A16%2E66666667%25%7D%2Ecol%2Dlg%2D1%7Bwidth%3A8%2E33333333%25%7D%2Ecol%2Dlg%2Dpull%2D12%7Bright%3A100%25%7D%2Ecol%2Dlg%2Dpull%2D11%7Bright%3A91%2E66666667%25%7D%2Ecol%2Dlg%2Dpull%2D10%7Bright%3A83%2E33333333%25%7D%2Ecol%2Dlg%2Dpull%2D9%7Bright%3A75%25%7D%2Ecol%2Dlg%2Dpull%2D8%7Bright%3A66%2E66666667%25%7D%2Ecol%2Dlg%2Dpull%2D7%7Bright%3A58%2E33333333%25%7D%2Ecol%2Dlg%2Dpull%2D6%7Bright%3A50%25%7D%2Ecol%2Dlg%2Dpull%2D5%7Bright%3A41%2E66666667%25%7D%2Ecol%2Dlg%2Dpull%2D4%7Bright%3A33%2E33333333%25%7D%2Ecol%2Dlg%2Dpull%2D3%7Bright%3A25%25%7D%2Ecol%2Dlg%2Dpull%2D2%7Bright%3A16%2E66666667%25%7D%2Ecol%2Dlg%2Dpull%2D1%7Bright%3A8%2E33333333%25%7D%2Ecol%2Dlg%2Dpull%2D0%7Bright%3Aauto%7D%2Ecol%2Dlg%2Dpush%2D12%7Bleft%3A100%25%7D%2Ecol%2Dlg%2Dpush%2D11%7Bleft%3A91%2E66666667%25%7D%2Ecol%2Dlg%2Dpush%2D10%7Bleft%3A83%2E33333333%25%7D%2Ecol%2Dlg%2Dpush%2D9%7Bleft%3A75%25%7D%2Ecol%2Dlg%2Dpush%2D8%7Bleft%3A66%2E66666667%25%7D%2Ecol%2Dlg%2Dpush%2D7%7Bleft%3A58%2E33333333%25%7D%2Ecol%2Dlg%2Dpush%2D6%7Bleft%3A50%25%7D%2Ecol%2Dlg%2Dpush%2D5%7Bleft%3A41%2E66666667%25%7D%2Ecol%2Dlg%2Dpush%2D4%7Bleft%3A33%2E33333333%25%7D%2Ecol%2Dlg%2Dpush%2D3%7Bleft%3A25%25%7D%2Ecol%2Dlg%2Dpush%2D2%7Bleft%3A16%2E66666667%25%7D%2Ecol%2Dlg%2Dpush%2D1%7Bleft%3A8%2E33333333%25%7D%2Ecol%2Dlg%2Dpush%2D0%7Bleft%3Aauto%7D%2Ecol%2Dlg%2Doffset%2D12%7Bmargin%2Dleft%3A100%25%7D%2Ecol%2Dlg%2Doffset%2D11%7Bmargin%2Dleft%3A91%2E66666667%25%7D%2Ecol%2Dlg%2Doffset%2D10%7Bmargin%2Dleft%3A83%2E33333333%25%7D%2Ecol%2Dlg%2Doffset%2D9%7Bmargin%2Dleft%3A75%25%7D%2Ecol%2Dlg%2Doffset%2D8%7Bmargin%2Dleft%3A66%2E66666667%25%7D%2Ecol%2Dlg%2Doffset%2D7%7Bmargin%2Dleft%3A58%2E33333333%25%7D%2Ecol%2Dlg%2Doffset%2D6%7Bmargin%2Dleft%3A50%25%7D%2Ecol%2Dlg%2Doffset%2D5%7Bmargin%2Dleft%3A41%2E66666667%25%7D%2Ecol%2Dlg%2Doffset%2D4%7Bmargin%2Dleft%3A33%2E33333333%25%7D%2Ecol%2Dlg%2Doffset%2D3%7Bmargin%2Dleft%3A25%25%7D%2Ecol%2Dlg%2Doffset%2D2%7Bmargin%2Dleft%3A16%2E66666667%25%7D%2Ecol%2Dlg%2Doffset%2D1%7Bmargin%2Dleft%3A8%2E33333333%25%7D%2Ecol%2Dlg%2Doffset%2D0%7Bmargin%2Dleft%3A0%7D%7Dtable%7Bbackground%2Dcolor%3Atransparent%7Dcaption%7Bpadding%2Dtop%3A8px%3Bpadding%2Dbottom%3A8px%3Bcolor%3A%23777%3Btext%2Dalign%3Aleft%7Dth%7B%7D%2Etable%7Bwidth%3A100%25%3Bmax%2Dwidth%3A100%25%3Bmargin%2Dbottom%3A20px%7D%2Etable%3Etbody%3Etr%3Etd%2C%2Etable%3Etbody%3Etr%3Eth%2C%2Etable%3Etfoot%3Etr%3Etd%2C%2Etable%3Etfoot%3Etr%3Eth%2C%2Etable%3Ethead%3Etr%3Etd%2C%2Etable%3Ethead%3Etr%3Eth%7Bpadding%3A8px%3Bline%2Dheight%3A1%2E42857143%3Bvertical%2Dalign%3Atop%3Bborder%2Dtop%3A1px%20solid%20%23ddd%7D%2Etable%3Ethead%3Etr%3Eth%7Bvertical%2Dalign%3Abottom%3Bborder%2Dbottom%3A2px%20solid%20%23ddd%7D%2Etable%3Ecaption%2Bthead%3Etr%3Afirst%2Dchild%3Etd%2C%2Etable%3Ecaption%2Bthead%3Etr%3Afirst%2Dchild%3Eth%2C%2Etable%3Ecolgroup%2Bthead%3Etr%3Afirst%2Dchild%3Etd%2C%2Etable%3Ecolgroup%2Bthead%3Etr%3Afirst%2Dchild%3Eth%2C%2Etable%3Ethead%3Afirst%2Dchild%3Etr%3Afirst%2Dchild%3Etd%2C%2Etable%3Ethead%3Afirst%2Dchild%3Etr%3Afirst%2Dchild%3Eth%7Bborder%2Dtop%3A0%7D%2Etable%3Etbody%2Btbody%7Bborder%2Dtop%3A2px%20solid%20%23ddd%7D%2Etable%20%2Etable%7Bbackground%2Dcolor%3A%23fff%7D%2Etable%2Dcondensed%3Etbody%3Etr%3Etd%2C%2Etable%2Dcondensed%3Etbody%3Etr%3Eth%2C%2Etable%2Dcondensed%3Etfoot%3Etr%3Etd%2C%2Etable%2Dcondensed%3Etfoot%3Etr%3Eth%2C%2Etable%2Dcondensed%3Ethead%3Etr%3Etd%2C%2Etable%2Dcondensed%3Ethead%3Etr%3Eth%7Bpadding%3A5px%7D%2Etable%2Dbordered%7Bborder%3A1px%20solid%20%23ddd%7D%2Etable%2Dbordered%3Etbody%3Etr%3Etd%2C%2Etable%2Dbordered%3Etbody%3Etr%3Eth%2C%2Etable%2Dbordered%3Etfoot%3Etr%3Etd%2C%2Etable%2Dbordered%3Etfoot%3Etr%3Eth%2C%2Etable%2Dbordered%3Ethead%3Etr%3Etd%2C%2Etable%2Dbordered%3Ethead%3Etr%3Eth%7Bborder%3A1px%20solid%20%23ddd%7D%2Etable%2Dbordered%3Ethead%3Etr%3Etd%2C%2Etable%2Dbordered%3Ethead%3Etr%3Eth%7Bborder%2Dbottom%2Dwidth%3A2px%7D%2Etable%2Dstriped%3Etbody%3Etr%3Anth%2Dof%2Dtype%28odd%29%7Bbackground%2Dcolor%3A%23f9f9f9%7D%2Etable%2Dhover%3Etbody%3Etr%3Ahover%7Bbackground%2Dcolor%3A%23f5f5f5%7Dtable%20col%5Bclass%2A%3Dcol%2D%5D%7Bposition%3Astatic%3Bdisplay%3Atable%2Dcolumn%3Bfloat%3Anone%7Dtable%20td%5Bclass%2A%3Dcol%2D%5D%2Ctable%20th%5Bclass%2A%3Dcol%2D%5D%7Bposition%3Astatic%3Bdisplay%3Atable%2Dcell%3Bfloat%3Anone%7D%2Etable%3Etbody%3Etr%2Eactive%3Etd%2C%2Etable%3Etbody%3Etr%2Eactive%3Eth%2C%2Etable%3Etbody%3Etr%3Etd%2Eactive%2C%2Etable%3Etbody%3Etr%3Eth%2Eactive%2C%2Etable%3Etfoot%3Etr%2Eactive%3Etd%2C%2Etable%3Etfoot%3Etr%2Eactive%3Eth%2C%2Etable%3Etfoot%3Etr%3Etd%2Eactive%2C%2Etable%3Etfoot%3Etr%3Eth%2Eactive%2C%2Etable%3Ethead%3Etr%2Eactive%3Etd%2C%2Etable%3Ethead%3Etr%2Eactive%3Eth%2C%2Etable%3Ethead%3Etr%3Etd%2Eactive%2C%2Etable%3Ethead%3Etr%3Eth%2Eactive%7Bbackground%2Dcolor%3A%23f5f5f5%7D%2Etable%2Dhover%3Etbody%3Etr%2Eactive%3Ahover%3Etd%2C%2Etable%2Dhover%3Etbody%3Etr%2Eactive%3Ahover%3Eth%2C%2Etable%2Dhover%3Etbody%3Etr%3Ahover%3E%2Eactive%2C%2Etable%2Dhover%3Etbody%3Etr%3Etd%2Eactive%3Ahover%2C%2Etable%2Dhover%3Etbody%3Etr%3Eth%2Eactive%3Ahover%7Bbackground%2Dcolor%3A%23e8e8e8%7D%2Etable%3Etbody%3Etr%2Esuccess%3Etd%2C%2Etable%3Etbody%3Etr%2Esuccess%3Eth%2C%2Etable%3Etbody%3Etr%3Etd%2Esuccess%2C%2Etable%3Etbody%3Etr%3Eth%2Esuccess%2C%2Etable%3Etfoot%3Etr%2Esuccess%3Etd%2C%2Etable%3Etfoot%3Etr%2Esuccess%3Eth%2C%2Etable%3Etfoot%3Etr%3Etd%2Esuccess%2C%2Etable%3Etfoot%3Etr%3Eth%2Esuccess%2C%2Etable%3Ethead%3Etr%2Esuccess%3Etd%2C%2Etable%3Ethead%3Etr%2Esuccess%3Eth%2C%2Etable%3Ethead%3Etr%3Etd%2Esuccess%2C%2Etable%3Ethead%3Etr%3Eth%2Esuccess%7Bbackground%2Dcolor%3A%23dff0d8%7D%2Etable%2Dhover%3Etbody%3Etr%2Esuccess%3Ahover%3Etd%2C%2Etable%2Dhover%3Etbody%3Etr%2Esuccess%3Ahover%3Eth%2C%2Etable%2Dhover%3Etbody%3Etr%3Ahover%3E%2Esuccess%2C%2Etable%2Dhover%3Etbody%3Etr%3Etd%2Esuccess%3Ahover%2C%2Etable%2Dhover%3Etbody%3Etr%3Eth%2Esuccess%3Ahover%7Bbackground%2Dcolor%3A%23d0e9c6%7D%2Etable%3Etbody%3Etr%2Einfo%3Etd%2C%2Etable%3Etbody%3Etr%2Einfo%3Eth%2C%2Etable%3Etbody%3Etr%3Etd%2Einfo%2C%2Etable%3Etbody%3Etr%3Eth%2Einfo%2C%2Etable%3Etfoot%3Etr%2Einfo%3Etd%2C%2Etable%3Etfoot%3Etr%2Einfo%3Eth%2C%2Etable%3Etfoot%3Etr%3Etd%2Einfo%2C%2Etable%3Etfoot%3Etr%3Eth%2Einfo%2C%2Etable%3Ethead%3Etr%2Einfo%3Etd%2C%2Etable%3Ethead%3Etr%2Einfo%3Eth%2C%2Etable%3Ethead%3Etr%3Etd%2Einfo%2C%2Etable%3Ethead%3Etr%3Eth%2Einfo%7Bbackground%2Dcolor%3A%23d9edf7%7D%2Etable%2Dhover%3Etbody%3Etr%2Einfo%3Ahover%3Etd%2C%2Etable%2Dhover%3Etbody%3Etr%2Einfo%3Ahover%3Eth%2C%2Etable%2Dhover%3Etbody%3Etr%3Ahover%3E%2Einfo%2C%2Etable%2Dhover%3Etbody%3Etr%3Etd%2Einfo%3Ahover%2C%2Etable%2Dhover%3Etbody%3Etr%3Eth%2Einfo%3Ahover%7Bbackground%2Dcolor%3A%23c4e3f3%7D%2Etable%3Etbody%3Etr%2Ewarning%3Etd%2C%2Etable%3Etbody%3Etr%2Ewarning%3Eth%2C%2Etable%3Etbody%3Etr%3Etd%2Ewarning%2C%2Etable%3Etbody%3Etr%3Eth%2Ewarning%2C%2Etable%3Etfoot%3Etr%2Ewarning%3Etd%2C%2Etable%3Etfoot%3Etr%2Ewarning%3Eth%2C%2Etable%3Etfoot%3Etr%3Etd%2Ewarning%2C%2Etable%3Etfoot%3Etr%3Eth%2Ewarning%2C%2Etable%3Ethead%3Etr%2Ewarning%3Etd%2C%2Etable%3Ethead%3Etr%2Ewarning%3Eth%2C%2Etable%3Ethead%3Etr%3Etd%2Ewarning%2C%2Etable%3Ethead%3Etr%3Eth%2Ewarning%7Bbackground%2Dcolor%3A%23fcf8e3%7D%2Etable%2Dhover%3Etbody%3Etr%2Ewarning%3Ahover%3Etd%2C%2Etable%2Dhover%3Etbody%3Etr%2Ewarning%3Ahover%3Eth%2C%2Etable%2Dhover%3Etbody%3Etr%3Ahover%3E%2Ewarning%2C%2Etable%2Dhover%3Etbody%3Etr%3Etd%2Ewarning%3Ahover%2C%2Etable%2Dhover%3Etbody%3Etr%3Eth%2Ewarning%3Ahover%7Bbackground%2Dcolor%3A%23faf2cc%7D%2Etable%3Etbody%3Etr%2Edanger%3Etd%2C%2Etable%3Etbody%3Etr%2Edanger%3Eth%2C%2Etable%3Etbody%3Etr%3Etd%2Edanger%2C%2Etable%3Etbody%3Etr%3Eth%2Edanger%2C%2Etable%3Etfoot%3Etr%2Edanger%3Etd%2C%2Etable%3Etfoot%3Etr%2Edanger%3Eth%2C%2Etable%3Etfoot%3Etr%3Etd%2Edanger%2C%2Etable%3Etfoot%3Etr%3Eth%2Edanger%2C%2Etable%3Ethead%3Etr%2Edanger%3Etd%2C%2Etable%3Ethead%3Etr%2Edanger%3Eth%2C%2Etable%3Ethead%3Etr%3Etd%2Edanger%2C%2Etable%3Ethead%3Etr%3Eth%2Edanger%7Bbackground%2Dcolor%3A%23f2dede%7D%2Etable%2Dhover%3Etbody%3Etr%2Edanger%3Ahover%3Etd%2C%2Etable%2Dhover%3Etbody%3Etr%2Edanger%3Ahover%3Eth%2C%2Etable%2Dhover%3Etbody%3Etr%3Ahover%3E%2Edanger%2C%2Etable%2Dhover%3Etbody%3Etr%3Etd%2Edanger%3Ahover%2C%2Etable%2Dhover%3Etbody%3Etr%3Eth%2Edanger%3Ahover%7Bbackground%2Dcolor%3A%23ebcccc%7D%2Etable%2Dresponsive%7Bmin%2Dheight%3A%2E01%25%3Boverflow%2Dx%3Aauto%7D%40media%20screen%20and%20%28max%2Dwidth%3A767px%29%7B%2Etable%2Dresponsive%7Bwidth%3A100%25%3Bmargin%2Dbottom%3A15px%3Boverflow%2Dy%3Ahidden%3B%2Dms%2Doverflow%2Dstyle%3A%2Dms%2Dautohiding%2Dscrollbar%3Bborder%3A1px%20solid%20%23ddd%7D%2Etable%2Dresponsive%3E%2Etable%7Bmargin%2Dbottom%3A0%7D%2Etable%2Dresponsive%3E%2Etable%3Etbody%3Etr%3Etd%2C%2Etable%2Dresponsive%3E%2Etable%3Etbody%3Etr%3Eth%2C%2Etable%2Dresponsive%3E%2Etable%3Etfoot%3Etr%3Etd%2C%2Etable%2Dresponsive%3E%2Etable%3Etfoot%3Etr%3Eth%2C%2Etable%2Dresponsive%3E%2Etable%3Ethead%3Etr%3Etd%2C%2Etable%2Dresponsive%3E%2Etable%3Ethead%3Etr%3Eth%7Bwhite%2Dspace%3Anowrap%7D%2Etable%2Dresponsive%3E%2Etable%2Dbordered%7Bborder%3A0%7D%2Etable%2Dresponsive%3E%2Etable%2Dbordered%3Etbody%3Etr%3Etd%3Afirst%2Dchild%2C%2Etable%2Dresponsive%3E%2Etable%2Dbordered%3Etbody%3Etr%3Eth%3Afirst%2Dchild%2C%2Etable%2Dresponsive%3E%2Etable%2Dbordered%3Etfoot%3Etr%3Etd%3Afirst%2Dchild%2C%2Etable%2Dresponsive%3E%2Etable%2Dbordered%3Etfoot%3Etr%3Eth%3Afirst%2Dchild%2C%2Etable%2Dresponsive%3E%2Etable%2Dbordered%3Ethead%3Etr%3Etd%3Afirst%2Dchild%2C%2Etable%2Dresponsive%3E%2Etable%2Dbordered%3Ethead%3Etr%3Eth%3Afirst%2Dchild%7Bborder%2Dleft%3A0%7D%2Etable%2Dresponsive%3E%2Etable%2Dbordered%3Etbody%3Etr%3Etd%3Alast%2Dchild%2C%2Etable%2Dresponsive%3E%2Etable%2Dbordered%3Etbody%3Etr%3Eth%3Alast%2Dchild%2C%2Etable%2Dresponsive%3E%2Etable%2Dbordered%3Etfoot%3Etr%3Etd%3Alast%2Dchild%2C%2Etable%2Dresponsive%3E%2Etable%2Dbordered%3Etfoot%3Etr%3Eth%3Alast%2Dchild%2C%2Etable%2Dresponsive%3E%2Etable%2Dbordered%3Ethead%3Etr%3Etd%3Alast%2Dchild%2C%2Etable%2Dresponsive%3E%2Etable%2Dbordered%3Ethead%3Etr%3Eth%3Alast%2Dchild%7Bborder%2Dright%3A0%7D%2Etable%2Dresponsive%3E%2Etable%2Dbordered%3Etbody%3Etr%3Alast%2Dchild%3Etd%2C%2Etable%2Dresponsive%3E%2Etable%2Dbordered%3Etbody%3Etr%3Alast%2Dchild%3Eth%2C%2Etable%2Dresponsive%3E%2Etable%2Dbordered%3Etfoot%3Etr%3Alast%2Dchild%3Etd%2C%2Etable%2Dresponsive%3E%2Etable%2Dbordered%3Etfoot%3Etr%3Alast%2Dchild%3Eth%7Bborder%2Dbottom%3A0%7D%7Dfieldset%7Bmin%2Dwidth%3A0%3Bpadding%3A0%3Bmargin%3A0%3Bborder%3A0%7Dlegend%7Bdisplay%3Ablock%3Bwidth%3A100%25%3Bpadding%3A0%3Bmargin%2Dbottom%3A20px%3Bfont%2Dsize%3A21px%3Bline%2Dheight%3Ainherit%3Bcolor%3A%23333%3Bborder%3A0%3Bborder%2Dbottom%3A1px%20solid%20%23e5e5e5%7Dlabel%7Bdisplay%3Ainline%2Dblock%3Bmax%2Dwidth%3A100%25%3Bmargin%2Dbottom%3A5px%3Bfont%2Dweight%3A700%7Dinput%5Btype%3Dsearch%5D%7B%2Dwebkit%2Dbox%2Dsizing%3Aborder%2Dbox%3B%2Dmoz%2Dbox%2Dsizing%3Aborder%2Dbox%3Bbox%2Dsizing%3Aborder%2Dbox%7Dinput%5Btype%3Dcheckbox%5D%2Cinput%5Btype%3Dradio%5D%7Bmargin%3A4px%200%200%3Bmargin%2Dtop%3A1px%5C9%3Bline%2Dheight%3Anormal%7Dinput%5Btype%3Dfile%5D%7Bdisplay%3Ablock%7Dinput%5Btype%3Drange%5D%7Bdisplay%3Ablock%3Bwidth%3A100%25%7Dselect%5Bmultiple%5D%2Cselect%5Bsize%5D%7Bheight%3Aauto%7Dinput%5Btype%3Dfile%5D%3Afocus%2Cinput%5Btype%3Dcheckbox%5D%3Afocus%2Cinput%5Btype%3Dradio%5D%3Afocus%7Boutline%3Athin%20dotted%3Boutline%3A5px%20auto%20%2Dwebkit%2Dfocus%2Dring%2Dcolor%3Boutline%2Doffset%3A%2D2px%7Doutput%7Bdisplay%3Ablock%3Bpadding%2Dtop%3A7px%3Bfont%2Dsize%3A14px%3Bline%2Dheight%3A1%2E42857143%3Bcolor%3A%23555%7D%2Eform%2Dcontrol%7Bdisplay%3Ablock%3Bwidth%3A100%25%3Bheight%3A34px%3Bpadding%3A6px%2012px%3Bfont%2Dsize%3A14px%3Bline%2Dheight%3A1%2E42857143%3Bcolor%3A%23555%3Bbackground%2Dcolor%3A%23fff%3Bbackground%2Dimage%3Anone%3Bborder%3A1px%20solid%20%23ccc%3Bborder%2Dradius%3A4px%3B%2Dwebkit%2Dbox%2Dshadow%3Ainset%200%201px%201px%20rgba%280%2C0%2C0%2C%2E075%29%3Bbox%2Dshadow%3Ainset%200%201px%201px%20rgba%280%2C0%2C0%2C%2E075%29%3B%2Dwebkit%2Dtransition%3Aborder%2Dcolor%20ease%2Din%2Dout%20%2E15s%2C%2Dwebkit%2Dbox%2Dshadow%20ease%2Din%2Dout%20%2E15s%3B%2Do%2Dtransition%3Aborder%2Dcolor%20ease%2Din%2Dout%20%2E15s%2Cbox%2Dshadow%20ease%2Din%2Dout%20%2E15s%3Btransition%3Aborder%2Dcolor%20ease%2Din%2Dout%20%2E15s%2Cbox%2Dshadow%20ease%2Din%2Dout%20%2E15s%7D%2Eform%2Dcontrol%3Afocus%7Bborder%2Dcolor%3A%2366afe9%3Boutline%3A0%3B%2Dwebkit%2Dbox%2Dshadow%3Ainset%200%201px%201px%20rgba%280%2C0%2C0%2C%2E075%29%2C0%200%208px%20rgba%28102%2C175%2C233%2C%2E6%29%3Bbox%2Dshadow%3Ainset%200%201px%201px%20rgba%280%2C0%2C0%2C%2E075%29%2C0%200%208px%20rgba%28102%2C175%2C233%2C%2E6%29%7D%2Eform%2Dcontrol%3A%3A%2Dmoz%2Dplaceholder%7Bcolor%3A%23999%3Bopacity%3A1%7D%2Eform%2Dcontrol%3A%2Dms%2Dinput%2Dplaceholder%7Bcolor%3A%23999%7D%2Eform%2Dcontrol%3A%3A%2Dwebkit%2Dinput%2Dplaceholder%7Bcolor%3A%23999%7D%2Eform%2Dcontrol%5Bdisabled%5D%2C%2Eform%2Dcontrol%5Breadonly%5D%2Cfieldset%5Bdisabled%5D%20%2Eform%2Dcontrol%7Bbackground%2Dcolor%3A%23eee%3Bopacity%3A1%7D%2Eform%2Dcontrol%5Bdisabled%5D%2Cfieldset%5Bdisabled%5D%20%2Eform%2Dcontrol%7Bcursor%3Anot%2Dallowed%7Dtextarea%2Eform%2Dcontrol%7Bheight%3Aauto%7Dinput%5Btype%3Dsearch%5D%7B%2Dwebkit%2Dappearance%3Anone%7D%40media%20screen%20and%20%28%2Dwebkit%2Dmin%2Ddevice%2Dpixel%2Dratio%3A0%29%7Binput%5Btype%3Ddate%5D%2Eform%2Dcontrol%2Cinput%5Btype%3Dtime%5D%2Eform%2Dcontrol%2Cinput%5Btype%3Ddatetime%2Dlocal%5D%2Eform%2Dcontrol%2Cinput%5Btype%3Dmonth%5D%2Eform%2Dcontrol%7Bline%2Dheight%3A34px%7D%2Einput%2Dgroup%2Dsm%20input%5Btype%3Ddate%5D%2C%2Einput%2Dgroup%2Dsm%20input%5Btype%3Dtime%5D%2C%2Einput%2Dgroup%2Dsm%20input%5Btype%3Ddatetime%2Dlocal%5D%2C%2Einput%2Dgroup%2Dsm%20input%5Btype%3Dmonth%5D%2Cinput%5Btype%3Ddate%5D%2Einput%2Dsm%2Cinput%5Btype%3Dtime%5D%2Einput%2Dsm%2Cinput%5Btype%3Ddatetime%2Dlocal%5D%2Einput%2Dsm%2Cinput%5Btype%3Dmonth%5D%2Einput%2Dsm%7Bline%2Dheight%3A30px%7D%2Einput%2Dgroup%2Dlg%20input%5Btype%3Ddate%5D%2C%2Einput%2Dgroup%2Dlg%20input%5Btype%3Dtime%5D%2C%2Einput%2Dgroup%2Dlg%20input%5Btype%3Ddatetime%2Dlocal%5D%2C%2Einput%2Dgroup%2Dlg%20input%5Btype%3Dmonth%5D%2Cinput%5Btype%3Ddate%5D%2Einput%2Dlg%2Cinput%5Btype%3Dtime%5D%2Einput%2Dlg%2Cinput%5Btype%3Ddatetime%2Dlocal%5D%2Einput%2Dlg%2Cinput%5Btype%3Dmonth%5D%2Einput%2Dlg%7Bline%2Dheight%3A46px%7D%7D%2Eform%2Dgroup%7Bmargin%2Dbottom%3A15px%7D%2Echeckbox%2C%2Eradio%7Bposition%3Arelative%3Bdisplay%3Ablock%3Bmargin%2Dtop%3A10px%3Bmargin%2Dbottom%3A10px%7D%2Echeckbox%20label%2C%2Eradio%20label%7Bmin%2Dheight%3A20px%3Bpadding%2Dleft%3A20px%3Bmargin%2Dbottom%3A0%3Bfont%2Dweight%3A400%3Bcursor%3Apointer%7D%2Echeckbox%20input%5Btype%3Dcheckbox%5D%2C%2Echeckbox%2Dinline%20input%5Btype%3Dcheckbox%5D%2C%2Eradio%20input%5Btype%3Dradio%5D%2C%2Eradio%2Dinline%20input%5Btype%3Dradio%5D%7Bposition%3Aabsolute%3Bmargin%2Dtop%3A4px%5C9%3Bmargin%2Dleft%3A%2D20px%7D%2Echeckbox%2B%2Echeckbox%2C%2Eradio%2B%2Eradio%7Bmargin%2Dtop%3A%2D5px%7D%2Echeckbox%2Dinline%2C%2Eradio%2Dinline%7Bposition%3Arelative%3Bdisplay%3Ainline%2Dblock%3Bpadding%2Dleft%3A20px%3Bmargin%2Dbottom%3A0%3Bfont%2Dweight%3A400%3Bvertical%2Dalign%3Amiddle%3Bcursor%3Apointer%7D%2Echeckbox%2Dinline%2B%2Echeckbox%2Dinline%2C%2Eradio%2Dinline%2B%2Eradio%2Dinline%7Bmargin%2Dtop%3A0%3Bmargin%2Dleft%3A10px%7Dfieldset%5Bdisabled%5D%20input%5Btype%3Dcheckbox%5D%2Cfieldset%5Bdisabled%5D%20input%5Btype%3Dradio%5D%2Cinput%5Btype%3Dcheckbox%5D%2Edisabled%2Cinput%5Btype%3Dcheckbox%5D%5Bdisabled%5D%2Cinput%5Btype%3Dradio%5D%2Edisabled%2Cinput%5Btype%3Dradio%5D%5Bdisabled%5D%7Bcursor%3Anot%2Dallowed%7D%2Echeckbox%2Dinline%2Edisabled%2C%2Eradio%2Dinline%2Edisabled%2Cfieldset%5Bdisabled%5D%20%2Echeckbox%2Dinline%2Cfieldset%5Bdisabled%5D%20%2Eradio%2Dinline%7Bcursor%3Anot%2Dallowed%7D%2Echeckbox%2Edisabled%20label%2C%2Eradio%2Edisabled%20label%2Cfieldset%5Bdisabled%5D%20%2Echeckbox%20label%2Cfieldset%5Bdisabled%5D%20%2Eradio%20label%7Bcursor%3Anot%2Dallowed%7D%2Eform%2Dcontrol%2Dstatic%7Bmin%2Dheight%3A34px%3Bpadding%2Dtop%3A7px%3Bpadding%2Dbottom%3A7px%3Bmargin%2Dbottom%3A0%7D%2Eform%2Dcontrol%2Dstatic%2Einput%2Dlg%2C%2Eform%2Dcontrol%2Dstatic%2Einput%2Dsm%7Bpadding%2Dright%3A0%3Bpadding%2Dleft%3A0%7D%2Einput%2Dsm%7Bheight%3A30px%3Bpadding%3A5px%2010px%3Bfont%2Dsize%3A12px%3Bline%2Dheight%3A1%2E5%3Bborder%2Dradius%3A3px%7Dselect%2Einput%2Dsm%7Bheight%3A30px%3Bline%2Dheight%3A30px%7Dselect%5Bmultiple%5D%2Einput%2Dsm%2Ctextarea%2Einput%2Dsm%7Bheight%3Aauto%7D%2Eform%2Dgroup%2Dsm%20%2Eform%2Dcontrol%7Bheight%3A30px%3Bpadding%3A5px%2010px%3Bfont%2Dsize%3A12px%3Bline%2Dheight%3A1%2E5%3Bborder%2Dradius%3A3px%7D%2Eform%2Dgroup%2Dsm%20select%2Eform%2Dcontrol%7Bheight%3A30px%3Bline%2Dheight%3A30px%7D%2Eform%2Dgroup%2Dsm%20select%5Bmultiple%5D%2Eform%2Dcontrol%2C%2Eform%2Dgroup%2Dsm%20textarea%2Eform%2Dcontrol%7Bheight%3Aauto%7D%2Eform%2Dgroup%2Dsm%20%2Eform%2Dcontrol%2Dstatic%7Bheight%3A30px%3Bmin%2Dheight%3A32px%3Bpadding%3A6px%2010px%3Bfont%2Dsize%3A12px%3Bline%2Dheight%3A1%2E5%7D%2Einput%2Dlg%7Bheight%3A46px%3Bpadding%3A10px%2016px%3Bfont%2Dsize%3A18px%3Bline%2Dheight%3A1%2E3333333%3Bborder%2Dradius%3A6px%7Dselect%2Einput%2Dlg%7Bheight%3A46px%3Bline%2Dheight%3A46px%7Dselect%5Bmultiple%5D%2Einput%2Dlg%2Ctextarea%2Einput%2Dlg%7Bheight%3Aauto%7D%2Eform%2Dgroup%2Dlg%20%2Eform%2Dcontrol%7Bheight%3A46px%3Bpadding%3A10px%2016px%3Bfont%2Dsize%3A18px%3Bline%2Dheight%3A1%2E3333333%3Bborder%2Dradius%3A6px%7D%2Eform%2Dgroup%2Dlg%20select%2Eform%2Dcontrol%7Bheight%3A46px%3Bline%2Dheight%3A46px%7D%2Eform%2Dgroup%2Dlg%20select%5Bmultiple%5D%2Eform%2Dcontrol%2C%2Eform%2Dgroup%2Dlg%20textarea%2Eform%2Dcontrol%7Bheight%3Aauto%7D%2Eform%2Dgroup%2Dlg%20%2Eform%2Dcontrol%2Dstatic%7Bheight%3A46px%3Bmin%2Dheight%3A38px%3Bpadding%3A11px%2016px%3Bfont%2Dsize%3A18px%3Bline%2Dheight%3A1%2E3333333%7D%2Ehas%2Dfeedback%7Bposition%3Arelative%7D%2Ehas%2Dfeedback%20%2Eform%2Dcontrol%7Bpadding%2Dright%3A42%2E5px%7D%2Eform%2Dcontrol%2Dfeedback%7Bposition%3Aabsolute%3Btop%3A0%3Bright%3A0%3Bz%2Dindex%3A2%3Bdisplay%3Ablock%3Bwidth%3A34px%3Bheight%3A34px%3Bline%2Dheight%3A34px%3Btext%2Dalign%3Acenter%3Bpointer%2Devents%3Anone%7D%2Eform%2Dgroup%2Dlg%20%2Eform%2Dcontrol%2B%2Eform%2Dcontrol%2Dfeedback%2C%2Einput%2Dgroup%2Dlg%2B%2Eform%2Dcontrol%2Dfeedback%2C%2Einput%2Dlg%2B%2Eform%2Dcontrol%2Dfeedback%7Bwidth%3A46px%3Bheight%3A46px%3Bline%2Dheight%3A46px%7D%2Eform%2Dgroup%2Dsm%20%2Eform%2Dcontrol%2B%2Eform%2Dcontrol%2Dfeedback%2C%2Einput%2Dgroup%2Dsm%2B%2Eform%2Dcontrol%2Dfeedback%2C%2Einput%2Dsm%2B%2Eform%2Dcontrol%2Dfeedback%7Bwidth%3A30px%3Bheight%3A30px%3Bline%2Dheight%3A30px%7D%2Ehas%2Dsuccess%20%2Echeckbox%2C%2Ehas%2Dsuccess%20%2Echeckbox%2Dinline%2C%2Ehas%2Dsuccess%20%2Econtrol%2Dlabel%2C%2Ehas%2Dsuccess%20%2Ehelp%2Dblock%2C%2Ehas%2Dsuccess%20%2Eradio%2C%2Ehas%2Dsuccess%20%2Eradio%2Dinline%2C%2Ehas%2Dsuccess%2Echeckbox%20label%2C%2Ehas%2Dsuccess%2Echeckbox%2Dinline%20label%2C%2Ehas%2Dsuccess%2Eradio%20label%2C%2Ehas%2Dsuccess%2Eradio%2Dinline%20label%7Bcolor%3A%233c763d%7D%2Ehas%2Dsuccess%20%2Eform%2Dcontrol%7Bborder%2Dcolor%3A%233c763d%3B%2Dwebkit%2Dbox%2Dshadow%3Ainset%200%201px%201px%20rgba%280%2C0%2C0%2C%2E075%29%3Bbox%2Dshadow%3Ainset%200%201px%201px%20rgba%280%2C0%2C0%2C%2E075%29%7D%2Ehas%2Dsuccess%20%2Eform%2Dcontrol%3Afocus%7Bborder%2Dcolor%3A%232b542c%3B%2Dwebkit%2Dbox%2Dshadow%3Ainset%200%201px%201px%20rgba%280%2C0%2C0%2C%2E075%29%2C0%200%206px%20%2367b168%3Bbox%2Dshadow%3Ainset%200%201px%201px%20rgba%280%2C0%2C0%2C%2E075%29%2C0%200%206px%20%2367b168%7D%2Ehas%2Dsuccess%20%2Einput%2Dgroup%2Daddon%7Bcolor%3A%233c763d%3Bbackground%2Dcolor%3A%23dff0d8%3Bborder%2Dcolor%3A%233c763d%7D%2Ehas%2Dsuccess%20%2Eform%2Dcontrol%2Dfeedback%7Bcolor%3A%233c763d%7D%2Ehas%2Dwarning%20%2Echeckbox%2C%2Ehas%2Dwarning%20%2Echeckbox%2Dinline%2C%2Ehas%2Dwarning%20%2Econtrol%2Dlabel%2C%2Ehas%2Dwarning%20%2Ehelp%2Dblock%2C%2Ehas%2Dwarning%20%2Eradio%2C%2Ehas%2Dwarning%20%2Eradio%2Dinline%2C%2Ehas%2Dwarning%2Echeckbox%20label%2C%2Ehas%2Dwarning%2Echeckbox%2Dinline%20label%2C%2Ehas%2Dwarning%2Eradio%20label%2C%2Ehas%2Dwarning%2Eradio%2Dinline%20label%7Bcolor%3A%238a6d3b%7D%2Ehas%2Dwarning%20%2Eform%2Dcontrol%7Bborder%2Dcolor%3A%238a6d3b%3B%2Dwebkit%2Dbox%2Dshadow%3Ainset%200%201px%201px%20rgba%280%2C0%2C0%2C%2E075%29%3Bbox%2Dshadow%3Ainset%200%201px%201px%20rgba%280%2C0%2C0%2C%2E075%29%7D%2Ehas%2Dwarning%20%2Eform%2Dcontrol%3Afocus%7Bborder%2Dcolor%3A%2366512c%3B%2Dwebkit%2Dbox%2Dshadow%3Ainset%200%201px%201px%20rgba%280%2C0%2C0%2C%2E075%29%2C0%200%206px%20%23c0a16b%3Bbox%2Dshadow%3Ainset%200%201px%201px%20rgba%280%2C0%2C0%2C%2E075%29%2C0%200%206px%20%23c0a16b%7D%2Ehas%2Dwarning%20%2Einput%2Dgroup%2Daddon%7Bcolor%3A%238a6d3b%3Bbackground%2Dcolor%3A%23fcf8e3%3Bborder%2Dcolor%3A%238a6d3b%7D%2Ehas%2Dwarning%20%2Eform%2Dcontrol%2Dfeedback%7Bcolor%3A%238a6d3b%7D%2Ehas%2Derror%20%2Echeckbox%2C%2Ehas%2Derror%20%2Echeckbox%2Dinline%2C%2Ehas%2Derror%20%2Econtrol%2Dlabel%2C%2Ehas%2Derror%20%2Ehelp%2Dblock%2C%2Ehas%2Derror%20%2Eradio%2C%2Ehas%2Derror%20%2Eradio%2Dinline%2C%2Ehas%2Derror%2Echeckbox%20label%2C%2Ehas%2Derror%2Echeckbox%2Dinline%20label%2C%2Ehas%2Derror%2Eradio%20label%2C%2Ehas%2Derror%2Eradio%2Dinline%20label%7Bcolor%3A%23a94442%7D%2Ehas%2Derror%20%2Eform%2Dcontrol%7Bborder%2Dcolor%3A%23a94442%3B%2Dwebkit%2Dbox%2Dshadow%3Ainset%200%201px%201px%20rgba%280%2C0%2C0%2C%2E075%29%3Bbox%2Dshadow%3Ainset%200%201px%201px%20rgba%280%2C0%2C0%2C%2E075%29%7D%2Ehas%2Derror%20%2Eform%2Dcontrol%3Afocus%7Bborder%2Dcolor%3A%23843534%3B%2Dwebkit%2Dbox%2Dshadow%3Ainset%200%201px%201px%20rgba%280%2C0%2C0%2C%2E075%29%2C0%200%206px%20%23ce8483%3Bbox%2Dshadow%3Ainset%200%201px%201px%20rgba%280%2C0%2C0%2C%2E075%29%2C0%200%206px%20%23ce8483%7D%2Ehas%2Derror%20%2Einput%2Dgroup%2Daddon%7Bcolor%3A%23a94442%3Bbackground%2Dcolor%3A%23f2dede%3Bborder%2Dcolor%3A%23a94442%7D%2Ehas%2Derror%20%2Eform%2Dcontrol%2Dfeedback%7Bcolor%3A%23a94442%7D%2Ehas%2Dfeedback%20label%7E%2Eform%2Dcontrol%2Dfeedback%7Btop%3A25px%7D%2Ehas%2Dfeedback%20label%2Esr%2Donly%7E%2Eform%2Dcontrol%2Dfeedback%7Btop%3A0%7D%2Ehelp%2Dblock%7Bdisplay%3Ablock%3Bmargin%2Dtop%3A5px%3Bmargin%2Dbottom%3A10px%3Bcolor%3A%23737373%7D%40media%20%28min%2Dwidth%3A768px%29%7B%2Eform%2Dinline%20%2Eform%2Dgroup%7Bdisplay%3Ainline%2Dblock%3Bmargin%2Dbottom%3A0%3Bvertical%2Dalign%3Amiddle%7D%2Eform%2Dinline%20%2Eform%2Dcontrol%7Bdisplay%3Ainline%2Dblock%3Bwidth%3Aauto%3Bvertical%2Dalign%3Amiddle%7D%2Eform%2Dinline%20%2Eform%2Dcontrol%2Dstatic%7Bdisplay%3Ainline%2Dblock%7D%2Eform%2Dinline%20%2Einput%2Dgroup%7Bdisplay%3Ainline%2Dtable%3Bvertical%2Dalign%3Amiddle%7D%2Eform%2Dinline%20%2Einput%2Dgroup%20%2Eform%2Dcontrol%2C%2Eform%2Dinline%20%2Einput%2Dgroup%20%2Einput%2Dgroup%2Daddon%2C%2Eform%2Dinline%20%2Einput%2Dgroup%20%2Einput%2Dgroup%2Dbtn%7Bwidth%3Aauto%7D%2Eform%2Dinline%20%2Einput%2Dgroup%3E%2Eform%2Dcontrol%7Bwidth%3A100%25%7D%2Eform%2Dinline%20%2Econtrol%2Dlabel%7Bmargin%2Dbottom%3A0%3Bvertical%2Dalign%3Amiddle%7D%2Eform%2Dinline%20%2Echeckbox%2C%2Eform%2Dinline%20%2Eradio%7Bdisplay%3Ainline%2Dblock%3Bmargin%2Dtop%3A0%3Bmargin%2Dbottom%3A0%3Bvertical%2Dalign%3Amiddle%7D%2Eform%2Dinline%20%2Echeckbox%20label%2C%2Eform%2Dinline%20%2Eradio%20label%7Bpadding%2Dleft%3A0%7D%2Eform%2Dinline%20%2Echeckbox%20input%5Btype%3Dcheckbox%5D%2C%2Eform%2Dinline%20%2Eradio%20input%5Btype%3Dradio%5D%7Bposition%3Arelative%3Bmargin%2Dleft%3A0%7D%2Eform%2Dinline%20%2Ehas%2Dfeedback%20%2Eform%2Dcontrol%2Dfeedback%7Btop%3A0%7D%7D%2Eform%2Dhorizontal%20%2Echeckbox%2C%2Eform%2Dhorizontal%20%2Echeckbox%2Dinline%2C%2Eform%2Dhorizontal%20%2Eradio%2C%2Eform%2Dhorizontal%20%2Eradio%2Dinline%7Bpadding%2Dtop%3A7px%3Bmargin%2Dtop%3A0%3Bmargin%2Dbottom%3A0%7D%2Eform%2Dhorizontal%20%2Echeckbox%2C%2Eform%2Dhorizontal%20%2Eradio%7Bmin%2Dheight%3A27px%7D%2Eform%2Dhorizontal%20%2Eform%2Dgroup%7Bmargin%2Dright%3A%2D15px%3Bmargin%2Dleft%3A%2D15px%7D%40media%20%28min%2Dwidth%3A768px%29%7B%2Eform%2Dhorizontal%20%2Econtrol%2Dlabel%7Bpadding%2Dtop%3A7px%3Bmargin%2Dbottom%3A0%3Btext%2Dalign%3Aright%7D%7D%2Eform%2Dhorizontal%20%2Ehas%2Dfeedback%20%2Eform%2Dcontrol%2Dfeedback%7Bright%3A15px%7D%40media%20%28min%2Dwidth%3A768px%29%7B%2Eform%2Dhorizontal%20%2Eform%2Dgroup%2Dlg%20%2Econtrol%2Dlabel%7Bpadding%2Dtop%3A14%2E33px%3Bfont%2Dsize%3A18px%7D%7D%40media%20%28min%2Dwidth%3A768px%29%7B%2Eform%2Dhorizontal%20%2Eform%2Dgroup%2Dsm%20%2Econtrol%2Dlabel%7Bpadding%2Dtop%3A6px%3Bfont%2Dsize%3A12px%7D%7D%2Ebtn%7Bdisplay%3Ainline%2Dblock%3Bpadding%3A6px%2012px%3Bmargin%2Dbottom%3A0%3Bfont%2Dsize%3A14px%3Bfont%2Dweight%3A400%3Bline%2Dheight%3A1%2E42857143%3Btext%2Dalign%3Acenter%3Bwhite%2Dspace%3Anowrap%3Bvertical%2Dalign%3Amiddle%3B%2Dms%2Dtouch%2Daction%3Amanipulation%3Btouch%2Daction%3Amanipulation%3Bcursor%3Apointer%3B%2Dwebkit%2Duser%2Dselect%3Anone%3B%2Dmoz%2Duser%2Dselect%3Anone%3B%2Dms%2Duser%2Dselect%3Anone%3Buser%2Dselect%3Anone%3Bbackground%2Dimage%3Anone%3Bborder%3A1px%20solid%20transparent%3Bborder%2Dradius%3A4px%7D%2Ebtn%2Eactive%2Efocus%2C%2Ebtn%2Eactive%3Afocus%2C%2Ebtn%2Efocus%2C%2Ebtn%3Aactive%2Efocus%2C%2Ebtn%3Aactive%3Afocus%2C%2Ebtn%3Afocus%7Boutline%3Athin%20dotted%3Boutline%3A5px%20auto%20%2Dwebkit%2Dfocus%2Dring%2Dcolor%3Boutline%2Doffset%3A%2D2px%7D%2Ebtn%2Efocus%2C%2Ebtn%3Afocus%2C%2Ebtn%3Ahover%7Bcolor%3A%23333%3Btext%2Ddecoration%3Anone%7D%2Ebtn%2Eactive%2C%2Ebtn%3Aactive%7Bbackground%2Dimage%3Anone%3Boutline%3A0%3B%2Dwebkit%2Dbox%2Dshadow%3Ainset%200%203px%205px%20rgba%280%2C0%2C0%2C%2E125%29%3Bbox%2Dshadow%3Ainset%200%203px%205px%20rgba%280%2C0%2C0%2C%2E125%29%7D%2Ebtn%2Edisabled%2C%2Ebtn%5Bdisabled%5D%2Cfieldset%5Bdisabled%5D%20%2Ebtn%7Bcursor%3Anot%2Dallowed%3Bfilter%3Aalpha%28opacity%3D65%29%3B%2Dwebkit%2Dbox%2Dshadow%3Anone%3Bbox%2Dshadow%3Anone%3Bopacity%3A%2E65%7Da%2Ebtn%2Edisabled%2Cfieldset%5Bdisabled%5D%20a%2Ebtn%7Bpointer%2Devents%3Anone%7D%2Ebtn%2Ddefault%7Bcolor%3A%23333%3Bbackground%2Dcolor%3A%23fff%3Bborder%2Dcolor%3A%23ccc%7D%2Ebtn%2Ddefault%2Efocus%2C%2Ebtn%2Ddefault%3Afocus%7Bcolor%3A%23333%3Bbackground%2Dcolor%3A%23e6e6e6%3Bborder%2Dcolor%3A%238c8c8c%7D%2Ebtn%2Ddefault%3Ahover%7Bcolor%3A%23333%3Bbackground%2Dcolor%3A%23e6e6e6%3Bborder%2Dcolor%3A%23adadad%7D%2Ebtn%2Ddefault%2Eactive%2C%2Ebtn%2Ddefault%3Aactive%2C%2Eopen%3E%2Edropdown%2Dtoggle%2Ebtn%2Ddefault%7Bcolor%3A%23333%3Bbackground%2Dcolor%3A%23e6e6e6%3Bborder%2Dcolor%3A%23adadad%7D%2Ebtn%2Ddefault%2Eactive%2Efocus%2C%2Ebtn%2Ddefault%2Eactive%3Afocus%2C%2Ebtn%2Ddefault%2Eactive%3Ahover%2C%2Ebtn%2Ddefault%3Aactive%2Efocus%2C%2Ebtn%2Ddefault%3Aactive%3Afocus%2C%2Ebtn%2Ddefault%3Aactive%3Ahover%2C%2Eopen%3E%2Edropdown%2Dtoggle%2Ebtn%2Ddefault%2Efocus%2C%2Eopen%3E%2Edropdown%2Dtoggle%2Ebtn%2Ddefault%3Afocus%2C%2Eopen%3E%2Edropdown%2Dtoggle%2Ebtn%2Ddefault%3Ahover%7Bcolor%3A%23333%3Bbackground%2Dcolor%3A%23d4d4d4%3Bborder%2Dcolor%3A%238c8c8c%7D%2Ebtn%2Ddefault%2Eactive%2C%2Ebtn%2Ddefault%3Aactive%2C%2Eopen%3E%2Edropdown%2Dtoggle%2Ebtn%2Ddefault%7Bbackground%2Dimage%3Anone%7D%2Ebtn%2Ddefault%2Edisabled%2C%2Ebtn%2Ddefault%2Edisabled%2Eactive%2C%2Ebtn%2Ddefault%2Edisabled%2Efocus%2C%2Ebtn%2Ddefault%2Edisabled%3Aactive%2C%2Ebtn%2Ddefault%2Edisabled%3Afocus%2C%2Ebtn%2Ddefault%2Edisabled%3Ahover%2C%2Ebtn%2Ddefault%5Bdisabled%5D%2C%2Ebtn%2Ddefault%5Bdisabled%5D%2Eactive%2C%2Ebtn%2Ddefault%5Bdisabled%5D%2Efocus%2C%2Ebtn%2Ddefault%5Bdisabled%5D%3Aactive%2C%2Ebtn%2Ddefault%5Bdisabled%5D%3Afocus%2C%2Ebtn%2Ddefault%5Bdisabled%5D%3Ahover%2Cfieldset%5Bdisabled%5D%20%2Ebtn%2Ddefault%2Cfieldset%5Bdisabled%5D%20%2Ebtn%2Ddefault%2Eactive%2Cfieldset%5Bdisabled%5D%20%2Ebtn%2Ddefault%2Efocus%2Cfieldset%5Bdisabled%5D%20%2Ebtn%2Ddefault%3Aactive%2Cfieldset%5Bdisabled%5D%20%2Ebtn%2Ddefault%3Afocus%2Cfieldset%5Bdisabled%5D%20%2Ebtn%2Ddefault%3Ahover%7Bbackground%2Dcolor%3A%23fff%3Bborder%2Dcolor%3A%23ccc%7D%2Ebtn%2Ddefault%20%2Ebadge%7Bcolor%3A%23fff%3Bbackground%2Dcolor%3A%23333%7D%2Ebtn%2Dprimary%7Bcolor%3A%23fff%3Bbackground%2Dcolor%3A%23337ab7%3Bborder%2Dcolor%3A%232e6da4%7D%2Ebtn%2Dprimary%2Efocus%2C%2Ebtn%2Dprimary%3Afocus%7Bcolor%3A%23fff%3Bbackground%2Dcolor%3A%23286090%3Bborder%2Dcolor%3A%23122b40%7D%2Ebtn%2Dprimary%3Ahover%7Bcolor%3A%23fff%3Bbackground%2Dcolor%3A%23286090%3Bborder%2Dcolor%3A%23204d74%7D%2Ebtn%2Dprimary%2Eactive%2C%2Ebtn%2Dprimary%3Aactive%2C%2Eopen%3E%2Edropdown%2Dtoggle%2Ebtn%2Dprimary%7Bcolor%3A%23fff%3Bbackground%2Dcolor%3A%23286090%3Bborder%2Dcolor%3A%23204d74%7D%2Ebtn%2Dprimary%2Eactive%2Efocus%2C%2Ebtn%2Dprimary%2Eactive%3Afocus%2C%2Ebtn%2Dprimary%2Eactive%3Ahover%2C%2Ebtn%2Dprimary%3Aactive%2Efocus%2C%2Ebtn%2Dprimary%3Aactive%3Afocus%2C%2Ebtn%2Dprimary%3Aactive%3Ahover%2C%2Eopen%3E%2Edropdown%2Dtoggle%2Ebtn%2Dprimary%2Efocus%2C%2Eopen%3E%2Edropdown%2Dtoggle%2Ebtn%2Dprimary%3Afocus%2C%2Eopen%3E%2Edropdown%2Dtoggle%2Ebtn%2Dprimary%3Ahover%7Bcolor%3A%23fff%3Bbackground%2Dcolor%3A%23204d74%3Bborder%2Dcolor%3A%23122b40%7D%2Ebtn%2Dprimary%2Eactive%2C%2Ebtn%2Dprimary%3Aactive%2C%2Eopen%3E%2Edropdown%2Dtoggle%2Ebtn%2Dprimary%7Bbackground%2Dimage%3Anone%7D%2Ebtn%2Dprimary%2Edisabled%2C%2Ebtn%2Dprimary%2Edisabled%2Eactive%2C%2Ebtn%2Dprimary%2Edisabled%2Efocus%2C%2Ebtn%2Dprimary%2Edisabled%3Aactive%2C%2Ebtn%2Dprimary%2Edisabled%3Afocus%2C%2Ebtn%2Dprimary%2Edisabled%3Ahover%2C%2Ebtn%2Dprimary%5Bdisabled%5D%2C%2Ebtn%2Dprimary%5Bdisabled%5D%2Eactive%2C%2Ebtn%2Dprimary%5Bdisabled%5D%2Efocus%2C%2Ebtn%2Dprimary%5Bdisabled%5D%3Aactive%2C%2Ebtn%2Dprimary%5Bdisabled%5D%3Afocus%2C%2Ebtn%2Dprimary%5Bdisabled%5D%3Ahover%2Cfieldset%5Bdisabled%5D%20%2Ebtn%2Dprimary%2Cfieldset%5Bdisabled%5D%20%2Ebtn%2Dprimary%2Eactive%2Cfieldset%5Bdisabled%5D%20%2Ebtn%2Dprimary%2Efocus%2Cfieldset%5Bdisabled%5D%20%2Ebtn%2Dprimary%3Aactive%2Cfieldset%5Bdisabled%5D%20%2Ebtn%2Dprimary%3Afocus%2Cfieldset%5Bdisabled%5D%20%2Ebtn%2Dprimary%3Ahover%7Bbackground%2Dcolor%3A%23337ab7%3Bborder%2Dcolor%3A%232e6da4%7D%2Ebtn%2Dprimary%20%2Ebadge%7Bcolor%3A%23337ab7%3Bbackground%2Dcolor%3A%23fff%7D%2Ebtn%2Dsuccess%7Bcolor%3A%23fff%3Bbackground%2Dcolor%3A%235cb85c%3Bborder%2Dcolor%3A%234cae4c%7D%2Ebtn%2Dsuccess%2Efocus%2C%2Ebtn%2Dsuccess%3Afocus%7Bcolor%3A%23fff%3Bbackground%2Dcolor%3A%23449d44%3Bborder%2Dcolor%3A%23255625%7D%2Ebtn%2Dsuccess%3Ahover%7Bcolor%3A%23fff%3Bbackground%2Dcolor%3A%23449d44%3Bborder%2Dcolor%3A%23398439%7D%2Ebtn%2Dsuccess%2Eactive%2C%2Ebtn%2Dsuccess%3Aactive%2C%2Eopen%3E%2Edropdown%2Dtoggle%2Ebtn%2Dsuccess%7Bcolor%3A%23fff%3Bbackground%2Dcolor%3A%23449d44%3Bborder%2Dcolor%3A%23398439%7D%2Ebtn%2Dsuccess%2Eactive%2Efocus%2C%2Ebtn%2Dsuccess%2Eactive%3Afocus%2C%2Ebtn%2Dsuccess%2Eactive%3Ahover%2C%2Ebtn%2Dsuccess%3Aactive%2Efocus%2C%2Ebtn%2Dsuccess%3Aactive%3Afocus%2C%2Ebtn%2Dsuccess%3Aactive%3Ahover%2C%2Eopen%3E%2Edropdown%2Dtoggle%2Ebtn%2Dsuccess%2Efocus%2C%2Eopen%3E%2Edropdown%2Dtoggle%2Ebtn%2Dsuccess%3Afocus%2C%2Eopen%3E%2Edropdown%2Dtoggle%2Ebtn%2Dsuccess%3Ahover%7Bcolor%3A%23fff%3Bbackground%2Dcolor%3A%23398439%3Bborder%2Dcolor%3A%23255625%7D%2Ebtn%2Dsuccess%2Eactive%2C%2Ebtn%2Dsuccess%3Aactive%2C%2Eopen%3E%2Edropdown%2Dtoggle%2Ebtn%2Dsuccess%7Bbackground%2Dimage%3Anone%7D%2Ebtn%2Dsuccess%2Edisabled%2C%2Ebtn%2Dsuccess%2Edisabled%2Eactive%2C%2Ebtn%2Dsuccess%2Edisabled%2Efocus%2C%2Ebtn%2Dsuccess%2Edisabled%3Aactive%2C%2Ebtn%2Dsuccess%2Edisabled%3Afocus%2C%2Ebtn%2Dsuccess%2Edisabled%3Ahover%2C%2Ebtn%2Dsuccess%5Bdisabled%5D%2C%2Ebtn%2Dsuccess%5Bdisabled%5D%2Eactive%2C%2Ebtn%2Dsuccess%5Bdisabled%5D%2Efocus%2C%2Ebtn%2Dsuccess%5Bdisabled%5D%3Aactive%2C%2Ebtn%2Dsuccess%5Bdisabled%5D%3Afocus%2C%2Ebtn%2Dsuccess%5Bdisabled%5D%3Ahover%2Cfieldset%5Bdisabled%5D%20%2Ebtn%2Dsuccess%2Cfieldset%5Bdisabled%5D%20%2Ebtn%2Dsuccess%2Eactive%2Cfieldset%5Bdisabled%5D%20%2Ebtn%2Dsuccess%2Efocus%2Cfieldset%5Bdisabled%5D%20%2Ebtn%2Dsuccess%3Aactive%2Cfieldset%5Bdisabled%5D%20%2Ebtn%2Dsuccess%3Afocus%2Cfieldset%5Bdisabled%5D%20%2Ebtn%2Dsuccess%3Ahover%7Bbackground%2Dcolor%3A%235cb85c%3Bborder%2Dcolor%3A%234cae4c%7D%2Ebtn%2Dsuccess%20%2Ebadge%7Bcolor%3A%235cb85c%3Bbackground%2Dcolor%3A%23fff%7D%2Ebtn%2Dinfo%7Bcolor%3A%23fff%3Bbackground%2Dcolor%3A%235bc0de%3Bborder%2Dcolor%3A%2346b8da%7D%2Ebtn%2Dinfo%2Efocus%2C%2Ebtn%2Dinfo%3Afocus%7Bcolor%3A%23fff%3Bbackground%2Dcolor%3A%2331b0d5%3Bborder%2Dcolor%3A%231b6d85%7D%2Ebtn%2Dinfo%3Ahover%7Bcolor%3A%23fff%3Bbackground%2Dcolor%3A%2331b0d5%3Bborder%2Dcolor%3A%23269abc%7D%2Ebtn%2Dinfo%2Eactive%2C%2Ebtn%2Dinfo%3Aactive%2C%2Eopen%3E%2Edropdown%2Dtoggle%2Ebtn%2Dinfo%7Bcolor%3A%23fff%3Bbackground%2Dcolor%3A%2331b0d5%3Bborder%2Dcolor%3A%23269abc%7D%2Ebtn%2Dinfo%2Eactive%2Efocus%2C%2Ebtn%2Dinfo%2Eactive%3Afocus%2C%2Ebtn%2Dinfo%2Eactive%3Ahover%2C%2Ebtn%2Dinfo%3Aactive%2Efocus%2C%2Ebtn%2Dinfo%3Aactive%3Afocus%2C%2Ebtn%2Dinfo%3Aactive%3Ahover%2C%2Eopen%3E%2Edropdown%2Dtoggle%2Ebtn%2Dinfo%2Efocus%2C%2Eopen%3E%2Edropdown%2Dtoggle%2Ebtn%2Dinfo%3Afocus%2C%2Eopen%3E%2Edropdown%2Dtoggle%2Ebtn%2Dinfo%3Ahover%7Bcolor%3A%23fff%3Bbackground%2Dcolor%3A%23269abc%3Bborder%2Dcolor%3A%231b6d85%7D%2Ebtn%2Dinfo%2Eactive%2C%2Ebtn%2Dinfo%3Aactive%2C%2Eopen%3E%2Edropdown%2Dtoggle%2Ebtn%2Dinfo%7Bbackground%2Dimage%3Anone%7D%2Ebtn%2Dinfo%2Edisabled%2C%2Ebtn%2Dinfo%2Edisabled%2Eactive%2C%2Ebtn%2Dinfo%2Edisabled%2Efocus%2C%2Ebtn%2Dinfo%2Edisabled%3Aactive%2C%2Ebtn%2Dinfo%2Edisabled%3Afocus%2C%2Ebtn%2Dinfo%2Edisabled%3Ahover%2C%2Ebtn%2Dinfo%5Bdisabled%5D%2C%2Ebtn%2Dinfo%5Bdisabled%5D%2Eactive%2C%2Ebtn%2Dinfo%5Bdisabled%5D%2Efocus%2C%2Ebtn%2Dinfo%5Bdisabled%5D%3Aactive%2C%2Ebtn%2Dinfo%5Bdisabled%5D%3Afocus%2C%2Ebtn%2Dinfo%5Bdisabled%5D%3Ahover%2Cfieldset%5Bdisabled%5D%20%2Ebtn%2Dinfo%2Cfieldset%5Bdisabled%5D%20%2Ebtn%2Dinfo%2Eactive%2Cfieldset%5Bdisabled%5D%20%2Ebtn%2Dinfo%2Efocus%2Cfieldset%5Bdisabled%5D%20%2Ebtn%2Dinfo%3Aactive%2Cfieldset%5Bdisabled%5D%20%2Ebtn%2Dinfo%3Afocus%2Cfieldset%5Bdisabled%5D%20%2Ebtn%2Dinfo%3Ahover%7Bbackground%2Dcolor%3A%235bc0de%3Bborder%2Dcolor%3A%2346b8da%7D%2Ebtn%2Dinfo%20%2Ebadge%7Bcolor%3A%235bc0de%3Bbackground%2Dcolor%3A%23fff%7D%2Ebtn%2Dwarning%7Bcolor%3A%23fff%3Bbackground%2Dcolor%3A%23f0ad4e%3Bborder%2Dcolor%3A%23eea236%7D%2Ebtn%2Dwarning%2Efocus%2C%2Ebtn%2Dwarning%3Afocus%7Bcolor%3A%23fff%3Bbackground%2Dcolor%3A%23ec971f%3Bborder%2Dcolor%3A%23985f0d%7D%2Ebtn%2Dwarning%3Ahover%7Bcolor%3A%23fff%3Bbackground%2Dcolor%3A%23ec971f%3Bborder%2Dcolor%3A%23d58512%7D%2Ebtn%2Dwarning%2Eactive%2C%2Ebtn%2Dwarning%3Aactive%2C%2Eopen%3E%2Edropdown%2Dtoggle%2Ebtn%2Dwarning%7Bcolor%3A%23fff%3Bbackground%2Dcolor%3A%23ec971f%3Bborder%2Dcolor%3A%23d58512%7D%2Ebtn%2Dwarning%2Eactive%2Efocus%2C%2Ebtn%2Dwarning%2Eactive%3Afocus%2C%2Ebtn%2Dwarning%2Eactive%3Ahover%2C%2Ebtn%2Dwarning%3Aactive%2Efocus%2C%2Ebtn%2Dwarning%3Aactive%3Afocus%2C%2Ebtn%2Dwarning%3Aactive%3Ahover%2C%2Eopen%3E%2Edropdown%2Dtoggle%2Ebtn%2Dwarning%2Efocus%2C%2Eopen%3E%2Edropdown%2Dtoggle%2Ebtn%2Dwarning%3Afocus%2C%2Eopen%3E%2Edropdown%2Dtoggle%2Ebtn%2Dwarning%3Ahover%7Bcolor%3A%23fff%3Bbackground%2Dcolor%3A%23d58512%3Bborder%2Dcolor%3A%23985f0d%7D%2Ebtn%2Dwarning%2Eactive%2C%2Ebtn%2Dwarning%3Aactive%2C%2Eopen%3E%2Edropdown%2Dtoggle%2Ebtn%2Dwarning%7Bbackground%2Dimage%3Anone%7D%2Ebtn%2Dwarning%2Edisabled%2C%2Ebtn%2Dwarning%2Edisabled%2Eactive%2C%2Ebtn%2Dwarning%2Edisabled%2Efocus%2C%2Ebtn%2Dwarning%2Edisabled%3Aactive%2C%2Ebtn%2Dwarning%2Edisabled%3Afocus%2C%2Ebtn%2Dwarning%2Edisabled%3Ahover%2C%2Ebtn%2Dwarning%5Bdisabled%5D%2C%2Ebtn%2Dwarning%5Bdisabled%5D%2Eactive%2C%2Ebtn%2Dwarning%5Bdisabled%5D%2Efocus%2C%2Ebtn%2Dwarning%5Bdisabled%5D%3Aactive%2C%2Ebtn%2Dwarning%5Bdisabled%5D%3Afocus%2C%2Ebtn%2Dwarning%5Bdisabled%5D%3Ahover%2Cfieldset%5Bdisabled%5D%20%2Ebtn%2Dwarning%2Cfieldset%5Bdisabled%5D%20%2Ebtn%2Dwarning%2Eactive%2Cfieldset%5Bdisabled%5D%20%2Ebtn%2Dwarning%2Efocus%2Cfieldset%5Bdisabled%5D%20%2Ebtn%2Dwarning%3Aactive%2Cfieldset%5Bdisabled%5D%20%2Ebtn%2Dwarning%3Afocus%2Cfieldset%5Bdisabled%5D%20%2Ebtn%2Dwarning%3Ahover%7Bbackground%2Dcolor%3A%23f0ad4e%3Bborder%2Dcolor%3A%23eea236%7D%2Ebtn%2Dwarning%20%2Ebadge%7Bcolor%3A%23f0ad4e%3Bbackground%2Dcolor%3A%23fff%7D%2Ebtn%2Ddanger%7Bcolor%3A%23fff%3Bbackground%2Dcolor%3A%23d9534f%3Bborder%2Dcolor%3A%23d43f3a%7D%2Ebtn%2Ddanger%2Efocus%2C%2Ebtn%2Ddanger%3Afocus%7Bcolor%3A%23fff%3Bbackground%2Dcolor%3A%23c9302c%3Bborder%2Dcolor%3A%23761c19%7D%2Ebtn%2Ddanger%3Ahover%7Bcolor%3A%23fff%3Bbackground%2Dcolor%3A%23c9302c%3Bborder%2Dcolor%3A%23ac2925%7D%2Ebtn%2Ddanger%2Eactive%2C%2Ebtn%2Ddanger%3Aactive%2C%2Eopen%3E%2Edropdown%2Dtoggle%2Ebtn%2Ddanger%7Bcolor%3A%23fff%3Bbackground%2Dcolor%3A%23c9302c%3Bborder%2Dcolor%3A%23ac2925%7D%2Ebtn%2Ddanger%2Eactive%2Efocus%2C%2Ebtn%2Ddanger%2Eactive%3Afocus%2C%2Ebtn%2Ddanger%2Eactive%3Ahover%2C%2Ebtn%2Ddanger%3Aactive%2Efocus%2C%2Ebtn%2Ddanger%3Aactive%3Afocus%2C%2Ebtn%2Ddanger%3Aactive%3Ahover%2C%2Eopen%3E%2Edropdown%2Dtoggle%2Ebtn%2Ddanger%2Efocus%2C%2Eopen%3E%2Edropdown%2Dtoggle%2Ebtn%2Ddanger%3Afocus%2C%2Eopen%3E%2Edropdown%2Dtoggle%2Ebtn%2Ddanger%3Ahover%7Bcolor%3A%23fff%3Bbackground%2Dcolor%3A%23ac2925%3Bborder%2Dcolor%3A%23761c19%7D%2Ebtn%2Ddanger%2Eactive%2C%2Ebtn%2Ddanger%3Aactive%2C%2Eopen%3E%2Edropdown%2Dtoggle%2Ebtn%2Ddanger%7Bbackground%2Dimage%3Anone%7D%2Ebtn%2Ddanger%2Edisabled%2C%2Ebtn%2Ddanger%2Edisabled%2Eactive%2C%2Ebtn%2Ddanger%2Edisabled%2Efocus%2C%2Ebtn%2Ddanger%2Edisabled%3Aactive%2C%2Ebtn%2Ddanger%2Edisabled%3Afocus%2C%2Ebtn%2Ddanger%2Edisabled%3Ahover%2C%2Ebtn%2Ddanger%5Bdisabled%5D%2C%2Ebtn%2Ddanger%5Bdisabled%5D%2Eactive%2C%2Ebtn%2Ddanger%5Bdisabled%5D%2Efocus%2C%2Ebtn%2Ddanger%5Bdisabled%5D%3Aactive%2C%2Ebtn%2Ddanger%5Bdisabled%5D%3Afocus%2C%2Ebtn%2Ddanger%5Bdisabled%5D%3Ahover%2Cfieldset%5Bdisabled%5D%20%2Ebtn%2Ddanger%2Cfieldset%5Bdisabled%5D%20%2Ebtn%2Ddanger%2Eactive%2Cfieldset%5Bdisabled%5D%20%2Ebtn%2Ddanger%2Efocus%2Cfieldset%5Bdisabled%5D%20%2Ebtn%2Ddanger%3Aactive%2Cfieldset%5Bdisabled%5D%20%2Ebtn%2Ddanger%3Afocus%2Cfieldset%5Bdisabled%5D%20%2Ebtn%2Ddanger%3Ahover%7Bbackground%2Dcolor%3A%23d9534f%3Bborder%2Dcolor%3A%23d43f3a%7D%2Ebtn%2Ddanger%20%2Ebadge%7Bcolor%3A%23d9534f%3Bbackground%2Dcolor%3A%23fff%7D%2Ebtn%2Dlink%7Bfont%2Dweight%3A400%3Bcolor%3A%23337ab7%3Bborder%2Dradius%3A0%7D%2Ebtn%2Dlink%2C%2Ebtn%2Dlink%2Eactive%2C%2Ebtn%2Dlink%3Aactive%2C%2Ebtn%2Dlink%5Bdisabled%5D%2Cfieldset%5Bdisabled%5D%20%2Ebtn%2Dlink%7Bbackground%2Dcolor%3Atransparent%3B%2Dwebkit%2Dbox%2Dshadow%3Anone%3Bbox%2Dshadow%3Anone%7D%2Ebtn%2Dlink%2C%2Ebtn%2Dlink%3Aactive%2C%2Ebtn%2Dlink%3Afocus%2C%2Ebtn%2Dlink%3Ahover%7Bborder%2Dcolor%3Atransparent%7D%2Ebtn%2Dlink%3Afocus%2C%2Ebtn%2Dlink%3Ahover%7Bcolor%3A%2323527c%3Btext%2Ddecoration%3Aunderline%3Bbackground%2Dcolor%3Atransparent%7D%2Ebtn%2Dlink%5Bdisabled%5D%3Afocus%2C%2Ebtn%2Dlink%5Bdisabled%5D%3Ahover%2Cfieldset%5Bdisabled%5D%20%2Ebtn%2Dlink%3Afocus%2Cfieldset%5Bdisabled%5D%20%2Ebtn%2Dlink%3Ahover%7Bcolor%3A%23777%3Btext%2Ddecoration%3Anone%7D%2Ebtn%2Dgroup%2Dlg%3E%2Ebtn%2C%2Ebtn%2Dlg%7Bpadding%3A10px%2016px%3Bfont%2Dsize%3A18px%3Bline%2Dheight%3A1%2E3333333%3Bborder%2Dradius%3A6px%7D%2Ebtn%2Dgroup%2Dsm%3E%2Ebtn%2C%2Ebtn%2Dsm%7Bpadding%3A5px%2010px%3Bfont%2Dsize%3A12px%3Bline%2Dheight%3A1%2E5%3Bborder%2Dradius%3A3px%7D%2Ebtn%2Dgroup%2Dxs%3E%2Ebtn%2C%2Ebtn%2Dxs%7Bpadding%3A1px%205px%3Bfont%2Dsize%3A12px%3Bline%2Dheight%3A1%2E5%3Bborder%2Dradius%3A3px%7D%2Ebtn%2Dblock%7Bdisplay%3Ablock%3Bwidth%3A100%25%7D%2Ebtn%2Dblock%2B%2Ebtn%2Dblock%7Bmargin%2Dtop%3A5px%7Dinput%5Btype%3Dbutton%5D%2Ebtn%2Dblock%2Cinput%5Btype%3Dreset%5D%2Ebtn%2Dblock%2Cinput%5Btype%3Dsubmit%5D%2Ebtn%2Dblock%7Bwidth%3A100%25%7D%2Efade%7Bopacity%3A0%3B%2Dwebkit%2Dtransition%3Aopacity%20%2E15s%20linear%3B%2Do%2Dtransition%3Aopacity%20%2E15s%20linear%3Btransition%3Aopacity%20%2E15s%20linear%7D%2Efade%2Ein%7Bopacity%3A1%7D%2Ecollapse%7Bdisplay%3Anone%7D%2Ecollapse%2Ein%7Bdisplay%3Ablock%7Dtr%2Ecollapse%2Ein%7Bdisplay%3Atable%2Drow%7Dtbody%2Ecollapse%2Ein%7Bdisplay%3Atable%2Drow%2Dgroup%7D%2Ecollapsing%7Bposition%3Arelative%3Bheight%3A0%3Boverflow%3Ahidden%3B%2Dwebkit%2Dtransition%2Dtiming%2Dfunction%3Aease%3B%2Do%2Dtransition%2Dtiming%2Dfunction%3Aease%3Btransition%2Dtiming%2Dfunction%3Aease%3B%2Dwebkit%2Dtransition%2Dduration%3A%2E35s%3B%2Do%2Dtransition%2Dduration%3A%2E35s%3Btransition%2Dduration%3A%2E35s%3B%2Dwebkit%2Dtransition%2Dproperty%3Aheight%2Cvisibility%3B%2Do%2Dtransition%2Dproperty%3Aheight%2Cvisibility%3Btransition%2Dproperty%3Aheight%2Cvisibility%7D%2Ecaret%7Bdisplay%3Ainline%2Dblock%3Bwidth%3A0%3Bheight%3A0%3Bmargin%2Dleft%3A2px%3Bvertical%2Dalign%3Amiddle%3Bborder%2Dtop%3A4px%20dashed%3Bborder%2Dtop%3A4px%20solid%5C9%3Bborder%2Dright%3A4px%20solid%20transparent%3Bborder%2Dleft%3A4px%20solid%20transparent%7D%2Edropdown%2C%2Edropup%7Bposition%3Arelative%7D%2Edropdown%2Dtoggle%3Afocus%7Boutline%3A0%7D%2Edropdown%2Dmenu%7Bposition%3Aabsolute%3Btop%3A100%25%3Bleft%3A0%3Bz%2Dindex%3A1000%3Bdisplay%3Anone%3Bfloat%3Aleft%3Bmin%2Dwidth%3A160px%3Bpadding%3A5px%200%3Bmargin%3A2px%200%200%3Bfont%2Dsize%3A14px%3Btext%2Dalign%3Aleft%3Blist%2Dstyle%3Anone%3Bbackground%2Dcolor%3A%23fff%3B%2Dwebkit%2Dbackground%2Dclip%3Apadding%2Dbox%3Bbackground%2Dclip%3Apadding%2Dbox%3Bborder%3A1px%20solid%20%23ccc%3Bborder%3A1px%20solid%20rgba%280%2C0%2C0%2C%2E15%29%3Bborder%2Dradius%3A4px%3B%2Dwebkit%2Dbox%2Dshadow%3A0%206px%2012px%20rgba%280%2C0%2C0%2C%2E175%29%3Bbox%2Dshadow%3A0%206px%2012px%20rgba%280%2C0%2C0%2C%2E175%29%7D%2Edropdown%2Dmenu%2Epull%2Dright%7Bright%3A0%3Bleft%3Aauto%7D%2Edropdown%2Dmenu%20%2Edivider%7Bheight%3A1px%3Bmargin%3A9px%200%3Boverflow%3Ahidden%3Bbackground%2Dcolor%3A%23e5e5e5%7D%2Edropdown%2Dmenu%3Eli%3Ea%7Bdisplay%3Ablock%3Bpadding%3A3px%2020px%3Bclear%3Aboth%3Bfont%2Dweight%3A400%3Bline%2Dheight%3A1%2E42857143%3Bcolor%3A%23333%3Bwhite%2Dspace%3Anowrap%7D%2Edropdown%2Dmenu%3Eli%3Ea%3Afocus%2C%2Edropdown%2Dmenu%3Eli%3Ea%3Ahover%7Bcolor%3A%23262626%3Btext%2Ddecoration%3Anone%3Bbackground%2Dcolor%3A%23f5f5f5%7D%2Edropdown%2Dmenu%3E%2Eactive%3Ea%2C%2Edropdown%2Dmenu%3E%2Eactive%3Ea%3Afocus%2C%2Edropdown%2Dmenu%3E%2Eactive%3Ea%3Ahover%7Bcolor%3A%23fff%3Btext%2Ddecoration%3Anone%3Bbackground%2Dcolor%3A%23337ab7%3Boutline%3A0%7D%2Edropdown%2Dmenu%3E%2Edisabled%3Ea%2C%2Edropdown%2Dmenu%3E%2Edisabled%3Ea%3Afocus%2C%2Edropdown%2Dmenu%3E%2Edisabled%3Ea%3Ahover%7Bcolor%3A%23777%7D%2Edropdown%2Dmenu%3E%2Edisabled%3Ea%3Afocus%2C%2Edropdown%2Dmenu%3E%2Edisabled%3Ea%3Ahover%7Btext%2Ddecoration%3Anone%3Bcursor%3Anot%2Dallowed%3Bbackground%2Dcolor%3Atransparent%3Bbackground%2Dimage%3Anone%3Bfilter%3Aprogid%3ADXImageTransform%2EMicrosoft%2Egradient%28enabled%3Dfalse%29%7D%2Eopen%3E%2Edropdown%2Dmenu%7Bdisplay%3Ablock%7D%2Eopen%3Ea%7Boutline%3A0%7D%2Edropdown%2Dmenu%2Dright%7Bright%3A0%3Bleft%3Aauto%7D%2Edropdown%2Dmenu%2Dleft%7Bright%3Aauto%3Bleft%3A0%7D%2Edropdown%2Dheader%7Bdisplay%3Ablock%3Bpadding%3A3px%2020px%3Bfont%2Dsize%3A12px%3Bline%2Dheight%3A1%2E42857143%3Bcolor%3A%23777%3Bwhite%2Dspace%3Anowrap%7D%2Edropdown%2Dbackdrop%7Bposition%3Afixed%3Btop%3A0%3Bright%3A0%3Bbottom%3A0%3Bleft%3A0%3Bz%2Dindex%3A990%7D%2Epull%2Dright%3E%2Edropdown%2Dmenu%7Bright%3A0%3Bleft%3Aauto%7D%2Edropup%20%2Ecaret%2C%2Enavbar%2Dfixed%2Dbottom%20%2Edropdown%20%2Ecaret%7Bcontent%3A%22%22%3Bborder%2Dtop%3A0%3Bborder%2Dbottom%3A4px%20dashed%3Bborder%2Dbottom%3A4px%20solid%5C9%7D%2Edropup%20%2Edropdown%2Dmenu%2C%2Enavbar%2Dfixed%2Dbottom%20%2Edropdown%20%2Edropdown%2Dmenu%7Btop%3Aauto%3Bbottom%3A100%25%3Bmargin%2Dbottom%3A2px%7D%40media%20%28min%2Dwidth%3A768px%29%7B%2Enavbar%2Dright%20%2Edropdown%2Dmenu%7Bright%3A0%3Bleft%3Aauto%7D%2Enavbar%2Dright%20%2Edropdown%2Dmenu%2Dleft%7Bright%3Aauto%3Bleft%3A0%7D%7D%2Ebtn%2Dgroup%2C%2Ebtn%2Dgroup%2Dvertical%7Bposition%3Arelative%3Bdisplay%3Ainline%2Dblock%3Bvertical%2Dalign%3Amiddle%7D%2Ebtn%2Dgroup%2Dvertical%3E%2Ebtn%2C%2Ebtn%2Dgroup%3E%2Ebtn%7Bposition%3Arelative%3Bfloat%3Aleft%7D%2Ebtn%2Dgroup%2Dvertical%3E%2Ebtn%2Eactive%2C%2Ebtn%2Dgroup%2Dvertical%3E%2Ebtn%3Aactive%2C%2Ebtn%2Dgroup%2Dvertical%3E%2Ebtn%3Afocus%2C%2Ebtn%2Dgroup%2Dvertical%3E%2Ebtn%3Ahover%2C%2Ebtn%2Dgroup%3E%2Ebtn%2Eactive%2C%2Ebtn%2Dgroup%3E%2Ebtn%3Aactive%2C%2Ebtn%2Dgroup%3E%2Ebtn%3Afocus%2C%2Ebtn%2Dgroup%3E%2Ebtn%3Ahover%7Bz%2Dindex%3A2%7D%2Ebtn%2Dgroup%20%2Ebtn%2B%2Ebtn%2C%2Ebtn%2Dgroup%20%2Ebtn%2B%2Ebtn%2Dgroup%2C%2Ebtn%2Dgroup%20%2Ebtn%2Dgroup%2B%2Ebtn%2C%2Ebtn%2Dgroup%20%2Ebtn%2Dgroup%2B%2Ebtn%2Dgroup%7Bmargin%2Dleft%3A%2D1px%7D%2Ebtn%2Dtoolbar%7Bmargin%2Dleft%3A%2D5px%7D%2Ebtn%2Dtoolbar%20%2Ebtn%2C%2Ebtn%2Dtoolbar%20%2Ebtn%2Dgroup%2C%2Ebtn%2Dtoolbar%20%2Einput%2Dgroup%7Bfloat%3Aleft%7D%2Ebtn%2Dtoolbar%3E%2Ebtn%2C%2Ebtn%2Dtoolbar%3E%2Ebtn%2Dgroup%2C%2Ebtn%2Dtoolbar%3E%2Einput%2Dgroup%7Bmargin%2Dleft%3A5px%7D%2Ebtn%2Dgroup%3E%2Ebtn%3Anot%28%3Afirst%2Dchild%29%3Anot%28%3Alast%2Dchild%29%3Anot%28%2Edropdown%2Dtoggle%29%7Bborder%2Dradius%3A0%7D%2Ebtn%2Dgroup%3E%2Ebtn%3Afirst%2Dchild%7Bmargin%2Dleft%3A0%7D%2Ebtn%2Dgroup%3E%2Ebtn%3Afirst%2Dchild%3Anot%28%3Alast%2Dchild%29%3Anot%28%2Edropdown%2Dtoggle%29%7Bborder%2Dtop%2Dright%2Dradius%3A0%3Bborder%2Dbottom%2Dright%2Dradius%3A0%7D%2Ebtn%2Dgroup%3E%2Ebtn%3Alast%2Dchild%3Anot%28%3Afirst%2Dchild%29%2C%2Ebtn%2Dgroup%3E%2Edropdown%2Dtoggle%3Anot%28%3Afirst%2Dchild%29%7Bborder%2Dtop%2Dleft%2Dradius%3A0%3Bborder%2Dbottom%2Dleft%2Dradius%3A0%7D%2Ebtn%2Dgroup%3E%2Ebtn%2Dgroup%7Bfloat%3Aleft%7D%2Ebtn%2Dgroup%3E%2Ebtn%2Dgroup%3Anot%28%3Afirst%2Dchild%29%3Anot%28%3Alast%2Dchild%29%3E%2Ebtn%7Bborder%2Dradius%3A0%7D%2Ebtn%2Dgroup%3E%2Ebtn%2Dgroup%3Afirst%2Dchild%3Anot%28%3Alast%2Dchild%29%3E%2Ebtn%3Alast%2Dchild%2C%2Ebtn%2Dgroup%3E%2Ebtn%2Dgroup%3Afirst%2Dchild%3Anot%28%3Alast%2Dchild%29%3E%2Edropdown%2Dtoggle%7Bborder%2Dtop%2Dright%2Dradius%3A0%3Bborder%2Dbottom%2Dright%2Dradius%3A0%7D%2Ebtn%2Dgroup%3E%2Ebtn%2Dgroup%3Alast%2Dchild%3Anot%28%3Afirst%2Dchild%29%3E%2Ebtn%3Afirst%2Dchild%7Bborder%2Dtop%2Dleft%2Dradius%3A0%3Bborder%2Dbottom%2Dleft%2Dradius%3A0%7D%2Ebtn%2Dgroup%20%2Edropdown%2Dtoggle%3Aactive%2C%2Ebtn%2Dgroup%2Eopen%20%2Edropdown%2Dtoggle%7Boutline%3A0%7D%2Ebtn%2Dgroup%3E%2Ebtn%2B%2Edropdown%2Dtoggle%7Bpadding%2Dright%3A8px%3Bpadding%2Dleft%3A8px%7D%2Ebtn%2Dgroup%3E%2Ebtn%2Dlg%2B%2Edropdown%2Dtoggle%7Bpadding%2Dright%3A12px%3Bpadding%2Dleft%3A12px%7D%2Ebtn%2Dgroup%2Eopen%20%2Edropdown%2Dtoggle%7B%2Dwebkit%2Dbox%2Dshadow%3Ainset%200%203px%205px%20rgba%280%2C0%2C0%2C%2E125%29%3Bbox%2Dshadow%3Ainset%200%203px%205px%20rgba%280%2C0%2C0%2C%2E125%29%7D%2Ebtn%2Dgroup%2Eopen%20%2Edropdown%2Dtoggle%2Ebtn%2Dlink%7B%2Dwebkit%2Dbox%2Dshadow%3Anone%3Bbox%2Dshadow%3Anone%7D%2Ebtn%20%2Ecaret%7Bmargin%2Dleft%3A0%7D%2Ebtn%2Dlg%20%2Ecaret%7Bborder%2Dwidth%3A5px%205px%200%3Bborder%2Dbottom%2Dwidth%3A0%7D%2Edropup%20%2Ebtn%2Dlg%20%2Ecaret%7Bborder%2Dwidth%3A0%205px%205px%7D%2Ebtn%2Dgroup%2Dvertical%3E%2Ebtn%2C%2Ebtn%2Dgroup%2Dvertical%3E%2Ebtn%2Dgroup%2C%2Ebtn%2Dgroup%2Dvertical%3E%2Ebtn%2Dgroup%3E%2Ebtn%7Bdisplay%3Ablock%3Bfloat%3Anone%3Bwidth%3A100%25%3Bmax%2Dwidth%3A100%25%7D%2Ebtn%2Dgroup%2Dvertical%3E%2Ebtn%2Dgroup%3E%2Ebtn%7Bfloat%3Anone%7D%2Ebtn%2Dgroup%2Dvertical%3E%2Ebtn%2B%2Ebtn%2C%2Ebtn%2Dgroup%2Dvertical%3E%2Ebtn%2B%2Ebtn%2Dgroup%2C%2Ebtn%2Dgroup%2Dvertical%3E%2Ebtn%2Dgroup%2B%2Ebtn%2C%2Ebtn%2Dgroup%2Dvertical%3E%2Ebtn%2Dgroup%2B%2Ebtn%2Dgroup%7Bmargin%2Dtop%3A%2D1px%3Bmargin%2Dleft%3A0%7D%2Ebtn%2Dgroup%2Dvertical%3E%2Ebtn%3Anot%28%3Afirst%2Dchild%29%3Anot%28%3Alast%2Dchild%29%7Bborder%2Dradius%3A0%7D%2Ebtn%2Dgroup%2Dvertical%3E%2Ebtn%3Afirst%2Dchild%3Anot%28%3Alast%2Dchild%29%7Bborder%2Dtop%2Dright%2Dradius%3A4px%3Bborder%2Dbottom%2Dright%2Dradius%3A0%3Bborder%2Dbottom%2Dleft%2Dradius%3A0%7D%2Ebtn%2Dgroup%2Dvertical%3E%2Ebtn%3Alast%2Dchild%3Anot%28%3Afirst%2Dchild%29%7Bborder%2Dtop%2Dleft%2Dradius%3A0%3Bborder%2Dtop%2Dright%2Dradius%3A0%3Bborder%2Dbottom%2Dleft%2Dradius%3A4px%7D%2Ebtn%2Dgroup%2Dvertical%3E%2Ebtn%2Dgroup%3Anot%28%3Afirst%2Dchild%29%3Anot%28%3Alast%2Dchild%29%3E%2Ebtn%7Bborder%2Dradius%3A0%7D%2Ebtn%2Dgroup%2Dvertical%3E%2Ebtn%2Dgroup%3Afirst%2Dchild%3Anot%28%3Alast%2Dchild%29%3E%2Ebtn%3Alast%2Dchild%2C%2Ebtn%2Dgroup%2Dvertical%3E%2Ebtn%2Dgroup%3Afirst%2Dchild%3Anot%28%3Alast%2Dchild%29%3E%2Edropdown%2Dtoggle%7Bborder%2Dbottom%2Dright%2Dradius%3A0%3Bborder%2Dbottom%2Dleft%2Dradius%3A0%7D%2Ebtn%2Dgroup%2Dvertical%3E%2Ebtn%2Dgroup%3Alast%2Dchild%3Anot%28%3Afirst%2Dchild%29%3E%2Ebtn%3Afirst%2Dchild%7Bborder%2Dtop%2Dleft%2Dradius%3A0%3Bborder%2Dtop%2Dright%2Dradius%3A0%7D%2Ebtn%2Dgroup%2Djustified%7Bdisplay%3Atable%3Bwidth%3A100%25%3Btable%2Dlayout%3Afixed%3Bborder%2Dcollapse%3Aseparate%7D%2Ebtn%2Dgroup%2Djustified%3E%2Ebtn%2C%2Ebtn%2Dgroup%2Djustified%3E%2Ebtn%2Dgroup%7Bdisplay%3Atable%2Dcell%3Bfloat%3Anone%3Bwidth%3A1%25%7D%2Ebtn%2Dgroup%2Djustified%3E%2Ebtn%2Dgroup%20%2Ebtn%7Bwidth%3A100%25%7D%2Ebtn%2Dgroup%2Djustified%3E%2Ebtn%2Dgroup%20%2Edropdown%2Dmenu%7Bleft%3Aauto%7D%5Bdata%2Dtoggle%3Dbuttons%5D%3E%2Ebtn%20input%5Btype%3Dcheckbox%5D%2C%5Bdata%2Dtoggle%3Dbuttons%5D%3E%2Ebtn%20input%5Btype%3Dradio%5D%2C%5Bdata%2Dtoggle%3Dbuttons%5D%3E%2Ebtn%2Dgroup%3E%2Ebtn%20input%5Btype%3Dcheckbox%5D%2C%5Bdata%2Dtoggle%3Dbuttons%5D%3E%2Ebtn%2Dgroup%3E%2Ebtn%20input%5Btype%3Dradio%5D%7Bposition%3Aabsolute%3Bclip%3Arect%280%2C0%2C0%2C0%29%3Bpointer%2Devents%3Anone%7D%2Einput%2Dgroup%7Bposition%3Arelative%3Bdisplay%3Atable%3Bborder%2Dcollapse%3Aseparate%7D%2Einput%2Dgroup%5Bclass%2A%3Dcol%2D%5D%7Bfloat%3Anone%3Bpadding%2Dright%3A0%3Bpadding%2Dleft%3A0%7D%2Einput%2Dgroup%20%2Eform%2Dcontrol%7Bposition%3Arelative%3Bz%2Dindex%3A2%3Bfloat%3Aleft%3Bwidth%3A100%25%3Bmargin%2Dbottom%3A0%7D%2Einput%2Dgroup%2Dlg%3E%2Eform%2Dcontrol%2C%2Einput%2Dgroup%2Dlg%3E%2Einput%2Dgroup%2Daddon%2C%2Einput%2Dgroup%2Dlg%3E%2Einput%2Dgroup%2Dbtn%3E%2Ebtn%7Bheight%3A46px%3Bpadding%3A10px%2016px%3Bfont%2Dsize%3A18px%3Bline%2Dheight%3A1%2E3333333%3Bborder%2Dradius%3A6px%7Dselect%2Einput%2Dgroup%2Dlg%3E%2Eform%2Dcontrol%2Cselect%2Einput%2Dgroup%2Dlg%3E%2Einput%2Dgroup%2Daddon%2Cselect%2Einput%2Dgroup%2Dlg%3E%2Einput%2Dgroup%2Dbtn%3E%2Ebtn%7Bheight%3A46px%3Bline%2Dheight%3A46px%7Dselect%5Bmultiple%5D%2Einput%2Dgroup%2Dlg%3E%2Eform%2Dcontrol%2Cselect%5Bmultiple%5D%2Einput%2Dgroup%2Dlg%3E%2Einput%2Dgroup%2Daddon%2Cselect%5Bmultiple%5D%2Einput%2Dgroup%2Dlg%3E%2Einput%2Dgroup%2Dbtn%3E%2Ebtn%2Ctextarea%2Einput%2Dgroup%2Dlg%3E%2Eform%2Dcontrol%2Ctextarea%2Einput%2Dgroup%2Dlg%3E%2Einput%2Dgroup%2Daddon%2Ctextarea%2Einput%2Dgroup%2Dlg%3E%2Einput%2Dgroup%2Dbtn%3E%2Ebtn%7Bheight%3Aauto%7D%2Einput%2Dgroup%2Dsm%3E%2Eform%2Dcontrol%2C%2Einput%2Dgroup%2Dsm%3E%2Einput%2Dgroup%2Daddon%2C%2Einput%2Dgroup%2Dsm%3E%2Einput%2Dgroup%2Dbtn%3E%2Ebtn%7Bheight%3A30px%3Bpadding%3A5px%2010px%3Bfont%2Dsize%3A12px%3Bline%2Dheight%3A1%2E5%3Bborder%2Dradius%3A3px%7Dselect%2Einput%2Dgroup%2Dsm%3E%2Eform%2Dcontrol%2Cselect%2Einput%2Dgroup%2Dsm%3E%2Einput%2Dgroup%2Daddon%2Cselect%2Einput%2Dgroup%2Dsm%3E%2Einput%2Dgroup%2Dbtn%3E%2Ebtn%7Bheight%3A30px%3Bline%2Dheight%3A30px%7Dselect%5Bmultiple%5D%2Einput%2Dgroup%2Dsm%3E%2Eform%2Dcontrol%2Cselect%5Bmultiple%5D%2Einput%2Dgroup%2Dsm%3E%2Einput%2Dgroup%2Daddon%2Cselect%5Bmultiple%5D%2Einput%2Dgroup%2Dsm%3E%2Einput%2Dgroup%2Dbtn%3E%2Ebtn%2Ctextarea%2Einput%2Dgroup%2Dsm%3E%2Eform%2Dcontrol%2Ctextarea%2Einput%2Dgroup%2Dsm%3E%2Einput%2Dgroup%2Daddon%2Ctextarea%2Einput%2Dgroup%2Dsm%3E%2Einput%2Dgroup%2Dbtn%3E%2Ebtn%7Bheight%3Aauto%7D%2Einput%2Dgroup%20%2Eform%2Dcontrol%2C%2Einput%2Dgroup%2Daddon%2C%2Einput%2Dgroup%2Dbtn%7Bdisplay%3Atable%2Dcell%7D%2Einput%2Dgroup%20%2Eform%2Dcontrol%3Anot%28%3Afirst%2Dchild%29%3Anot%28%3Alast%2Dchild%29%2C%2Einput%2Dgroup%2Daddon%3Anot%28%3Afirst%2Dchild%29%3Anot%28%3Alast%2Dchild%29%2C%2Einput%2Dgroup%2Dbtn%3Anot%28%3Afirst%2Dchild%29%3Anot%28%3Alast%2Dchild%29%7Bborder%2Dradius%3A0%7D%2Einput%2Dgroup%2Daddon%2C%2Einput%2Dgroup%2Dbtn%7Bwidth%3A1%25%3Bwhite%2Dspace%3Anowrap%3Bvertical%2Dalign%3Amiddle%7D%2Einput%2Dgroup%2Daddon%7Bpadding%3A6px%2012px%3Bfont%2Dsize%3A14px%3Bfont%2Dweight%3A400%3Bline%2Dheight%3A1%3Bcolor%3A%23555%3Btext%2Dalign%3Acenter%3Bbackground%2Dcolor%3A%23eee%3Bborder%3A1px%20solid%20%23ccc%3Bborder%2Dradius%3A4px%7D%2Einput%2Dgroup%2Daddon%2Einput%2Dsm%7Bpadding%3A5px%2010px%3Bfont%2Dsize%3A12px%3Bborder%2Dradius%3A3px%7D%2Einput%2Dgroup%2Daddon%2Einput%2Dlg%7Bpadding%3A10px%2016px%3Bfont%2Dsize%3A18px%3Bborder%2Dradius%3A6px%7D%2Einput%2Dgroup%2Daddon%20input%5Btype%3Dcheckbox%5D%2C%2Einput%2Dgroup%2Daddon%20input%5Btype%3Dradio%5D%7Bmargin%2Dtop%3A0%7D%2Einput%2Dgroup%20%2Eform%2Dcontrol%3Afirst%2Dchild%2C%2Einput%2Dgroup%2Daddon%3Afirst%2Dchild%2C%2Einput%2Dgroup%2Dbtn%3Afirst%2Dchild%3E%2Ebtn%2C%2Einput%2Dgroup%2Dbtn%3Afirst%2Dchild%3E%2Ebtn%2Dgroup%3E%2Ebtn%2C%2Einput%2Dgroup%2Dbtn%3Afirst%2Dchild%3E%2Edropdown%2Dtoggle%2C%2Einput%2Dgroup%2Dbtn%3Alast%2Dchild%3E%2Ebtn%2Dgroup%3Anot%28%3Alast%2Dchild%29%3E%2Ebtn%2C%2Einput%2Dgroup%2Dbtn%3Alast%2Dchild%3E%2Ebtn%3Anot%28%3Alast%2Dchild%29%3Anot%28%2Edropdown%2Dtoggle%29%7Bborder%2Dtop%2Dright%2Dradius%3A0%3Bborder%2Dbottom%2Dright%2Dradius%3A0%7D%2Einput%2Dgroup%2Daddon%3Afirst%2Dchild%7Bborder%2Dright%3A0%7D%2Einput%2Dgroup%20%2Eform%2Dcontrol%3Alast%2Dchild%2C%2Einput%2Dgroup%2Daddon%3Alast%2Dchild%2C%2Einput%2Dgroup%2Dbtn%3Afirst%2Dchild%3E%2Ebtn%2Dgroup%3Anot%28%3Afirst%2Dchild%29%3E%2Ebtn%2C%2Einput%2Dgroup%2Dbtn%3Afirst%2Dchild%3E%2Ebtn%3Anot%28%3Afirst%2Dchild%29%2C%2Einput%2Dgroup%2Dbtn%3Alast%2Dchild%3E%2Ebtn%2C%2Einput%2Dgroup%2Dbtn%3Alast%2Dchild%3E%2Ebtn%2Dgroup%3E%2Ebtn%2C%2Einput%2Dgroup%2Dbtn%3Alast%2Dchild%3E%2Edropdown%2Dtoggle%7Bborder%2Dtop%2Dleft%2Dradius%3A0%3Bborder%2Dbottom%2Dleft%2Dradius%3A0%7D%2Einput%2Dgroup%2Daddon%3Alast%2Dchild%7Bborder%2Dleft%3A0%7D%2Einput%2Dgroup%2Dbtn%7Bposition%3Arelative%3Bfont%2Dsize%3A0%3Bwhite%2Dspace%3Anowrap%7D%2Einput%2Dgroup%2Dbtn%3E%2Ebtn%7Bposition%3Arelative%7D%2Einput%2Dgroup%2Dbtn%3E%2Ebtn%2B%2Ebtn%7Bmargin%2Dleft%3A%2D1px%7D%2Einput%2Dgroup%2Dbtn%3E%2Ebtn%3Aactive%2C%2Einput%2Dgroup%2Dbtn%3E%2Ebtn%3Afocus%2C%2Einput%2Dgroup%2Dbtn%3E%2Ebtn%3Ahover%7Bz%2Dindex%3A2%7D%2Einput%2Dgroup%2Dbtn%3Afirst%2Dchild%3E%2Ebtn%2C%2Einput%2Dgroup%2Dbtn%3Afirst%2Dchild%3E%2Ebtn%2Dgroup%7Bmargin%2Dright%3A%2D1px%7D%2Einput%2Dgroup%2Dbtn%3Alast%2Dchild%3E%2Ebtn%2C%2Einput%2Dgroup%2Dbtn%3Alast%2Dchild%3E%2Ebtn%2Dgroup%7Bz%2Dindex%3A2%3Bmargin%2Dleft%3A%2D1px%7D%2Enav%7Bpadding%2Dleft%3A0%3Bmargin%2Dbottom%3A0%3Blist%2Dstyle%3Anone%7D%2Enav%3Eli%7Bposition%3Arelative%3Bdisplay%3Ablock%7D%2Enav%3Eli%3Ea%7Bposition%3Arelative%3Bdisplay%3Ablock%3Bpadding%3A10px%2015px%7D%2Enav%3Eli%3Ea%3Afocus%2C%2Enav%3Eli%3Ea%3Ahover%7Btext%2Ddecoration%3Anone%3Bbackground%2Dcolor%3A%23eee%7D%2Enav%3Eli%2Edisabled%3Ea%7Bcolor%3A%23777%7D%2Enav%3Eli%2Edisabled%3Ea%3Afocus%2C%2Enav%3Eli%2Edisabled%3Ea%3Ahover%7Bcolor%3A%23777%3Btext%2Ddecoration%3Anone%3Bcursor%3Anot%2Dallowed%3Bbackground%2Dcolor%3Atransparent%7D%2Enav%20%2Eopen%3Ea%2C%2Enav%20%2Eopen%3Ea%3Afocus%2C%2Enav%20%2Eopen%3Ea%3Ahover%7Bbackground%2Dcolor%3A%23eee%3Bborder%2Dcolor%3A%23337ab7%7D%2Enav%20%2Enav%2Ddivider%7Bheight%3A1px%3Bmargin%3A9px%200%3Boverflow%3Ahidden%3Bbackground%2Dcolor%3A%23e5e5e5%7D%2Enav%3Eli%3Ea%3Eimg%7Bmax%2Dwidth%3Anone%7D%2Enav%2Dtabs%7Bborder%2Dbottom%3A1px%20solid%20%23ddd%7D%2Enav%2Dtabs%3Eli%7Bfloat%3Aleft%3Bmargin%2Dbottom%3A%2D1px%7D%2Enav%2Dtabs%3Eli%3Ea%7Bmargin%2Dright%3A2px%3Bline%2Dheight%3A1%2E42857143%3Bborder%3A1px%20solid%20transparent%3Bborder%2Dradius%3A4px%204px%200%200%7D%2Enav%2Dtabs%3Eli%3Ea%3Ahover%7Bborder%2Dcolor%3A%23eee%20%23eee%20%23ddd%7D%2Enav%2Dtabs%3Eli%2Eactive%3Ea%2C%2Enav%2Dtabs%3Eli%2Eactive%3Ea%3Afocus%2C%2Enav%2Dtabs%3Eli%2Eactive%3Ea%3Ahover%7Bcolor%3A%23555%3Bcursor%3Adefault%3Bbackground%2Dcolor%3A%23fff%3Bborder%3A1px%20solid%20%23ddd%3Bborder%2Dbottom%2Dcolor%3Atransparent%7D%2Enav%2Dtabs%2Enav%2Djustified%7Bwidth%3A100%25%3Bborder%2Dbottom%3A0%7D%2Enav%2Dtabs%2Enav%2Djustified%3Eli%7Bfloat%3Anone%7D%2Enav%2Dtabs%2Enav%2Djustified%3Eli%3Ea%7Bmargin%2Dbottom%3A5px%3Btext%2Dalign%3Acenter%7D%2Enav%2Dtabs%2Enav%2Djustified%3E%2Edropdown%20%2Edropdown%2Dmenu%7Btop%3Aauto%3Bleft%3Aauto%7D%40media%20%28min%2Dwidth%3A768px%29%7B%2Enav%2Dtabs%2Enav%2Djustified%3Eli%7Bdisplay%3Atable%2Dcell%3Bwidth%3A1%25%7D%2Enav%2Dtabs%2Enav%2Djustified%3Eli%3Ea%7Bmargin%2Dbottom%3A0%7D%7D%2Enav%2Dtabs%2Enav%2Djustified%3Eli%3Ea%7Bmargin%2Dright%3A0%3Bborder%2Dradius%3A4px%7D%2Enav%2Dtabs%2Enav%2Djustified%3E%2Eactive%3Ea%2C%2Enav%2Dtabs%2Enav%2Djustified%3E%2Eactive%3Ea%3Afocus%2C%2Enav%2Dtabs%2Enav%2Djustified%3E%2Eactive%3Ea%3Ahover%7Bborder%3A1px%20solid%20%23ddd%7D%40media%20%28min%2Dwidth%3A768px%29%7B%2Enav%2Dtabs%2Enav%2Djustified%3Eli%3Ea%7Bborder%2Dbottom%3A1px%20solid%20%23ddd%3Bborder%2Dradius%3A4px%204px%200%200%7D%2Enav%2Dtabs%2Enav%2Djustified%3E%2Eactive%3Ea%2C%2Enav%2Dtabs%2Enav%2Djustified%3E%2Eactive%3Ea%3Afocus%2C%2Enav%2Dtabs%2Enav%2Djustified%3E%2Eactive%3Ea%3Ahover%7Bborder%2Dbottom%2Dcolor%3A%23fff%7D%7D%2Enav%2Dpills%3Eli%7Bfloat%3Aleft%7D%2Enav%2Dpills%3Eli%3Ea%7Bborder%2Dradius%3A4px%7D%2Enav%2Dpills%3Eli%2Bli%7Bmargin%2Dleft%3A2px%7D%2Enav%2Dpills%3Eli%2Eactive%3Ea%2C%2Enav%2Dpills%3Eli%2Eactive%3Ea%3Afocus%2C%2Enav%2Dpills%3Eli%2Eactive%3Ea%3Ahover%7Bcolor%3A%23fff%3Bbackground%2Dcolor%3A%23337ab7%7D%2Enav%2Dstacked%3Eli%7Bfloat%3Anone%7D%2Enav%2Dstacked%3Eli%2Bli%7Bmargin%2Dtop%3A2px%3Bmargin%2Dleft%3A0%7D%2Enav%2Djustified%7Bwidth%3A100%25%7D%2Enav%2Djustified%3Eli%7Bfloat%3Anone%7D%2Enav%2Djustified%3Eli%3Ea%7Bmargin%2Dbottom%3A5px%3Btext%2Dalign%3Acenter%7D%2Enav%2Djustified%3E%2Edropdown%20%2Edropdown%2Dmenu%7Btop%3Aauto%3Bleft%3Aauto%7D%40media%20%28min%2Dwidth%3A768px%29%7B%2Enav%2Djustified%3Eli%7Bdisplay%3Atable%2Dcell%3Bwidth%3A1%25%7D%2Enav%2Djustified%3Eli%3Ea%7Bmargin%2Dbottom%3A0%7D%7D%2Enav%2Dtabs%2Djustified%7Bborder%2Dbottom%3A0%7D%2Enav%2Dtabs%2Djustified%3Eli%3Ea%7Bmargin%2Dright%3A0%3Bborder%2Dradius%3A4px%7D%2Enav%2Dtabs%2Djustified%3E%2Eactive%3Ea%2C%2Enav%2Dtabs%2Djustified%3E%2Eactive%3Ea%3Afocus%2C%2Enav%2Dtabs%2Djustified%3E%2Eactive%3Ea%3Ahover%7Bborder%3A1px%20solid%20%23ddd%7D%40media%20%28min%2Dwidth%3A768px%29%7B%2Enav%2Dtabs%2Djustified%3Eli%3Ea%7Bborder%2Dbottom%3A1px%20solid%20%23ddd%3Bborder%2Dradius%3A4px%204px%200%200%7D%2Enav%2Dtabs%2Djustified%3E%2Eactive%3Ea%2C%2Enav%2Dtabs%2Djustified%3E%2Eactive%3Ea%3Afocus%2C%2Enav%2Dtabs%2Djustified%3E%2Eactive%3Ea%3Ahover%7Bborder%2Dbottom%2Dcolor%3A%23fff%7D%7D%2Etab%2Dcontent%3E%2Etab%2Dpane%7Bdisplay%3Anone%7D%2Etab%2Dcontent%3E%2Eactive%7Bdisplay%3Ablock%7D%2Enav%2Dtabs%20%2Edropdown%2Dmenu%7Bmargin%2Dtop%3A%2D1px%3Bborder%2Dtop%2Dleft%2Dradius%3A0%3Bborder%2Dtop%2Dright%2Dradius%3A0%7D%2Enavbar%7Bposition%3Arelative%3Bmin%2Dheight%3A50px%3Bmargin%2Dbottom%3A20px%3Bborder%3A1px%20solid%20transparent%7D%40media%20%28min%2Dwidth%3A768px%29%7B%2Enavbar%7Bborder%2Dradius%3A4px%7D%7D%40media%20%28min%2Dwidth%3A768px%29%7B%2Enavbar%2Dheader%7Bfloat%3Aleft%7D%7D%2Enavbar%2Dcollapse%7Bpadding%2Dright%3A15px%3Bpadding%2Dleft%3A15px%3Boverflow%2Dx%3Avisible%3B%2Dwebkit%2Doverflow%2Dscrolling%3Atouch%3Bborder%2Dtop%3A1px%20solid%20transparent%3B%2Dwebkit%2Dbox%2Dshadow%3Ainset%200%201px%200%20rgba%28255%2C255%2C255%2C%2E1%29%3Bbox%2Dshadow%3Ainset%200%201px%200%20rgba%28255%2C255%2C255%2C%2E1%29%7D%2Enavbar%2Dcollapse%2Ein%7Boverflow%2Dy%3Aauto%7D%40media%20%28min%2Dwidth%3A768px%29%7B%2Enavbar%2Dcollapse%7Bwidth%3Aauto%3Bborder%2Dtop%3A0%3B%2Dwebkit%2Dbox%2Dshadow%3Anone%3Bbox%2Dshadow%3Anone%7D%2Enavbar%2Dcollapse%2Ecollapse%7Bdisplay%3Ablock%21important%3Bheight%3Aauto%21important%3Bpadding%2Dbottom%3A0%3Boverflow%3Avisible%21important%7D%2Enavbar%2Dcollapse%2Ein%7Boverflow%2Dy%3Avisible%7D%2Enavbar%2Dfixed%2Dbottom%20%2Enavbar%2Dcollapse%2C%2Enavbar%2Dfixed%2Dtop%20%2Enavbar%2Dcollapse%2C%2Enavbar%2Dstatic%2Dtop%20%2Enavbar%2Dcollapse%7Bpadding%2Dright%3A0%3Bpadding%2Dleft%3A0%7D%7D%2Enavbar%2Dfixed%2Dbottom%20%2Enavbar%2Dcollapse%2C%2Enavbar%2Dfixed%2Dtop%20%2Enavbar%2Dcollapse%7Bmax%2Dheight%3A340px%7D%40media%20%28max%2Ddevice%2Dwidth%3A480px%29%20and%20%28orientation%3Alandscape%29%7B%2Enavbar%2Dfixed%2Dbottom%20%2Enavbar%2Dcollapse%2C%2Enavbar%2Dfixed%2Dtop%20%2Enavbar%2Dcollapse%7Bmax%2Dheight%3A200px%7D%7D%2Econtainer%2Dfluid%3E%2Enavbar%2Dcollapse%2C%2Econtainer%2Dfluid%3E%2Enavbar%2Dheader%2C%2Econtainer%3E%2Enavbar%2Dcollapse%2C%2Econtainer%3E%2Enavbar%2Dheader%7Bmargin%2Dright%3A%2D15px%3Bmargin%2Dleft%3A%2D15px%7D%40media%20%28min%2Dwidth%3A768px%29%7B%2Econtainer%2Dfluid%3E%2Enavbar%2Dcollapse%2C%2Econtainer%2Dfluid%3E%2Enavbar%2Dheader%2C%2Econtainer%3E%2Enavbar%2Dcollapse%2C%2Econtainer%3E%2Enavbar%2Dheader%7Bmargin%2Dright%3A0%3Bmargin%2Dleft%3A0%7D%7D%2Enavbar%2Dstatic%2Dtop%7Bz%2Dindex%3A1000%3Bborder%2Dwidth%3A0%200%201px%7D%40media%20%28min%2Dwidth%3A768px%29%7B%2Enavbar%2Dstatic%2Dtop%7Bborder%2Dradius%3A0%7D%7D%2Enavbar%2Dfixed%2Dbottom%2C%2Enavbar%2Dfixed%2Dtop%7Bposition%3Afixed%3Bright%3A0%3Bleft%3A0%3Bz%2Dindex%3A1030%7D%40media%20%28min%2Dwidth%3A768px%29%7B%2Enavbar%2Dfixed%2Dbottom%2C%2Enavbar%2Dfixed%2Dtop%7Bborder%2Dradius%3A0%7D%7D%2Enavbar%2Dfixed%2Dtop%7Btop%3A0%3Bborder%2Dwidth%3A0%200%201px%7D%2Enavbar%2Dfixed%2Dbottom%7Bbottom%3A0%3Bmargin%2Dbottom%3A0%3Bborder%2Dwidth%3A1px%200%200%7D%2Enavbar%2Dbrand%7Bfloat%3Aleft%3Bheight%3A50px%3Bpadding%3A15px%2015px%3Bfont%2Dsize%3A18px%3Bline%2Dheight%3A20px%7D%2Enavbar%2Dbrand%3Afocus%2C%2Enavbar%2Dbrand%3Ahover%7Btext%2Ddecoration%3Anone%7D%2Enavbar%2Dbrand%3Eimg%7Bdisplay%3Ablock%7D%40media%20%28min%2Dwidth%3A768px%29%7B%2Enavbar%3E%2Econtainer%20%2Enavbar%2Dbrand%2C%2Enavbar%3E%2Econtainer%2Dfluid%20%2Enavbar%2Dbrand%7Bmargin%2Dleft%3A%2D15px%7D%7D%2Enavbar%2Dtoggle%7Bposition%3Arelative%3Bfloat%3Aright%3Bpadding%3A9px%2010px%3Bmargin%2Dtop%3A8px%3Bmargin%2Dright%3A15px%3Bmargin%2Dbottom%3A8px%3Bbackground%2Dcolor%3Atransparent%3Bbackground%2Dimage%3Anone%3Bborder%3A1px%20solid%20transparent%3Bborder%2Dradius%3A4px%7D%2Enavbar%2Dtoggle%3Afocus%7Boutline%3A0%7D%2Enavbar%2Dtoggle%20%2Eicon%2Dbar%7Bdisplay%3Ablock%3Bwidth%3A22px%3Bheight%3A2px%3Bborder%2Dradius%3A1px%7D%2Enavbar%2Dtoggle%20%2Eicon%2Dbar%2B%2Eicon%2Dbar%7Bmargin%2Dtop%3A4px%7D%40media%20%28min%2Dwidth%3A768px%29%7B%2Enavbar%2Dtoggle%7Bdisplay%3Anone%7D%7D%2Enavbar%2Dnav%7Bmargin%3A7%2E5px%20%2D15px%7D%2Enavbar%2Dnav%3Eli%3Ea%7Bpadding%2Dtop%3A10px%3Bpadding%2Dbottom%3A10px%3Bline%2Dheight%3A20px%7D%40media%20%28max%2Dwidth%3A767px%29%7B%2Enavbar%2Dnav%20%2Eopen%20%2Edropdown%2Dmenu%7Bposition%3Astatic%3Bfloat%3Anone%3Bwidth%3Aauto%3Bmargin%2Dtop%3A0%3Bbackground%2Dcolor%3Atransparent%3Bborder%3A0%3B%2Dwebkit%2Dbox%2Dshadow%3Anone%3Bbox%2Dshadow%3Anone%7D%2Enavbar%2Dnav%20%2Eopen%20%2Edropdown%2Dmenu%20%2Edropdown%2Dheader%2C%2Enavbar%2Dnav%20%2Eopen%20%2Edropdown%2Dmenu%3Eli%3Ea%7Bpadding%3A5px%2015px%205px%2025px%7D%2Enavbar%2Dnav%20%2Eopen%20%2Edropdown%2Dmenu%3Eli%3Ea%7Bline%2Dheight%3A20px%7D%2Enavbar%2Dnav%20%2Eopen%20%2Edropdown%2Dmenu%3Eli%3Ea%3Afocus%2C%2Enavbar%2Dnav%20%2Eopen%20%2Edropdown%2Dmenu%3Eli%3Ea%3Ahover%7Bbackground%2Dimage%3Anone%7D%7D%40media%20%28min%2Dwidth%3A768px%29%7B%2Enavbar%2Dnav%7Bfloat%3Aleft%3Bmargin%3A0%7D%2Enavbar%2Dnav%3Eli%7Bfloat%3Aleft%7D%2Enavbar%2Dnav%3Eli%3Ea%7Bpadding%2Dtop%3A15px%3Bpadding%2Dbottom%3A15px%7D%7D%2Enavbar%2Dform%7Bpadding%3A10px%2015px%3Bmargin%2Dtop%3A8px%3Bmargin%2Dright%3A%2D15px%3Bmargin%2Dbottom%3A8px%3Bmargin%2Dleft%3A%2D15px%3Bborder%2Dtop%3A1px%20solid%20transparent%3Bborder%2Dbottom%3A1px%20solid%20transparent%3B%2Dwebkit%2Dbox%2Dshadow%3Ainset%200%201px%200%20rgba%28255%2C255%2C255%2C%2E1%29%2C0%201px%200%20rgba%28255%2C255%2C255%2C%2E1%29%3Bbox%2Dshadow%3Ainset%200%201px%200%20rgba%28255%2C255%2C255%2C%2E1%29%2C0%201px%200%20rgba%28255%2C255%2C255%2C%2E1%29%7D%40media%20%28min%2Dwidth%3A768px%29%7B%2Enavbar%2Dform%20%2Eform%2Dgroup%7Bdisplay%3Ainline%2Dblock%3Bmargin%2Dbottom%3A0%3Bvertical%2Dalign%3Amiddle%7D%2Enavbar%2Dform%20%2Eform%2Dcontrol%7Bdisplay%3Ainline%2Dblock%3Bwidth%3Aauto%3Bvertical%2Dalign%3Amiddle%7D%2Enavbar%2Dform%20%2Eform%2Dcontrol%2Dstatic%7Bdisplay%3Ainline%2Dblock%7D%2Enavbar%2Dform%20%2Einput%2Dgroup%7Bdisplay%3Ainline%2Dtable%3Bvertical%2Dalign%3Amiddle%7D%2Enavbar%2Dform%20%2Einput%2Dgroup%20%2Eform%2Dcontrol%2C%2Enavbar%2Dform%20%2Einput%2Dgroup%20%2Einput%2Dgroup%2Daddon%2C%2Enavbar%2Dform%20%2Einput%2Dgroup%20%2Einput%2Dgroup%2Dbtn%7Bwidth%3Aauto%7D%2Enavbar%2Dform%20%2Einput%2Dgroup%3E%2Eform%2Dcontrol%7Bwidth%3A100%25%7D%2Enavbar%2Dform%20%2Econtrol%2Dlabel%7Bmargin%2Dbottom%3A0%3Bvertical%2Dalign%3Amiddle%7D%2Enavbar%2Dform%20%2Echeckbox%2C%2Enavbar%2Dform%20%2Eradio%7Bdisplay%3Ainline%2Dblock%3Bmargin%2Dtop%3A0%3Bmargin%2Dbottom%3A0%3Bvertical%2Dalign%3Amiddle%7D%2Enavbar%2Dform%20%2Echeckbox%20label%2C%2Enavbar%2Dform%20%2Eradio%20label%7Bpadding%2Dleft%3A0%7D%2Enavbar%2Dform%20%2Echeckbox%20input%5Btype%3Dcheckbox%5D%2C%2Enavbar%2Dform%20%2Eradio%20input%5Btype%3Dradio%5D%7Bposition%3Arelative%3Bmargin%2Dleft%3A0%7D%2Enavbar%2Dform%20%2Ehas%2Dfeedback%20%2Eform%2Dcontrol%2Dfeedback%7Btop%3A0%7D%7D%40media%20%28max%2Dwidth%3A767px%29%7B%2Enavbar%2Dform%20%2Eform%2Dgroup%7Bmargin%2Dbottom%3A5px%7D%2Enavbar%2Dform%20%2Eform%2Dgroup%3Alast%2Dchild%7Bmargin%2Dbottom%3A0%7D%7D%40media%20%28min%2Dwidth%3A768px%29%7B%2Enavbar%2Dform%7Bwidth%3Aauto%3Bpadding%2Dtop%3A0%3Bpadding%2Dbottom%3A0%3Bmargin%2Dright%3A0%3Bmargin%2Dleft%3A0%3Bborder%3A0%3B%2Dwebkit%2Dbox%2Dshadow%3Anone%3Bbox%2Dshadow%3Anone%7D%7D%2Enavbar%2Dnav%3Eli%3E%2Edropdown%2Dmenu%7Bmargin%2Dtop%3A0%3Bborder%2Dtop%2Dleft%2Dradius%3A0%3Bborder%2Dtop%2Dright%2Dradius%3A0%7D%2Enavbar%2Dfixed%2Dbottom%20%2Enavbar%2Dnav%3Eli%3E%2Edropdown%2Dmenu%7Bmargin%2Dbottom%3A0%3Bborder%2Dtop%2Dleft%2Dradius%3A4px%3Bborder%2Dtop%2Dright%2Dradius%3A4px%3Bborder%2Dbottom%2Dright%2Dradius%3A0%3Bborder%2Dbottom%2Dleft%2Dradius%3A0%7D%2Enavbar%2Dbtn%7Bmargin%2Dtop%3A8px%3Bmargin%2Dbottom%3A8px%7D%2Enavbar%2Dbtn%2Ebtn%2Dsm%7Bmargin%2Dtop%3A10px%3Bmargin%2Dbottom%3A10px%7D%2Enavbar%2Dbtn%2Ebtn%2Dxs%7Bmargin%2Dtop%3A14px%3Bmargin%2Dbottom%3A14px%7D%2Enavbar%2Dtext%7Bmargin%2Dtop%3A15px%3Bmargin%2Dbottom%3A15px%7D%40media%20%28min%2Dwidth%3A768px%29%7B%2Enavbar%2Dtext%7Bfloat%3Aleft%3Bmargin%2Dright%3A15px%3Bmargin%2Dleft%3A15px%7D%7D%40media%20%28min%2Dwidth%3A768px%29%7B%2Enavbar%2Dleft%7Bfloat%3Aleft%21important%7D%2Enavbar%2Dright%7Bfloat%3Aright%21important%3Bmargin%2Dright%3A%2D15px%7D%2Enavbar%2Dright%7E%2Enavbar%2Dright%7Bmargin%2Dright%3A0%7D%7D%2Enavbar%2Ddefault%7Bbackground%2Dcolor%3A%23f8f8f8%3Bborder%2Dcolor%3A%23e7e7e7%7D%2Enavbar%2Ddefault%20%2Enavbar%2Dbrand%7Bcolor%3A%23777%7D%2Enavbar%2Ddefault%20%2Enavbar%2Dbrand%3Afocus%2C%2Enavbar%2Ddefault%20%2Enavbar%2Dbrand%3Ahover%7Bcolor%3A%235e5e5e%3Bbackground%2Dcolor%3Atransparent%7D%2Enavbar%2Ddefault%20%2Enavbar%2Dtext%7Bcolor%3A%23777%7D%2Enavbar%2Ddefault%20%2Enavbar%2Dnav%3Eli%3Ea%7Bcolor%3A%23777%7D%2Enavbar%2Ddefault%20%2Enavbar%2Dnav%3Eli%3Ea%3Afocus%2C%2Enavbar%2Ddefault%20%2Enavbar%2Dnav%3Eli%3Ea%3Ahover%7Bcolor%3A%23333%3Bbackground%2Dcolor%3Atransparent%7D%2Enavbar%2Ddefault%20%2Enavbar%2Dnav%3E%2Eactive%3Ea%2C%2Enavbar%2Ddefault%20%2Enavbar%2Dnav%3E%2Eactive%3Ea%3Afocus%2C%2Enavbar%2Ddefault%20%2Enavbar%2Dnav%3E%2Eactive%3Ea%3Ahover%7Bcolor%3A%23555%3Bbackground%2Dcolor%3A%23e7e7e7%7D%2Enavbar%2Ddefault%20%2Enavbar%2Dnav%3E%2Edisabled%3Ea%2C%2Enavbar%2Ddefault%20%2Enavbar%2Dnav%3E%2Edisabled%3Ea%3Afocus%2C%2Enavbar%2Ddefault%20%2Enavbar%2Dnav%3E%2Edisabled%3Ea%3Ahover%7Bcolor%3A%23ccc%3Bbackground%2Dcolor%3Atransparent%7D%2Enavbar%2Ddefault%20%2Enavbar%2Dtoggle%7Bborder%2Dcolor%3A%23ddd%7D%2Enavbar%2Ddefault%20%2Enavbar%2Dtoggle%3Afocus%2C%2Enavbar%2Ddefault%20%2Enavbar%2Dtoggle%3Ahover%7Bbackground%2Dcolor%3A%23ddd%7D%2Enavbar%2Ddefault%20%2Enavbar%2Dtoggle%20%2Eicon%2Dbar%7Bbackground%2Dcolor%3A%23888%7D%2Enavbar%2Ddefault%20%2Enavbar%2Dcollapse%2C%2Enavbar%2Ddefault%20%2Enavbar%2Dform%7Bborder%2Dcolor%3A%23e7e7e7%7D%2Enavbar%2Ddefault%20%2Enavbar%2Dnav%3E%2Eopen%3Ea%2C%2Enavbar%2Ddefault%20%2Enavbar%2Dnav%3E%2Eopen%3Ea%3Afocus%2C%2Enavbar%2Ddefault%20%2Enavbar%2Dnav%3E%2Eopen%3Ea%3Ahover%7Bcolor%3A%23555%3Bbackground%2Dcolor%3A%23e7e7e7%7D%40media%20%28max%2Dwidth%3A767px%29%7B%2Enavbar%2Ddefault%20%2Enavbar%2Dnav%20%2Eopen%20%2Edropdown%2Dmenu%3Eli%3Ea%7Bcolor%3A%23777%7D%2Enavbar%2Ddefault%20%2Enavbar%2Dnav%20%2Eopen%20%2Edropdown%2Dmenu%3Eli%3Ea%3Afocus%2C%2Enavbar%2Ddefault%20%2Enavbar%2Dnav%20%2Eopen%20%2Edropdown%2Dmenu%3Eli%3Ea%3Ahover%7Bcolor%3A%23333%3Bbackground%2Dcolor%3Atransparent%7D%2Enavbar%2Ddefault%20%2Enavbar%2Dnav%20%2Eopen%20%2Edropdown%2Dmenu%3E%2Eactive%3Ea%2C%2Enavbar%2Ddefault%20%2Enavbar%2Dnav%20%2Eopen%20%2Edropdown%2Dmenu%3E%2Eactive%3Ea%3Afocus%2C%2Enavbar%2Ddefault%20%2Enavbar%2Dnav%20%2Eopen%20%2Edropdown%2Dmenu%3E%2Eactive%3Ea%3Ahover%7Bcolor%3A%23555%3Bbackground%2Dcolor%3A%23e7e7e7%7D%2Enavbar%2Ddefault%20%2Enavbar%2Dnav%20%2Eopen%20%2Edropdown%2Dmenu%3E%2Edisabled%3Ea%2C%2Enavbar%2Ddefault%20%2Enavbar%2Dnav%20%2Eopen%20%2Edropdown%2Dmenu%3E%2Edisabled%3Ea%3Afocus%2C%2Enavbar%2Ddefault%20%2Enavbar%2Dnav%20%2Eopen%20%2Edropdown%2Dmenu%3E%2Edisabled%3Ea%3Ahover%7Bcolor%3A%23ccc%3Bbackground%2Dcolor%3Atransparent%7D%7D%2Enavbar%2Ddefault%20%2Enavbar%2Dlink%7Bcolor%3A%23777%7D%2Enavbar%2Ddefault%20%2Enavbar%2Dlink%3Ahover%7Bcolor%3A%23333%7D%2Enavbar%2Ddefault%20%2Ebtn%2Dlink%7Bcolor%3A%23777%7D%2Enavbar%2Ddefault%20%2Ebtn%2Dlink%3Afocus%2C%2Enavbar%2Ddefault%20%2Ebtn%2Dlink%3Ahover%7Bcolor%3A%23333%7D%2Enavbar%2Ddefault%20%2Ebtn%2Dlink%5Bdisabled%5D%3Afocus%2C%2Enavbar%2Ddefault%20%2Ebtn%2Dlink%5Bdisabled%5D%3Ahover%2Cfieldset%5Bdisabled%5D%20%2Enavbar%2Ddefault%20%2Ebtn%2Dlink%3Afocus%2Cfieldset%5Bdisabled%5D%20%2Enavbar%2Ddefault%20%2Ebtn%2Dlink%3Ahover%7Bcolor%3A%23ccc%7D%2Enavbar%2Dinverse%7Bbackground%2Dcolor%3A%23222%3Bborder%2Dcolor%3A%23080808%7D%2Enavbar%2Dinverse%20%2Enavbar%2Dbrand%7Bcolor%3A%239d9d9d%7D%2Enavbar%2Dinverse%20%2Enavbar%2Dbrand%3Afocus%2C%2Enavbar%2Dinverse%20%2Enavbar%2Dbrand%3Ahover%7Bcolor%3A%23fff%3Bbackground%2Dcolor%3Atransparent%7D%2Enavbar%2Dinverse%20%2Enavbar%2Dtext%7Bcolor%3A%239d9d9d%7D%2Enavbar%2Dinverse%20%2Enavbar%2Dnav%3Eli%3Ea%7Bcolor%3A%239d9d9d%7D%2Enavbar%2Dinverse%20%2Enavbar%2Dnav%3Eli%3Ea%3Afocus%2C%2Enavbar%2Dinverse%20%2Enavbar%2Dnav%3Eli%3Ea%3Ahover%7Bcolor%3A%23fff%3Bbackground%2Dcolor%3Atransparent%7D%2Enavbar%2Dinverse%20%2Enavbar%2Dnav%3E%2Eactive%3Ea%2C%2Enavbar%2Dinverse%20%2Enavbar%2Dnav%3E%2Eactive%3Ea%3Afocus%2C%2Enavbar%2Dinverse%20%2Enavbar%2Dnav%3E%2Eactive%3Ea%3Ahover%7Bcolor%3A%23fff%3Bbackground%2Dcolor%3A%23080808%7D%2Enavbar%2Dinverse%20%2Enavbar%2Dnav%3E%2Edisabled%3Ea%2C%2Enavbar%2Dinverse%20%2Enavbar%2Dnav%3E%2Edisabled%3Ea%3Afocus%2C%2Enavbar%2Dinverse%20%2Enavbar%2Dnav%3E%2Edisabled%3Ea%3Ahover%7Bcolor%3A%23444%3Bbackground%2Dcolor%3Atransparent%7D%2Enavbar%2Dinverse%20%2Enavbar%2Dtoggle%7Bborder%2Dcolor%3A%23333%7D%2Enavbar%2Dinverse%20%2Enavbar%2Dtoggle%3Afocus%2C%2Enavbar%2Dinverse%20%2Enavbar%2Dtoggle%3Ahover%7Bbackground%2Dcolor%3A%23333%7D%2Enavbar%2Dinverse%20%2Enavbar%2Dtoggle%20%2Eicon%2Dbar%7Bbackground%2Dcolor%3A%23fff%7D%2Enavbar%2Dinverse%20%2Enavbar%2Dcollapse%2C%2Enavbar%2Dinverse%20%2Enavbar%2Dform%7Bborder%2Dcolor%3A%23101010%7D%2Enavbar%2Dinverse%20%2Enavbar%2Dnav%3E%2Eopen%3Ea%2C%2Enavbar%2Dinverse%20%2Enavbar%2Dnav%3E%2Eopen%3Ea%3Afocus%2C%2Enavbar%2Dinverse%20%2Enavbar%2Dnav%3E%2Eopen%3Ea%3Ahover%7Bcolor%3A%23fff%3Bbackground%2Dcolor%3A%23080808%7D%40media%20%28max%2Dwidth%3A767px%29%7B%2Enavbar%2Dinverse%20%2Enavbar%2Dnav%20%2Eopen%20%2Edropdown%2Dmenu%3E%2Edropdown%2Dheader%7Bborder%2Dcolor%3A%23080808%7D%2Enavbar%2Dinverse%20%2Enavbar%2Dnav%20%2Eopen%20%2Edropdown%2Dmenu%20%2Edivider%7Bbackground%2Dcolor%3A%23080808%7D%2Enavbar%2Dinverse%20%2Enavbar%2Dnav%20%2Eopen%20%2Edropdown%2Dmenu%3Eli%3Ea%7Bcolor%3A%239d9d9d%7D%2Enavbar%2Dinverse%20%2Enavbar%2Dnav%20%2Eopen%20%2Edropdown%2Dmenu%3Eli%3Ea%3Afocus%2C%2Enavbar%2Dinverse%20%2Enavbar%2Dnav%20%2Eopen%20%2Edropdown%2Dmenu%3Eli%3Ea%3Ahover%7Bcolor%3A%23fff%3Bbackground%2Dcolor%3Atransparent%7D%2Enavbar%2Dinverse%20%2Enavbar%2Dnav%20%2Eopen%20%2Edropdown%2Dmenu%3E%2Eactive%3Ea%2C%2Enavbar%2Dinverse%20%2Enavbar%2Dnav%20%2Eopen%20%2Edropdown%2Dmenu%3E%2Eactive%3Ea%3Afocus%2C%2Enavbar%2Dinverse%20%2Enavbar%2Dnav%20%2Eopen%20%2Edropdown%2Dmenu%3E%2Eactive%3Ea%3Ahover%7Bcolor%3A%23fff%3Bbackground%2Dcolor%3A%23080808%7D%2Enavbar%2Dinverse%20%2Enavbar%2Dnav%20%2Eopen%20%2Edropdown%2Dmenu%3E%2Edisabled%3Ea%2C%2Enavbar%2Dinverse%20%2Enavbar%2Dnav%20%2Eopen%20%2Edropdown%2Dmenu%3E%2Edisabled%3Ea%3Afocus%2C%2Enavbar%2Dinverse%20%2Enavbar%2Dnav%20%2Eopen%20%2Edropdown%2Dmenu%3E%2Edisabled%3Ea%3Ahover%7Bcolor%3A%23444%3Bbackground%2Dcolor%3Atransparent%7D%7D%2Enavbar%2Dinverse%20%2Enavbar%2Dlink%7Bcolor%3A%239d9d9d%7D%2Enavbar%2Dinverse%20%2Enavbar%2Dlink%3Ahover%7Bcolor%3A%23fff%7D%2Enavbar%2Dinverse%20%2Ebtn%2Dlink%7Bcolor%3A%239d9d9d%7D%2Enavbar%2Dinverse%20%2Ebtn%2Dlink%3Afocus%2C%2Enavbar%2Dinverse%20%2Ebtn%2Dlink%3Ahover%7Bcolor%3A%23fff%7D%2Enavbar%2Dinverse%20%2Ebtn%2Dlink%5Bdisabled%5D%3Afocus%2C%2Enavbar%2Dinverse%20%2Ebtn%2Dlink%5Bdisabled%5D%3Ahover%2Cfieldset%5Bdisabled%5D%20%2Enavbar%2Dinverse%20%2Ebtn%2Dlink%3Afocus%2Cfieldset%5Bdisabled%5D%20%2Enavbar%2Dinverse%20%2Ebtn%2Dlink%3Ahover%7Bcolor%3A%23444%7D%2Ebreadcrumb%7Bpadding%3A8px%2015px%3Bmargin%2Dbottom%3A20px%3Blist%2Dstyle%3Anone%3Bbackground%2Dcolor%3A%23f5f5f5%3Bborder%2Dradius%3A4px%7D%2Ebreadcrumb%3Eli%7Bdisplay%3Ainline%2Dblock%7D%2Ebreadcrumb%3Eli%2Bli%3Abefore%7Bpadding%3A0%205px%3Bcolor%3A%23ccc%3Bcontent%3A%22%2F%5C00a0%22%7D%2Ebreadcrumb%3E%2Eactive%7Bcolor%3A%23777%7D%2Epagination%7Bdisplay%3Ainline%2Dblock%3Bpadding%2Dleft%3A0%3Bmargin%3A20px%200%3Bborder%2Dradius%3A4px%7D%2Epagination%3Eli%7Bdisplay%3Ainline%7D%2Epagination%3Eli%3Ea%2C%2Epagination%3Eli%3Espan%7Bposition%3Arelative%3Bfloat%3Aleft%3Bpadding%3A6px%2012px%3Bmargin%2Dleft%3A%2D1px%3Bline%2Dheight%3A1%2E42857143%3Bcolor%3A%23337ab7%3Btext%2Ddecoration%3Anone%3Bbackground%2Dcolor%3A%23fff%3Bborder%3A1px%20solid%20%23ddd%7D%2Epagination%3Eli%3Afirst%2Dchild%3Ea%2C%2Epagination%3Eli%3Afirst%2Dchild%3Espan%7Bmargin%2Dleft%3A0%3Bborder%2Dtop%2Dleft%2Dradius%3A4px%3Bborder%2Dbottom%2Dleft%2Dradius%3A4px%7D%2Epagination%3Eli%3Alast%2Dchild%3Ea%2C%2Epagination%3Eli%3Alast%2Dchild%3Espan%7Bborder%2Dtop%2Dright%2Dradius%3A4px%3Bborder%2Dbottom%2Dright%2Dradius%3A4px%7D%2Epagination%3Eli%3Ea%3Afocus%2C%2Epagination%3Eli%3Ea%3Ahover%2C%2Epagination%3Eli%3Espan%3Afocus%2C%2Epagination%3Eli%3Espan%3Ahover%7Bz%2Dindex%3A3%3Bcolor%3A%2323527c%3Bbackground%2Dcolor%3A%23eee%3Bborder%2Dcolor%3A%23ddd%7D%2Epagination%3E%2Eactive%3Ea%2C%2Epagination%3E%2Eactive%3Ea%3Afocus%2C%2Epagination%3E%2Eactive%3Ea%3Ahover%2C%2Epagination%3E%2Eactive%3Espan%2C%2Epagination%3E%2Eactive%3Espan%3Afocus%2C%2Epagination%3E%2Eactive%3Espan%3Ahover%7Bz%2Dindex%3A2%3Bcolor%3A%23fff%3Bcursor%3Adefault%3Bbackground%2Dcolor%3A%23337ab7%3Bborder%2Dcolor%3A%23337ab7%7D%2Epagination%3E%2Edisabled%3Ea%2C%2Epagination%3E%2Edisabled%3Ea%3Afocus%2C%2Epagination%3E%2Edisabled%3Ea%3Ahover%2C%2Epagination%3E%2Edisabled%3Espan%2C%2Epagination%3E%2Edisabled%3Espan%3Afocus%2C%2Epagination%3E%2Edisabled%3Espan%3Ahover%7Bcolor%3A%23777%3Bcursor%3Anot%2Dallowed%3Bbackground%2Dcolor%3A%23fff%3Bborder%2Dcolor%3A%23ddd%7D%2Epagination%2Dlg%3Eli%3Ea%2C%2Epagination%2Dlg%3Eli%3Espan%7Bpadding%3A10px%2016px%3Bfont%2Dsize%3A18px%3Bline%2Dheight%3A1%2E3333333%7D%2Epagination%2Dlg%3Eli%3Afirst%2Dchild%3Ea%2C%2Epagination%2Dlg%3Eli%3Afirst%2Dchild%3Espan%7Bborder%2Dtop%2Dleft%2Dradius%3A6px%3Bborder%2Dbottom%2Dleft%2Dradius%3A6px%7D%2Epagination%2Dlg%3Eli%3Alast%2Dchild%3Ea%2C%2Epagination%2Dlg%3Eli%3Alast%2Dchild%3Espan%7Bborder%2Dtop%2Dright%2Dradius%3A6px%3Bborder%2Dbottom%2Dright%2Dradius%3A6px%7D%2Epagination%2Dsm%3Eli%3Ea%2C%2Epagination%2Dsm%3Eli%3Espan%7Bpadding%3A5px%2010px%3Bfont%2Dsize%3A12px%3Bline%2Dheight%3A1%2E5%7D%2Epagination%2Dsm%3Eli%3Afirst%2Dchild%3Ea%2C%2Epagination%2Dsm%3Eli%3Afirst%2Dchild%3Espan%7Bborder%2Dtop%2Dleft%2Dradius%3A3px%3Bborder%2Dbottom%2Dleft%2Dradius%3A3px%7D%2Epagination%2Dsm%3Eli%3Alast%2Dchild%3Ea%2C%2Epagination%2Dsm%3Eli%3Alast%2Dchild%3Espan%7Bborder%2Dtop%2Dright%2Dradius%3A3px%3Bborder%2Dbottom%2Dright%2Dradius%3A3px%7D%2Epager%7Bpadding%2Dleft%3A0%3Bmargin%3A20px%200%3Btext%2Dalign%3Acenter%3Blist%2Dstyle%3Anone%7D%2Epager%20li%7Bdisplay%3Ainline%7D%2Epager%20li%3Ea%2C%2Epager%20li%3Espan%7Bdisplay%3Ainline%2Dblock%3Bpadding%3A5px%2014px%3Bbackground%2Dcolor%3A%23fff%3Bborder%3A1px%20solid%20%23ddd%3Bborder%2Dradius%3A15px%7D%2Epager%20li%3Ea%3Afocus%2C%2Epager%20li%3Ea%3Ahover%7Btext%2Ddecoration%3Anone%3Bbackground%2Dcolor%3A%23eee%7D%2Epager%20%2Enext%3Ea%2C%2Epager%20%2Enext%3Espan%7Bfloat%3Aright%7D%2Epager%20%2Eprevious%3Ea%2C%2Epager%20%2Eprevious%3Espan%7Bfloat%3Aleft%7D%2Epager%20%2Edisabled%3Ea%2C%2Epager%20%2Edisabled%3Ea%3Afocus%2C%2Epager%20%2Edisabled%3Ea%3Ahover%2C%2Epager%20%2Edisabled%3Espan%7Bcolor%3A%23777%3Bcursor%3Anot%2Dallowed%3Bbackground%2Dcolor%3A%23fff%7D%2Elabel%7Bdisplay%3Ainline%3Bpadding%3A%2E2em%20%2E6em%20%2E3em%3Bfont%2Dsize%3A75%25%3Bfont%2Dweight%3A700%3Bline%2Dheight%3A1%3Bcolor%3A%23fff%3Btext%2Dalign%3Acenter%3Bwhite%2Dspace%3Anowrap%3Bvertical%2Dalign%3Abaseline%3Bborder%2Dradius%3A%2E25em%7Da%2Elabel%3Afocus%2Ca%2Elabel%3Ahover%7Bcolor%3A%23fff%3Btext%2Ddecoration%3Anone%3Bcursor%3Apointer%7D%2Elabel%3Aempty%7Bdisplay%3Anone%7D%2Ebtn%20%2Elabel%7Bposition%3Arelative%3Btop%3A%2D1px%7D%2Elabel%2Ddefault%7Bbackground%2Dcolor%3A%23777%7D%2Elabel%2Ddefault%5Bhref%5D%3Afocus%2C%2Elabel%2Ddefault%5Bhref%5D%3Ahover%7Bbackground%2Dcolor%3A%235e5e5e%7D%2Elabel%2Dprimary%7Bbackground%2Dcolor%3A%23337ab7%7D%2Elabel%2Dprimary%5Bhref%5D%3Afocus%2C%2Elabel%2Dprimary%5Bhref%5D%3Ahover%7Bbackground%2Dcolor%3A%23286090%7D%2Elabel%2Dsuccess%7Bbackground%2Dcolor%3A%235cb85c%7D%2Elabel%2Dsuccess%5Bhref%5D%3Afocus%2C%2Elabel%2Dsuccess%5Bhref%5D%3Ahover%7Bbackground%2Dcolor%3A%23449d44%7D%2Elabel%2Dinfo%7Bbackground%2Dcolor%3A%235bc0de%7D%2Elabel%2Dinfo%5Bhref%5D%3Afocus%2C%2Elabel%2Dinfo%5Bhref%5D%3Ahover%7Bbackground%2Dcolor%3A%2331b0d5%7D%2Elabel%2Dwarning%7Bbackground%2Dcolor%3A%23f0ad4e%7D%2Elabel%2Dwarning%5Bhref%5D%3Afocus%2C%2Elabel%2Dwarning%5Bhref%5D%3Ahover%7Bbackground%2Dcolor%3A%23ec971f%7D%2Elabel%2Ddanger%7Bbackground%2Dcolor%3A%23d9534f%7D%2Elabel%2Ddanger%5Bhref%5D%3Afocus%2C%2Elabel%2Ddanger%5Bhref%5D%3Ahover%7Bbackground%2Dcolor%3A%23c9302c%7D%2Ebadge%7Bdisplay%3Ainline%2Dblock%3Bmin%2Dwidth%3A10px%3Bpadding%3A3px%207px%3Bfont%2Dsize%3A12px%3Bfont%2Dweight%3A700%3Bline%2Dheight%3A1%3Bcolor%3A%23fff%3Btext%2Dalign%3Acenter%3Bwhite%2Dspace%3Anowrap%3Bvertical%2Dalign%3Amiddle%3Bbackground%2Dcolor%3A%23777%3Bborder%2Dradius%3A10px%7D%2Ebadge%3Aempty%7Bdisplay%3Anone%7D%2Ebtn%20%2Ebadge%7Bposition%3Arelative%3Btop%3A%2D1px%7D%2Ebtn%2Dgroup%2Dxs%3E%2Ebtn%20%2Ebadge%2C%2Ebtn%2Dxs%20%2Ebadge%7Btop%3A0%3Bpadding%3A1px%205px%7Da%2Ebadge%3Afocus%2Ca%2Ebadge%3Ahover%7Bcolor%3A%23fff%3Btext%2Ddecoration%3Anone%3Bcursor%3Apointer%7D%2Elist%2Dgroup%2Ditem%2Eactive%3E%2Ebadge%2C%2Enav%2Dpills%3E%2Eactive%3Ea%3E%2Ebadge%7Bcolor%3A%23337ab7%3Bbackground%2Dcolor%3A%23fff%7D%2Elist%2Dgroup%2Ditem%3E%2Ebadge%7Bfloat%3Aright%7D%2Elist%2Dgroup%2Ditem%3E%2Ebadge%2B%2Ebadge%7Bmargin%2Dright%3A5px%7D%2Enav%2Dpills%3Eli%3Ea%3E%2Ebadge%7Bmargin%2Dleft%3A3px%7D%2Ejumbotron%7Bpadding%2Dtop%3A30px%3Bpadding%2Dbottom%3A30px%3Bmargin%2Dbottom%3A30px%3Bcolor%3Ainherit%3Bbackground%2Dcolor%3A%23eee%7D%2Ejumbotron%20%2Eh1%2C%2Ejumbotron%20h1%7Bcolor%3Ainherit%7D%2Ejumbotron%20p%7Bmargin%2Dbottom%3A15px%3Bfont%2Dsize%3A21px%3Bfont%2Dweight%3A200%7D%2Ejumbotron%3Ehr%7Bborder%2Dtop%2Dcolor%3A%23d5d5d5%7D%2Econtainer%20%2Ejumbotron%2C%2Econtainer%2Dfluid%20%2Ejumbotron%7Bborder%2Dradius%3A6px%7D%2Ejumbotron%20%2Econtainer%7Bmax%2Dwidth%3A100%25%7D%40media%20screen%20and%20%28min%2Dwidth%3A768px%29%7B%2Ejumbotron%7Bpadding%2Dtop%3A48px%3Bpadding%2Dbottom%3A48px%7D%2Econtainer%20%2Ejumbotron%2C%2Econtainer%2Dfluid%20%2Ejumbotron%7Bpadding%2Dright%3A60px%3Bpadding%2Dleft%3A60px%7D%2Ejumbotron%20%2Eh1%2C%2Ejumbotron%20h1%7Bfont%2Dsize%3A63px%7D%7D%2Ethumbnail%7Bdisplay%3Ablock%3Bpadding%3A4px%3Bmargin%2Dbottom%3A20px%3Bline%2Dheight%3A1%2E42857143%3Bbackground%2Dcolor%3A%23fff%3Bborder%3A1px%20solid%20%23ddd%3Bborder%2Dradius%3A4px%3B%2Dwebkit%2Dtransition%3Aborder%20%2E2s%20ease%2Din%2Dout%3B%2Do%2Dtransition%3Aborder%20%2E2s%20ease%2Din%2Dout%3Btransition%3Aborder%20%2E2s%20ease%2Din%2Dout%7D%2Ethumbnail%20a%3Eimg%2C%2Ethumbnail%3Eimg%7Bmargin%2Dright%3Aauto%3Bmargin%2Dleft%3Aauto%7Da%2Ethumbnail%2Eactive%2Ca%2Ethumbnail%3Afocus%2Ca%2Ethumbnail%3Ahover%7Bborder%2Dcolor%3A%23337ab7%7D%2Ethumbnail%20%2Ecaption%7Bpadding%3A9px%3Bcolor%3A%23333%7D%2Ealert%7Bpadding%3A15px%3Bmargin%2Dbottom%3A20px%3Bborder%3A1px%20solid%20transparent%3Bborder%2Dradius%3A4px%7D%2Ealert%20h4%7Bmargin%2Dtop%3A0%3Bcolor%3Ainherit%7D%2Ealert%20%2Ealert%2Dlink%7Bfont%2Dweight%3A700%7D%2Ealert%3Ep%2C%2Ealert%3Eul%7Bmargin%2Dbottom%3A0%7D%2Ealert%3Ep%2Bp%7Bmargin%2Dtop%3A5px%7D%2Ealert%2Ddismissable%2C%2Ealert%2Ddismissible%7Bpadding%2Dright%3A35px%7D%2Ealert%2Ddismissable%20%2Eclose%2C%2Ealert%2Ddismissible%20%2Eclose%7Bposition%3Arelative%3Btop%3A%2D2px%3Bright%3A%2D21px%3Bcolor%3Ainherit%7D%2Ealert%2Dsuccess%7Bcolor%3A%233c763d%3Bbackground%2Dcolor%3A%23dff0d8%3Bborder%2Dcolor%3A%23d6e9c6%7D%2Ealert%2Dsuccess%20hr%7Bborder%2Dtop%2Dcolor%3A%23c9e2b3%7D%2Ealert%2Dsuccess%20%2Ealert%2Dlink%7Bcolor%3A%232b542c%7D%2Ealert%2Dinfo%7Bcolor%3A%2331708f%3Bbackground%2Dcolor%3A%23d9edf7%3Bborder%2Dcolor%3A%23bce8f1%7D%2Ealert%2Dinfo%20hr%7Bborder%2Dtop%2Dcolor%3A%23a6e1ec%7D%2Ealert%2Dinfo%20%2Ealert%2Dlink%7Bcolor%3A%23245269%7D%2Ealert%2Dwarning%7Bcolor%3A%238a6d3b%3Bbackground%2Dcolor%3A%23fcf8e3%3Bborder%2Dcolor%3A%23faebcc%7D%2Ealert%2Dwarning%20hr%7Bborder%2Dtop%2Dcolor%3A%23f7e1b5%7D%2Ealert%2Dwarning%20%2Ealert%2Dlink%7Bcolor%3A%2366512c%7D%2Ealert%2Ddanger%7Bcolor%3A%23a94442%3Bbackground%2Dcolor%3A%23f2dede%3Bborder%2Dcolor%3A%23ebccd1%7D%2Ealert%2Ddanger%20hr%7Bborder%2Dtop%2Dcolor%3A%23e4b9c0%7D%2Ealert%2Ddanger%20%2Ealert%2Dlink%7Bcolor%3A%23843534%7D%40%2Dwebkit%2Dkeyframes%20progress%2Dbar%2Dstripes%7Bfrom%7Bbackground%2Dposition%3A40px%200%7Dto%7Bbackground%2Dposition%3A0%200%7D%7D%40%2Do%2Dkeyframes%20progress%2Dbar%2Dstripes%7Bfrom%7Bbackground%2Dposition%3A40px%200%7Dto%7Bbackground%2Dposition%3A0%200%7D%7D%40keyframes%20progress%2Dbar%2Dstripes%7Bfrom%7Bbackground%2Dposition%3A40px%200%7Dto%7Bbackground%2Dposition%3A0%200%7D%7D%2Eprogress%7Bheight%3A20px%3Bmargin%2Dbottom%3A20px%3Boverflow%3Ahidden%3Bbackground%2Dcolor%3A%23f5f5f5%3Bborder%2Dradius%3A4px%3B%2Dwebkit%2Dbox%2Dshadow%3Ainset%200%201px%202px%20rgba%280%2C0%2C0%2C%2E1%29%3Bbox%2Dshadow%3Ainset%200%201px%202px%20rgba%280%2C0%2C0%2C%2E1%29%7D%2Eprogress%2Dbar%7Bfloat%3Aleft%3Bwidth%3A0%3Bheight%3A100%25%3Bfont%2Dsize%3A12px%3Bline%2Dheight%3A20px%3Bcolor%3A%23fff%3Btext%2Dalign%3Acenter%3Bbackground%2Dcolor%3A%23337ab7%3B%2Dwebkit%2Dbox%2Dshadow%3Ainset%200%20%2D1px%200%20rgba%280%2C0%2C0%2C%2E15%29%3Bbox%2Dshadow%3Ainset%200%20%2D1px%200%20rgba%280%2C0%2C0%2C%2E15%29%3B%2Dwebkit%2Dtransition%3Awidth%20%2E6s%20ease%3B%2Do%2Dtransition%3Awidth%20%2E6s%20ease%3Btransition%3Awidth%20%2E6s%20ease%7D%2Eprogress%2Dbar%2Dstriped%2C%2Eprogress%2Dstriped%20%2Eprogress%2Dbar%7Bbackground%2Dimage%3A%2Dwebkit%2Dlinear%2Dgradient%2845deg%2Crgba%28255%2C255%2C255%2C%2E15%29%2025%25%2Ctransparent%2025%25%2Ctransparent%2050%25%2Crgba%28255%2C255%2C255%2C%2E15%29%2050%25%2Crgba%28255%2C255%2C255%2C%2E15%29%2075%25%2Ctransparent%2075%25%2Ctransparent%29%3Bbackground%2Dimage%3A%2Do%2Dlinear%2Dgradient%2845deg%2Crgba%28255%2C255%2C255%2C%2E15%29%2025%25%2Ctransparent%2025%25%2Ctransparent%2050%25%2Crgba%28255%2C255%2C255%2C%2E15%29%2050%25%2Crgba%28255%2C255%2C255%2C%2E15%29%2075%25%2Ctransparent%2075%25%2Ctransparent%29%3Bbackground%2Dimage%3Alinear%2Dgradient%2845deg%2Crgba%28255%2C255%2C255%2C%2E15%29%2025%25%2Ctransparent%2025%25%2Ctransparent%2050%25%2Crgba%28255%2C255%2C255%2C%2E15%29%2050%25%2Crgba%28255%2C255%2C255%2C%2E15%29%2075%25%2Ctransparent%2075%25%2Ctransparent%29%3B%2Dwebkit%2Dbackground%2Dsize%3A40px%2040px%3Bbackground%2Dsize%3A40px%2040px%7D%2Eprogress%2Dbar%2Eactive%2C%2Eprogress%2Eactive%20%2Eprogress%2Dbar%7B%2Dwebkit%2Danimation%3Aprogress%2Dbar%2Dstripes%202s%20linear%20infinite%3B%2Do%2Danimation%3Aprogress%2Dbar%2Dstripes%202s%20linear%20infinite%3Banimation%3Aprogress%2Dbar%2Dstripes%202s%20linear%20infinite%7D%2Eprogress%2Dbar%2Dsuccess%7Bbackground%2Dcolor%3A%235cb85c%7D%2Eprogress%2Dstriped%20%2Eprogress%2Dbar%2Dsuccess%7Bbackground%2Dimage%3A%2Dwebkit%2Dlinear%2Dgradient%2845deg%2Crgba%28255%2C255%2C255%2C%2E15%29%2025%25%2Ctransparent%2025%25%2Ctransparent%2050%25%2Crgba%28255%2C255%2C255%2C%2E15%29%2050%25%2Crgba%28255%2C255%2C255%2C%2E15%29%2075%25%2Ctransparent%2075%25%2Ctransparent%29%3Bbackground%2Dimage%3A%2Do%2Dlinear%2Dgradient%2845deg%2Crgba%28255%2C255%2C255%2C%2E15%29%2025%25%2Ctransparent%2025%25%2Ctransparent%2050%25%2Crgba%28255%2C255%2C255%2C%2E15%29%2050%25%2Crgba%28255%2C255%2C255%2C%2E15%29%2075%25%2Ctransparent%2075%25%2Ctransparent%29%3Bbackground%2Dimage%3Alinear%2Dgradient%2845deg%2Crgba%28255%2C255%2C255%2C%2E15%29%2025%25%2Ctransparent%2025%25%2Ctransparent%2050%25%2Crgba%28255%2C255%2C255%2C%2E15%29%2050%25%2Crgba%28255%2C255%2C255%2C%2E15%29%2075%25%2Ctransparent%2075%25%2Ctransparent%29%7D%2Eprogress%2Dbar%2Dinfo%7Bbackground%2Dcolor%3A%235bc0de%7D%2Eprogress%2Dstriped%20%2Eprogress%2Dbar%2Dinfo%7Bbackground%2Dimage%3A%2Dwebkit%2Dlinear%2Dgradient%2845deg%2Crgba%28255%2C255%2C255%2C%2E15%29%2025%25%2Ctransparent%2025%25%2Ctransparent%2050%25%2Crgba%28255%2C255%2C255%2C%2E15%29%2050%25%2Crgba%28255%2C255%2C255%2C%2E15%29%2075%25%2Ctransparent%2075%25%2Ctransparent%29%3Bbackground%2Dimage%3A%2Do%2Dlinear%2Dgradient%2845deg%2Crgba%28255%2C255%2C255%2C%2E15%29%2025%25%2Ctransparent%2025%25%2Ctransparent%2050%25%2Crgba%28255%2C255%2C255%2C%2E15%29%2050%25%2Crgba%28255%2C255%2C255%2C%2E15%29%2075%25%2Ctransparent%2075%25%2Ctransparent%29%3Bbackground%2Dimage%3Alinear%2Dgradient%2845deg%2Crgba%28255%2C255%2C255%2C%2E15%29%2025%25%2Ctransparent%2025%25%2Ctransparent%2050%25%2Crgba%28255%2C255%2C255%2C%2E15%29%2050%25%2Crgba%28255%2C255%2C255%2C%2E15%29%2075%25%2Ctransparent%2075%25%2Ctransparent%29%7D%2Eprogress%2Dbar%2Dwarning%7Bbackground%2Dcolor%3A%23f0ad4e%7D%2Eprogress%2Dstriped%20%2Eprogress%2Dbar%2Dwarning%7Bbackground%2Dimage%3A%2Dwebkit%2Dlinear%2Dgradient%2845deg%2Crgba%28255%2C255%2C255%2C%2E15%29%2025%25%2Ctransparent%2025%25%2Ctransparent%2050%25%2Crgba%28255%2C255%2C255%2C%2E15%29%2050%25%2Crgba%28255%2C255%2C255%2C%2E15%29%2075%25%2Ctransparent%2075%25%2Ctransparent%29%3Bbackground%2Dimage%3A%2Do%2Dlinear%2Dgradient%2845deg%2Crgba%28255%2C255%2C255%2C%2E15%29%2025%25%2Ctransparent%2025%25%2Ctransparent%2050%25%2Crgba%28255%2C255%2C255%2C%2E15%29%2050%25%2Crgba%28255%2C255%2C255%2C%2E15%29%2075%25%2Ctransparent%2075%25%2Ctransparent%29%3Bbackground%2Dimage%3Alinear%2Dgradient%2845deg%2Crgba%28255%2C255%2C255%2C%2E15%29%2025%25%2Ctransparent%2025%25%2Ctransparent%2050%25%2Crgba%28255%2C255%2C255%2C%2E15%29%2050%25%2Crgba%28255%2C255%2C255%2C%2E15%29%2075%25%2Ctransparent%2075%25%2Ctransparent%29%7D%2Eprogress%2Dbar%2Ddanger%7Bbackground%2Dcolor%3A%23d9534f%7D%2Eprogress%2Dstriped%20%2Eprogress%2Dbar%2Ddanger%7Bbackground%2Dimage%3A%2Dwebkit%2Dlinear%2Dgradient%2845deg%2Crgba%28255%2C255%2C255%2C%2E15%29%2025%25%2Ctransparent%2025%25%2Ctransparent%2050%25%2Crgba%28255%2C255%2C255%2C%2E15%29%2050%25%2Crgba%28255%2C255%2C255%2C%2E15%29%2075%25%2Ctransparent%2075%25%2Ctransparent%29%3Bbackground%2Dimage%3A%2Do%2Dlinear%2Dgradient%2845deg%2Crgba%28255%2C255%2C255%2C%2E15%29%2025%25%2Ctransparent%2025%25%2Ctransparent%2050%25%2Crgba%28255%2C255%2C255%2C%2E15%29%2050%25%2Crgba%28255%2C255%2C255%2C%2E15%29%2075%25%2Ctransparent%2075%25%2Ctransparent%29%3Bbackground%2Dimage%3Alinear%2Dgradient%2845deg%2Crgba%28255%2C255%2C255%2C%2E15%29%2025%25%2Ctransparent%2025%25%2Ctransparent%2050%25%2Crgba%28255%2C255%2C255%2C%2E15%29%2050%25%2Crgba%28255%2C255%2C255%2C%2E15%29%2075%25%2Ctransparent%2075%25%2Ctransparent%29%7D%2Emedia%7Bmargin%2Dtop%3A15px%7D%2Emedia%3Afirst%2Dchild%7Bmargin%2Dtop%3A0%7D%2Emedia%2C%2Emedia%2Dbody%7Boverflow%3Ahidden%3Bzoom%3A1%7D%2Emedia%2Dbody%7Bwidth%3A10000px%7D%2Emedia%2Dobject%7Bdisplay%3Ablock%7D%2Emedia%2Dobject%2Eimg%2Dthumbnail%7Bmax%2Dwidth%3Anone%7D%2Emedia%2Dright%2C%2Emedia%3E%2Epull%2Dright%7Bpadding%2Dleft%3A10px%7D%2Emedia%2Dleft%2C%2Emedia%3E%2Epull%2Dleft%7Bpadding%2Dright%3A10px%7D%2Emedia%2Dbody%2C%2Emedia%2Dleft%2C%2Emedia%2Dright%7Bdisplay%3Atable%2Dcell%3Bvertical%2Dalign%3Atop%7D%2Emedia%2Dmiddle%7Bvertical%2Dalign%3Amiddle%7D%2Emedia%2Dbottom%7Bvertical%2Dalign%3Abottom%7D%2Emedia%2Dheading%7Bmargin%2Dtop%3A0%3Bmargin%2Dbottom%3A5px%7D%2Emedia%2Dlist%7Bpadding%2Dleft%3A0%3Blist%2Dstyle%3Anone%7D%2Elist%2Dgroup%7Bpadding%2Dleft%3A0%3Bmargin%2Dbottom%3A20px%7D%2Elist%2Dgroup%2Ditem%7Bposition%3Arelative%3Bdisplay%3Ablock%3Bpadding%3A10px%2015px%3Bmargin%2Dbottom%3A%2D1px%3Bbackground%2Dcolor%3A%23fff%3Bborder%3A1px%20solid%20%23ddd%7D%2Elist%2Dgroup%2Ditem%3Afirst%2Dchild%7Bborder%2Dtop%2Dleft%2Dradius%3A4px%3Bborder%2Dtop%2Dright%2Dradius%3A4px%7D%2Elist%2Dgroup%2Ditem%3Alast%2Dchild%7Bmargin%2Dbottom%3A0%3Bborder%2Dbottom%2Dright%2Dradius%3A4px%3Bborder%2Dbottom%2Dleft%2Dradius%3A4px%7Da%2Elist%2Dgroup%2Ditem%2Cbutton%2Elist%2Dgroup%2Ditem%7Bcolor%3A%23555%7Da%2Elist%2Dgroup%2Ditem%20%2Elist%2Dgroup%2Ditem%2Dheading%2Cbutton%2Elist%2Dgroup%2Ditem%20%2Elist%2Dgroup%2Ditem%2Dheading%7Bcolor%3A%23333%7Da%2Elist%2Dgroup%2Ditem%3Afocus%2Ca%2Elist%2Dgroup%2Ditem%3Ahover%2Cbutton%2Elist%2Dgroup%2Ditem%3Afocus%2Cbutton%2Elist%2Dgroup%2Ditem%3Ahover%7Bcolor%3A%23555%3Btext%2Ddecoration%3Anone%3Bbackground%2Dcolor%3A%23f5f5f5%7Dbutton%2Elist%2Dgroup%2Ditem%7Bwidth%3A100%25%3Btext%2Dalign%3Aleft%7D%2Elist%2Dgroup%2Ditem%2Edisabled%2C%2Elist%2Dgroup%2Ditem%2Edisabled%3Afocus%2C%2Elist%2Dgroup%2Ditem%2Edisabled%3Ahover%7Bcolor%3A%23777%3Bcursor%3Anot%2Dallowed%3Bbackground%2Dcolor%3A%23eee%7D%2Elist%2Dgroup%2Ditem%2Edisabled%20%2Elist%2Dgroup%2Ditem%2Dheading%2C%2Elist%2Dgroup%2Ditem%2Edisabled%3Afocus%20%2Elist%2Dgroup%2Ditem%2Dheading%2C%2Elist%2Dgroup%2Ditem%2Edisabled%3Ahover%20%2Elist%2Dgroup%2Ditem%2Dheading%7Bcolor%3Ainherit%7D%2Elist%2Dgroup%2Ditem%2Edisabled%20%2Elist%2Dgroup%2Ditem%2Dtext%2C%2Elist%2Dgroup%2Ditem%2Edisabled%3Afocus%20%2Elist%2Dgroup%2Ditem%2Dtext%2C%2Elist%2Dgroup%2Ditem%2Edisabled%3Ahover%20%2Elist%2Dgroup%2Ditem%2Dtext%7Bcolor%3A%23777%7D%2Elist%2Dgroup%2Ditem%2Eactive%2C%2Elist%2Dgroup%2Ditem%2Eactive%3Afocus%2C%2Elist%2Dgroup%2Ditem%2Eactive%3Ahover%7Bz%2Dindex%3A2%3Bcolor%3A%23fff%3Bbackground%2Dcolor%3A%23337ab7%3Bborder%2Dcolor%3A%23337ab7%7D%2Elist%2Dgroup%2Ditem%2Eactive%20%2Elist%2Dgroup%2Ditem%2Dheading%2C%2Elist%2Dgroup%2Ditem%2Eactive%20%2Elist%2Dgroup%2Ditem%2Dheading%3E%2Esmall%2C%2Elist%2Dgroup%2Ditem%2Eactive%20%2Elist%2Dgroup%2Ditem%2Dheading%3Esmall%2C%2Elist%2Dgroup%2Ditem%2Eactive%3Afocus%20%2Elist%2Dgroup%2Ditem%2Dheading%2C%2Elist%2Dgroup%2Ditem%2Eactive%3Afocus%20%2Elist%2Dgroup%2Ditem%2Dheading%3E%2Esmall%2C%2Elist%2Dgroup%2Ditem%2Eactive%3Afocus%20%2Elist%2Dgroup%2Ditem%2Dheading%3Esmall%2C%2Elist%2Dgroup%2Ditem%2Eactive%3Ahover%20%2Elist%2Dgroup%2Ditem%2Dheading%2C%2Elist%2Dgroup%2Ditem%2Eactive%3Ahover%20%2Elist%2Dgroup%2Ditem%2Dheading%3E%2Esmall%2C%2Elist%2Dgroup%2Ditem%2Eactive%3Ahover%20%2Elist%2Dgroup%2Ditem%2Dheading%3Esmall%7Bcolor%3Ainherit%7D%2Elist%2Dgroup%2Ditem%2Eactive%20%2Elist%2Dgroup%2Ditem%2Dtext%2C%2Elist%2Dgroup%2Ditem%2Eactive%3Afocus%20%2Elist%2Dgroup%2Ditem%2Dtext%2C%2Elist%2Dgroup%2Ditem%2Eactive%3Ahover%20%2Elist%2Dgroup%2Ditem%2Dtext%7Bcolor%3A%23c7ddef%7D%2Elist%2Dgroup%2Ditem%2Dsuccess%7Bcolor%3A%233c763d%3Bbackground%2Dcolor%3A%23dff0d8%7Da%2Elist%2Dgroup%2Ditem%2Dsuccess%2Cbutton%2Elist%2Dgroup%2Ditem%2Dsuccess%7Bcolor%3A%233c763d%7Da%2Elist%2Dgroup%2Ditem%2Dsuccess%20%2Elist%2Dgroup%2Ditem%2Dheading%2Cbutton%2Elist%2Dgroup%2Ditem%2Dsuccess%20%2Elist%2Dgroup%2Ditem%2Dheading%7Bcolor%3Ainherit%7Da%2Elist%2Dgroup%2Ditem%2Dsuccess%3Afocus%2Ca%2Elist%2Dgroup%2Ditem%2Dsuccess%3Ahover%2Cbutton%2Elist%2Dgroup%2Ditem%2Dsuccess%3Afocus%2Cbutton%2Elist%2Dgroup%2Ditem%2Dsuccess%3Ahover%7Bcolor%3A%233c763d%3Bbackground%2Dcolor%3A%23d0e9c6%7Da%2Elist%2Dgroup%2Ditem%2Dsuccess%2Eactive%2Ca%2Elist%2Dgroup%2Ditem%2Dsuccess%2Eactive%3Afocus%2Ca%2Elist%2Dgroup%2Ditem%2Dsuccess%2Eactive%3Ahover%2Cbutton%2Elist%2Dgroup%2Ditem%2Dsuccess%2Eactive%2Cbutton%2Elist%2Dgroup%2Ditem%2Dsuccess%2Eactive%3Afocus%2Cbutton%2Elist%2Dgroup%2Ditem%2Dsuccess%2Eactive%3Ahover%7Bcolor%3A%23fff%3Bbackground%2Dcolor%3A%233c763d%3Bborder%2Dcolor%3A%233c763d%7D%2Elist%2Dgroup%2Ditem%2Dinfo%7Bcolor%3A%2331708f%3Bbackground%2Dcolor%3A%23d9edf7%7Da%2Elist%2Dgroup%2Ditem%2Dinfo%2Cbutton%2Elist%2Dgroup%2Ditem%2Dinfo%7Bcolor%3A%2331708f%7Da%2Elist%2Dgroup%2Ditem%2Dinfo%20%2Elist%2Dgroup%2Ditem%2Dheading%2Cbutton%2Elist%2Dgroup%2Ditem%2Dinfo%20%2Elist%2Dgroup%2Ditem%2Dheading%7Bcolor%3Ainherit%7Da%2Elist%2Dgroup%2Ditem%2Dinfo%3Afocus%2Ca%2Elist%2Dgroup%2Ditem%2Dinfo%3Ahover%2Cbutton%2Elist%2Dgroup%2Ditem%2Dinfo%3Afocus%2Cbutton%2Elist%2Dgroup%2Ditem%2Dinfo%3Ahover%7Bcolor%3A%2331708f%3Bbackground%2Dcolor%3A%23c4e3f3%7Da%2Elist%2Dgroup%2Ditem%2Dinfo%2Eactive%2Ca%2Elist%2Dgroup%2Ditem%2Dinfo%2Eactive%3Afocus%2Ca%2Elist%2Dgroup%2Ditem%2Dinfo%2Eactive%3Ahover%2Cbutton%2Elist%2Dgroup%2Ditem%2Dinfo%2Eactive%2Cbutton%2Elist%2Dgroup%2Ditem%2Dinfo%2Eactive%3Afocus%2Cbutton%2Elist%2Dgroup%2Ditem%2Dinfo%2Eactive%3Ahover%7Bcolor%3A%23fff%3Bbackground%2Dcolor%3A%2331708f%3Bborder%2Dcolor%3A%2331708f%7D%2Elist%2Dgroup%2Ditem%2Dwarning%7Bcolor%3A%238a6d3b%3Bbackground%2Dcolor%3A%23fcf8e3%7Da%2Elist%2Dgroup%2Ditem%2Dwarning%2Cbutton%2Elist%2Dgroup%2Ditem%2Dwarning%7Bcolor%3A%238a6d3b%7Da%2Elist%2Dgroup%2Ditem%2Dwarning%20%2Elist%2Dgroup%2Ditem%2Dheading%2Cbutton%2Elist%2Dgroup%2Ditem%2Dwarning%20%2Elist%2Dgroup%2Ditem%2Dheading%7Bcolor%3Ainherit%7Da%2Elist%2Dgroup%2Ditem%2Dwarning%3Afocus%2Ca%2Elist%2Dgroup%2Ditem%2Dwarning%3Ahover%2Cbutton%2Elist%2Dgroup%2Ditem%2Dwarning%3Afocus%2Cbutton%2Elist%2Dgroup%2Ditem%2Dwarning%3Ahover%7Bcolor%3A%238a6d3b%3Bbackground%2Dcolor%3A%23faf2cc%7Da%2Elist%2Dgroup%2Ditem%2Dwarning%2Eactive%2Ca%2Elist%2Dgroup%2Ditem%2Dwarning%2Eactive%3Afocus%2Ca%2Elist%2Dgroup%2Ditem%2Dwarning%2Eactive%3Ahover%2Cbutton%2Elist%2Dgroup%2Ditem%2Dwarning%2Eactive%2Cbutton%2Elist%2Dgroup%2Ditem%2Dwarning%2Eactive%3Afocus%2Cbutton%2Elist%2Dgroup%2Ditem%2Dwarning%2Eactive%3Ahover%7Bcolor%3A%23fff%3Bbackground%2Dcolor%3A%238a6d3b%3Bborder%2Dcolor%3A%238a6d3b%7D%2Elist%2Dgroup%2Ditem%2Ddanger%7Bcolor%3A%23a94442%3Bbackground%2Dcolor%3A%23f2dede%7Da%2Elist%2Dgroup%2Ditem%2Ddanger%2Cbutton%2Elist%2Dgroup%2Ditem%2Ddanger%7Bcolor%3A%23a94442%7Da%2Elist%2Dgroup%2Ditem%2Ddanger%20%2Elist%2Dgroup%2Ditem%2Dheading%2Cbutton%2Elist%2Dgroup%2Ditem%2Ddanger%20%2Elist%2Dgroup%2Ditem%2Dheading%7Bcolor%3Ainherit%7Da%2Elist%2Dgroup%2Ditem%2Ddanger%3Afocus%2Ca%2Elist%2Dgroup%2Ditem%2Ddanger%3Ahover%2Cbutton%2Elist%2Dgroup%2Ditem%2Ddanger%3Afocus%2Cbutton%2Elist%2Dgroup%2Ditem%2Ddanger%3Ahover%7Bcolor%3A%23a94442%3Bbackground%2Dcolor%3A%23ebcccc%7Da%2Elist%2Dgroup%2Ditem%2Ddanger%2Eactive%2Ca%2Elist%2Dgroup%2Ditem%2Ddanger%2Eactive%3Afocus%2Ca%2Elist%2Dgroup%2Ditem%2Ddanger%2Eactive%3Ahover%2Cbutton%2Elist%2Dgroup%2Ditem%2Ddanger%2Eactive%2Cbutton%2Elist%2Dgroup%2Ditem%2Ddanger%2Eactive%3Afocus%2Cbutton%2Elist%2Dgroup%2Ditem%2Ddanger%2Eactive%3Ahover%7Bcolor%3A%23fff%3Bbackground%2Dcolor%3A%23a94442%3Bborder%2Dcolor%3A%23a94442%7D%2Elist%2Dgroup%2Ditem%2Dheading%7Bmargin%2Dtop%3A0%3Bmargin%2Dbottom%3A5px%7D%2Elist%2Dgroup%2Ditem%2Dtext%7Bmargin%2Dbottom%3A0%3Bline%2Dheight%3A1%2E3%7D%2Epanel%7Bmargin%2Dbottom%3A20px%3Bbackground%2Dcolor%3A%23fff%3Bborder%3A1px%20solid%20transparent%3Bborder%2Dradius%3A4px%3B%2Dwebkit%2Dbox%2Dshadow%3A0%201px%201px%20rgba%280%2C0%2C0%2C%2E05%29%3Bbox%2Dshadow%3A0%201px%201px%20rgba%280%2C0%2C0%2C%2E05%29%7D%2Epanel%2Dbody%7Bpadding%3A15px%7D%2Epanel%2Dheading%7Bpadding%3A10px%2015px%3Bborder%2Dbottom%3A1px%20solid%20transparent%3Bborder%2Dtop%2Dleft%2Dradius%3A3px%3Bborder%2Dtop%2Dright%2Dradius%3A3px%7D%2Epanel%2Dheading%3E%2Edropdown%20%2Edropdown%2Dtoggle%7Bcolor%3Ainherit%7D%2Epanel%2Dtitle%7Bmargin%2Dtop%3A0%3Bmargin%2Dbottom%3A0%3Bfont%2Dsize%3A16px%3Bcolor%3Ainherit%7D%2Epanel%2Dtitle%3E%2Esmall%2C%2Epanel%2Dtitle%3E%2Esmall%3Ea%2C%2Epanel%2Dtitle%3Ea%2C%2Epanel%2Dtitle%3Esmall%2C%2Epanel%2Dtitle%3Esmall%3Ea%7Bcolor%3Ainherit%7D%2Epanel%2Dfooter%7Bpadding%3A10px%2015px%3Bbackground%2Dcolor%3A%23f5f5f5%3Bborder%2Dtop%3A1px%20solid%20%23ddd%3Bborder%2Dbottom%2Dright%2Dradius%3A3px%3Bborder%2Dbottom%2Dleft%2Dradius%3A3px%7D%2Epanel%3E%2Elist%2Dgroup%2C%2Epanel%3E%2Epanel%2Dcollapse%3E%2Elist%2Dgroup%7Bmargin%2Dbottom%3A0%7D%2Epanel%3E%2Elist%2Dgroup%20%2Elist%2Dgroup%2Ditem%2C%2Epanel%3E%2Epanel%2Dcollapse%3E%2Elist%2Dgroup%20%2Elist%2Dgroup%2Ditem%7Bborder%2Dwidth%3A1px%200%3Bborder%2Dradius%3A0%7D%2Epanel%3E%2Elist%2Dgroup%3Afirst%2Dchild%20%2Elist%2Dgroup%2Ditem%3Afirst%2Dchild%2C%2Epanel%3E%2Epanel%2Dcollapse%3E%2Elist%2Dgroup%3Afirst%2Dchild%20%2Elist%2Dgroup%2Ditem%3Afirst%2Dchild%7Bborder%2Dtop%3A0%3Bborder%2Dtop%2Dleft%2Dradius%3A3px%3Bborder%2Dtop%2Dright%2Dradius%3A3px%7D%2Epanel%3E%2Elist%2Dgroup%3Alast%2Dchild%20%2Elist%2Dgroup%2Ditem%3Alast%2Dchild%2C%2Epanel%3E%2Epanel%2Dcollapse%3E%2Elist%2Dgroup%3Alast%2Dchild%20%2Elist%2Dgroup%2Ditem%3Alast%2Dchild%7Bborder%2Dbottom%3A0%3Bborder%2Dbottom%2Dright%2Dradius%3A3px%3Bborder%2Dbottom%2Dleft%2Dradius%3A3px%7D%2Epanel%3E%2Epanel%2Dheading%2B%2Epanel%2Dcollapse%3E%2Elist%2Dgroup%20%2Elist%2Dgroup%2Ditem%3Afirst%2Dchild%7Bborder%2Dtop%2Dleft%2Dradius%3A0%3Bborder%2Dtop%2Dright%2Dradius%3A0%7D%2Epanel%2Dheading%2B%2Elist%2Dgroup%20%2Elist%2Dgroup%2Ditem%3Afirst%2Dchild%7Bborder%2Dtop%2Dwidth%3A0%7D%2Elist%2Dgroup%2B%2Epanel%2Dfooter%7Bborder%2Dtop%2Dwidth%3A0%7D%2Epanel%3E%2Epanel%2Dcollapse%3E%2Etable%2C%2Epanel%3E%2Etable%2C%2Epanel%3E%2Etable%2Dresponsive%3E%2Etable%7Bmargin%2Dbottom%3A0%7D%2Epanel%3E%2Epanel%2Dcollapse%3E%2Etable%20caption%2C%2Epanel%3E%2Etable%20caption%2C%2Epanel%3E%2Etable%2Dresponsive%3E%2Etable%20caption%7Bpadding%2Dright%3A15px%3Bpadding%2Dleft%3A15px%7D%2Epanel%3E%2Etable%2Dresponsive%3Afirst%2Dchild%3E%2Etable%3Afirst%2Dchild%2C%2Epanel%3E%2Etable%3Afirst%2Dchild%7Bborder%2Dtop%2Dleft%2Dradius%3A3px%3Bborder%2Dtop%2Dright%2Dradius%3A3px%7D%2Epanel%3E%2Etable%2Dresponsive%3Afirst%2Dchild%3E%2Etable%3Afirst%2Dchild%3Etbody%3Afirst%2Dchild%3Etr%3Afirst%2Dchild%2C%2Epanel%3E%2Etable%2Dresponsive%3Afirst%2Dchild%3E%2Etable%3Afirst%2Dchild%3Ethead%3Afirst%2Dchild%3Etr%3Afirst%2Dchild%2C%2Epanel%3E%2Etable%3Afirst%2Dchild%3Etbody%3Afirst%2Dchild%3Etr%3Afirst%2Dchild%2C%2Epanel%3E%2Etable%3Afirst%2Dchild%3Ethead%3Afirst%2Dchild%3Etr%3Afirst%2Dchild%7Bborder%2Dtop%2Dleft%2Dradius%3A3px%3Bborder%2Dtop%2Dright%2Dradius%3A3px%7D%2Epanel%3E%2Etable%2Dresponsive%3Afirst%2Dchild%3E%2Etable%3Afirst%2Dchild%3Etbody%3Afirst%2Dchild%3Etr%3Afirst%2Dchild%20td%3Afirst%2Dchild%2C%2Epanel%3E%2Etable%2Dresponsive%3Afirst%2Dchild%3E%2Etable%3Afirst%2Dchild%3Etbody%3Afirst%2Dchild%3Etr%3Afirst%2Dchild%20th%3Afirst%2Dchild%2C%2Epanel%3E%2Etable%2Dresponsive%3Afirst%2Dchild%3E%2Etable%3Afirst%2Dchild%3Ethead%3Afirst%2Dchild%3Etr%3Afirst%2Dchild%20td%3Afirst%2Dchild%2C%2Epanel%3E%2Etable%2Dresponsive%3Afirst%2Dchild%3E%2Etable%3Afirst%2Dchild%3Ethead%3Afirst%2Dchild%3Etr%3Afirst%2Dchild%20th%3Afirst%2Dchild%2C%2Epanel%3E%2Etable%3Afirst%2Dchild%3Etbody%3Afirst%2Dchild%3Etr%3Afirst%2Dchild%20td%3Afirst%2Dchild%2C%2Epanel%3E%2Etable%3Afirst%2Dchild%3Etbody%3Afirst%2Dchild%3Etr%3Afirst%2Dchild%20th%3Afirst%2Dchild%2C%2Epanel%3E%2Etable%3Afirst%2Dchild%3Ethead%3Afirst%2Dchild%3Etr%3Afirst%2Dchild%20td%3Afirst%2Dchild%2C%2Epanel%3E%2Etable%3Afirst%2Dchild%3Ethead%3Afirst%2Dchild%3Etr%3Afirst%2Dchild%20th%3Afirst%2Dchild%7Bborder%2Dtop%2Dleft%2Dradius%3A3px%7D%2Epanel%3E%2Etable%2Dresponsive%3Afirst%2Dchild%3E%2Etable%3Afirst%2Dchild%3Etbody%3Afirst%2Dchild%3Etr%3Afirst%2Dchild%20td%3Alast%2Dchild%2C%2Epanel%3E%2Etable%2Dresponsive%3Afirst%2Dchild%3E%2Etable%3Afirst%2Dchild%3Etbody%3Afirst%2Dchild%3Etr%3Afirst%2Dchild%20th%3Alast%2Dchild%2C%2Epanel%3E%2Etable%2Dresponsive%3Afirst%2Dchild%3E%2Etable%3Afirst%2Dchild%3Ethead%3Afirst%2Dchild%3Etr%3Afirst%2Dchild%20td%3Alast%2Dchild%2C%2Epanel%3E%2Etable%2Dresponsive%3Afirst%2Dchild%3E%2Etable%3Afirst%2Dchild%3Ethead%3Afirst%2Dchild%3Etr%3Afirst%2Dchild%20th%3Alast%2Dchild%2C%2Epanel%3E%2Etable%3Afirst%2Dchild%3Etbody%3Afirst%2Dchild%3Etr%3Afirst%2Dchild%20td%3Alast%2Dchild%2C%2Epanel%3E%2Etable%3Afirst%2Dchild%3Etbody%3Afirst%2Dchild%3Etr%3Afirst%2Dchild%20th%3Alast%2Dchild%2C%2Epanel%3E%2Etable%3Afirst%2Dchild%3Ethead%3Afirst%2Dchild%3Etr%3Afirst%2Dchild%20td%3Alast%2Dchild%2C%2Epanel%3E%2Etable%3Afirst%2Dchild%3Ethead%3Afirst%2Dchild%3Etr%3Afirst%2Dchild%20th%3Alast%2Dchild%7Bborder%2Dtop%2Dright%2Dradius%3A3px%7D%2Epanel%3E%2Etable%2Dresponsive%3Alast%2Dchild%3E%2Etable%3Alast%2Dchild%2C%2Epanel%3E%2Etable%3Alast%2Dchild%7Bborder%2Dbottom%2Dright%2Dradius%3A3px%3Bborder%2Dbottom%2Dleft%2Dradius%3A3px%7D%2Epanel%3E%2Etable%2Dresponsive%3Alast%2Dchild%3E%2Etable%3Alast%2Dchild%3Etbody%3Alast%2Dchild%3Etr%3Alast%2Dchild%2C%2Epanel%3E%2Etable%2Dresponsive%3Alast%2Dchild%3E%2Etable%3Alast%2Dchild%3Etfoot%3Alast%2Dchild%3Etr%3Alast%2Dchild%2C%2Epanel%3E%2Etable%3Alast%2Dchild%3Etbody%3Alast%2Dchild%3Etr%3Alast%2Dchild%2C%2Epanel%3E%2Etable%3Alast%2Dchild%3Etfoot%3Alast%2Dchild%3Etr%3Alast%2Dchild%7Bborder%2Dbottom%2Dright%2Dradius%3A3px%3Bborder%2Dbottom%2Dleft%2Dradius%3A3px%7D%2Epanel%3E%2Etable%2Dresponsive%3Alast%2Dchild%3E%2Etable%3Alast%2Dchild%3Etbody%3Alast%2Dchild%3Etr%3Alast%2Dchild%20td%3Afirst%2Dchild%2C%2Epanel%3E%2Etable%2Dresponsive%3Alast%2Dchild%3E%2Etable%3Alast%2Dchild%3Etbody%3Alast%2Dchild%3Etr%3Alast%2Dchild%20th%3Afirst%2Dchild%2C%2Epanel%3E%2Etable%2Dresponsive%3Alast%2Dchild%3E%2Etable%3Alast%2Dchild%3Etfoot%3Alast%2Dchild%3Etr%3Alast%2Dchild%20td%3Afirst%2Dchild%2C%2Epanel%3E%2Etable%2Dresponsive%3Alast%2Dchild%3E%2Etable%3Alast%2Dchild%3Etfoot%3Alast%2Dchild%3Etr%3Alast%2Dchild%20th%3Afirst%2Dchild%2C%2Epanel%3E%2Etable%3Alast%2Dchild%3Etbody%3Alast%2Dchild%3Etr%3Alast%2Dchild%20td%3Afirst%2Dchild%2C%2Epanel%3E%2Etable%3Alast%2Dchild%3Etbody%3Alast%2Dchild%3Etr%3Alast%2Dchild%20th%3Afirst%2Dchild%2C%2Epanel%3E%2Etable%3Alast%2Dchild%3Etfoot%3Alast%2Dchild%3Etr%3Alast%2Dchild%20td%3Afirst%2Dchild%2C%2Epanel%3E%2Etable%3Alast%2Dchild%3Etfoot%3Alast%2Dchild%3Etr%3Alast%2Dchild%20th%3Afirst%2Dchild%7Bborder%2Dbottom%2Dleft%2Dradius%3A3px%7D%2Epanel%3E%2Etable%2Dresponsive%3Alast%2Dchild%3E%2Etable%3Alast%2Dchild%3Etbody%3Alast%2Dchild%3Etr%3Alast%2Dchild%20td%3Alast%2Dchild%2C%2Epanel%3E%2Etable%2Dresponsive%3Alast%2Dchild%3E%2Etable%3Alast%2Dchild%3Etbody%3Alast%2Dchild%3Etr%3Alast%2Dchild%20th%3Alast%2Dchild%2C%2Epanel%3E%2Etable%2Dresponsive%3Alast%2Dchild%3E%2Etable%3Alast%2Dchild%3Etfoot%3Alast%2Dchild%3Etr%3Alast%2Dchild%20td%3Alast%2Dchild%2C%2Epanel%3E%2Etable%2Dresponsive%3Alast%2Dchild%3E%2Etable%3Alast%2Dchild%3Etfoot%3Alast%2Dchild%3Etr%3Alast%2Dchild%20th%3Alast%2Dchild%2C%2Epanel%3E%2Etable%3Alast%2Dchild%3Etbody%3Alast%2Dchild%3Etr%3Alast%2Dchild%20td%3Alast%2Dchild%2C%2Epanel%3E%2Etable%3Alast%2Dchild%3Etbody%3Alast%2Dchild%3Etr%3Alast%2Dchild%20th%3Alast%2Dchild%2C%2Epanel%3E%2Etable%3Alast%2Dchild%3Etfoot%3Alast%2Dchild%3Etr%3Alast%2Dchild%20td%3Alast%2Dchild%2C%2Epanel%3E%2Etable%3Alast%2Dchild%3Etfoot%3Alast%2Dchild%3Etr%3Alast%2Dchild%20th%3Alast%2Dchild%7Bborder%2Dbottom%2Dright%2Dradius%3A3px%7D%2Epanel%3E%2Epanel%2Dbody%2B%2Etable%2C%2Epanel%3E%2Epanel%2Dbody%2B%2Etable%2Dresponsive%2C%2Epanel%3E%2Etable%2B%2Epanel%2Dbody%2C%2Epanel%3E%2Etable%2Dresponsive%2B%2Epanel%2Dbody%7Bborder%2Dtop%3A1px%20solid%20%23ddd%7D%2Epanel%3E%2Etable%3Etbody%3Afirst%2Dchild%3Etr%3Afirst%2Dchild%20td%2C%2Epanel%3E%2Etable%3Etbody%3Afirst%2Dchild%3Etr%3Afirst%2Dchild%20th%7Bborder%2Dtop%3A0%7D%2Epanel%3E%2Etable%2Dbordered%2C%2Epanel%3E%2Etable%2Dresponsive%3E%2Etable%2Dbordered%7Bborder%3A0%7D%2Epanel%3E%2Etable%2Dbordered%3Etbody%3Etr%3Etd%3Afirst%2Dchild%2C%2Epanel%3E%2Etable%2Dbordered%3Etbody%3Etr%3Eth%3Afirst%2Dchild%2C%2Epanel%3E%2Etable%2Dbordered%3Etfoot%3Etr%3Etd%3Afirst%2Dchild%2C%2Epanel%3E%2Etable%2Dbordered%3Etfoot%3Etr%3Eth%3Afirst%2Dchild%2C%2Epanel%3E%2Etable%2Dbordered%3Ethead%3Etr%3Etd%3Afirst%2Dchild%2C%2Epanel%3E%2Etable%2Dbordered%3Ethead%3Etr%3Eth%3Afirst%2Dchild%2C%2Epanel%3E%2Etable%2Dresponsive%3E%2Etable%2Dbordered%3Etbody%3Etr%3Etd%3Afirst%2Dchild%2C%2Epanel%3E%2Etable%2Dresponsive%3E%2Etable%2Dbordered%3Etbody%3Etr%3Eth%3Afirst%2Dchild%2C%2Epanel%3E%2Etable%2Dresponsive%3E%2Etable%2Dbordered%3Etfoot%3Etr%3Etd%3Afirst%2Dchild%2C%2Epanel%3E%2Etable%2Dresponsive%3E%2Etable%2Dbordered%3Etfoot%3Etr%3Eth%3Afirst%2Dchild%2C%2Epanel%3E%2Etable%2Dresponsive%3E%2Etable%2Dbordered%3Ethead%3Etr%3Etd%3Afirst%2Dchild%2C%2Epanel%3E%2Etable%2Dresponsive%3E%2Etable%2Dbordered%3Ethead%3Etr%3Eth%3Afirst%2Dchild%7Bborder%2Dleft%3A0%7D%2Epanel%3E%2Etable%2Dbordered%3Etbody%3Etr%3Etd%3Alast%2Dchild%2C%2Epanel%3E%2Etable%2Dbordered%3Etbody%3Etr%3Eth%3Alast%2Dchild%2C%2Epanel%3E%2Etable%2Dbordered%3Etfoot%3Etr%3Etd%3Alast%2Dchild%2C%2Epanel%3E%2Etable%2Dbordered%3Etfoot%3Etr%3Eth%3Alast%2Dchild%2C%2Epanel%3E%2Etable%2Dbordered%3Ethead%3Etr%3Etd%3Alast%2Dchild%2C%2Epanel%3E%2Etable%2Dbordered%3Ethead%3Etr%3Eth%3Alast%2Dchild%2C%2Epanel%3E%2Etable%2Dresponsive%3E%2Etable%2Dbordered%3Etbody%3Etr%3Etd%3Alast%2Dchild%2C%2Epanel%3E%2Etable%2Dresponsive%3E%2Etable%2Dbordered%3Etbody%3Etr%3Eth%3Alast%2Dchild%2C%2Epanel%3E%2Etable%2Dresponsive%3E%2Etable%2Dbordered%3Etfoot%3Etr%3Etd%3Alast%2Dchild%2C%2Epanel%3E%2Etable%2Dresponsive%3E%2Etable%2Dbordered%3Etfoot%3Etr%3Eth%3Alast%2Dchild%2C%2Epanel%3E%2Etable%2Dresponsive%3E%2Etable%2Dbordered%3Ethead%3Etr%3Etd%3Alast%2Dchild%2C%2Epanel%3E%2Etable%2Dresponsive%3E%2Etable%2Dbordered%3Ethead%3Etr%3Eth%3Alast%2Dchild%7Bborder%2Dright%3A0%7D%2Epanel%3E%2Etable%2Dbordered%3Etbody%3Etr%3Afirst%2Dchild%3Etd%2C%2Epanel%3E%2Etable%2Dbordered%3Etbody%3Etr%3Afirst%2Dchild%3Eth%2C%2Epanel%3E%2Etable%2Dbordered%3Ethead%3Etr%3Afirst%2Dchild%3Etd%2C%2Epanel%3E%2Etable%2Dbordered%3Ethead%3Etr%3Afirst%2Dchild%3Eth%2C%2Epanel%3E%2Etable%2Dresponsive%3E%2Etable%2Dbordered%3Etbody%3Etr%3Afirst%2Dchild%3Etd%2C%2Epanel%3E%2Etable%2Dresponsive%3E%2Etable%2Dbordered%3Etbody%3Etr%3Afirst%2Dchild%3Eth%2C%2Epanel%3E%2Etable%2Dresponsive%3E%2Etable%2Dbordered%3Ethead%3Etr%3Afirst%2Dchild%3Etd%2C%2Epanel%3E%2Etable%2Dresponsive%3E%2Etable%2Dbordered%3Ethead%3Etr%3Afirst%2Dchild%3Eth%7Bborder%2Dbottom%3A0%7D%2Epanel%3E%2Etable%2Dbordered%3Etbody%3Etr%3Alast%2Dchild%3Etd%2C%2Epanel%3E%2Etable%2Dbordered%3Etbody%3Etr%3Alast%2Dchild%3Eth%2C%2Epanel%3E%2Etable%2Dbordered%3Etfoot%3Etr%3Alast%2Dchild%3Etd%2C%2Epanel%3E%2Etable%2Dbordered%3Etfoot%3Etr%3Alast%2Dchild%3Eth%2C%2Epanel%3E%2Etable%2Dresponsive%3E%2Etable%2Dbordered%3Etbody%3Etr%3Alast%2Dchild%3Etd%2C%2Epanel%3E%2Etable%2Dresponsive%3E%2Etable%2Dbordered%3Etbody%3Etr%3Alast%2Dchild%3Eth%2C%2Epanel%3E%2Etable%2Dresponsive%3E%2Etable%2Dbordered%3Etfoot%3Etr%3Alast%2Dchild%3Etd%2C%2Epanel%3E%2Etable%2Dresponsive%3E%2Etable%2Dbordered%3Etfoot%3Etr%3Alast%2Dchild%3Eth%7Bborder%2Dbottom%3A0%7D%2Epanel%3E%2Etable%2Dresponsive%7Bmargin%2Dbottom%3A0%3Bborder%3A0%7D%2Epanel%2Dgroup%7Bmargin%2Dbottom%3A20px%7D%2Epanel%2Dgroup%20%2Epanel%7Bmargin%2Dbottom%3A0%3Bborder%2Dradius%3A4px%7D%2Epanel%2Dgroup%20%2Epanel%2B%2Epanel%7Bmargin%2Dtop%3A5px%7D%2Epanel%2Dgroup%20%2Epanel%2Dheading%7Bborder%2Dbottom%3A0%7D%2Epanel%2Dgroup%20%2Epanel%2Dheading%2B%2Epanel%2Dcollapse%3E%2Elist%2Dgroup%2C%2Epanel%2Dgroup%20%2Epanel%2Dheading%2B%2Epanel%2Dcollapse%3E%2Epanel%2Dbody%7Bborder%2Dtop%3A1px%20solid%20%23ddd%7D%2Epanel%2Dgroup%20%2Epanel%2Dfooter%7Bborder%2Dtop%3A0%7D%2Epanel%2Dgroup%20%2Epanel%2Dfooter%2B%2Epanel%2Dcollapse%20%2Epanel%2Dbody%7Bborder%2Dbottom%3A1px%20solid%20%23ddd%7D%2Epanel%2Ddefault%7Bborder%2Dcolor%3A%23ddd%7D%2Epanel%2Ddefault%3E%2Epanel%2Dheading%7Bcolor%3A%23333%3Bbackground%2Dcolor%3A%23f5f5f5%3Bborder%2Dcolor%3A%23ddd%7D%2Epanel%2Ddefault%3E%2Epanel%2Dheading%2B%2Epanel%2Dcollapse%3E%2Epanel%2Dbody%7Bborder%2Dtop%2Dcolor%3A%23ddd%7D%2Epanel%2Ddefault%3E%2Epanel%2Dheading%20%2Ebadge%7Bcolor%3A%23f5f5f5%3Bbackground%2Dcolor%3A%23333%7D%2Epanel%2Ddefault%3E%2Epanel%2Dfooter%2B%2Epanel%2Dcollapse%3E%2Epanel%2Dbody%7Bborder%2Dbottom%2Dcolor%3A%23ddd%7D%2Epanel%2Dprimary%7Bborder%2Dcolor%3A%23337ab7%7D%2Epanel%2Dprimary%3E%2Epanel%2Dheading%7Bcolor%3A%23fff%3Bbackground%2Dcolor%3A%23337ab7%3Bborder%2Dcolor%3A%23337ab7%7D%2Epanel%2Dprimary%3E%2Epanel%2Dheading%2B%2Epanel%2Dcollapse%3E%2Epanel%2Dbody%7Bborder%2Dtop%2Dcolor%3A%23337ab7%7D%2Epanel%2Dprimary%3E%2Epanel%2Dheading%20%2Ebadge%7Bcolor%3A%23337ab7%3Bbackground%2Dcolor%3A%23fff%7D%2Epanel%2Dprimary%3E%2Epanel%2Dfooter%2B%2Epanel%2Dcollapse%3E%2Epanel%2Dbody%7Bborder%2Dbottom%2Dcolor%3A%23337ab7%7D%2Epanel%2Dsuccess%7Bborder%2Dcolor%3A%23d6e9c6%7D%2Epanel%2Dsuccess%3E%2Epanel%2Dheading%7Bcolor%3A%233c763d%3Bbackground%2Dcolor%3A%23dff0d8%3Bborder%2Dcolor%3A%23d6e9c6%7D%2Epanel%2Dsuccess%3E%2Epanel%2Dheading%2B%2Epanel%2Dcollapse%3E%2Epanel%2Dbody%7Bborder%2Dtop%2Dcolor%3A%23d6e9c6%7D%2Epanel%2Dsuccess%3E%2Epanel%2Dheading%20%2Ebadge%7Bcolor%3A%23dff0d8%3Bbackground%2Dcolor%3A%233c763d%7D%2Epanel%2Dsuccess%3E%2Epanel%2Dfooter%2B%2Epanel%2Dcollapse%3E%2Epanel%2Dbody%7Bborder%2Dbottom%2Dcolor%3A%23d6e9c6%7D%2Epanel%2Dinfo%7Bborder%2Dcolor%3A%23bce8f1%7D%2Epanel%2Dinfo%3E%2Epanel%2Dheading%7Bcolor%3A%2331708f%3Bbackground%2Dcolor%3A%23d9edf7%3Bborder%2Dcolor%3A%23bce8f1%7D%2Epanel%2Dinfo%3E%2Epanel%2Dheading%2B%2Epanel%2Dcollapse%3E%2Epanel%2Dbody%7Bborder%2Dtop%2Dcolor%3A%23bce8f1%7D%2Epanel%2Dinfo%3E%2Epanel%2Dheading%20%2Ebadge%7Bcolor%3A%23d9edf7%3Bbackground%2Dcolor%3A%2331708f%7D%2Epanel%2Dinfo%3E%2Epanel%2Dfooter%2B%2Epanel%2Dcollapse%3E%2Epanel%2Dbody%7Bborder%2Dbottom%2Dcolor%3A%23bce8f1%7D%2Epanel%2Dwarning%7Bborder%2Dcolor%3A%23faebcc%7D%2Epanel%2Dwarning%3E%2Epanel%2Dheading%7Bcolor%3A%238a6d3b%3Bbackground%2Dcolor%3A%23fcf8e3%3Bborder%2Dcolor%3A%23faebcc%7D%2Epanel%2Dwarning%3E%2Epanel%2Dheading%2B%2Epanel%2Dcollapse%3E%2Epanel%2Dbody%7Bborder%2Dtop%2Dcolor%3A%23faebcc%7D%2Epanel%2Dwarning%3E%2Epanel%2Dheading%20%2Ebadge%7Bcolor%3A%23fcf8e3%3Bbackground%2Dcolor%3A%238a6d3b%7D%2Epanel%2Dwarning%3E%2Epanel%2Dfooter%2B%2Epanel%2Dcollapse%3E%2Epanel%2Dbody%7Bborder%2Dbottom%2Dcolor%3A%23faebcc%7D%2Epanel%2Ddanger%7Bborder%2Dcolor%3A%23ebccd1%7D%2Epanel%2Ddanger%3E%2Epanel%2Dheading%7Bcolor%3A%23a94442%3Bbackground%2Dcolor%3A%23f2dede%3Bborder%2Dcolor%3A%23ebccd1%7D%2Epanel%2Ddanger%3E%2Epanel%2Dheading%2B%2Epanel%2Dcollapse%3E%2Epanel%2Dbody%7Bborder%2Dtop%2Dcolor%3A%23ebccd1%7D%2Epanel%2Ddanger%3E%2Epanel%2Dheading%20%2Ebadge%7Bcolor%3A%23f2dede%3Bbackground%2Dcolor%3A%23a94442%7D%2Epanel%2Ddanger%3E%2Epanel%2Dfooter%2B%2Epanel%2Dcollapse%3E%2Epanel%2Dbody%7Bborder%2Dbottom%2Dcolor%3A%23ebccd1%7D%2Eembed%2Dresponsive%7Bposition%3Arelative%3Bdisplay%3Ablock%3Bheight%3A0%3Bpadding%3A0%3Boverflow%3Ahidden%7D%2Eembed%2Dresponsive%20%2Eembed%2Dresponsive%2Ditem%2C%2Eembed%2Dresponsive%20embed%2C%2Eembed%2Dresponsive%20iframe%2C%2Eembed%2Dresponsive%20object%2C%2Eembed%2Dresponsive%20video%7Bposition%3Aabsolute%3Btop%3A0%3Bbottom%3A0%3Bleft%3A0%3Bwidth%3A100%25%3Bheight%3A100%25%3Bborder%3A0%7D%2Eembed%2Dresponsive%2D16by9%7Bpadding%2Dbottom%3A56%2E25%25%7D%2Eembed%2Dresponsive%2D4by3%7Bpadding%2Dbottom%3A75%25%7D%2Ewell%7Bmin%2Dheight%3A20px%3Bpadding%3A19px%3Bmargin%2Dbottom%3A20px%3Bbackground%2Dcolor%3A%23f5f5f5%3Bborder%3A1px%20solid%20%23e3e3e3%3Bborder%2Dradius%3A4px%3B%2Dwebkit%2Dbox%2Dshadow%3Ainset%200%201px%201px%20rgba%280%2C0%2C0%2C%2E05%29%3Bbox%2Dshadow%3Ainset%200%201px%201px%20rgba%280%2C0%2C0%2C%2E05%29%7D%2Ewell%20blockquote%7Bborder%2Dcolor%3A%23ddd%3Bborder%2Dcolor%3Argba%280%2C0%2C0%2C%2E15%29%7D%2Ewell%2Dlg%7Bpadding%3A24px%3Bborder%2Dradius%3A6px%7D%2Ewell%2Dsm%7Bpadding%3A9px%3Bborder%2Dradius%3A3px%7D%2Eclose%7Bfloat%3Aright%3Bfont%2Dsize%3A21px%3Bfont%2Dweight%3A700%3Bline%2Dheight%3A1%3Bcolor%3A%23000%3Btext%2Dshadow%3A0%201px%200%20%23fff%3Bfilter%3Aalpha%28opacity%3D20%29%3Bopacity%3A%2E2%7D%2Eclose%3Afocus%2C%2Eclose%3Ahover%7Bcolor%3A%23000%3Btext%2Ddecoration%3Anone%3Bcursor%3Apointer%3Bfilter%3Aalpha%28opacity%3D50%29%3Bopacity%3A%2E5%7Dbutton%2Eclose%7B%2Dwebkit%2Dappearance%3Anone%3Bpadding%3A0%3Bcursor%3Apointer%3Bbackground%3A0%200%3Bborder%3A0%7D%2Emodal%2Dopen%7Boverflow%3Ahidden%7D%2Emodal%7Bposition%3Afixed%3Btop%3A0%3Bright%3A0%3Bbottom%3A0%3Bleft%3A0%3Bz%2Dindex%3A1050%3Bdisplay%3Anone%3Boverflow%3Ahidden%3B%2Dwebkit%2Doverflow%2Dscrolling%3Atouch%3Boutline%3A0%7D%2Emodal%2Efade%20%2Emodal%2Ddialog%7B%2Dwebkit%2Dtransition%3A%2Dwebkit%2Dtransform%20%2E3s%20ease%2Dout%3B%2Do%2Dtransition%3A%2Do%2Dtransform%20%2E3s%20ease%2Dout%3Btransition%3Atransform%20%2E3s%20ease%2Dout%3B%2Dwebkit%2Dtransform%3Atranslate%280%2C%2D25%25%29%3B%2Dms%2Dtransform%3Atranslate%280%2C%2D25%25%29%3B%2Do%2Dtransform%3Atranslate%280%2C%2D25%25%29%3Btransform%3Atranslate%280%2C%2D25%25%29%7D%2Emodal%2Ein%20%2Emodal%2Ddialog%7B%2Dwebkit%2Dtransform%3Atranslate%280%2C0%29%3B%2Dms%2Dtransform%3Atranslate%280%2C0%29%3B%2Do%2Dtransform%3Atranslate%280%2C0%29%3Btransform%3Atranslate%280%2C0%29%7D%2Emodal%2Dopen%20%2Emodal%7Boverflow%2Dx%3Ahidden%3Boverflow%2Dy%3Aauto%7D%2Emodal%2Ddialog%7Bposition%3Arelative%3Bwidth%3Aauto%3Bmargin%3A10px%7D%2Emodal%2Dcontent%7Bposition%3Arelative%3Bbackground%2Dcolor%3A%23fff%3B%2Dwebkit%2Dbackground%2Dclip%3Apadding%2Dbox%3Bbackground%2Dclip%3Apadding%2Dbox%3Bborder%3A1px%20solid%20%23999%3Bborder%3A1px%20solid%20rgba%280%2C0%2C0%2C%2E2%29%3Bborder%2Dradius%3A6px%3Boutline%3A0%3B%2Dwebkit%2Dbox%2Dshadow%3A0%203px%209px%20rgba%280%2C0%2C0%2C%2E5%29%3Bbox%2Dshadow%3A0%203px%209px%20rgba%280%2C0%2C0%2C%2E5%29%7D%2Emodal%2Dbackdrop%7Bposition%3Afixed%3Btop%3A0%3Bright%3A0%3Bbottom%3A0%3Bleft%3A0%3Bz%2Dindex%3A1040%3Bbackground%2Dcolor%3A%23000%7D%2Emodal%2Dbackdrop%2Efade%7Bfilter%3Aalpha%28opacity%3D0%29%3Bopacity%3A0%7D%2Emodal%2Dbackdrop%2Ein%7Bfilter%3Aalpha%28opacity%3D50%29%3Bopacity%3A%2E5%7D%2Emodal%2Dheader%7Bmin%2Dheight%3A16%2E43px%3Bpadding%3A15px%3Bborder%2Dbottom%3A1px%20solid%20%23e5e5e5%7D%2Emodal%2Dheader%20%2Eclose%7Bmargin%2Dtop%3A%2D2px%7D%2Emodal%2Dtitle%7Bmargin%3A0%3Bline%2Dheight%3A1%2E42857143%7D%2Emodal%2Dbody%7Bposition%3Arelative%3Bpadding%3A15px%7D%2Emodal%2Dfooter%7Bpadding%3A15px%3Btext%2Dalign%3Aright%3Bborder%2Dtop%3A1px%20solid%20%23e5e5e5%7D%2Emodal%2Dfooter%20%2Ebtn%2B%2Ebtn%7Bmargin%2Dbottom%3A0%3Bmargin%2Dleft%3A5px%7D%2Emodal%2Dfooter%20%2Ebtn%2Dgroup%20%2Ebtn%2B%2Ebtn%7Bmargin%2Dleft%3A%2D1px%7D%2Emodal%2Dfooter%20%2Ebtn%2Dblock%2B%2Ebtn%2Dblock%7Bmargin%2Dleft%3A0%7D%2Emodal%2Dscrollbar%2Dmeasure%7Bposition%3Aabsolute%3Btop%3A%2D9999px%3Bwidth%3A50px%3Bheight%3A50px%3Boverflow%3Ascroll%7D%40media%20%28min%2Dwidth%3A768px%29%7B%2Emodal%2Ddialog%7Bwidth%3A600px%3Bmargin%3A30px%20auto%7D%2Emodal%2Dcontent%7B%2Dwebkit%2Dbox%2Dshadow%3A0%205px%2015px%20rgba%280%2C0%2C0%2C%2E5%29%3Bbox%2Dshadow%3A0%205px%2015px%20rgba%280%2C0%2C0%2C%2E5%29%7D%2Emodal%2Dsm%7Bwidth%3A300px%7D%7D%40media%20%28min%2Dwidth%3A992px%29%7B%2Emodal%2Dlg%7Bwidth%3A900px%7D%7D%2Etooltip%7Bposition%3Aabsolute%3Bz%2Dindex%3A1070%3Bdisplay%3Ablock%3Bfont%2Dfamily%3A%22Helvetica%20Neue%22%2CHelvetica%2CArial%2Csans%2Dserif%3Bfont%2Dsize%3A12px%3Bfont%2Dstyle%3Anormal%3Bfont%2Dweight%3A400%3Bline%2Dheight%3A1%2E42857143%3Btext%2Dalign%3Aleft%3Btext%2Dalign%3Astart%3Btext%2Ddecoration%3Anone%3Btext%2Dshadow%3Anone%3Btext%2Dtransform%3Anone%3Bletter%2Dspacing%3Anormal%3Bword%2Dbreak%3Anormal%3Bword%2Dspacing%3Anormal%3Bword%2Dwrap%3Anormal%3Bwhite%2Dspace%3Anormal%3Bfilter%3Aalpha%28opacity%3D0%29%3Bopacity%3A0%3Bline%2Dbreak%3Aauto%7D%2Etooltip%2Ein%7Bfilter%3Aalpha%28opacity%3D90%29%3Bopacity%3A%2E9%7D%2Etooltip%2Etop%7Bpadding%3A5px%200%3Bmargin%2Dtop%3A%2D3px%7D%2Etooltip%2Eright%7Bpadding%3A0%205px%3Bmargin%2Dleft%3A3px%7D%2Etooltip%2Ebottom%7Bpadding%3A5px%200%3Bmargin%2Dtop%3A3px%7D%2Etooltip%2Eleft%7Bpadding%3A0%205px%3Bmargin%2Dleft%3A%2D3px%7D%2Etooltip%2Dinner%7Bmax%2Dwidth%3A200px%3Bpadding%3A3px%208px%3Bcolor%3A%23fff%3Btext%2Dalign%3Acenter%3Bbackground%2Dcolor%3A%23000%3Bborder%2Dradius%3A4px%7D%2Etooltip%2Darrow%7Bposition%3Aabsolute%3Bwidth%3A0%3Bheight%3A0%3Bborder%2Dcolor%3Atransparent%3Bborder%2Dstyle%3Asolid%7D%2Etooltip%2Etop%20%2Etooltip%2Darrow%7Bbottom%3A0%3Bleft%3A50%25%3Bmargin%2Dleft%3A%2D5px%3Bborder%2Dwidth%3A5px%205px%200%3Bborder%2Dtop%2Dcolor%3A%23000%7D%2Etooltip%2Etop%2Dleft%20%2Etooltip%2Darrow%7Bright%3A5px%3Bbottom%3A0%3Bmargin%2Dbottom%3A%2D5px%3Bborder%2Dwidth%3A5px%205px%200%3Bborder%2Dtop%2Dcolor%3A%23000%7D%2Etooltip%2Etop%2Dright%20%2Etooltip%2Darrow%7Bbottom%3A0%3Bleft%3A5px%3Bmargin%2Dbottom%3A%2D5px%3Bborder%2Dwidth%3A5px%205px%200%3Bborder%2Dtop%2Dcolor%3A%23000%7D%2Etooltip%2Eright%20%2Etooltip%2Darrow%7Btop%3A50%25%3Bleft%3A0%3Bmargin%2Dtop%3A%2D5px%3Bborder%2Dwidth%3A5px%205px%205px%200%3Bborder%2Dright%2Dcolor%3A%23000%7D%2Etooltip%2Eleft%20%2Etooltip%2Darrow%7Btop%3A50%25%3Bright%3A0%3Bmargin%2Dtop%3A%2D5px%3Bborder%2Dwidth%3A5px%200%205px%205px%3Bborder%2Dleft%2Dcolor%3A%23000%7D%2Etooltip%2Ebottom%20%2Etooltip%2Darrow%7Btop%3A0%3Bleft%3A50%25%3Bmargin%2Dleft%3A%2D5px%3Bborder%2Dwidth%3A0%205px%205px%3Bborder%2Dbottom%2Dcolor%3A%23000%7D%2Etooltip%2Ebottom%2Dleft%20%2Etooltip%2Darrow%7Btop%3A0%3Bright%3A5px%3Bmargin%2Dtop%3A%2D5px%3Bborder%2Dwidth%3A0%205px%205px%3Bborder%2Dbottom%2Dcolor%3A%23000%7D%2Etooltip%2Ebottom%2Dright%20%2Etooltip%2Darrow%7Btop%3A0%3Bleft%3A5px%3Bmargin%2Dtop%3A%2D5px%3Bborder%2Dwidth%3A0%205px%205px%3Bborder%2Dbottom%2Dcolor%3A%23000%7D%2Epopover%7Bposition%3Aabsolute%3Btop%3A0%3Bleft%3A0%3Bz%2Dindex%3A1060%3Bdisplay%3Anone%3Bmax%2Dwidth%3A276px%3Bpadding%3A1px%3Bfont%2Dfamily%3A%22Helvetica%20Neue%22%2CHelvetica%2CArial%2Csans%2Dserif%3Bfont%2Dsize%3A14px%3Bfont%2Dstyle%3Anormal%3Bfont%2Dweight%3A400%3Bline%2Dheight%3A1%2E42857143%3Btext%2Dalign%3Aleft%3Btext%2Dalign%3Astart%3Btext%2Ddecoration%3Anone%3Btext%2Dshadow%3Anone%3Btext%2Dtransform%3Anone%3Bletter%2Dspacing%3Anormal%3Bword%2Dbreak%3Anormal%3Bword%2Dspacing%3Anormal%3Bword%2Dwrap%3Anormal%3Bwhite%2Dspace%3Anormal%3Bbackground%2Dcolor%3A%23fff%3B%2Dwebkit%2Dbackground%2Dclip%3Apadding%2Dbox%3Bbackground%2Dclip%3Apadding%2Dbox%3Bborder%3A1px%20solid%20%23ccc%3Bborder%3A1px%20solid%20rgba%280%2C0%2C0%2C%2E2%29%3Bborder%2Dradius%3A6px%3B%2Dwebkit%2Dbox%2Dshadow%3A0%205px%2010px%20rgba%280%2C0%2C0%2C%2E2%29%3Bbox%2Dshadow%3A0%205px%2010px%20rgba%280%2C0%2C0%2C%2E2%29%3Bline%2Dbreak%3Aauto%7D%2Epopover%2Etop%7Bmargin%2Dtop%3A%2D10px%7D%2Epopover%2Eright%7Bmargin%2Dleft%3A10px%7D%2Epopover%2Ebottom%7Bmargin%2Dtop%3A10px%7D%2Epopover%2Eleft%7Bmargin%2Dleft%3A%2D10px%7D%2Epopover%2Dtitle%7Bpadding%3A8px%2014px%3Bmargin%3A0%3Bfont%2Dsize%3A14px%3Bbackground%2Dcolor%3A%23f7f7f7%3Bborder%2Dbottom%3A1px%20solid%20%23ebebeb%3Bborder%2Dradius%3A5px%205px%200%200%7D%2Epopover%2Dcontent%7Bpadding%3A9px%2014px%7D%2Epopover%3E%2Earrow%2C%2Epopover%3E%2Earrow%3Aafter%7Bposition%3Aabsolute%3Bdisplay%3Ablock%3Bwidth%3A0%3Bheight%3A0%3Bborder%2Dcolor%3Atransparent%3Bborder%2Dstyle%3Asolid%7D%2Epopover%3E%2Earrow%7Bborder%2Dwidth%3A11px%7D%2Epopover%3E%2Earrow%3Aafter%7Bcontent%3A%22%22%3Bborder%2Dwidth%3A10px%7D%2Epopover%2Etop%3E%2Earrow%7Bbottom%3A%2D11px%3Bleft%3A50%25%3Bmargin%2Dleft%3A%2D11px%3Bborder%2Dtop%2Dcolor%3A%23999%3Bborder%2Dtop%2Dcolor%3Argba%280%2C0%2C0%2C%2E25%29%3Bborder%2Dbottom%2Dwidth%3A0%7D%2Epopover%2Etop%3E%2Earrow%3Aafter%7Bbottom%3A1px%3Bmargin%2Dleft%3A%2D10px%3Bcontent%3A%22%20%22%3Bborder%2Dtop%2Dcolor%3A%23fff%3Bborder%2Dbottom%2Dwidth%3A0%7D%2Epopover%2Eright%3E%2Earrow%7Btop%3A50%25%3Bleft%3A%2D11px%3Bmargin%2Dtop%3A%2D11px%3Bborder%2Dright%2Dcolor%3A%23999%3Bborder%2Dright%2Dcolor%3Argba%280%2C0%2C0%2C%2E25%29%3Bborder%2Dleft%2Dwidth%3A0%7D%2Epopover%2Eright%3E%2Earrow%3Aafter%7Bbottom%3A%2D10px%3Bleft%3A1px%3Bcontent%3A%22%20%22%3Bborder%2Dright%2Dcolor%3A%23fff%3Bborder%2Dleft%2Dwidth%3A0%7D%2Epopover%2Ebottom%3E%2Earrow%7Btop%3A%2D11px%3Bleft%3A50%25%3Bmargin%2Dleft%3A%2D11px%3Bborder%2Dtop%2Dwidth%3A0%3Bborder%2Dbottom%2Dcolor%3A%23999%3Bborder%2Dbottom%2Dcolor%3Argba%280%2C0%2C0%2C%2E25%29%7D%2Epopover%2Ebottom%3E%2Earrow%3Aafter%7Btop%3A1px%3Bmargin%2Dleft%3A%2D10px%3Bcontent%3A%22%20%22%3Bborder%2Dtop%2Dwidth%3A0%3Bborder%2Dbottom%2Dcolor%3A%23fff%7D%2Epopover%2Eleft%3E%2Earrow%7Btop%3A50%25%3Bright%3A%2D11px%3Bmargin%2Dtop%3A%2D11px%3Bborder%2Dright%2Dwidth%3A0%3Bborder%2Dleft%2Dcolor%3A%23999%3Bborder%2Dleft%2Dcolor%3Argba%280%2C0%2C0%2C%2E25%29%7D%2Epopover%2Eleft%3E%2Earrow%3Aafter%7Bright%3A1px%3Bbottom%3A%2D10px%3Bcontent%3A%22%20%22%3Bborder%2Dright%2Dwidth%3A0%3Bborder%2Dleft%2Dcolor%3A%23fff%7D%2Ecarousel%7Bposition%3Arelative%7D%2Ecarousel%2Dinner%7Bposition%3Arelative%3Bwidth%3A100%25%3Boverflow%3Ahidden%7D%2Ecarousel%2Dinner%3E%2Eitem%7Bposition%3Arelative%3Bdisplay%3Anone%3B%2Dwebkit%2Dtransition%3A%2E6s%20ease%2Din%2Dout%20left%3B%2Do%2Dtransition%3A%2E6s%20ease%2Din%2Dout%20left%3Btransition%3A%2E6s%20ease%2Din%2Dout%20left%7D%2Ecarousel%2Dinner%3E%2Eitem%3Ea%3Eimg%2C%2Ecarousel%2Dinner%3E%2Eitem%3Eimg%7Bline%2Dheight%3A1%7D%40media%20all%20and%20%28transform%2D3d%29%2C%28%2Dwebkit%2Dtransform%2D3d%29%7B%2Ecarousel%2Dinner%3E%2Eitem%7B%2Dwebkit%2Dtransition%3A%2Dwebkit%2Dtransform%20%2E6s%20ease%2Din%2Dout%3B%2Do%2Dtransition%3A%2Do%2Dtransform%20%2E6s%20ease%2Din%2Dout%3Btransition%3Atransform%20%2E6s%20ease%2Din%2Dout%3B%2Dwebkit%2Dbackface%2Dvisibility%3Ahidden%3Bbackface%2Dvisibility%3Ahidden%3B%2Dwebkit%2Dperspective%3A1000px%3Bperspective%3A1000px%7D%2Ecarousel%2Dinner%3E%2Eitem%2Eactive%2Eright%2C%2Ecarousel%2Dinner%3E%2Eitem%2Enext%7Bleft%3A0%3B%2Dwebkit%2Dtransform%3Atranslate3d%28100%25%2C0%2C0%29%3Btransform%3Atranslate3d%28100%25%2C0%2C0%29%7D%2Ecarousel%2Dinner%3E%2Eitem%2Eactive%2Eleft%2C%2Ecarousel%2Dinner%3E%2Eitem%2Eprev%7Bleft%3A0%3B%2Dwebkit%2Dtransform%3Atranslate3d%28%2D100%25%2C0%2C0%29%3Btransform%3Atranslate3d%28%2D100%25%2C0%2C0%29%7D%2Ecarousel%2Dinner%3E%2Eitem%2Eactive%2C%2Ecarousel%2Dinner%3E%2Eitem%2Enext%2Eleft%2C%2Ecarousel%2Dinner%3E%2Eitem%2Eprev%2Eright%7Bleft%3A0%3B%2Dwebkit%2Dtransform%3Atranslate3d%280%2C0%2C0%29%3Btransform%3Atranslate3d%280%2C0%2C0%29%7D%7D%2Ecarousel%2Dinner%3E%2Eactive%2C%2Ecarousel%2Dinner%3E%2Enext%2C%2Ecarousel%2Dinner%3E%2Eprev%7Bdisplay%3Ablock%7D%2Ecarousel%2Dinner%3E%2Eactive%7Bleft%3A0%7D%2Ecarousel%2Dinner%3E%2Enext%2C%2Ecarousel%2Dinner%3E%2Eprev%7Bposition%3Aabsolute%3Btop%3A0%3Bwidth%3A100%25%7D%2Ecarousel%2Dinner%3E%2Enext%7Bleft%3A100%25%7D%2Ecarousel%2Dinner%3E%2Eprev%7Bleft%3A%2D100%25%7D%2Ecarousel%2Dinner%3E%2Enext%2Eleft%2C%2Ecarousel%2Dinner%3E%2Eprev%2Eright%7Bleft%3A0%7D%2Ecarousel%2Dinner%3E%2Eactive%2Eleft%7Bleft%3A%2D100%25%7D%2Ecarousel%2Dinner%3E%2Eactive%2Eright%7Bleft%3A100%25%7D%2Ecarousel%2Dcontrol%7Bposition%3Aabsolute%3Btop%3A0%3Bbottom%3A0%3Bleft%3A0%3Bwidth%3A15%25%3Bfont%2Dsize%3A20px%3Bcolor%3A%23fff%3Btext%2Dalign%3Acenter%3Btext%2Dshadow%3A0%201px%202px%20rgba%280%2C0%2C0%2C%2E6%29%3Bfilter%3Aalpha%28opacity%3D50%29%3Bopacity%3A%2E5%7D%2Ecarousel%2Dcontrol%2Eleft%7Bbackground%2Dimage%3A%2Dwebkit%2Dlinear%2Dgradient%28left%2Crgba%280%2C0%2C0%2C%2E5%29%200%2Crgba%280%2C0%2C0%2C%2E0001%29%20100%25%29%3Bbackground%2Dimage%3A%2Do%2Dlinear%2Dgradient%28left%2Crgba%280%2C0%2C0%2C%2E5%29%200%2Crgba%280%2C0%2C0%2C%2E0001%29%20100%25%29%3Bbackground%2Dimage%3A%2Dwebkit%2Dgradient%28linear%2Cleft%20top%2Cright%20top%2Cfrom%28rgba%280%2C0%2C0%2C%2E5%29%29%2Cto%28rgba%280%2C0%2C0%2C%2E0001%29%29%29%3Bbackground%2Dimage%3Alinear%2Dgradient%28to%20right%2Crgba%280%2C0%2C0%2C%2E5%29%200%2Crgba%280%2C0%2C0%2C%2E0001%29%20100%25%29%3Bfilter%3Aprogid%3ADXImageTransform%2EMicrosoft%2Egradient%28startColorstr%3D%27%2380000000%27%2C%20endColorstr%3D%27%2300000000%27%2C%20GradientType%3D1%29%3Bbackground%2Drepeat%3Arepeat%2Dx%7D%2Ecarousel%2Dcontrol%2Eright%7Bright%3A0%3Bleft%3Aauto%3Bbackground%2Dimage%3A%2Dwebkit%2Dlinear%2Dgradient%28left%2Crgba%280%2C0%2C0%2C%2E0001%29%200%2Crgba%280%2C0%2C0%2C%2E5%29%20100%25%29%3Bbackground%2Dimage%3A%2Do%2Dlinear%2Dgradient%28left%2Crgba%280%2C0%2C0%2C%2E0001%29%200%2Crgba%280%2C0%2C0%2C%2E5%29%20100%25%29%3Bbackground%2Dimage%3A%2Dwebkit%2Dgradient%28linear%2Cleft%20top%2Cright%20top%2Cfrom%28rgba%280%2C0%2C0%2C%2E0001%29%29%2Cto%28rgba%280%2C0%2C0%2C%2E5%29%29%29%3Bbackground%2Dimage%3Alinear%2Dgradient%28to%20right%2Crgba%280%2C0%2C0%2C%2E0001%29%200%2Crgba%280%2C0%2C0%2C%2E5%29%20100%25%29%3Bfilter%3Aprogid%3ADXImageTransform%2EMicrosoft%2Egradient%28startColorstr%3D%27%2300000000%27%2C%20endColorstr%3D%27%2380000000%27%2C%20GradientType%3D1%29%3Bbackground%2Drepeat%3Arepeat%2Dx%7D%2Ecarousel%2Dcontrol%3Afocus%2C%2Ecarousel%2Dcontrol%3Ahover%7Bcolor%3A%23fff%3Btext%2Ddecoration%3Anone%3Bfilter%3Aalpha%28opacity%3D90%29%3Boutline%3A0%3Bopacity%3A%2E9%7D%2Ecarousel%2Dcontrol%20%2Eglyphicon%2Dchevron%2Dleft%2C%2Ecarousel%2Dcontrol%20%2Eglyphicon%2Dchevron%2Dright%2C%2Ecarousel%2Dcontrol%20%2Eicon%2Dnext%2C%2Ecarousel%2Dcontrol%20%2Eicon%2Dprev%7Bposition%3Aabsolute%3Btop%3A50%25%3Bz%2Dindex%3A5%3Bdisplay%3Ainline%2Dblock%3Bmargin%2Dtop%3A%2D10px%7D%2Ecarousel%2Dcontrol%20%2Eglyphicon%2Dchevron%2Dleft%2C%2Ecarousel%2Dcontrol%20%2Eicon%2Dprev%7Bleft%3A50%25%3Bmargin%2Dleft%3A%2D10px%7D%2Ecarousel%2Dcontrol%20%2Eglyphicon%2Dchevron%2Dright%2C%2Ecarousel%2Dcontrol%20%2Eicon%2Dnext%7Bright%3A50%25%3Bmargin%2Dright%3A%2D10px%7D%2Ecarousel%2Dcontrol%20%2Eicon%2Dnext%2C%2Ecarousel%2Dcontrol%20%2Eicon%2Dprev%7Bwidth%3A20px%3Bheight%3A20px%3Bfont%2Dfamily%3Aserif%3Bline%2Dheight%3A1%7D%2Ecarousel%2Dcontrol%20%2Eicon%2Dprev%3Abefore%7Bcontent%3A%27%5C2039%27%7D%2Ecarousel%2Dcontrol%20%2Eicon%2Dnext%3Abefore%7Bcontent%3A%27%5C203a%27%7D%2Ecarousel%2Dindicators%7Bposition%3Aabsolute%3Bbottom%3A10px%3Bleft%3A50%25%3Bz%2Dindex%3A15%3Bwidth%3A60%25%3Bpadding%2Dleft%3A0%3Bmargin%2Dleft%3A%2D30%25%3Btext%2Dalign%3Acenter%3Blist%2Dstyle%3Anone%7D%2Ecarousel%2Dindicators%20li%7Bdisplay%3Ainline%2Dblock%3Bwidth%3A10px%3Bheight%3A10px%3Bmargin%3A1px%3Btext%2Dindent%3A%2D999px%3Bcursor%3Apointer%3Bbackground%2Dcolor%3A%23000%5C9%3Bbackground%2Dcolor%3Argba%280%2C0%2C0%2C0%29%3Bborder%3A1px%20solid%20%23fff%3Bborder%2Dradius%3A10px%7D%2Ecarousel%2Dindicators%20%2Eactive%7Bwidth%3A12px%3Bheight%3A12px%3Bmargin%3A0%3Bbackground%2Dcolor%3A%23fff%7D%2Ecarousel%2Dcaption%7Bposition%3Aabsolute%3Bright%3A15%25%3Bbottom%3A20px%3Bleft%3A15%25%3Bz%2Dindex%3A10%3Bpadding%2Dtop%3A20px%3Bpadding%2Dbottom%3A20px%3Bcolor%3A%23fff%3Btext%2Dalign%3Acenter%3Btext%2Dshadow%3A0%201px%202px%20rgba%280%2C0%2C0%2C%2E6%29%7D%2Ecarousel%2Dcaption%20%2Ebtn%7Btext%2Dshadow%3Anone%7D%40media%20screen%20and%20%28min%2Dwidth%3A768px%29%7B%2Ecarousel%2Dcontrol%20%2Eglyphicon%2Dchevron%2Dleft%2C%2Ecarousel%2Dcontrol%20%2Eglyphicon%2Dchevron%2Dright%2C%2Ecarousel%2Dcontrol%20%2Eicon%2Dnext%2C%2Ecarousel%2Dcontrol%20%2Eicon%2Dprev%7Bwidth%3A30px%3Bheight%3A30px%3Bmargin%2Dtop%3A%2D15px%3Bfont%2Dsize%3A30px%7D%2Ecarousel%2Dcontrol%20%2Eglyphicon%2Dchevron%2Dleft%2C%2Ecarousel%2Dcontrol%20%2Eicon%2Dprev%7Bmargin%2Dleft%3A%2D15px%7D%2Ecarousel%2Dcontrol%20%2Eglyphicon%2Dchevron%2Dright%2C%2Ecarousel%2Dcontrol%20%2Eicon%2Dnext%7Bmargin%2Dright%3A%2D15px%7D%2Ecarousel%2Dcaption%7Bright%3A20%25%3Bleft%3A20%25%3Bpadding%2Dbottom%3A30px%7D%2Ecarousel%2Dindicators%7Bbottom%3A20px%7D%7D%2Ebtn%2Dgroup%2Dvertical%3E%2Ebtn%2Dgroup%3Aafter%2C%2Ebtn%2Dgroup%2Dvertical%3E%2Ebtn%2Dgroup%3Abefore%2C%2Ebtn%2Dtoolbar%3Aafter%2C%2Ebtn%2Dtoolbar%3Abefore%2C%2Eclearfix%3Aafter%2C%2Eclearfix%3Abefore%2C%2Econtainer%2Dfluid%3Aafter%2C%2Econtainer%2Dfluid%3Abefore%2C%2Econtainer%3Aafter%2C%2Econtainer%3Abefore%2C%2Edl%2Dhorizontal%20dd%3Aafter%2C%2Edl%2Dhorizontal%20dd%3Abefore%2C%2Eform%2Dhorizontal%20%2Eform%2Dgroup%3Aafter%2C%2Eform%2Dhorizontal%20%2Eform%2Dgroup%3Abefore%2C%2Emodal%2Dfooter%3Aafter%2C%2Emodal%2Dfooter%3Abefore%2C%2Enav%3Aafter%2C%2Enav%3Abefore%2C%2Enavbar%2Dcollapse%3Aafter%2C%2Enavbar%2Dcollapse%3Abefore%2C%2Enavbar%2Dheader%3Aafter%2C%2Enavbar%2Dheader%3Abefore%2C%2Enavbar%3Aafter%2C%2Enavbar%3Abefore%2C%2Epager%3Aafter%2C%2Epager%3Abefore%2C%2Epanel%2Dbody%3Aafter%2C%2Epanel%2Dbody%3Abefore%2C%2Erow%3Aafter%2C%2Erow%3Abefore%7Bdisplay%3Atable%3Bcontent%3A%22%20%22%7D%2Ebtn%2Dgroup%2Dvertical%3E%2Ebtn%2Dgroup%3Aafter%2C%2Ebtn%2Dtoolbar%3Aafter%2C%2Eclearfix%3Aafter%2C%2Econtainer%2Dfluid%3Aafter%2C%2Econtainer%3Aafter%2C%2Edl%2Dhorizontal%20dd%3Aafter%2C%2Eform%2Dhorizontal%20%2Eform%2Dgroup%3Aafter%2C%2Emodal%2Dfooter%3Aafter%2C%2Enav%3Aafter%2C%2Enavbar%2Dcollapse%3Aafter%2C%2Enavbar%2Dheader%3Aafter%2C%2Enavbar%3Aafter%2C%2Epager%3Aafter%2C%2Epanel%2Dbody%3Aafter%2C%2Erow%3Aafter%7Bclear%3Aboth%7D%2Ecenter%2Dblock%7Bdisplay%3Ablock%3Bmargin%2Dright%3Aauto%3Bmargin%2Dleft%3Aauto%7D%2Epull%2Dright%7Bfloat%3Aright%21important%7D%2Epull%2Dleft%7Bfloat%3Aleft%21important%7D%2Ehide%7Bdisplay%3Anone%21important%7D%2Eshow%7Bdisplay%3Ablock%21important%7D%2Einvisible%7Bvisibility%3Ahidden%7D%2Etext%2Dhide%7Bfont%3A0%2F0%20a%3Bcolor%3Atransparent%3Btext%2Dshadow%3Anone%3Bbackground%2Dcolor%3Atransparent%3Bborder%3A0%7D%2Ehidden%7Bdisplay%3Anone%21important%7D%2Eaffix%7Bposition%3Afixed%7D%40%2Dms%2Dviewport%7Bwidth%3Adevice%2Dwidth%7D%2Evisible%2Dlg%2C%2Evisible%2Dmd%2C%2Evisible%2Dsm%2C%2Evisible%2Dxs%7Bdisplay%3Anone%21important%7D%2Evisible%2Dlg%2Dblock%2C%2Evisible%2Dlg%2Dinline%2C%2Evisible%2Dlg%2Dinline%2Dblock%2C%2Evisible%2Dmd%2Dblock%2C%2Evisible%2Dmd%2Dinline%2C%2Evisible%2Dmd%2Dinline%2Dblock%2C%2Evisible%2Dsm%2Dblock%2C%2Evisible%2Dsm%2Dinline%2C%2Evisible%2Dsm%2Dinline%2Dblock%2C%2Evisible%2Dxs%2Dblock%2C%2Evisible%2Dxs%2Dinline%2C%2Evisible%2Dxs%2Dinline%2Dblock%7Bdisplay%3Anone%21important%7D%40media%20%28max%2Dwidth%3A767px%29%7B%2Evisible%2Dxs%7Bdisplay%3Ablock%21important%7Dtable%2Evisible%2Dxs%7Bdisplay%3Atable%21important%7Dtr%2Evisible%2Dxs%7Bdisplay%3Atable%2Drow%21important%7Dtd%2Evisible%2Dxs%2Cth%2Evisible%2Dxs%7Bdisplay%3Atable%2Dcell%21important%7D%7D%40media%20%28max%2Dwidth%3A767px%29%7B%2Evisible%2Dxs%2Dblock%7Bdisplay%3Ablock%21important%7D%7D%40media%20%28max%2Dwidth%3A767px%29%7B%2Evisible%2Dxs%2Dinline%7Bdisplay%3Ainline%21important%7D%7D%40media%20%28max%2Dwidth%3A767px%29%7B%2Evisible%2Dxs%2Dinline%2Dblock%7Bdisplay%3Ainline%2Dblock%21important%7D%7D%40media%20%28min%2Dwidth%3A768px%29%20and%20%28max%2Dwidth%3A991px%29%7B%2Evisible%2Dsm%7Bdisplay%3Ablock%21important%7Dtable%2Evisible%2Dsm%7Bdisplay%3Atable%21important%7Dtr%2Evisible%2Dsm%7Bdisplay%3Atable%2Drow%21important%7Dtd%2Evisible%2Dsm%2Cth%2Evisible%2Dsm%7Bdisplay%3Atable%2Dcell%21important%7D%7D%40media%20%28min%2Dwidth%3A768px%29%20and%20%28max%2Dwidth%3A991px%29%7B%2Evisible%2Dsm%2Dblock%7Bdisplay%3Ablock%21important%7D%7D%40media%20%28min%2Dwidth%3A768px%29%20and%20%28max%2Dwidth%3A991px%29%7B%2Evisible%2Dsm%2Dinline%7Bdisplay%3Ainline%21important%7D%7D%40media%20%28min%2Dwidth%3A768px%29%20and%20%28max%2Dwidth%3A991px%29%7B%2Evisible%2Dsm%2Dinline%2Dblock%7Bdisplay%3Ainline%2Dblock%21important%7D%7D%40media%20%28min%2Dwidth%3A992px%29%20and%20%28max%2Dwidth%3A1199px%29%7B%2Evisible%2Dmd%7Bdisplay%3Ablock%21important%7Dtable%2Evisible%2Dmd%7Bdisplay%3Atable%21important%7Dtr%2Evisible%2Dmd%7Bdisplay%3Atable%2Drow%21important%7Dtd%2Evisible%2Dmd%2Cth%2Evisible%2Dmd%7Bdisplay%3Atable%2Dcell%21important%7D%7D%40media%20%28min%2Dwidth%3A992px%29%20and%20%28max%2Dwidth%3A1199px%29%7B%2Evisible%2Dmd%2Dblock%7Bdisplay%3Ablock%21important%7D%7D%40media%20%28min%2Dwidth%3A992px%29%20and%20%28max%2Dwidth%3A1199px%29%7B%2Evisible%2Dmd%2Dinline%7Bdisplay%3Ainline%21important%7D%7D%40media%20%28min%2Dwidth%3A992px%29%20and%20%28max%2Dwidth%3A1199px%29%7B%2Evisible%2Dmd%2Dinline%2Dblock%7Bdisplay%3Ainline%2Dblock%21important%7D%7D%40media%20%28min%2Dwidth%3A1200px%29%7B%2Evisible%2Dlg%7Bdisplay%3Ablock%21important%7Dtable%2Evisible%2Dlg%7Bdisplay%3Atable%21important%7Dtr%2Evisible%2Dlg%7Bdisplay%3Atable%2Drow%21important%7Dtd%2Evisible%2Dlg%2Cth%2Evisible%2Dlg%7Bdisplay%3Atable%2Dcell%21important%7D%7D%40media%20%28min%2Dwidth%3A1200px%29%7B%2Evisible%2Dlg%2Dblock%7Bdisplay%3Ablock%21important%7D%7D%40media%20%28min%2Dwidth%3A1200px%29%7B%2Evisible%2Dlg%2Dinline%7Bdisplay%3Ainline%21important%7D%7D%40media%20%28min%2Dwidth%3A1200px%29%7B%2Evisible%2Dlg%2Dinline%2Dblock%7Bdisplay%3Ainline%2Dblock%21important%7D%7D%40media%20%28max%2Dwidth%3A767px%29%7B%2Ehidden%2Dxs%7Bdisplay%3Anone%21important%7D%7D%40media%20%28min%2Dwidth%3A768px%29%20and%20%28max%2Dwidth%3A991px%29%7B%2Ehidden%2Dsm%7Bdisplay%3Anone%21important%7D%7D%40media%20%28min%2Dwidth%3A992px%29%20and%20%28max%2Dwidth%3A1199px%29%7B%2Ehidden%2Dmd%7Bdisplay%3Anone%21important%7D%7D%40media%20%28min%2Dwidth%3A1200px%29%7B%2Ehidden%2Dlg%7Bdisplay%3Anone%21important%7D%7D%2Evisible%2Dprint%7Bdisplay%3Anone%21important%7D%40media%20print%7B%2Evisible%2Dprint%7Bdisplay%3Ablock%21important%7Dtable%2Evisible%2Dprint%7Bdisplay%3Atable%21important%7Dtr%2Evisible%2Dprint%7Bdisplay%3Atable%2Drow%21important%7Dtd%2Evisible%2Dprint%2Cth%2Evisible%2Dprint%7Bdisplay%3Atable%2Dcell%21important%7D%7D%2Evisible%2Dprint%2Dblock%7Bdisplay%3Anone%21important%7D%40media%20print%7B%2Evisible%2Dprint%2Dblock%7Bdisplay%3Ablock%21important%7D%7D%2Evisible%2Dprint%2Dinline%7Bdisplay%3Anone%21important%7D%40media%20print%7B%2Evisible%2Dprint%2Dinline%7Bdisplay%3Ainline%21important%7D%7D%2Evisible%2Dprint%2Dinline%2Dblock%7Bdisplay%3Anone%21important%7D%40media%20print%7B%2Evisible%2Dprint%2Dinline%2Dblock%7Bdisplay%3Ainline%2Dblock%21important%7D%7D%40media%20print%7B%2Ehidden%2Dprint%7Bdisplay%3Anone%21important%7D%7D%0A" rel="stylesheet" />
+<script src="data:application/x-javascript;base64,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"></script>
+<script src="data:application/x-javascript;base64,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"></script>
+<script src="data:application/x-javascript;base64,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"></script>
+<script src="data:application/x-javascript;base64,CgovKioKICogalF1ZXJ5IFBsdWdpbjogU3RpY2t5IFRhYnMKICoKICogQGF1dGhvciBBaWRhbiBMaXN0ZXIgPGFpZGFuQHBocC5uZXQ+CiAqIGFkYXB0ZWQgYnkgUnViZW4gQXJzbGFuIHRvIGFjdGl2YXRlIHBhcmVudCB0YWJzIHRvbwogKiBodHRwOi8vd3d3LmFpZGFubGlzdGVyLmNvbS8yMDE0LzAzL3BlcnNpc3RpbmctdGhlLXRhYi1zdGF0ZS1pbi1ib290c3RyYXAvCiAqLwooZnVuY3Rpb24oJCkgewogICJ1c2Ugc3RyaWN0IjsKICAkLmZuLnJtYXJrZG93blN0aWNreVRhYnMgPSBmdW5jdGlvbigpIHsKICAgIHZhciBjb250ZXh0ID0gdGhpczsKICAgIC8vIFNob3cgdGhlIHRhYiBjb3JyZXNwb25kaW5nIHdpdGggdGhlIGhhc2ggaW4gdGhlIFVSTCwgb3IgdGhlIGZpcnN0IHRhYgogICAgdmFyIHNob3dTdHVmZkZyb21IYXNoID0gZnVuY3Rpb24oKSB7CiAgICAgIHZhciBoYXNoID0gd2luZG93LmxvY2F0aW9uLmhhc2g7CiAgICAgIHZhciBzZWxlY3RvciA9IGhhc2ggPyAnYVtocmVmPSInICsgaGFzaCArICciXScgOiAnbGkuYWN0aXZlID4gYSc7CiAgICAgIHZhciAkc2VsZWN0b3IgPSAkKHNlbGVjdG9yLCBjb250ZXh0KTsKICAgICAgaWYoJHNlbGVjdG9yLmRhdGEoJ3RvZ2dsZScpID09PSAidGFiIikgewogICAgICAgICRzZWxlY3Rvci50YWIoJ3Nob3cnKTsKICAgICAgICAvLyB3YWxrIHVwIHRoZSBhbmNlc3RvcnMgb2YgdGhpcyBlbGVtZW50LCBzaG93IGFueSBoaWRkZW4gdGFicwogICAgICAgICRzZWxlY3Rvci5wYXJlbnRzKCcuc2VjdGlvbi50YWJzZXQnKS5lYWNoKGZ1bmN0aW9uKGksIGVsbSkgewogICAgICAgICAgdmFyIGxpbmsgPSAkKCdhW2hyZWY9IiMnICsgJChlbG0pLmF0dHIoJ2lkJykgKyAnIl0nKTsKICAgICAgICAgIGlmKGxpbmsuZGF0YSgndG9nZ2xlJykgPT09ICJ0YWIiKSB7CiAgICAgICAgICAgIGxpbmsudGFiKCJzaG93Iik7CiAgICAgICAgICB9CiAgICAgICAgfSk7CiAgICAgIH0KICAgIH07CgoKICAgIC8vIFNldCB0aGUgY29ycmVjdCB0YWIgd2hlbiB0aGUgcGFnZSBsb2FkcwogICAgc2hvd1N0dWZmRnJvbUhhc2goY29udGV4dCk7CgogICAgLy8gU2V0IHRoZSBjb3JyZWN0IHRhYiB3aGVuIGEgdXNlciB1c2VzIHRoZWlyIGJhY2svZm9yd2FyZCBidXR0b24KICAgICQod2luZG93KS5vbignaGFzaGNoYW5nZScsIGZ1bmN0aW9uKCkgewogICAgICBzaG93U3R1ZmZGcm9tSGFzaChjb250ZXh0KTsKICAgIH0pOwoKICAgIC8vIENoYW5nZSB0aGUgVVJMIHdoZW4gdGFicyBhcmUgY2xpY2tlZAogICAgJCgnYScsIGNvbnRleHQpLm9uKCdjbGljaycsIGZ1bmN0aW9uKGUpIHsKICAgICAgaGlzdG9yeS5wdXNoU3RhdGUobnVsbCwgbnVsbCwgdGhpcy5ocmVmKTsKICAgICAgc2hvd1N0dWZmRnJvbUhhc2goY29udGV4dCk7CiAgICB9KTsKCiAgICByZXR1cm4gdGhpczsKICB9Owp9KGpRdWVyeSkpOwoKd2luZG93LmJ1aWxkVGFic2V0cyA9IGZ1bmN0aW9uKHRvY0lEKSB7CgogIC8vIGJ1aWxkIGEgdGFic2V0IGZyb20gYSBzZWN0aW9uIGRpdiB3aXRoIHRoZSAudGFic2V0IGNsYXNzCiAgZnVuY3Rpb24gYnVpbGRUYWJzZXQodGFic2V0KSB7CgogICAgLy8gY2hlY2sgZm9yIGZhZGUgYW5kIHBpbGxzIG9wdGlvbnMKICAgIHZhciBmYWRlID0gdGFic2V0Lmhhc0NsYXNzKCJ0YWJzZXQtZmFkZSIpOwogICAgdmFyIHBpbGxzID0gdGFic2V0Lmhhc0NsYXNzKCJ0YWJzZXQtcGlsbHMiKTsKICAgIHZhciBuYXZDbGFzcyA9IHBpbGxzID8gIm5hdi1waWxscyIgOiAibmF2LXRhYnMiOwoKICAgIC8vIGRldGVybWluZSB0aGUgaGVhZGluZyBsZXZlbCBvZiB0aGUgdGFic2V0IGFuZCB0YWJzCiAgICB2YXIgbWF0Y2ggPSB0YWJzZXQuYXR0cignY2xhc3MnKS5tYXRjaCgvbGV2ZWwoXGQpIC8pOwogICAgaWYgKG1hdGNoID09PSBudWxsKQogICAgICByZXR1cm47CiAgICB2YXIgdGFic2V0TGV2ZWwgPSBOdW1iZXIobWF0Y2hbMV0pOwogICAgdmFyIHRhYkxldmVsID0gdGFic2V0TGV2ZWwgKyAxOwoKICAgIC8vIGZpbmQgYWxsIHN1YmhlYWRpbmdzIGltbWVkaWF0ZWx5IGJlbG93CiAgICB2YXIgdGFicyA9IHRhYnNldC5maW5kKCJkaXYuc2VjdGlvbi5sZXZlbCIgKyB0YWJMZXZlbCk7CiAgICBpZiAoIXRhYnMubGVuZ3RoKQogICAgICByZXR1cm47CgogICAgLy8gY3JlYXRlIHRhYmxpc3QgYW5kIHRhYi1jb250ZW50IGVsZW1lbnRzCiAgICB2YXIgdGFiTGlzdCA9ICQoJzx1bCBjbGFzcz0ibmF2ICcgKyBuYXZDbGFzcyArICciIHJvbGU9InRhYmxpc3QiPjwvdWw+Jyk7CiAgICAkKHRhYnNbMF0pLmJlZm9yZSh0YWJMaXN0KTsKICAgIHZhciB0YWJDb250ZW50ID0gJCgnPGRpdiBjbGFzcz0idGFiLWNvbnRlbnQiPjwvZGl2PicpOwogICAgJCh0YWJzWzBdKS5iZWZvcmUodGFiQ29udGVudCk7CgogICAgLy8gYnVpbGQgdGhlIHRhYnNldAogICAgdmFyIGFjdGl2ZVRhYiA9IDA7CiAgICB0YWJzLmVhY2goZnVuY3Rpb24oaSkgewoKICAgICAgLy8gZ2V0IHRoZSB0YWIgZGl2CiAgICAgIHZhciB0YWIgPSAkKHRhYnNbaV0pOwoKICAgICAgLy8gZ2V0IHRoZSBpZCB0aGVuIHNhbml0aXplIGl0IGZvciB1c2Ugd2l0aCBib290c3RyYXAgdGFicwogICAgICB2YXIgaWQgPSB0YWIuYXR0cignaWQnKTsKCiAgICAgIC8vIHNlZSBpZiB0aGlzIGlzIG1hcmtlZCBhcyB0aGUgYWN0aXZlIHRhYgogICAgICBpZiAodGFiLmhhc0NsYXNzKCdhY3RpdmUnKSkKICAgICAgICBhY3RpdmVUYWIgPSBpOwoKICAgICAgLy8gcmVtb3ZlIGFueSB0YWJsZSBvZiBjb250ZW50cyBlbnRyaWVzIGFzc29jaWF0ZWQgd2l0aAogICAgICAvLyB0aGlzIElEIChzaW5jZSB3ZSdsbCBiZSByZW1vdmluZyB0aGUgaGVhZGluZyBlbGVtZW50KQogICAgICAkKCJkaXYjIiArIHRvY0lEICsgIiBsaSBhW2hyZWY9JyMiICsgaWQgKyAiJ10iKS5wYXJlbnQoKS5yZW1vdmUoKTsKCiAgICAgIC8vIHNhbml0aXplIHRoZSBpZCBmb3IgdXNlIHdpdGggYm9vdHN0cmFwIHRhYnMKICAgICAgaWQgPSBpZC5yZXBsYWNlKC9bLlwvPyYhIzw+XS9nLCAnJykucmVwbGFjZSgvXHMvZywgJ18nKTsKICAgICAgdGFiLmF0dHIoJ2lkJywgaWQpOwoKICAgICAgLy8gZ2V0IHRoZSBoZWFkaW5nIGVsZW1lbnQgd2l0aGluIGl0LCBncmFiIGl0J3MgdGV4dCwgdGhlbiByZW1vdmUgaXQKICAgICAgdmFyIGhlYWRpbmcgPSB0YWIuZmluZCgnaCcgKyB0YWJMZXZlbCArICc6Zmlyc3QnKTsKICAgICAgdmFyIGhlYWRpbmdUZXh0ID0gaGVhZGluZy5odG1sKCk7CiAgICAgIGhlYWRpbmcucmVtb3ZlKCk7CgogICAgICAvLyBidWlsZCBhbmQgYXBwZW5kIHRoZSB0YWIgbGlzdCBpdGVtCiAgICAgIHZhciBhID0gJCgnPGEgcm9sZT0idGFiIiBkYXRhLXRvZ2dsZT0idGFiIj4nICsgaGVhZGluZ1RleHQgKyAnPC9hPicpOwogICAgICBhLmF0dHIoJ2hyZWYnLCAnIycgKyBpZCk7CiAgICAgIGEuYXR0cignYXJpYS1jb250cm9scycsIGlkKTsKICAgICAgdmFyIGxpID0gJCgnPGxpIHJvbGU9InByZXNlbnRhdGlvbiI+PC9saT4nKTsKICAgICAgbGkuYXBwZW5kKGEpOwogICAgICB0YWJMaXN0LmFwcGVuZChsaSk7CgogICAgICAvLyBzZXQgaXQncyBhdHRyaWJ1dGVzCiAgICAgIHRhYi5hdHRyKCdyb2xlJywgJ3RhYnBhbmVsJyk7CiAgICAgIHRhYi5hZGRDbGFzcygndGFiLXBhbmUnKTsKICAgICAgdGFiLmFkZENsYXNzKCd0YWJiZWQtcGFuZScpOwogICAgICBpZiAoZmFkZSkKICAgICAgICB0YWIuYWRkQ2xhc3MoJ2ZhZGUnKTsKCiAgICAgIC8vIG1vdmUgaXQgaW50byB0aGUgdGFiIGNvbnRlbnQgZGl2CiAgICAgIHRhYi5kZXRhY2goKS5hcHBlbmRUbyh0YWJDb250ZW50KTsKICAgIH0pOwoKICAgIC8vIHNldCBhY3RpdmUgdGFiCiAgICAkKHRhYkxpc3QuY2hpbGRyZW4oJ2xpJylbYWN0aXZlVGFiXSkuYWRkQ2xhc3MoJ2FjdGl2ZScpOwogICAgdmFyIGFjdGl2ZSA9ICQodGFiQ29udGVudC5jaGlsZHJlbignZGl2LnNlY3Rpb24nKVthY3RpdmVUYWJdKTsKICAgIGFjdGl2ZS5hZGRDbGFzcygnYWN0aXZlJyk7CiAgICBpZiAoZmFkZSkKICAgICAgYWN0aXZlLmFkZENsYXNzKCdpbicpOwoKICAgIGlmICh0YWJzZXQuaGFzQ2xhc3MoInRhYnNldC1zdGlja3kiKSkKICAgICAgdGFic2V0LnJtYXJrZG93blN0aWNreVRhYnMoKTsKICB9CgogIC8vIGNvbnZlcnQgc2VjdGlvbiBkaXZzIHdpdGggdGhlIC50YWJzZXQgY2xhc3MgdG8gdGFic2V0cwogIHZhciB0YWJzZXRzID0gJCgiZGl2LnNlY3Rpb24udGFic2V0Iik7CiAgdGFic2V0cy5lYWNoKGZ1bmN0aW9uKGkpIHsKICAgIGJ1aWxkVGFic2V0KCQodGFic2V0c1tpXSkpOwogIH0pOwp9OwoK"></script>
+<link href="data:text/css;charset=utf-8,%2Ehljs%2Dliteral%20%7B%0Acolor%3A%20%23990073%3B%0A%7D%0A%2Ehljs%2Dnumber%20%7B%0Acolor%3A%20%23099%3B%0A%7D%0A%2Ehljs%2Dcomment%20%7B%0Acolor%3A%20%23998%3B%0Afont%2Dstyle%3A%20italic%3B%0A%7D%0A%2Ehljs%2Dkeyword%20%7B%0Acolor%3A%20%23900%3B%0Afont%2Dweight%3A%20bold%3B%0A%7D%0A%2Ehljs%2Dstring%20%7B%0Acolor%3A%20%23d14%3B%0A%7D%0A" rel="stylesheet" />
+<script src="data:application/x-javascript;base64,LyohIGhpZ2hsaWdodC5qcyB2OS4xMi4wIHwgQlNEMyBMaWNlbnNlIHwgZ2l0LmlvL2hsanNsaWNlbnNlICovCiFmdW5jdGlvbihlKXt2YXIgbj0ib2JqZWN0Ij09dHlwZW9mIHdpbmRvdyYmd2luZG93fHwib2JqZWN0Ij09dHlwZW9mIHNlbGYmJnNlbGY7InVuZGVmaW5lZCIhPXR5cGVvZiBleHBvcnRzP2UoZXhwb3J0cyk6biYmKG4uaGxqcz1lKHt9KSwiZnVuY3Rpb24iPT10eXBlb2YgZGVmaW5lJiZkZWZpbmUuYW1kJiZkZWZpbmUoW10sZnVuY3Rpb24oKXtyZXR1cm4gbi5obGpzfSkpfShmdW5jdGlvbihlKXtmdW5jdGlvbiBuKGUpe3JldHVybiBlLnJlcGxhY2UoLyYvZywiJmFtcDsiKS5yZXBsYWNlKC88L2csIiZsdDsiKS5yZXBsYWNlKC8+L2csIiZndDsiKX1mdW5jdGlvbiB0KGUpe3JldHVybiBlLm5vZGVOYW1lLnRvTG93ZXJDYXNlKCl9ZnVuY3Rpb24gcihlLG4pe3ZhciB0PWUmJmUuZXhlYyhuKTtyZXR1cm4gdCYmMD09PXQuaW5kZXh9ZnVuY3Rpb24gYShlKXtyZXR1cm4gay50ZXN0KGUpfWZ1bmN0aW9uIGkoZSl7dmFyIG4sdCxyLGksbz1lLmNsYXNzTmFtZSsiICI7aWYobys9ZS5wYXJlbnROb2RlP2UucGFyZW50Tm9kZS5jbGFzc05hbWU6IiIsdD1CLmV4ZWMobykpcmV0dXJuIHcodFsxXSk/dFsxXToibm8taGlnaGxpZ2h0Ijtmb3Iobz1vLnNwbGl0KC9ccysvKSxuPTAscj1vLmxlbmd0aDtyPm47bisrKWlmKGk9b1tuXSxhKGkpfHx3KGkpKXJldHVybiBpfWZ1bmN0aW9uIG8oZSl7dmFyIG4sdD17fSxyPUFycmF5LnByb3RvdHlwZS5zbGljZS5jYWxsKGFyZ3VtZW50cywxKTtmb3IobiBpbiBlKXRbbl09ZVtuXTtyZXR1cm4gci5mb3JFYWNoKGZ1bmN0aW9uKGUpe2ZvcihuIGluIGUpdFtuXT1lW25dfSksdH1mdW5jdGlvbiB1KGUpe3ZhciBuPVtdO3JldHVybiBmdW5jdGlvbiByKGUsYSl7Zm9yKHZhciBpPWUuZmlyc3RDaGlsZDtpO2k9aS5uZXh0U2libGluZykzPT09aS5ub2RlVHlwZT9hKz1pLm5vZGVWYWx1ZS5sZW5ndGg6MT09PWkubm9kZVR5cGUmJihuLnB1c2goe2V2ZW50OiJzdGFydCIsb2Zmc2V0OmEsbm9kZTppfSksYT1yKGksYSksdChpKS5tYXRjaCgvYnJ8aHJ8aW1nfGlucHV0Lyl8fG4ucHVzaCh7ZXZlbnQ6InN0b3AiLG9mZnNldDphLG5vZGU6aX0pKTtyZXR1cm4gYX0oZSwwKSxufWZ1bmN0aW9uIGMoZSxyLGEpe2Z1bmN0aW9uIGkoKXtyZXR1cm4gZS5sZW5ndGgmJnIubGVuZ3RoP2VbMF0ub2Zmc2V0IT09clswXS5vZmZzZXQ/ZVswXS5vZmZzZXQ8clswXS5vZmZzZXQ/ZTpyOiJzdGFydCI9PT1yWzBdLmV2ZW50P2U6cjplLmxlbmd0aD9lOnJ9ZnVuY3Rpb24gbyhlKXtmdW5jdGlvbiByKGUpe3JldHVybiIgIitlLm5vZGVOYW1lKyc9IicrbihlLnZhbHVlKS5yZXBsYWNlKCciJywiJnF1b3Q7IikrJyInfXMrPSI8Iit0KGUpK0UubWFwLmNhbGwoZS5hdHRyaWJ1dGVzLHIpLmpvaW4oIiIpKyI+In1mdW5jdGlvbiB1KGUpe3MrPSI8LyIrdChlKSsiPiJ9ZnVuY3Rpb24gYyhlKXsoInN0YXJ0Ij09PWUuZXZlbnQ/bzp1KShlLm5vZGUpfWZvcih2YXIgbD0wLHM9IiIsZj1bXTtlLmxlbmd0aHx8ci5sZW5ndGg7KXt2YXIgZz1pKCk7aWYocys9bihhLnN1YnN0cmluZyhsLGdbMF0ub2Zmc2V0KSksbD1nWzBdLm9mZnNldCxnPT09ZSl7Zi5yZXZlcnNlKCkuZm9yRWFjaCh1KTtkbyBjKGcuc3BsaWNlKDAsMSlbMF0pLGc9aSgpO3doaWxlKGc9PT1lJiZnLmxlbmd0aCYmZ1swXS5vZmZzZXQ9PT1sKTtmLnJldmVyc2UoKS5mb3JFYWNoKG8pfWVsc2Uic3RhcnQiPT09Z1swXS5ldmVudD9mLnB1c2goZ1swXS5ub2RlKTpmLnBvcCgpLGMoZy5zcGxpY2UoMCwxKVswXSl9cmV0dXJuIHMrbihhLnN1YnN0cihsKSl9ZnVuY3Rpb24gbChlKXtyZXR1cm4gZS52JiYhZS5jYWNoZWRfdmFyaWFudHMmJihlLmNhY2hlZF92YXJpYW50cz1lLnYubWFwKGZ1bmN0aW9uKG4pe3JldHVybiBvKGUse3Y6bnVsbH0sbil9KSksZS5jYWNoZWRfdmFyaWFudHN8fGUuZVcmJltvKGUpXXx8W2VdfWZ1bmN0aW9uIHMoZSl7ZnVuY3Rpb24gbihlKXtyZXR1cm4gZSYmZS5zb3VyY2V8fGV9ZnVuY3Rpb24gdCh0LHIpe3JldHVybiBuZXcgUmVnRXhwKG4odCksIm0iKyhlLmNJPyJpIjoiIikrKHI/ImciOiIiKSl9ZnVuY3Rpb24gcihhLGkpe2lmKCFhLmNvbXBpbGVkKXtpZihhLmNvbXBpbGVkPSEwLGEuaz1hLmt8fGEuYkssYS5rKXt2YXIgbz17fSx1PWZ1bmN0aW9uKG4sdCl7ZS5jSSYmKHQ9dC50b0xvd2VyQ2FzZSgpKSx0LnNwbGl0KCIgIikuZm9yRWFjaChmdW5jdGlvbihlKXt2YXIgdD1lLnNwbGl0KCJ8Iik7b1t0WzBdXT1bbix0WzFdP051bWJlcih0WzFdKToxXX0pfTsic3RyaW5nIj09dHlwZW9mIGEuaz91KCJrZXl3b3JkIixhLmspOngoYS5rKS5mb3JFYWNoKGZ1bmN0aW9uKGUpe3UoZSxhLmtbZV0pfSksYS5rPW99YS5sUj10KGEubHx8L1x3Ky8sITApLGkmJihhLmJLJiYoYS5iPSJcXGIoIithLmJLLnNwbGl0KCIgIikuam9pbigifCIpKyIpXFxiIiksYS5ifHwoYS5iPS9cQnxcYi8pLGEuYlI9dChhLmIpLGEuZXx8YS5lV3x8KGEuZT0vXEJ8XGIvKSxhLmUmJihhLmVSPXQoYS5lKSksYS50RT1uKGEuZSl8fCIiLGEuZVcmJmkudEUmJihhLnRFKz0oYS5lPyJ8IjoiIikraS50RSkpLGEuaSYmKGEuaVI9dChhLmkpKSxudWxsPT1hLnImJihhLnI9MSksYS5jfHwoYS5jPVtdKSxhLmM9QXJyYXkucHJvdG90eXBlLmNvbmNhdC5hcHBseShbXSxhLmMubWFwKGZ1bmN0aW9uKGUpe3JldHVybiBsKCJzZWxmIj09PWU/YTplKX0pKSxhLmMuZm9yRWFjaChmdW5jdGlvbihlKXtyKGUsYSl9KSxhLnN0YXJ0cyYmcihhLnN0YXJ0cyxpKTt2YXIgYz1hLmMubWFwKGZ1bmN0aW9uKGUpe3JldHVybiBlLmJLPyJcXC4/KCIrZS5iKyIpXFwuPyI6ZS5ifSkuY29uY2F0KFthLnRFLGEuaV0pLm1hcChuKS5maWx0ZXIoQm9vbGVhbik7YS50PWMubGVuZ3RoP3QoYy5qb2luKCJ8IiksITApOntleGVjOmZ1bmN0aW9uKCl7cmV0dXJuIG51bGx9fX19cihlKX1mdW5jdGlvbiBmKGUsdCxhLGkpe2Z1bmN0aW9uIG8oZSxuKXt2YXIgdCxhO2Zvcih0PTAsYT1uLmMubGVuZ3RoO2E+dDt0KyspaWYocihuLmNbdF0uYlIsZSkpcmV0dXJuIG4uY1t0XX1mdW5jdGlvbiB1KGUsbil7aWYocihlLmVSLG4pKXtmb3IoO2UuZW5kc1BhcmVudCYmZS5wYXJlbnQ7KWU9ZS5wYXJlbnQ7cmV0dXJuIGV9cmV0dXJuIGUuZVc/dShlLnBhcmVudCxuKTp2b2lkIDB9ZnVuY3Rpb24gYyhlLG4pe3JldHVybiFhJiZyKG4uaVIsZSl9ZnVuY3Rpb24gbChlLG4pe3ZhciB0PU4uY0k/blswXS50b0xvd2VyQ2FzZSgpOm5bMF07cmV0dXJuIGUuay5oYXNPd25Qcm9wZXJ0eSh0KSYmZS5rW3RdfWZ1bmN0aW9uIHAoZSxuLHQscil7dmFyIGE9cj8iIjpJLmNsYXNzUHJlZml4LGk9JzxzcGFuIGNsYXNzPSInK2Esbz10PyIiOkM7cmV0dXJuIGkrPWUrJyI+JyxpK24rb31mdW5jdGlvbiBoKCl7dmFyIGUsdCxyLGE7aWYoIUUuaylyZXR1cm4gbihrKTtmb3IoYT0iIix0PTAsRS5sUi5sYXN0SW5kZXg9MCxyPUUubFIuZXhlYyhrKTtyOylhKz1uKGsuc3Vic3RyaW5nKHQsci5pbmRleCkpLGU9bChFLHIpLGU/KEIrPWVbMV0sYSs9cChlWzBdLG4oclswXSkpKTphKz1uKHJbMF0pLHQ9RS5sUi5sYXN0SW5kZXgscj1FLmxSLmV4ZWMoayk7cmV0dXJuIGErbihrLnN1YnN0cih0KSl9ZnVuY3Rpb24gZCgpe3ZhciBlPSJzdHJpbmciPT10eXBlb2YgRS5zTDtpZihlJiYheVtFLnNMXSlyZXR1cm4gbihrKTt2YXIgdD1lP2YoRS5zTCxrLCEwLHhbRS5zTF0pOmcoayxFLnNMLmxlbmd0aD9FLnNMOnZvaWQgMCk7cmV0dXJuIEUucj4wJiYoQis9dC5yKSxlJiYoeFtFLnNMXT10LnRvcCkscCh0Lmxhbmd1YWdlLHQudmFsdWUsITEsITApfWZ1bmN0aW9uIGIoKXtMKz1udWxsIT1FLnNMP2QoKTpoKCksaz0iIn1mdW5jdGlvbiB2KGUpe0wrPWUuY04/cChlLmNOLCIiLCEwKToiIixFPU9iamVjdC5jcmVhdGUoZSx7cGFyZW50Ont2YWx1ZTpFfX0pfWZ1bmN0aW9uIG0oZSxuKXtpZihrKz1lLG51bGw9PW4pcmV0dXJuIGIoKSwwO3ZhciB0PW8obixFKTtpZih0KXJldHVybiB0LnNraXA/ays9bjoodC5lQiYmKGsrPW4pLGIoKSx0LnJCfHx0LmVCfHwoaz1uKSksdih0LG4pLHQuckI/MDpuLmxlbmd0aDt2YXIgcj11KEUsbik7aWYocil7dmFyIGE9RTthLnNraXA/ays9bjooYS5yRXx8YS5lRXx8KGsrPW4pLGIoKSxhLmVFJiYoaz1uKSk7ZG8gRS5jTiYmKEwrPUMpLEUuc2tpcHx8KEIrPUUuciksRT1FLnBhcmVudDt3aGlsZShFIT09ci5wYXJlbnQpO3JldHVybiByLnN0YXJ0cyYmdihyLnN0YXJ0cywiIiksYS5yRT8wOm4ubGVuZ3RofWlmKGMobixFKSl0aHJvdyBuZXcgRXJyb3IoJ0lsbGVnYWwgbGV4ZW1lICInK24rJyIgZm9yIG1vZGUgIicrKEUuY058fCI8dW5uYW1lZD4iKSsnIicpO3JldHVybiBrKz1uLG4ubGVuZ3RofHwxfXZhciBOPXcoZSk7aWYoIU4pdGhyb3cgbmV3IEVycm9yKCdVbmtub3duIGxhbmd1YWdlOiAiJytlKyciJyk7cyhOKTt2YXIgUixFPWl8fE4seD17fSxMPSIiO2ZvcihSPUU7UiE9PU47Uj1SLnBhcmVudClSLmNOJiYoTD1wKFIuY04sIiIsITApK0wpO3ZhciBrPSIiLEI9MDt0cnl7Zm9yKHZhciBNLGosTz0wOzspe2lmKEUudC5sYXN0SW5kZXg9TyxNPUUudC5leGVjKHQpLCFNKWJyZWFrO2o9bSh0LnN1YnN0cmluZyhPLE0uaW5kZXgpLE1bMF0pLE89TS5pbmRleCtqfWZvcihtKHQuc3Vic3RyKE8pKSxSPUU7Ui5wYXJlbnQ7Uj1SLnBhcmVudClSLmNOJiYoTCs9Qyk7cmV0dXJue3I6Qix2YWx1ZTpMLGxhbmd1YWdlOmUsdG9wOkV9fWNhdGNoKFQpe2lmKFQubWVzc2FnZSYmLTEhPT1ULm1lc3NhZ2UuaW5kZXhPZigiSWxsZWdhbCIpKXJldHVybntyOjAsdmFsdWU6bih0KX07dGhyb3cgVH19ZnVuY3Rpb24gZyhlLHQpe3Q9dHx8SS5sYW5ndWFnZXN8fHgoeSk7dmFyIHI9e3I6MCx2YWx1ZTpuKGUpfSxhPXI7cmV0dXJuIHQuZmlsdGVyKHcpLmZvckVhY2goZnVuY3Rpb24obil7dmFyIHQ9ZihuLGUsITEpO3QubGFuZ3VhZ2U9bix0LnI+YS5yJiYoYT10KSx0LnI+ci5yJiYoYT1yLHI9dCl9KSxhLmxhbmd1YWdlJiYoci5zZWNvbmRfYmVzdD1hKSxyfWZ1bmN0aW9uIHAoZSl7cmV0dXJuIEkudGFiUmVwbGFjZXx8SS51c2VCUj9lLnJlcGxhY2UoTSxmdW5jdGlvbihlLG4pe3JldHVybiBJLnVzZUJSJiYiXG4iPT09ZT8iPGJyPiI6SS50YWJSZXBsYWNlP24ucmVwbGFjZSgvXHQvZyxJLnRhYlJlcGxhY2UpOiIifSk6ZX1mdW5jdGlvbiBoKGUsbix0KXt2YXIgcj1uP0xbbl06dCxhPVtlLnRyaW0oKV07cmV0dXJuIGUubWF0Y2goL1xiaGxqc1xiLyl8fGEucHVzaCgiaGxqcyIpLC0xPT09ZS5pbmRleE9mKHIpJiZhLnB1c2gociksYS5qb2luKCIgIikudHJpbSgpfWZ1bmN0aW9uIGQoZSl7dmFyIG4sdCxyLG8sbCxzPWkoZSk7YShzKXx8KEkudXNlQlI/KG49ZG9jdW1lbnQuY3JlYXRlRWxlbWVudE5TKCJodHRwOi8vd3d3LnczLm9yZy8xOTk5L3hodG1sIiwiZGl2Iiksbi5pbm5lckhUTUw9ZS5pbm5lckhUTUwucmVwbGFjZSgvXG4vZywiIikucmVwbGFjZSgvPGJyWyBcL10qPi9nLCJcbiIpKTpuPWUsbD1uLnRleHRDb250ZW50LHI9cz9mKHMsbCwhMCk6ZyhsKSx0PXUobiksdC5sZW5ndGgmJihvPWRvY3VtZW50LmNyZWF0ZUVsZW1lbnROUygiaHR0cDovL3d3dy53My5vcmcvMTk5OS94aHRtbCIsImRpdiIpLG8uaW5uZXJIVE1MPXIudmFsdWUsci52YWx1ZT1jKHQsdShvKSxsKSksci52YWx1ZT1wKHIudmFsdWUpLGUuaW5uZXJIVE1MPXIudmFsdWUsZS5jbGFzc05hbWU9aChlLmNsYXNzTmFtZSxzLHIubGFuZ3VhZ2UpLGUucmVzdWx0PXtsYW5ndWFnZTpyLmxhbmd1YWdlLHJlOnIucn0sci5zZWNvbmRfYmVzdCYmKGUuc2Vjb25kX2Jlc3Q9e2xhbmd1YWdlOnIuc2Vjb25kX2Jlc3QubGFuZ3VhZ2UscmU6ci5zZWNvbmRfYmVzdC5yfSkpfWZ1bmN0aW9uIGIoZSl7ST1vKEksZSl9ZnVuY3Rpb24gdigpe2lmKCF2LmNhbGxlZCl7di5jYWxsZWQ9ITA7dmFyIGU9ZG9jdW1lbnQucXVlcnlTZWxlY3RvckFsbCgicHJlIGNvZGUiKTtFLmZvckVhY2guY2FsbChlLGQpfX1mdW5jdGlvbiBtKCl7YWRkRXZlbnRMaXN0ZW5lcigiRE9NQ29udGVudExvYWRlZCIsdiwhMSksYWRkRXZlbnRMaXN0ZW5lcigibG9hZCIsdiwhMSl9ZnVuY3Rpb24gTihuLHQpe3ZhciByPXlbbl09dChlKTtyLmFsaWFzZXMmJnIuYWxpYXNlcy5mb3JFYWNoKGZ1bmN0aW9uKGUpe0xbZV09bn0pfWZ1bmN0aW9uIFIoKXtyZXR1cm4geCh5KX1mdW5jdGlvbiB3KGUpe3JldHVybiBlPShlfHwiIikudG9Mb3dlckNhc2UoKSx5W2VdfHx5W0xbZV1dfXZhciBFPVtdLHg9T2JqZWN0LmtleXMseT17fSxMPXt9LGs9L14obm8tP2hpZ2hsaWdodHxwbGFpbnx0ZXh0KSQvaSxCPS9cYmxhbmcoPzp1YWdlKT8tKFtcdy1dKylcYi9pLE09LygoXig8W14+XSs+fFx0fCkrfCg/OlxuKSkpL2dtLEM9Ijwvc3Bhbj4iLEk9e2NsYXNzUHJlZml4OiJobGpzLSIsdGFiUmVwbGFjZTpudWxsLHVzZUJSOiExLGxhbmd1YWdlczp2b2lkIDB9O3JldHVybiBlLmhpZ2hsaWdodD1mLGUuaGlnaGxpZ2h0QXV0bz1nLGUuZml4TWFya3VwPXAsZS5oaWdobGlnaHRCbG9jaz1kLGUuY29uZmlndXJlPWIsZS5pbml0SGlnaGxpZ2h0aW5nPXYsZS5pbml0SGlnaGxpZ2h0aW5nT25Mb2FkPW0sZS5yZWdpc3Rlckxhbmd1YWdlPU4sZS5saXN0TGFuZ3VhZ2VzPVIsZS5nZXRMYW5ndWFnZT13LGUuaW5oZXJpdD1vLGUuSVI9IlthLXpBLVpdXFx3KiIsZS5VSVI9IlthLXpBLVpfXVxcdyoiLGUuTlI9IlxcYlxcZCsoXFwuXFxkKyk/IixlLkNOUj0iKC0/KShcXGIwW3hYXVthLWZBLUYwLTldK3woXFxiXFxkKyhcXC5cXGQqKT98XFwuXFxkKykoW2VFXVstK10/XFxkKyk/KSIsZS5CTlI9IlxcYigwYlswMV0rKSIsZS5SU1I9IiF8IT18IT09fCV8JT18JnwmJnwmPXxcXCp8XFwqPXxcXCt8XFwrPXwsfC18LT18Lz18L3w6fDt8PDx8PDw9fDw9fDx8PT09fD09fD18Pj4+PXw+Pj18Pj18Pj4+fD4+fD58XFw/fFxcW3xcXHt8XFwofFxcXnxcXF49fFxcfHxcXHw9fFxcfFxcfHx+IixlLkJFPXtiOiJcXFxcW1xcc1xcU10iLHI6MH0sZS5BU009e2NOOiJzdHJpbmciLGI6IiciLGU6IiciLGk6IlxcbiIsYzpbZS5CRV19LGUuUVNNPXtjTjoic3RyaW5nIixiOiciJyxlOiciJyxpOiJcXG4iLGM6W2UuQkVdfSxlLlBXTT17YjovXGIoYXxhbnx0aGV8YXJlfEknbXxpc24ndHxkb24ndHxkb2Vzbid0fHdvbid0fGJ1dHxqdXN0fHNob3VsZHxwcmV0dHl8c2ltcGx5fGVub3VnaHxnb25uYXxnb2luZ3x3dGZ8c298c3VjaHx3aWxsfHlvdXx5b3VyfHRoZXl8bGlrZXxtb3JlKVxiL30sZS5DPWZ1bmN0aW9uKG4sdCxyKXt2YXIgYT1lLmluaGVyaXQoe2NOOiJjb21tZW50IixiOm4sZTp0LGM6W119LHJ8fHt9KTtyZXR1cm4gYS5jLnB1c2goZS5QV00pLGEuYy5wdXNoKHtjTjoiZG9jdGFnIixiOiIoPzpUT0RPfEZJWE1FfE5PVEV8QlVHfFhYWCk6IixyOjB9KSxhfSxlLkNMQ009ZS5DKCIvLyIsIiQiKSxlLkNCQ009ZS5DKCIvXFwqIiwiXFwqLyIpLGUuSENNPWUuQygiIyIsIiQiKSxlLk5NPXtjTjoibnVtYmVyIixiOmUuTlIscjowfSxlLkNOTT17Y046Im51bWJlciIsYjplLkNOUixyOjB9LGUuQk5NPXtjTjoibnVtYmVyIixiOmUuQk5SLHI6MH0sZS5DU1NOTT17Y046Im51bWJlciIsYjplLk5SKyIoJXxlbXxleHxjaHxyZW18dnd8dmh8dm1pbnx2bWF4fGNtfG1tfGlufHB0fHBjfHB4fGRlZ3xncmFkfHJhZHx0dXJufHN8bXN8SHp8a0h6fGRwaXxkcGNtfGRwcHgpPyIscjowfSxlLlJNPXtjTjoicmVnZXhwIixiOi9cLy8sZTovXC9bZ2ltdXldKi8saTovXG4vLGM6W2UuQkUse2I6L1xbLyxlOi9cXS8scjowLGM6W2UuQkVdfV19LGUuVE09e2NOOiJ0aXRsZSIsYjplLklSLHI6MH0sZS5VVE09e2NOOiJ0aXRsZSIsYjplLlVJUixyOjB9LGUuTUVUSE9EX0dVQVJEPXtiOiJcXC5cXHMqIitlLlVJUixyOjB9LGV9KTtobGpzLnJlZ2lzdGVyTGFuZ3VhZ2UoInNxbCIsZnVuY3Rpb24oZSl7dmFyIHQ9ZS5DKCItLSIsIiQiKTtyZXR1cm57Y0k6ITAsaTovWzw+e30qI10vLGM6W3tiSzoiYmVnaW4gZW5kIHN0YXJ0IGNvbW1pdCByb2xsYmFjayBzYXZlcG9pbnQgbG9jayBhbHRlciBjcmVhdGUgZHJvcCByZW5hbWUgY2FsbCBkZWxldGUgZG8gaGFuZGxlciBpbnNlcnQgbG9hZCByZXBsYWNlIHNlbGVjdCB0cnVuY2F0ZSB1cGRhdGUgc2V0IHNob3cgcHJhZ21hIGdyYW50IG1lcmdlIGRlc2NyaWJlIHVzZSBleHBsYWluIGhlbHAgZGVjbGFyZSBwcmVwYXJlIGV4ZWN1dGUgZGVhbGxvY2F0ZSByZWxlYXNlIHVubG9jayBwdXJnZSByZXNldCBjaGFuZ2Ugc3RvcCBhbmFseXplIGNhY2hlIGZsdXNoIG9wdGltaXplIHJlcGFpciBraWxsIGluc3RhbGwgdW5pbnN0YWxsIGNoZWNrc3VtIHJlc3RvcmUgY2hlY2sgYmFja3VwIHJldm9rZSBjb21tZW50IixlOi87LyxlVzohMCxsOi9bXHdcLl0rLyxrOntrZXl3b3JkOiJhYm9ydCBhYnMgYWJzb2x1dGUgYWNjIGFjY2UgYWNjZXAgYWNjZXB0IGFjY2VzcyBhY2Nlc3NlZCBhY2Nlc3NpYmxlIGFjY291bnQgYWNvcyBhY3Rpb24gYWN0aXZhdGUgYWRkIGFkZHRpbWUgYWRtaW4gYWRtaW5pc3RlciBhZHZhbmNlZCBhZHZpc2UgYWVzX2RlY3J5cHQgYWVzX2VuY3J5cHQgYWZ0ZXIgYWdlbnQgYWdncmVnYXRlIGFsaSBhbGlhIGFsaWFzIGFsbG9jYXRlIGFsbG93IGFsdGVyIGFsd2F5cyBhbmFseXplIGFuY2lsbGFyeSBhbmQgYW55IGFueWRhdGEgYW55ZGF0YXNldCBhbnlzY2hlbWEgYW55dHlwZSBhcHBseSBhcmNoaXZlIGFyY2hpdmVkIGFyY2hpdmVsb2cgYXJlIGFzIGFzYyBhc2NpaSBhc2luIGFzc2VtYmx5IGFzc2VydGlvbiBhc3NvY2lhdGUgYXN5bmNocm9ub3VzIGF0IGF0YW4gYXRuMiBhdHRyIGF0dHJpIGF0dHJpYiBhdHRyaWJ1IGF0dHJpYnV0IGF0dHJpYnV0ZSBhdHRyaWJ1dGVzIGF1ZGl0IGF1dGhlbnRpY2F0ZWQgYXV0aGVudGljYXRpb24gYXV0aGlkIGF1dGhvcnMgYXV0byBhdXRvYWxsb2NhdGUgYXV0b2RibGluayBhdXRvZXh0ZW5kIGF1dG9tYXRpYyBhdmFpbGFiaWxpdHkgYXZnIGJhY2t1cCBiYWRmaWxlIGJhc2ljZmlsZSBiZWZvcmUgYmVnaW4gYmVnaW5uaW5nIGJlbmNobWFyayBiZXR3ZWVuIGJmaWxlIGJmaWxlX2Jhc2UgYmlnIGJpZ2ZpbGUgYmluIGJpbmFyeV9kb3VibGUgYmluYXJ5X2Zsb2F0IGJpbmxvZyBiaXRfYW5kIGJpdF9jb3VudCBiaXRfbGVuZ3RoIGJpdF9vciBiaXRfeG9yIGJpdG1hcCBibG9iX2Jhc2UgYmxvY2sgYmxvY2tzaXplIGJvZHkgYm90aCBib3VuZCBidWZmZXJfY2FjaGUgYnVmZmVyX3Bvb2wgYnVpbGQgYnVsayBieSBieXRlIGJ5dGVvcmRlcm1hcmsgYnl0ZXMgY2FjaGUgY2FjaGluZyBjYWxsIGNhbGxpbmcgY2FuY2VsIGNhcGFjaXR5IGNhc2NhZGUgY2FzY2FkZWQgY2FzZSBjYXN0IGNhdGFsb2cgY2F0ZWdvcnkgY2VpbCBjZWlsaW5nIGNoYWluIGNoYW5nZSBjaGFuZ2VkIGNoYXJfYmFzZSBjaGFyX2xlbmd0aCBjaGFyYWN0ZXJfbGVuZ3RoIGNoYXJhY3RlcnMgY2hhcmFjdGVyc2V0IGNoYXJpbmRleCBjaGFyc2V0IGNoYXJzZXRmb3JtIGNoYXJzZXRpZCBjaGVjayBjaGVja3N1bSBjaGVja3N1bV9hZ2cgY2hpbGQgY2hvb3NlIGNociBjaHVuayBjbGFzcyBjbGVhbnVwIGNsZWFyIGNsaWVudCBjbG9iIGNsb2JfYmFzZSBjbG9uZSBjbG9zZSBjbHVzdGVyX2lkIGNsdXN0ZXJfcHJvYmFiaWxpdHkgY2x1c3Rlcl9zZXQgY2x1c3RlcmluZyBjb2FsZXNjZSBjb2VyY2liaWxpdHkgY29sIGNvbGxhdGUgY29sbGF0aW9uIGNvbGxlY3QgY29sdSBjb2x1bSBjb2x1bW4gY29sdW1uX3ZhbHVlIGNvbHVtbnMgY29sdW1uc191cGRhdGVkIGNvbW1lbnQgY29tbWl0IGNvbXBhY3QgY29tcGF0aWJpbGl0eSBjb21waWxlZCBjb21wbGV0ZSBjb21wb3NpdGVfbGltaXQgY29tcG91bmQgY29tcHJlc3MgY29tcHV0ZSBjb25jYXQgY29uY2F0X3dzIGNvbmN1cnJlbnQgY29uZmlybSBjb25uIGNvbm5lYyBjb25uZWN0IGNvbm5lY3RfYnlfaXNjeWNsZSBjb25uZWN0X2J5X2lzbGVhZiBjb25uZWN0X2J5X3Jvb3QgY29ubmVjdF90aW1lIGNvbm5lY3Rpb24gY29uc2lkZXIgY29uc2lzdGVudCBjb25zdGFudCBjb25zdHJhaW50IGNvbnN0cmFpbnRzIGNvbnN0cnVjdG9yIGNvbnRhaW5lciBjb250ZW50IGNvbnRlbnRzIGNvbnRleHQgY29udHJpYnV0b3JzIGNvbnRyb2xmaWxlIGNvbnYgY29udmVydCBjb252ZXJ0X3R6IGNvcnIgY29ycl9rIGNvcnJfcyBjb3JyZXNwb25kaW5nIGNvcnJ1cHRpb24gY29zIGNvc3QgY291bnQgY291bnRfYmlnIGNvdW50ZWQgY292YXJfcG9wIGNvdmFyX3NhbXAgY3B1X3Blcl9jYWxsIGNwdV9wZXJfc2Vzc2lvbiBjcmMzMiBjcmVhdGUgY3JlYXRpb24gY3JpdGljYWwgY3Jvc3MgY3ViZSBjdW1lX2Rpc3QgY3VyZGF0ZSBjdXJyZW50IGN1cnJlbnRfZGF0ZSBjdXJyZW50X3RpbWUgY3VycmVudF90aW1lc3RhbXAgY3VycmVudF91c2VyIGN1cnNvciBjdXJ0aW1lIGN1c3RvbWRhdHVtIGN5Y2xlIGRhdGEgZGF0YWJhc2UgZGF0YWJhc2VzIGRhdGFmaWxlIGRhdGFmaWxlcyBkYXRhbGVuZ3RoIGRhdGVfYWRkIGRhdGVfY2FjaGUgZGF0ZV9mb3JtYXQgZGF0ZV9zdWIgZGF0ZWFkZCBkYXRlZGlmZiBkYXRlZnJvbXBhcnRzIGRhdGVuYW1lIGRhdGVwYXJ0IGRhdGV0aW1lMmZyb21wYXJ0cyBkYXkgZGF5X3RvX3NlY29uZCBkYXluYW1lIGRheW9mbW9udGggZGF5b2Z3ZWVrIGRheW9meWVhciBkYXlzIGRiX3JvbGVfY2hhbmdlIGRidGltZXpvbmUgZGRsIGRlYWxsb2NhdGUgZGVjbGFyZSBkZWNvZGUgZGVjb21wb3NlIGRlY3JlbWVudCBkZWNyeXB0IGRlZHVwbGljYXRlIGRlZiBkZWZhIGRlZmF1IGRlZmF1bCBkZWZhdWx0IGRlZmF1bHRzIGRlZmVycmVkIGRlZmkgZGVmaW4gZGVmaW5lIGRlZ3JlZXMgZGVsYXllZCBkZWxlZ2F0ZSBkZWxldGUgZGVsZXRlX2FsbCBkZWxpbWl0ZWQgZGVtYW5kIGRlbnNlX3JhbmsgZGVwdGggZGVxdWV1ZSBkZXNfZGVjcnlwdCBkZXNfZW5jcnlwdCBkZXNfa2V5X2ZpbGUgZGVzYyBkZXNjciBkZXNjcmkgZGVzY3JpYiBkZXNjcmliZSBkZXNjcmlwdG9yIGRldGVybWluaXN0aWMgZGlhZ25vc3RpY3MgZGlmZmVyZW5jZSBkaW1lbnNpb24gZGlyZWN0X2xvYWQgZGlyZWN0b3J5IGRpc2FibGUgZGlzYWJsZV9hbGwgZGlzYWxsb3cgZGlzYXNzb2NpYXRlIGRpc2NhcmRmaWxlIGRpc2Nvbm5lY3QgZGlza2dyb3VwIGRpc3RpbmN0IGRpc3RpbmN0cm93IGRpc3RyaWJ1dGUgZGlzdHJpYnV0ZWQgZGl2IGRvIGRvY3VtZW50IGRvbWFpbiBkb3RuZXQgZG91YmxlIGRvd25ncmFkZSBkcm9wIGR1bXBmaWxlIGR1cGxpY2F0ZSBkdXJhdGlvbiBlYWNoIGVkaXRpb24gZWRpdGlvbmFibGUgZWRpdGlvbnMgZWxlbWVudCBlbGxpcHNpcyBlbHNlIGVsc2lmIGVsdCBlbXB0eSBlbmFibGUgZW5hYmxlX2FsbCBlbmNsb3NlZCBlbmNvZGUgZW5jb2RpbmcgZW5jcnlwdCBlbmQgZW5kLWV4ZWMgZW5kaWFuIGVuZm9yY2VkIGVuZ2luZSBlbmdpbmVzIGVucXVldWUgZW50ZXJwcmlzZSBlbnRpdHllc2NhcGluZyBlb21vbnRoIGVycm9yIGVycm9ycyBlc2NhcGVkIGV2YWxuYW1lIGV2YWx1YXRlIGV2ZW50IGV2ZW50ZGF0YSBldmVudHMgZXhjZXB0IGV4Y2VwdGlvbiBleGNlcHRpb25zIGV4Y2hhbmdlIGV4Y2x1ZGUgZXhjbHVkaW5nIGV4ZWN1IGV4ZWN1dCBleGVjdXRlIGV4ZW1wdCBleGlzdHMgZXhpdCBleHAgZXhwaXJlIGV4cGxhaW4gZXhwb3J0IGV4cG9ydF9zZXQgZXh0ZW5kZWQgZXh0ZW50IGV4dGVybmFsIGV4dGVybmFsXzEgZXh0ZXJuYWxfMiBleHRlcm5hbGx5IGV4dHJhY3QgZmFpbGVkIGZhaWxlZF9sb2dpbl9hdHRlbXB0cyBmYWlsb3ZlciBmYWlsdXJlIGZhciBmYXN0IGZlYXR1cmVfc2V0IGZlYXR1cmVfdmFsdWUgZmV0Y2ggZmllbGQgZmllbGRzIGZpbGUgZmlsZV9uYW1lX2NvbnZlcnQgZmlsZXN5c3RlbV9saWtlX2xvZ2dpbmcgZmluYWwgZmluaXNoIGZpcnN0IGZpcnN0X3ZhbHVlIGZpeGVkIGZsYXNoX2NhY2hlIGZsYXNoYmFjayBmbG9vciBmbHVzaCBmb2xsb3dpbmcgZm9sbG93cyBmb3IgZm9yYWxsIGZvcmNlIGZvcm0gZm9ybWEgZm9ybWF0IGZvdW5kIGZvdW5kX3Jvd3MgZnJlZWxpc3QgZnJlZWxpc3RzIGZyZWVwb29scyBmcmVzaCBmcm9tIGZyb21fYmFzZTY0IGZyb21fZGF5cyBmdHAgZnVsbCBmdW5jdGlvbiBnZW5lcmFsIGdlbmVyYXRlZCBnZXQgZ2V0X2Zvcm1hdCBnZXRfbG9jayBnZXRkYXRlIGdldHV0Y2RhdGUgZ2xvYmFsIGdsb2JhbF9uYW1lIGdsb2JhbGx5IGdvIGdvdG8gZ3JhbnQgZ3JhbnRzIGdyZWF0ZXN0IGdyb3VwIGdyb3VwX2NvbmNhdCBncm91cF9pZCBncm91cGluZyBncm91cGluZ19pZCBncm91cHMgZ3RpZF9zdWJ0cmFjdCBndWFyYW50ZWUgZ3VhcmQgaGFuZGxlciBoYXNoIGhhc2hrZXlzIGhhdmluZyBoZWEgaGVhZCBoZWFkaSBoZWFkaW4gaGVhZGluZyBoZWFwIGhlbHAgaGV4IGhpZXJhcmNoeSBoaWdoIGhpZ2hfcHJpb3JpdHkgaG9zdHMgaG91ciBodHRwIGlkIGlkZW50X2N1cnJlbnQgaWRlbnRfaW5jciBpZGVudF9zZWVkIGlkZW50aWZpZWQgaWRlbnRpdHkgaWRsZV90aW1lIGlmIGlmbnVsbCBpZ25vcmUgaWlmIGlsaWtlIGlsbSBpbW1lZGlhdGUgaW1wb3J0IGluIGluY2x1ZGUgaW5jbHVkaW5nIGluY3JlbWVudCBpbmRleCBpbmRleGVzIGluZGV4aW5nIGluZGV4dHlwZSBpbmRpY2F0b3IgaW5kaWNlcyBpbmV0Nl9hdG9uIGluZXQ2X250b2EgaW5ldF9hdG9uIGluZXRfbnRvYSBpbmZpbGUgaW5pdGlhbCBpbml0aWFsaXplZCBpbml0aWFsbHkgaW5pdHJhbnMgaW5tZW1vcnkgaW5uZXIgaW5ub2RiIGlucHV0IGluc2VydCBpbnN0YWxsIGluc3RhbmNlIGluc3RhbnRpYWJsZSBpbnN0ciBpbnRlcmZhY2UgaW50ZXJsZWF2ZWQgaW50ZXJzZWN0IGludG8gaW52YWxpZGF0ZSBpbnZpc2libGUgaXMgaXNfZnJlZV9sb2NrIGlzX2lwdjQgaXNfaXB2NF9jb21wYXQgaXNfbm90IGlzX25vdF9udWxsIGlzX3VzZWRfbG9jayBpc2RhdGUgaXNudWxsIGlzb2xhdGlvbiBpdGVyYXRlIGphdmEgam9pbiBqc29uIGpzb25fZXhpc3RzIGtlZXAga2VlcF9kdXBsaWNhdGVzIGtleSBrZXlzIGtpbGwgbGFuZ3VhZ2UgbGFyZ2UgbGFzdCBsYXN0X2RheSBsYXN0X2luc2VydF9pZCBsYXN0X3ZhbHVlIGxheCBsY2FzZSBsZWFkIGxlYWRpbmcgbGVhc3QgbGVhdmVzIGxlZnQgbGVuIGxlbmdodCBsZW5ndGggbGVzcyBsZXZlbCBsZXZlbHMgbGlicmFyeSBsaWtlIGxpa2UyIGxpa2U0IGxpa2VjIGxpbWl0IGxpbmVzIGxpbmsgbGlzdCBsaXN0YWdnIGxpdHRsZSBsbiBsb2FkIGxvYWRfZmlsZSBsb2IgbG9icyBsb2NhbCBsb2NhbHRpbWUgbG9jYWx0aW1lc3RhbXAgbG9jYXRlIGxvY2F0b3IgbG9jayBsb2NrZWQgbG9nIGxvZzEwIGxvZzIgbG9nZmlsZSBsb2dmaWxlcyBsb2dnaW5nIGxvZ2ljYWwgbG9naWNhbF9yZWFkc19wZXJfY2FsbCBsb2dvZmYgbG9nb24gbG9ncyBsb25nIGxvb3AgbG93IGxvd19wcmlvcml0eSBsb3dlciBscGFkIGxydHJpbSBsdHJpbSBtYWluIG1ha2Vfc2V0IG1ha2VkYXRlIG1ha2V0aW1lIG1hbmFnZWQgbWFuYWdlbWVudCBtYW51YWwgbWFwIG1hcHBpbmcgbWFzayBtYXN0ZXIgbWFzdGVyX3Bvc193YWl0IG1hdGNoIG1hdGNoZWQgbWF0ZXJpYWxpemVkIG1heCBtYXhleHRlbnRzIG1heGltaXplIG1heGluc3RhbmNlcyBtYXhsZW4gbWF4bG9nZmlsZXMgbWF4bG9naGlzdG9yeSBtYXhsb2dtZW1iZXJzIG1heHNpemUgbWF4dHJhbnMgbWQ1IG1lYXN1cmVzIG1lZGlhbiBtZWRpdW0gbWVtYmVyIG1lbWNvbXByZXNzIG1lbW9yeSBtZXJnZSBtaWNyb3NlY29uZCBtaWQgbWlncmF0aW9uIG1pbiBtaW5leHRlbnRzIG1pbmltdW0gbWluaW5nIG1pbnVzIG1pbnV0ZSBtaW52YWx1ZSBtaXNzaW5nIG1vZCBtb2RlIG1vZGVsIG1vZGlmaWNhdGlvbiBtb2RpZnkgbW9kdWxlIG1vbml0b3JpbmcgbW9udGggbW9udGhzIG1vdW50IG1vdmUgbW92ZW1lbnQgbXVsdGlzZXQgbXV0ZXggbmFtZSBuYW1lX2NvbnN0IG5hbWVzIG5hbiBuYXRpb25hbCBuYXRpdmUgbmF0dXJhbCBuYXYgbmNoYXIgbmNsb2IgbmVzdGVkIG5ldmVyIG5ldyBuZXdsaW5lIG5leHQgbmV4dHZhbCBubyBub193cml0ZV90b19iaW5sb2cgbm9hcmNoaXZlbG9nIG5vYXVkaXQgbm9iYWRmaWxlIG5vY2hlY2sgbm9jb21wcmVzcyBub2NvcHkgbm9jeWNsZSBub2RlbGF5IG5vZGlzY2FyZGZpbGUgbm9lbnRpdHllc2NhcGluZyBub2d1YXJhbnRlZSBub2tlZXAgbm9sb2dmaWxlIG5vbWFwcGluZyBub21heHZhbHVlIG5vbWluaW1pemUgbm9taW52YWx1ZSBub21vbml0b3Jpbmcgbm9uZSBub25lZGl0aW9uYWJsZSBub25zY2hlbWEgbm9vcmRlciBub3ByIG5vcHJvIG5vcHJvbSBub3Byb21wIG5vcHJvbXB0IG5vcmVseSBub3Jlc2V0bG9ncyBub3JldmVyc2Ugbm9ybWFsIG5vcm93ZGVwZW5kZW5jaWVzIG5vc2NoZW1hY2hlY2sgbm9zd2l0Y2ggbm90IG5vdGhpbmcgbm90aWNlIG5vdHJpbSBub3ZhbGlkYXRlIG5vdyBub3dhaXQgbnRoX3ZhbHVlIG51bGxpZiBudWxscyBudW0gbnVtYiBudW1iZSBudmFyY2hhciBudmFyY2hhcjIgb2JqZWN0IG9jaWNvbGwgb2NpZGF0ZSBvY2lkYXRldGltZSBvY2lkdXJhdGlvbiBvY2lpbnRlcnZhbCBvY2lsb2Jsb2NhdG9yIG9jaW51bWJlciBvY2lyZWYgb2NpcmVmY3Vyc29yIG9jaXJvd2lkIG9jaXN0cmluZyBvY2l0eXBlIG9jdCBvY3RldF9sZW5ndGggb2Ygb2ZmIG9mZmxpbmUgb2Zmc2V0IG9pZCBvaWRpbmRleCBvbGQgb24gb25saW5lIG9ubHkgb3BhcXVlIG9wZW4gb3BlcmF0aW9ucyBvcGVyYXRvciBvcHRpbWFsIG9wdGltaXplIG9wdGlvbiBvcHRpb25hbGx5IG9yIG9yYWNsZSBvcmFjbGVfZGF0ZSBvcmFkYXRhIG9yZCBvcmRhdWRpbyBvcmRkaWNvbSBvcmRkb2Mgb3JkZXIgb3JkaW1hZ2Ugb3JkaW5hbGl0eSBvcmR2aWRlbyBvcmdhbml6YXRpb24gb3JsYW55IG9ybHZhcnkgb3V0IG91dGVyIG91dGZpbGUgb3V0bGluZSBvdXRwdXQgb3ZlciBvdmVyZmxvdyBvdmVycmlkaW5nIHBhY2thZ2UgcGFkIHBhcmFsbGVsIHBhcmFsbGVsX2VuYWJsZSBwYXJhbWV0ZXJzIHBhcmVudCBwYXJzZSBwYXJ0aWFsIHBhcnRpdGlvbiBwYXJ0aXRpb25zIHBhc2NhbCBwYXNzaW5nIHBhc3N3b3JkIHBhc3N3b3JkX2dyYWNlX3RpbWUgcGFzc3dvcmRfbG9ja190aW1lIHBhc3N3b3JkX3JldXNlX21heCBwYXNzd29yZF9yZXVzZV90aW1lIHBhc3N3b3JkX3ZlcmlmeV9mdW5jdGlvbiBwYXRjaCBwYXRoIHBhdGluZGV4IHBjdGluY3JlYXNlIHBjdHRocmVzaG9sZCBwY3R1c2VkIHBjdHZlcnNpb24gcGVyY2VudCBwZXJjZW50X3JhbmsgcGVyY2VudGlsZV9jb250IHBlcmNlbnRpbGVfZGlzYyBwZXJmb3JtYW5jZSBwZXJpb2QgcGVyaW9kX2FkZCBwZXJpb2RfZGlmZiBwZXJtYW5lbnQgcGh5c2ljYWwgcGkgcGlwZSBwaXBlbGluZWQgcGl2b3QgcGx1Z2dhYmxlIHBsdWdpbiBwb2xpY3kgcG9zaXRpb24gcG9zdF90cmFuc2FjdGlvbiBwb3cgcG93ZXIgcHJhZ21hIHByZWJ1aWx0IHByZWNlZGVzIHByZWNlZGluZyBwcmVjaXNpb24gcHJlZGljdGlvbiBwcmVkaWN0aW9uX2Nvc3QgcHJlZGljdGlvbl9kZXRhaWxzIHByZWRpY3Rpb25fcHJvYmFiaWxpdHkgcHJlZGljdGlvbl9zZXQgcHJlcGFyZSBwcmVzZW50IHByZXNlcnZlIHByaW9yIHByaW9yaXR5IHByaXZhdGUgcHJpdmF0ZV9zZ2EgcHJpdmlsZWdlcyBwcm9jZWR1cmFsIHByb2NlZHVyZSBwcm9jZWR1cmVfYW5hbHl6ZSBwcm9jZXNzbGlzdCBwcm9maWxlcyBwcm9qZWN0IHByb21wdCBwcm90ZWN0aW9uIHB1YmxpYyBwdWJsaXNoaW5nc2VydmVybmFtZSBwdXJnZSBxdWFydGVyIHF1ZXJ5IHF1aWNrIHF1aWVzY2UgcXVvdGEgcXVvdGVuYW1lIHJhZGlhbnMgcmFpc2UgcmFuZCByYW5nZSByYW5rIHJhdyByZWFkIHJlYWRzIHJlYWRzaXplIHJlYnVpbGQgcmVjb3JkIHJlY29yZHMgcmVjb3ZlciByZWNvdmVyeSByZWN1cnNpdmUgcmVjeWNsZSByZWRvIHJlZHVjZWQgcmVmIHJlZmVyZW5jZSByZWZlcmVuY2VkIHJlZmVyZW5jZXMgcmVmZXJlbmNpbmcgcmVmcmVzaCByZWdleHBfbGlrZSByZWdpc3RlciByZWdyX2F2Z3ggcmVncl9hdmd5IHJlZ3JfY291bnQgcmVncl9pbnRlcmNlcHQgcmVncl9yMiByZWdyX3Nsb3BlIHJlZ3Jfc3h4IHJlZ3Jfc3h5IHJlamVjdCByZWtleSByZWxhdGlvbmFsIHJlbGF0aXZlIHJlbGF5bG9nIHJlbGVhc2UgcmVsZWFzZV9sb2NrIHJlbGllc19vbiByZWxvY2F0ZSByZWx5IHJlbSByZW1haW5kZXIgcmVuYW1lIHJlcGFpciByZXBlYXQgcmVwbGFjZSByZXBsaWNhdGUgcmVwbGljYXRpb24gcmVxdWlyZWQgcmVzZXQgcmVzZXRsb2dzIHJlc2l6ZSByZXNvdXJjZSByZXNwZWN0IHJlc3RvcmUgcmVzdHJpY3RlZCByZXN1bHQgcmVzdWx0X2NhY2hlIHJlc3VtYWJsZSByZXN1bWUgcmV0ZW50aW9uIHJldHVybiByZXR1cm5pbmcgcmV0dXJucyByZXVzZSByZXZlcnNlIHJldm9rZSByaWdodCBybGlrZSByb2xlIHJvbGVzIHJvbGxiYWNrIHJvbGxpbmcgcm9sbHVwIHJvdW5kIHJvdyByb3dfY291bnQgcm93ZGVwZW5kZW5jaWVzIHJvd2lkIHJvd251bSByb3dzIHJ0cmltIHJ1bGVzIHNhZmUgc2FsdCBzYW1wbGUgc2F2ZSBzYXZlcG9pbnQgc2IxIHNiMiBzYjQgc2NhbiBzY2hlbWEgc2NoZW1hY2hlY2sgc2NuIHNjb3BlIHNjcm9sbCBzZG9fZ2VvcmFzdGVyIHNkb190b3BvX2dlb21ldHJ5IHNlYXJjaCBzZWNfdG9fdGltZSBzZWNvbmQgc2VjdGlvbiBzZWN1cmVmaWxlIHNlY3VyaXR5IHNlZWQgc2VnbWVudCBzZWxlY3Qgc2VsZiBzZXF1ZW5jZSBzZXF1ZW50aWFsIHNlcmlhbGl6YWJsZSBzZXJ2ZXIgc2VydmVyZXJyb3Igc2Vzc2lvbiBzZXNzaW9uX3VzZXIgc2Vzc2lvbnNfcGVyX3VzZXIgc2V0IHNldHMgc2V0dGluZ3Mgc2hhIHNoYTEgc2hhMiBzaGFyZSBzaGFyZWQgc2hhcmVkX3Bvb2wgc2hvcnQgc2hvdyBzaHJpbmsgc2h1dGRvd24gc2lfYXZlcmFnZWNvbG9yIHNpX2NvbG9yaGlzdG9ncmFtIHNpX2ZlYXR1cmVsaXN0IHNpX3Bvc2l0aW9uYWxjb2xvciBzaV9zdGlsbGltYWdlIHNpX3RleHR1cmUgc2libGluZ3Mgc2lkIHNpZ24gc2luIHNpemUgc2l6ZV90IHNpemVzIHNraXAgc2xhdmUgc2xlZXAgc21hbGxkYXRldGltZWZyb21wYXJ0cyBzbWFsbGZpbGUgc25hcHNob3Qgc29tZSBzb25hbWUgc29ydCBzb3VuZGV4IHNvdXJjZSBzcGFjZSBzcGFyc2Ugc3BmaWxlIHNwbGl0IHNxbCBzcWxfYmlnX3Jlc3VsdCBzcWxfYnVmZmVyX3Jlc3VsdCBzcWxfY2FjaGUgc3FsX2NhbGNfZm91bmRfcm93cyBzcWxfc21hbGxfcmVzdWx0IHNxbF92YXJpYW50X3Byb3BlcnR5IHNxbGNvZGUgc3FsZGF0YSBzcWxlcnJvciBzcWxuYW1lIHNxbHN0YXRlIHNxcnQgc3F1YXJlIHN0YW5kYWxvbmUgc3RhbmRieSBzdGFydCBzdGFydGluZyBzdGFydHVwIHN0YXRlbWVudCBzdGF0aWMgc3RhdGlzdGljcyBzdGF0c19iaW5vbWlhbF90ZXN0IHN0YXRzX2Nyb3NzdGFiIHN0YXRzX2tzX3Rlc3Qgc3RhdHNfbW9kZSBzdGF0c19td190ZXN0IHN0YXRzX29uZV93YXlfYW5vdmEgc3RhdHNfdF90ZXN0XyBzdGF0c190X3Rlc3RfaW5kZXAgc3RhdHNfdF90ZXN0X29uZSBzdGF0c190X3Rlc3RfcGFpcmVkIHN0YXRzX3dzcl90ZXN0IHN0YXR1cyBzdGQgc3RkZGV2IHN0ZGRldl9wb3Agc3RkZGV2X3NhbXAgc3RkZXYgc3RvcCBzdG9yYWdlIHN0b3JlIHN0b3JlZCBzdHIgc3RyX3RvX2RhdGUgc3RyYWlnaHRfam9pbiBzdHJjbXAgc3RyaWN0IHN0cmluZyBzdHJ1Y3Qgc3R1ZmYgc3R5bGUgc3ViZGF0ZSBzdWJwYXJ0aXRpb24gc3VicGFydGl0aW9ucyBzdWJzdGl0dXRhYmxlIHN1YnN0ciBzdWJzdHJpbmcgc3VidGltZSBzdWJ0cmluZ19pbmRleCBzdWJ0eXBlIHN1Y2Nlc3Mgc3VtIHN1c3BlbmQgc3dpdGNoIHN3aXRjaG9mZnNldCBzd2l0Y2hvdmVyIHN5bmMgc3luY2hyb25vdXMgc3lub255bSBzeXMgc3lzX3htbGFnZyBzeXNhc20gc3lzYXV4IHN5c2RhdGUgc3lzZGF0ZXRpbWVvZmZzZXQgc3lzZGJhIHN5c29wZXIgc3lzdGVtIHN5c3RlbV91c2VyIHN5c3V0Y2RhdGV0aW1lIHRhYmxlIHRhYmxlcyB0YWJsZXNwYWNlIHRhbiB0ZG8gdGVtcGxhdGUgdGVtcG9yYXJ5IHRlcm1pbmF0ZWQgdGVydGlhcnlfd2VpZ2h0cyB0ZXN0IHRoYW4gdGhlbiB0aHJlYWQgdGhyb3VnaCB0aWVyIHRpZXMgdGltZSB0aW1lX2Zvcm1hdCB0aW1lX3pvbmUgdGltZWRpZmYgdGltZWZyb21wYXJ0cyB0aW1lb3V0IHRpbWVzdGFtcCB0aW1lc3RhbXBhZGQgdGltZXN0YW1wZGlmZiB0aW1lem9uZV9hYmJyIHRpbWV6b25lX21pbnV0ZSB0aW1lem9uZV9yZWdpb24gdG8gdG9fYmFzZTY0IHRvX2RhdGUgdG9fZGF5cyB0b19zZWNvbmRzIHRvZGF0ZXRpbWVvZmZzZXQgdHJhY2UgdHJhY2tpbmcgdHJhbnNhY3Rpb24gdHJhbnNhY3Rpb25hbCB0cmFuc2xhdGUgdHJhbnNsYXRpb24gdHJlYXQgdHJpZ2dlciB0cmlnZ2VyX25lc3RsZXZlbCB0cmlnZ2VycyB0cmltIHRydW5jYXRlIHRyeV9jYXN0IHRyeV9jb252ZXJ0IHRyeV9wYXJzZSB0eXBlIHViMSB1YjIgdWI0IHVjYXNlIHVuYXJjaGl2ZWQgdW5ib3VuZGVkIHVuY29tcHJlc3MgdW5kZXIgdW5kbyB1bmhleCB1bmljb2RlIHVuaWZvcm0gdW5pbnN0YWxsIHVuaW9uIHVuaXF1ZSB1bml4X3RpbWVzdGFtcCB1bmtub3duIHVubGltaXRlZCB1bmxvY2sgdW5waXZvdCB1bnJlY292ZXJhYmxlIHVuc2FmZSB1bnNpZ25lZCB1bnRpbCB1bnRydXN0ZWQgdW51c2FibGUgdW51c2VkIHVwZGF0ZSB1cGRhdGVkIHVwZ3JhZGUgdXBwZWQgdXBwZXIgdXBzZXJ0IHVybCB1cm93aWQgdXNhYmxlIHVzYWdlIHVzZSB1c2Vfc3RvcmVkX291dGxpbmVzIHVzZXIgdXNlcl9kYXRhIHVzZXJfcmVzb3VyY2VzIHVzZXJzIHVzaW5nIHV0Y19kYXRlIHV0Y190aW1lc3RhbXAgdXVpZCB1dWlkX3Nob3J0IHZhbGlkYXRlIHZhbGlkYXRlX3Bhc3N3b3JkX3N0cmVuZ3RoIHZhbGlkYXRpb24gdmFsaXN0IHZhbHVlIHZhbHVlcyB2YXIgdmFyX3NhbXAgdmFyY2hhcmMgdmFyaSB2YXJpYSB2YXJpYWIgdmFyaWFibCB2YXJpYWJsZSB2YXJpYWJsZXMgdmFyaWFuY2UgdmFycCB2YXJyYXcgdmFycmF3YyB2YXJyYXkgdmVyaWZ5IHZlcnNpb24gdmVyc2lvbnMgdmlldyB2aXJ0dWFsIHZpc2libGUgdm9pZCB3YWl0IHdhbGxldCB3YXJuaW5nIHdhcm5pbmdzIHdlZWsgd2Vla2RheSB3ZWVrb2Z5ZWFyIHdlbGxmb3JtZWQgd2hlbiB3aGVuZSB3aGVuZXYgd2hlbmV2ZSB3aGVuZXZlciB3aGVyZSB3aGlsZSB3aGl0ZXNwYWNlIHdpdGggd2l0aGluIHdpdGhvdXQgd29yayB3cmFwcGVkIHhkYiB4bWwgeG1sYWdnIHhtbGF0dHJpYnV0ZXMgeG1sY2FzdCB4bWxjb2xhdHR2YWwgeG1sZWxlbWVudCB4bWxleGlzdHMgeG1sZm9yZXN0IHhtbGluZGV4IHhtbG5hbWVzcGFjZXMgeG1scGkgeG1scXVlcnkgeG1scm9vdCB4bWxzY2hlbWEgeG1sc2VyaWFsaXplIHhtbHRhYmxlIHhtbHR5cGUgeG9yIHllYXIgeWVhcl90b19tb250aCB5ZWFycyB5ZWFyd2VlayIsbGl0ZXJhbDoidHJ1ZSBmYWxzZSBudWxsIixidWlsdF9pbjoiYXJyYXkgYmlnaW50IGJpbmFyeSBiaXQgYmxvYiBib29sZWFuIGNoYXIgY2hhcmFjdGVyIGRhdGUgZGVjIGRlY2ltYWwgZmxvYXQgaW50IGludDggaW50ZWdlciBpbnRlcnZhbCBudW1iZXIgbnVtZXJpYyByZWFsIHJlY29yZCBzZXJpYWwgc2VyaWFsOCBzbWFsbGludCB0ZXh0IHZhcmNoYXIgdmFyeWluZyB2b2lkIn0sYzpbe2NOOiJzdHJpbmciLGI6IiciLGU6IiciLGM6W2UuQkUse2I6IicnIn1dfSx7Y046InN0cmluZyIsYjonIicsZTonIicsYzpbZS5CRSx7YjonIiInfV19LHtjTjoic3RyaW5nIixiOiJgIixlOiJgIixjOltlLkJFXX0sZS5DTk0sZS5DQkNNLHRdfSxlLkNCQ00sdF19fSk7aGxqcy5yZWdpc3Rlckxhbmd1YWdlKCJyIixmdW5jdGlvbihlKXt2YXIgcj0iKFthLXpBLVpdfFxcLlthLXpBLVouXSlbYS16QS1aMC05Ll9dKiI7cmV0dXJue2M6W2UuSENNLHtiOnIsbDpyLGs6e2tleXdvcmQ6ImZ1bmN0aW9uIGlmIGluIGJyZWFrIG5leHQgcmVwZWF0IGVsc2UgZm9yIHJldHVybiBzd2l0Y2ggd2hpbGUgdHJ5IHRyeUNhdGNoIHN0b3Agd2FybmluZyByZXF1aXJlIGxpYnJhcnkgYXR0YWNoIGRldGFjaCBzb3VyY2Ugc2V0TWV0aG9kIHNldEdlbmVyaWMgc2V0R3JvdXBHZW5lcmljIHNldENsYXNzIC4uLiIsbGl0ZXJhbDoiTlVMTCBOQSBUUlVFIEZBTFNFIFQgRiBJbmYgTmFOIE5BX2ludGVnZXJffDEwIE5BX3JlYWxffDEwIE5BX2NoYXJhY3Rlcl98MTAgTkFfY29tcGxleF98MTAifSxyOjB9LHtjTjoibnVtYmVyIixiOiIwW3hYXVswLTlhLWZBLUZdK1tMaV0/XFxiIixyOjB9LHtjTjoibnVtYmVyIixiOiJcXGQrKD86W2VFXVsrXFwtXT9cXGQqKT9MXFxiIixyOjB9LHtjTjoibnVtYmVyIixiOiJcXGQrXFwuKD8hXFxkKSg/OmlcXGIpPyIscjowfSx7Y046Im51bWJlciIsYjoiXFxkKyg/OlxcLlxcZCopPyg/OltlRV1bK1xcLV0/XFxkKik/aT9cXGIiLHI6MH0se2NOOiJudW1iZXIiLGI6IlxcLlxcZCsoPzpbZUVdWytcXC1dP1xcZCopP2k/XFxiIixyOjB9LHtiOiJgIixlOiJgIixyOjB9LHtjTjoic3RyaW5nIixjOltlLkJFXSx2Olt7YjonIicsZTonIid9LHtiOiInIixlOiInIn1dfV19fSk7aGxqcy5yZWdpc3Rlckxhbmd1YWdlKCJwZXJsIixmdW5jdGlvbihlKXt2YXIgdD0iZ2V0cHdlbnQgZ2V0c2VydmVudCBxdW90ZW1ldGEgbXNncmN2IHNjYWxhciBraWxsIGRibWNsb3NlIHVuZGVmIGxjIG1hIHN5c3dyaXRlIHRyIHNlbmQgdW1hc2sgc3lzb3BlbiBzaG13cml0ZSB2ZWMgcXggdXRpbWUgbG9jYWwgb2N0IHNlbWN0bCBsb2NhbHRpbWUgcmVhZHBpcGUgZG8gcmV0dXJuIGZvcm1hdCByZWFkIHNwcmludGYgZGJtb3BlbiBwb3AgZ2V0cGdycCBub3QgZ2V0cHduYW0gcmV3aW5kZGlyIHFxZmlsZW5vIHF3IGVuZHByb3RvZW50IHdhaXQgc2V0aG9zdGVudCBibGVzcyBzfDAgb3BlbmRpciBjb250aW51ZSBlYWNoIHNsZWVwIGVuZGdyZW50IHNodXRkb3duIGR1bXAgY2hvbXAgY29ubmVjdCBnZXRzb2NrbmFtZSBkaWUgc29ja2V0cGFpciBjbG9zZSBmbG9jayBleGlzdHMgaW5kZXggc2htZ2V0c3ViIGZvciBlbmRwd2VudCByZWRvIGxzdGF0IG1zZ2N0bCBzZXRwZ3JwIGFicyBleGl0IHNlbGVjdCBwcmludCByZWYgZ2V0aG9zdGJ5YWRkciB1bnNoaWZ0IGZjbnRsIHN5c2NhbGwgZ290byBnZXRuZXRieWFkZHIgam9pbiBnbXRpbWUgc3ltbGluayBzZW1nZXQgc3BsaWNlIHh8MCBnZXRwZWVybmFtZSByZWN2IGxvZyBzZXRzb2Nrb3B0IGNvcyBsYXN0IHJldmVyc2UgZ2V0aG9zdGJ5bmFtZSBnZXRncm5hbSBzdHVkeSBmb3JtbGluZSBlbmRob3N0ZW50IHRpbWVzIGNob3AgbGVuZ3RoIGdldGhvc3RlbnQgZ2V0bmV0ZW50IHBhY2sgZ2V0cHJvdG9lbnQgZ2V0c2VydmJ5bmFtZSByYW5kIG1rZGlyIHBvcyBjaG1vZCB5fDAgc3Vic3RyIGVuZG5ldGVudCBwcmludGYgbmV4dCBvcGVuIG1zZ3NuZCByZWFkZGlyIHVzZSB1bmxpbmsgZ2V0c29ja29wdCBnZXRwcmlvcml0eSByaW5kZXggd2FudGFycmF5IGhleCBzeXN0ZW0gZ2V0c2VydmJ5cG9ydCBlbmRzZXJ2ZW50IGludCBjaHIgdW50aWUgcm1kaXIgcHJvdG90eXBlIHRlbGwgbGlzdGVuIGZvcmsgc2htcmVhZCB1Y2ZpcnN0IHNldHByb3RvZW50IGVsc2Ugc3lzc2VlayBsaW5rIGdldGdyZ2lkIHNobWN0bCB3YWl0cGlkIHVucGFjayBnZXRuZXRieW5hbWUgcmVzZXQgY2hkaXIgZ3JlcCBzcGxpdCByZXF1aXJlIGNhbGxlciBsY2ZpcnN0IHVudGlsIHdhcm4gd2hpbGUgdmFsdWVzIHNoaWZ0IHRlbGxkaXIgZ2V0cHd1aWQgbXkgZ2V0cHJvdG9ieW51bWJlciBkZWxldGUgYW5kIHNvcnQgdWMgZGVmaW5lZCBzcmFuZCBhY2NlcHQgcGFja2FnZSBzZWVrZGlyIGdldHByb3RvYnluYW1lIHNlbW9wIG91ciByZW5hbWUgc2VlayBpZiBxfDAgY2hyb290IHN5c3JlYWQgc2V0cHdlbnQgbm8gY3J5cHQgZ2V0YyBjaG93biBzcXJ0IHdyaXRlIHNldG5ldGVudCBzZXRwcmlvcml0eSBmb3JlYWNoIHRpZSBzaW4gbXNnZ2V0IG1hcCBzdGF0IGdldGxvZ2luIHVubGVzcyBlbHNpZiB0cnVuY2F0ZSBleGVjIGtleXMgZ2xvYiB0aWVkIGNsb3NlZGlyaW9jdGwgc29ja2V0IHJlYWRsaW5rIGV2YWwgeG9yIHJlYWRsaW5lIGJpbm1vZGUgc2V0c2VydmVudCBlb2Ygb3JkIGJpbmQgYWxhcm0gcGlwZSBhdGFuMiBnZXRncmVudCBleHAgdGltZSBwdXNoIHNldGdyZW50IGd0IGx0IG9yIG5lIG18MCBicmVhayBnaXZlbiBzYXkgc3RhdGUgd2hlbiIscj17Y046InN1YnN0IixiOiJbJEBdXFx7IixlOiJcXH0iLGs6dH0scz17YjoiLT57IixlOiJ9In0sbj17djpbe2I6L1wkXGQvfSx7YjovW1wkJUBdKFxeXHdcYnwjXHcrKDo6XHcrKSp8e1x3K318XHcrKDo6XHcqKSopL30se2I6L1tcJCVAXVteXHNcd3tdLyxyOjB9XX0saT1bZS5CRSxyLG5dLG89W24sZS5IQ00sZS5DKCJeXFw9XFx3IiwiXFw9Y3V0Iix7ZVc6ITB9KSxzLHtjTjoic3RyaW5nIixjOmksdjpbe2I6InFbcXd4cl0/XFxzKlxcKCIsZToiXFwpIixyOjV9LHtiOiJxW3F3eHJdP1xccypcXFsiLGU6IlxcXSIscjo1fSx7YjoicVtxd3hyXT9cXHMqXFx7IixlOiJcXH0iLHI6NX0se2I6InFbcXd4cl0/XFxzKlxcfCIsZToiXFx8IixyOjV9LHtiOiJxW3F3eHJdP1xccypcXDwiLGU6IlxcPiIscjo1fSx7YjoicXdcXHMrcSIsZToicSIscjo1fSx7YjoiJyIsZToiJyIsYzpbZS5CRV19LHtiOiciJyxlOiciJ30se2I6ImAiLGU6ImAiLGM6W2UuQkVdfSx7Yjoie1xcdyt9IixjOltdLHI6MH0se2I6Ii0/XFx3K1xccypcXD1cXD4iLGM6W10scjowfV19LHtjTjoibnVtYmVyIixiOiIoXFxiMFswLTdfXSspfChcXGIweFswLTlhLWZBLUZfXSspfChcXGJbMS05XVswLTlfXSooXFwuWzAtOV9dKyk/KXxbMF9dXFxiIixyOjB9LHtiOiIoXFwvXFwvfCIrZS5SU1IrInxcXGIoc3BsaXR8cmV0dXJufHByaW50fHJldmVyc2V8Z3JlcClcXGIpXFxzKiIsazoic3BsaXQgcmV0dXJuIHByaW50IHJldmVyc2UgZ3JlcCIscjowLGM6W2UuSENNLHtjTjoicmVnZXhwIixiOiIoc3x0cnx5KS8oXFxcXC58W14vXSkqLyhcXFxcLnxbXi9dKSovW2Etel0qIixyOjEwfSx7Y046InJlZ2V4cCIsYjoiKG18cXIpPy8iLGU6Ii9bYS16XSoiLGM6W2UuQkVdLHI6MH1dfSx7Y046ImZ1bmN0aW9uIixiSzoic3ViIixlOiIoXFxzKlxcKC4qP1xcKSk/Wzt7XSIsZUU6ITAscjo1LGM6W2UuVE1dfSx7YjoiLVxcd1xcYiIscjowfSx7YjoiXl9fREFUQV9fJCIsZToiXl9fRU5EX18kIixzTDoibW9qb2xpY2lvdXMiLGM6W3tiOiJeQEAuKiIsZToiJCIsY046ImNvbW1lbnQifV19XTtyZXR1cm4gci5jPW8scy5jPW8se2FsaWFzZXM6WyJwbCIsInBtIl0sbDovW1x3XC5dKy8sazp0LGM6b319KTtobGpzLnJlZ2lzdGVyTGFuZ3VhZ2UoImluaSIsZnVuY3Rpb24oZSl7dmFyIGI9e2NOOiJzdHJpbmciLGM6W2UuQkVdLHY6W3tiOiInJyciLGU6IicnJyIscjoxMH0se2I6JyIiIicsZTonIiIiJyxyOjEwfSx7YjonIicsZTonIid9LHtiOiInIixlOiInIn1dfTtyZXR1cm57YWxpYXNlczpbInRvbWwiXSxjSTohMCxpOi9cUy8sYzpbZS5DKCI7IiwiJCIpLGUuSENNLHtjTjoic2VjdGlvbiIsYjovXlxzKlxbKy8sZTovXF0rL30se2I6L15bYS16MC05XFtcXV8tXStccyo9XHMqLyxlOiIkIixyQjohMCxjOlt7Y046ImF0dHIiLGI6L1thLXowLTlcW1xdXy1dKy99LHtiOi89LyxlVzohMCxyOjAsYzpbe2NOOiJsaXRlcmFsIixiOi9cYm9ufG9mZnx0cnVlfGZhbHNlfHllc3xub1xiL30se2NOOiJ2YXJpYWJsZSIsdjpbe2I6L1wkW1x3XGQiXVtcd1xkX10qL30se2I6L1wkXHsoLio/KX0vfV19LGIse2NOOiJudW1iZXIiLGI6LyhbXCtcLV0rKT9bXGRdK19bXGRfXSsvfSxlLk5NXX1dfV19fSk7aGxqcy5yZWdpc3Rlckxhbmd1YWdlKCJkaWZmIixmdW5jdGlvbihlKXtyZXR1cm57YWxpYXNlczpbInBhdGNoIl0sYzpbe2NOOiJtZXRhIixyOjEwLHY6W3tiOi9eQEAgK1wtXGQrLFxkKyArXCtcZCssXGQrICtAQCQvfSx7YjovXlwqXCpcKiArXGQrLFxkKyArXCpcKlwqXCokL30se2I6L15cLVwtXC0gK1xkKyxcZCsgK1wtXC1cLVwtJC99XX0se2NOOiJjb21tZW50Iix2Olt7YjovSW5kZXg6IC8sZTovJC99LHtiOi89ezMsfS8sZTovJC99LHtiOi9eXC17M30vLGU6LyQvfSx7YjovXlwqezN9IC8sZTovJC99LHtiOi9eXCt7M30vLGU6LyQvfSx7YjovXCp7NX0vLGU6L1wqezV9JC99XX0se2NOOiJhZGRpdGlvbiIsYjoiXlxcKyIsZToiJCJ9LHtjTjoiZGVsZXRpb24iLGI6Il5cXC0iLGU6IiQifSx7Y046ImFkZGl0aW9uIixiOiJeXFwhIixlOiIkIn1dfX0pO2hsanMucmVnaXN0ZXJMYW5ndWFnZSgiZ28iLGZ1bmN0aW9uKGUpe3ZhciB0PXtrZXl3b3JkOiJicmVhayBkZWZhdWx0IGZ1bmMgaW50ZXJmYWNlIHNlbGVjdCBjYXNlIG1hcCBzdHJ1Y3QgY2hhbiBlbHNlIGdvdG8gcGFja2FnZSBzd2l0Y2ggY29uc3QgZmFsbHRocm91Z2ggaWYgcmFuZ2UgdHlwZSBjb250aW51ZSBmb3IgaW1wb3J0IHJldHVybiB2YXIgZ28gZGVmZXIgYm9vbCBieXRlIGNvbXBsZXg2NCBjb21wbGV4MTI4IGZsb2F0MzIgZmxvYXQ2NCBpbnQ4IGludDE2IGludDMyIGludDY0IHN0cmluZyB1aW50OCB1aW50MTYgdWludDMyIHVpbnQ2NCBpbnQgdWludCB1aW50cHRyIHJ1bmUiLGxpdGVyYWw6InRydWUgZmFsc2UgaW90YSBuaWwiLGJ1aWx0X2luOiJhcHBlbmQgY2FwIGNsb3NlIGNvbXBsZXggY29weSBpbWFnIGxlbiBtYWtlIG5ldyBwYW5pYyBwcmludCBwcmludGxuIHJlYWwgcmVjb3ZlciBkZWxldGUifTtyZXR1cm57YWxpYXNlczpbImdvbGFuZyJdLGs6dCxpOiI8LyIsYzpbZS5DTENNLGUuQ0JDTSx7Y046InN0cmluZyIsdjpbZS5RU00se2I6IiciLGU6IlteXFxcXF0nIn0se2I6ImAiLGU6ImAifV19LHtjTjoibnVtYmVyIix2Olt7YjplLkNOUisiW2RmbHNpXSIscjoxfSxlLkNOTV19LHtiOi86PS99LHtjTjoiZnVuY3Rpb24iLGJLOiJmdW5jIixlOi9ccypcey8sZUU6ITAsYzpbZS5UTSx7Y046InBhcmFtcyIsYjovXCgvLGU6L1wpLyxrOnQsaTovWyInXS99XX1dfX0pO2hsanMucmVnaXN0ZXJMYW5ndWFnZSgiYmFzaCIsZnVuY3Rpb24oZSl7dmFyIHQ9e2NOOiJ2YXJpYWJsZSIsdjpbe2I6L1wkW1x3XGQjQF1bXHdcZF9dKi99LHtiOi9cJFx7KC4qPyl9L31dfSxzPXtjTjoic3RyaW5nIixiOi8iLyxlOi8iLyxjOltlLkJFLHQse2NOOiJ2YXJpYWJsZSIsYjovXCRcKC8sZTovXCkvLGM6W2UuQkVdfV19LGE9e2NOOiJzdHJpbmciLGI6LycvLGU6LycvfTtyZXR1cm57YWxpYXNlczpbInNoIiwienNoIl0sbDovXGItP1thLXpcLl9dK1xiLyxrOntrZXl3b3JkOiJpZiB0aGVuIGVsc2UgZWxpZiBmaSBmb3Igd2hpbGUgaW4gZG8gZG9uZSBjYXNlIGVzYWMgZnVuY3Rpb24iLGxpdGVyYWw6InRydWUgZmFsc2UiLGJ1aWx0X2luOiJicmVhayBjZCBjb250aW51ZSBldmFsIGV4ZWMgZXhpdCBleHBvcnQgZ2V0b3B0cyBoYXNoIHB3ZCByZWFkb25seSByZXR1cm4gc2hpZnQgdGVzdCB0aW1lcyB0cmFwIHVtYXNrIHVuc2V0IGFsaWFzIGJpbmQgYnVpbHRpbiBjYWxsZXIgY29tbWFuZCBkZWNsYXJlIGVjaG8gZW5hYmxlIGhlbHAgbGV0IGxvY2FsIGxvZ291dCBtYXBmaWxlIHByaW50ZiByZWFkIHJlYWRhcnJheSBzb3VyY2UgdHlwZSB0eXBlc2V0IHVsaW1pdCB1bmFsaWFzIHNldCBzaG9wdCBhdXRvbG9hZCBiZyBiaW5ka2V5IGJ5ZSBjYXAgY2hkaXIgY2xvbmUgY29tcGFyZ3VtZW50cyBjb21wY2FsbCBjb21wY3RsIGNvbXBkZXNjcmliZSBjb21wZmlsZXMgY29tcGdyb3VwcyBjb21wcXVvdGUgY29tcHRhZ3MgY29tcHRyeSBjb21wdmFsdWVzIGRpcnMgZGlzYWJsZSBkaXNvd24gZWNob3RjIGVjaG90aSBlbXVsYXRlIGZjIGZnIGZsb2F0IGZ1bmN0aW9ucyBnZXRjYXAgZ2V0bG4gaGlzdG9yeSBpbnRlZ2VyIGpvYnMga2lsbCBsaW1pdCBsb2cgbm9nbG9iIHBvcGQgcHJpbnQgcHVzaGQgcHVzaGxuIHJlaGFzaCBzY2hlZCBzZXRjYXAgc2V0b3B0IHN0YXQgc3VzcGVuZCB0dHljdGwgdW5mdW5jdGlvbiB1bmhhc2ggdW5saW1pdCB1bnNldG9wdCB2YXJlZCB3YWl0IHdoZW5jZSB3aGVyZSB3aGljaCB6Y29tcGlsZSB6Zm9ybWF0IHpmdHAgemxlIHptb2Rsb2FkIHpwYXJzZW9wdHMgenByb2YgenB0eSB6cmVnZXhwYXJzZSB6c29ja2V0IHpzdHlsZSB6dGNwIixfOiItbmUgLWVxIC1sdCAtZ3QgLWYgLWQgLWUgLXMgLWwgLWEifSxjOlt7Y046Im1ldGEiLGI6L14jIVteXG5dK3NoXHMqJC8scjoxMH0se2NOOiJmdW5jdGlvbiIsYjovXHdbXHdcZF9dKlxzKlwoXHMqXClccypcey8sckI6ITAsYzpbZS5pbmhlcml0KGUuVE0se2I6L1x3W1x3XGRfXSovfSldLHI6MH0sZS5IQ00scyxhLHRdfX0pO2hsanMucmVnaXN0ZXJMYW5ndWFnZSgicHl0aG9uIixmdW5jdGlvbihlKXt2YXIgcj17a2V5d29yZDoiYW5kIGVsaWYgaXMgZ2xvYmFsIGFzIGluIGlmIGZyb20gcmFpc2UgZm9yIGV4Y2VwdCBmaW5hbGx5IHByaW50IGltcG9ydCBwYXNzIHJldHVybiBleGVjIGVsc2UgYnJlYWsgbm90IHdpdGggY2xhc3MgYXNzZXJ0IHlpZWxkIHRyeSB3aGlsZSBjb250aW51ZSBkZWwgb3IgZGVmIGxhbWJkYSBhc3luYyBhd2FpdCBub25sb2NhbHwxMCBOb25lIFRydWUgRmFsc2UiLGJ1aWx0X2luOiJFbGxpcHNpcyBOb3RJbXBsZW1lbnRlZCJ9LGI9e2NOOiJtZXRhIixiOi9eKD4+PnxcLlwuXC4pIC99LGM9e2NOOiJzdWJzdCIsYjovXHsvLGU6L1x9LyxrOnIsaTovIy99LGE9e2NOOiJzdHJpbmciLGM6W2UuQkVdLHY6W3tiOi8odXxiKT9yPycnJy8sZTovJycnLyxjOltiXSxyOjEwfSx7YjovKHV8Yik/cj8iIiIvLGU6LyIiIi8sYzpbYl0scjoxMH0se2I6LyhmcnxyZnxmKScnJy8sZTovJycnLyxjOltiLGNdfSx7YjovKGZyfHJmfGYpIiIiLyxlOi8iIiIvLGM6W2IsY119LHtiOi8odXxyfHVyKScvLGU6LycvLHI6MTB9LHtiOi8odXxyfHVyKSIvLGU6LyIvLHI6MTB9LHtiOi8oYnxiciknLyxlOi8nL30se2I6LyhifGJyKSIvLGU6LyIvfSx7YjovKGZyfHJmfGYpJy8sZTovJy8sYzpbY119LHtiOi8oZnJ8cmZ8ZikiLyxlOi8iLyxjOltjXX0sZS5BU00sZS5RU01dfSxzPXtjTjoibnVtYmVyIixyOjAsdjpbe2I6ZS5CTlIrIltsTGpKXT8ifSx7YjoiXFxiKDBvWzAtN10rKVtsTGpKXT8ifSx7YjplLkNOUisiW2xMakpdPyJ9XX0saT17Y046InBhcmFtcyIsYjovXCgvLGU6L1wpLyxjOlsic2VsZiIsYixzLGFdfTtyZXR1cm4gYy5jPVthLHMsYl0se2FsaWFzZXM6WyJweSIsImd5cCJdLGs6cixpOi8oPFwvfC0+fFw/KXw9Pi8sYzpbYixzLGEsZS5IQ00se3Y6W3tjTjoiZnVuY3Rpb24iLGJLOiJkZWYifSx7Y046ImNsYXNzIixiSzoiY2xhc3MifV0sZTovOi8saTovWyR7PTtcbixdLyxjOltlLlVUTSxpLHtiOi8tPi8sZVc6ITAsazoiTm9uZSJ9XX0se2NOOiJtZXRhIixiOi9eW1x0IF0qQC8sZTovJC99LHtiOi9cYihwcmludHxleGVjKVwoL31dfX0pO2hsanMucmVnaXN0ZXJMYW5ndWFnZSgianVsaWEiLGZ1bmN0aW9uKGUpe3ZhciByPXtrZXl3b3JkOiJpbiBpc2Egd2hlcmUgYmFyZW1vZHVsZSBiZWdpbiBicmVhayBjYXRjaCBjY2FsbCBjb25zdCBjb250aW51ZSBkbyBlbHNlIGVsc2VpZiBlbmQgZXhwb3J0IGZhbHNlIGZpbmFsbHkgZm9yIGZ1bmN0aW9uIGdsb2JhbCBpZiBpbXBvcnQgaW1wb3J0YWxsIGxldCBsb2NhbCBtYWNybyBtb2R1bGUgcXVvdGUgcmV0dXJuIHRydWUgdHJ5IHVzaW5nIHdoaWxlIHR5cGUgaW1tdXRhYmxlIGFic3RyYWN0IGJpdHN0eXBlIHR5cGVhbGlhcyAiLGxpdGVyYWw6InRydWUgZmFsc2UgQVJHUyBDX05VTEwgRGV2TnVsbCBFTkRJQU5fQk9NIEVOViBJIEluZiBJbmYxNiBJbmYzMiBJbmY2NCBJbnNlcnRpb25Tb3J0IEpVTElBX0hPTUUgTE9BRF9QQVRIIE1lcmdlU29ydCBOYU4gTmFOMTYgTmFOMzIgTmFONjQgUFJPR1JBTV9GSUxFIFF1aWNrU29ydCBSb3VuZERvd24gUm91bmRGcm9tWmVybyBSb3VuZE5lYXJlc3QgUm91bmROZWFyZXN0VGllc0F3YXkgUm91bmROZWFyZXN0VGllc1VwIFJvdW5kVG9aZXJvIFJvdW5kVXAgU1RERVJSIFNURElOIFNURE9VVCBWRVJTSU9OIGNhdGFsYW4gZXwwIGV1fDAgZXVsZXJnYW1tYSBnb2xkZW4gaW0gbm90aGluZyBwaSDOsyDPgCDPhiAiLGJ1aWx0X2luOiJBTlkgQWJzdHJhY3RBcnJheSBBYnN0cmFjdENoYW5uZWwgQWJzdHJhY3RGbG9hdCBBYnN0cmFjdE1hdHJpeCBBYnN0cmFjdFJORyBBYnN0cmFjdFNlcmlhbGl6ZXIgQWJzdHJhY3RTZXQgQWJzdHJhY3RTcGFyc2VBcnJheSBBYnN0cmFjdFNwYXJzZU1hdHJpeCBBYnN0cmFjdFNwYXJzZVZlY3RvciBBYnN0cmFjdFN0cmluZyBBYnN0cmFjdFVuaXRSYW5nZSBBYnN0cmFjdFZlY09yTWF0IEFic3RyYWN0VmVjdG9yIEFueSBBcmd1bWVudEVycm9yIEFycmF5IEFzc2VydGlvbkVycm9yIEFzc29jaWF0aXZlIEJhc2U2NERlY29kZVBpcGUgQmFzZTY0RW5jb2RlUGlwZSBCaWRpYWdvbmFsIEJpZ0Zsb2F0IEJpZ0ludCBCaXRBcnJheSBCaXRNYXRyaXggQml0VmVjdG9yIEJvb2wgQm91bmRzRXJyb3IgQnVmZmVyU3RyZWFtIENhY2hpbmdQb29sIENhcHR1cmVkRXhjZXB0aW9uIENhcnRlc2lhbkluZGV4IENhcnRlc2lhblJhbmdlIENjaGFyIENkb3VibGUgQ2Zsb2F0IENoYW5uZWwgQ2hhciBDaW50IENpbnRtYXhfdCBDbG9uZyBDbG9uZ2xvbmcgQ2x1c3Rlck1hbmFnZXIgQ21kIENvZGVJbmZvIENvbG9uIENvbXBsZXggQ29tcGxleDEyOCBDb21wbGV4MzIgQ29tcGxleDY0IENvbXBvc2l0ZUV4Y2VwdGlvbiBDb25kaXRpb24gQ29uakFycmF5IENvbmpNYXRyaXggQ29ualZlY3RvciBDcHRyZGlmZl90IENzaG9ydCBDc2l6ZV90IENzc2l6ZV90IENzdHJpbmcgQ3VjaGFyIEN1aW50IEN1aW50bWF4X3QgQ3Vsb25nIEN1bG9uZ2xvbmcgQ3VzaG9ydCBDd2NoYXJfdCBDd3N0cmluZyBEYXRhVHlwZSBEYXRlIERhdGVGb3JtYXQgRGF0ZVRpbWUgRGVuc2VBcnJheSBEZW5zZU1hdHJpeCBEZW5zZVZlY09yTWF0IERlbnNlVmVjdG9yIERpYWdvbmFsIERpY3QgRGltZW5zaW9uTWlzbWF0Y2ggRGltcyBEaXJlY3RJbmRleFN0cmluZyBEaXNwbGF5IERpdmlkZUVycm9yIERvbWFpbkVycm9yIEVPRkVycm9yIEVhY2hMaW5lIEVudW0gRW51bWVyYXRlIEVycm9yRXhjZXB0aW9uIEV4Y2VwdGlvbiBFeHBvbmVudGlhbEJhY2tPZmYgRXhwciBGYWN0b3JpemF0aW9uIEZpbGVNb25pdG9yIEZsb2F0MTYgRmxvYXQzMiBGbG9hdDY0IEZ1bmN0aW9uIEZ1dHVyZSBHbG9iYWxSZWYgR290b05vZGUgSFRNTCBIZXJtaXRpYW4gSU8gSU9CdWZmZXIgSU9Db250ZXh0IElPU3RyZWFtIElQQWRkciBJUHY0IElQdjYgSW5kZXhDYXJ0ZXNpYW4gSW5kZXhMaW5lYXIgSW5kZXhTdHlsZSBJbmV4YWN0RXJyb3IgSW5pdEVycm9yIEludCBJbnQxMjggSW50MTYgSW50MzIgSW50NjQgSW50OCBJbnRTZXQgSW50ZWdlciBJbnRlcnJ1cHRFeGNlcHRpb24gSW52YWxpZFN0YXRlRXhjZXB0aW9uIElycmF0aW9uYWwgS2V5RXJyb3IgTGFiZWxOb2RlIExpblNwYWNlIExpbmVOdW1iZXJOb2RlIExvYWRFcnJvciBMb3dlclRyaWFuZ3VsYXIgTUlNRSBNYXRyaXggTWVyc2VubmVUd2lzdGVyIE1ldGhvZCBNZXRob2RFcnJvciBNZXRob2RUYWJsZSBNb2R1bGUgTlR1cGxlIE5ld3Zhck5vZGUgTnVsbEV4Y2VwdGlvbiBOdWxsYWJsZSBOdW1iZXIgT2JqZWN0SWREaWN0IE9yZGluYWxSYW5nZSBPdXRPZk1lbW9yeUVycm9yIE92ZXJmbG93RXJyb3IgUGFpciBQYXJzZUVycm9yIFBhcnRpYWxRdWlja1NvcnQgUGVybXV0ZWREaW1zQXJyYXkgUGlwZSBQb2xsaW5nRmlsZVdhdGNoZXIgUHJvY2Vzc0V4aXRlZEV4Y2VwdGlvbiBQdHIgUXVvdGVOb2RlIFJhbmRvbURldmljZSBSYW5nZSBSYW5nZUluZGV4IFJhdGlvbmFsIFJhd0ZEIFJlYWRPbmx5TWVtb3J5RXJyb3IgUmVhbCBSZWVudHJhbnRMb2NrIFJlZiBSZWdleCBSZWdleE1hdGNoIFJlbW90ZUNoYW5uZWwgUmVtb3RlRXhjZXB0aW9uIFJldlN0cmluZyBSb3VuZGluZ01vZGUgUm93VmVjdG9yIFNTQVZhbHVlIFNlZ21lbnRhdGlvbkZhdWx0IFNlcmlhbGl6YXRpb25TdGF0ZSBTZXQgU2hhcmVkQXJyYXkgU2hhcmVkTWF0cml4IFNoYXJlZFZlY3RvciBTaWduZWQgU2ltcGxlVmVjdG9yIFNsb3QgU2xvdE51bWJlciBTcGFyc2VNYXRyaXhDU0MgU3BhcnNlVmVjdG9yIFN0YWNrRnJhbWUgU3RhY2tPdmVyZmxvd0Vycm9yIFN0YWNrVHJhY2UgU3RlcFJhbmdlIFN0ZXBSYW5nZUxlbiBTdHJpZGVkQXJyYXkgU3RyaWRlZE1hdHJpeCBTdHJpZGVkVmVjT3JNYXQgU3RyaWRlZFZlY3RvciBTdHJpbmcgU3ViQXJyYXkgU3ViU3RyaW5nIFN5bVRyaWRpYWdvbmFsIFN5bWJvbCBTeW1tZXRyaWMgU3lzdGVtRXJyb3IgVENQU29ja2V0IFRhc2sgVGV4dCBUZXh0RGlzcGxheSBUaW1lciBUcmlkaWFnb25hbCBUdXBsZSBUeXBlIFR5cGVFcnJvciBUeXBlTWFwRW50cnkgVHlwZU1hcExldmVsIFR5cGVOYW1lIFR5cGVWYXIgVHlwZWRTbG90IFVEUFNvY2tldCBVSW50IFVJbnQxMjggVUludDE2IFVJbnQzMiBVSW50NjQgVUludDggVW5kZWZSZWZFcnJvciBVbmRlZlZhckVycm9yIFVuaWNvZGVFcnJvciBVbmlmb3JtU2NhbGluZyBVbmlvbiBVbmlvbkFsbCBVbml0UmFuZ2UgVW5zaWduZWQgVXBwZXJUcmlhbmd1bGFyIFZhbCBWYXJhcmcgVmVjRWxlbWVudCBWZWNPck1hdCBWZWN0b3IgVmVyc2lvbk51bWJlciBWb2lkIFdlYWtLZXlEaWN0IFdlYWtSZWYgV29ya2VyQ29uZmlnIFdvcmtlclBvb2wgIn0sdD0iW0EtWmEtel9cXHUwMEExLVxcdUZGRkZdW0EtWmEtel8wLTlcXHUwMEExLVxcdUZGRkZdKiIsYT17bDp0LGs6cixpOi88XC8vfSxuPXtjTjoibnVtYmVyIixiOi8oXGIweFtcZF9dKihcLltcZF9dKik/fDB4XC5cZFtcZF9dKilwWy0rXT9cZCt8XGIwW2JveF1bYS1mQS1GMC05XVthLWZBLUYwLTlfXSp8KFxiXGRbXGRfXSooXC5bXGRfXSopP3xcLlxkW1xkX10qKShbZUVmRl1bLStdP1xkKyk/LyxyOjB9LG89e2NOOiJzdHJpbmciLGI6LycoLnxcXFt4WHVVXVthLXpBLVowLTldKyknL30saT17Y046InN1YnN0IixiOi9cJFwoLyxlOi9cKS8sazpyfSxsPXtjTjoidmFyaWFibGUiLGI6IlxcJCIrdH0sYz17Y046InN0cmluZyIsYzpbZS5CRSxpLGxdLHY6W3tiOi9cdyoiIiIvLGU6LyIiIlx3Ki8scjoxMH0se2I6L1x3KiIvLGU6LyJcdyovfV19LHM9e2NOOiJzdHJpbmciLGM6W2UuQkUsaSxsXSxiOiJgIixlOiJgIn0sZD17Y046Im1ldGEiLGI6IkAiK3R9LHU9e2NOOiJjb21tZW50Iix2Olt7YjoiIz0iLGU6Ij0jIixyOjEwfSx7YjoiIyIsZToiJCJ9XX07cmV0dXJuIGEuYz1bbixvLGMscyxkLHUsZS5IQ00se2NOOiJrZXl3b3JkIixiOiJcXGIoKChhYnN0cmFjdHxwcmltaXRpdmUpXFxzKyl0eXBlfChtdXRhYmxlXFxzKyk/c3RydWN0KVxcYiJ9LHtiOi88Oi99XSxpLmM9YS5jLGF9KTtobGpzLnJlZ2lzdGVyTGFuZ3VhZ2UoImNvZmZlZXNjcmlwdCIsZnVuY3Rpb24oZSl7dmFyIGM9e2tleXdvcmQ6ImluIGlmIGZvciB3aGlsZSBmaW5hbGx5IG5ldyBkbyByZXR1cm4gZWxzZSBicmVhayBjYXRjaCBpbnN0YW5jZW9mIHRocm93IHRyeSB0aGlzIHN3aXRjaCBjb250aW51ZSB0eXBlb2YgZGVsZXRlIGRlYnVnZ2VyIHN1cGVyIHlpZWxkIGltcG9ydCBleHBvcnQgZnJvbSBhcyBkZWZhdWx0IGF3YWl0IHRoZW4gdW5sZXNzIHVudGlsIGxvb3Agb2YgYnkgd2hlbiBhbmQgb3IgaXMgaXNudCBub3QiLGxpdGVyYWw6InRydWUgZmFsc2UgbnVsbCB1bmRlZmluZWQgeWVzIG5vIG9uIG9mZiIsYnVpbHRfaW46Im5wbSByZXF1aXJlIGNvbnNvbGUgcHJpbnQgbW9kdWxlIGdsb2JhbCB3aW5kb3cgZG9jdW1lbnQifSxuPSJbQS1aYS16JF9dWzAtOUEtWmEteiRfXSoiLHI9e2NOOiJzdWJzdCIsYjovI1x7LyxlOi99LyxrOmN9LGk9W2UuQk5NLGUuaW5oZXJpdChlLkNOTSx7c3RhcnRzOntlOiIoXFxzKi8pPyIscjowfX0pLHtjTjoic3RyaW5nIix2Olt7YjovJycnLyxlOi8nJycvLGM6W2UuQkVdfSx7YjovJy8sZTovJy8sYzpbZS5CRV19LHtiOi8iIiIvLGU6LyIiIi8sYzpbZS5CRSxyXX0se2I6LyIvLGU6LyIvLGM6W2UuQkUscl19XX0se2NOOiJyZWdleHAiLHY6W3tiOiIvLy8iLGU6Ii8vLyIsYzpbcixlLkhDTV19LHtiOiIvL1tnaW1dKiIscjowfSx7YjovXC8oPyFbICpdKShcXFwvfC4pKj9cL1tnaW1dKig/PVxXfCQpL31dfSx7YjoiQCIrbn0se3NMOiJqYXZhc2NyaXB0IixlQjohMCxlRTohMCx2Olt7YjoiYGBgIixlOiJgYGAifSx7YjoiYCIsZToiYCJ9XX1dO3IuYz1pO3ZhciBzPWUuaW5oZXJpdChlLlRNLHtiOm59KSx0PSIoXFwoLipcXCkpP1xccypcXEJbLT1dPiIsbz17Y046InBhcmFtcyIsYjoiXFwoW15cXChdIixyQjohMCxjOlt7YjovXCgvLGU6L1wpLyxrOmMsYzpbInNlbGYiXS5jb25jYXQoaSl9XX07cmV0dXJue2FsaWFzZXM6WyJjb2ZmZWUiLCJjc29uIiwiaWNlZCJdLGs6YyxpOi9cL1wqLyxjOmkuY29uY2F0KFtlLkMoIiMjIyIsIiMjIyIpLGUuSENNLHtjTjoiZnVuY3Rpb24iLGI6Il5cXHMqIituKyJcXHMqPVxccyoiK3QsZToiWy09XT4iLHJCOiEwLGM6W3Msb119LHtiOi9bOlwoLD1dXHMqLyxyOjAsYzpbe2NOOiJmdW5jdGlvbiIsYjp0LGU6IlstPV0+IixyQjohMCxjOltvXX1dfSx7Y046ImNsYXNzIixiSzoiY2xhc3MiLGU6IiQiLGk6L1s6PSJcW1xdXS8sYzpbe2JLOiJleHRlbmRzIixlVzohMCxpOi9bOj0iXFtcXV0vLGM6W3NdfSxzXX0se2I6bisiOiIsZToiOiIsckI6ITAsckU6ITAscjowfV0pfX0pO2hsanMucmVnaXN0ZXJMYW5ndWFnZSgiY3BwIixmdW5jdGlvbih0KXt2YXIgZT17Y046ImtleXdvcmQiLGI6IlxcYlthLXpcXGRfXSpfdFxcYiJ9LHI9e2NOOiJzdHJpbmciLHY6W3tiOicodTg/fFUpP0w/IicsZTonIicsaToiXFxuIixjOlt0LkJFXX0se2I6Jyh1OD98VSk/UiInLGU6JyInLGM6W3QuQkVdfSx7YjoiJ1xcXFw/LiIsZToiJyIsaToiLiJ9XX0scz17Y046Im51bWJlciIsdjpbe2I6IlxcYigwYlswMSddKykifSx7YjoiKC0/KVxcYihbXFxkJ10rKFxcLltcXGQnXSopP3xcXC5bXFxkJ10rKSh1fFV8bHxMfHVsfFVMfGZ8RnxifEIpIn0se2I6IigtPykoXFxiMFt4WF1bYS1mQS1GMC05J10rfChcXGJbXFxkJ10rKFxcLltcXGQnXSopP3xcXC5bXFxkJ10rKShbZUVdWy0rXT9bXFxkJ10rKT8pIn1dLHI6MH0saT17Y046Im1ldGEiLGI6LyNccypbYS16XStcYi8sZTovJC8sazp7Im1ldGEta2V5d29yZCI6ImlmIGVsc2UgZWxpZiBlbmRpZiBkZWZpbmUgdW5kZWYgd2FybmluZyBlcnJvciBsaW5lIHByYWdtYSBpZmRlZiBpZm5kZWYgaW5jbHVkZSJ9LGM6W3tiOi9cXFxuLyxyOjB9LHQuaW5oZXJpdChyLHtjTjoibWV0YS1zdHJpbmcifSkse2NOOiJtZXRhLXN0cmluZyIsYjovPFteXG4+XSo+LyxlOi8kLyxpOiJcXG4ifSx0LkNMQ00sdC5DQkNNXX0sYT10LklSKyJcXHMqXFwoIixjPXtrZXl3b3JkOiJpbnQgZmxvYXQgd2hpbGUgcHJpdmF0ZSBjaGFyIGNhdGNoIGltcG9ydCBtb2R1bGUgZXhwb3J0IHZpcnR1YWwgb3BlcmF0b3Igc2l6ZW9mIGR5bmFtaWNfY2FzdHwxMCB0eXBlZGVmIGNvbnN0X2Nhc3R8MTAgY29uc3QgZm9yIHN0YXRpY19jYXN0fDEwIHVuaW9uIG5hbWVzcGFjZSB1bnNpZ25lZCBsb25nIHZvbGF0aWxlIHN0YXRpYyBwcm90ZWN0ZWQgYm9vbCB0ZW1wbGF0ZSBtdXRhYmxlIGlmIHB1YmxpYyBmcmllbmQgZG8gZ290byBhdXRvIHZvaWQgZW51bSBlbHNlIGJyZWFrIGV4dGVybiB1c2luZyBhc20gY2FzZSB0eXBlaWQgc2hvcnQgcmVpbnRlcnByZXRfY2FzdHwxMCBkZWZhdWx0IGRvdWJsZSByZWdpc3RlciBleHBsaWNpdCBzaWduZWQgdHlwZW5hbWUgdHJ5IHRoaXMgc3dpdGNoIGNvbnRpbnVlIGlubGluZSBkZWxldGUgYWxpZ25vZiBjb25zdGV4cHIgZGVjbHR5cGUgbm9leGNlcHQgc3RhdGljX2Fzc2VydCB0aHJlYWRfbG9jYWwgcmVzdHJpY3QgX0Jvb2wgY29tcGxleCBfQ29tcGxleCBfSW1hZ2luYXJ5IGF0b21pY19ib29sIGF0b21pY19jaGFyIGF0b21pY19zY2hhciBhdG9taWNfdWNoYXIgYXRvbWljX3Nob3J0IGF0b21pY191c2hvcnQgYXRvbWljX2ludCBhdG9taWNfdWludCBhdG9taWNfbG9uZyBhdG9taWNfdWxvbmcgYXRvbWljX2xsb25nIGF0b21pY191bGxvbmcgbmV3IHRocm93IHJldHVybiBhbmQgb3Igbm90IixidWlsdF9pbjoic3RkIHN0cmluZyBjaW4gY291dCBjZXJyIGNsb2cgc3RkaW4gc3Rkb3V0IHN0ZGVyciBzdHJpbmdzdHJlYW0gaXN0cmluZ3N0cmVhbSBvc3RyaW5nc3RyZWFtIGF1dG9fcHRyIGRlcXVlIGxpc3QgcXVldWUgc3RhY2sgdmVjdG9yIG1hcCBzZXQgYml0c2V0IG11bHRpc2V0IG11bHRpbWFwIHVub3JkZXJlZF9zZXQgdW5vcmRlcmVkX21hcCB1bm9yZGVyZWRfbXVsdGlzZXQgdW5vcmRlcmVkX211bHRpbWFwIGFycmF5IHNoYXJlZF9wdHIgYWJvcnQgYWJzIGFjb3MgYXNpbiBhdGFuMiBhdGFuIGNhbGxvYyBjZWlsIGNvc2ggY29zIGV4aXQgZXhwIGZhYnMgZmxvb3IgZm1vZCBmcHJpbnRmIGZwdXRzIGZyZWUgZnJleHAgZnNjYW5mIGlzYWxudW0gaXNhbHBoYSBpc2NudHJsIGlzZGlnaXQgaXNncmFwaCBpc2xvd2VyIGlzcHJpbnQgaXNwdW5jdCBpc3NwYWNlIGlzdXBwZXIgaXN4ZGlnaXQgdG9sb3dlciB0b3VwcGVyIGxhYnMgbGRleHAgbG9nMTAgbG9nIG1hbGxvYyByZWFsbG9jIG1lbWNociBtZW1jbXAgbWVtY3B5IG1lbXNldCBtb2RmIHBvdyBwcmludGYgcHV0Y2hhciBwdXRzIHNjYW5mIHNpbmggc2luIHNucHJpbnRmIHNwcmludGYgc3FydCBzc2NhbmYgc3RyY2F0IHN0cmNociBzdHJjbXAgc3RyY3B5IHN0cmNzcG4gc3RybGVuIHN0cm5jYXQgc3RybmNtcCBzdHJuY3B5IHN0cnBicmsgc3RycmNociBzdHJzcG4gc3Ryc3RyIHRhbmggdGFuIHZmcHJpbnRmIHZwcmludGYgdnNwcmludGYgZW5kbCBpbml0aWFsaXplcl9saXN0IHVuaXF1ZV9wdHIiLGxpdGVyYWw6InRydWUgZmFsc2UgbnVsbHB0ciBOVUxMIn0sbj1bZSx0LkNMQ00sdC5DQkNNLHMscl07cmV0dXJue2FsaWFzZXM6WyJjIiwiY2MiLCJoIiwiYysrIiwiaCsrIiwiaHBwIl0sazpjLGk6IjwvIixjOm4uY29uY2F0KFtpLHtiOiJcXGIoZGVxdWV8bGlzdHxxdWV1ZXxzdGFja3x2ZWN0b3J8bWFwfHNldHxiaXRzZXR8bXVsdGlzZXR8bXVsdGltYXB8dW5vcmRlcmVkX21hcHx1bm9yZGVyZWRfc2V0fHVub3JkZXJlZF9tdWx0aXNldHx1bm9yZGVyZWRfbXVsdGltYXB8YXJyYXkpXFxzKjwiLGU6Ij4iLGs6YyxjOlsic2VsZiIsZV19LHtiOnQuSVIrIjo6IixrOmN9LHt2Olt7YjovPS8sZTovOy99LHtiOi9cKC8sZTovXCkvfSx7Yks6Im5ldyB0aHJvdyByZXR1cm4gZWxzZSIsZTovOy99XSxrOmMsYzpuLmNvbmNhdChbe2I6L1woLyxlOi9cKS8sazpjLGM6bi5jb25jYXQoWyJzZWxmIl0pLHI6MH1dKSxyOjB9LHtjTjoiZnVuY3Rpb24iLGI6IigiK3QuSVIrIltcXComXFxzXSspKyIrYSxyQjohMCxlOi9bezs9XS8sZUU6ITAsazpjLGk6L1teXHdcc1wqJl0vLGM6W3tiOmEsckI6ITAsYzpbdC5UTV0scjowfSx7Y046InBhcmFtcyIsYjovXCgvLGU6L1wpLyxrOmMscjowLGM6W3QuQ0xDTSx0LkNCQ00scixzLGVdfSx0LkNMQ00sdC5DQkNNLGldfSx7Y046ImNsYXNzIixiSzoiY2xhc3Mgc3RydWN0IixlOi9bezs6XS8sYzpbe2I6LzwvLGU6Lz4vLGM6WyJzZWxmIl19LHQuVE1dfV0pLGV4cG9ydHM6e3ByZXByb2Nlc3NvcjppLHN0cmluZ3M6cixrOmN9fX0pO2hsanMucmVnaXN0ZXJMYW5ndWFnZSgicnVieSIsZnVuY3Rpb24oZSl7dmFyIGI9IlthLXpBLVpfXVxcdypbIT89XT98Wy0rfl1cXEB8PDx8Pj58PX58PT09P3w8PT58Wzw+XT0/fFxcKlxcKnxbLS8rJV4mKn5gfF18XFxbXFxdPT8iLHI9e2tleXdvcmQ6ImFuZCB0aGVuIGRlZmluZWQgbW9kdWxlIGluIHJldHVybiByZWRvIGlmIEJFR0lOIHJldHJ5IGVuZCBmb3Igc2VsZiB3aGVuIG5leHQgdW50aWwgZG8gYmVnaW4gdW5sZXNzIEVORCByZXNjdWUgZWxzZSBicmVhayB1bmRlZiBub3Qgc3VwZXIgY2xhc3MgY2FzZSByZXF1aXJlIHlpZWxkIGFsaWFzIHdoaWxlIGVuc3VyZSBlbHNpZiBvciBpbmNsdWRlIGF0dHJfcmVhZGVyIGF0dHJfd3JpdGVyIGF0dHJfYWNjZXNzb3IiLGxpdGVyYWw6InRydWUgZmFsc2UgbmlsIn0sYz17Y046ImRvY3RhZyIsYjoiQFtBLVphLXpdKyJ9LGE9e2I6IiM8IixlOiI+In0scz1bZS5DKCIjIiwiJCIse2M6W2NdfSksZS5DKCJeXFw9YmVnaW4iLCJeXFw9ZW5kIix7YzpbY10scjoxMH0pLGUuQygiXl9fRU5EX18iLCJcXG4kIildLG49e2NOOiJzdWJzdCIsYjoiI1xceyIsZToifSIsazpyfSx0PXtjTjoic3RyaW5nIixjOltlLkJFLG5dLHY6W3tiOi8nLyxlOi8nL30se2I6LyIvLGU6LyIvfSx7YjovYC8sZTovYC99LHtiOiIlW3FRd1d4XT9cXCgiLGU6IlxcKSJ9LHtiOiIlW3FRd1d4XT9cXFsiLGU6IlxcXSJ9LHtiOiIlW3FRd1d4XT97IixlOiJ9In0se2I6IiVbcVF3V3hdPzwiLGU6Ij4ifSx7YjoiJVtxUXdXeF0/LyIsZToiLyJ9LHtiOiIlW3FRd1d4XT8lIixlOiIlIn0se2I6IiVbcVF3V3hdPy0iLGU6Ii0ifSx7YjoiJVtxUXdXeF0/XFx8IixlOiJcXHwifSx7YjovXEJcPyhcXFxkezEsM318XFx4W0EtRmEtZjAtOV17MSwyfXxcXHVbQS1GYS1mMC05XXs0fXxcXD9cUylcYi99LHtiOi88PCgtPylcdyskLyxlOi9eXHMqXHcrJC99XX0saT17Y046InBhcmFtcyIsYjoiXFwoIixlOiJcXCkiLGVuZHNQYXJlbnQ6ITAsazpyfSxkPVt0LGEse2NOOiJjbGFzcyIsYks6ImNsYXNzIG1vZHVsZSIsZToiJHw7IixpOi89LyxjOltlLmluaGVyaXQoZS5UTSx7YjoiW0EtWmEtel9dXFx3Kig6OlxcdyspKihcXD98XFwhKT8ifSkse2I6IjxcXHMqIixjOlt7YjoiKCIrZS5JUisiOjopPyIrZS5JUn1dfV0uY29uY2F0KHMpfSx7Y046ImZ1bmN0aW9uIixiSzoiZGVmIixlOiIkfDsiLGM6W2UuaW5oZXJpdChlLlRNLHtiOmJ9KSxpXS5jb25jYXQocyl9LHtiOmUuSVIrIjo6In0se2NOOiJzeW1ib2wiLGI6ZS5VSVIrIihcXCF8XFw/KT86IixyOjB9LHtjTjoic3ltYm9sIixiOiI6KD8hXFxzKSIsYzpbdCx7YjpifV0scjowfSx7Y046Im51bWJlciIsYjoiKFxcYjBbMC03X10rKXwoXFxiMHhbMC05YS1mQS1GX10rKXwoXFxiWzEtOV1bMC05X10qKFxcLlswLTlfXSspPyl8WzBfXVxcYiIscjowfSx7YjoiKFxcJFxcVyl8KChcXCR8XFxAXFxAPykoXFx3KykpIn0se2NOOiJwYXJhbXMiLGI6L1x8LyxlOi9cfC8sazpyfSx7YjoiKCIrZS5SU1IrInx1bmxlc3MpXFxzKiIsazoidW5sZXNzIixjOlthLHtjTjoicmVnZXhwIixjOltlLkJFLG5dLGk6L1xuLyx2Olt7YjoiLyIsZToiL1thLXpdKiJ9LHtiOiIlcnsiLGU6In1bYS16XSoifSx7YjoiJXJcXCgiLGU6IlxcKVthLXpdKiJ9LHtiOiIlciEiLGU6IiFbYS16XSoifSx7YjoiJXJcXFsiLGU6IlxcXVthLXpdKiJ9XX1dLmNvbmNhdChzKSxyOjB9XS5jb25jYXQocyk7bi5jPWQsaS5jPWQ7dmFyIGw9Ils+P10+IixvPSJbXFx3I10rXFwoXFx3K1xcKTpcXGQrOlxcZCs+Iix1PSIoXFx3Ky0pP1xcZCtcXC5cXGQrXFwuXFxkKHBcXGQrKT9bXj5dKz4iLHc9W3tiOi9eXHMqPT4vLHN0YXJ0czp7ZToiJCIsYzpkfX0se2NOOiJtZXRhIixiOiJeKCIrbCsifCIrbysifCIrdSsiKSIsc3RhcnRzOntlOiIkIixjOmR9fV07cmV0dXJue2FsaWFzZXM6WyJyYiIsImdlbXNwZWMiLCJwb2RzcGVjIiwidGhvciIsImlyYiJdLGs6cixpOi9cL1wqLyxjOnMuY29uY2F0KHcpLmNvbmNhdChkKX19KTtobGpzLnJlZ2lzdGVyTGFuZ3VhZ2UoInlhbWwiLGZ1bmN0aW9uKGUpe3ZhciBiPSJ0cnVlIGZhbHNlIHllcyBubyBudWxsIixhPSJeWyBcXC1dKiIscj0iW2EtekEtWl9dW1xcd1xcLV0qIix0PXtjTjoiYXR0ciIsdjpbe2I6YStyKyI6In0se2I6YSsnIicrcisnIjonfSx7YjphKyInIityKyInOiJ9XX0sYz17Y046InRlbXBsYXRlLXZhcmlhYmxlIix2Olt7Yjoie3siLGU6In19In0se2I6IiV7IixlOiJ9In1dfSxsPXtjTjoic3RyaW5nIixyOjAsdjpbe2I6LycvLGU6LycvfSx7YjovIi8sZTovIi99LHtiOi9cUysvfV0sYzpbZS5CRSxjXX07cmV0dXJue2NJOiEwLGFsaWFzZXM6WyJ5bWwiLCJZQU1MIiwieWFtbCJdLGM6W3Qse2NOOiJtZXRhIixiOiJeLS0tcyokIixyOjEwfSx7Y046InN0cmluZyIsYjoiW1xcfD5dICokIixyRTohMCxjOmwuYyxlOnQudlswXS5ifSx7YjoiPCVbJT0tXT8iLGU6IlslLV0/JT4iLHNMOiJydWJ5IixlQjohMCxlRTohMCxyOjB9LHtjTjoidHlwZSIsYjoiISEiK2UuVUlSfSx7Y046Im1ldGEiLGI6IiYiK2UuVUlSKyIkIn0se2NOOiJtZXRhIixiOiJcXCoiK2UuVUlSKyIkIn0se2NOOiJidWxsZXQiLGI6Il4gKi0iLHI6MH0sZS5IQ00se2JLOmIsazp7bGl0ZXJhbDpifX0sZS5DTk0sbF19fSk7aGxqcy5yZWdpc3Rlckxhbmd1YWdlKCJjc3MiLGZ1bmN0aW9uKGUpe3ZhciBjPSJbYS16QS1aLV1bYS16QS1aMC05Xy1dKiIsdD17YjovW0EtWlxfXC5cLV0rXHMqOi8sckI6ITAsZToiOyIsZVc6ITAsYzpbe2NOOiJhdHRyaWJ1dGUiLGI6L1xTLyxlOiI6IixlRTohMCxzdGFydHM6e2VXOiEwLGVFOiEwLGM6W3tiOi9bXHctXStcKC8sckI6ITAsYzpbe2NOOiJidWlsdF9pbiIsYjovW1x3LV0rL30se2I6L1woLyxlOi9cKS8sYzpbZS5BU00sZS5RU01dfV19LGUuQ1NTTk0sZS5RU00sZS5BU00sZS5DQkNNLHtjTjoibnVtYmVyIixiOiIjWzAtOUEtRmEtZl0rIn0se2NOOiJtZXRhIixiOiIhaW1wb3J0YW50In1dfX1dfTtyZXR1cm57Y0k6ITAsaTovWz1cL3wnXCRdLyxjOltlLkNCQ00se2NOOiJzZWxlY3Rvci1pZCIsYjovI1tBLVphLXowLTlfLV0rL30se2NOOiJzZWxlY3Rvci1jbGFzcyIsYjovXC5bQS1aYS16MC05Xy1dKy99LHtjTjoic2VsZWN0b3ItYXR0ciIsYjovXFsvLGU6L1xdLyxpOiIkIn0se2NOOiJzZWxlY3Rvci1wc2V1ZG8iLGI6LzooOik/W2EtekEtWjAtOVxfXC1cK1woXCkiJy5dKy99LHtiOiJAKGZvbnQtZmFjZXxwYWdlKSIsbDoiW2Etei1dKyIsazoiZm9udC1mYWNlIHBhZ2UifSx7YjoiQCIsZToiW3s7XSIsaTovOi8sYzpbe2NOOiJrZXl3b3JkIixiOi9cdysvfSx7YjovXHMvLGVXOiEwLGVFOiEwLHI6MCxjOltlLkFTTSxlLlFTTSxlLkNTU05NXX1dfSx7Y046InNlbGVjdG9yLXRhZyIsYjpjLHI6MH0se2I6InsiLGU6In0iLGk6L1xTLyxjOltlLkNCQ00sdF19XX19KTtobGpzLnJlZ2lzdGVyTGFuZ3VhZ2UoImZvcnRyYW4iLGZ1bmN0aW9uKGUpe3ZhciB0PXtjTjoicGFyYW1zIixiOiJcXCgiLGU6IlxcKSJ9LG49e2xpdGVyYWw6Ii5GYWxzZS4gLlRydWUuIixrZXl3b3JkOiJraW5kIGRvIHdoaWxlIHByaXZhdGUgY2FsbCBpbnRyaW5zaWMgd2hlcmUgZWxzZXdoZXJlIHR5cGUgZW5kdHlwZSBlbmRtb2R1bGUgZW5kc2VsZWN0IGVuZGludGVyZmFjZSBlbmQgZW5kZG8gZW5kaWYgaWYgZm9yYWxsIGVuZGZvcmFsbCBvbmx5IGNvbnRhaW5zIGRlZmF1bHQgcmV0dXJuIHN0b3AgdGhlbiBwdWJsaWMgc3Vicm91dGluZXwxMCBmdW5jdGlvbiBwcm9ncmFtIC5hbmQuIC5vci4gLm5vdC4gLmxlLiAuZXEuIC5nZS4gLmd0LiAubHQuIGdvdG8gc2F2ZSBlbHNlIHVzZSBtb2R1bGUgc2VsZWN0IGNhc2UgYWNjZXNzIGJsYW5rIGRpcmVjdCBleGlzdCBmaWxlIGZtdCBmb3JtIGZvcm1hdHRlZCBpb3N0YXQgbmFtZSBuYW1lZCBuZXh0cmVjIG51bWJlciBvcGVuZWQgcmVjIHJlY2wgc2VxdWVudGlhbCBzdGF0dXMgdW5mb3JtYXR0ZWQgdW5pdCBjb250aW51ZSBmb3JtYXQgcGF1c2UgY3ljbGUgZXhpdCBjX251bGxfY2hhciBjX2FsZXJ0IGNfYmFja3NwYWNlIGNfZm9ybV9mZWVkIGZsdXNoIHdhaXQgZGVjaW1hbCByb3VuZCBpb21zZyBzeW5jaHJvbm91cyBub3Bhc3Mgbm9uX292ZXJyaWRhYmxlIHBhc3MgcHJvdGVjdGVkIHZvbGF0aWxlIGFic3RyYWN0IGV4dGVuZHMgaW1wb3J0IG5vbl9pbnRyaW5zaWMgdmFsdWUgZGVmZXJyZWQgZ2VuZXJpYyBmaW5hbCBlbnVtZXJhdG9yIGNsYXNzIGFzc29jaWF0ZSBiaW5kIGVudW0gY19pbnQgY19zaG9ydCBjX2xvbmcgY19sb25nX2xvbmcgY19zaWduZWRfY2hhciBjX3NpemVfdCBjX2ludDhfdCBjX2ludDE2X3QgY19pbnQzMl90IGNfaW50NjRfdCBjX2ludF9sZWFzdDhfdCBjX2ludF9sZWFzdDE2X3QgY19pbnRfbGVhc3QzMl90IGNfaW50X2xlYXN0NjRfdCBjX2ludF9mYXN0OF90IGNfaW50X2Zhc3QxNl90IGNfaW50X2Zhc3QzMl90IGNfaW50X2Zhc3Q2NF90IGNfaW50bWF4X3QgQ19pbnRwdHJfdCBjX2Zsb2F0IGNfZG91YmxlIGNfbG9uZ19kb3VibGUgY19mbG9hdF9jb21wbGV4IGNfZG91YmxlX2NvbXBsZXggY19sb25nX2RvdWJsZV9jb21wbGV4IGNfYm9vbCBjX2NoYXIgY19udWxsX3B0ciBjX251bGxfZnVucHRyIGNfbmV3X2xpbmUgY19jYXJyaWFnZV9yZXR1cm4gY19ob3Jpem9udGFsX3RhYiBjX3ZlcnRpY2FsX3RhYiBpc29fY19iaW5kaW5nIGNfbG9jIGNfZnVubG9jIGNfYXNzb2NpYXRlZCAgY19mX3BvaW50ZXIgY19wdHIgY19mdW5wdHIgaXNvX2ZvcnRyYW5fZW52IGNoYXJhY3Rlcl9zdG9yYWdlX3NpemUgZXJyb3JfdW5pdCBmaWxlX3N0b3JhZ2Vfc2l6ZSBpbnB1dF91bml0IGlvc3RhdF9lbmQgaW9zdGF0X2VvciBudW1lcmljX3N0b3JhZ2Vfc2l6ZSBvdXRwdXRfdW5pdCBjX2ZfcHJvY3BvaW50ZXIgaWVlZV9hcml0aG1ldGljIGllZWVfc3VwcG9ydF91bmRlcmZsb3dfY29udHJvbCBpZWVlX2dldF91bmRlcmZsb3dfbW9kZSBpZWVlX3NldF91bmRlcmZsb3dfbW9kZSBuZXd1bml0IGNvbnRpZ3VvdXMgcmVjdXJzaXZlIHBhZCBwb3NpdGlvbiBhY3Rpb24gZGVsaW0gcmVhZHdyaXRlIGVvciBhZHZhbmNlIG5tbCBpbnRlcmZhY2UgcHJvY2VkdXJlIG5hbWVsaXN0IGluY2x1ZGUgc2VxdWVuY2UgZWxlbWVudGFsIHB1cmUgaW50ZWdlciByZWFsIGNoYXJhY3RlciBjb21wbGV4IGxvZ2ljYWwgZGltZW5zaW9uIGFsbG9jYXRhYmxlfDEwIHBhcmFtZXRlciBleHRlcm5hbCBpbXBsaWNpdHwxMCBub25lIGRvdWJsZSBwcmVjaXNpb24gYXNzaWduIGludGVudCBvcHRpb25hbCBwb2ludGVyIHRhcmdldCBpbiBvdXQgY29tbW9uIGVxdWl2YWxlbmNlIGRhdGEiLGJ1aWx0X2luOiJhbG9nIGFsb2cxMCBhbWF4MCBhbWF4MSBhbWluMCBhbWluMSBhbW9kIGNhYnMgY2NvcyBjZXhwIGNsb2cgY3NpbiBjc3FydCBkYWJzIGRhY29zIGRhc2luIGRhdGFuIGRhdGFuMiBkY29zIGRjb3NoIGRkaW0gZGV4cCBkaW50IGRsb2cgZGxvZzEwIGRtYXgxIGRtaW4xIGRtb2QgZG5pbnQgZHNpZ24gZHNpbiBkc2luaCBkc3FydCBkdGFuIGR0YW5oIGZsb2F0IGlhYnMgaWRpbSBpZGludCBpZG5pbnQgaWZpeCBpc2lnbiBtYXgwIG1heDEgbWluMCBtaW4xIHNuZ2wgYWxnYW1hIGNkYWJzIGNkY29zIGNkZXhwIGNkbG9nIGNkc2luIGNkc3FydCBjcWFicyBjcWNvcyBjcWV4cCBjcWxvZyBjcXNpbiBjcXNxcnQgZGNtcGx4IGRjb25qZyBkZXJmIGRlcmZjIGRmbG9hdCBkZ2FtbWEgZGltYWcgZGxnYW1hIGlxaW50IHFhYnMgcWFjb3MgcWFzaW4gcWF0YW4gcWF0YW4yIHFjbXBseCBxY29uamcgcWNvcyBxY29zaCBxZGltIHFlcmYgcWVyZmMgcWV4cCBxZ2FtbWEgcWltYWcgcWxnYW1hIHFsb2cgcWxvZzEwIHFtYXgxIHFtaW4xIHFtb2QgcW5pbnQgcXNpZ24gcXNpbiBxc2luaCBxc3FydCBxdGFuIHF0YW5oIGFicyBhY29zIGFpbWFnIGFpbnQgYW5pbnQgYXNpbiBhdGFuIGF0YW4yIGNoYXIgY21wbHggY29uamcgY29zIGNvc2ggZXhwIGljaGFyIGluZGV4IGludCBsb2cgbG9nMTAgbWF4IG1pbiBuaW50IHNpZ24gc2luIHNpbmggc3FydCB0YW4gdGFuaCBwcmludCB3cml0ZSBkaW0gbGdlIGxndCBsbGUgbGx0IG1vZCBudWxsaWZ5IGFsbG9jYXRlIGRlYWxsb2NhdGUgYWRqdXN0bCBhZGp1c3RyIGFsbCBhbGxvY2F0ZWQgYW55IGFzc29jaWF0ZWQgYml0X3NpemUgYnRlc3QgY2VpbGluZyBjb3VudCBjc2hpZnQgZGF0ZV9hbmRfdGltZSBkaWdpdHMgZG90X3Byb2R1Y3QgZW9zaGlmdCBlcHNpbG9uIGV4cG9uZW50IGZsb29yIGZyYWN0aW9uIGh1Z2UgaWFuZCBpYmNsciBpYml0cyBpYnNldCBpZW9yIGlvciBpc2hmdCBpc2hmdGMgbGJvdW5kIGxlbl90cmltIG1hdG11bCBtYXhleHBvbmVudCBtYXhsb2MgbWF4dmFsIG1lcmdlIG1pbmV4cG9uZW50IG1pbmxvYyBtaW52YWwgbW9kdWxvIG12Yml0cyBuZWFyZXN0IHBhY2sgcHJlc2VudCBwcm9kdWN0IHJhZGl4IHJhbmRvbV9udW1iZXIgcmFuZG9tX3NlZWQgcmFuZ2UgcmVwZWF0IHJlc2hhcGUgcnJzcGFjaW5nIHNjYWxlIHNjYW4gc2VsZWN0ZWRfaW50X2tpbmQgc2VsZWN0ZWRfcmVhbF9raW5kIHNldF9leHBvbmVudCBzaGFwZSBzaXplIHNwYWNpbmcgc3ByZWFkIHN1bSBzeXN0ZW1fY2xvY2sgdGlueSB0cmFuc3Bvc2UgdHJpbSB1Ym91bmQgdW5wYWNrIHZlcmlmeSBhY2hhciBpYWNoYXIgdHJhbnNmZXIgZGJsZSBlbnRyeSBkcHJvZCBjcHVfdGltZSBjb21tYW5kX2FyZ3VtZW50X2NvdW50IGdldF9jb21tYW5kIGdldF9jb21tYW5kX2FyZ3VtZW50IGdldF9lbnZpcm9ubWVudF92YXJpYWJsZSBpc19pb3N0YXRfZW5kIGllZWVfYXJpdGhtZXRpYyBpZWVlX3N1cHBvcnRfdW5kZXJmbG93X2NvbnRyb2wgaWVlZV9nZXRfdW5kZXJmbG93X21vZGUgaWVlZV9zZXRfdW5kZXJmbG93X21vZGUgaXNfaW9zdGF0X2VvciBtb3ZlX2FsbG9jIG5ld19saW5lIHNlbGVjdGVkX2NoYXJfa2luZCBzYW1lX3R5cGVfYXMgZXh0ZW5kc190eXBlX29mYWNvc2ggYXNpbmggYXRhbmggYmVzc2VsX2owIGJlc3NlbF9qMSBiZXNzZWxfam4gYmVzc2VsX3kwIGJlc3NlbF95MSBiZXNzZWxfeW4gZXJmIGVyZmMgZXJmY19zY2FsZWQgZ2FtbWEgbG9nX2dhbW1hIGh5cG90IG5vcm0yIGF0b21pY19kZWZpbmUgYXRvbWljX3JlZiBleGVjdXRlX2NvbW1hbmRfbGluZSBsZWFkeiB0cmFpbHogc3RvcmFnZV9zaXplIG1lcmdlX2JpdHMgYmdlIGJndCBibGUgYmx0IGRzaGlmdGwgZHNoaWZ0ciBmaW5kbG9jIGlhbGwgaWFueSBpcGFyaXR5IGltYWdlX2luZGV4IGxjb2JvdW5kIHVjb2JvdW5kIG1hc2tsIG1hc2tyIG51bV9pbWFnZXMgcGFyaXR5IHBvcGNudCBwb3BwYXIgc2hpZnRhIHNoaWZ0bCBzaGlmdHIgdGhpc19pbWFnZSJ9O3JldHVybntjSTohMCxhbGlhc2VzOlsiZjkwIiwiZjk1Il0sazpuLGk6L1wvXCovLGM6W2UuaW5oZXJpdChlLkFTTSx7Y046InN0cmluZyIscjowfSksZS5pbmhlcml0KGUuUVNNLHtjTjoic3RyaW5nIixyOjB9KSx7Y046ImZ1bmN0aW9uIixiSzoic3Vicm91dGluZSBmdW5jdGlvbiBwcm9ncmFtIixpOiJbJHs9XFxuXSIsYzpbZS5VVE0sdF19LGUuQygiISIsIiQiLHtyOjB9KSx7Y046Im51bWJlciIsYjoiKD89XFxifFxcK3xcXC18XFwuKSg/PVxcLlxcZHxcXGQpKD86XFxkKyk/KD86XFwuP1xcZCopKD86W2RlXVsrLV0/XFxkKyk/XFxiXFwuPyIscjowfV19fSk7aGxqcy5yZWdpc3Rlckxhbmd1YWdlKCJhd2siLGZ1bmN0aW9uKGUpe3ZhciByPXtjTjoidmFyaWFibGUiLHY6W3tiOi9cJFtcd1xkI0BdW1x3XGRfXSovfSx7YjovXCRceyguKj8pfS99XX0sYj0iQkVHSU4gRU5EIGlmIGVsc2Ugd2hpbGUgZG8gZm9yIGluIGJyZWFrIGNvbnRpbnVlIGRlbGV0ZSBuZXh0IG5leHRmaWxlIGZ1bmN0aW9uIGZ1bmMgZXhpdHwxMCIsbj17Y046InN0cmluZyIsYzpbZS5CRV0sdjpbe2I6Lyh1fGIpP3I/JycnLyxlOi8nJycvLHI6MTB9LHtiOi8odXxiKT9yPyIiIi8sZTovIiIiLyxyOjEwfSx7YjovKHV8cnx1ciknLyxlOi8nLyxyOjEwfSx7YjovKHV8cnx1cikiLyxlOi8iLyxyOjEwfSx7YjovKGJ8YnIpJy8sZTovJy99LHtiOi8oYnxicikiLyxlOi8iL30sZS5BU00sZS5RU01dfTtyZXR1cm57azp7a2V5d29yZDpifSxjOltyLG4sZS5STSxlLkhDTSxlLk5NXX19KTtobGpzLnJlZ2lzdGVyTGFuZ3VhZ2UoIm1ha2VmaWxlIixmdW5jdGlvbihlKXt2YXIgaT17Y046InZhcmlhYmxlIix2Olt7YjoiXFwkXFwoIitlLlVJUisiXFwpIixjOltlLkJFXX0se2I6L1wkW0AlPD9cXlwrXCpdL31dfSxyPXtjTjoic3RyaW5nIixiOi8iLyxlOi8iLyxjOltlLkJFLGldfSxhPXtjTjoidmFyaWFibGUiLGI6L1wkXChbXHctXStccy8sZTovXCkvLGs6e2J1aWx0X2luOiJzdWJzdCBwYXRzdWJzdCBzdHJpcCBmaW5kc3RyaW5nIGZpbHRlciBmaWx0ZXItb3V0IHNvcnQgd29yZCB3b3JkbGlzdCBmaXJzdHdvcmQgbGFzdHdvcmQgZGlyIG5vdGRpciBzdWZmaXggYmFzZW5hbWUgYWRkc3VmZml4IGFkZHByZWZpeCBqb2luIHdpbGRjYXJkIHJlYWxwYXRoIGFic3BhdGggZXJyb3Igd2FybmluZyBzaGVsbCBvcmlnaW4gZmxhdm9yIGZvcmVhY2ggaWYgb3IgYW5kIGNhbGwgZXZhbCBmaWxlIHZhbHVlIn0sYzpbaV19LG49e2I6Il4iK2UuVUlSKyJcXHMqWzorP10/PSIsaToiXFxuIixyQjohMCxjOlt7YjoiXiIrZS5VSVIsZToiWzorP10/PSIsZUU6ITB9XX0sdD17Y046Im1ldGEiLGI6L15cLlBIT05ZOi8sZTovJC8sazp7Im1ldGEta2V5d29yZCI6Ii5QSE9OWSJ9LGw6L1tcLlx3XSsvfSxsPXtjTjoic2VjdGlvbiIsYjovXlteXHNdKzovLGU6LyQvLGM6W2ldfTtyZXR1cm57YWxpYXNlczpbIm1rIiwibWFrIl0sazoiZGVmaW5lIGVuZGVmIHVuZGVmaW5lIGlmZGVmIGlmbmRlZiBpZmVxIGlmbmVxIGVsc2UgZW5kaWYgaW5jbHVkZSAtaW5jbHVkZSBzaW5jbHVkZSBvdmVycmlkZSBleHBvcnQgdW5leHBvcnQgcHJpdmF0ZSB2cGF0aCIsbDovW1x3LV0rLyxjOltlLkhDTSxpLHIsYSxuLHQsbF19fSk7aGxqcy5yZWdpc3Rlckxhbmd1YWdlKCJqYXZhIixmdW5jdGlvbihlKXt2YXIgYT0iW8OALcq4YS16QS1aXyRdW8OALcq4YS16QS1aXyQwLTldKiIsdD1hKyIoPCIrYSsiKFxccyosXFxzKiIrYSsiKSo+KT8iLHI9ImZhbHNlIHN5bmNocm9uaXplZCBpbnQgYWJzdHJhY3QgZmxvYXQgcHJpdmF0ZSBjaGFyIGJvb2xlYW4gc3RhdGljIG51bGwgaWYgY29uc3QgZm9yIHRydWUgd2hpbGUgbG9uZyBzdHJpY3RmcCBmaW5hbGx5IHByb3RlY3RlZCBpbXBvcnQgbmF0aXZlIGZpbmFsIHZvaWQgZW51bSBlbHNlIGJyZWFrIHRyYW5zaWVudCBjYXRjaCBpbnN0YW5jZW9mIGJ5dGUgc3VwZXIgdm9sYXRpbGUgY2FzZSBhc3NlcnQgc2hvcnQgcGFja2FnZSBkZWZhdWx0IGRvdWJsZSBwdWJsaWMgdHJ5IHRoaXMgc3dpdGNoIGNvbnRpbnVlIHRocm93cyBwcm90ZWN0ZWQgcHVibGljIHByaXZhdGUgbW9kdWxlIHJlcXVpcmVzIGV4cG9ydHMgZG8iLHM9IlxcYigwW2JCXShbMDFdK1swMV9dK1swMV0rfFswMV0rKXwwW3hYXShbYS1mQS1GMC05XStbYS1mQS1GMC05X10rW2EtZkEtRjAtOV0rfFthLWZBLUYwLTldKyl8KChbXFxkXStbXFxkX10rW1xcZF0rfFtcXGRdKykoXFwuKFtcXGRdK1tcXGRfXStbXFxkXSt8W1xcZF0rKSk/fFxcLihbXFxkXStbXFxkX10rW1xcZF0rfFtcXGRdKykpKFtlRV1bLStdP1xcZCspPylbbExmRl0/IixjPXtjTjoibnVtYmVyIixiOnMscjowfTtyZXR1cm57YWxpYXNlczpbImpzcCJdLGs6cixpOi88XC98Iy8sYzpbZS5DKCIvXFwqXFwqIiwiXFwqLyIse3I6MCxjOlt7YjovXHcrQC8scjowfSx7Y046ImRvY3RhZyIsYjoiQFtBLVphLXpdKyJ9XX0pLGUuQ0xDTSxlLkNCQ00sZS5BU00sZS5RU00se2NOOiJjbGFzcyIsYks6ImNsYXNzIGludGVyZmFjZSIsZTovW3s7PV0vLGVFOiEwLGs6ImNsYXNzIGludGVyZmFjZSIsaTovWzoiXFtcXV0vLGM6W3tiSzoiZXh0ZW5kcyBpbXBsZW1lbnRzIn0sZS5VVE1dfSx7Yks6Im5ldyB0aHJvdyByZXR1cm4gZWxzZSIscjowfSx7Y046ImZ1bmN0aW9uIixiOiIoIit0KyJcXHMrKSsiK2UuVUlSKyJcXHMqXFwoIixyQjohMCxlOi9bezs9XS8sZUU6ITAsazpyLGM6W3tiOmUuVUlSKyJcXHMqXFwoIixyQjohMCxyOjAsYzpbZS5VVE1dfSx7Y046InBhcmFtcyIsYjovXCgvLGU6L1wpLyxrOnIscjowLGM6W2UuQVNNLGUuUVNNLGUuQ05NLGUuQ0JDTV19LGUuQ0xDTSxlLkNCQ01dfSxjLHtjTjoibWV0YSIsYjoiQFtBLVphLXpdKyJ9XX19KTtobGpzLnJlZ2lzdGVyTGFuZ3VhZ2UoInN0YW4iLGZ1bmN0aW9uKGUpe3JldHVybntjOltlLkhDTSxlLkNMQ00sZS5DQkNNLHtiOmUuVUlSLGw6ZS5VSVIsazp7bmFtZToiZm9yIGluIHdoaWxlIHJlcGVhdCB1bnRpbCBpZiB0aGVuIGVsc2UiLHN5bWJvbDoiYmVybm91bGxpIGJlcm5vdWxsaV9sb2dpdCBiaW5vbWlhbCBiaW5vbWlhbF9sb2dpdCBiZXRhX2Jpbm9taWFsIGh5cGVyZ2VvbWV0cmljIGNhdGVnb3JpY2FsIGNhdGVnb3JpY2FsX2xvZ2l0IG9yZGVyZWRfbG9naXN0aWMgbmVnX2Jpbm9taWFsIG5lZ19iaW5vbWlhbF8yIG5lZ19iaW5vbWlhbF8yX2xvZyBwb2lzc29uIHBvaXNzb25fbG9nIG11bHRpbm9taWFsIG5vcm1hbCBleHBfbW9kX25vcm1hbCBza2V3X25vcm1hbCBzdHVkZW50X3QgY2F1Y2h5IGRvdWJsZV9leHBvbmVudGlhbCBsb2dpc3RpYyBndW1iZWwgbG9nbm9ybWFsIGNoaV9zcXVhcmUgaW52X2NoaV9zcXVhcmUgc2NhbGVkX2ludl9jaGlfc3F1YXJlIGV4cG9uZW50aWFsIGludl9nYW1tYSB3ZWlidWxsIGZyZWNoZXQgcmF5bGVpZ2ggd2llbmVyIHBhcmV0byBwYXJldG9fdHlwZV8yIHZvbl9taXNlcyB1bmlmb3JtIG11bHRpX25vcm1hbCBtdWx0aV9ub3JtYWxfcHJlYyBtdWx0aV9ub3JtYWxfY2hvbGVza3kgbXVsdGlfZ3AgbXVsdGlfZ3BfY2hvbGVza3kgbXVsdGlfc3R1ZGVudF90IGdhdXNzaWFuX2RsbV9vYnMgZGlyaWNobGV0IGxral9jb3JyIGxral9jb3JyX2Nob2xlc2t5IHdpc2hhcnQgaW52X3dpc2hhcnQiLCJzZWxlY3Rvci10YWciOiJpbnQgcmVhbCB2ZWN0b3Igc2ltcGxleCB1bml0X3ZlY3RvciBvcmRlcmVkIHBvc2l0aXZlX29yZGVyZWQgcm93X3ZlY3RvciBtYXRyaXggY2hvbGVza3lfZmFjdG9yX2NvcnIgY2hvbGVza3lfZmFjdG9yX2NvdiBjb3JyX21hdHJpeCBjb3ZfbWF0cml4Iix0aXRsZToiZnVuY3Rpb25zIG1vZGVsIGRhdGEgcGFyYW1ldGVycyBxdWFudGl0aWVzIHRyYW5zZm9ybWVkIGdlbmVyYXRlZCIsbGl0ZXJhbDoidHJ1ZSBmYWxzZSJ9LHI6MH0se2NOOiJudW1iZXIiLGI6IjBbeFhdWzAtOWEtZkEtRl0rW0xpXT9cXGIiLHI6MH0se2NOOiJudW1iZXIiLGI6IjBbeFhdWzAtOWEtZkEtRl0rW0xpXT9cXGIiLHI6MH0se2NOOiJudW1iZXIiLGI6IlxcZCsoPzpbZUVdWytcXC1dP1xcZCopP0xcXGIiLHI6MH0se2NOOiJudW1iZXIiLGI6IlxcZCtcXC4oPyFcXGQpKD86aVxcYik/IixyOjB9LHtjTjoibnVtYmVyIixiOiJcXGQrKD86XFwuXFxkKik/KD86W2VFXVsrXFwtXT9cXGQqKT9pP1xcYiIscjowfSx7Y046Im51bWJlciIsYjoiXFwuXFxkKyg/OltlRV1bK1xcLV0/XFxkKik/aT9cXGIiLHI6MH1dfX0pO2hsanMucmVnaXN0ZXJMYW5ndWFnZSgiamF2YXNjcmlwdCIsZnVuY3Rpb24oZSl7dmFyIHI9IltBLVphLXokX11bMC05QS1aYS16JF9dKiIsdD17a2V5d29yZDoiaW4gb2YgaWYgZm9yIHdoaWxlIGZpbmFsbHkgdmFyIG5ldyBmdW5jdGlvbiBkbyByZXR1cm4gdm9pZCBlbHNlIGJyZWFrIGNhdGNoIGluc3RhbmNlb2Ygd2l0aCB0aHJvdyBjYXNlIGRlZmF1bHQgdHJ5IHRoaXMgc3dpdGNoIGNvbnRpbnVlIHR5cGVvZiBkZWxldGUgbGV0IHlpZWxkIGNvbnN0IGV4cG9ydCBzdXBlciBkZWJ1Z2dlciBhcyBhc3luYyBhd2FpdCBzdGF0aWMgaW1wb3J0IGZyb20gYXMiLGxpdGVyYWw6InRydWUgZmFsc2UgbnVsbCB1bmRlZmluZWQgTmFOIEluZmluaXR5IixidWlsdF9pbjoiZXZhbCBpc0Zpbml0ZSBpc05hTiBwYXJzZUZsb2F0IHBhcnNlSW50IGRlY29kZVVSSSBkZWNvZGVVUklDb21wb25lbnQgZW5jb2RlVVJJIGVuY29kZVVSSUNvbXBvbmVudCBlc2NhcGUgdW5lc2NhcGUgT2JqZWN0IEZ1bmN0aW9uIEJvb2xlYW4gRXJyb3IgRXZhbEVycm9yIEludGVybmFsRXJyb3IgUmFuZ2VFcnJvciBSZWZlcmVuY2VFcnJvciBTdG9wSXRlcmF0aW9uIFN5bnRheEVycm9yIFR5cGVFcnJvciBVUklFcnJvciBOdW1iZXIgTWF0aCBEYXRlIFN0cmluZyBSZWdFeHAgQXJyYXkgRmxvYXQzMkFycmF5IEZsb2F0NjRBcnJheSBJbnQxNkFycmF5IEludDMyQXJyYXkgSW50OEFycmF5IFVpbnQxNkFycmF5IFVpbnQzMkFycmF5IFVpbnQ4QXJyYXkgVWludDhDbGFtcGVkQXJyYXkgQXJyYXlCdWZmZXIgRGF0YVZpZXcgSlNPTiBJbnRsIGFyZ3VtZW50cyByZXF1aXJlIG1vZHVsZSBjb25zb2xlIHdpbmRvdyBkb2N1bWVudCBTeW1ib2wgU2V0IE1hcCBXZWFrU2V0IFdlYWtNYXAgUHJveHkgUmVmbGVjdCBQcm9taXNlIn0sYT17Y046Im51bWJlciIsdjpbe2I6IlxcYigwW2JCXVswMV0rKSJ9LHtiOiJcXGIoMFtvT11bMC03XSspIn0se2I6ZS5DTlJ9XSxyOjB9LG49e2NOOiJzdWJzdCIsYjoiXFwkXFx7IixlOiJcXH0iLGs6dCxjOltdfSxjPXtjTjoic3RyaW5nIixiOiJgIixlOiJgIixjOltlLkJFLG5dfTtuLmM9W2UuQVNNLGUuUVNNLGMsYSxlLlJNXTt2YXIgcz1uLmMuY29uY2F0KFtlLkNCQ00sZS5DTENNXSk7cmV0dXJue2FsaWFzZXM6WyJqcyIsImpzeCJdLGs6dCxjOlt7Y046Im1ldGEiLHI6MTAsYjovXlxzKlsnIl11c2UgKHN0cmljdHxhc20pWyciXS99LHtjTjoibWV0YSIsYjovXiMhLyxlOi8kL30sZS5BU00sZS5RU00sYyxlLkNMQ00sZS5DQkNNLGEse2I6L1t7LF1ccyovLHI6MCxjOlt7YjpyKyJcXHMqOiIsckI6ITAscjowLGM6W3tjTjoiYXR0ciIsYjpyLHI6MH1dfV19LHtiOiIoIitlLlJTUisifFxcYihjYXNlfHJldHVybnx0aHJvdylcXGIpXFxzKiIsazoicmV0dXJuIHRocm93IGNhc2UiLGM6W2UuQ0xDTSxlLkNCQ00sZS5STSx7Y046ImZ1bmN0aW9uIixiOiIoXFwoLio/XFwpfCIrcisiKVxccyo9PiIsckI6ITAsZToiXFxzKj0+IixjOlt7Y046InBhcmFtcyIsdjpbe2I6cn0se2I6L1woXHMqXCkvfSx7YjovXCgvLGU6L1wpLyxlQjohMCxlRTohMCxrOnQsYzpzfV19XX0se2I6LzwvLGU6LyhcL1x3K3xcdytcLyk+LyxzTDoieG1sIixjOlt7YjovPFx3K1xzKlwvPi8sc2tpcDohMH0se2I6LzxcdysvLGU6LyhcL1x3K3xcdytcLyk+Lyxza2lwOiEwLGM6W3tiOi88XHcrXHMqXC8+Lyxza2lwOiEwfSwic2VsZiJdfV19XSxyOjB9LHtjTjoiZnVuY3Rpb24iLGJLOiJmdW5jdGlvbiIsZTovXHsvLGVFOiEwLGM6W2UuaW5oZXJpdChlLlRNLHtiOnJ9KSx7Y046InBhcmFtcyIsYjovXCgvLGU6L1wpLyxlQjohMCxlRTohMCxjOnN9XSxpOi9cW3wlL30se2I6L1wkWyguXS99LGUuTUVUSE9EX0dVQVJELHtjTjoiY2xhc3MiLGJLOiJjbGFzcyIsZTovW3s7PV0vLGVFOiEwLGk6L1s6IlxbXF1dLyxjOlt7Yks6ImV4dGVuZHMifSxlLlVUTV19LHtiSzoiY29uc3RydWN0b3IiLGU6L1x7LyxlRTohMH1dLGk6LyMoPyEhKS99fSk7aGxqcy5yZWdpc3Rlckxhbmd1YWdlKCJ0ZXgiLGZ1bmN0aW9uKGMpe3ZhciBlPXtjTjoidGFnIixiOi9cXC8scjowLGM6W3tjTjoibmFtZSIsdjpbe2I6L1thLXpBLVrQsC3Rj9CQLdGPXStbKl0/L30se2I6L1teYS16QS1a0LAt0Y/QkC3RjzAtOV0vfV0sc3RhcnRzOntlVzohMCxyOjAsYzpbe2NOOiJzdHJpbmciLHY6W3tiOi9cWy8sZTovXF0vfSx7YjovXHsvLGU6L1x9L31dfSx7YjovXHMqPVxzKi8sZVc6ITAscjowLGM6W3tjTjoibnVtYmVyIixiOi8tP1xkKlwuP1xkKyhwdHxwY3xtbXxjbXxpbnxkZHxjY3xleHxlbSk/L31dfV19fV19O3JldHVybntjOltlLHtjTjoiZm9ybXVsYSIsYzpbZV0scjowLHY6W3tiOi9cJFwkLyxlOi9cJFwkL30se2I6L1wkLyxlOi9cJC99XX0sYy5DKCIlIiwiJCIse3I6MH0pXX19KTtobGpzLnJlZ2lzdGVyTGFuZ3VhZ2UoInhtbCIsZnVuY3Rpb24ocyl7dmFyIGU9IltBLVphLXowLTlcXC5fOi1dKyIsdD17ZVc6ITAsaTovPC8scjowLGM6W3tjTjoiYXR0ciIsYjplLHI6MH0se2I6Lz1ccyovLHI6MCxjOlt7Y046InN0cmluZyIsZW5kc1BhcmVudDohMCx2Olt7YjovIi8sZTovIi99LHtiOi8nLyxlOi8nL30se2I6L1teXHMiJz08PmBdKy99XX1dfV19O3JldHVybnthbGlhc2VzOlsiaHRtbCIsInhodG1sIiwicnNzIiwiYXRvbSIsInhqYiIsInhzZCIsInhzbCIsInBsaXN0Il0sY0k6ITAsYzpbe2NOOiJtZXRhIixiOiI8IURPQ1RZUEUiLGU6Ij4iLHI6MTAsYzpbe2I6IlxcWyIsZToiXFxdIn1dfSxzLkMoIjwhLS0iLCItLT4iLHtyOjEwfSkse2I6IjxcXCFcXFtDREFUQVxcWyIsZToiXFxdXFxdPiIscjoxMH0se2I6LzxcPyhwaHApPy8sZTovXD8+LyxzTDoicGhwIixjOlt7YjoiL1xcKiIsZToiXFwqLyIsc2tpcDohMH1dfSx7Y046InRhZyIsYjoiPHN0eWxlKD89XFxzfD58JCkiLGU6Ij4iLGs6e25hbWU6InN0eWxlIn0sYzpbdF0sc3RhcnRzOntlOiI8L3N0eWxlPiIsckU6ITAsc0w6WyJjc3MiLCJ4bWwiXX19LHtjTjoidGFnIixiOiI8c2NyaXB0KD89XFxzfD58JCkiLGU6Ij4iLGs6e25hbWU6InNjcmlwdCJ9LGM6W3RdLHN0YXJ0czp7ZToiPC9zY3JpcHQ+IixyRTohMCxzTDpbImFjdGlvbnNjcmlwdCIsImphdmFzY3JpcHQiLCJoYW5kbGViYXJzIiwieG1sIl19fSx7Y046Im1ldGEiLHY6W3tiOi88XD94bWwvLGU6L1w/Pi8scjoxMH0se2I6LzxcP1x3Ky8sZTovXD8+L31dfSx7Y046InRhZyIsYjoiPC8/IixlOiIvPz4iLGM6W3tjTjoibmFtZSIsYjovW15cLz48XHNdKy8scjowfSx0XX1dfX0pO2hsanMucmVnaXN0ZXJMYW5ndWFnZSgibWFya2Rvd24iLGZ1bmN0aW9uKGUpe3JldHVybnthbGlhc2VzOlsibWQiLCJta2Rvd24iLCJta2QiXSxjOlt7Y046InNlY3Rpb24iLHY6W3tiOiJeI3sxLDZ9IixlOiIkIn0se2I6Il4uKz9cXG5bPS1dezIsfSQifV19LHtiOiI8IixlOiI+IixzTDoieG1sIixyOjB9LHtjTjoiYnVsbGV0IixiOiJeKFsqKy1dfChcXGQrXFwuKSlcXHMrIn0se2NOOiJzdHJvbmciLGI6IlsqX117Mn0uKz9bKl9dezJ9In0se2NOOiJlbXBoYXNpcyIsdjpbe2I6IlxcKi4rP1xcKiJ9LHtiOiJfLis/XyIscjowfV19LHtjTjoicXVvdGUiLGI6Il4+XFxzKyIsZToiJCJ9LHtjTjoiY29kZSIsdjpbe2I6Il5gYGB3KnMqJCIsZToiXmBgYHMqJCJ9LHtiOiJgLis/YCJ9LHtiOiJeKCB7NH18CSkiLGU6IiQiLHI6MH1dfSx7YjoiXlstXFwqXXszLH0iLGU6IiQifSx7YjoiXFxbLis/XFxdW1xcKFxcW10uKj9bXFwpXFxdXSIsckI6ITAsYzpbe2NOOiJzdHJpbmciLGI6IlxcWyIsZToiXFxdIixlQjohMCxyRTohMCxyOjB9LHtjTjoibGluayIsYjoiXFxdXFwoIixlOiJcXCkiLGVCOiEwLGVFOiEwfSx7Y046InN5bWJvbCIsYjoiXFxdXFxbIixlOiJcXF0iLGVCOiEwLGVFOiEwfV0scjoxMH0se2I6L15cW1teXG5dK1xdOi8sckI6ITAsYzpbe2NOOiJzeW1ib2wiLGI6L1xbLyxlOi9cXS8sZUI6ITAsZUU6ITB9LHtjTjoibGluayIsYjovOlxzKi8sZTovJC8sZUI6ITB9XX1dfX0pO2hsanMucmVnaXN0ZXJMYW5ndWFnZSgianNvbiIsZnVuY3Rpb24oZSl7dmFyIGk9e2xpdGVyYWw6InRydWUgZmFsc2UgbnVsbCJ9LG49W2UuUVNNLGUuQ05NXSxyPXtlOiIsIixlVzohMCxlRTohMCxjOm4sazppfSx0PXtiOiJ7IixlOiJ9IixjOlt7Y046ImF0dHIiLGI6LyIvLGU6LyIvLGM6W2UuQkVdLGk6IlxcbiJ9LGUuaW5oZXJpdChyLHtiOi86L30pXSxpOiJcXFMifSxjPXtiOiJcXFsiLGU6IlxcXSIsYzpbZS5pbmhlcml0KHIpXSxpOiJcXFMifTtyZXR1cm4gbi5zcGxpY2Uobi5sZW5ndGgsMCx0LGMpLHtjOm4sazppLGk6IlxcUyJ9fSk7"></script>
 
 <style type="text/css">code{white-space: pre;}</style>
 <style type="text/css">
@@ -256,9 +71,7 @@ h6 {
 </style>
 
 
-</head>
 
-<body>
 
 <style type="text/css">
 .main-container {
@@ -283,17 +96,79 @@ img {
 button.code-folding-btn:focus {
   outline: none;
 }
+summary {
+  display: list-item;
+}
 </style>
 
 
 
-<div class="container-fluid main-container">
-
 <!-- tabsets -->
+
+<style type="text/css">
+.tabset-dropdown > .nav-tabs {
+  display: inline-table;
+  max-height: 500px;
+  min-height: 44px;
+  overflow-y: auto;
+  background: white;
+  border: 1px solid #ddd;
+  border-radius: 4px;
+}
+
+.tabset-dropdown > .nav-tabs > li.active:before {
+  content: "";
+  font-family: 'Glyphicons Halflings';
+  display: inline-block;
+  padding: 10px;
+  border-right: 1px solid #ddd;
+}
+
+.tabset-dropdown > .nav-tabs.nav-tabs-open > li.active:before {
+  content: "";
+  border: none;
+}
+
+.tabset-dropdown > .nav-tabs.nav-tabs-open:before {
+  content: "";
+  font-family: 'Glyphicons Halflings';
+  display: inline-block;
+  padding: 10px;
+  border-right: 1px solid #ddd;
+}
+
+.tabset-dropdown > .nav-tabs > li.active {
+  display: block;
+}
+
+.tabset-dropdown > .nav-tabs > li > a,
+.tabset-dropdown > .nav-tabs > li > a:focus,
+.tabset-dropdown > .nav-tabs > li > a:hover {
+  border: none;
+  display: inline-block;
+  border-radius: 4px;
+}
+
+.tabset-dropdown > .nav-tabs.nav-tabs-open > li {
+  display: block;
+  float: none;
+}
+
+.tabset-dropdown > .nav-tabs > li {
+  display: none;
+}
+</style>
+
 <script>
 $(document).ready(function () {
   window.buildTabsets("TOC");
 });
+
+$(document).ready(function () {
+  $('.tabset-dropdown > .nav-tabs > li').click(function () {
+    $(this).parent().toggleClass('nav-tabs-open')
+  });
+});
 </script>
 
 <!-- code folding -->
@@ -301,23 +176,37 @@ $(document).ready(function () {
 
 
 
+</head>
+
+<body>
+
+
+<div class="container-fluid main-container">
+
+
 
 
 <div class="fluid-row" id="header">
 
 
 
-<h1 class="title toc-ignore">Week 6 Worked Examples</h1>
-<h4 class="author"><em>Jeremy Foote</em></h4>
-<h4 class="date"><em>April 11, 2019</em></h4>
+<h1 class="title toc-ignore">Week 6 problem set: Worked solutions</h1>
+<h3 class="subtitle">Statistics and statistical programming<br />
+Northwestern University<br />
+MTS 525</h3>
+<h4 class="author">Jeremy Foote</h4>
+<h4 class="date">April 11, 2019</h4>
 
 </div>
 
 
 <div id="programming-questions" class="section level2">
 <h2>Programming Questions</h2>
-<p>PC0. First we import the data.</p>
-<pre class="r"><code>raw_df = read.csv(&quot;~/Desktop/DeleteMe/Teaching/owan03.csv&quot;) # Note that I saved the file as a CSV for importing to R
+<p>Note that Jeremy created this solution set with some code that uses functions drawn from the <code>tidyverse</code>. Wherever possible, we have also provided solution code using base R functions or alternatives since the course has not really provided an introduction to the tidyverse.</p>
+<div id="pc0" class="section level3">
+<h3>PC0</h3>
+<p>First we import the data.</p>
+<pre class="r"><code>raw_df = read.csv(&quot;/tmp/owan03.csv&quot;) # Note that I saved the file as a CSV for importing to R
 head(raw_df)</code></pre>
 <pre><code>##   X1 X2 X3 X4
 ## 1 70 49 30 34
@@ -326,35 +215,43 @@ head(raw_df)</code></pre>
 ## 4 87 67 65 48
 ## 5 92 70 76 65
 ## 6 93 74 83 91</code></pre>
-<p>PC1. Let’s reshape the data</p>
-<pre class="r"><code>library(tidyverse)</code></pre>
-<pre><code>## ── Attaching packages ───────────────────────────────────────────────────────────────────────────────────────────────────────── tidyverse 1.2.1 ──</code></pre>
-<pre><code>## ✔ ggplot2 3.1.0     ✔ purrr   0.2.5
-## ✔ tibble  2.1.1     ✔ dplyr   0.7.7
-## ✔ tidyr   0.8.2     ✔ stringr 1.3.1
-## ✔ readr   1.1.1     ✔ forcats 0.3.0</code></pre>
-<pre><code>## ── Conflicts ──────────────────────────────────────────────────────────────────────────────────────────────────────────── tidyverse_conflicts() ──
-## ✖ dplyr::filter() masks stats::filter()
-## ✖ dplyr::lag()    masks stats::lag()</code></pre>
+</div>
+<div id="pc1" class="section level3">
+<h3>PC1</h3>
+<p>Let’s organize the data. We want this in “long” format where the survival times in weeks for each of the three dosage levels are all arranged in a single column and another variable labels the dosage level. There are (surprise!) many ways to do this. Here are two examples using a function called <code>melt()</code> from a library called <code>reshape</code> and another function called <code>gather()</code> from the <code>tidyverse</code> library. Note that you’ll need to install the respective library and load it before you can call either function!</p>
+<pre class="r"><code>library(reshape)
+library(tidyr)</code></pre>
+<pre><code>## 
+## Attaching package: 'tidyr'</code></pre>
+<pre><code>## The following objects are masked from 'package:reshape':
+## 
+##     expand, smiths</code></pre>
 <pre class="r"><code># Rename the columns
 colnames(raw_df) &lt;- c(&quot;None&quot;, &quot;Low&quot;, &quot;Medium&quot;,&quot;High&quot;)
 
-# Gather gets all of the columns from the dataframe and 
-# use them as keys, with the value as the value from that cell.
-# This gives us a &quot;long&quot; dataframe
-df &lt;- gather(raw_df, dose, weeks_alive)
+# Here's a version using &quot;melt&quot;
+d &lt;- melt(raw_df, na.rm=TRUE)</code></pre>
+<pre><code>## Using  as id variables</code></pre>
+<pre class="r"><code>names(d) &lt;- c(&quot;dose&quot;, &quot;weeks_alive&quot;)
 
-# We want to treat dose as a factor, so we can group by it easier
-df$dose &lt;- as.factor(df$dose)
+# &quot;gather&quot; similarly takes all of the columns from the dataframe and uses them as keys, with the value as the value from that # cell. This gives us a &quot;long&quot; dataframe
+df &lt;- gather(raw_df, dose, weeks_alive)
+# We want to treat dose as a factor, so we can group by it easier. Specifying the levels helps with labels later on.
+df$dose &lt;- factor(df$dose, levels=c(&quot;None&quot;, &quot;Low&quot;, &quot;Medium&quot;, &quot;High&quot;))
+# Finally, remove NAs
+df &lt;- df[complete.cases(df),]
 
-# Finally, when we look at it, we see some missing data (NA); this is because not all group sizes were the same.
-# We can safely remove these.
-df &lt;- df[complete.cases(df),]</code></pre>
-<p>PC2: Now we’re goint to get statistics and create some visualizations</p>
-<pre class="r"><code>tapply(df$weeks_alive, df$dose, summary)</code></pre>
-<pre><code>## $High
+# Uncomment this to see that the results are the same:
+## df==d</code></pre>
+</div>
+<div id="pc2" class="section level3">
+<h3>PC2</h3>
+<p>Now we’re going to calculate summary statistics and create some visualizations</p>
+<pre class="r"><code># First, using the tapply function
+tapply(df$weeks_alive, df$dose, summary)</code></pre>
+<pre><code>## $None
 ##    Min. 1st Qu.  Median    Mean 3rd Qu.    Max. 
-##   34.00   45.00   56.50   65.25   92.75  102.00 
+##   70.00   85.00   93.00   91.36  101.00  103.00 
 ## 
 ## $Low
 ##    Min. 1st Qu.  Median    Mean 3rd Qu.    Max. 
@@ -364,81 +261,100 @@ df &lt;- df[complete.cases(df),]</code></pre>
 ##    Min. 1st Qu.  Median    Mean 3rd Qu.    Max. 
 ##   30.00   58.25   79.50   71.50   89.25   97.00 
 ## 
-## $None
+## $High
 ##    Min. 1st Qu.  Median    Mean 3rd Qu.    Max. 
-##   70.00   85.00   93.00   91.36  101.00  103.00</code></pre>
-<pre class="r"><code># Alternative way to do this using tidyverse
-
-df %&gt;% group_by(dose) %&gt;% summarize_all(c('min','max','mean', 'IQR'))</code></pre>
+##   34.00   45.00   56.50   65.25   92.75  102.00</code></pre>
+<pre class="r"><code># Alternative way to do this using tidyverse &quot;group_by&quot; function:
+library(dplyr)</code></pre>
+<pre><code>## 
+## Attaching package: 'dplyr'</code></pre>
+<pre><code>## The following object is masked from 'package:reshape':
+## 
+##     rename</code></pre>
+<pre><code>## The following objects are masked from 'package:stats':
+## 
+##     filter, lag</code></pre>
+<pre><code>## The following objects are masked from 'package:base':
+## 
+##     intersect, setdiff, setequal, union</code></pre>
+<pre class="r"><code>df %&gt;% group_by(dose) %&gt;% summarize_all(c('min','max','mean', 'IQR'))</code></pre>
 <pre><code>## # A tibble: 4 x 5
 ##   dose     min   max  mean   IQR
 ##   &lt;fct&gt;  &lt;dbl&gt; &lt;dbl&gt; &lt;dbl&gt; &lt;dbl&gt;
-## 1 High      34   102  65.2  47.8
+## 1 None      70   103  91.4  16  
 ## 2 Low       49    89  69.9  14  
 ## 3 Medium    30    97  71.5  31  
-## 4 None      70   103  91.4  16</code></pre>
+## 4 High      34   102  65.2  47.8</code></pre>
 <p>When it comes to visualizations, we definitely want to use ggplot. We have lots of options for what to do.</p>
+<pre class="r"><code>library(ggplot2)</code></pre>
+<pre><code>## Registered S3 methods overwritten by 'ggplot2':
+##   method         from 
+##   [.quosures     rlang
+##   c.quosures     rlang
+##   print.quosures rlang</code></pre>
 <pre class="r"><code># Histograms
 
-h_plot = df %&gt;% ggplot(aes(x=weeks_alive, # What to summarize
-                      fill = dose # How to group by color
-                      )) + geom_histogram(position = 'dodge', bins = 5)
+h_plot &lt;- ggplot(data=df, aes(x=weeks_alive, fill = dose)) + geom_histogram(position = 'dodge', bins = 5)
 h_plot</code></pre>
-<p><img src="data:image/png;base64,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" width="672" /></p>
+<p><img src="data:image/png;base64,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" width="672" /></p>
 <pre class="r"><code># In this case, faceted histograms is probably better
-
-h_facet = df %&gt;% ggplot(aes(x=weeks_alive # What to summarize
-                      )) + geom_histogram(bins = 5) + facet_grid(~dose)
-
+h_facet &lt;- ggplot(data=df, aes(x=weeks_alive)) + geom_histogram(bins = 5) + facet_grid(~dose)
 h_facet</code></pre>
-<p><img src="data:image/png;base64,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" width="672" /></p>
-<pre class="r"><code># Density plots
-d_plot = df %&gt;% ggplot(aes(x=weeks_alive, # What to summarize
-                      fill = dose # How to group by color
-                      )) + geom_density(alpha = .2)
+<p><img src="data:image/png;base64,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" width="672" /></p>
+<pre class="r"><code># Even easier to compare if we flip the coordinates (rotate the axes)
+h_facet+coord_flip()</code></pre>
+<p><img src="data:image/png;base64,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" width="672" /></p>
+<pre class="r"><code># Density plots. Note that setting a value &lt;1 for &quot;alpha&quot; makes the fill more transparent
+d_plot &lt;- ggplot(data=df, aes(x=weeks_alive, fill = dose)) + geom_density(alpha = .2)
 d_plot</code></pre>
-<p><img src="data:image/png;base64,iVBORw0KGgoAAAANSUhEUgAABUAAAAPACAIAAAB7BESOAAAACXBIWXMAAB2HAAAdhwGP5fFlAAAgAElEQVR4nOzdZ2CTVd/H8X9GV+gulL1lIypLkL03LVDAsvcGFZXbrSggisqytJQl07JlCLJURFSogAqCyBIBEbAt0NKRNMnz4uLu462IjDQn4/t5dVqa9NeQXs0v57rO0dntdgEAAAAAAK5NrzoAAAAAAAD4dxR4AAAAAADcAAUeAAAAAAA3QIEHAAAAAMANUOABAAAAAHADFHgAAAAAANwABR4AAAAAADdAgQcAAAAAwA1Q4AEAAAAAcAMUeAAAAAAA3AAFHgAAAAAAN0CBBwAAAADADVDgAQAAAABwAxR4AAAAAADcgFF1AI+SkpJit9tVpxCTyRQQEJCSkqI6iNcJDQ01Go3iMs8ErxIUFKTX669du6Y6iNeJiIjQ6XQ2my01NVV1Fq8TGhqam5ubkZGhOoh30ev14eHhImKxWDjmOF9ERERmZmZWVpbqIN7Fx8cnJCRERLKzs73zmFOwYEHVEYCbmIEHAAAAAMANUOABAAAAAHADFHgAAAAAANwABR4AAAAAADdAgQcAAAAAwA1Q4AEAAAAAcAMUeAAAAAAA3AAFHgAAAAAAN0CBBwAAAADADVDgAQAAAABwAxR4AAAAAADcAAUeAAAAAAA3QIEHAAAAAMANUOABAAAAAHADFHgAAAAAANwABR4AAAAAADdAgQcAAAAAwA1Q4AEAAAAAcAMUeAAAAAAA3AAFHgAAAAAAN0CBBwAAAADADVDgAQAAAABwAxR4AAAAAADcAAUeAAAAAAA3oLPb7aozeA6r1arXu8R7Ijod/7MK6HQ6bcCD73zag88j73w87RXiaa8KT3uFeIWjipc/7fN+fEA5o+oAHiU7O1t1BBERHx8fHx+fzMxM1UG8TkBAgPYOTlZWlnf+eVPIz89Pp9O5yO+gVzGZTNrraY45zhcQEGCz2XJyclQH8S46nc5kMomI1WrlmON8BQoUsFgsFotFdRDvYjAY/P39RSQ3N9c7jzkFChRQHQG4iQLvSNnZ2a5Q23Q6nY+PT1ZWluogXsfPz48Cr4rRaNTr9TztnU9rMna7nQff+fz8/HJzc3nknUyv12tPe5vNxoPvfCaTyWKx8Mg7mY+PT16B984HnwIP1+ES53sDAAAAAIDbo8ADAAAAAOAGKPAAAAAAALgBCjwAAAAAAG6AAg8AAAAAgBugwAMAAAAA4AYo8AAAAAAAuAEKPAAAAAAAboACDwAAAACAGzCqDgAAAADgvpw7d27//v0//PDDsWPHfv311/T0dLPZHBISEhQUVKxYsWrVqjVs2LBevXq+vr6qkwK4LxR4AAAAwC2lpaWtWrVq9erV33///d//9erVqyJy5MiR7du3T58+PTAwsGXLln369GncuLFOp3N6WAAOQIEHAAAA3MyZM2dmzpy5du3a7Ozsv/9rYGCQQW/IzsnKycnJ+2RGRsZHH3300UcfValS5bnnnmvfvr0T8wJwDAo8AAAA4DZSUlImTZqUlJSUm5ub98nSJcu2aNy65kN1qlaqXrZ0OaPRR/v8jcwbv57/Zf+Br3fv3bXn68+zc7JF5NixY/37969bt+5bb71VvXp1NT8GgHtCgQcAAADcgM1mW7x48ZQpU7Rz40XEz8+vc9uuvXsMrPVQnVvepICpQJWK1apUrNY/dkhmVua6TSsXLI0/eeaEiOzfv79Vq1bDhw9/8cUXfXx8nPdjALgPrEIPAAAAuLrTp0937tx5woQJWnv39/Mf2m/U19t+eG/ynH9q739hCjD16TFw54av33ljdpHIoiKSm5sbFxcXFRV18eLF/E0PwEEo8AAAAIBL27x5c8uWLfft26d92Kpp2083fvPKhMmFCkbe7V0Z9IaeXfp8tmn/0P6jDXqDiCQnJ7ds2fLAgQMODg0gH1DgAQAAABdls9kmTZo0aNCg9PR0ESlUMDLhvQ8Wvv9hyeKl7+duAwsEvvLspKVz10SEFRSRy5cvR0VFbdy40TGhAeQbCjwAAADgijIyMvr27Ttz5ky73S4ijR9rvnP9Vx1aRznq/hvVb7pl9efVq9QQkZycnGHDhq1evdpRdw4gP1DgAQAAAJejzYpv375dRHQ63RMjnl2asDo8LMKx36VYkeJrl2xt3by9iFit1jFjxtDhAVdGgQcAAABcy7lz59q3b//DDz+ISIB/QPy7i54Z84Jeny8v3U0BprnTF3dq21VEbDbbuHHjtm3blh/fCMD9o8ADAAAALuTMmTOdO3c+e/asiESEFVy5cKMDT5u/JaPBOPvtRK3D5+bmDh48OG/BPAAuhQIPAAAAuIpffvklOjr6/PnzIlKsSPF1S7c+UqO2E76vQW+YNTWhWaOWIpKTk9OvX7/Tp0874fsCuCsUeAAAAMAlnD17Njo6+rfffhOREsVLrVn8cbkyDzjtuxuNPnOnL3n4wVoikpqaGhsbe/36dad9dwB3ggIPAAAAqPfHH3/ExMRcuHBBREoUL7V60eb73CvuHgT4Byx6f0WJ4qVE5PTp00888YS2AD4AF0GBBwAAABS7ceNGbGzsL7/8IiLFi5ZYvXBTiWIllSQpGBH5wftJAf4BIrJ58+Z58+YpiQHglijwAAAAgEpWq3Xo0KHfffediISFhi+ft06bA1elUoUqk19+VxtPnDjx+++/VxgGwJ9R4AEAAACVXnzxxR07doiIv5//orik8mUqqE4k3aNie3TpLSJms3nIkCE3btxQnQiACAUeAAAAUGjBggULFiwQEb1eP3Pq3FoP1VGd6KY3XnhbeyvhxIkTTz31lOo4AEQo8AAAAIAqX3755csvv6yNXxj/WvtWndXm+TNTgGn22/N8fHxFZP78+bt27VKdCAAFHgAAAFDh/PnzgwcPtlgsItKzS5/hA8aqTvRXD1Z96MkRz4qI3W4fOnQoJ9IDylHgAQAAAGczm82DBw9OTU0VkVoP1Zny8juqE93aqMFPVKtSQ0TOnDnz+uuvq44DeDsKPAAAAOBsL7/88sGDB0WkUMHIxBlLfX39VCe6NaPRZ/rkOKPRKCJz5sw5evSo6kSAV6PAAwAAAE61fv36hQsXiojRYJzzzsLIQoVVJ7qdB6s+PHzAGBHJzc195pln7Ha76kSA96LAAwAAAM5z6tSp8ePHa+MJT7xUr3YDtXnuxItPTyxauJiIJCcnr1y5UnUcwHtR4AEAAAAnMZvNQ4cOzcjIEJEWjVuPGDhOdaI7EhQUPPW16dp40qRJWn4AzkeBBwAAAJxk6tSphw8fFpFiRYpPnxKv0+lUJ7pTXTv1aFCvsYhcunRp+vTpquMAXooCDwAAADjD3r174+LiRESv1894MyEsNFx1orsz5eV3DXqDiCQkJPz666+q4wDeiAIPAAAA5Lv09PSxY8fabDYRGT5gbP06DVUnumvVq9SIjeknImaz+a233lIdB/BGFHgAAAAg373wwgvnzp0TkWqVH3x27Auq49yjp0b+xxRgEpE1a9YcO3ZMdRzA61DgAQAAgPy1devWpKQkEfH19Zv55lwfH1/Vie5RZKHCg/oMFxGbzfbmm2+qjgN4HQo8AAAAkI/S0tKeffZZbTxh3IuVKlRRm+c+jRg4LiQ4VEQ++eSTQ4cOqY4DeBcKPAAAAJCPXnrppUuXLolInUceHdpvtOo49yskOFTb/c5ut0+dOlV1HMC7UOABAACA/LJz585Vq1aJiL+f/7Q3Zuv1nvDye2DvYRFhBUXk008/3b9/v+o4gBfxhCMIAAAA4IJu3LiRd/L8M2NfKF+mgto8jlLAVGDU4Ce0McvRA85EgQcAAADyxeTJk8+fPy8iNao9MqTfKNVxHKlf7JBCBSNF5Isvvjh48KDqOIC3oMADAAAAjnfw4MEFCxaIiNFgnPb6LIPeoDqRI/n7+Q/rf/N6/pkzZ6oNA3gPCjwAAADgYLm5uePHj7fZbCIyfOCYqpWqq07keH16DNKWo9+6detPP/2kOg7gFSjwAAAAgIMlJCT8+OOPIlK6ZNknR/5HdZx8EVggsH/sEBGx2+1xcXGq4wBegQIPAAAAONKFCxemTZumjd985V1/P3+1efLPoD7DtZ9u3bp1Fy9eVB0H8HwUeAAAAMCRXnnllczMTBGJ7hDTqH4z1XHyUURYwe7RvUTEbDbPnz9fdRzA81HgAQAAAIfZvXv3xo0bRSQwMOilp99QHSffDe03StvcfvHixRkZGarjAB6OAg8AAAA4hsVieeGFF7Tx+JH/KRxZRG0eJyhbunyrZu1E5Nq1ax9++KHqOICHo8ADAAAAjpGYmPjzzz+LSMUHKg/sPUx1HCcZ3n+MNpg3b5628D6AfEKBBwAAABzg999/f+edd7Tx68+/ZTT6qM3jNHVq1nuw6sMicubMmR07dqiOA3gyCjwAAADgAG+88YZ2EXjHNtENHm2sOo5TDe47QhskJiaqTQJ4Ngo8AAAAcL8OHDiwevVqEQnwD3jpGc9fu+4vOrftUqhgpIjs2bPnp59+Uh0H8FgUeAAAAOC+2O32l156yW63i8iIQeOKFy2hOpGz+fj49u0xSETsdvvixYtVxwE8FgUeAAAAuC/r16//9ttvRaRYkeIjBz2hOo4asTH9jAajiKxevTorK0t1HMAzUeABAACAe5eTkzNp0iRt/PxTrwX4B6jNo0qRyKItm7YVkWvXrq1fv151HMAzUeABAACAexcfH3/u3DkRqflQ7aj23VTHUalPjwHagLPogXxCgQcAAADu0R9//DFr1iwR0el0Lz87WafTqU6kUqP6zUqVKCMiBw8ePHz4sOo4gAeiwAMAAAD36K233kpPTxeRjq2jaz9cV3UcxfR6fWy3ftp46dKlasMAHokCDwAAANyLEydOLFu2TER8fHz/8+TLquO4hJ5dehuNPiKydu3azMxM1XEAT0OBBwAAAO7Fa6+9lpubKyIDew8rXbKs6jguoVDByJZN2ojI9evXN27cqDoO4Gko8AAAAMBd+/rrr7dv3y4ioSFh44Y9rTqOC+EseiD/UOABAACAu/bGG29og7FDx4cEh6oN41KaNGxerEhxEUlOTj59+rTqOIBHocADAAAAd2fLli3JyckiUrxoif6xQ1THcS0GvaFb58dFxG63JyUlqY4DeBQKPAAAAHAXrFbrlClTtPHTo5/38/NXm8cFdY+K1XbUW716tc1mUx0H8BwUeAAAAOAurFq16vjx4yJS8YHKXTv3VB3HFZUtXb7WQ3VE5Pz5819++aXqOIDnoMADAAAAdyonJ+ftt9/Wxs898YpBb1Cbx2XFRMVqg5UrV6pNAngSCjwAAABwpxYtWnT+/HkRqf1w3VbN2qmO47o6te3i5+cnIh9//DEbwgOOQoEHAAAA7khGRsaMGTO08XNPvqo2jIsLDgpp2aStiNy4cWPLli2q4wAeggIPAAAA3JGEhISUlBQRadKgxaO1H1Mdx9V16dBdG6xZs0ZtEsBjUOABAACAf5eWlhYfHy8iOp1uwrgXVcdxA80btwoLDReR3bt3X7lyRXUcwBNQ4AEAAIB/N2fOnOvXr4tI2xYda1R7RHUcN+Dj49uxTbSI5Obmrl+/XnUcwBNQ4AEAAIB/kZKSMm/ePBHR6/XPjHlBdRy30aVjD22wbt06tUkAz0CBBwAAAP7FzJkzb9y4ISJR7btVfKCy6jhuo/bDdUsULyUiBw4cOH36tOo4gNujwAMAAAC3c+nSpUWLFomI0WB8csQE1XHciU6n69y2qzbesGGD2jCAB6DAAwAAALczY8aM7OxsEenSqUe5Mg+ojuNmotp30wacRQ/cPwo8AAAA8I9+++23pUuXiojR6PPkiGdVx3E/VStVr1C+koj89NNPx44dUx0HcG8UeAAAAOAfzZgxIycnR0R6RPcqVaKM6jhuKardzUl4zqIH7hMFHgAAALi1CxcuLF++XER8fHzHDntadRx31aFNtDbYuHGj2iSAu6PAAwAAALc2c+ZMs9ksIo936V2iWEnVcdzVA2UrVKpQRUROnDjBWfTA/aDAAwAAALeQN/1uNPqMGvyk6jjurWPrm5PwmzZtUpsEcGsUeAAAAOAWZs2adXP6vWsfbTNz3LMOraO0AWfRA/fDqDoAAAC4R7m5uUePHj1+/Pjx48dPnz59/vz5tLS0tLS0zMzMrKwsEQkICAgICAgLCwsPDy9ZsmTp0qUrVapUpUqVihUrGo28BgBu5+LFi8uWLRMRHx/f0UOeUh3H7VUoX6lC+UonTh0/fvz4iRMnKlSooDoR4Jb44w0AgDvJzc1NTk7es2fPN998c+DAgczMzNt8cVZWVlZWVmpq6qlTp5KTk/M+HxAQUKtWrUaNGjVp0uThhx82GAz5HxxwM3nT7z2ie3H1u0N0aB01I/5tEdm8efNTT/GeCHAvdHa7XXUGz5GSkuIKj6fJZAoICEhJSVEdxOuEhoZqM1ou8kzwKkFBQXq9/tq1a6qDeJ2IiAidTmez2VJTU1Vn8XBms/nTTz9dt27dzp0709PTb/OVwSFBJpO/0ccoIjarLSMjM/16xm0OSuHh4R06dIiJialfv75Op3N8dM+i1+vDw8NFxGKxcMxxvoiIiLwTTPLV5cuXa9WqlZ2dbTT6fPFxcsnipfP7O7oyo9EYGBgoImaz+fZvGt7esZ9/bN21oYg8+OCDn376qcPy5b+CBQuqjgDcxAw8AAAu7ejRo0uWLFm/fv3f3yUJDQ2uVqNi5aoPlCtfqkzZEiVKFQ0LD9Xrb1HCU1Ounvv14tkz50+dPHvsx5NHfjh+/drNdwFSU1OXLl26dOnSUqVK9erVq2/fvpGRkfn+UwGu7f3338/OzhaRbp16enl7d6AqFauVLV3+zNlThw8fPnv2bOnSPLDAXaPAAwDgimw227Zt2+bMmfPNN9/8+fN+/n516z3UoFHtZq0aPlCh9B1ORYZHhIZHhD70SJX/3rn9+LFT3+w9+OXu5APJh3Nzc0Xk119/nTp16nvvvRcdHT1u3LhKlSo5/IcC3EJKSsrixYtFxGgwjh02XnUcj9KuZac5C2aIyObNm0ePHq06DuB+KPAAALiW3NzctWvXzpw588SJE3mf1Bv0DRrV7hDVonmrx0ymABEJCgqyWq339i30el2Vag9UqfbAwGE90tKubft494a12w9//5OImM3mVatWrVmzpmPHjs8++2zlypUd8kMBbiQhIUE7UTy6Q0zpkmVVx/Eo7VtR4IH7wjXwjuQiVz5zDbwqXAOvENfAq8I18A63efPmN9544/Tp03mfKVykUM/eHbt0b1coMvzPX6kV+Pu5HvUvThw/8+HSDZs+2pmVma19Rq/X9+jR46WXXipcuLCjvou74xp4tZxwDfzVq1dr1qyZnp6u1+t3bfjmgbIslu6wa+BFxG63129d48LF83q9/ocffnCXYwvXwMN1sA88AAAu4fvvv+/YsePAgQPz2nuFSmWnvvfcti+WDhvd+y/tPT9UqFT2lUlPbv9i+bDRvQMDTSJis9mSkpLq1as3Z84c7TR7wOPNmzdPWyeyQ+to2rvD6XS6Vs3aiYjNZvvkk09UxwHcDwUeAADFrl69+swzz7Ru3Xrfvn3aZypVKT9r7uvrtiR2jG7p5A3bw8JDxj09cNsXy4eOjPXz9xORjIyMV199tXnz5gcPHnRmEsD5MjIy5s2bJyI6nY6r3/NJ62bttcHWrVvVJgHcEQUeAACVNm7c+Nhjjy1evNhms4lIZJGCk95+dvWmhOatHlO4qVtIaNATzw7evHNRm/ZNtM8cO3asffv2r7/+urYzNuCRFi5cmJaWJiKtmrWrUrGa6jieqX6dBiHBoSKyZ8+e22+KCeDvKPAAAKiRmpo6ePDgwYMHX7lyRUR8fX2Gje798c4PomPa3HIrOOcrWizy3fdfTlw8tVTp4iJitVpnz57dtm3bP6+uB3iM7OzshIQEbTx2KNPv+cVo9GnWqKWImM3mzz77THUcwM1Q4AEAUGDnzp2NGjXauHGj9mHtR2us25I47umBASZ/tcH+7rFGtddtSew/OEZv0IvI4cOHW7ZsmZSUpDoX4GDLli3T3k1rVL/Zww/WUh3Hk3EWPXDPKPAAADhVbm7u66+/3qtXr8uXL4tIgMn/xYljF614t0y5kqqj/SP/AL9nXxyxcPk7RYtFikhmZubYsWOfeOIJTqeHx7BYLHFxcdqYq9/zW7OGLX18fEVk165dLJAJ3BUKPAAAznP58uUuXbrMnj1b22zy4ZrV1n48N7ZvlMLL3e9c7bo11n48t2WbhtqHK1asiI6OvnTpktpUgEOsXr36/PnzIvJIjdr16zRUHcfDBQYG1avdQETS0tKSk5NVxwHcCQUeAAAnSU5Obt68+TfffCMier1u+JjeS1ZO1y4vdxfBIUEz4l97/pXR2tr4ycnJbdq0+fHHH1XnAu6LzWabPXu2Nh43/Bm1YbxEq6ZttcG2bdvUJgHcCwUeAABnSEpKypuvDg0LnrNgytjxA7Wryt1O7wFd5i15KywsREQuXLjQoUMHVqKCW/v4449PnjwpIlUrVW/RuLXqOF6BAg/cG7d83QAAgBux2+2TJ08eO3asdsV41eoVVm6Y07BJHdW57kudeg+tWD+7bPmSInLjxo3evXuvWrVKdSjgHs2aNUsbjBn6lFtcz+IBShQvVblCVRE5efLkqVOnVMcB3AYFHgCAfGQ2m0eOHDljxgztw7Ydmy5dNaN4iSJqUzlEyVLFVqydXbf+wyJisVjGjBkzf/581aGAu/b5559/9913IlKmVLn2raNUx/EieZPw27dvV5sEcCNG1QHuyOXLl5OSkg4dOnT9+vWwsLDatWvHxsaGhIQ48LYHDhzYtGnT+fPnr1+/XqRIkYoVK/bs2bNQoUL58NMAALxFenr6gAEDvvjiC+3DkeP6jnqinyfN7wUFByYsevP5p6du+3i33W5//vnnMzMzx40bpzoXcBdmzpypDUYMHGvQG9SG8SotmradPe89EdmxY8fIkSNVxwHcg05bBdeVnTlzRntBICKhoaFXr14VkbCwsGnTpkVGRjrktgsWLNiwYYOImEymkJCQy5cvW61WX1/fiRMnVqtW7c6jpqSkuMLjaTKZAgICUlJSVAfxOqGhodqqTi7yTPAqQUFBer3+2rVrqoN4nYiICJ1OZ7PZUlNTVWdxOampqd27d//hhx9ExGg0vjrlyS4xbR14/0FBQVarVfsbp5bNZp/0ysxVKzZrHz7//PPjx3vsLlx6vT48PFxELBYLxxzni4iIyMzMzMrKctQdHjhwoG3btiISWajw19u+9/X1c9Q9exKj0RgYGCgiZrPZgcccq81as0ml1LQUHx+f48ePBwUFOeqeHa5gwYKqIwA3ucEp9LNnz87MzKxdu/aSJUuWLFmyYMGCChUqpKWlJSYmOuS2P/3008aNG/39/V988cWkpKS5c+cuX768cePGZrN5+vTpVqs1P384AIBnunLlSnR0tNbeA0z+sxInOra9uxS9XvfyG0/0Hxyjffjmm2++++67aiMBdyjv6vchfUfS3p3MoDc0a9hSRCwWy+7du1XHAdyDqxf4n3/++eTJkxEREc8991xoaKiIFCpU6NVXX/X19f32229vP8l8h7fdsWOH3W6Piop69NFHtc+YTKZx48YFBgZevnz53Llz+fwjAgA8zaVLl6Kioo4dOyYiIaFBC5ZNa9z0UdWh8pdOp3v2xRGDhvfUPpw6dWpCQoLaSMC/OnHixCeffCIigYFBfXoMVB3HG7VocnPNfy6DB+6Qqxf4nTt3ishjjz3m6+ub98ng4OBatWrZbLbbb1pzh7f99ddfReTBBx/88219fX2LFy8uIhcuXHDYDwMA8AKXLl3q0qXLiRMnRCQsPGTBsndqPFxFdSgnGf+foYNHPK6NX3nllRUrVqjNA9ze7NmzbTabiAyMHRoUGKw6jjdq0qCF0WAUkV27dmn/FwBuz9ULvNafH3roob98vkaNGiJy/vz5+79t//79X3vttYoVK/75a6xW65UrV0SEdewAAHfujz/+6Nq1q9bewyNCF654t3LV8qpDOdVTE4bE9o0SEbvdPn78+C1btqhOBNzaxYsX165dKyL+fv6D+gxXHcdLBQeF1H7kURG5fPny999/rzoO4AZcfRV6bVWkvy8ar50Sn5aWdv+3rV69et4/Wa3WjIyM8+fPb9iwITU1tUaNGn8p9prjx49PmjTp75+fPXu2tsKHWnq9XqfTaT8mnMlguLl07R1ukQAH0h58nvbOpy2ortfrefBFJC0trWfPnj///LOIFCwU/uG6uIqVy+Xft9OO9i647NOUd57LzjavX73VarUOHz5869atDRs2VB3K8YxGI09759PpdAEBAX5+DrhYfcqUKWazWUR69xhQtox3vdF2t/L2zjAajQ4/5rRr1fGbb/eKyFdffdWsWTPH3jngeVy9wGtXqv/9SBEcHCz/regOvO2TTz559uxZbdymTZvBgwff8p4zMzO1Kxv/QqfTaYuQuwLXSeKFePBV4ZFXiAc/MzMzKipKW7UuLDwkaf2cylUfyO9v6po70hkMhunvv3o17dpnO7/Kzs6OiYnZu3dvpUqVVOdyMJf6o+9V9Hq9Xn+/55CmpaXNnz9fRAwGw7jhT+e9BY/bu/9H/u9aN2//6pvPi8j27dtfffVVh98/4GFc/RR6zd9foGh7dOXm5jr2thUqVKhdu3ZMTEz9+vX37Nnz6aef3mNiAIA3sVgs3bt3//rrr0UkKDhw+Zr3ndDeXZnRx5i4+O1HalUXkZSUlPbt22sXpgEuIj4+Pj09XUSiO8SUK+PVv63KVav8YLEixUVk37597EgK/CtXf+c4IiLiwoUL6enpRYsW/fPntWOuthGrA287bty4vPHs2bMTEhLCw8Pr1av3ly+rUaPGLbu9xWJxhd3XTSaTv78/R0DnCwkJ0WZjUlNT2QfeyQIDA/V6/fXr11UH8Trh4eHaPvC3v6bJs5Yv6p0AACAASURBVNnt9rFjx2oXe/v5+c5OfL102WJO2CE8MDDQarU6cENsh5s1d2Js1zHnf714+vTpzp07r1u37s/LyropvV4fFhYmIhaLhWOO84WHh2dlZd3n0z4nJ2fGjBnaeEjfUU74bXV3RqOxQIECImI2m/PjmNP4sWZJ65ZZrdb169dHR0c7/P7vX0REhOoIwE2uXuDDw8MvXLjw9z+Q2mcKFix4n7dNT08/cOCAyWSqW7fuX76sdu3aO3bs2L59+98LvMFg0M7D/4uUlBRXqG1aBldI4rXsdjuPvxI87Ap584M/adKkpKQkEdEb9G/PfLF23RrOfDRc+ZEPCw9JWDilV7dx16+lf/PNN88++2xea3Jff37AXfnB92D3/0d2xYoV2ikhjeo3q17Fqb+wbiq/n/ZNGrRIWrdMRHbt2hUVFeXw+wc8iaufQl+sWDER+fHHH//y+aNHj4rIX6bW7+G2drv9vffemzZt2t8PRjk5OfcVHQDgBZYtWzZz5kxt/MobT7Zo3UBtHldTplzJd2e/pF1gvHz5cu2qY0Ahq9UaHx+vjUcOGqs2DDSNH2umbSb36aef8n4KcHuuXuBbtmwpInv37v3zzpBmszk5OVmv1zdv3vw+bxscHFyoUKGcnJxDhw795eb79u0TkZIlSzrupwEAeJQvv/xywoQJ2njUE/1iHm+vNo9rqt+w1oSXRmrjV1555csvv1SbB15u69atp0+fFpHqVWo0qs+a5y4hOCjkkRq1ROTSpUvaTBuAf+LqBb5SpUrly5f//fff4+Pj8xafe/vttzMzM+vWrRsZGZn3lR9//PGmTZtOnjx5t7dt166diMTFxeXN1ZvN5uXLl+/du9ff31/7VwAA/uKXX34ZPHiwxWIRkY7RLUeO66s6kevq3T+6S/e2ImKxWIYMGXLx4kXVieC93n//fW0wYuC4238lnKlJgxbagDWkgdvTuf5pKqdPn37++eezsrICAwNLlix55syZ7Ozs8PDwadOmFSpUKO/LYmJizGZz//79u3Xrdle3tVqtL7zwgrYtnMlkCgoKunLlis1m8/X1HTduXOPGje88qotcA28ymQICAlxhOT1vExoaqi1i5yLPBK8SFBSk1+tZiMj5IiIitEXsvG3hzPT09Hbt2h0/flxEHq5ZbcHyaX5+zl6eLSgoyGq1ZmZmOvn73huz2TIwdvz3h46JSN26dT/66CMfHx/Voe6FXq/X1sG1WCwcc5wvIiIiMzPzntdR++qrr7RLrEsUL7VnywHttG38K6PRGBgYKCJmszmfjjnfHT7QKbaliDRq1GjdunX58S3ux+0X3gKcydVn4EWkXLlyM2fObN68uY+Pz4kTJ4KCgjp06DBz5sw/t/f7ua3BYJg6derIkSOrVavm5+d37dq1smXLtmnTJj4+/q7aOwDAS9hstpEjR2rtvWixyJkJrzm/vbsdX1+fd99/JSwsRET2798/ZcoU1YngjeLi4rTB0H6jaO8upUa1R8JCw0Vk37597vK+JKCEG8zAuxEXmXdlBl4VZuAVYgZeFe+cgZ82bdrbb78tIgEm/6WrZlauWl5JDPeagdd8uTt51OAXbDa7TqdbsWKFtlqNe2EGXq37mYE/fvx4o0aN7HZ7WGj4NzsOmwJMDo/nqZwwAy8io58dvHHrOhFZvnx569at8+m73Btm4OE63GAGHgAA17Fjx4533nlHG7/+5tOq2rubatikzpCRvUTEbrePHj2ai+HhTHPmzNHeXu/bcxDt3QU1eezm6tSfffaZ2iSAK6PAAwBwp86dOzd69Ghtc5P+g2PadWIJ67s2+sl+teo8KCKpqal5DyaQ3y5durRmzRoR8fPzG9hrqOo4uIUmDVrodDqhwAO3RYEHAOCOmM3moUOHpqWliUideg+Nf44OcC8MBsPbM18IDQsWkT179uQtCQ7kq/nz55vNZhHp1unxghGR//r1cL7CkUUqPlBZRE6dOnXu3DnVcQAXRYEHAOCOTJw48cCBAyJSsFD4tJkvGgwG1YncVeEihSZOGa+Np06d+v3336vNA49348aNDz74QET0ev3Q/qNVx8E/alSvqTb44osvlAYBXBcFHgCAf7dly5Z58+aJiN6gf3vGCwULhatO5N5atGnYPbaDiFgsltGjR+fk5KhOBE+2bNmyq1evikjLpm0fKFtBdRz8o4b1m2oDCjzwTyjwAAD8iwsXLjzxxBPa8lcjx/atW/9h1Yk8wYQXR5YqXVxEjh8/PnnyZNVx4LGsVqv27puIDO8/Rm0Y3F79Og19fHxF5PPPP2eBDOCWKPAAANyOzWYbPXq0Nn1X77FHho/pozqRhwgw+U95d4LeoBeRuXPn7tu3T3UieKaNGzeePXtWRB5+sFbdWvVVx8HtmAJMNR+qLSKpqalHjx5VHQdwRRR4AABu5/3339+7d6+IhIWHvPne83q9TnUiz/FwzWqDhvYQEZvNNnbsWPfa0x7uIj4+XhsMH8D0uxvIuwx+9+7dSoMALooCDwDAPzp8+PBbb72ljSe++XShSC59d7BRT/avULGMiJw5c+aNN95QHQee5uuvvz506JCIlCpRpl2rTqrj4N/lXQZPgQduiQIPAMCt5eTkjBo1Stt6Kubx9s1bPaY6kQfy9fWZ/M4Eo9EoIgsXLty/f7/qRPAoc+bM0QZD+o406Nk5wg08VP2RoMBgEdm3b592+AXwZxR4AABubfLkyT/99JOIlCxVbMJLI1XH8VhVq1ccNLyniNhstvHjx/OSHY5y8uTJ7du3i0hoSFjPrqxe4R6MBmO9Og1EJDMzU9u5E8CfUeABALiFffv2zZ07V0T0Bv2b7/3HZApQnciTDR/Tu0y5kiJy/PjxGTNmqI4DD5GQkKCtZN6nxwBTgEl1HNyphvWaaAM2kwP+jgIPAMBfZWVljR07VnvpP3hYz4drVlOdyMP5+flOfHO8TqcTkZkzZ544cUJ1Iri9lJSUlStXioivr9/AXsNVx8FdaPgoBR74RxR4AAD+atKkSWfOnBGRChXLjHyin+o4XqFWnQdjHm8vImaz+ZlnnrHb7aoTwb0tXLgwOztbRKI7xEQWKqw6Du5ChfKVtP+ygwcPpqenq44DuBYKPAAA/yM5OXn+/PkiYjAYJr8zwdfXR3Uib/HUhCERBcNE5KuvvtLmToF7k5OTs2jRIhHR6XRD+41SHQd3R6fTNXi0sYjk5uZ+8803quMAroUCDwDA/zObzU899ZR28vzAod2rVq+oOpEXCQ4Jylss8LXXXrt69araPHBfq1evvnLliog0qt+scoWqquPgrj1Wt5E20HYBBJCHAg8AwP+bPn368ePHRaRMuZKcPO98HTo3r9+gpoikpKRMnjxZdRy4JbvdnpCQoI1HDByjNgzuTWhImDawWCxqkwCuhgIPAMBNP/3006xZs0REr9dNfHO8n5+v6kTe6MWJY7XLFpYsWfLdd9+pjgP3s3PnTu1tuEoVqjSs11R1HABwJAo8AAAi/7sJeUxsx1p1HlSdyEuVKVey/5DuImKz2Z5//nlWs8Pdio+P1wbD+4/RtjYAAI9BgQcAQERk6dKlycnJIhJZOGL8hCGq43i14aN7FylaSES+/fZbVrPDXTly5MiePXtEJLJQ4aj23VTHAQAHo8ADAPA/V1y/8NrYwKACavN4Of8Av2eev7lx9xtvvME+UrhzedPvA2KH+vr6qQ0DAA5HgQcAQCZOnJiWliYijZs92rJNQ9VxIG07Nq39aA0RuXz58vTp01XHgXv4/fffP/roIxEJ8A/o3X2A6jgA4HgUeACAt0tOTk5KShIRP3+/F15lzWpX8dzLo/UGvYjMnTv3l19+UR0HbmDBggU3l7GIig0Pi1AdBwAcjwIPAPBqVqt1woQJ2kppQ0fGlihVVHUi3FS5avluPdqJiNlsfvXVV1XHgavLyspasmSJiOh0uoG9h6uOAwD5ggIPAPBqH3zwwZEjR0SkZKliA4f1UB0H/2Ps+IHaegRbtmz5+uuvVceBS1uxYkVqaqqItGzatkK5iqrjAEC+oMADALxXamrq1KlTtfHzr41m43dXEx4ROnx0b2382muvsaUc/onNZluwYIE2HtpvlNowAJB/KPAAAO81ZcqUq1evikjTFvUbN31UdRzcQq/+0cVLFBGRgwcPrl+/XnUcuKhPPvnkxIkTIlKlYrX6dViHEoDHosADALzUkSNHli1bJiK+vj4TXhqhOg5uzc/Pd9wzg7Tx5MmTtSXKgL+YO3euNhg+cKzaJACQryjwAAAv9dJLL1mtVhHpO6hbqdLFVcfBP2rfqVm1ByuKyK+//rpo0SLVceByfvjhh6+++kpECkcW6dy2i+o4AJCPKPAAAG+0ZcuWvXv3ikihyPBho3upjoPb0el0458bqo3fe++969evq80DVxMfH68NBvYa7uPDShYAPBkFHgDgdcxm88SJE7Xx2PEDCxQwqc2Df/Vo/UcaNa0rIqmpqbNnz1YdBy7k4sWLGzduFJEA/4BeMf1UxwGA/EWBBwB4nUWLFp0+fVpEKlctHx3TVnUc3JGnJgzR63UikpiYePnyZdVx4CoWLFigrYzQPbpXWGi46jgAkL8o8AAA73L16tV3331XGz/74gitE8L1VaxcrkNUSxHJzMx87733VMeBS8jMzFyyZImI6PX6wX1YihKA56PAAwC8y7vvvpuWliYiTZrXe7T+I6rj4C6MfqKfj49RRJYuXXr27FnVcaDeihUrtF/nFk3alCvzgOo4AJDvKPAAAC9y9uzZhQsXiojBYHj6uWGq4+DulChVNObxDiJiNpvfeecd1XGgmN1uX7BggTYe2m+U2jAA4BwUeACAF3nzzTdvXi4b26HcA6VUx8FdGz6mt3+An4isXr365MmTquNApe3bt2vPgepVatSv01B1HABwBgo8AMBbHD16dP369SJiMgWMHNdXdRzci4KFwh/vEyUiVqt12rRpquNApYSEBG0whOl3AF6DAg8A8Bavv/66zWYTkf5DYiIKhqmOg3s0eERPbee/jz766NixY6rjQI1jx47t3btXRApHFunctovqOADgJBR4AIBX+Oqrr3bt2iUiYeEh/Yd0Vx0H9y4sLKT3gC4iYrPZ3n77bdVxoEZcXJzdbheR/rFDfXx8VccBACehwAMAvMKkSZO0wdBRvQIDTWrD4D4NGBITGFRARD7++OMjR46ojgNnu3TpknY5TIB/QJ/uA1THAQDnocADADzftm3bkpOTRaRoscievTupjoP7FRwS1G9QNxGx2+1MwnuhBQsWaKtRduv8eFhouOo4AOA8FHgAgIez2WxTpkzRxiPG9vHz42xbT9B3YNfgkCAR+eSTT5iE9ypZWVmLFy8WEZ1ON6jPCNVxAMCpKPAAAA+3adOmo0ePikiZciWjY9qojgPHCAoO7Duwq4jY7fb33ntPdRw4z/Lly1NTU0WkeaNWFcpVVB0HAJyKAg8A8GRWqzXvFOvRT/QzGAxq88CB+g7sGhQcKCKbN29mOXovYbfb4+PjtfHgviPVhgEA56PAAwA82Zo1a37++WcRqVCpbJsOTVXHgSMFBhXo1S9KROx2+4wZM1THgTN88sknx48fF5HKFao2rNdEdRwAcDYKPADAY+Xm5r777rvaeMyT/fV6ndo8cLi+A7uZTAEismHDhtOnT6uOg3w3ffp0bTCs/2idjt9oAF6HAg8A8Fhr1qw5c+aMiFStXqF56waq48DxQsOCe/bpJCJWq3XmzJmq4yB/HTt2bNeuXSJSMLxQ53ZdVccBAAUo8AAAz2S1WvNOqx45ri+TdZ5qwJDufv5+IrJ69epz586pjoN8lJCQYLfbRaR/7BA/P3/VcQBAAQo8AMAzrV69+tSpUyJStXqFpi3qq46D/BJRMKxbj3YiYrFY4uLiVMdBfvnjjz/Wrl0rIr6+fn16DFAdBwDUoMADADxQbm5u3tZiTL97vIHDevj4GEVk+fLlV65cUR0H+WLhwoU5OTki0iM6tmBEpOo4AKAGBR4A4IHWrl2rXf1euWp5pt89XtFikR2iWohIdnZ2YmKi6jhwPLPZvHjxYm08pO8otWEAQCEKPADA0/zP1e9jmX73CkNGPK7tMrBgwYJr166pjgMHW7t27eXLl0WkWaNWVStXVx0HAJShwAMAPM2GDRtOnjwpIhUrl2PxeS9RplzJFq0bikh6enreVC08g91uj4+P18Zjhj6pNgwAqEWBBwB4FLvdnjf9PnxMb6bfvceQkbHaIDEx0Ww2qw0DB9q9e/exY8dEpHyZCi2atFEdBwBUosADADzKtm3btNf6ZcuXbNW2seo4cJ5qD1Z8tP4jInLp0qVVq1apjgOHSUhI0AbDBozW63ntCsCrcRAEAHiUWbNmaYMhI2K1i6LhPQYO66EN4uLibDab2jBwiBMnTnz66aciEh4W0bVTT9VxAEAxCjwAwHN8+eWXycnJIlKseOEOUc1Vx4GzNWhcu2LlciJy8uTJbdu2qY4DB0hMTLTb7SLSp8dAfz9/1XEAQDEKPADAc+RNvw8a3tNoNKoNA+fT6XSD/jQJrzYM7l9aWpp2NYSPj2//x4eojgMA6lHgAQAe4siRI59//rmIhEeEdolhpSsv1bZj06LFIkVk37592ukYcF8ffPBBZmamiES37xZZqLDqOACgHgUeAOAhpk+frp1q23dQNz9/P9VxoIbRaOwzsKs2njNnjtowuB8Wi+WDDz7QxoP7jlSaBQBcBQUeAOAJTp06tXnzZhEJDCoQ26ez6jhQKebx9kHBgSKyZcuWM2fOqI6De7Ru3brffvtNRBo82rha5QdVxwEAl0CBBwB4gjlz5mirjvfo1TEwqIDqOFCpQAFTzOPtRcRmsyUmJqqOg3s0d+5cbTCk3yi1SQDAdVDgAQBu78qVKytXrhQRX1+fPgO6qo4D9foO6KqtYrh8+fLU1FTVcXDX9u7de/jwYREpW7p880atVMcBAFdBgQcAuL3ExMScnBwR6RjdMrJwhOo4UC+ySMHW7RqLSFZW1vLly1XHwV1LSEjQBoP7jNDreb0KADdxQAQAuLcbN25oK13p9bqBQ3uojgNX0W9wN22wYMGC3NxctWFwV06fPr19+3YRCQkO7R7dS3UcAHAhFHgAgHv78MMPr169KiJNmtcvW76k6jhwFdVrVHqkVjURuXDhgrbAIdzF3LlztSUt+vQYYAowqY4DAC6EAg8AcGNWqzVvpasBQ7urDQNX03fQzUn4uLg4tUlw565evZqUlCQiRqNPv8eHqI4DAK6FAg8AcGMff/zxL7/8IiLVa1SqVYeNpvA/WrZuWKx4YRH57rvvvv32W9VxcEcWL16cmZkpIp3aRhcrUlx1HABwLRR4AIAbi4+P1wYDhjD9jr/SG/S9+kdrY/aTcwsWi2XBggXaeCi7xwHA31DgAQDuKjk5WZtWLVa8cKt2jVTHgSvq1qOdyRQgIps3b7548aLqOPgX69ev1/6b6tVu8GDVh1XHAQCXQ4EHALirOXPmaIM+A7saDAa1YeCagoIDo7q1FhGLxbJo0SLVcfAv5s+frw2YfgeAW6LAAwDc0tmzZ7du3SoigYGmrj3aqY4D19W7f7ROpxORJUuW5OTkqI6Df/TVV18dOnRIRMqUKteyaVvVcQDAFVHgAQBuaf78+VarVUS69WwfGMhGU/hHZcqVbNC4toikpKSsXbtWdRz8o4SEBG0wuM8IvZ7XqABwCxwcAQDuJyMjY8WKFSKiN+h7D+iiOg5cXe/+N58keWdow9WcPn1627ZtIhISHNo9upfqOADgoijwAAD38+GHH16/fl3+tE8YcBsNm9QpU7aEiBw+fHj//v2q4+AW5s6da7PZRKRPjwEFTAVUxwEAF0WBBwC4GbvdvnDhQm3cZ2BXtWHgFnQ6XY/enbRx3i5lcB1paWlJSUkiYjT69I8dqjoOALguCjwAwM3s3Lnz5MmTIlK1eoWataurjgP30CWmTYDJX0Q2b9586dIl1XHwP5YsWZKZmSkiHdtEFS1cTHUcAHBdFHgAgJuZN2+eNujVL1ptEriRoODATtEtRcRsNi9dulR1HPw/i8WSd07NkL4j1YYBABdHgQcAuJNTp059/vnnIhIWHtKuUzPVceBOevfvou0n98EHH1gsFtVxcNO6det+++03Ealfp+FD1WuqjgMALo0CDwBwJ4mJiXa7XUR69u7k5+erOg7cSfkKpWvVeVBELl26tHXrVtVxcFPe7nFD+41SmwQAXB8FHgDgNq5fv75y5UoRMRqNPWI7qo4D9/N43yhtkHfONtT68ssvjxw5IiLlyjzQokkb1XEAwNVR4AEAbmPlypU3btwQkZZtG0YWKag6DtxPyzYNIgtHiMjevXt/+ukn1XEg8fHx2mBI35F6Pa9LAeBfcKAEALgHu92etwFY7H/nUYG7YjQaYx7voI0XL16sNgxOnjy5c+dOEQkNCevW+XHVcQDADVDgAQDu4bPPPjt16pSIVKpSXruSGbgH3WM7Go1GEUlKSsrIyFAdx6slJCTYbDYR6dtzkCnApDoOALgBCjwAwD3kXbTM9DvuR6HI8KYt6olIRkbGunXrVMfxXikpKdqSFr6+fgNih6qOAwDugQIPAHAD586d0061DQ4J6hjVQnUcuLeefTprg0WLFqlN4s0WLVqUnZ0tIlHtu0UWKqw6DgC4Bwo8AMANLFmyxGq1ikiXmDb+AX6q48C91XvskTJlS4jIkSNHDhw4oDqONzKbzXnvngzpO1JtGABwIxR4AICrM5vNy5YtExGdTte9F7vH4X7pdLqY2JtL2S1ZskRtGO+0cuXKy5cvi0iTBi2qVqquOg4AuA0KPADA1W3cuPGPP/4QkfoNa2kTp8B9iu7Wxs/PV0TWr19/9epV1XG8i91unzt3rjYe1n+U2jAA4F4o8AAAV5d3qu3j/710GbhPoWHBrds3EZGsrKxVq1apjuNddu3adfz4cRGpXKFqo/rNVMcBAHdCgQcAuLSjR4/u379fRIoULdS0eT3VceA5eva+eTkGG8I7WVxcnDYYNmCMTqdTGwYA3AsFHgDg0vKm33v06qg38GcLDvNwzWoVKpYRkZ9//vmbb75RHcdbHD16dO/evSJSOLJIdPtuquMAgJvhlRAAwHXduHFj7dq1ImI0GqNj2qqOA08TE3tzEp6l7Jxm9uzZdrtdRPrHDvXx8VUdBwDcDAUeAOC6Vq9enZ6eLiItWjeILByhOg48TecuLbVdCTdu3Jiamqo6jue7cOHChg0bRMQUYOrbY6DqOADgfijwAADXlTcv2qM3u8fB8YKCA9t2aCoiOTk5LGXnBImJiRaLRUR6du0TGhKmOg4AuB8KPADARR08ePDw4cMiUqZsibr1HlYdB56p++M3N4RfunSp2iQeLyMjY9myZSJi0BsG9xmhOg4AuCUKPADAReVNv8c83p6lqpFPHqpZtVKV8iLy888/79u3T3UcT7Z06dLr16+LSJsWHUqXLKs6DgC4JQo8AMAVpaenr1+/XkR8fX2iurVRHQeerGuPdtqApezyj8ViSUxM1MbDBoxRGwYA3BcFHgDgitasWZOZmSkiLdo0DAsPUR0HnqxTdAs//5tL2V29elV1HM/00UcfnT9/XkTq1KxX66E6quMAgLuiwAMAXFHeBck9Ylm+DvkrOCSodbvGIpKdnb1mzRrVcTxTfHy8Nhjen+l3ALh3FHgAgMs5dOiQtnxdyVLFaj9aQ3UceL5uPW+eRc9Sdvlh9+7d2m902dLlWzVrpzoOALgxCjwAwOXklaievTuyfB2coHbdGuUeKCUiR48ePXTokOo4niYuLk4bDO03Sq/nxScA3Duj6gAexUX+Jmkvdg0Gg+og3stgMNjtdtUpvItOp9PpdDztFXLgg5+RkbFu3ToR8fExRndv6yKHVhekHe15fBylW4/206YkiMiyZctq1679T1+W94BzzLlDR48e/fzzz0UkIrxgzy697/8Zq9fredo72Z8fcOc8+Hlv3fKLBvwFBd6RQkNDXWemKCwsTHUE7xUaGqo6gpfiaa+KXq934IO/Zs2aGzduiEi7js3LlCnlqLv1SL6+vr6+vqpTeIje/bvOeGeBxWxZv359XFxcYGDg7b/eaDRyzLkT8+bN097UHjZgdGRk4fu/Qz8/Pz8/v/u/H9wDpx1zTCaTNvD39+cXDfgzCrwjpaenu8K8q/aHTdtqFc4UGBiovUl8/fp1V3gmeBWTyaTX6zMyMlQH8TrBwcE6nc5utzvwmDN37lxtEN29Df+nt2EymaxWa05OjuogHsI/wLd5y8e2bdmdnp6+ePHiPn363PLLdDpdcHCwiOTm5mrvNOE2Ll68+OGHH4qIv39A75gB9/8bXaBAAYvFYjabHZEOd8pgMAQEBIiIxWJxzjEnOytbG+Tk5Fy7ds0J3/H2QkLYDAWuggLvSBaLxRVqm4+PjxZGdRCvk/e/7yLPBK9is9mEp706drvdUQ/+sWPHDhw4ICLFSxapXbdGbm6uQ+7WI9ntdrvdzkPkQF26t922ZbeILFmypGfPnrf8mrzzhx34tPdgc+bM0cp296jYkOBQhzxdrVYrT3tVnHbMsdqs2sBms/GLBvwZVxABAFzIkiVLtEHX7u30ele5KAle4rGGtYoULSQi+/fvP3HihOo4bi8jI0P7jTboDcP6j1YdBwA8AQUeAOAqcnJy1q5dKyJ6g75LTBvVceB19AZ99H+feMuXL1cbxgMsWbJEO/m5TYsOZUqVUx0HADwBBR4A4Cq2bNmSlpYmIg0b14ksUlB1HHijLjFttVM/Vq1axYm798NiseStZzF8wFi1YQDAY1DgAQCuIm/Os1uPdmqTwGsVL1nk0fqPiMiVK1d27NihOo4bW7du3W+//SYidWrWq/nQP27LBwC4KxR4AIBLOHfu3J49e0QkPCK0SYt6quPAe0V3b6sNVqxY5uoVAwAAIABJREFUoTaJ+7Lb7fHx8dp41OAn1YYBAE9CgQcAuIQVK1Zouwl07traaGSTFCjTsk3D4JAgEdm1a9fvv/+uOo5b2rVr148//igiD5Sr2KJxa9VxAMBzUOABAOrZbLakpCRt3PW/85+AEn5+vh06NxeR3NzclStXqo7jlt5//31tMGLgWJ2O7SQAwGEo8AAA9Xbv3n3+/HkReahm1XIPlFIdB96uy3/fRUpKSrLb7WrDuJ1Dhw7t3btXRIpEFu3asYfqOADgUSjwAAD1WL4OLqVq9QqVq5YXkZMnTyYnJ6uO42ZmzZqlDQb3HeHj46s2DAB4GAo8AECxtLS0rVu3ikiAyb9Nhyaq4wAiItHdbm4Iz1J2d+XMmTPar3NgYFDv7gNUxwEAT0OBBwAotnbtWrPZLCJt2zctUMCkOg4gItIxuqWvr4+IbNiwITMzU3UctxEXF2e1WkWkT/eBQYHBquMAgKehwAMAFPvwww+1QXT3NmqTAHlCw4KbtqgvIhkZGRs3blQdxz388ccf2rJ/RqPPwN7DVMcBAA9EgQcAqHT06NEffvhBREqVLl6zdnXVcYD/l/eOUt57TLi9uXPnZmdni0i3Tj2LFSmuOg4AeCAKPABApf+ffo9pzXZTcCkNG9WJLBwhIl9//fUvv/yiOo6ru3HjxgcffCAiOp1u+IAxquMAgGeiwAMAlMnNzV23bp2I6PW6Tl1aqY4D/A+9Qd8xuqWI2O12NoT/V0uWLLl69aqItGzatkL5SqrjAIBnosADAJTZsWPH5cuXReTRx2oWLRapOg7wV3lr0a9cudJms6kN48osFktiYqI2HjX4SbVhAMCDUeABAMokJSVpg+gYlq+DKyr3QKmHHqkiIufOnfvyyy9Vx3Fdq1evPn/+vIg8Wvux2g/XVR0HADwWBR4AoEZKSsrOnTtFJDCoQMvWDVTHAW4t6r+T8HnvN+EvbDZbXFycNh456Am1YQDAs1HgAQBq5G3/3q5jUz9/P9VxgFtr36mZf4CfiGzatOn69euq47iiTz755OeffxaRyhWqNm/EYhYAkI8o8AAANfLmMzt35RU/XFdgUIHmLRuISHZ29qZNm1THcUXvv/++Nhg1+En2kgCAfEWBBwAocPTo0cOHD4tI6TLFH65ZTXUc4HaiY1prAzaE/7u9e/cmJyeLSInipTq166I6DgB4OAo8AOD/2LvzuBrT/g/g33M67XtpkyKiVCokpbSLNgozGOuMZRjLY8Yy1sc2M9ZhbMNgHsxglERSyJayhKwRoZQmkpS0nzrn/P6453eenmxxlrs6n/cf87rc577v63N6Tcv3XNd9XSwQb8o1YBC2f4emzq1XNxNTIyK6cuXK48eP2Y7TtGzcuJFpTBg1mafEYzcMAECLhwIeAADkra6uLjo6mrD9OzQTXCVu/whsCP8Wd+/ePXPmDBEZ6rcaOmgk23EAAFo+FPAAACBvZ8+eZbZ/d3Vzxvbv0CwMGNyXmSqyf/9+bAgvtn79epFIRERfjfhaXU2d7TgAAC0fCngAAJA38Rhm/4GB7CYBaKR2Vm26ONkSUV5eXnJyMttxmoTc3FxmVT8tTa3Rw8axHQcAQCGggAcAALkqLS09fvw4EWloqPfp15vtOACNNWDQP5837d69m90kTcSGDRvq6uqIaMTnX+nq6LEdBwBAIaCABwAAuYqNja2pqSGigH6e6hpqbMcBaKygUB8VFWUiio6OrqioYDsOywoKCpidIFVUVMeNnMR2HAAARYECHgAA5CoqKoppYP48NC86uto+/u5EVF5eHhMTw3Yclm3ZsoXP5xPR5+FfmBibsh0HAEBRoIAHAAD5ycvLu3LlChEZm7Zy7enEdhyAj9N/4D+bJvzxxx/sJmFXSUkJ8xVQ4ipNGD2Z7TgAAAoEBTwAAMjP/v37mTWr+4cHcJXwOwiaGU/vHgaGekR05syZv//+m+04rNmxY0d5eTkRhfYLt2rbge04AAAKBH88AQCAnIhEIvH8eWz/Ds0Rj8cLHeBPREKhcM+ePWzHYUdFRcWOHTuIiMPhTB73LdtxAAAUCwp4AACQk7S0tJycHCKyc+jUoWNbtuMAfIrwwX2Zxt69e9lNwpbdu3cXFxcTUYB3386d7NmOAwCgWFDAAwCAnIi3fx8wEMPv0FzZ2ll3tu9IRJmZmdevX2c7jrzx+fwtW7Yw7W8w/A4AIHco4AEAQB74fH5cXBwRKSkp9Qv1YTsOwKcbNCSYaRw4cIDdJPK3f//+goICInLv4eni7Mp2HAAAhYMCHgAA5CExMZGZduvh5WLYSp/tOACfbuBnQUpKXCKKiYlhtlJTEHV1dRs2bGDaU8Zj+B0AgAUo4AEAQB7EY5X9sXwdNHPGJq16+7gRUXFx8cmTJ9mOIz8xMTG5ublE1MXOqbe7L9txAAAUEQp4AACQueLi4lOnThGRlpaGb4A723EAJDV4aAjTUJxZ9EKhUDz8/q+vZ3E4HHbzAAAoJhTwAAAgc7GxscxM4z5BXqpqqmzHAZBUvxAfbR0tIjp58iTzbEiLl5CQkJmZSUQ2HTsH+gWzHQcAQEGhgAcAAJkTj1KGhQewmwRAKtTUVPuF+BARn8+PjY1lO448rF+/nmlMHT8Dw+8AAGxBAQ8AALKVm5ublpZGRCamRi49ndiOAyAdgz5XoLXoT506dfPmTSJqa2EV0ncA23EAABQXCngAAJCt6OhokUhERCED/LhcDNxBC+Hq7tza3ISI0tLSsrOz2Y4jW2vXrmUaU8Z/y1PisRsGAECRoYAHAADZio6OZhqYPw8tCZfLDRngT0QikahlD8KnpKRcvXqViMzN2gwKG8J2HAAAhYYCHgAAZOjGjRuPHj0iIpvOHTraWLEdB0CaBgwKZBqRkZHMNJMWSTz8Pumrfykrq7AbBgBAwaGABwAAGRIPv4cO8Gc3CYDUtbNq4+BoQ0R5eXlXrlxhO45MXL169fz580Rk1Mp4yMARbMcBAFB0KOABAEBW6urqDh06RERcLie4vy/bcQCkr//APkwjMjKS3SQy8vPPPzONiWOmqqmqsRsGAABQwAMAgKykpKS8ePGCiHr0dDYxNWI7DoD0BYX68ng8Ijpy5EhNTQ3bcaTs5s2bp0+fJiJD/VYjhnzFdhwAAEABDwAAMvPf+fPhmD8PLZO+gW5vH1ciKi0tTUxMZDuOlImH38ePnqyhrsFuGAAAIBTwAAAgI1VVVfHx8USkqqoS0NeT7TgAsiL+fKqFrUV/9+7dEydOEJGerv7ooWPZjgMAAEQo4AEAQEYSEhIqKiqIyMffXVtHi+04ALIi/j/81KlTxcXFbMeRmjVr1jBL6381/GstLW224wAAABEKeAAAkJGDBw8yjeD+fuwmAZApVVWVvsFeRFRbW8us2tgCZGZmJiQkEJGWlvZXI75mOw4AAPwDBTwAAEhfcXFxUlISEenqaXv59mQ7DoBshYYHMA3xug/N3Zo1a4RCIRGNHTFRV0eP7TgAAPAPFPAAACB9hw8frq2tJaLAIC9lZR7bcQBkq3uPLuZtTIkoLS0tKyuL7TiSevDgwZEjR4hIS1Nr7IiJbMcBAID/QgEPAADSJ54/HzIA689Dy8fhcILDfJl2CxiEX7t2LTP8PnrYOH09A7bjAADAf6GABwAAKcvLy7t69SoRmZoZdXPpwnYcAHkIi+jDNCIjI5m135qp7Ozs2NhYItJQ1xg/ejLbcQAA4H+ggAcAACk7ePAgU8AE9/fjcjlsxwGQh/bWlnYOnajeB1jN1Jo1a+rq6ohozBcTDPVbsR0HAAD+Bwp4AACQspiYGKYRgvXnQZGENf8N4bOyspjvXw11jfGjv2E7DgAANIQCHgAApCkjI+PevXtEZN2xnU3nDmzHAZCf4P5+SkpKRHTkyBE+n892nE+xdu1agUBARCOHjG1lYMR2HAAAaAgFPAAASFO95esw/A6KxbCVfq/e3YmouLj49OnTbMf5aI8fP2aG39XV1Cd+OYXtOAAA8BYo4AEAQGpEItGhQ4fon0W5UcCDwhFvCN8cZ9GLn34fOXRsK0NjtuMAAMBboIAHAACpSU1NzcvLIyKnrnbmFqZsxwGQt4BADy1tTSJKTEwsLS1lO85HyMrKYqbPqKupT/pyKttxAADg7VDAAwCA1IiXrwvu78tuEgBWqKqp+gd6EFFNTc2RI0fYjvMR1qxZwzz9PnrYeAy/AwA0WSjgAQBAOmpra5mKRUlJqW+wN9txANgh3hA+KiqK3SSNl52dffjwYSLSUNf4Gk+/AwA0YSjgAQBAOs6cOVNcXExE7p7dDFvpsx0HgB2ubs4mpkZEdPny5SdPnrAdp1FWrlzJPP0+eth4LD4PANCUoYAHAADp+O/8eSxfBwqMy+UEhfkSkUgkEn9TNGWZmZnM8LumhiaG3wGaNSMjIw6Hs3DhQraDgAyhgAcAACmorKw8ceIEEamqqvj16cV2HAA2hQ7wZxrNYi36VatWCYVCIvpy+ARD/VZsxwEAgPdBAQ8AAFJw/PjxiooKIvLxd2dW4QZQWLZ2HTp2akdEDx48uHXrFttx3ufevXtHjx4lIi1NrQmjMfwOANDUoYAHAAApqDd/HuvPAzSbDeHFw+9fjZior2fAdhwAAPgAFPAAACCpkpKSs2fPEpGWloanjyvbcQDYFxYewOVyiOjgwYPM+nBNUHp6enx8PBHpaOtOGD2Z7TgAAPBhKOABAEBS0dHRfD6fiPoEeamqqrAdB4B9xqaturs6ElFRUVFKSgrbcd5uxYoVIpGIiMaP+kZXR4/tOAAA8GEo4AEAQFJ//fUX0wjuj/XnAf4hXsouOjqa3SRvde3atcTERCLS1zMYN3IS23EA4CNERkYGBQWZmJioqalZWVl9/fXXDx48eM/59+/fnzx5sq2trZaWlra2dufOnadMmZKZmfnWk1+/fr1s2TIfH582bdpoamo6ODh89tlnly5detfNy8vLV65c6e7ubmhoqK2t7eTkNGnSpPfnAUmggAcAAInk5+czA4ytjAx6ujmzHQegqegb7M1MSImPj6+srGQ7TkMrVqxgGl+Pmaqlpc1uGABopIqKiqCgoKFDhx4/frywsLCmpiYnJ2fbtm3Ozs6xsbFvvWTt2rVdunT59ddfMzMzKyoqysvL79+/v3nzZgcHh7Vr1zY4+dKlS1ZWVv/+97/PnTuXn59fWVl59+7d6OjoXr16zZ49+82bX7t2zcbGZs6cOampqcXFxeXl5bdv3966dWvnzp1XrVol/fcPRDy2AwAAQPMWFRXFLILVN8Sbq4TPhQH+oaWt6ePvfiLhXEVFRUJCwuDBg9lO9F+XL19OSkoiIgN9w9HDxrEdBwAaa/jw4cePHyciAwODL774okePHs+ePUtISEhOTh42bJhAIGhw/s6dO2fMmEFEqqqqX375pZubG4fDSU1N/c9//lNTUzNjxgwDA4MxY8YwJxcWFg4cOLC4uFhdXf2rr77y8PDQ0NDIyMjYuHHjs2fPVq9e7efn169fP/HNs7Oz/fz8Xr9+TUSfffaZv7+/np5eamrq9u3bKyoqvv/+ey6XO3PmTPl8ZRQHCngAAJDIvn37mEYI5s8D/K+QAf4nEs4RUXR0dJMq4H/44QemMWX8d1qaWuyGAYBGOnLkCDPMbm9vHxcXZ2VlxRyfPXv2vHnzxNNqxF6/fj19+nQiMjIyiouL69mzJ3N81KhRo0ePDg0NLSoq+vbbbwcPHqylpUVEx44dKygoIKLo6Ojg4GDm5AEDBgwePNje3r62tvbgwYP1C/iJEye+fv1aSUkpKipq4MCBzMEhQ4ZMmjQpODg4Kytr8eLFw4YNMzc3l93XRAFhqAQAAD5dVlbWtWvXiKiNpVkXJ1u24wA0LV6+rvr6ukR07ty5wsJCtuP84+zZs6mpqURkamw2ashXbMcBgMbasmUL09i1a5e4eiciDoezfPlyFxeXBufv2bOHGR5ftGiRuHpn9OzZc8GCBUT06tUr8Qfx169fJyIVFZWgoKD6J3fs2HHSpEnh4eGGhobig3fv3j158iQRTZ06VVy9Mzp16rR9+3Yiqqio2LZtmyRvGd6EAh4AAD7d3r17mVWsg0J9ORwO23EAmhYejxcY7EVEdXV1hw4dYjsOEZFIJBIP002bOEtVVY3dPADQeEyB3bdv3zdrdSKaNWtWgyOnT58mIj09vbFjx755/rhx45iB9zNnzjBHmPqcz+cfOHCgwcnr168/dOhQ/UH+uLg4pvHdd9+9eXNfX18HBwciunjxYmPeGjQeCngAAPh0UVFRTCM4zJfdJABNU1hEANN48w9iVhw7doypAdqYWw6NGM52HABorNevXzMTeVxdXd96wpvH09PTicjW1lZN7S0f1Wlqanbs2JGIbt26xRwJDw9XVlYmoiFDhnTr1m3z5s1ZWVnvynP16lUiMjc3t7CweOsJjo6ORHTnzp0PvDH4SCjgAQDgE929ezcjI4OIOtm272hj9cHzARSQU1c7C8vWRHTr1i3W91USCoXiAbTvvpmjrKzCbh4AaLxHjx4xjfbt27/1BAsLCx7vfxY4KykpIaJ27dq9657MS8xpROTo6Lhr1y59fX0iunHjxpQpU6ytrc3MzEaOHHn48GE+n1//2ufPnxNRfn4+5x2YmfllZWWf9n7hXVDAAwDAJzp48CDTCB/Ul90kAE0Wh8MJDf9nQ3jxtwxbYmJi7t27R0TW7TsNDPuc3TAA8FHeXGG+AQ6Ho6SkVP8I84zbezAFf11dnfjIF1988eDBgzVr1vTu3Zu5W0FBwZ49eyIiIpycnJhRd0ZFRUVjYldUVDBb1YC0oIAHAIBPIRKJDh8+zLRDwwPYDQPQlIVFBDArRBw4cOCDf0/LTm1t7cqVK5n2rKnzlbhK7z8fAJoUAwMDpvH48eO3nvD06dOampo3L8nNzX3XPZlbie/MaNWq1YwZM5KTk1++fJmQkDBr1ixTU1Miun//fnh4+KtXr5jTdHV1iahbt26iD+FyUXJKE76aAADwKa5evZqXl0dE3Vy6tLN6+/NvAEBElm3NHRxtiCgvL+/KlStsxdi7d29OTg4RdbFzDgoIYysGAHwaKysrbW1t+v+Hz99048aNBkfs7e2JKDMzs8Hsd0Z1dfXDhw/FpxFRaWlpaWlpVVUV809dXd2goKBVq1bl5eXNnz+fiJ4+fXru3Dnm1U6dOhFRRkbGB6cGgHShgAcAgE8hXlJ7wMBAdpMANH3iWSpsLWVXXV39888/M+3v/7UAe0YANDtcLtfX15fqLUXZwOrVqxsc8fPzI6Li4uJdu3a9ef6OHTtKS0uJyN//n8d8bGxs9PT0wsPDG5zJ4/FGjRrFtIuKipgGc1V1dXV0dPSbN6+oqHB2drayslq+fHkj3yA0Egp4AAD4aAKBIDY2loiUlLhhEX3YjgPQ1AWF+jBPkx45cuStQ2Gytn379oKCAiJyc/Hw9vCXfwAAkNykSZOYxpgxYxpMjP/pp59SUlIanD9y5Ehmo7hFixY1qPmvXLmydOlSItLR0Rk5ciRzsFu3bkSUlJT05tLx4ipdvNZ9RESEmZkZEU2bNi0zM7P+ySKR6Lvvvrt161ZOTk7//v0/7c3Cu/A+fAoAAMD/Sk5OfvHiBRG5e3Q3NmmF9WkA3s/AUM+jt0ty0uWSkpJTp04FBwfLs/fXr19v3LiRaX8//d/y7BoApKhfv36DBg06ePBgenq6i4vLyJEjXVxcXrx4kZCQkJiYqKam1r59e2Z3GIaent7atWsnTJhQUFDg6ek5fvx4Nzc3kUh06dKl7du3Mw/M//LLL8zT7EQ0atSoY8eO8fl8Pz+/b775pmvXrurq6k+fPo2Li4uJiSEid3d38Xx7FRWV3377bcCAAYWFhY6OjhMmTHB1dTUyMsrJydm9e3dqaioRTZ8+XXw+SAsKeAAA+Gj/nT+P9ecBGick3D856TIRRUdHy7mA37hxI7NNVB+ffi7Ob99BGgCahT/++KOiouL48eNFRUXr1q0TH1dXV9+zZ8+BAwfqF/BENH78+OLi4vnz51dVVW3YsGHDhg3il3g83qpVq7788kvxkaFDh6akpPz6668vXrxYsmRJg667dOkSGxtbf0W6sLCw3bt3T5gwobq6etOmTfVP5nA4X3/99dq1a6XyrqE+FPAAAPBx+Hz+8ePHiYjH4/UL8WU7DkDz4N/HQ1NTo6KiMjExsbS0VDzkJWuFhYXbtm0jIi6XO2vaAvl0CgAyoqGhcezYsf379+/evfv69euvXr0yMjIKCAiYPXu2nZ1deXm5qampu7t7/Uu+//770NDQjRs3njlzhtm23dzcPCAgYNq0aTY2Ng3uv3nz5hEjRvzyyy/379/Py8urra21tLTs0KHDmDFjwsPD31xPfuTIkT4+Phs2bDh+/Dgzq9/a2trR0XHGjBldunSR6ZdCYaGABwCAj3P69GlmNM/Dy0VPX4ftOADNg5q6qn9fjyMxJ2tqauLi4kaMGCGfftesWVNZWUlEESGfde6EuawALcHQoUOHDh365vFRo0aJV5urz97efuvWrY28ubu7e4OPAN7PwsJi9erVby6hBzKCRewAAODjiOfPB/f3YzcJQPMS9v9r0UdFRcmnx5ycnD179hCRsrLKd5PnyKdTAACQHRTwAADwESorK0+cOEFEauqqvgEf8Qk9APTs1c3YtBURpaam5uXlyaHHn376qba2loiGfzbask07OfQIAAAyhQIeAAA+wrFjx5jpuL4BvTQ01NmOA9CccLmcoBAfIhKJRAcPHpR1d3fu3GG2e9TU0Jw2YaasuwMAADlAAQ8AAB9BPH8+KBTL1wF8tND/n0V/4MABWfe1ePFiZovHCWOmGLUylnV3AAAgByjgAQCgsV69enX27Fki0tbR8vTuwXYcgOans721dcd2RPTgwYNbt27JrqOkpKRz584RUSsDowmjp8iuIwAAkCcU8AAA0Fjx8fF8Pp+IAgI9VVSU2Y4D0CyFRvwzCB8dHS2jLoRC4bJly5j29EmztTS1ZNQRAADIGQp4AABorJiYGKYRFObDahCAZix0gD+XyyGimJgYgUAgiy4OHTp0+/ZtIrJs0+6LwW/ZUwoAAJopFPAAANAohYWFFy5cICJ9A92evbqyHQeguTI1M+ru6khEhYWFycnJUr8/n89fvnw5054zfaGysorUuwAAALaggAcAgEY5fPgwM1oYFOqrpKTEdhyAZky8IbwsZtHv3LkzNzeXiBztu4b2jZD6/QEAgEUo4AEAoFEOHz7MNPqF+rAaBKDZCwzyUlVVIaL4+HhmX0ZpKS0tXbt2LdOe++0iDocjxZsDAADrUMADAMCH5eXlpaWlEZFZa+Ou3e3ZjgPQvGlpa/r4uxNRRUVFfHy8FO+8fv364uJiIvL3CvR085binQEAoClAAQ8AAB8WExMjEomIqF+oD8b0ACQXGu7PNKS4IXxeXt62bduISImrNPfbxdK6LQAANB0o4AEA4MMOHjzINEL6+7ObBKBl6O3jqq+vS0TJycmFhYVSueePP/5YU1NDREMGjrDp2Fkq9wQAgCYFBTwAAHzAw4cP7927R0RWHSxs7TqwHQegJeDxeH1DvIlIIBCIN2iUxK1btw4dOkREGuoa330zR/IbAgBAE4QCHgAAPkA8/B4U6stuEoCWRLwWfVRUlOR3W7x4sVAoJKIJY6aYGJtKfkMAAGiCUMADAMAHMMN6RBQchgIeQGocu3a2bGtOROnp6ZmZmZLc6sSJE+fPnyciYyOTiV9Ok04+AABoelDAAwDA+9y8eTM7O5uIOttbt2tvwXYcgJaDw+GERfwzCC/JUnZ1dXXLli1j2jOnzNPU0JRCOAAAaJJ4bAdolMLCwv3799+4ceP169f6+vouLi7Dhg3T1dWV4rV5eXmRkZG5ubkFBQWGhoZt27aNiIiwtbWVwbsBAGhOxE/nhvT3YzcJQMvTf2CfX9f/IRKJoqKi5s2bx+V+ysjK7t27mQF82452n0cMl3ZGAGgSSktLX758yXYK0tDQMDXFQzpsagYF/OPHj+fOnVtZWUlEenp6hYWFCQkJly5dWr16tbGxsVSuPXPmzMaNGwUCAY/HMzIyKioqevr0aWpq6qBBg0aNGiXTdwcA0JQJhcLY2Fgi4nA4gcHYUxpAyszbmDp1tbt5/e6zZ88uXbrk4eHxsXcoLS1dtWoV054/Y6kSV0naGQGgSdixY8fMmTPZTkH9+/dn/jAAtjSDKfQbN26srKx0cXH5448//vjjj99//71jx44lJSXMTqeSX8v8UyAQDBo0KCoq6rfffouKipo0aRKXy42Ojk5NTZXlmwMAaNIuX7789OlTIura3b61uQnbcQBaoP4REi1lt27duuLiYiLy7R3g44ldHgEAWrimPgL/4MGDR48eGRoazpkzR0VFhYiMjIwWLVr01VdfpaWlvXz50tDQUMJrT58+XVlZ6eDgMHr0aOZCLpcbFBRUWlq6b9++48ePu7m5yeW9AgA0OeL580FYvg5ANvqG+KxY9iufXxsXF7dy5Uo1NbXGX5uTk7N9+3Yi4inxFsxYJrOMANCE2HTo0NrkA9OQpa6yqvryjRty7hTeqqkX8KdOnSKiXr16MRU4Q0dHp3v37pcuXTp79uzgwYMlvDY3N5eI3py05urqum/fvkePHkn1DQEANBu1tbVHjhwhIiUlpb6YPw8gG7p62l6+PU+dOF9WVnbs2LGIiIjGX7tkyRI+n09EwwaN7GSNhXsAFMKUMaPHfP6ZnDt9lJPTtW+QnDuFt2rqU+jz8/OJyMnJqcFxR0dHIvr7778lv7ZDhw4hISHMwfrKysqISFMTS7kCgII6d+4cMzXXzaOrgaEe23EAWizxhvAftRb9xYsXjx49SkRamlrffjNHJskAAKCJaeoj8Mzfjm8uGq+np0dEJSUlkl8bHh7+5rV8Pp95FM3FxeXNV29jNg7fAAAgAElEQVTcuDF+/Pg3j58+fbqRa+PLQatWrdiOoLje82QHyBT+t5eu+Ph4pjF4SCjzk/NduFzu+08AGVFSUqo/ywzkicfjSet/+/4RfRfPX1dSXHr27FmBQGBi8uH1JoRC4ZIlS5j29/9a2NG6k1SSNAvq6urq6upsp1BQKioq8vmZIx5CU1dXxy93gPqa+gg8s1mCtrZ2g+M6Ojr0/yW61K8tLi6eN29eenq6kZHR559//knBAQCat6qqKmaZWVVVlX4hPmzHAWjJlFWUQwcEEFFdXV1kZGRjLtm1a9f169eJqJ1l+0nj/iXbfAAA0GQ09RF4BofDaXBEJBIRUV1dnXSvra6ujo2NjYmJqaqqatu27fz58986oq6qqmpubv7mcaFQKBAIPhhJ1rhcLofDaQpJFA3zlScifPHlj9k8WSgUsh2k5Thy5AjzJJFvQC9NLY13fW3F21bjiy9/zA8c5pcayBPzv71IJJLiF3/gZ0F/7jxIRH/88cfkyZPff3JZWdmCBQuY9g8LVivzlBXnG5DL5Ur3Kw+NJIv/7d9DKPynF5FI1BT+rFJSwgaN0FQ09QLe0NAwPz+/rKzMzMys/nHmz0oDAwMpXnvhwoVt27aVlJRoamqOGjUqIiLiXd+rdnZ2b93/8OXLl++f1S8fGhoa6urqTSGJotHT0+PxeET06tUr/G0hZ9ra2lwut7S0lO0gLceff/7JNPoE9X79+vW7TtPV1eVwOEKh8D3ngIxoa2sLBILKykq2gygWLpfLTOUTCATl5eXSuq21TVsLy9Z5T55eu3YtNTXVxsbmPScvW7bs2bNnROTew9O7l59Cfffp6upWV1fX1NSwHUSx8Hg8LS0tIqqtrZXPz5yqqn96qa6ubgp/02IaPzQdTX0KPVNmv/mbiTny/u+lxl/L5/OXL1++cuXKsrKywYMHb9++ffDgwfikDQAUVllZGbORh6amho8fttIEkDkOhxP2/xvCR0dHv+fMnJycrVu3EpESV2nxnOXyCAcAAE1GUy/gW7duTUR3795tcDwjI4OIGgytf9q1AoFgxYoVly5dsrCwWLNmzahRo5iPGAEAFNbRo0eZAS7/vh6qaqpsxwFQCGERAcxjEZGRke+ZM7xo0SJm67gvBo+ys3GQXz4AAGgCmnoBHxAQQEQXLlyo/3AXn8+/evUql8v18/OT/Npz586lpaV17tx53bp17du3l8nbAABoVg4dOsQ0gkJ92U0CoDgsLFs7dbUjomfPnl28ePGt5yQnJyckJBCRjrbujKnz5JoPAACagKZewNvY2HTo0KGgoGDLli3ixedWrVpVWVnp6upqbGwsPjM+Pj4uLu7Ro0cfey2zh+qQIUM4HE7tGxqzTh4AQEvy4sWL5ORkItLX13X37MZ2HAAF0n9gH6bB7GXbQF1d3fz585n2t998b6iPh3IBABROU1/EjoimTp06d+7cEydOXLhwwcLC4vHjx9XV1QYGBg12Yt+5cyefzx89erS1tXXjrxUIBI8fPyaixYsXv7V3MzOz3377TVbvDQCg6YmNjWWm7wYGezHrMgKAfPQL8V6xdDOfX3v06NFVq1Y12O18586d9+/fJyJrq45jvhj/jnsAAEBL1tRH4Imoffv269ev9/PzU1ZWfvjwoba2dkhIyPr1642MjCS/9sWLF01hawoAgKYjJiaGaQSFYf48gFzp6Gp7+7kRUXl5eXx8fP2XXr58uWrVKqa9eM5ynhI+XAOA5qG0tJTD4XA4nNzc3Lee8PLlS+aE/Px8STrau3cvh8OZOnWqJDdp+prHT39TU9Pp06e//5x3Ldn6/mtNTU2PHDkiUTgAgBYkLy8vLS2NiEzNjLq5dGE7DoDCCYvoc/J4ChFFRUUNHjxYfPyHH3549eoVEfXxDfL28GctHwCA7E2aNGnr1q22trb37t1jO0uT0wxG4AEAQG4OHjzILBoSFObL5XLYjgOgcLx8XfUNdIkoOTm5oKCAOXjz5s19+/YRkYqK6qLZP7KZDwAAWIUCHgAA/ks8fz447H3bfACAjPB4vOAwXyISCATM7EKRSDR37lxmS52vx0xpa2HFckQAABkbNGjQmjVrZs+ezXaQpggFPAAA/OPevXvMXDWrDhad7a0/eD4AyEJYxP+sRR8ZGck82NLa1HzK+O/YTAYAIBfPnz+fOXPm9evX6x+Mj48fOHCgmZmZgYFB//797969GxkZyeFwVq5c2eDyV69eTZs2zcLCQk1NrVOnTt99911JSYkc48tW83gGHgAA5ADD7wBNgYOjTXtry+xHT+7du3fp0qWlS5cyxxfMXKahrsFuNgAAVvz4448LFy4UiUQ6OjoaGhpxcXFnz57t37//m2cWFRW5u7s/fPjQwcHBwMDg9u3b69atu3bt2tmzZ7ncljB63RLeAwAASE4kEmH9eYAmQjwIP3/+/BcvXhCRR0+vsH4RrIYCAGDHhQsXFixYoK6uHhkZWVJS8uzZs6tXr+rp6TGLgzSwf/9+Lpf74MGDmzdv3rp16+TJkzweLzk5mZnK1AKggAcAACKitLS0J0+eEJGDo007qzZsxwFQaP0jAphVJNPT04mIx1NeOrfhHFEAgOala9euRm/TqVOn91+4cOFCIlqxYsXnn3/OjKK7uLjs37//Xefv3bu3ffv2TDsgICA0NJSIWsyC9phCDwAARPXnz/fH/HkAlpmYGvV073rpwj/Pf375xfhO1rbsRgIAkNCnPYheWVmZlJSkrq4+duzY+sc9PDwcHBzu3LnT4PwePXo4OzvXP8IU83V1dZ/QexOEEXgAAKC6urrDhw8TEVeJ2y/Eh+04AEBtLMyYhoa61nffzGE3DACA5HJyckRvU1RU9J6rsrOzRSKRlZWVhkbDRUDs7e3fPP/N8fyW8ei7mKRvxt7e/qeffsrJyZFGGAAAYEdKSgrz69O1p5OxiSHbcQAUXdnr8tMnLzBtQw1ToVDEbh4AALYwz/cZGxu/+ZKZmdmbB/X19WWeiVWSFvAZGRnz589v3769p6fnli1bXr58KZVYAAAgTwcPHmQaIeH+7CYBACLa8PPO4peviEhDRVtDWTcpKYntRAAA7DAxMSEiZjnPBt56sMWTtIB3cXEhIpFIdOHChW+++cbMzKx///6RkZFVVVXSiAcAADJXXV0dHx9PRKqqKn369mY7DoCiy7jzIHJfHBHxeDwTHUsiOnnyJNuhAADY0bFjRyJ6/PhxdXV1g5cyMzPZSMQySQv4q1evPnz48IcffnBwcCCi2trauLi4oUOHmpiYjB49OjExUSAQSCMnAADIyokTJ8rLy4nIy7enlrYm23EAFJpQKFq6YL1QICSikV8Mb9PanIju3buXl5fHdjQAABbo6Oh069atsrJy165d9Y9fu3atxewM91Gk8EC/tbX1/Pnz09PT79y5s2DBAmtrayIqKyv7448/+vbta25u/u233yrmFxcAoFn47/z5AZg/D8CyqH1xd25nEpGZqenXY7/s7duTOY5BeABQWD/++CMRzZ49+8iRIyKRiIju3r07ZMgQZWVlIuJwOCznky9prshnb2+/bNmyhw8fpqWlzZw508LCgoieP3/+yy+/9OjRw8bGZunSpY8ePZJijwAAIKGSkpLTp08TkbaOVm8fV7bjACi0ohfF69f8h2l/P2OGurq6t19P5m/TU6dOCYVCVtMBALCjX79+3377bVlZ2YABAwwNDdu2bevg4KCvr7969WoiatWqFdsB5UomS+p379599erVubm558+fnzx5sqamJhE9ePBg0aJFHTt2dHNz27RpE5a7AwBoCuLi4vh8PhH16ddbVVWF7TgACm3VD1vLXpcTkY9Xb19vLyIyNTPqZGtFRIWFhTdv3mQ5HwAAS9auXfvXX38FBgaKRKKampqJEyeePXuW2dr9rWvRt2Cy2hNPJBKlpaXFx8efOnWqoqKi/kuXL1+eOnVqu3btFi9ejM+SAQDYJZ4/H9zfj90kAAru0vlrCXFniEhdXX3OjJni495+7kwjMTGRnWQAABLQ1dVl9ntv27btW08wNDRkTjA3N2eODB8+XCQSbdy4sf5pQ4cOPXHiRElJSUFBwZYtW7S0tB4+fEhEpqam77mKiFavXi0SicaOHSvlN8YSKRfwAoEgKSlp2rRplpaWrq6uy5cvZ9YGVFFRCQ4O/v3339etW+fs7ExE5eXlS5YsmThxonQDAABA4+Xn56emphKRsYmha08ntuMAKK6aGv6yhRuY9tfjxpqZmYpf6uXZXUVFhYhSUlIqKyvZyQcAwB5PT09lZeXk5OT6B4uKivbt26ejo2NnZ8dWMFZIp4Cvrq4+evTo2LFjTUxMfH19N27c+PfffxORurp6eHj4nj17Xrx4ER8f/9VXX02fPv3GjRuXLl1iyvgdO3bcv39fKhkAAOBjxcTEMDOhgkJ9uUqympMFAB+0/dd9T3LziaijdYeRw4bVf0lDU62HmxMR1dTUNPj7FQBAEXz++ed1dXVffvkls29OSUnJyZMnfX19y8rKJkyYoKqqynZAueJJeP3+/fsPHTqUkJDAbEHE0NLSCgkJGTRoUHBwMPMAfANubm67du1ydnYWiUTXr1+3tbWVMAYAAHyC6OhophEyAPPnAVjzOCvvP79FEhGXy104dy6P1/DPM28/twvJV4noxIkT/fr1YyEiAAB7pk2blpmZuWXLlgY/AIOCgpYsWcJWKrZIWsAPq/chsZ6eXlhY2KBBg/r27aumpvb+C9u3b880mMXqAQBAzu7fv5+RkUFEbduZ2zl0YjsOgIISiURLF/7C59cS0cDwAU5durx5jqNzZ8NW+i+LSu7cufP06dPWrVvLPSYAAJs2b948ZcqU2NjYnJwcJSUla2vrbt26eXt7s52LBZIW8ETUqlWrAQMGDBo0KCAggNmLrzGEQuGmTZuIqEePHpJnAACAj3XgwAGmERbRh90kAIrs0IHjV1NvEVErQ8Ppkye/9Rwul3p79zh8MFEkEiUmJo4ZM0auEQEAmoDOnTt37tyZ7RTsk7SAP336tLe3t5KS0sdeqKurO/kdv6UAAEDWhEIhs/48h8MJHeDPdhwABfWyqGTN8m1Me9a332pra7/rTG9/98MHE4no5MmTo0aN4nKxaAUAgCKS9Kd/eno6M5D+fs+ePVu/fv3evXsl7A4AAKTi0qVL+fn5ROTU1a6NpWJtoArQdCxfuvl1aRkR9fbw6Bf4vrkw5m1MOtm2J6Lnz59jQ3gAAIUlaQE/ffr06dOnf/C0oqKi6dOnz5kzR8LuAABAKsTL14WGY/gdgB3nzqQeP5pERBoa6vNnz/7g+T5+bkwDG8IDACgsecy/EolEKSkpRPTixQs5dAcAAO9XU1MTFxdHRDwer2+IIi4AA8C6qsrqn5b8M4dxysRJ9Td+f5devV1UVFWIKDk5uf7uPwAAoDg+5Rl4IyOjDx6pr6qqqqKigoiwaCoAQFNw8uTJ0tJSIvL07qGvr8t2HABF9MvqHfl5BURkb2c37PPPGnOJhqaaa0+n88lX+Xx+cnJycHCwjDMCAECT8ykFfFFR0QePvNXEiRM/oTsAAJAu8frzoeEB7CYBUEw3r9/9689YIuLxeIvmz2v8inTe/m7nk68S0fHjx1HAAyim1Vu2/md/pJw7rebXyLlHeJdPKeAXLFggbv/www8Njry9Gx7PxcUlJCTkE7oDAAApKikpOXXqFBFpaWn4+ruxHQdA4fD5tYvmrhUKRUT05ciRNh07Nv7aLk7/bAifkZGRl5dnYWEhs5gA0EQ9efr0ydOnbKcA1nxKAb9s2TJxmyng6x8BAICmLDY2ls/nE1Gffl6qaqpsxwFQOL9t2pP1MJeIrNq1Gz/2q4+6lsslbz+3mKhjRJSYmDh27FiZRASApmfy5MlfffVxPzFkQVlZme0Iik7SRewmTpyIifEAAM2IeP58CNafB5C7zHtZv2+NJCIul7to3jxVFZWPvYOPvxuHwyGixMREoVAo/YgAANCEfcoIfH1btmyRSg4AAJCD3Nzcq1evEpGJqZGrmzPbcQAUi0AgWPj9mrq6OiIaMnhQV2enT7iJqZlRZzvrjLsPX758mZaW5urqKu2YANAUbd68eebMmWynoP79+8fGxrKdQqHJYxs5AABoIg4cOCASiYgoZIAfl8thOw6AYvnPb5EZdx4SkZmZ6bRvJn/yfXz83ZnG8ePHpZMMAACaiU8Zge/RowfTOHv27Pr16z/q2vnz539CjwAAIBXR0dFMIwzrzwPIV/ajJ1s2/ElEHA5n0bx5Ghrqn3wrN89uO7dHVVVVX7p06fXr1zo6OtKLCQBNXes2pgYG8t4Ctqaa//DBYzl3Cm/1KQV8Wloa0xAIBB9cf74BFPAAAGy5du1aVlYWEdl07tDRxortOAAKRCgQLpyzhs+vJaL+oSHuPXtKcjc1NRV3z25nTl6sra09ffp0RESElGICQDPwzbSRn38RJudOc7LzAr2Gy7lTeCtJn4HX0tKSSg4AAJA18fJ1/Qf2YTcJgKL5Y+fBW9cziMjIqNWs6dMlv6FPgPuZkxeJ6Pjx4yjgAQAUx6cU8ElJSUxDS0urrKxMmnEAAEA2+Hz+oUOHiIirxA0O82M7DoACycnO27h2F9NeOGeOtra25Pe0se3Q2tzkaf7zrKyshw8fdvyYzeQBAKD5+pQC3tvbW+o5AABApk6dOlVcXExEHr1djIwN2I4DoCiEAuH82atrqmuIKDQ4yLt3b6nclsMhH3/3fX8cJqITJ06ggAcAUBBYhR4AQCGI58+HYvk6ADna/Xu0ePL89zNmSPHO3n5uXC6XiE6fPs3n86V4ZwAAaLJkWMC/evUqNjb2999/T09PFwqFsusIAADer6SkJDExkYi0tDT8+3iwHQdAUWQ/erJp3S6m/e+5c3WkMXleTN9Ax7m7PRGVlZWdP39eincGAIAmSzoF/LFjx8aPHz9nzhzxkStXrtjZ2YWHh48bN87R0bF3795FRUVS6QsAAD7W4cOHmQG6Pv281NRV2Y4DoBAEAsH8WatqavhENCA01MvTU+pd+AVgQ3gAAMUihQJ+1qxZwcHBO3bsyMzMZI7w+fxRo0Y9e/ZMfM7Fixf9/PwwDg8AwAqsPw8gf79v3Z9+6z4RmRgbz/pWCivPv6lbjy46utpEdPPmzYKCAll0AQAATYqkBfz58+fXrFlDRMrKypaWlszBo0ePMsX8ggULMjMzf/jhByJKT08/cuSIhN0BAMDHys7OTktLI6LW5ibdXR3ZjgOgEDLvZW3Z8CcRcTicJQsXSGXl+TfxeErevj2JSCgUnjhxQhZdAABAkyJpAb927Voiat26dUZGxvr165mDMTExRGRmZrZo0aJOnTrNnz8/PDyciHbt2iVhdwAA8LGioqJEIhERhYYHcLkctuMAtHx8fu3cGStra+uIaHBEhHvPnrLry7dPL6Zx4sQJTHUEAGjxJC3g79+/T0RTp061trYWHzx16hQRjRgxgsf7Z5u6sLAwIsrOzpawOwAA+ChCoTAqKopp94/A+vMA8rD5l90P7mcTURtz8++mTZNpX20sTDvZtieiwsLCa9euybQvAABgnaQFfG5uLhHZ2dmJjzx69Oj58+dE5OfnJz5obm5ORI8fP5awOwAA+CgXL17My8sjIqdudu3aW7AdB6Dlu552Z+f2KCLicrnLFv1bQ0Nd1j36B/6ztcSxY8dk3RcAwMfKzc3lcDgcDqeiooLtLC2BpAW8np4eEVVVVYmPMI9gKSkp9erVS3ywsrKSiFRVsfQxAIBc7d+/n2kMGBjIbhIARVBZWTVv5kqhQEhEI78Y1s3ZWQ6dunt219BQI6JLly6VlpbKoUcAAGCLpAV8hw4diOjChQviIzt27CAiT09PHR0d8cEbN24QkYUFBn8AAOSnsrLy6NGjRKSqqtIvxJvtOAAt36oftvz95BkRWXfoMGXiRPl0qqam4u7ZnYhqa2tPnjwpn04BAIAVkhbwAQEBRLRjx45jx46JRKLIyMibN28SEbNqHSM9PX3r1q1EZGtrK2F3AADQeEeOHGGmq/n26cXsNQUAspN0+lL0/gQiUlZW/mnJYhUVFbl17dcHs+gBABSCpAX8v/71L319/aqqquDgYDU1taFDhxKRgYHByJEjiaiqqqpv374uLi4vXrxgTpY8MQAANBLmzwPITfHLV/+e8zPT/mbCeJtOneTZe0ebdpZtWxNRbm5uRkaGPLsGAJCipKSk4cOHOzg4aGtr9+zZc9KkScz25Iy+fftyOJxff/1VfEQgEOjo6HA4nCFDhtS/T1BQEIfDWblypfyiy4ukBbyuru7Ro0dNTU2JiM/nE5GKisqmTZsMDQ2ZI4mJiczxyZMnu7m5SRwYAAAa5cmTJxcvXiQiYxNDj94ubMcBaMlEItG/5/xc/PIVEXVzdh4zcqT8M4gH4ePj4+XfOwCA5NauXevr67tv374HDx4YGRmlpaVt3bq1a9euiYmJzAl9+/YloqSkJPElt2/fLisrI6KUlBTxQYFAwDzizZzfwkhawBNRr169bt26tXPnzmnTpi1duvTq1avDhg0Tv2pqahocHBwZGblp0ybJ+wIAgEbav38/s/17WEQfrpIUftoDwLsc2Hc06fQlItLU1PxxyWIul4XvuN4+rioqykSUlJRUXl4u/wAAAJLIzs6eM2cOh8NZunTp69evs7Ozi4uLv/jii6qqqgkTJjBDwoGBgUR07tw58VXnz58nIi6X++zZs6ysLObgzZs3y8rKTExMnJyc2HgrsiWdXzDGxsZjxoxZv379woULHR0dxcd1dXWfPXsWHx//+eefS6UjAABoDJFIJN7+PTQc278DyFBOdt6qn7Yy7TkzZ7Q2M2MlhraOZg83ZyKqqampPzwFANAsLFq0qLa2dty4cQsXLlRTUyMiXV3dvXv3du3aNTc3d/v27UTk4OBgZmZWWFh4//595ipmpH3gwIFUbxA+OTmZiAIDAzkcDivvRaYwJgMA0AJdvHgxNzeXiLo42Xbs1I7tOAAtVm1t3ezpy6uraogo0N+/f0gIi2H8A//ZwTchIYHFGAAAn4AZV58+fXqD41OnTqV60+YbDMKfP3++TZs2I0aMoDcK+BY5f56IeFK5i0gkys7OzsrKYqZrvouWlpaHh4dUegQAgPf466+/mMaAQVi+DkCGNq3blXHnARGZGBsvmDuH3TD2XWxMzYwLnhU+ePDg4cOHHTt2ZDcPAEAj1dTU5Ofn83g8GxubBi85ODgQUXZ2NvPPwMDA3bt3JyUlff311zk5Ofn5+cOGDevduzeHw2EKeJFIlJKSwuFw+vTpI+d3IR9SKODPnz8/YsQIZqjn/aysrMRfegAAkJHy8vK4uDgiUlVVCQ7zZTsOQIt15dLNndsiiYjL5f64ZLGujg67eTgc8uvTa98fh4koISEBu/8AQHNRUFAgFAqNjY2VlJQavGRmZkZEeXl5zD/79OnD4XCYEXhm/ryHh4eBgYGDg0N6evrz58+LiopevnzZrVs3Y2Nj+b4JOZF0Cv3jx4/9/f0bU70TUVFRkYTdAQDAB8XGxlZWVhKRX6AHtn8HkJGSktI5M1YIhSIiGjNiRI/u3dlORETkE+DO/Pl7+vTpqqoqtuMAADSKqakpl8stKioSCAQNXiosLGROYP5pZGTUtWvXZ8+ePXz4kFnBztPTk4i8vb2J6Pz58y17/jxJPgK/dOlSPp+vra09f/58Ly8vAwMDqcQCAIBPtm/fPqYRPqjF/vYCYN33038qLCgiIns7u8kTv2Y7zj/09LS7u3a5culmZWXluXPn+vXrx3YiAIAPU1VVNTc3z8vLe/DgQefOneu/dPfuXSKq/0xQYGDg9evXz507d/78eR0dnS5duhCRl5fXpk2bUlJSnj9/Tijg3+Py5ctEFBsb6+uLWZoAAOzLysq6evUqEZmYGrl7dGM7DkDLtPv3A4nHzhGRhob6imXLeDzpLCokFQGBnlcu3SSihIQEFPAA0Fx4eXnt3bt3/fr1W7durX98w4YNRNS7d2/xkcDAwBUrVhw+fPju3buBgYHMzp1eXl5ElJKSUlBQoKWl1atXL/nGlx8pTKG3tbVF9Q4A0ETs27ePWU90wKBAbP8OIAuZ97KWLfyFac+dNcvSog27eRpw7GpnZGxARBkZGY8fP2Y7DgBAoyxevFhZWXn79u2rVq2qra0lovLy8tGjR6elpVlaWk6cOFF8poeHh4aGRnx8vEgkYubPE5GJiYmNjc3169efPn3q5+enrKzMztuQPUk/MNbT02vVqpVUogAAgITq6uqY7d85HE74YKw/DyB91dU134ybV11dQ0RBgYHs7hv3Vlwu+QZ4RO2LI6L4+PgpU6bUf1UgELx8+fLFixcvXrx49erVq1evSktLS0tLy8vLy8vLKysrq6ura2pqiKi8vLz+7kJcLldTU5OINDU11dXVVVVVtbW1dXR0dHV19fT0DAwM9PX1jY2NTUxMNDQ05PuOAaB5aNu27Vs3Zp89e/asWbOsra1/+umnWbNmff/994sWLbK0tMzKyhIIBEZGRrt27WJ2hmeoqKj4+Pgw+2WKC3gi8vb2zszMpBY9f54kL+BdXV3PnDkjEAjeXDAQAADk7NSpUwUFBUTUw83Jsq0523EAWqAVSzdn3ssiojbm5gvmfM92nLfzD/Q4GJkgEAgSExO7dOny/Pnzv//+++nTp8+ePXvrGlGNIRQKy8rKiIj573toaWmZmpq2adPGwsLC0tKybdu2FhYWLXg0DAAa6eXLl289zqy8S0QzZ87s1q3bjh07bt269eTJE2dn5x49eixatEi8gp1YYGBgQkICj8dzdXUVH/Ty8tq2bRuhgH+/GTNmHD16dOXKlfPmzZNKIAAA+GTi7d8jBrfkX10AbDkefy5q31Ei4vF461at0tLSYjvR/6isrM7J/js768mTnF71VUwAACAASURBVHxVNZXKiqrKysply5Z91E3UNDS4SkoamppvviQUCqsrK/k1NXW1te+5Q3l5+aNHjx49eiQ+oqSkZGlpaW1t3aFDh06dOtnY2Kiqqn5UKgBovtq2bVt/Os/7+fn5+fn5ffC0f/3rX2/ulDl8+PDhw4d/dL7mRtIC3svLa9WqVbNnz+bz+d99950O2zugAgAorMLCwpMnTxKRlrZmn35ebMcBaGn+fvJs8by1THvm9OlOjo5VVZXsRhIKKe9Jfua97IeZjx89zHmW/5zZ1u5dlJSU9I2M9I2MDIyMDIyMtPX0tHR1dfT0NXV0NLW1VdXV1DUb9ZGEUFBXXVVdVVFe9upV+evX5aWvX70sev3qVXHB8+KiF8UvXlT+7yi9QCB4/Pjx48ePmZ9RPB6vQ4cO9vb2Xbp0cXR01NXVleSLAACgUCQt4OPj421tbceOHbtkyZINGzZ4eHi0b9/eyMjorY83ENH8+fMl7BEAAN4qMjKSWfQluL+fmjpGtwCkqba2bua0H8rLKojIy9Nz3JdfCoVCVpIIBaLsrNw7tx/cz3h0/152ZcU7P0TgKisLlXikrNwvLLSTnb1RazODVkYcaTzzyFXiaWhpaWhpGZo0nNfKqCwrK8zPf5aX9/zvv589yc3PySkuLBS/WldXl5mZmZmZGRMTw+FwrKysnJ2du3fv7ujoqK6uLnk8AIAWTNICPjQ0VNwuKSk5evTo+89HAQ8AICP79+9nGoOHBLGbBKDlWbdy+53bmURkZGS0ZsWKdw1UyE7Bsxe3btxLv3nvTvrDdxXtrcyMzTu0Ne9g2bpdG7O25qmp6SePJxNRFY9n27WrPNNqaGu3s7VtZ2srPlJZVpaXnZX3KCsnMzPrXkZpcTFzXCQSZWdnZ2dnx8TEKCsrOzk5ubm5ubu7m5iYyDMwAEBzIWkB39Se/gIAUEyXL19+8OABEXW0sbJz6MR2HIAW5eypi3/ujCEiLpe7fOlSQwMD+fRbW1t37+6ja1fTb6TdLXhW+OYJKqoqljbt29l2sOhoZdHJSkPrf5Z/7+HmdPrkBaFAcDn18oAB4fXXcJY/DW1tGydnGydn5p8vnz/Pzsh4cCf94e3bhU+fMgdra2vT0tLS0tI2bdrUvn17Ly8vLy8vS0tL9lIDADQ5khbwH1yJFAAA5GDPnj1MY/CQYHaTALQwz54WLpi9mlmBacLYr1xdXGTdY2Vl9fW0O1cv3bxx7S6zX119yioqVp2tOzjaWtlZt+nQlqvEfdd9dHQ1O9tb372dWV1dfe1amoeH57vOlD9DExNDE5Mevr5EVFJYeP/mzXs3rmfcuCF+eJ4Zlt+1a5eVlZWvr6+vr6+ZmRmrkQEAmgRJC3gAAGDd69evY2NjiUhVVSU0PIDtOAAtR11d3cypy0pflRFRj+7dvx47VnZ9VZRXXU29lXrhevrt+7W1dQ1eNWvXplNXu05Odm1t2vNUGrslW89ezndvZxJRSsr5JlXA16dvbOweGOgeGCgU1GVn3Eu/cvnW5cuF+fnMq8zqdzt37rS3tw8MDAwLC+Px8OcrACguWf0EFAqFXO47PxIGAAApOnjwYFVVFREF9PXU1dNmOw5Ay7F2xfZbN+4RkaGBwYplS2Xxt011Nf/aldsXktNu3choULerqKhYO3e27eZg081e11D/E27esZOVYSuDl0XFT3Jzn+TmWrZtK6XUMsFV4ll36WLdpUvE2HFPc3KuXzh/PSWlIC+PiEQi0Z07d+7cubN582Zvb+9+/fo5OjqynRcAgAVSK+Dz8/O3bdt24cKFu3fvlpWVVVRUMJPNli9fPmDAADs7O2l1BAAADfx3/vzQEHaTALQkp0+c/++j78uWtmrVSoo3Fwrpzu37586kXkm9WVPNr/+SuqaGbXcHB7euHZ3tVVQbO9j+VhwO9XBzPH40iYhSzqcMb9oFfH2t27Vr3a5d6PAR+Y8fXz137mrS2ZIXL4iopqYmMTExMTGxbdu2YWFhffr00XzblvUAAC2VdAr4NWvWLFq0qLLyLWuibtu2bd68ed9+++3PP/8s/yVbAQBavFu3bt2+fZuI2rYzd+mJISkA6ch78nTB92uY0YiJ48b17NFDWnd+ml+YdOpictLV4pcl9Y+raajbuTo59epu7dRZiSeFzd4YLq5OJ4+nCOoEV6+mDRo4SK257dNmbmVlbmUVPmrU/du3Uk+fvnnhAr+mhohyc3M3bdr0+++/9+nTJyIiwsLCgu2kAADyIIUCfuXKlXPmzGHaDg4Ojo6O+/btE7+qq6tLROvWraupqdm8ebPk3QEAQH1//vkn0xj4eRA+JwWQipoa/owpy8pelxORe0/X8V99Kfk9+TW1qRevnz5x4f69LOZzAYayikpnly5Oni42Xe0b/3B742lpq9s72ty+nsGvqbl85bK3t4/Uu5AHLtfWuautc1fujJnJCQlJR+Oe5uYSUVVV1ZEjR44ePerq6jpo0KCu8t0tDwBA/iQt4DMzM+fOnUtEHTp02L59u6+vLxHVL+AvXLjw3Xffbdu2bevWrd988429vb2EPQIAgFhFRUVMTAwRKSvzwgf3ZTsOQAuxfMmmjDsPicjYyGj5Ukkffc978uzksZTkpCv192/ncDhtbdt383Zz7NVdTVO2o+Juvbrevp5BRCkp55trAf//NLS0/AYM8AwOfnT3TvLRozcuXBAIBEKhMDU1NTU11dra+osvvvD09MRKTNCC7dwelRB3Vs6dVldVy7lHeBdJC/h169aJRCItLa1jx4517NjxzRM0NTW3bt16+/bt1NTUtWvX/v777xL2CAAAYjExMcx2nn59PAxbfcoaVwDQwOHoE9H7E4iIx+Ot+vFHff1P/M6qqxNcTb114lhyRvqD+sd1DPRcfN27+7kbmhpJIW4jtO9gYWpqVFDw4ml+flbWow4drOXTr0xZ2ztY2zuUvnyZdDTu/PHjFa9fE9GjR4+WLl1qaWk5dOhQf39/JSWpPYkA0HRkPczNepjLdgpgjaQF/IULF4ho/Pjxb63eGRwOZ9SoUampqcxTmgAAIC3i+fODh2L7dwApyLyXtezfG5j29CmTuzo7fcJNSl+Vnzpx/uSxc8XFpeKDXC7XtrtDjz6eNs4OXCV5P+3i2sv5SMxJIko+l9wyCniGrqHhgNFjgocOTT11+vThQ4VPnxLRkydPVq1atWfPnuHDhwcEBKCMh5Zh3LhxERERbKcgDQ0NtiMoOkkL+MePHxNRz549339au3btiOjBgwfvPw0AABovPT39xo0bRNTG0qxnr25sxwFo9srLKr79ZmlNdQ0RBfj5jhg27GPvkJuTnxB79nzylfobwukY6LoG9HYN8NAx1JNm3I/RzaXL8fgkfk3tjRs3Pisr09JuUftNKquq9Q4J8ezX79r58ycOROU/fkxET58+Xb169b59+0aPHu3j44NJ9dDc6erqMouLgYKTtIBXVlYmotevX7//tMLCQiLC6koAAFK0e/dupjFoSDCXix+wABIRiUTzZq58kptPRG0tLZcuXNj4v1tEIrp1PSPu0Mn025n1j7e37+Qe5GPv6shlexBYTV3FuavdldRbdXV1Fy5e6Nu3H7t5ZIGjpOTi7e3i5XX7yuWEffuePHpERPn5+T/99NNff/01ZswYDw+P/2PvvuOp7P8/gL/PtI+9VyqhzCijaG932nuqpKGppWQlGWl3d1caqvtuioaGJGmTCpVEREL2wcHhnPP74+r27Sd1N3AdvJ9/3I/rus51zud13HK8fRbZGRH6dfv27XN3dyc7BYwYMeLUqVNkp+jQfreA19XVjY+Pj46OXrBgwXdue/LkCQBot53dRxFCSMg1LF9Hp9PHjMfl6xD6XYcPnL4d9QAAxMTEgv23/eDu4vX1vHux8VfCb2W//9hwkc5kmNn26jNqkIq2WkvF/XlWfc2fPHoBAHF344YOGUpprz3SFIqxpZVxb8uU+CeXT5zMeZcBAJmZmR4eHoaGhgsXLjQwMCA7IkK/oqamprS09L/va2GVlZVkR+jofreAHzt2bHx8/JkzZyZNmjRu3Lgm70lNTT18+DAADB/eDv/cixBCpDh//jyxfN2goX0UleTIjoNQ2/bwfuKe4KPEsfuG9V27dPnPp9TU1F6NuH05PPrL7dylZFjWw/tZDrWVkBa6Mepq6kpandSzs3JLSkpSUlKMjI3JTtSSKBTD3paGvXo/e3D/8okT+Tk5AJCSkrJs2TI7O7v58+erqqqSHRGhXyQjLy8uKdnKjdbX1RFrTCDS/W4Bv3Tp0j179uTl5U2aNGnZsmWrVq3S0NBoeJTH4507d2758uVcLldMTMzFxeU3m0MIIUQIDQ0lDiZOsyc3CUJt3cfcgrXLfPk8PgBMnTRp1H/1N1RWVl2+GBVx4Qab/b+eKCVNVbs/Bpva9aYzfveXq5Zj3bdndlYuANyNu9vOC3gChWLWp6+plfWj27cvnwgtKy4WCASxsbEPHz4cP378tGnTxMRadgM/hFrCgg1uf8yY0cqN5mRkTO9j08qNoib97meMlJRURETE4MGD2Wz2jh07duzY0VDA9+zZ8/Xr1zU1NQBAoVAOHjz4ZW2PEELolyUmJhL7emh3Ure0NiU7DkJtWG0td8Uir9LScgAwNTZ2XbH8OzezyysjL92+fjWWw6luuKjTvaudw1B9c0PhX+vHyET/Snh0VSXn9avXRYWFCoqttI8duSg0mvWQIRZ2ttHh4TfPn6/hcLhc7j///HPz5k0nJ6eBAwcK//84hBBq0AzTn3r16pWYmDh48GDi9MOHD8TBs2fPiOpdTU3t8uXLM1r9D0UIIdReHT36eazvpOl/4K+eCP0OH/ddr1LSAEBBQSHIz49Ob7pvo7ys8uSxi0sWbAo7d52o3ikUil5PQ+ctrgt9VhtYGLWJf4l0Os3C0hgA+Hx+XFwc2XFaFUNEdPjkKV6HDvcZPpxYkb64uNjPz2/lypUZGRlkp0MIoR/VPKO8unTpEhUVlZKScuXKlSdPnhQUFHA4HGlp6W7dug0aNGjs2LFMJrNZGkIIIVReXh4REQEAIqIiDuOGkh0HoTbszKnL4edvAACdTg/aulVRUeHre9jllRFhUTevxdbWcIkrFCrVxMZi8CR7BfW214Nt3cc8LuYJn8+//+CBvb09o4P9hiYlIzPdZVm/kaPO/nUg/eVLAEhJSVm8eLGDg8OcOXNwg2uEkPBrzmlahoaGhoaGzfiCCCGEvnbq1Knq6moAGD6qn4wsi+w4CLVVzxNfbvPeRxy7rlhuZmrS6IbKCs7li1HXrtypqaklrlCpVNO+vYZPH6uiqcbj82pra1o1cXOQkZXqZtA59WU6p6rq6dOnVtbWZCcigUaXLqv8AxJiYy+EHC4vKeHxeGFhYffu3VuyZAluNYcQEnLCu84KQgihrwkEgobl6yZN+4PcMAi1XZ8Kilcu9qqrqwcA+5Ejpk6a9OWjHE7N1YjoqxG3G+a6U6lUU9teAyeMVFRXERNt2yufWdv0TH2ZDgB3YmM7ZgEPAEChWPTvb9i799W/T8VcusTn8T59+uTh4WFjY7N8+XJ5eXmy8yGEUNN+roB3d3f/ncasra1Hjhz5O6+AEEIdXGxsLDFd06BHVxMz3M0YoV/B5datXOxZ+KkEAAz09TZv2PC/h2rrbkTGhl+4UcGuIq5QqBTTvr0GTRyloKZETtzmpqvfWU5BtqSoNPv9+8x373Q6dyY7EWlExcXHz19gOWDgP/v3ZaamAsCDBw+SkpIWLlw4fPjwNrGuAUKoo/m5An7Lli2/09iKFSuwgEcIod9x5MgR4mDqTAdykyDUdm3ZvPvFs9cAICsruyMgQEREBAB4PP6d6Ifn/ols2NedQqEYWpkNnmyvrNmu9gynUsGqj1lkxG0AiI2N7cgFPEGjS5c1gUFx165dPHa0hsOprKzcvn17TEzM6tWrlZWVyU6HEEL/z88V8AoKTSzuAgBsNpvL5f7vRel0Go1WW1vbcKV79+7GxsZmZma/lhIhhBAAfPjw4ebNmwAgxZIcYT+A7DgItUn/nIgIO3sNAGg0WtDWraoqKgIBPHn0/J8TER8/FDTcpm9uNHTKH2qdNclL2oJ69Ta5dT2OW1uXmJg4btw4lrQ02YnIRqXajhplZGn5z769yU+eAEBiYuKCBQucnJxGjRqFXfEIIeHxc9vIFTbl0KFDNBoNACwsLM6cOZOVlVVTU8PhcHJyciIiIvr27QsA7969mzRp0qxZs1rkTSCEUMcQGhrK4/EAYMyEYWLiomTHQajtSXic5O/zJ3G8ZuUKC/Oeqa8y3NcFbfc72FC9dzLo4rzFdY7b4vZavQOAmLiIqVl3AKivr3/w4AHZcYSFjILCIg/Pua5rJFksAOBwODt37nRzcysuLiY7GkIIffa7+8CnpaVNmzaturp6/fr18fHxkyZN0tbWptFoVCpVQ0Nj9OjRcXFxvr6+NTU106dPf/XqVbOERgihDojL5Z44cQIAKBTK5Om4fB1CP+1jbsHKJV719fUA4GBv38+mX6Dvgc3rt6elviNuUNFSm+O22HmLayeDLqQmbQ3WthbEQVxcHJ/HIzeMUOk1YID7nwdMbWyI0/j4+AULFsTExJCbCiGECL9bwAcFBVVXV9vZ2W3duvVb97i5uQ0ZMqS6ujooKOg3m0MIoQ7r4sWLRUVFANDHzqKTjgbZcRBqY2qqa5ct3FxaUg4A3Q0MNOS7rHbZEv84iXhURkFuksvs5ds36ZsbkRqz9aiqKXbuogUApaWlL168IDuOcJGSkXHauMlxzVpxSUkAYLPZvr6+vr6+VVVVZEdDqO2JjY2lUCgUCkVeXr6uru5bt505c4a4zcCg2dboJV5zwYIFzfWCwuB3C3hiNuZ/zg4i1q67devWbzaHEEIdVkhICHEwbdYYcpMg1OYIBIKNawNSX2UAgJQki8qRunXjHjEhRUxCfMTMca57PHv2t6JQO9ZUZ2tbc+IAu5ebZNG//6Z9+7v37EmcxsTEODk54XhShH5ZSUlJVFTUtx49e/ZsSwdYtGhR8/6BgBS/W8Dn5eUBgKrqf6zOqqioCAAFBQXfvw0hhFCTnj59+uzZMwDQ0FLt26832XEQamMO7vv7xtVYAKBSqXIianW1PACg0mg2Iweu2efdb8wQOpNBdkYS9DDsxpJhAUB6enpu7gey4wgjGQWFpd4+kxctZjKZAFBQULBy5coTJ07w+XyyoyHUxlCpVAA4c+ZMk49WVlZeu3aNuAd93+9+jVgsFgAkJCR8/7b4+HgAkJOT+83mEEKoYzp8+DBxMGXGaGoH6yRE6Dfdjnqwd8cx4lhJSkuUIQEAPSxNV+3yGD1voriUBJnhSEWlUaxsPu8QFHMbO+G/gULpZ2+/bucudR0dAODxeMePH3d1dS0sLCQ7GUJtiYGBgYKCQnh4eE1NzdePXr58mZiX3aIZxo8fHxQUtHbt2hZtpaX9bgFP7Ax36tSprKysb92TlZV18uRJAOj57xgkhBBCP66goODSpUsAICYuOm7icLLjINSWxD96sWqJl0AgAAA5CWVpMXn1zloLvVfNXLtQQVWR7HTk621lSqfTACA+Pr6qqpLsOMJLVVt7XfCOAaNHE5NGk5KSFi5c+OTJE7JzIdRmMBiM8ePHs9ns69evf/0oMX5+0qRJTT63oKDAxcXF0tJSUlKyS5cuc+bMSUlJ+fq20NDQkSNHKioqysvLjxw58ustNgoKClxdXRMTE4lTX19fCoXi7+/f6LaZM2dSKJSrV68Sp8RE+l27dmVnZ0+bNk1BQUFMTMzCwuLKlSsAkJWVNXXqVEVFRQkJCTMzM6LsbVG/W8DPnTsXAIqLi+3t7Z8+ffr1DYmJiaNHjya235gzZ85vNocQQh3QsWPHuFwuAPwxdghLWorsOAi1DfX19X/uOTFvhmt9PQ8AJERYnbW6T1gya6n/ep0eumSnExaSUmLGZt0BoK6uLi7uHtlxhBqdyZy40NnZ3V2CxQIANpu9cePGkJAQHq7hj9CPmTx5MgCcPn260fWKiorr16937dqV6Btu5O7du6ampnv37k1MTFRWVs7Ozj5+/Li5ufmXc+b5fP7cuXNnz5597do1NptNpVKvXbtma2sbHh7eXOHT09N79eoFAB4eHhMnTnz69KmDg8O+ffssLCzYbPamTZtmz56dnJw8c+bMy5cvN1ejTfrdAn7KlClDhw4FgJcvX1pYWAwfPnzz5s3Hjh07duzY5s2bR4wYYW5unpycDAAjR46cOHFiM0RGCKGOhMvlHj9+HAAoFMq0WQ5kx0GobXgQlzB2xIJ9O47z+QIAEGGITZqxYM1eb4uB1h1tpbr/1Offpezu4X5yP8DI0mrjnr26hoYAIBAI/vnnH1dXV2KLEITQ9/Xr109FReXy5cuNNnSIiIioqamZPHny18uiczicKVOm5Ofnu7q6lpeXZ2RksNlsNzc3Lpc7a9asd+8+bwJ66tSpY8eOMZnMv/76i81mFxYWPnv2rHPnzl//seCX7d27193d/e+//3ZxcQkNDXV2dubz+UuXLp0zZ87Vq1eXL1++f/9+T09PADh27FhzNdqk3y3gKRRKWFiYg8Pn3ylv3Ljh4+Mzd+7cuXPn+vj4NAyQmDBhwrlz536zLYQQ6oDCw8OJmZaW1qZddTuRHQchYZebk7/c2cNp9vrMjBziCoMh4rFjz2jHyUxREXKzCSd1TRVtHXUAKCkpwf3kfoSMgsLyrVuHTZpEFBvJycnOzs7Pnz8nOxdCwo5KpU6cOJHD4RCDzxsQfelE/3wjwcHBeXl5U6dODQwMFBcXBwAxMTFfX19HR8fa2trAwEAA4PP5Hh4eABAUFOTk5CQiIgIApqamN2/epNPpzRW+S5cuS5YsaTglNlljsVg+Pj4NF0eNGgUAGRkZzdVok5phoT8JCYnw8PCIiIjhw4crKSl9+RCLxXJwcLh+/fq5c+eIrzhCCKGf8tdffxEH0+eMJTcJQkKutqZ2/67Q0cPmRd+833CRSqVtDAwyMDMmMZjw62PXizjA/eR+EJVGd5g9Z7GnlySLBQBlZWXr1q07e/YssdoCQuhbiCr9y7Xoy8vLb9y4oa+vb2Rk9PX9RH+wi4tLo+tOTk4AEBsbCwDZ2dmZmZmysrKN9nvX0dEZM6bZdt41Nzf/coAAsTq7oaGhmJhYo4stPa2m2f4mMXr06NGjRwMAm81OT0/n8XhycnI6Ojq4GQBCCP2yBw8eJCUlAYCWtnq/gdZkx0FIeMXceuC/5c8P2XmNrjsuX2nSy5KUSG2IoZGetAyrvIydnp6e/f69lrY22Ynahh4WFm679xzc6puVlsbj8Q4ePJiamrpmzZovf6FHCH3JxsZGU1MzMjKSzWYT25lFRERwudwmu98BIC0tDQAOHToUGhr65fXq6moA+PDhAwCkp6cDgIGBgaioaKOnm5mZnT9/vlmSE2l/5GJLa7YCvgGLxcLV5hFCqFkcPHiQOJg+ZyzuHodQkz5k5/l574u9/Yg4pVAoNBqtvr4eAIaNGTdqQtNrGqMvUWkU6749r1+5AwB37tyZNXs22YnaDBlFxVUBgef++ivuWiQA3L17Nycnx8vLS01NjexoCAkjCoUyadKk7du3R0REzJw5E767/jyXyyVmER49erTJV6usrOTz+QUFBQCgqqr69Q3t8l8ido8jhJCQys7OJkaOSUpJjJkwlOw4CAkdLrfuwJ6TDsPnN1TvWnqdZRTliOrdyNxiwUpXUgO2Jb2tTBgMBgAkJCRUsNlkx2lL6AzG1KVLZ65YyWAyASAzM3PJkiUJCQlk50JISE2ZMgX+XYu+rKwsKirK0NCwe/fuX9/JZDLl5eUBoKKiQtAUPp9PpVI1NTUBID8//+tXIGr7n1VSUvILz2o1zd8D35ExmUyyIwAA0Gg0ACDWb0CtqWFijIiICM6Ca2VUKpVKpbazb/vDhw8T06gmTrWXlZUhO873UCgUIfkB2KFQKBQqldphv/L3YuO9N+3IyvxAnEpKS01eNvd+ZEz2m3cAoKaptd4vkCnSeDhlM6JSqTRa+/k9SoolZWZh+OThs/r6+vv37/8x+g+yE30TlUZjgNB9yNqNHKGt23W/p2dJYWFFRYWbm5uzszNRqLQDDVNiW+1nTsPaYzQarZ19uCMLC4vOnTtHRUWVlJRcunTpO+PnAaBLly7FxcWvX78mtnBrQHTOi4iIKCgo6OjoAMCrV69qa2sbfbcQu6F939e/tL99+/Yn3k+raz8fPMJAUlLy680PyCIlhZtFk0ZSUpLsCB1Ue/q2Z7PZp06dAgAajbpg0XQhXweUQqEIecL2ikqlNuMSu21FQX6h18Ydly7eJE4pFMrgiaPmurkc8d2d+jQZACRZLJ9d+xUUlL77Mr+LAhQRZruqKwYP7Rv/6LlAILgTGztm7FgGQ0i/tRgMunBm0zMy8jkcsmeze+qLF3w+f//+/e/fv9+4cWN7+isbnU5vnZ85DWUYk8lsTx/uiDB58mQ/P7+wsLALFy4AwHf2Gu/fv/+TJ0/27t1LbKnbwM/Pz9PTc+3atf7+/hoaGnp6em/evDl06NDSpUsb7snNzf3+PmjEN/P79++/vHj37l0s4DuQ0tJSYeh3FRMTExMTE/KxH+2StLQ08YOgpKREGL4TOhRJSUkajVZeXk52kGazZ88eNpsNAENH2EnLSAjtW2OxWBQKhc/nV1RUkJ2lw5GUlOTxeMRCPh0Ej8c/fTJid9CRykoOcUWrW+f5Hsv1zY3CD/5z69xVAKDR6Wt8tskpK3OqOS2RgUKhiImKAQCPz6utrW2JJsjCkpHsqqv9Ni2LXV5+584da2srshM1QUJCso7L5dZxyQ7SNCqDvtRny9kDf96NjASAq1evZmZment7y8rKkh3tt9DpdAkJCQDgcrmt8zOHU/X53291dXVxcXErZTWY9gAAIABJREFUtPh9xEBu1FymTJni5+f3119/vXjxwtTUVE9P71t3rlu37uDBgydOnNDR0XFzc2MymQKB4MyZM35+fkwm09HREQAoFIqHh8e0adNcXV0lJCRmzJjBYDBSU1MnTJhAzKj6Fn19fQA4efLkwoULTU1NASA1NXXevHkiIiLC/OMdC/jmREzGIDvFZ8KTpAMSqu+EjqM9fdnr6uoOHz5MHM90HN8m3lebCNkudZyv/MvkNK+NO16lfO4YERUXm+gye+SscTQa7XHU3X92fv4ns2Clq7GFBbTgEOsvh9q1ty++jV2vt2lZABBz+7aVlTAW8PD5pz3ZIb6NSqNNWbJUXUfn7IEDPB7v1atXS5Ys2bJlCzHKt4368iveOl99AbR2i6g1GRsb6+vrE0tFNLl8XQM5Obljx47NmjXLy8tr27ZtnTt3LioqKiwspFKpJ06caKj8J0+eHB0dHRIS4ujouHjxYllZ2by8PBqNFhAQ4Or6zcVQhg0bZmxsnJSU1KtXLwsLi5qamuTkZD09PVdXV19f3+Z9y80IF7FDCCGhc+nSpdzcXAAwM+9hbGpAdhyESFZVxfHz3jd13NKG6r33kL47rh79Y+5EGo2WkfJm7xo/AV8AAKMmTh42ZhypYds2PYMuCopyAJCTk5Mu3INIhZztyFHLtvhKsFgAUFBQsHz58sePH5MdCiEh0rBCxPcLeABwcHB4/vy5o6OjgYHB+/fvpaWlp06dmpSUNG3atIZ7qFTq4cOHQ0NDR4wYwWKxqqqqBg0adPv27dmzZ0+ZMqXR/PkGoqKiUVFRixcv1tLSevbsWUlJiYuLy8OHD62srKZMmfJrK9irq6tbW7fsvr8U/JtWMyouLhaGr6e4uLiYmJgwDDfqaGRkZIgh9ELyndChSElJUalUoR1n/rMGDRpEbP++64DXoKF9yI7zPdLS0sQQejYuW93qpKSkeDweh9MiA8WFR/SNe75eez/lFxGniuoq89xdevb/3DlcnF/oNnFJaWExAJhb93EL2N6w2lYLoVCoXwyhr2nRtkjx4F7ipQs3AcDYxMTZ2ZnsOI1JSkpya2u5dXVkB/khhR8/7vfyLPjwAQCoVOqiRYvGjh1LdqhfQafTifV9uFxu6/zMuR59ZcHymQCwYsWKjRs3tkKL36egoEB2BNi+fTvRmbwmaPsfM2a0cus5GRnT+9gAwOjRoyMiIlq5dfQl7IFHCCHhEhcXR1Tv2p3UBwy2ITsOQqQpyC9ctnDz8kWeRPVOo9FGz5scfPVIQ/Vew6n2X7SJqN61u3Rd5bWlpav3jsCil7G4pBgApCQnFxUWkh2nbVNUU1uzPVjPxBQA+Hz+vn379u3bx+fzyc6FEGrD8HMOIYSEy/79+4mDmY7jqVRh2dgCodbE5/FPHbs4esi821EPiCu6Jt23hR2YscZJRPTz2tQCvmC369as1+kAICMvvzFgu7iEBGmJ2xGmCN2itwkA8Pn827dvkx2nzROXlFzq491n+HDi9OLFi15eXsK8PhZCSMhhAY8QQkLk9evX0dHRACArKz1m/DCy4yBEgjevM6ZPWObnva+qigMA4lIS8zYv2/LPbm29zl/ediLwQMLtBwDAFBHZsC1QUUWVnLjtkY2tOTGW4eGjh9XtfY5GK6DR6NNdlo2ZO5fYbPj+/fuurq7tZs4XQqiVYQGPEEJCZPfu3cQCCtNmjxEVa1dbTCP0n2prancGHJ7ssCT5RSpxxWqY3Y6rR4dNc6D8/9EoUacvXzl6HgAoFMryTZ7duhuSELf9kpGRMjI1AIDamtp79+6RHaedGDph4rx16xhMJgC8fv3axcWFWKwUIYR+ChbwCCEkLHJzc4mFYUTFRKbMHE12HIRa1eOHz8aOcDp84DSxba+8quK6P7es2uUhq9R4++XncfFHfPYQx9OcnG0GDmrtrB2AXf/PizbHxMTweN/bSBn9uJ62dsu2+IpLSQHAx48fly1blpqaSnYohFAbgwU8QggJi3379tXV1QHA+MkjZWWlyY6DUCthl1e4rwuaP2Nt9vtcAKBQKSNmjA2+csR8QBM78WSnZe5Y6c3j8QBg4Ej7CbPmtnbcjkFdU6VTZw0AKCsre5b4jOw47UeXHj3WBAXJKysDQHl5uaura3x8PNmhEEJtCRbwCCEkFIqLi0+ePAkAdDp99rwJZMdBqJXcvHZ39NB5F89dJyaPaHXT2fLP7rmblopJiH99c1lRybaFbtWVHAAwNDNftHZDa8ftSGz79SYObkVHk5uknVHW0FyzPVijc2cAqKmpcXd3v3nzJtmhEEJtBhbwCCEkFA4ePFhdXQ0AI0cPUFNXJjsOQi3uU0HxcmePVUu8iwpLAIAhwpyywtE/7ICuSfcm7+fW1Pov2lSU9wkA1LW01231pzMYrZq4gzEw7CavIAcA2e/fp799S3acdoUlK7vKP0DP2AQA6uvrAwMDz549S3YohFDbgAU8QgiRj8PhHDt2DAAoFMpcp8lkx0GoZQkEgvOnI0cPdYy+eZ+4YmBhFBh+cJzzdBqd3vRT+II9a7dlJL8BAJa0zMbAYEkWq/USd0hUKvSxMyeOsRO+2YmKiy/18bGwswMAgUBw8ODBQ4cOEeNQEELoO5r+mEQIIdSajh49WlJSAgD9B1nrdutEdhyEWlD2+1yPDcHxj14Qp2KS4jNcnQZPtid22PqWU9sPPb55FwAYTOb6bQGqGpqtkbXDs+htfOtGHKeqJiU5+VNBgZIyDg5qTjQ63XHNWikZmZhLlwDgzJkzFRUVK1asIPbwQwihJmEBjxBCJKutrf3zzz+J4/nOU8gNg1DL4fP4oUcv7N1xrKa6lrhi3t9qvucKeRXF7z/x1tkrl0LOAACFQnFxczcwNm3xrAgAAJgiDEtrs5hbD/l8fnR09NRp08hO1O5QqRMXOkuypC+fPAEAkZGRlZWVGzZsYOD0EPRtl0JDE2JjW7lRTlVlK7eIvgULeIQQItmpU6cKCgoAwNLazKRn07N/EWrr3r7JdF8XlJL0hjhlycnM3bikz6iB//nEF/fiQ7x2E8dT5y+0HTKsBVOir9jY9boXG19XV//o0SN7e3spnLnQAkZMnSrBkjpz4ICAz7979y6Hw/H09BQVFSU7FxJSb5JevEl6QXYKRBos4BFCiEx1dXV79+4ljucvnkpuGIRaQl1d/aH9fx/a/3dd3eftxG1HD56zYbHUD+yV+CH9/c5VWxo2jZs4x7Fls6KvSEmJG5t1f/okqa6u7m7c3VGj7MlO1D7ZjbIXl5A8Hrydx+MlJCSsW7du69atEhISZOdCQmTChAkmJiZkpwBFxf8YM4VaGhbwCCFEpvPnz+fk5ACAkYm+dZ+eZMdBqJmlJL1xXxf09k0mcSqvqujkudKsn+WPPLe0sHjrgvVV7EoAMOqJm8aRxrZfr8T4ZIFAcDf27rChw3Dx/xZi0b+/qLjYYT8/Lpf78uVLV1dXf39/Fg55QP/S1tbW1tYmOwUiHxbwCCFEmvr6+h07dhDHC5dOJzcMQs2rtqZ2787jx0PO83l8AKBQKIMn289wdRKTbGKD9yaeXl3j7/zvpnHandZuDcC6kSwqaoq6ejppqe8qKioePXrU19aW7ETtlmFvy6XePvu9vWo4nLdv365atSogIEBOTo7sXEgohISEbN26lewUMGjQoIMHD5KdokPDAh4hhEhz4cKFzMxMANDv3qXfQCuy4yDUbJ7GJ3us356V+YE4VdFSd/Zd3b3Xjw7+5PP4u123vnuZBgAsGVn3oB2SUlItlRX9ALsBlmmp7wDg1q1bffr0oeAy6S2mq5HRsi2+eze7cyors7KyVq5cGRgYqKSkRHYuRL6ysrJ3796RnQIMDQ3JjtDR4c9fhBAiB4/H27lzJ3G8yGXm9/fQQqit4HCqfT32zJ26iqjeqTSq/dwJgZcO/Xj1DgAnAv+Kj74PAAwmc8O2QGU19ZaKi35M127aahrKAPDp06ek5CSy47RznfT0Vm7zZ8nIAEBubu7KlSvz8vLIDoUQEhbYA48QQuQIDw9PT08HgG76nQcO7UN2HISawaMHzzw2bM/NySdONbt2cvZ11TUx+KkXufF3xNVj5wGAQqEsd/fUNzJu/qDo59n1tzp9MgIAom5GmZjgTn4tS11HZ6V/wO6NbqVFRQUFBatWrQoKClJXx79kIQCA7Vv2zpg0t5Ubzch6azMU/+ELBeyBRwghEvD5/ODgYOJ40TLsfkdtXmVFladb8IKZa4nqnUanj3Oevi3swM9W789iHx/1/bwvw4yFi/sMHNz8WdEvMTbVl5WTBoB37969y8ggO077p6yhsXKbv5ySEgAUFhauWrUqOzub7FAIIfJhAY8QQiSIiIhIS0sDgK66nQYN7Ut2HIR+y907jx2GzT9/OlIgEACATveuW8/um7LCkcH8uWXn3qdm7Fy1hVj0brD96HEzZ7dIXPRLqDRKHzsL4vhmVBS5YToIBVXVVf7+CioqAFBcXLx69eqsrCyyQyGESIYFPEIItTYej7d9+3bieNHymVQqdr+jtqq8rMLN1X+x48aC/EIAYDAZk5fP3Xp2n073rj/7UqWfirc5b6yu4gCAsUUv5zXrmz8u+j29rUzFxEUBICU5OT8/n+w4HYKckvIq/wAlNTUAKC0tdXV1JZY+RQh1WFjAI4RQawsLC3vz5g0A6HbrNGS4HdlxEPpF0TfvOwybdynsc2esronBtrAD4xfNoNF/eoWd2uqabc4bi/MLAUCjU6e1vtt+4UVQS2OKMKxsegIAn8+/hZ3wrUVGQWHlNn9lDQ0AKCsrwxoeoQ4OC3iEEGpV9fX1QUFBxPHiFbOx+x21RaWl5WuXb13u7FFUWAIATFGRmWsX+vy9W7Nrp194NT6Pv3PVlsxXbwFARk5uU9BOCUncNE5I2dhZMBh0AHjy5El5eRnZcToKaXn5ldv8VTQ1AaC8vBxreIQ6MizgEUKoVZ09e5bYx9WgR9fBw3D2O2p7rl+NdRg6L/LybeJUr6dhwMW//nCcRKX94i8Vof5/Po15CABMEZEN/kHKqmrNlhU1NykpcTMLQwCor6+PuR1DdpwOhCUru3Kbv6qWFmANj1DHhgU8Qgi1Hi6X29D9vmTFbFx8HrUtxUWlKxd7ubr4lBSXAYCImOgctyXeJ3eq6Wj+8mtePxUeGRoGABQqdfkmz27dDZstLmoZtv0tiaFDcXFxNdXVZMfpQKRkZFb4bcMaHqEODgt4hBBqPaGhoTk5OQBgYmbQf5A12XEQ+glXL912GDYv6noccdqjt0lQxKGRs8ZRfmMayLPYx8e27iOOZzgtshk4qBmCohamqCTb3agbAFRXV8fFxZEdp2NpVMOvWbMG95ZDqKPBAh4hhFpJdXX1jh07iGOX1Y7khkHoxxV+KnFx2rxuxdayUjYAiIqLzdu8bPPx7cpavzXW/f2bdw2bxg3CTePalH4DrIiD6Nu36+vryQ3T0RA1PDEfnljT7sOHD2SHQgi1HizgEUKolRw8ePDTp08AYGltZmVjRnYchH5IxIWbDsPmxdx6QJwa2fTcfvnwsGkOvzkBpLSweJuzG7FpnJG5xSLcNK5N0dRW7aKrDQDs8vJHjx6RHafDkZKRWbHVT0ldHQBKSkpcXV0/fvxIdiiEUCvBAh4hhFpDWVnZ3r17ieNlrtj9jtqAgvzCxY4bN64JYJdXAICYpPhC71WbQgIU1VV+85Vra2oDFrkX5xUCgLp2p7W+/rhpXJvTb+DnTvhbUVECPp/cMB0QS05uxVY/RVVVACgqKnJ1dS0oKCA7FEJNe/z4MYVCoVAogYGB37pHVVWVQqEQm+yi78MCHiGEWsPevXvLysoAYNDQPiZmBmTHQeh7BALBhTORDsPm373zmLhiatsr+PKRQZNG/f7KiwK+YM+arRkpbwCAJSPrHrRDUgo3jWt7uunrqGkoA8CnT5+ePXtGdpyOSEZBYcVWP3klJQD49OmTq6trUVER2aEQ+h4PD4/09HSyU7R5WMAjhFCLy8/PP3jwIABQaVSX1XPJjoPQ93zMLXCavd5jQ3BlRRUASLAkF/utdTu0TV5VsVle/+/gQ0+i7gEAg8nc4BegrKbeLC+LWl//fzvhb968SW6SDktWSWnFtm2yiooAkJeXt2bNmtLSUrJDIfRN1dXVTk5OZKdo87CARwihFufv719dXQ0A9g6Duup2IjsOQk0TCARnT10eO3zBw3tPiSvm/a2CrxzpP3ZYczVx+/y1iMNnAIBCoSzd4K5vbNJcr4xan6GxvryCHABkZ2e/fvWK7DgdlLyyynJfX5asLADk5OSsW7eOzWaTHQqhJqiqqiopKcXExBw+fJjsLG0bFvAIIdSyUlNT//nnHwAQEWEuXTmH7DgINe1Ddt68GWu83XdVVXEAQFJayiVgw7oDvrJK8s3VxMvHzw957iSOJ82dbze02f4ugEhBpVHsBvQmjm9gJzx5lNQ1lvtulWSxAODdu3fr16/ncDhkh0KoMWlp6V27dgHAmjVr8vLyfuQpd+7cmT59uqGhoZSUlKWl5aJFixpNkj916hSFQtm1a1dVVdXatWt1dHTExMT09PRcXV2/Ho1SUFDg4uJiaWkpKSnZpUuXOXPmpKSkNNe7a01YwCOEUMvy8fHh8XgAMHXWGDV1ZbLjINQYny84dezi2JELnjx8TlyxHGIbfPWI7ejBzdhKXtaH7cs8efX1AGA7ZOhkx/nN+OKILD17GbGkJQAg7c2brMxMsuN0XKra2su2bBGXkACAtLQ0Nze3mpoaskMh1NiUKVPs7e3LysqWLFnynzcHBwcPGDDg77//TktLU1RUTEhIOHDggJmZ2ddzdjgcTr9+/WJjYxcsWODh4SEQCLZv325vb//lJpd37941NTXdu3dvYmKisrJydnb28ePHzc3Nz54928xvsuVhAY8QQi3o/v37xCeNtIyU0+KpZMdBqLGszA9zpqz0895XzakBAJaczMod7qv3eMooyDVjK5XlFducN1aWVwCAvpGxi9vm318MDwkDBoPW186SOL5x8wa5YTo4jS5dl3h5i4iJAUBKSoqHh0ddXR3ZoRBqbP/+/VJSUhcvXgwLC/vObcRYEgqF4u3tzWaz3717V1JSMm3aNGIWPZfL/fLmwMBAPT29+/fvu7m5rV+/PiEhQU1N7cGDB/fv3ydu4HA4U6ZMyc/Pd3V1LS8vz8jIYLPZbm5uXC531qxZ7969a8E33AKwgEcIoZYiEAg8PT2JY6cl01nSuNQ2EiJ8Hv/owbPjRy1MTPg8htBmZP/gKyHWI/o3b0O8+vrtyzzzsj4AgLKq2nq/QAaT2bxNIBJZ9jEVExcFgOSk5PwfGxaLWoiOgcHizR5MJhMAnj59unXrVj7u8IeEjKam5rZt2wBg6dKlxO48TSL+AjV//nx3d3dRUVEAkJaWPnXqlJmZ2fv37w8dOvTlzbW1tTt27KD/ux0pi8UaOXIkALx+/Zq4EhwcnJeXN3Xq1MDAQHFxcQAQExPz9fV1dHSsra39zuZ2wgkLeIQQailhYWHPnz8HAHVNlakzHciOg9D/pL/Nmj5h2fZtB2tragFAWl529W6PFcHuLDmZZm/rsPful4+fA4C4hIRbYLC0rGyzN4FIJCLCtO5jDgB8Ph874Umna2zsvNmDzmAAQFxcXGBgINbwSNgsWrSoT58+eXl5rq6u37onNjYWAFasWNHououLCwDcuXPny4uWlpZKSkpfXlFQUAAAYgIjAFy/fr3huV8ilsQn2mpD6GQHQAih9qmmpmbLli3E8QrXeUwmg9w8CBHq6+sP/3n64L5TXO7n4bX9xgydvWGxZMuMELl6/EL02asAQKVSV3v7aul0bolWELn62FnExT6p49bFxyfY29vLyyuQnahD0zczm7N69ZGAAD6fHxUVJSkp+SPzjRFqNRQK5dChQ2ZmZiEhIdOmTRs4cGCjG2pra3Nzc+l0up6eXqOHDA0NAaDRoPdOnTp9v8W0tDQAOHToUGho6JfXiR2CPnz48CtvgzxYwCOEUIvYv38/8ZFgbGow3L4/2XEQAgB4/TJ909rAN68ziFN5VUUnz5Vm/SxbqLlnsY9PBBwgjh2XrexpZdNCDSFySUiK9bIyeXA3gc/jRUXdmjJlCtmJOrqetnbVHM7fe/YIBIKLFy9KSkrOnj2b7FAI/Y+BgcHGjRs3b97s5OSUnJwsJib25aP5+fl8Pl9JSYlGozV6oqqqKgDk5OR8eVFCQuI7bXG53MLCQgA4evRokzdUVlby+Xwqtc2MTG8zQRFCqA0pLCzcs2cPAFAoFFe3hbheFyJdbS13V9CRKWOWENU7hUIZNGlU8OUjLVe956Rn7Vy9hc/jA8CwMeNGTZzcQg0hYdBvgCWNTgOAB/fvl5d/c14rajV9hg0f6ziPOD5x4sT3FwxDqPWtX7/e0NAwIyPDw8Oj0UMqKipUKrWoqKhhDHyDT58+ETf8eENMJlNeXh4AKioqBE1pW9U7YAGPEEItwdvbu7KyEgCG2/fvaWFIdhzU0b1IfDXxD+dD+/8mfhlS0lDZFBKw0HuVmKR4C7VYUVoesGhTdSUHAIzMLRas/OZER9Q+SMtIET/r6uvrb0ffJjsOAgAYPG7c8Emf/3D2559/RkVFkZsHoS8xGIzDhw9TqdTg4OCnT59++ZCIiIi6unp9fT0x9P1LL1++BABdXd2faqtLly7wxZp2Dbhcbm5ublFR0U+nJxUW8Agh1MwSExOJbUVFxURWr3ciOw7q0Gqqa/23/Dlz8op36dkAQKFSRswct/1yiJFNz5ZrtL6uPmiZZ0FOHgCoamqu3bKNRscpe+1fv4FWRC9WXFxcVVUl2XEQAMDo2bPtRo4EAIFAEBQU9PDhQ7ITIfQ/lpaWy5Yt4/F48+bN+3LPdgCws7MDgF27djV6yu7duwHA1tb2pxrq378/AOzdu7fRdT8/Pw0NDVyFHiGEOjSBQODm5kas+uvoNFlFVZHsRKjjevzw2ZgR808cuUCMY1fT0fQ6sXPuxiUiYqIt2m6Iz+7X8UkAICEptdF/uySL1aLNISGhoChrZGoAADU1NY3WiEYkmrJosYWdHQDweDwfH5+kpCSyEyH0P1u2bOnUqdOLFy8adYN7enoyGIxDhw4FBATU1dUBQGVl5ezZsxMSErS0tJydnX+qlXXr1snIyJw4ccLT05PYQ14gEJw+fdrPz4/JZDo6OjbjO2oFWMAjhFBzOnv2LDESTEVV0dEJJ/0iclRWVHlt3DF/xtoP2XkAQKPRHBZMCbj4l755i0/oiAwNI5adp9Forj5b1bU7tXSLSHgMGGRFLPlx586dmpoasuMgAACgUme7unY3NwcALpfr7u6ekZFBdiaEPpOQkDhw4MDX17t27bp161Y+n79u3ToWi6WnpycjIxMaGqqoqHjs2DFiZ/gfJycnd+zYMSkpKS8vLxaL1b17d2Vl5alTp9bV1R09evTrte6FHBbwCCHUbCoqKry9vYlj1w0LRcVEyM2DOqY70Q9HD5t37p+rAoEAALT1u/ie3Tt99QKmaIt/QyY9eHrC//OvYnOWLjft3VIr5CHhpKKmaNCjKwBUVVbF3b1Ldhz0GY1Gd3Lb2NnAAACqqqo2bNjw8eNHskMh9NmwYcNmzpz59XVXV9fo6OipU6d27tz548ePpqamzs7OSUlJAwYM+M/XZDKZRkZGX24v5+Dg8Pz5c0dHRwMDg/fv30tLS0+dOjUpKWnatGnN+F5aB85JQwihZhMUFESsj2phaTxsVD+y46AOp6S4zM9737XLMcQpg8kYt2jGmAVTWmcKet773B0rfIh18ob84WA/CfcS64gGDLZ5lfIWAKKjowcMGEBnMMhOhAAAmKKiiz08g9et/fj+fUlJyfr163ft2iUrK0t2LtQhWFpaEn9Q/pbQ0NBGO7QTBg4c+PUu8V+aPn369OnTv77u5eXl5eXV6KKOjk5ISMgP5BV22AOPEELNIy0t7dChQwBAo9HcPJbi1nGolV0Jv+UwdF5D9d7NtLt/2F/jF81oneq9upITsHhTFbsCALqbmDqtXtsKjSIhpKmtqqunAwBsNvve/ftkx0H/Iy4ltdTHR05JCQA+fvzo5ubG4XDIDoUQ+mlYwCOEUDMQCATr1q0j1lmZPP2PbvqdyU6EOpC8j58WObqtX7WttLQcAERERWZvWOx9apdGV+3WCSDgC3a5+uZmZAOAoorqWl9/7HftyAYMsSEObkVFNVpZGpFLRl7BxceHWFfy7du3Hh4exMcWQqgNwQIeIYSaQVhY2L179wBAXkHWZdUcsuOgjoLP5586Hu4wdF7cnSfEFSObnkGXQ0bNHk+ltd5H/D87QxLvPAIAUVGxDdsCpXFcbsfWuYtmp84aAFBSUvL48SOy46D/R1lDc7Gnl4iYGAA8e/bMz8+P2DYFIdRWYAGPEEK/i81me3h4EMeuG5ykWJLk5kEdxNs3mZMcFvl57eVwqgFAgiW12G/tppAAZU3V1oxx/2pMxKHTAEChUFw2btbR7daarSPhNHBIH+Lg5o2bfB6P3DCokU56ek4b3Gg0GgDcvXt3//79ZCdCCP0ELOARQuh3bd26taCgAAAsehvbjxlMdhzU/tXV1e/fFTpy4KzE+GTiitUwux2RR/qPHdbKiy9kvko/sDGQWJ1owuy5NgMHtWbrSGh109fR0FIFgMLCwifx8WTHQY0ZmJvPWLGC+HERHh5++vRpshMhhH4UrkKPUAdVXV1dW1vL5/MrKiqIK2VlZV/fJiMjQxzQ6XQJCQk6nS4pid3L/8/z58+PHTsGAHQ6fZPPMly7DrW0F4mvPDYEp7/NIk5lleTnuS/rPaRv6yepKC0PWrq5tqYWAHr3tZs6z6n1MyChNXCITWjIBQC4eeOGZe/eFCp2GgkXy4GDKkpLw44cAYCQkBA5ObmhQ4eSHQoh9N+wgEf3qv5/AAAgAElEQVSo/SguLi4qKiL+W1hYWFJSUv6vsrKyioqKioqKqqqqmpqaysrK32mIyWSKi4tLSUlJSEhISEhISUmxWCw5OTkZGRkZGRlZWVk5OTl5eXlFRUUlJSVxcfHmeoNCqL6+fvXq1cS+WXPmT+iq24nsRKg9q6zk7AoMOXPqEp8vAAAKhTJsqsOUlY7iUhKtH4bH4wWv8C78WAAAGp06Ld/shRUa+pJBD11VdeW83IL8/PyniYkWFhZkJ0KNDR4/oayk9Hb4RYFAEBwcLCcnh/+bEBJ+WMAj1MaUlZVlZ2dnZ2d/+PDhw4cPeXl5+fn5eXl5BQUFXC63dTJwuVwul9tkj/3XxMTEVFRUlJWV1dTUVFRU1NTU1NTUNDQ0NDU1FRQUWjpqSzt48GBSUhIAqGuoOLvMJDsOas/uRD/0cd9dkF9InKp20lgeuLF7L+OamhpS8pzwP/Dy8XMAkJCU2rAtSFyChD8iIGFGocCAwdZ/Hw8HgOvXr1uYmwMOUBI+E+bNY5cUJ9y9W19f7+3tvX37dl1dXbJDIYS+Bwt4hIRXaWlpenp6RkZGRkbGu3fvMjMz379/z2azf/kFqVQqS1JSXExMRESEJSlJoVCkpaQAQITJEBMV+9azauu41dU1AFDF4dTV13Oqq+vq6iqqKis51bW1tf/ZaHV1dWZmZmZm5tcPiYqKamtra2pqamtr6+jo6OjodO7cWUtLi8lk/vJ7bE05OTkBAQHE8QaPJaJiIuTmQe1VUWGJn9e+G5GxxCmNTneYP3n8ohnSsrJ8PjnLg92NiIoMDQMACpW60tNbTVOLlBhIyBka66uoKObnF37MzX3+4oWpqSnZidBXqNRZq1azy8rSkpI4HM7GjRt3796toqJCdiyE0DdhAY+QsGCz2a9evXr9+nVqampaWtrr16+Li4t/8LlMBkNJQUFVSUleTlZBVlZJQUFeRkZOVlZORlpaSkqGxWJJSrGkJMVERZs3cz2PV1lVVcZmsysq2RUVpeXlZWx2cVlZcUlJUWlpcWlpQWFRUWlJYVExr6ldampqat68efPmzZsvL9LpdA0NDV1d3W7dunXt2lVPT09XV7dhKr5QWb16dVVVFQCM/GNg/0HWZMdB7ZBAIDh/OjLY/1AF+/O0F10Tg4U+q7W66ZCYKvPV24MeO4jjaQsWmlv3ITEMEmZUKvQbbH3m5CUAuBYZaWpigp3wQojOYDhvct++bm1uZmZJScmGDRt27drFYrHIzoUQahoW8AiRJi8v7/nz58nJyS9fvkxJScnOzv7Pp8hKS2upq2trqGupqWmpqamrqqopK6soKiiRNBadTqPJsFgy//Uxz+PzC4uK8z59Kigs/JCf9zH/U07ex5yPedkfcwsKixrdXF9fn5WVlZWVFRUV1XBRTU1NT0+vR48e+vr6PXr06NatG+m99GfOnImJiQEAGVnWOvdF5IZB7VLG2/deG3ckJqQQp6LiYlNXOg6bNqY1N3j/WkUZO8jFk1tTCwDW/QeOnzmHxDBI+JmYGty6fq+4qCQnJyc5OdnI2JjsRKgJohISS7y8AlevLi0szMnJ2bRpU1BQEOmfs+hbzkecfp6c2MqNsit+fQQoal5YwCPUesrLy58+ffrs2bNnz549f/6c2HjsW8RERXV1OnXt1ElXR6eLtnYXbS0dLS1ixHubQ6NSVZQUVZQUv36otrY2++PHd9k5mdnZ73JysnJy3r3Pzs7NbdRj//Hjx48fPxIFMwAwGAw9PT0jIyNDQ0MTExNDQ0Op1v3KFBUVbd68mThe4+YsryDbmq2jdq+2lnto/98hB07X1dUTV8wHWM/bvExBVYncYHwef+cqn8LcfADQ0um8bNNm3HYBfR+VRhkw2Pr86asAEHktEgt4oSUjr7DUy2v7mjWcqqpXr15t3bp18+bNVFyZUig9jL/3MP4e2SkQabCAR6hlZWRkxMfHP3nyJD4+Pi0tjd/USHIAoNNoXTppG+rpde/WTb9LF/2uXbTU1DrCB6eIiIiujo6uzv8bD1zL5aZnZaVnZaVnZqVmZKSmZ7zNzKyrr2+4oa6uLiUlJSXlc88kjUbT09OzsLDo3r27qampkZFRS/cbrF+/vqSkBABsbC0cxuO+O6g5PX74zGfTrqzMD8SprKL83E1LrYbZkZuK8M+Ow8kPEgFAXFJynV+AqFh73mMCNRczix63o+6VFJe/z3r/8mVKjx6GZCdCTVPV7rRwk/ueze71dXX37t37888/lyxZQnYohFBjWMAj1MwEAkFqaur9+/cfPnz44MGDT58+NXkbg07v3q1bT8MexgYGxgb6+l27iorgEmifiTCZPbp169GtW8OVuvr69Kys1PSMlDdvXqWlJae+yfviC8vj8V69evXq1SvilMlkmpiYmJubW1hYWFhYqKurN2+8K1euREREAIC4uJiH74rmfXHUkZWWlAduPXD54i2BQAAAFCpl8CT7aavmS7AkyY4GAPD45t1LIWcBgEKhLHf3xIXr0A+i0aj9B9mEnb0GANcir2EBL8x0jY1nrVx1NDBAIBBcvHhRRUVl/PjxZIdCn02YMMHExITsFKCo2MSAStSasIBHqHnk5ORcuHAhJiYmJibmW0W7jpZWb1MTc0ND0x49DPX1RHB22Q9j0OkGXbsadO06dvgw4kpRSUnKmzcvXr9+8fL185cv3+fmNtzM5XLj4+Pj4+OJU3V1dUtLy969e1tbW+vr6//muIaSkpJ169YRxyvWzlPXwKV6UTMQCAThF25s9ztYVvp5kqGmro6T90o9sx7kBmuQ+y57/4ZA4i8LE2bP7d1XKEYEoLbCvLfR7agHZaXl7969e/3qlUH37mQnQt9k0a9faWHhxaNHAOCvv/5SUlKytbUlOxQCANDW1tbW1iY7BSIfFvAI/brKysq4uLiYmJg7d+40uU0ag0437dHD0szUqqdZLxMTBTm51g/ZXinIyfW3tu5v/Xnt9+ra2ucvXz6IT3ianPw0KbmotLThztzc3LCwsLCwMABgsViWlpY2NjbW1tYmJiZ0+k//DFy/fj3xBxrzXkZTZjg007tBHVrG2/c+m3clPE4iTpmiIuMXzxjtOIn289+fLaSGU719mWd1FQcAzKysp85zIjsRamNoNOqAQVYXz98AgKuRkVjAC7khEyYUF+TfjYzk8/l+fn7y8vLGuHgBQkJDWH45QKgNefPmza1bt6Kjox8/fszlchs9yqDTzY2NbCws+liY9zY1FRf75v7qqBnJy8oOsbOz7tmTOM3Mzk5ITk54kfzk+bOXb9IalsRjs9lRUVHE+vYSEhJWVlZ9+/a1tbU1MjL6kZ75K1euXLx4EQBExUS8/V2pVFy+C/2W2praA3tPHTt0tmGxOpM+FvM9litrqZEbrJE/NwZ9SH8PAMqqaqs8fCgdYHkO1OzMLY2jbz1kl7HfZWS8eZOqp6dPdiL0PZOdF5UWFSU/ecLlcjdt2rR//359ffxfhpBQwAIeoR9SX1//4MGD69ev37hx4+v93igUSndd3SF2tkP72fXt3bu+ro4YaIrIoqOlpaOlNXHUKACorKqKf5H0+Nmzh4mJT5OSq2tqiHuqqqqio6Ojo6MBQEZGxsbGxs7Orl+/fl27dm3yNYuKilxdXYnjlWvma3dq5qn1qKOJu/PE13PPh+w84lRGQW72hsV9Rg0gN9XXIkPDHl67AwAMJnOt7zZJ3B0a/RI6ndZ/kNWlCzcBIPJqJBbwQo5Co81bt37HhvXv09LYbPa6deuOHz8uIyNDdi6EEBbwCH0Xh8O5detWZGRkdHR0WVlZo0flZWUGWNsM6ttngI21ory8lJQUjUYDgPLycjLCoqZJSkgMsLEeYGMNAHX19c9SUh4kPH3w9OmjxGdVHA5xT1lZWWRkZGRkJABoamr269evf//+tra2cl/MenB1dS0uLgaAXlYmU2eNIeOtoHbiU37RNp/9N6/dJU4pVMqQyX9MXTlPSBar+9KbZy9PBPxFHDutXtsZiy70G3pZmtyJfsguq3j79u3btDTdL1YqRUKIKSq62MMzYOWK4k+fcnNzV69e/eeff5IdCiGEBTxCTamoqLh58+aVK1du3bpV82+HLYFCoRjp6w+1sx1iZ2tmaEjDoaRtCoNO721q2tvUdMX8efU83vOXL+MeP7n7+Mnj589ra2uJe3Jyck6ePHny5EkajWZqajpo0KCBAwemp6dfvXoVACQkxLf4r8HB8+jX8Hi8k8cu7t8ZWlX1+Y9HnQy6LvBcoWtiQG6wJpUXl+1Y4c2rrweAwfajB9uPJjsRatsYDFr/gVaXwqIA4OrVqyuwgBd6UjIyS7y9g1xdOZWVL1688PT09PDwIDsUQh0dFvAI/U9VVdW1a9fCw8NjYmIaTW4XYTL79u41YsCA4f37qSopkZUQNSM6jWZhbGxhbLxywfza2tonz1/cefTozsNHSa9f8/l8AODxeE+fPn369GlAQACF8rlid1k9W10TV55Hv+LZ0xQf991pqe+IUzEJ8UnL5gyfMYYYuSNs+Dz+blffkoIiAOjcTc9p9VqyE6H2oJeV6Z3oh+zyyrS0tPS3b7vq6pKdCP0HFU2thRs37XHfVF9ff/PmTSUlpTlz5pAdCqEODQt4hIDL5UZFRYWFhd28ebNRf7uEuPhQO9s/hgwe1KePpIQEWQlRSxMREbG17G1r2dt9+bKSsrLYR4/vPHwY8+Bhbn4+cQOxqIGIOIT+/deTxFgrKysrK6vOnTuTmhq1GaWl5Tv8D188d71hdQyrYXZz3JbIKSuQG+w7zu0LTX6YCACSUlJrfbcxcNtL1BwYDFq/AVaXw28BwNXIq8uXryA7EfpvnzeHDwoUCAQnT55UVFQcNWoU2aEQ6riwgEcdl0AgePTo0dmzZy9fvtxo1jpLUnJ4/36jhw4ZaGMjIiJCVkJECjkZmbHDhxEbzr9OT/cK3hkVFwcAVBpIKwKfz3/16tWrV6+OHDmirKxsbW1tbW1tbGzMYDDIDo6EEZ8vOH/66q6gkPKyCuKKipb6vM0uJn17kRvs+17ciw87cBIAKBSKyyYPZTVcshE1m97WprExD9nlVW9S36Snp39r3VAkVKwGDSorKgo7egQAdu/eraSk1KuXUP8QQ6gdwwIedUQZGRlnz549d+5cTk7Ol9fFxcRGDOg/bvjwAX1sRLC7CQFwudzYR4+I40ULHGp4FY+ePSsu+bzJfEFBQXh4eHh4uISERK9evWxsbCwtLSVwpAb618vkNB/3XSlJb4hThghzzIKpYxZMYYgI9Y+X4vzCPWu3CfgCAHCYOqN3XzuyE6F2hcGk2w2wuhIeDQBXr17BTvi2YsycOZ8+frx34zqPx/Px8dm1a5eOjg7ZoRDqiLCARx1IZWVleHj433//HR8f/+V1JoMxsG+f8SNGjBjQX0xUlKx4SNhwqqud1m3g1tUBwMQxA5fMmwIAAoEgPSvrUeLzh4lP095lErPlq6qq7ty5c+fOHQaDYWJi0rdvXxsbmy9XsEcdTVkpe/f2I+dPX+XzP4+ZN7Pr7bjJRdg2eP8aj8fbtXoLu6QMAAyMTWc4LyY7EWqHLK3NYm8/qmBjJ3xbQqHAvLVri/LzUl+84HA4Gzdu3Lt3L37SIdT6sIBHHcLjx49Pnjx5+fLlqqqqL69bGBtP+sN+7PBhcri1KfrKxoDA9KwsANDRVlu/ahZxkUKh6Oro6OrozBw/tri07GFi4sOEp09TUohVD+vq6hISEhISEnbv3m1gYGBra2tra6usrEziu0CtjM8XXDgTuSsopKyUTVxRVFOevWFx7yF9yQ32g07vCEl9mgIALBnZ1V5bhHOBPdTWMZh0u4FWV7ETvq2hM+hLvbx9li4t+JDz6dOnTZs2BQcHi2LPB0KtCwt41J6VlpaePXs2NDQ0LS3ty+saqqqTR/8x+Q/7LtraZGVDQu7SzajQ8xcAgMGgB/gsFRNrYikEeVkZ+0ED7QcNrKmtTXiRdC8+4VFiIruyEgD4fP7Lly9fvnx54MCBbt262dra2tnZqavjROJ2LvlF6pbNu18mf/6BQ2fQ/5g7adziGSKibWMpjcQ7jy6FnAUACpW6YrOXPO64gVqMlbXZ3YZOeFyOvu0Ql5Ja4ukZsHpVZXl5Wlqan5+fh4cHFbfURagVYQGP2qdHjx6FhoZeunSpYXNvABARERk5oP/0sWP7WVnihw36jpyPeSu9vInj5Ysm9TD4j2l+oiIifXv36tu7Vz2Pn/T6VdyT+PvxCUUlJcSjaWlpaWlpISEhnTt37tevn52dnaamZsu+AdTqSorLdgaGhJ+/3jBm3qRvL8dNS1U7aZAb7McV5xXuW+9PrJM/fuZsM0srshOh9ozBpPcb+Hkm/JWrV1eswE74NkNBVdXZ3X2Xm1sdl3v//v1Dhw4tXLiQ7FAIdSBYwKN2paqq6ty5c0eOHHn9+vWX1w26dp09ccJE+/9j777jori6PoDf7buwfem9dxSQqigIKNg10dijxhJL1NiiSTQmmmI0JrHnUaPpGo3GFgVsiA0bIL2D9LpsY3t5/xhDfI3GBswC5/s8n2T27g78lugyZ+bOuSO4bDZe2UB3odFq5763WiSRIIT6h/nPnDry+fclk4hBfn5Bfn5LZs3MLym5evvO1bRbtY2N2LNlZWVlZWUHDx50cXGJjo6OioqCa/I9gE6nO/TTyV3bfpJKZNiIua3VjDULusuceYxOp9u28lOpSIIQ8g0InDR7Ht6JQM8XFhGYejlNIm4rKoSL8N2Mi7fP9HeXHdyy2WAwHD161NbWduTIF/hdCQB4FVDAgx6ipKTkwIEDv//+u0QiaR9k0Olj4+NnvjE+uE8fHLOB7uXzHTvv3L+PEBLwOZs+WUgkEl7iixAIBB93dx9397enTikur7hy61Zq2q3qujrsWaySP3DggLu7e3R0dHR0NNwn303dupnxxce7SoorsIcUGnX07Inj5k2mdpM58+2O7vix/db35XDrO+gSFCo5anAEtib8mTNn3l22DO9E4AUER0U119Wd+vknhNCOHTusra379euHdygAegUo4EH3ZjAYLl++/N1336WkpGAzPzEeLi5vTXzjjVEjOSwWjvFAt3Ph2rUdB39ACBGJhE2fLDQTdEB3Q3dnJ3dnpzmTJpY8eHAl7daVm2ntlXxxcXFxcfH+/fu9vb0HDx4cFRUFHX27i5rq+q1f7E0+l9o+EhI74M01CyztrXFM9XKyb6b/ufc3hBCBQFi6bj3fzBzvRKC3CI0IuHI5TSKWFRUVFRcVuXt44J0IvICEiRMbampuXbqo0+k2bNiwfft2R2gtBEDngwIedFdyufzIkSN79+4tLi5uHyQRiQmDo+dOnhwZGkIgvMyFU9Cb1TY0LPzgQ+xM0Jw3Rw8I7+CJG26Ojm6OjrMnvlHy4EHKzbSUGzdrGxoQQgaDIS8vLy8vb8+ePQEBATExMZGRkUwms2O/O+goSoVq/3eHDu47qlI+bLFh42w/68NFfSND8A32csQtoh2rvmhf9T0ovD/eiUAvQqGSo2MjTh0/jxA6c+bMsuXL8U4EXgSBMHXJEmFjQ3FOTltb24cffrhr1y4Oh4N3LAB6OCjgQfdTX1+/b9++n3/+ubW1tX3QjMebPv71WW9MsLWywjEb6L60Ot3c91a3tIoQQv0CvRbPn9B53wur5OdMmlhQWpZy40bKzbTGlhaEkF6vT09PT09P37ZtW2hoaGxsbHh4OJVK7bwk4IUYDIZzZy5v/WJfQ30TNsJgmoxf9OawaePIlG75+9RgMOxcvUnULEQIefj4TXt7Ad6JQK8TEh6QcilNIpIWFxcXFBR4eXnhnQi8ADKFMu/DtVtWLG+sra2vr1+3bt1XX30Fv7YA6FTd8oAD9FoFBQW7du06fvw4tuY2xsfd/e1pUyeMGE6jdbObToFR+XTb9rT0DIQQj8va8unirrkH2MvVxcvV5e1pU3MKiy7duHHlZhrWPE+j0Vy/fv369eumpqaRkZGxsbEBAQGwdAK+crOLvvhkV2Z6LvaQQCREj42fvHw216wb3/Vw5uDR+9fuIIRMmazlGz4lkeGoAHQ1CoUUHRtx6lgyQujMmTNQwHc7pmz2gvUfb1mxXC6T5eXlbd26dc2aNTALEoDOA7+qQfdw/fr1nTt3Xrx4sf1GdyKROGRg5Pzp0waFheGbDfQA5y5f3vnDj+jvW9+tLLq0JCMQCP5env5enotmzEjPzr5048a1O3fkcgVCqK2tLSkpKSkpSSAQxMTExMXFubq6dmU2gBBqbGjZvvXAqePJ7UvEeQT4zPrwHVd/T3yDvaLS7MJD33yPbS9c84GltQ2+eUCvFRre98rFNLFIUlZaWpCf7+XtjXci8GIs7ezmfvDBznXrdDrdxYsX7ezspk+fjncoAHosKOCBUdPr9UlJSdu2bbt37177IJ1GmzR61MIZb7pCrxTQEcorKxd9uA47NzRv1tiB/QPwSkImEUMD+oYG9FWr59xMT79w7frtjAyNVosQamlpOXr06NGjR52dnePi4mJiYszNodNYp1MpVT9+/8f+PYex8ykIIb6l2dQVcyNHxXb360uKNvm2FZ9pNVqE0JDRY/sPjsU7Eei9yGRSTFzEn38kIewiPBTw3ZBn34DJi975Zfs2hNBPP/1kb28fHR2NdygAeiYo4IGR0mg0x48f37FjR2FhYfuggMedPWnS7MmTzHg8HLOBnkSpUs1cvlIslSKEwkP8Fs0bj3cihBCiUilR4WFR4WESmSz11q0LV69nFxRgpxjKy8v37dv3/fffBwQExMXFDRw4kMFg4J23B8Jud//my/11tY3YCJVOG/XWG2PnTqIx6Phm6xAHNu6or6xBCNk7Oc9eCp3DAM6Cw/umXEprFYrLysrycnN9fH3xTgReWP/4+Iaa6vPHjhkMhs2bN1taWnrDuRgAOgEU8MDoqNXq3377bfv27VVVVe2Djra2i2a8OWXcWAa9Jxw6A+OxfMPGnMJChJClBf+rzxaTjOw+czaTOTI2dmRsbH1T88Vr185fvVZZU4MeaXe3ffv2gQMHDhkyBG6S70D3M/I3f7bnfnoe9pBAIPQfHj115Twzawt8g3WUa2cuXjmRjBCiUKnLP/mMBp+rAG8kEnFwXP/jR84hhM6cOQMFfDc1buasxtra+zdvqtXq9evX79y508Kih3xsAmA8oIAHRkShUPz88887d+6s+3uVbISQt5vbktlvvTYsgdwlTcVAr/L94d9/P3UaIUShkL/Z9C6fx8Y70VNZmZtNHTd26rixRWXl569eu3T9eqtYjBBSKpXnz58/f/68ubl5XFzckCFDHBwc8A7bjdVU1X+zZX/SX1fa22249fGa8f5Cz8CeU040Vtfv/2Qbtj1j4WInNzd88wCA6Rfqn3LxhrBFXFFRkZOd7efvj3ci8OKIxFkrV21dtaqqrFQoFK5du/bbb781MTHBOxYAPQoU8MAoyOXygwcP7tq1q6mpqX2wXx//5XPnxkcN6u73mgLjdOf+/bWbt2Dbq5dND/B3xzfPc/JwcfZwcX572tQ7mffPX029cS8dW5Shqanp0KFDhw4d8vLyGjp06ODBg1ksFt5huxOZtO1/u3799Yc/1WoNNiKwNp+yfG7kyJie9BGk0+l2rPpcLm1DCPWLGDB8/Bt4JwLgIRKJGDMk8o/DfyGEzpw54+fnh3rQX73eg0qnL1i//stl74qFwrKyss8//3zDhg0wQQyADgQFPMCZQqE4cODAzp07m5ub2wcHBAeveHtuVHg4jsFAz9bQ1Dxz+Qq1RoMQGj184JQJQ/FO9GLIJGJEv8CIfoGyNnlKWlrylSu5RcXYReOCgoKCgoI9e/ZERETEx8cHBwd3zZJ43ZdWq/391zPfbf+5tVWMjdBNGGPnTR45czyV3tMWpzy+59fCjFyEEJfPX/zhRz3p3AToAYKC/S5duCFsbq2srLyfdb9vX9xaioJXwTUzW/DR+q9Xv6dWqdLS0v73v/8tWLAA71AA9BxQwAPcKBSKH374YceOHY9edY+OiFg5f15EUBCOwUCPp9ZoZi5fUd/YhBDy9nT6+IM5eCd6eUxTk5GxMSNjY2rq68+nXk1OvVrf1IQQ0mg0qampqampfD4/Li5u6NChTk5OeIc1RucTr37z5f7KBzXYQyKJOPj1YROXzOzWq7s/TVFm3vE9vyCECATC4g8/4kA3UGBkiCRC7JD+Rw89vAjft09fuAjfTTm4u7+5fPn3mzYZDIZjx445OjoOHz4c71AA9BBQwAMcqNXqH3/88dtvv21sbGwfjBnQf9X8t0MD4HQ76HTvf7HpdmYmQojHZW3fspxOo+KdqAPYWlnNfGPCjAnj7+cXJKVcSb11S6FUIoSEQuGRI0eOHDkCU+sfk5me+9XnezPTc9tH+kaGTH/vbQcPZxxTdR5Fm3zHqs91Oh1CaPjrE4LC++OdCIAnCOznl3LhZlOTsKa6JiMjIxBO6HdbQZEDG6fVnPr5J4TQ9u3bbW1t+/bti3coAHoCKOBBl9JqtYcPH966dWt1dXX7YHRExOqF86F0B13jpz+O/XD0D4QQiUTa+vkSW+setZo6gUAI8PEO8PFeOntWatrtxCspWfkFer0e/T21/rvvvuvfv398fHy/fv167U2J5aVV327ZfzH5evuIk7fbtFXz+vTvh2OqzvbDZ7saquoQQg7OLjMWLcE7DgBPRiQRYhMGHv75JELor7/+CggIIPTWT6oeIGHixPqa6tuXLmm12k8++WTnzp02NjZ4hwKg24MCHnQRvV5//PjxLVu2lJWVtQ9Ghoa8v2hReFAgjsFAr3IrI3P1519g28vfmRwe4odvns5Dp9GGRg0cGjWwvqkp6Upq8pXUusZGhJBarU5JSUlJSTE3Nx8yZEhCQkKvOpxqahTu3vbj8SOJ2IVohJDA2nzS0rcGjVkz7H8AACAASURBVB5CIPbkmbq3klMvH09ECFEo1GUff0qh9oRZJ6Cn6hPgffn8jYb6ptra2nv37gWHhOCdCLwsAmHa4iXNdfVl+XkSiWTt2rXbt29nMpl4xwKge4OTmqArXLhwYfDgwQsWLGiv3kMDAk7s33fy+/1QvYMuU1NfP2PZcqxx3ciEAbOmjcA7UVewMjefMf71X7Z/+/VHHw0dNIhOe9iVramp6bfffpsxY8by5cuTk5MVCgW+OTubTNq27asDw2PePHroL6x6N2Uzp66Yu+3cj1Fjh/bs6r21qWXvR99g21PfXgDrxgEjRySiuIRIbPuvs38Z9Hp884BXQaZS569dy7ewQAhVVlZu3Lix/fwpAODlwBV40Lnu3LmzcePGmzdvto/08fZ6/513hg4aiGMq0AspVao3313e1NKCEPL1dt6wdh7eiboUgUAI8PUO8PVe8tbMlJtpSVeu5BQWGQwGg8GQlZWVlZW1c+fOQYMGDRs2zNe35yx4jlGp1Id/ObVv92+iVgk2QqFS4qeOeW3+NCan57cDMBgM3334lVQkQQj59wsePXEy3okAeDa/Pp7WtpZ1NQ0N9Q23bt8Oh1VpujMml7tw/cdfrVqplMvv3bu3e/fuxYsX4x0KgG4MCnjQWYqKij777LOzZ8+2j7g5OX2weNHoIUNg4SLQxQwGw+J1H2Xm5iKEBHzOji0rekbjupdgwmAMjxk8PGZwdV19YkpKcurVZqEQISSXyxMTExMTE+3t7ePj44cMGSIQCPAO+6r0Ov3J48m7vv2xvu7hUhcEImHgqLiJS2eZ21jim63LnD98OiP1NkLIlMlasnY93E4MugUCAQ2Jj/zpwDGE0LmzZ0NCgkkkOGTtxmycnGatXPndp58a9PqTJ086ODiMGTMG71AAdFfwaQg6XkNDw+bNm3/77TetVouNWFmYv7dg/tRx48iwHjXAw7f7vz9+LhEhRKVStm9ZbmXZ7UvTV2dnbTVn8qRZEyfezbqfePnKzXv3sJsLqqqq9u/ff+DAgdDQ0Pj4+PDwcAqFgnfYF2YwGC4kXdu+9UB5aVX7YGBU2JTlcxw9XXAM1sXqHtT8vPl/2Pa8FavMLHrLaQvQA3j7udvaW9VU1Tc1NaWlpQ0YEIl3IvBK/MPCx82cdfzA9wih3bt329nZ9evXk/uGAtB5oIAHHamtrW3Xrl27du2Sy+XYCIfFWjL7rbenTmHQ6fhmA73W2UuXPt+5C9tev+atwD4e+OYxKiQiISwgICwgQCyVXrx2PTElpaTiAUJIr9enpaWlpaVxOJy4uLiEhARn526zuNr11Lvbtx7IzS5qH/EI8Jm6Yq53SB8cU3U9nU63871NKoUSIdQ/JnbQ0AS8EwHwAggENCRh4A/7jiKEzp1LDAsLJ5PhqLV7i3v99frqqhvJyTqd7tNPP92+fbu9vT3eoQDofuCjEHQMnU73yy+/bN68uX1pdxqNNnfypHfnzOZxOPhmA71ZblHR/Pc/xNZRe3PysHGjovFOZKQ4LNZrwxJeG5ZQXF6RmJJy8dp1iUyGEBKLxceOHTt27JiXl1d8fHxMTIypqSneYZ/q3p3sHVsP3r2d1T7i4OE86d23gmN645rnJ/YeKr6fhxDiCczmr1yDdxwAXpiXj6uDk21lRY2wpeX69WtRUdF4JwKvavKidxprakpyc6VS6bp163bs2MFi9fxeJAB0LLgXDnSAy5cvDx48eOXKlVj1TiQSJ4wYcevUyU9WLIfqHeCoWSicunhJm1yOEIqM6LPq3Wl4J+oG3J2dFs+aeeS73euWLgnu06d9rfiCgoJt27ZNmDBh06ZNGRkZeiPrC52TVfj2zDUzJi5rr94tHWwWb35/y4l9vbN6L88r+WPXzwghAoHwzgdrWfBRDLqnIQkPW94mJiZqNRp8w4BXRyKT5334ocDSEiFUXV29cePG9tstAQDPCa7Ag1dSUFCwfv36S5cutY9EhoZsWLG8r48PjqkAQAip1Oo3311WVVuHEHJxstn62VIStO96blQKZXD/iMH9IxpbWpJSUpNSUmr/Xkb+woULFy5csLKyio+Pj4+P5+BdGRbml+785seUizcNBgM2wrc0e33BtJjxw0i9dcKtRq3ZuXqTTqtFCA0dMy4ovDeewgA9g7unk4ubQ1lJpVgkTk1NjYmNxTsReFVMDnfh+o+3rFyhlMvT09N37dq1dOlSvEMB0J3A4Sx4SUKh8L333hs8eHB79e7u7Pzbzh0nv98P1TswBss+2XArIxMhxGEzd329isUywTtRt2QhEEx/fdzP27/9+qN1QwZGUqkPu/fX19f/+OOP06ZNW7x4cXJyslqt7vpsxUUVyxdtGD9y/uULN7Dqnc3nvrl6wfakn4ZMGtVrq3eE0JHtB6uKyxFClja2M99ZgnccAF5J+0X4pKQklUqJbxjQIawdHd96bzU2w+v06dMnTpzAOxEA3UnvPb4BL02j0Xz//fdfffWVWCzGRgQ87uoFC2a8MQGazAMj8e3+738/dRohRCaTvt30rqO9Fd6JujcCgRDg6xPg67P4rbcu37iRmJKSX1yCHul1x2azo6OjExISPDy6okdgWUnlnh0/J/2Votc/vOpuymaNnv3GsOnj6CaMLghgzIoy804fPIoQIhCJS9etpzPg1BXo3pxd7d09nYsLy6VS6eXLKePHj8c7EegAfiEh496afWz/PoTQnj177O3toSk9AM+pexTwjY2Nhw8fzsjIkEgkPB4vODh48uTJzzlv80X3TUxM3L179969e62s4Ij/CS5cuPDRRx8VFxdjD6kUypzJk1bOf5sDPUiA0fjr4qXPduzEtteumhUW4otvnp6EacIYFRc7Ki62oqr63OXL569eE0kkCCGJRHLq1KlTp045OzsnJCTExsZyudzOCFBRVvXdjl/Onr7UXrozmCYjZ44fMWO8Cct4u+t1GZVStWvNl3qdHiE0euJk7z4BeCcCoAMMGTaouLAcIXTx4oURw4eT4GpBjxA7blxd5QNoSg/Ai+oGBXx5efn777+PLUvG5XIbGxvPnj178+bNLVu2WFhYdOy+Op0uMTGxM95FD1BaWrp27doLFy60jyQMjt64YoWLowOOqQB4TFZ+/vz3P2hvO//Ga3DDZKdwsrdb8Ob0uVOnZOblnz5//sbduzqdHiFUXl6+Z8+effv2hYeHx8fHh4aGdtShdnlp1f92/nL2zGWsOkUIMUxNEqaNHfXWG0wOnEB86NDX++sqqhFCdk5OU+ctwDsOAB3DwdHay9etILekTdaWmJg4YsQIvBOBjgFN6QF4Cd2ggN+xY4dcLg8ODl6yZAmXy21qatq0aVNxcfHevXvXrl3bUfvq9frS0tLff/+9rKysM99NtySTybZu3bp3797221y93dw+fW9ldEQEvsEAeEx9Y9OUd5bIFQqE0KABAdB2vrORSaToiPDo/hHNLcITiYmJl1Mqa2sRQlqt9tq1a9euXePxeHFxcfHx8U5OTi/9XUqKK/6389dHJ8zTTRgJ08aOmjWBxYPm6v/Iv5N17pc/EUIkEmnJh+spfzcsAKAHGJowqDCv1GAwJCYmRkdHwx/vngFrSv/lsmUtDQ1YU/ovvvgCZlgA8N+MvYldUVFRSUmJQCBYs2YNNiHT3Nx8/fr1VCr17t27LS0tHbLvTz/9NH78+BUrVty+fbuz31H3YjAYjh49GhERsXPnTqx653E4mz94P+WPI1C9A2MjVyimLF5S19iIEHJ3tf/qsyXQdr7LCHjcSaNH/fDN1h0bN4yMjTU1eXjfdWtr69GjR+fMmbNo0aJTp05JpdIX+rKF+aXLF214bdjcc6cvY9U73YQxZu6kXRd/nbJ8DlTvj1IplLs/2GzQGxBC46a+6e4Dd46AHsXGzsKvjydCSKFQJCUl4R0HdBgmh7tw/Xq6iQlCCGtKj3ciAIydsR/dYhO2+/fvT33kVCubze7Xr59er798+XKH7GtnZxcTEzN06NChQ4dSKJSOfxvdU05OzsiRIxcuXFhfX48QIhGJsydNvH3m1OzJk6BZHTA2er1+1vIV9/PyEEJ8Pnv3N6uYpr29mRkufD3cl8+b88f/vnv/nUWBfr7ty8gXFhZu37594sSJGzduvH379jOXkc/JKlw876PxI+cnn0vFSneGqcm4eVN2Xfx16oq5ULr/229b9zdU1SGEHF3d3nhrNt5xAOh4cQmR2EfKxUsXpRIJ3nFAh7F2dHpr1SrsPy7WTgXvRAAYNWOfQl9TU4MQ6tu372Pjffr0uXnzZnV1dYfsGxMTExMTg22npaVpNJr/TiWXyx88ePDvcQsLC7IRLFyEfQK+ShKxWPzZZ58dOHBAp9NhI/2Dg7/8YI2/l1fHROyhCAQCtkEikdqXpAZd4+Ot35xITEII0aiU3Vvfs7eFJpRdjUj857weg06Kj4qKj4qqb2pMTLmSfOVKbcPDZeSvXLly5coVPp8/dOjQhIQER0fHx77OvdtZ/9v169WUf+ZDMZimw6e/NnLWBBaX3TXvpbsgEAgIEYhEUkF6TuJvJ9Hfk+dpVDh71ZkIf/+bQCAS4HR217G2tgwI8k6/m6tSqpLPJ78x4Q28E/UiBOLDP/cEAuqM2W19wyNemz37j337EEK7du1ydHRsP/9LIBCM4egaAONh7H8fhEIhQujfTeOxKfGtra2dtO9/KywsnDt37r/HL168+Jy98bvAy7WANhgMP/744+rVqxsbG7ERG0vLLes+nDh6dHt1Cp6JyWTiHaF3Ofj7kS3ffYcQIhAIWz5f2j8COm93NQKRaGL6hOXKXEydFs50WjjjzfScnNPnL1y+fgPrUCAUCg8fPnz48GEfH5+RI0fGx8dzOJyUizd2fH3w1s2M9t2ZHNaY2ZPGzJ4Ebeqehkwm6zTaXas3GfR6hNDEWXP9AoLwDtVbEAlEOp2Od4reZcTouMyMfL1On3oldczoMTw+D+9EvQ6ZTO6kcnrM9OmNNTWpZ8/qdLpPPvlk2qzJ2DidTu+kZU0A6KaMvYDH7lT/d0dKNpuN/i7RO2Pf3un+/fsLFy68ceMG9pBCJi+dM3vt0qUsJizOBIxXys20he9/gG2/u2jy2JHRuMYBT0IgBPn7B/n7v7dg/sXr18+cv5iRk4PNUsnLy8vLy/tsw1a9itHaImvfg2vGGzdv6sg3xzOYsIz5M/ywaXfdg2qEkLObx9S58/GOA0AnMrcwC4sIvHntnkajOXHy5KxZM/FOBDrSzOUr6quqirKzJRLJwYMH8Y4DgJEy9gIe8+9rv9iRn1ar7dR9n4bL5cbFxf17XK/Xq1Sql/6yHYVMJpNIpBdKIpVKN27cuHv37vYfS1R4+DefrPd2c0MItTefB/+NTCZjM77gJ9ZlSioqxs+bp9ZoEEKjhw98Z9547bNugQEdi4z1DTEYnudDlUImJ0RFJURF1TY0nrt8+dzFy6WlDW0ipNXoEHpYvTPYzITpY6e8M4vGoCOEtFr4D/pUZDI55/b9UwePIIRIJNLSjz5GRIJW9/K/3cBzIpPICCEDMrTfaAa6TPzw6NtpmTqtLvXKlYSEBDMzM7wT9QpEAoFEJiGEDHqDtvP+2BMIiz7+ZOOihc319U1/TwXV6XTGcHRNo9HwjgDAQ8ZewAsEgpqaGqlUam1t/eg41sqYz+d30r7/zdnZedOmTf8eb2lpedEey53BxMSEwWA8f5LTp09/+OGHdXV12ENrC4sNK1e8NiwBISSXyzsrZU/UPt1DoVDAPfBdoFUsHvvWnFaRGCEU1Ndz88YlxvA7vrdhksmIQDAYDEql8vn3opIpKimxqkQlbv5nkMSgMhz5NEt2at6dog1VYWGhoaFhfJgi+3RkInnbio3Y5PmxU6Y7urqp1fBXoNMRCESsgNfr9fAD73p8Pid8QND1K3e0Wu3xY8fenDED70S9AplEYpAZCCGtTvdCn/YvikKnv7123VerVsqUImxErVYbw9E1FPDAeBh7Ac/n82tqaiT/6jWKjfz3addX2beXqKioWL169aVLl7CHZBJpzpTJ7y9ayDSFOfPA2Kk1mpnLV5Q+eIAQsrU2/9/292k0qkIB1x6NXVOz6OfD534/dkEq++f8oKO7U5+oIBFRVVBahjWor6+vP3ny1OnTZ1xdXUNDQ4OCgkxMoDHb4379el9NeRVCyN7JeeLsOXjHAaCLxA6NvH0zU6PW3L59Oz4h3tISupb2KLbOzrNWrfpq7QrsYWlpKb55ADA2xl7A29jYZGdn5+bmBgcHPzqel5eHEHrs0noH7tvjqdXqXbt2ff311+2nUUMDAras/cDP0xPfYAA8p5UbP712+w5CiGnK2PPtewK+sfSPBE9TWl79w69nT5+7plb/MyveN8h37JuvBYQ/7DsokkjuZGWl3Uuvqa9HCOn1+uLi4uLi4iNHjvj7+4eGhvr6+kA7YkxpduGf+35DCBGJxHc+XEehUJ+5CwA9A5vNjIgMSr10S6/XnzlzZjacvepx+oSFBw2IrDldghDCFjMGALQz9sOguLi4pKSk69evT58+vX09CbVafefOHSKR2L72W4fv27OlpaWtXLmysLAQe8jnctcve3fquLHQZx50F9sPHPz1zxMIIRKJ9PUXS91c7PBOBP7Lrbu5P/z6V+r1zPZbSwhEQuigsNHTxnj4eTz6Si6bPSQyckhkZFVdXdq99LvZWWKJFCGk0WjS09PT09NNTU379esXEhLi6uramz+xtBrtng+36HV6hNCoiZM9fPzwTgRAl4qOCb99I1OpVGWkZ9QkVNvawm+BnkZgaYl3BACMlLEX8J6enq6urqWlpXv27Fm4cCGBQNBqtZs3b5bL5eHh4RYWFu2v/Ouvv/R6vbe3t5ub24vu23u0trZ+8sknv/32G3YYTSAQJo0e/cmK5QIerM8Buo2zly5t3LYd216zfHpkRF9884Cn0el0iRdu/fDrmdz88vZBCpUSNTx69NQx1vb/NQ3K3trafuSI10cMLygpuZWRkZmbp1KrEUJtbW2pqampqakCAT84OCQ4ONjOzrbT34nxObH3UGVROULIxt5hCnSeB72PiSljQFTwxaTrer3+9Okz8+fD3wIAQG9h7AU8Qmjx4sXvv/8+di3d3t6+vLxcqVTy+fzHVmI/ePCgWq2eMWNGewH//Pv2En/88ce6deuamx/2jHJ3dt66bu2AkOD/3gsAo5KVn//2mg+w26SnToyf+kY83onAE0gkbUdPXPr1SFJ9Q0v7IIvDGvpa/LAJwzj85z1jSCQQfNzdfdzd1Rp1Zm7erczMguISnV6PEGppESYlJSUlJdnY2ISEBAcHh5iZCTrlzRifquLy49/9ghAiEAhL1q6nQmsl0CsNjAq7ee2evE2ZnZVVUVHh5OSEdyIAAOgK3aCAd3Fx2bZt2+HDhzMyMoqLi3k8Xmxs7KRJkzicZ9/y+ir79iSVlZWrVq1qb1ZHo9GWzZm95K1ZNCrcMwm6k/rGpinvLJErFAihgf0D1ix/E+9E4HEPqup/PnTuzzNXFIp/unNb2VmNmDRy8MgYGv0lS00qhRoaEBAaECCVye5mZ9/JvF9WWYk9VVtbe/LkqVOnTjs7O/fr1y8oKIjL7cmf8Hqd/ru1W7UaLUJo+OsT/IOCoRE66J3oDOqgweGJZ1IMBsPpU6cWL1mCdyIAAOgKBFjsqgO1tLQYw88TW0aupaUFIaTVavfu3btp0yaFQoE9Gxka8vVH61wdHXHN2DOxWCwSiYQQEovFxvAnoYdRKJUjZ76VmZuLEHJ3tf/1+0+Ypv+0JafT6QQCUaGAhQ+7GpPJRASCXqe7cPnWz4fPXb2Rqdf/84ffO8B75KRRIYNCCcQOvmG9SdhyNzPrdtb9uobGR8eJRKKbm1twcHBgYCCT2QMX1Dh98OjPX36HEOKbme/94yTD1BQK+C5GIBAZdAZCSKfXqVSduJ4WeCITholao9FqNQghtUqz+bPvZNI2hNCyZcvcPTyetTd4SWQSiWHCQAhpNNpOXUau3c97vv5h/5cIoZCQkLNnz3bBd/xvsHwVMB7d4Ao8eGnZ2dnvvvtuVlYW9pDH4XyyYvmUsWOgWR3odgwGw8IP12LVO5/P3v3Nqkerd4AjuUL55+mUAz+dKi6tah8kkUkRMREjJ4929XbtpO9rzhcMixk8LGZwVV3d3fv372ZltbSKEEJ6vb6oqKioqOj33w97enoFBQX17du3x1TyjdX1R7b/gG3Pf2+NKYuF3U4CQO9EpVEGx0acPnEBIXTy1KmVK1finQgAADodFPA9k0Kh2LBhw549e7Tah8tijx8+/LPVq8z4fHyDAfByPt+561TyeYQQjUrZsWWFrbU53okAqqpuOPTH+eOnUiTStvZBFocVN3ZI/OsJAosuuiPd3tra3tp6bEJ82YPKe9nZ97Kzscb1Op0+Ly8vLy/v0KFDXl6eQUH9+vTp090r+X3rv1EplAihgUOGhgwYiHccAPAX2j8wNeW2WCQpKy3Nzc3x9YUVGQAAPRwU8D3QtWvXFi1aVFJSgj10tLX9at3amAH98U0FwEv74+zZb/btRwgRCIQNa+cF9oFJknjS6w03bmX9diQ59UbGo7PlHVwdhr0xfFBCFJWGQ3MNAiK4Ojq6OjqOHzGiuKzsblZ2Rm6OrE2OENLpdLm5ebm5eSQS0dPTKzAwMCCgL5PJ7PqQr+jKieT71+8ihFgczuylK/COA4BRoFBIsUP6Hz+aiBA6feq0r48vgmmGAIAeDQr4nqahoWHUqFFqtRohRCIS506d8uHid0wYMNkYdFd3s7KWfPQx1lNg3qyxo4ZF4p2o9xJLZMdPXfn9+IXKqvr2QSKRGBYdNnLyKA9/TxyztSMSCJ6urp6urpPGjC4qLbubnZWZm9cmxyr5f67Ju7u7BwQEBAT05XK7xyKaEqHop017sO1Zi5dxeDx88wBgPPqF9bly+XZLs7CysjIjIyMwKAjvRAAA0ImggO9pamtrserd0dZ2/5bNQf4wlwx0YzX19dOXLlOpVAihoTGhS+ZPwDtRL5WdW3r4j/Nnk2+o1Jr2QRaHFTM6dszUMZa2Vga9XqEwrm5eJCLR293N291tytgxBSWl6Tk5mXl5bW1yhJBery8sLCwsLDxy5Iizs3NAQEBAQIC5uVE3KPrxi91SkQQh1DckbPCw4XjHAcCIkEjEuITI3385hRA6ffp0QEAAgUj898u0Wq1SoZDL5XKFQqlUKuRylVqtVCrVKpVcodBqNGq1WqVS6XQ6uVyOEFIqlXqDARvHvgI28h9JyCQSlUpFCJHIJBqNjhCiUihkMplGp5GIJBMTE2ybQWfQ6XRsw+RvpqamFFgbCADwHKCA77HCggKhegfdmlyhmLZkaWNzM0LIx8t50ycLof9iF5PLlWeSrv9+7EJ+YcWj467ervGvJwwYEkmlUU1MjH2CD4lI8vXw8PXwmDJ2bFFZWXp2TmZurrStDSFkMBjKysrKysqOHz9ua2vTp0/fgIAABwd7vCM/LvPqnaunLyKEaHT6gvfW4B0HAOOi0+ld3e3NzPnNTcL6+voffvhBIBDIZLK2tra2tjZZW5tCLm+Ty9UqY1+vgUKhmJqamjJN2Sw2i8ViMplMFovL5XLYbA6Hw+ZwuuO9PwCADgcFPADAGGFt57PyCxBCZgLuzq0r6S+7hDh4CfmFFUf+vPhX4nVZm6J9kEKl9I8bkDA+wc3HHcdsL41EJHq7uXm7uU0eO6akvCIzLzczJ1coFmPP1tTU1tTUnjt3js/n9+nTp0+fPu7ubmQy/r8lVQrlvo+/xbYnzZ5naWOLbx4Aup5Op5NKZVKpVCaTqVQqsVgskYj/rtDbsAvmmoeXydGdO3c6PACRSKTTnvA7iEKlaB6Zl4QQ0hv0SuVLninQaDQikUgkEtWgmie+gEwm83g8LpfLF/B5XB5fIDATCMzMzLg8njF8WAEAugb8bQcAGKNNu3afPn8BPWw7v9zKAhZQ6AptcsXZ5JtH/7yUk1f66Li1g82QsUOiRwxmcVh4ZetARALBw8XZw8V5wsgRD6prMnNyMvLyG5qasGeFQmFKSkpKSgqdTvfx8fH39/fz88Oxff3v2w821dQjhFw8vUZNnIxXDAA6m15vkEllYolYIpGIxWKZVCaRSqRSqVQqbWtre+buFBoiUZBO84SnqFSqKYNhwmCY0OkMBp1Bp9NoNAaNzmDQ6TQqlUqlUakMGp1MItNpVDKFQiGTqRQKhUImEYm0J9Xtz0Or1ak1aoRQm1xhMOiVSpVKq9FotAqlQqVSK5VKpUqtUCkVSqVcoWhTKBQKhaytTdYmV2ue9B4efk1tU1NTU1MTKv5/40Qikcvlmpmbm5ubWVpYmltg/7OAqh6AHgn+YgMAjM6JpOSte/dh2xvWzuvr3y2v93YvmdnFx05ePnf+plz+z63sJDIpZFDokHFD/YP9e+T9CwREcLKzc7KzG5uQUN/UdD8v735efkV1Nba4ulKpTE9PT09PJxKJzs7Ofn5+fn5+dnZdegG8PK/43E9/IoSIROLC1R+QSKSu/O4AdAasUBeJRWKxSCQSi8ViiVgiEotkUqlOr3/Rr0aj0pimJqYME1MThlKhz8woQwiZMBhzJk/isNmmDIapiQmFgsPhLplMIpMZWJgX2lGt1kjb2qQymUTWJmuTicQSiUwmkkokUplQJJJIpfp//ZT0er1QKBQKhUWFhe2DRCJRIBBYWlpaW1tbWVnZ2NpYWVlhd+YDALo1KOABAMYlK7/gnbXrsLbzc2aMhrbznUoolJw6d/XYyZTS8upHxy1treLGxEWPGMwVdI8m7a/OytzcKioqPipKLJVmFeRn5xXkl5ZqNBqEkF6vLy0tLS0tPXnyJJ/P9/Pz8/Hx8fLyonXygnl6nf5/677W6XQIoZFvTHL19OrUbwdAh1OrNCKRqFXUKvpba2urRCLB/lQ/tKqAiwAAIABJREFUJyqVwmayWEwm09SEzWRyWWxTUwaDRmeaMk1NGOT/f1ZL0nqirKJGrlBU19f7eHTLk79UKkVA5Qp4T/7s1en0EqlUKBK1iEStIlFLa2uzsLW5VSgUibTa//dT1ev12OX6nJwcbIRAIAgEAhtbW1sbGzs7O3t7ezMzM1h1D4BuBwp4AIARaWppmbZkqUKpRAgNHtTv3YWT8E7UM+l0utQb94+funzlWsajx3wUCiUkKjR2TFxPveT+PDgs1sCQ0IEhoWqNJr+kOCu/MKewQCyRYs8KhcLU1NTU1FQymezm5ubj4+Pj42Nra9MZSc798mdZbhFCyMLaevKctzvjWwDQUVQqtVAobG1tFbWKhK1CoVAoEomeZ+o7hkQkMplMLovFZrE4LBaLxWQzTTlMFovJpP3/3ux0Ol2j0ep02id+nfi4sD37jyOEklNTB4aGMnpc8xQSicjjcnhcjityfHRcrze0ikWNLS2NzS2Nzc0Nzc0NTc0tra2PXq43GAzNzc3Nzc1Z9+9jI3Q6Havk7R0cHB0crKysntjAHwBgVKCABwAYC7VGM2PZ8pr6eoSQm4vd5o2LiMReWkN2nqKSqhNnrpw+d61FKH503N7FPnZ03KBhUT3jLvcOQaVQ+nr79PX2MSBDZXVNTlFRdkFBZU0tdkCs1WoLCgoKCgqOHz/O5XK9vb19fLy9vLw6qk10c13j79sOYtvzVrxHf8FZuAB0Hq1Wh9XqLS0t7eX6c9bqRCKRxTTlsthcNovL5nDYLDaLzWWzmKbMDvm8d3Kw9nR3LCx+IJO1Xbx2bWRcbAd80e6ASCQIeDwBj+ft5tY+qNFq65uaGhqb6hob6xobq+vqm4XCR0t6pVJZUlJSUlKCPaTT6Q4ODo6Ojs7Ozk5OTlwer4vfBQDgeUABDwAwFqs//+JWRiZCiMNm7vp6lanRr0/WjQiFkr+Sr5/862peQfmj4yZMkwFDIgePjHH37ZZzTbsGAREc7ewc7exGxMRIZbK84uKcwsK8khJsVXmEkEgkunnz5s2bN4lEoq2trbe3t6enp7u7+6vceXtg4w6lXIEQGhAT1y9iQMe8EwBekMGAJBIJdn91S0sLVrRLn3Qb9r+RyWQ+h8PlcHhsFpfD5XE4XDaLw2aRiJ3byiE+NqyopNJgMFy4di0qPJyFXwdK3FHIZHtra3tr6/YRtUZb19BQU19fXV9fVVNbVVf7aM98pVJZVFRUVFSEPeTz+c4u2P9cnJ2dujQ6AODpoIAHABiFA78f+emPYwghEon09RdL7G0t8E7UEyhV6sup906fvXYt7f6jU+UJRIJfP7/oETHhg8OpnXwjdw/DYjLDAgPDAgP1BsOD6uqcwsL84pL2vnd6vb6qqqqqqio5OZlMJru4uHh5eXl6ejo4OJDJL1C03Dp/9e6lGwghEyZz9rvLO+vNAPD/qVWaFmFLq7C1uaVZKGxpaREKhUKt9smT1R9FIZP5XC6Py+GzOVwul8fh8Lkclik+lbOtjbmfr0t2TqlSqUpOTX19+DBcYhgnKoXsaGfr+HczToPB0NQirKqrraqpraiuflBdg93ChsFO3Ny7ew8hRKXR3N3dPD08XV1d7ezsKFT4xQEAbqCABwDg78a9ex9s+hLbXrV0akSoP755ujudXn/nXt7ps9fOX7796ELuCCFrB5uoYVFRw6LNrMzwitczEAkEZ3t7Z3v7UXFxcqWioKQ0v7gkr7i4pbUVe4FWq22/lkWjUV1d3dzd3Tw8nl3MK9rkBz/diW1Pn7+IJ4D/UqBTSMRSYWtLS4uwpbkFq9vFEvEz9yIRiVw2W8Dj8rhcPo/L53B5HA67g+4c6ShDY8Jz88r1en3KzbTYAQO4HDbeiYwUgUCwMBNYmAn6+fsjhAwGQ0NTc0VVdXl1VXllVXVdXftUC7VKlZuTm5uTixAik8mOTo4e7h4enh6uLq5kCgXP9wBA7wMFPAAAZzX19bOWr9RotQihsSMHvTkZrpa8vJy80r+Sbpw7f7OxqfXRcVMWs39c/6jh0Z7+nnhl68FM6IwgP78gPz+EUFNLc0FpWUFJaWFZqezvOfYqlTovLy8vLw8hRKVSXVxcXF1d3d3dnZ2dqdTHj31/33ZQ2NCMEPL0848fM65r3wromXQ6HXY5XSgUNjc1Y3W7Wq1+5o6mDIYZn8/jcgVcLp/HFXC5HDabZPR9zizMuEF9Pe5mFGg0mrOXL08ZOwbvRN0DgUCwsjC3sjAP7xeIEFKr1RVVNSUPKsoqK0srHrRfnNdqtaUlpaUlpefOnaNQKM4uLl5eXl5eXo4ODtADD4AuAAU8AABPKpVqxrLlzUIhQsjf13X9+3PwTtQtlZRVnzt/82zSjQdV9Y+OkynkoP5Bg4ZFBQ3oR4GLJF3CXGBmLjAbGBpqMBiq6+sLSkuLS8uKKsrbbzRVq9VY9zuEEIlEcnBwcHFxdnV1c3Fx4XDYZblFib+ewJ56e+VqOBoGL0HepmhuaRYKhcIWYXNzs7BVKBaLn3nXOolI4nE5Ai5PwOPyeVwBj2fG49G67UzpuMGhGdnFOq3u+p27QwYONBfw8U7U/VCpVA9XZw9XZ4QQARGaWlsLS0sLSkoKSkqksoc9CzUaTVFhYVFh4amTJ02Zpp6eXt7eXj7ePjw+/MAB6CxQwAMA8LR846cZObkIITMBd/uW5bR/XY0E/6Gyqj7xQtrZ5BtFJVWPjhOIBO++PgPjB0bERpiyjGtqa+9BIBCw9lFDIiP1BkNlTU1xeXlRWVnJIxeydDpdeXl5eXn5xYuXEEICPr/hepFep0cIjZo42dndA883ALoDnU4nEolamlua/+4wJxS2KBTKZ+7IYNAFXK6AxxdwuQI+T8DlctgcUg9a+IPHZYUF+95Iy9LpdGcuXJg18Q28E3VvBCLBwdbGwdZmcP8IlUpV19BYVF5WUFpWXFbeJn841ahN1pZ+7176vXsIIRsbG19fXx9fH1dXNzIZyg0AOhL8jQIA4Gb/oUOHT55CCJHJpG83vWtpDifsn8uDqvqkC2lJF2/lF1Y89pSzh3Pk0IEDhgwQWMKN00aESCA42dk52dkNGThQbzDU1NeXlJcXV1SUVjwQS6XtL6vJKm2rb0EI0UxM6iTyn3762dra2srKysbGmsfjw6qKQCqVCrFF2/5uMScRi3XPurROJBI5LBafyzXj8/g8noDLE/B4pgx612TGUcygfnfT89VqzZ37WfFRUTZWlngn6iEIBIKNlaWNlWV0RIReb6iurc0vKckvKSl9UKnRaLDX1NbW1tbWnj9/nk6ne3t7+/v7+/n7d9QqmwD0clDAAwDwkZaesXbzV9j2muVvBgXAvdnPUFZRe/7SraSLtwqKHjz2lK2j7YAhkQOGRNo42uCSDTw/4t9X5gf3748QahK2lD2oLKusLCgozr/6cDVmR9++BoRqampqamqwERqVZmllaW1lbWllaWFhIRAIoJ7v2eRyhahVhPWWE7YKW1tbhcLnumudRqXyuVwBjyfgcQU8Hp/L5XO5ZFLnrtxmnFhMkwHhfS6n3tPr9SfPX1gwfSreiXogIpHgYGfrYGcbHx2lVmuKy8vziotzC4vqm5qwFyiVyoyMjIyMDCKR6OTk1KdPnz59+1pZWeEbG4BuDQp4AAAO6hubZq142Lhu3MioKROG4p3IeOUVlF+8cvf8pdslZdWPPWVlZ9U/bkBEbH8ndyc8ooEOYM4XmPMFYYGB39zeatDqEELuPn4hYRG1DQ1tin9WEFCpVZWVlZWVldhDMplsaWFpbWNtaWnJ5/MtzM1hOcDuS96maBW1CoUtra0iUauoVdTa2tqqUCieuSORSGQzWQLePx3m+DweXou3GaeoAYE3b+colaqsvLyKqmonezu8E/VkVCrF19PD19NjwsgRzcLWnMKi3MKCwtIytUaDENLr9WVlZWVlZSdOnLCwsAgIDAwICHBydEQEOBcJwIuBAh4A0NXUGs2sFSsbm5sRQn7eLh+9PxvvREZHp9enZxZeTLlzMeVuTV3TY89a21uHx0RExPZ39nDGJR7ocJlpmTcu3kAI0RmM1e+vNTM3RwiJJNK6xoa6xqa6xob6xibVI1dftVptTW1NTW1N+wiHwzE3Nzc3Nzf72wutPA+6gE6nE4slIpFILBa3traKRCJRq0gkEqnUqufZ3YRB53O5Ai63vSc8j9NLL60/PwaDFhUZkHThlsFgOJGU/O6ct/BO1FuY8XnREWHREWEajbawrDQrLz+roFAkfrhOYWNjY3JSUnJSEo/H69u3b1BQkKurK/TsBOA5QQEPAOhq67ZsvZ2ZiRDi89jbNi+DxnXt5HLltbSsy6n3rlxLF4lljz1r72IfNjg8fHCEo5sjLvFAJ9FoNAe/+R7bnjx9Jla9I4S4bBaXzfJ2c0MIGRBqFYnqGpsam5vrmpoampsea1QmFovFYnFJycNJ+EQikc1mC/gCgZmAz+cL+AK+gM9kwrXZrqDT6aRSqUQs/btaF4vFYrFILJVJn9kKHkOj0vhcDp/L4XG4fB6Xz+XyuRw6ldbZyXukAeF9r6dly2TygpKSwpJSTzdXvBP1LhQK2c/T08/Tc7LBUFVTe7+g4H5uXnVdHfZsa2trSkpKSkoKm8MJCAjoFxTk5uYGlTwA/w0KeABAl/rj7Nn9hw4hhEhE4lefLba2gl5rqK6+OeVaRsrV9Nt3c1VqzaNPEQgEV2+3sOiwkKhQW0dbvBKCTnX8h2O1D2oRQk4uriOfsvA7ASHsZmZfD3dsRCKTtYol9U2NdY2Njc0trSLRo83M9Hq9SCQSiUSlZaXtg1Qqhcvl8Xg8LpfL5/O4XC6Hw2WzWSS4ivsyDDKZrLm5WSqRSmUSiUQqkUiwUl0ulz9noY4QYjDoPDaHx2HzOFweh4P939SE0anRexUalRIzqN+ps1cRQieTz78HBTxOCISHd8uPiottbGnJzM3LzM2rqKrC/rJIxOLUK1dSr1xhczhBgYHBwcF45wXAeEEBDwDoOvklJcs+3oBtL1nwRniIH755cKTT6+9nFadez7hyPbOw+PGmdCQyyTfQN2RQaEhUqMBCgEtC0DXqqupO/nQCIUQgEBYsfvf5a2k2k2nGF7g5O2rUGoSQVqdraW1tFrY2CVtahK1NwhaxRPpYf3K1WtPY2NjY2PjoIJFIZDFZbA6bw+Gw2Wxsm81iM1lMk15fRsrbFLI2mUwma2uTtbXJpVKpTCaTyR7+W6fTPf+XIhFJbBaTx+Fw2Swum8PlsHlsNpfD6b4LrXcjYSG+V29ktoqkZZWV9/Py+vr44J2ot7MQCIYOGjh00ECxRJqRm5uenVNSUdFeyWPX5LWSerxjAmCkoIAHAHQRqaxtxrvL5QoFQigmKnjOjNF4J8JBi1B87eb9qzfuX0/LEksenyRvyjINjAgMHhgSGBFkwjTBJSHoYge/PoAtvDRk2AhP75evK8gkkqWZmaWZGUIPL9Hr9LpWkbilVdQiam0RiYQikUgkbvtXazS9Xi+WiMUScVVV1WNPkUgkJsaUyWQyTUxNTBgm2Abjb920/5ROp1MolEqlUqGQK+QKuUIhl2P/kMvlCplMJpe3KeSKZ67Q9m9kMpnNNGUzWRw2i8tic9gsDovNYbNYpsxu+rPqAcgkUtzgkKN/XkIInUw+7+/lDYs4GAkOmxUdER4dES6RStNz8tKzs9sreeXfdwmJRCJcMwJgdKCABwB0BYPB8M66daUPHiCEHOytvli/gNBrDma1Wl1GVtH1m/ev3rxfUPTAYDA89gJre+t+kcH9IoO9A7xhMnOvcv38tYyb6QghNof75qw5HfvFSUSSGZ9vxuc/OqhSq0VisVAibhVLxGKJSCoVS8QSqUz7pIvJOp0OmxP+tG9BIBDodLoJw4TOoNNpNCqNRqPRGAw6lUqj0agUMpVKo1KpVDKZTKFQqFQKiUSiUmgEIgHbfpV3p9cbVCoVQkij1ugNOpVKrdfrVSqlTqfTaLRKpVKj0Wg0GrVapVAo1Wq1SqlSqVVqtUqpVCmVCvX/v1flRZGIJBaTyWYxmaYmLBMTNovFZjJZLBabyWSawKk3YxQU4HXlWmZjk7C2vuFOZmZYUCDeicD/w2axsKZ3IrEkPTv7TlZWdkMF9pRM9vjJbgB6OSjgAQBdYc/Pv5y5cBEhRKfTtn25jMXq+ce4ZeU1N25l37iVfTs9Ty5XPvYshULxDvQJjAjsN6CftQMs3t4bKdrkP277AdueOWcek8Xqgm9Ko1Itzc0t/+6ThzEgJGtrE0ukEplM2iaTSGVSmUwqb5NJZTK5Qqd/6kRxg8GgUCieZ8Gzp6FQKI92y6dSaf8+tafVah6drK5Uqv59FqzDUSkUU4aJiSnDlGHCMjU1NcH+yWCxmEwTpoDPRQjpdAbN8zWQB7gjEghDY0N/OZyIEDpz4WK/Pn1gmQbjxOWwYyIHxEQO2LSpoeZBOt5xADBGUMADADrd7czMDd98i22vX/OWp7sDvnk6T2NTa9qdnLQ7OTdv5TQ0Cf/9Agsbi4DwwMD+Qf79/GkM6Cndqx3ee7i1uRUh5O3rNzhuKI5JCAixTE2ftn54m1zRppDL2uRyhbxNocAeKhVKuUqJ/fOxfvgvBLtO3v7wVb7Uc6JSKHQalU7DJg3QGHSaCZ1hYsJg0OkmdDqDQcfuEKCQn3qA1HtmD/Uwfj6udnaW1dUNTULhtTt3oiPC8U4E/gsskQjA00ABDwDoXC2totkr39NotQihCWNjxowYhHeiDiYUSm6n5925l3frXl5Zec2/X0Bj0HyD/PqG9Q0IC7RxhIvtACGEKorKE4+eQwiRSKT5i9815prQ1IRhasKwEDy1maIBIZVapVCqsGnqSrVapVZrNFq1Rq1SqjQ6nVarVapVBr1BqVYbDHps+rpaq9VrdQghjV6r02ifJwmRSKRSqAghREB0Gg0hRCIRyWQKmUSkkClkMplMIlFpVDKJTKWQaVQqhUyhUik0KpVOpdFoFCqFRqdTSUSoCnopAkIJseH7fzyJEDp7+XL/fkFU6CAIAOiGoIAHAHQivV6/4IMPahsaEELenk4frJqJd6KO0dQsupdZcOde3p30/NLymn9P6CUSia7erv4hffqG9fXw8yRT4MMW/MNgMOzfsg9r1DRy7GuOTs54J3olBIToVBqdSkNsvKMA8J/cXe1cXWxLy2okEumlGzcToqPwTgQAAC8MjikBAJ3o2/3fX7x2HSHEYpp888VSGpWCd6KXV1FZl5FVdC+jID2zsKKy7omvsXO28w/u4x/i7xPoa8p68oRkAC6fvlSYXYgQ4vEFE6e+iXccAHqRhLiI3fuOGQyG5Cupg8JCTRi9fa1EAEC3AwU8AKCzXL9zd9Ou3dj2xrXzHOyt8M3zojQabV5BeUZWUcb9ovT7hS3CJ/fitnaw8evn5xvk6xPoyzPjdXFI0O3IJLJfdv+Cbc9ZsMgEmpYD0IUc7Cx9vJxz88vkCkXyldSxCfF4JwIAgBcDBTwAoFM0tbTMXb0aW0V52qSEobFheCd6LvUNLVk5JZnZxfezi/MKylVPWmuKQCDYOdt5B/h4B/j4BvnwzPj/fg0AT/Pr7l+kIglCKKBf8ICBMIMXgK4WHxuWX1iu1xsuXb8xuH9/DrsrFoAAAICOAgU8AKDj6fX6t9d80NDUjBDy83FdtWQq3omeSiqV5+SXZueVZeeWZOeWNja1PvFlJDLJ1dvN09/TO8Dbq683iwMHfOBllOaXXjx1ASFEJpPnLngH7zgA9EaWFvyAPh7pmYVqjeavS5emjB2DdyIAAHgBUMADADret/u/v5KWhhBisUy+/mIJxZhauEll8vzCitz88tz8stz8ssrqhqetKc3msj38PT39PT37eLl6u1Jp0K8YvBK9Xr/3y/8Z9AaE0GsTJtna2eOdCIBeakhM6P2cEp1Wd/3O3bjISAuzpy6yAAAAxsaIjqoBAD1DWnrGl7v3YNsb175tZ2OBb56mZlF+YUVBUUVeYUVB0YOqp1fsJDLJ2cPZzcfd3dfd3c/D2t66i6OCnu38n8llBaUIIUsrq/GTp+AdB4Dei89lhwX73kjL0ul0py9cmD1pIt6JAADgeUEBDwDoSEKRaN7qNVqdDiE0ZcLQoTGhXRxApdaUldcUlVQWlVQWFFUWFj8Qtkqe9mICkWBjb+Pq7erq4+bm4+7s4Uzpzn3ygTGTiqW/7z2Mbc+Z/w6VSsM3DwC9XGxU8N2MfLVKcy8re+igQfY2cMYWANA9QAEPAOgwBoNh8Ufra+rrEULenk7vvTuts7+jVqurqKwrLauuqKwvKq0qKKyorKrHOuc9EVaxO3u5uHq5uni5OHs4M0yhBzjoCj/v+EkqliKEgsPCQ8Ij8I4DQG/HNGUMjAi4mHJHr9efTE5+Z+YMvBMBAMBzgQIeANBh9v12KPFyCkLIxIS+9fMl1I6+mi2VySse1JVV1JRX1JY/qCspq66qadBqdf+xC5VGtXexd3J3dvJwcvZ0cXRzpDPoHZsKgGcqzC5M+esyQohKpc2dD73rADAKgwYE3LydLZcrcwoKiysq3J2c8E4EAADPBgU8AKBjZBcUfvz1N9j2+jWznRxeaTqiUqmqrG6srK6vrKp/UFlfUVlX/qCuuUX033sRCARza3MHFwcHN0dHdydHN0dre2sikfgqSQB4RXq9fv+WfVjnhdcnTra0hpm6ABgFOo06eFC/vxKvI4ROJCatmv823okAAODZoIAHAHQAuUIxZ9V7KrUaITRmxKBRwyKff98Wobi6prGqprGmtrGyurG6pqGyqqGhSfjMHQkEgpmVuZ2Tnb2LvZObo4O7s6WtBVxgB8Ym6VhiRVE5QsjK2mbcBOiVBYAR6R/mfz0tSySSllY8yMrL7+PjjXciAAB4BijgAQAdYM0Xm0oqKhBCjvZW696b9e8X6PT65hZRTW1TXX1LXX1zXX1zbX1zdU1TTV2TUql6nm9BpVFtHGysHWxsHGzsnOxsHG1sHG3by3UajYoIRJVS2XHvCYAOIBaK2nvXzV34DpUKixECYETIJNKQwSFH/7yEEDqRnOzn5UUkEvAOBQAA/8fefcc1de5/AP+eJGwIgUBYIoiKA0RlieLeVuverdW2jtbacTtux+399ba9nXbXqnW2jtZV9wD3niiCiuy9ISEhzISc/P6IpV6lrQN5TsLn/eq9ryeHhHyEQ8iH85zn/BUUeAB4WDtjD27csZOIrKwkb782OyU9t6xcXVKqLC5RlZSqikuURSXKsvKKvz5Z/XZWVlYKb4WXr5enr1fj/7t5uHEc3leBmVm/ZH21tpqIevWJDovoxToOANwptEfnE6evlpapCotLLsTH9w4LZZ0IAOCvoMADwH2oqa0tKSsrVarKlMristIypSo9O2fvkSOmj+r1Dc+98tl9fUIXNxd3L4XCy13h7eHh46nwVnj6eMoVcg7HQMD8JSfcPHngBBHZ2Ng8u2Ah6zgA0AQRx40c2mvdrweIaM/hw+HdQ6wkeHsMAMKFVygAuKVep1Op1cqKijKlqrxCpapQK9Xq0vLyMpVKqaooUypLystrH2iOusRK4uruKlfI3b0Ubh5ubh5ubp5uCi+FwtsD110HS2UwGFZ/serW2nXTZyo8PFknAoCmde0S0NbXMzevWFWhPnn+wpC+0awTAQD8KRR4AMunrapWadQVanWFprJCo6lQq1UajUqtrlBrVBq1Sq0uV6lUak11Tc3DPIutna3cw81FLnNVyF3dXF0VrnKFm6u7i1zhJpPLMPsdWpvY32Ky07KJyMvbZ/ykqazjAMCf4ohGDev945odRHTg+PE+4eF2tjasQwEANA0FHsAs1dTWqisr1ZWVGq1Wo6lUV1ZWaDTqykp1pVZTWWn6UIVGo9Zo1JWVDYZ7Pfn8L4jFYpmLi7OLi5u7u8zV1dVNrjfSjkOHebGYk0hefWVWVK+Qh38WAMugUam3rNxsGs99/gWsXQcgcAH+3p06+qWk5VRVVR86eWrs8KGsEwEANA0FHkAoqqqrKzSV6spKze/N/FZFr6zUVN4am/6r1Gp1en0zPrXEysrZ2VkqkznLZM4uLi6uLs4yF9PY1U1u6u0yF5fbH1JTWzv/n2/xjo5ENHJYb7R3gNs1rl0X1acv1q4DMAujhkelZeTyvPHw6dMDe/eSOjmxTgQA0AQUeIBHS6VWV2g0FRqNSq1RazSm4+QVGo1aU9k4NjX2ZjlOfgcnZ2cnJycnZ2dnZ2cnZ2cnZ6lU6uzkLJU6y5xlzlJnZ2eZi7OLzN7B4X4/8/drfy4sLSUiXx/Fk9NHNntyAPOVci3FtHadtbXN0/OfZx0HAO6Jl4db926B8QkpOp1u35FjM8aPZZ0IAKAJKPAAD6i2rq5cVVGmVJZXVKgqKpRqdWVVVbmqQllRUVperlKrTaea8zzfvM/r6OTkJJU6Sp2cnKROUqmjk5OjVOokdXKSSh2dbg1Mvd1RKn1EZ56fOH8h9sQJIrKykry8cLq1FRaiA7iF5/lVi1ea1q6bPGOmhyfWrgMwG8OHRCbeSDc0GE5fujSkb7TCTc46EQDAnVDgAZpWr9OVliuLSkvKlKri0tIylaq0XFmiLFeqKkwLs9fU1jbXc906Ti6V/l7FnU0t3Unq5CiV3ho7SR2dHB9dJ7935aqKr1euMo1nThnu54t+AvCH2N9islOziMjL22fC5Gms4wDAfXCVSXuFB509n2gwGHYdPDhv5gzWiQAA7oQCD61auUpVWFJaWFJSUFxcXFZWWFJSUlZWVFpaWq5UqdUP+cmdnJ2dZTKps7PpPydnqbOzzMnZdHhcKjUNpFInqbRZ/i0tw2g0frp0WWVVFRGFBHcYPRLX2gH4g6ZCs3nFJtP42ecWWmFyCoC5GTog/HJ8cn297sq16zn5BX5tfFgnAgD4HyjwYPn0DQ0FxcV5BYV5RYV5hUWPL3iUAAAgAElEQVS5hYUFxcUFxcUFxSX19fUP8Akb12N3kctdXF2dZTJnmUzm6urh5eXi6ipzdTGtCceJRM3+b2Hut/0xV65dIyJHB/sX5k/iCBeHA/jDhiXrTGvX9eoTHR4ZxToOANw3Bwe7/tE9Dh29aDQad8TEvDL3WdaJAAD+Bwo8WBRlhTorLzcrLy83vyA7Pz8nvyCnoKCopMRwnyeiS52dXd3c5O7ucjc3F7nczd3d2UVmunyazNX1jvXYG9nbO4jEIiKqqqoio7EZ/j0Ck52Xv2rTraOL858e5ypzZpsHQFCSE5NP7L+1dt2zCxayjgMAD6hfnx7nL97QVlUnp2ckpaZ1DezIOhEAwB9Q4MFcVVVXZ+TkpGVnp2dlZ+bmZuTkZuXmarTae3y4SCRykcs9vLzcFQo3D4XCw9NNoXBTKNwU7nJ3d1y0+W76hoaPlyzR6XRE1D+6Z+/IbqwTAQgIz/Orv1h1a+266TMUHlgbAsBc2VhbDR0YvmPvCSLaERPTuUMHkQjTzQBAKFDgwTyUq1Q309JTMzNTMjNTs7LSsrKKS8vu5YESKytPLy8Pb29Pb28PLy8PL09PL2+Fl6fc3V0iwf5/H37avDU9O4eI3OSyZ596nHUcAGE5+Fusae06Ty/v8Vi7DsDMRYR3PXXuarlSk1dYFJeYENmjB+tEAAC3oMCAEGmrqm+mpyWlpSelpiZnZCSnpysr/n5JOUcnJx9fXx9fX2/fNl4+bbza+Hi3aeOuUFjkuegt7Fpyyua9e4lIJOIWLZhsb2fLOhGAgGgqNJt+X7tu7vMvYAoPgLkTi0QjhkZt3BxLRLtiD/UMDrbCH/0BQBjwYgSCkFtYeD05+VpyyrXklKTU1JyCgr++v5W1ta9f2zZt/dr4+fn6+/n6+/v4+jrLZC2TtrWpqa397IelpgvajxnZL6hzAOtEAMKyYcn6am0VEfXqjbXrACxEt6AOvm2u5uWXKCsqTp6/MKQvrroCAIKAAg8MGI3G7Pz8hKSbCUlJV28kJd68qa6s/Iv729rZ+bVr59++vV9AgF+7dm0D2nn5+IhwXL2lLF23obC0lIj82npOnzyUdRwAYUm9lnJi/3Eisra2eQZr1wFYCo5o1PDeK9bsJKL9x473DgvD7DMAEAIUeGghZUrl5WvXr1y/fuXatfjrN/6isYtEojZt2wYEdmwfGNiuQ4d27Tt4enthGjwr56/E7z96lIisrCQvLpiKOYQAt+N5ftXilaa16yZNm+HhibXrACxHe3+fzoF+yak51dXVscdPTBg1gnUiAAAUeHhk9A0NiUk34xIT4xITLyUk5BUW/dk9be3sAjp27Ni5c4dOgR06d27XoQPOIBUIjVb7xYoVpvGUCYP9fFFOAP7Hwe2xWb+vXTdhCtauA7A0o4b3Tk3P43n+6NmzA3pH4fqpAMAcCjw0J41WeyH+6sWrVy9cib9y/XpdfX2Td7O1s+vYuVNg166BXboEduni6++P+fDC9M2qNaoKNRF16th23Oj+rOMACItWo92ycrNpPPc5rF0HYIE8FfLQHp3irtzU6/W7Dx2aM2Uy60QA0NqhwMPDKlepzsZdPnv58tm4yzfT001Lnd1BJBL5t2/fpVu3Lt2COwcF+QUEiMXilo8K9+XwmTMnzp8nIhsb60XzJ4s4/JEF4H+s/36dVqMlosioPuG9sHYdgGUaPjgy4VqaXt9wMf7qkL7Rvl5erBMBQKuGAg8PQlmhPhsXd+rSpdMXL6ZkZDZ5Hydn5+DuIV26hQSFhHQK6mrv4NDCIeFhlKsqvl+91jR+avpITw83tnkAhCY9Kf34/mNEZG1t8yzWrgOwXM5Sx369ux89eZnn+R0HYl565mnWiQCgVUOBh3tVU1t77vKVE+fPn7p48XpKapNH2r18fLr17NmtZ4/gHj3atmvHcVzL54SHZzQav/hxhba6moh6hHQcNqQX60QAwsLz/IpPlxt5IxFNnDrdA0fkACzagH6hFy7fqK6uS0pNu5me3qVDB9aJAKD1QoGHv8Lz/LXk5GNnzx09e/bS1QSdXn/3fXz9/UNCQ0NCQ7uHhSk8PVo+JDS7fUePXbx6lYjs7W2fnzuRI/whBuB/NK5d5+HlNXHqdNZxAODRsrWxHjIgfPf+00S040BMpxdeEInwmxEA2ECBhyaUV1QcPXP2yKnTx8+dK6+ouPsOXm18eoZH9IgI7xkZKXfD5GqLUlxWtnz9BtP42VmPY8VdgDtoKjSbftxkGs97bhHWrgNoDaIigk+fv6ZSaXILCi9dvdortCfrRADQSqHAwy1GozEhKengyVOHTp26eiPp7hnyzjJZz8iIsF69Qnv18vLxYRISHjWeN362dHlNbS0RRYZ17R+NNygAd9qwZH21toqIIqJ6Y+06gFZCLBaPGha1cXMsEe06eCi0WzcrK7yLBgAG8NLT2tXU1h4/dy72xMmDJ0+Vlpff8VGJRBLUvXtEn97hvXt37NSJw8XeLN2OmJiEpCQikkodFjw9nnUcAMFJTkw+sf84EVlb28x9bhHrOADQcroFdWjrm5CbV6xSq4+dOTt8IK6uCgAMoMC3UmVK5YFjxw8cO3biwsX6u67WrvD07NU3OqJPn7Bevezs7ZkkhJaXX1S06tffJwbPGSeVOrLNAyA0PM+v/mKV0WgkoknTZnh4erJOBAAthyMaPaLPslXbiejA8eO9w8OcHHGFHQBoaSjwrUtWXt6+I0f3HTkal5h4xyR5kUgU1KN77379e/WNboflVVsfA2/8bOnyep2OiPr2DokKD2adCEBwYrYdyE7NIiIvb58JU6axjgMALc2/rVdw1/bXkzJq6+r2Hz06bezjrBMBQKuDAt8qpGRk7jl8eM+hw9dTUu74kIOjY0Sf3n0GDIiMjpY6Y7my1mvbvn03UlOJyEXm9OxTY1nHARActVK9ecXvU1Sex9p1AK3UqOFRSSlZvIE/eeHioD69FVjKFwBaFgq8JbuZnr4r9uCug4dSMzPv+JCbQtFnwIC+gwb2CA+XWFkxiQfCkVNQsHbLVtN4/jMTHB1w3gTAndZ//3NNVQ0RRfXpGxoRyToOALDh5irrHdHtzPkEg8Hw24HY52c9wToRALQuKPAWK+bY8S179t6xsU3btv2GDOk3eFCnoCCOwyVMgYiowcB/tnSZTqcjokH9w8J7dGadCEBwkuKTTsWeIiIbG5tnn1vIOg4AsDRsUER8QkpNbV3CjRsp6RmdOrRnnQgAWhEUeEtTWVl5a1BV1bjRv337AcOG9h8yBCe3w9227NmTnJ5BRK6u0jkzR7OOAyA4hgbDqsUrTGvXTZnxpLvCg3UiAGDJzs5mYP/Q/bFniWhHbOyb7Z/HQREAaDEo8JamqKiocWzq7QOHDfMLCGAYCYQsKy9v3bbfiIjjuOefnWhvb8s6EYDg7P11T15mHhF5+/iMmzSFdRwAYC86KuT8hesqdWV2Xv6lhMTIHt1ZJwKA1gIF3mJ5BgSs2baVdQoQtAYD/9nS5Tq9nogGDwjr0S2QdSIAwVGWlG9bc+u19LkXX7HCoiEAQCQRi0cOi/pl60Ei2hkT2yMoyNoKb6oBoCWIWAeAR4UT4ZsLf2PTzl2mBQ7d5LLZMx9jHQdAiH76Zm1dbR0R9RswKKRHKOs4ACAUId06tvX1JCKVWn3k9GnWcQCgtUDHA2ilMnJy12/fTr9PnrezxeR5gDslXEg4f+w8EdnZ2c+Z/xzrOAAgIBzRmJHRprPfY0+cqNRqWScCgFYBBR6gNWowGD5ftlzf0EBEQwdFhARjdUOAO+l1+pWfrzCNZ85+Wi7H1Z4B4H/4+Xp2C+pARHV19bsOHmYdBwBaBRR4gNbol5270rKyiMjdTTZr+kjWcQCEaMe67SUFxUTkH9D+scfHsY4DAEL02PDeVhIxEZ27fDnvtoWEAQAeERR4gFYnIyd34/YdhMnzAH+uOL9457pbPybPLXpZLBazTgQAQuQic+rbpwcR8Ty/bd9+1nEAwPKhwAO0Lg0Gw2dLl5omzw8bFGGa+wcAd1i1eIVepyeiIcNHdu4axDoOAAjXoP6hjo72RJSSnpGQlMQ6DgBYOBR4gNZl446d6dk5RKRwc3kSk+cBmnL28JmECwlEJJU6P/XsPNZxAEDQbKythw+JNI1/23+gocHANg8AWDYUeIBWJD07p3Hy/HNzJ2DyPMDdaqtrfv72J9P4qWfnSaXOTOMAgBmICO3q5SknotJy5fHz51nHAQBLhgIP0Fo0GAyfL1vWYDCQafJ8V0yeB2jCr8t/VZWpiKhLUPCQ4ZilAgB/T8RxY0b2NY33Hz6irapmmwcALBgKPEBrsWH7jluT591dZ00fxToOgBBlJmfE/hZDRGKJ5PkXXzFd4RkA4G91CGgT1CWAiGrq6vYcxiXlAOBRQYEHaBXSs3N+2bGTTCvPz51ga2vDOhGA4Bh54+ovVvE8T0TjJk5u69+OdSIAMCejR/SWSMREdPripYLiEtZxAMAyocADWD59Q8NnS29Nnh8+ODK4S3vWiQCEKPa3mNTrqUTkrvCYOnMW6zgAYGbkrrLoqO5ExPP8lr17WccBAMuEAg9g+TZu35GRc2vy/JPTcE4vQBMqylW//viLaTx/4Yu2WOIRAO7fkIFhjZeUi79+g3UcALBAKPAAFi4tK/uXnbsIk+cB/tLar9fWVNUQUVSfvhFRvVnHAQCzZGNtPWpolGm8ff8Bvb6BbR4AsDwo8ACW7PbJ8yOG9MLkeYAmXTl75dyRs0RkZ2c/b+Ei1nEAwIyF9ezi4+NORGUq1aFTp1jHAQBLgwIPYMnW/7YjMzeXiBTurk9MHcE6DoAQ1dfWr1q80jSe+dQcuZs72zwAYNY4jsaO6me6hkXM8RMVag3rRABgUVDgASxWambWrzt3EpFIxC2cNxGT5wGatGX15rKiUiJq3zFw9LgJrOMAgNnzb+vVIySQiHQ63faYGNZxAMCioMADWCadXv/Z0mUGnieiEUOigjoHsE4EIETZqVn7ft1LRCKRaOFL/xCJ8GsRAJrBqOFR1jZWRBSXkJiWnc06DgBYDrxTAbBM637bnpWXR0QeCkyeB2gaz/M/fvqjwWAgotHjJrTvGMg6EQBYCGcnx8H9w4jIaDRu3rWH542sEwGAhUCBB7BAKRmZm3ftJiKRiHth3iQbG2vWiQCEKGbbgfSkNCJyVyieeOpp1nEAwKL069NDLncmovyiopMXLrKOAwAWAgUewNLo9PrPlt2aPD9yaO8undqxTgQgRMqS8k3LfzWN5y98ydbOjm0eALAwErF4zKi+pvGeQ4e0VdVs8wCAZUCBB7A0P2/Zlp2XT0ReHvInpg5nHQdAoFYuXllbU0tEffr1x4XfAeBR6Bro37mTHxFV19TsOniQdRwAsAQo8AAW5UZq2ua9phW5uIXzJ1lbY/I8QBPOHTl7+XQcETk4Os57Hhd+B4BHZeyovhKJmIjOxl3Ozi9gHQcAzB4KPIDlqNfpP1+2nOd5IhozMrpzR3/WiQCEqFpbtear1abxU8/Mc3GVs80DABZM7irrH92TiHie/3XnTqxmBwAPScI6gEWxs7PjOI5tBhubW9f65jhR4xhaRuM339bGxmhk8Bt6+cZf8goLicjXRzFr+mhr61b0A266+pe1tRXrIK0Ux3Fm9MVf8cMGtVJNREHdQkaPHcf8dfuBcRxxJLKyakU/6YLw+w4j4ghffAY4EonF5nXBx+FDel1NTFVVVObkF5yPjx8Y1Yt1ovvXuNuLxNZWLfFqz/3+PRaJRA4ODi3wjADmAr94mpO1tTXzN4ISya3vKcf9MYaW0fjdF4vFLf/s8ddvbNu7j4jEItErL8y0s2tdf74xffGZfOVbO9Nuz3Hm8sVPuJhweNdhIrKysnr59TfFYjN+nTTt9iKReXzlLQ/HcfjiMyHiiDhz+srb2ojHPz5wzbrdRLT9wIFePXo4mm0jFXHEtcjLJsf98UcanA8IcDszfuMiQBqNhslx19tVV99a45Tn+cYxtAx7eweRmCOi6poaatk9obau7oOvvjZNnh83ul/bNora2rqWDMCcjY01caL6utb1rxYCe3s74jgjz5vFLldfV//df74zvVBPmfmkwsOzvr6edagHZ21tYyRer9OzDtK6cBxnY2tDRAbeqNeZ8f5jpmxtbRsaDAZDA+sg96dTB9/Ajn6paTlVVdW/7tr15MQJrBPdH5FIZGtrS0QNBoNOp2uBZ+R//xbzPF9RUdECz/jX3NzcWEcAuMWsZiABwJ/4ceMvhaWlRNTW12PyhCGs4wAI1OYVm0oKionIz7/dxCnTWccBgFZk3Og/VrPLzM1lHQcAzBUKPIDZi0u8tufQYSISi0WL5k+xwqkTAE3JuJmxf/M+IhKJRItefQMnGQFAS3JzlQ3oG0pEPM9v2rUHq9kBwINBgQcwb1XVNYuXLTdNCZ48bnA7P2/WiQCEyNBgWPbRDwaDgYgenzCpY2An1okAoNUZ1D/U1dWZiHILCo6fO8c6DgCYJRR4APP2/U8/lalURNS+nc/EsQNYxwEQqB3rtuek5xCRl7fPzKeeZh0HAFojK4lk3GN9TePdhw6pNZVs8wCAOUKBBzBjpy9eOnTyFBFZW0kWLZiM9ZABmpSbkfvb2m1ExHHcC6+8hktsAgArnQP9g7u2J6K6uvqt+/axjgMA5gcFHsBcqTWVX61cZRpPnzy8jbcH2zwAwmQwGJb+d0mDvoGIho0aHRzSnXUiAGjVxj7W18bGmoguJ167npLCOg4AmBkUeABz9eXKVerKSiLq2qndmJHRrOMACNSejbszbmYQkZu7+5y5C1jHAYDWzlnqOHRQhGm8edceHa4ECQD3AwUewCzFnjh55tIlIrKztXlhwSSO41gnAhCiguz8Las2m8YLX37N3t6ebR4AACLqGxXi7elGRGUq1f6jR1nHAQBzggIPYH6Ky8qX/PSzaTx75miFmyvbPADCxPP8Dx8u0ev0RDRk+MjQ8AjWiQAAiIhEItGEsQNFIo6IDp06XVhcwjoRAJgNFHgAM8Pzxs+WLquuqSGisB6dhwwMZ50IQKD2bNyddiONiORyt2fmP886DgDAH9q28YiKCCYig8GwYcdOXBYeAO4RCjyAmdm2f39CUhIRSaUOz8+dyDoOgEAVZOdvXrnJNF74yqsOjo5s8wAA3GHksCip1IGIMnNyTl64yDoOAJgHFHgAc5KZm7dm060Tehc8Pd5Zik4C0ASe55d88Mfk+bCIXqwTAQDcycbaevzo/qbxztiYCrWGbR4AMAso8ABmQ6fXf/z9Ep1eT0SD+oVGhgWxTgQgUDvX7UhPwuR5ABC6oC4BQV0CiKiurv7X3XtYxwEAM4ACD2A21mzekpmbS0QKd9enZ41hHQdAoLLTsreu3kJEHMe98MprmDwPAEI2fkx/W1sbIkpMSrpy7TrrOAAgdCjwAObh6o2kbfv2E5FIJFo0f7KdrS3rRABC1KBvWPL+dw36BiIaNvKx0IhI1okAAP6K1MnhsWFRpvGm3XtqamvZ5gEAgUOBBzAD2urqT39YyvM8EY0f079LJ3/WiQAEavPKTTnpOUTk4en59PznWMcBAPh7keHB7fy9iahSqzX9sR4A4M+gwAOYga9XripVKokooJ331AlDWMcBEKiUaym7N+wiIo7jXnrtTTs7e9aJAAD+HsfRpHGDrCRiIjp3+UpSWjrrRAAgXCjwAEJ38MSp4+fOE5G1tfVLz00Ti8WsEwEIUV1t3ZL3vzNNVBk7cXJQtxDWiQAA7pW7XDZkYAQRGY3GDdu319XVs04EAAKFAg8gaIUlpd+tXWsaz5k5ysfLnW0eAMH6+ZufivOLiaitn/8Ts59hHQcA4P7079vT28udiFQV6h0xsazjAIBAocADCFeDgf/4+yWm9WzCe3YeOhjLcQE0Le7UpcO7DhGRRCJ5+Y23rK2tWScCALg/YpFo6sTBIrGIiE5euJCakcU6EQAIEQo8gHCt2/ZbUloaEbnInJ6fO5EjjnUiACHSqNTLP15mGk+fNbt9h45s8wAAPBgvD7ch/cOIyGg0rt++XafTsU4EAIKDAg8gUFdv3Pxl504yXct6/mSpE65lDdAEo9G49KOlmgoNEXUN7jZxynTWiQAAHtyg/mFenm5EVKZUYiI9ANwNBR5AiDRa7SdLlpiW4xozMrp7MI4oAjQt9reYK2cuE5G9g8Mrb7wlEuH3GgCYMbFYPGXCENNE+uPnzqdmZLJOBADCgjc6AIJjNBoXL19RplIRUYC/98wpw1knAhCovMy89d+vM40XvPCywsOTbR4AgIfn4+U2qN+tifQ/b/sNK9IDwO1Q4AEEZ9fBg2fj4ojI1tb6lYXTJRIJ60QAQqTX67977xtdvY6I+g0cPGDwENaJAACax5ABYd6ebkSkrKjYum8/6zgAICAo8ADCkp6Ts3zDL6bxs0+NNZ0IBwB32/D9+uy0bCJyV3g8t+hl1nEAAJqNWCyeOmmIWCImojOXLiXeTGGdCACEAgUeQEBq6+o+/OY706qz/fv0GNg3lHUiAIGKOx13YOt+IhKJRK+++Y6DI1Z5BACL4uXhNmxQhGm8Yft2bVU12zwAIBAo8AAC8u3qtXmFhUTk5SGfO2cs6zgAAqUsVS797w9Go5GIpj3xVJegYNaJAACa34C+oX5tPYmoUqv9ddcu1nEAQBBQ4AGEIub48YMnTxKRlZXkHy9Mt7O1ZZ0IQIh4nv/uvW+16koiCurWfcqMJ1gnAgB4JEQcN23iEGsbKyK6cu36hSvxrBMBAHso8ACCkJ2X/+3qtabxrOmj2vn7sM0DIFjb1mxNir9BRE5S6atvvo3rxgGABZO7ysaMjDaNN+3eo6xQs80DAMzhfQ8Ae3X19e9/8029TkdEvcKDRg6LYp0IQKCuX77+25ptRMRx3IuvviF3c2edCADg0YoMC+rcyY+Iauvq1m7ezPNG1okAgCUUeAD2vl61Oie/gIg8FK4L507iiGOdCECINCr1d+99w/M8EY0ZNyEyqg/rRAAAjxxHNHncYAdHOyJKz86JOXacdSIAYAkFHoCxfUeOHTp5ikynvi+abm+PU98BmsDz/LfvfVNRXkFEHQM7zZ67gHUiAIAW4uRoP2X8YI7jiGjvkSOZubmsEwEAMyjwACylZWV9/9NPpvFTMx5r79+GaRwA4dq6esu1S9eIyMHR8fV3/k8ikbBOBADQcroE+veODCYinufXbNpSU1fPOhEAsIECD8CMtrr6P199Y7rqe59eISOH4tR3gKZdPX/1t7W3Tn1/6dV/enh6sk4EANDSRo+M9vSUE1G5SvXrjp2s4wAAGyjwAGzwvPGTJT8UlZYSkY+34vlnJ7BOBCBQ5cXl3/3nWyNvJKKxEyf36hPNOhEAAAMSsXjm5OFWVhIiupSQcOZSHOtEAMAACjwAGxt37Dx/JZ6IbG2t33hppq2tDetEAELUoG/46l9fmK763iUo+Kmn57JOBADAjIfCdcyIW3/E3Lx7T2FJKds8ANDyUOABGLiUkPjztm2m8cK5k3y8FWzzAAjW2q/XpN1IIyKZi8sb7/yfGKe+A0DrFhUZHNy1PRHp9PpVv/6q0+lZJwKAFoUCD9DSCktKP/rue9OlsEaP6NM7shvrRAACdWzv0YPbY4lIJBK9+ua/XOVy1okAANibPG6Qq0xKRIXFJZt272YdBwBaFAo8QIuqq69/78uvKquqiKhr53azpo9inQhAoDJuZqz8fIVpPOuZuSE9erLNAwAgEHZ2NjOnjRBLxER0Nu7y+cvxrBMBQMtBgQdoOUaj8fNlyzNycojI1VX62qIZYrGYdSgAIdJUaBa/9blepyei6P4Dx0+ayjoRAICA+PooHhvW2zT+ZdeuwuIStnkAoMWgwAO0nF937zl+7jwRWVtJ3njpSanUkXUiACEyNBi+eudLZUk5Efn5t3vx1Tc4jmMdCgBAWKJ7dw/qEkBEOp1uxcZf6nBleIDWAQUeoIWcj49fs2mzaTz36XEdAtqwzQMgWGu+Wp0Uf4OIHJ2c3n7vA1tbW9aJAAAEhyOaOnGIq6szERWXla377Tej0cg6FAA8cijwAC0ht6Dwo++WmBaue2x4n0F9w1gnAhCog9tjGxeue/2tdz29vFknAgAQKFsb6yenjbSSiInoyrXrh06eYp0IAB45FHiAR66yqupfixdX19QQUXDXgNkzsXAdQNOux11b89Vq03j23AU9wsLZ5gEAEDgfL7fxYweaxjtjD6akZzCNAwCPHAo8wKPVYDD856uvC4qKichD4frqopkiERauA2hCcX7xl+98YWgwENHgYcPHTZzMOhEAgBkI79E5KiKIiHieX7Vpk0qtZp0IAB4hFHiAR+ubVWuu3kgiInt727deneXkaM86EYAQVWurPnnto6rKKiLq3DXo+ZdeZZ0IAMBsjH2sn19bTyLSVlUv37BRp29gnQgAHhUUeIBHaMveffuPHiUikUj0jxemt/H2YJ0IQIgMDYYv3/miMKeQiNwVHm/9+30rKyvWoQAAzIZYLH5y2kipkwMR5eYXrMeCdgCWCwUe4FE5fenSio2/mMZznhjdo1sg2zwAgvXjp8uvXbpGRPb29u9+8JHMxYV1IgAAMyN1cnhy+kiJRExEl64mxJ44yToRADwSKPAAj0RyRmbjsvMjhvQaNaw360QAArVtzdZje29NVHn97X/7+bdjnQgAwCz5+XpOGDPANN598FBCUhLbPADwKKDAAzS/4tLSdz9fXK/TEVGPkMBnZo1hnQhAoE4eOLFl5WbTeP7CF0MjItnmAQAwa+GhXaKjuhMRz/NrNm/NKypinQgAmhkKPEAz01RWvvnxp6Y1YP3aer26aDqWnQdoUuKlxGUfLzWdqDlh8rSRY8ayTgQAYPbGjOwT2LEtEdXX1y/7eX2lVss6EQA0JxR4gOZUX697/YP/5qZ9v8wAACAASURBVBUWEpHc1fmd15+ys7VlHQpAiLJSs7546/MGfQMR9R0w8Kln57FOBABgCUQi0RNTRijcXYlIpVYvW78Bi9IDWBIUeIBmw/OGdz9fbDrlzN7e9l+vz3aVObMOBSBEJQXFH73y39rqWiIK6hby8utvcRzHOhQAgIWwtbWe88RjDg62RJSVm7d282aex6L0ABYCBR6g2Sxe9uOJc+eIyMpK8s9XnvRt48k6EYAQaSo0H7/6kUalJqK2fv5v/98HuGgcAEDzkrs6z5o+yrQoffz1G9v272edCACaBwo8QPNYvXnLjpgYIhKJuBcXTAnqHMA6EYAQVWur//vSB42XfH/v488cnZxYhwIAsEDt/LynTBhimt909PSZQ6dOs04EAM0ABR6gGWzdt3/j9h2m8bynJ/Tu1Y1tHgBhqq+t//S1j7PTsonIWSZ7/5PP5XI31qEAACxWj24dGy9ku+NAzKWriWzzAMDDQ4EHeFj7jx5bvn6DaTxz6sixj/VnmwdAmPR6/eK3PktOTCYie3v7//vvp94+bViHAgCwcAP69uwTFUJEPM//vG1rUlo660QA8FBQ4AEeyrGz575aucp0HawxI/s+MW0k60QAQmRoMHz1zpcJFxKIyNra5l/vf9S+Q0fWoQAAWoXHR/UN7tqeiBoaDD9u2JidX8A6EQA8OBR4gAd3Ji7ukyU/8DxPRP2je85/eiLrRABCZDAYvvn3V3GnLhGRWCJ589/vBXULYR0KAKC1EHHcjCnDAvx9iKi+vn7JTz8VlpSyDgUADwgFHuABXbh69cNvvmswGIioV3jQC/Mm4jJYAHfjef679749f+w8EYnF4tff+ldYRC/WoQAAWheJWDz7ice8Pd2IqKqq+ts1a8qUKtahAOBBoMADPIi4hGvvffm1Tq8notDuga8snCYSiVmHAhAcnueXvP/92cNniEgkEr3yxtu9+2KRCAAABmxtrOfOHuvuJiMijaby29Vr1JpK1qEA4L6hwAPct7jEa+9+8YVOpyOikOAOr7/8pEQiYR0KQHAMBsN37317KvYkEXEc99Jr/+w3cBDrUAAArZeDg93c2eNkMiciKlepvlm1urKqinUoALg/KPAA9ycu8dq7i2+19+CuAW/+Y5YV2jvAXRr0Dd+8+9WZQ6eJiOO4F199Y+CQYaxDAQC0djJnx3lzxjo5OhBRcVnZ1ytXocMDmBcUeID7cO5y/O3t/e1Xn7K2smIdCkBw9Dr94rc+N533LhKJXn79zcHDRrAOBQAARERurrJ5cx53cLQjoqKSUhyHBzAvKPAA9+rE+Qv/+eqr/2nv1tasQwEITl1t3cevfnTlzGUiEovF//jnOzj2DgAgKB4K+bzZ4xwcbImosLjk65WrKrVa1qEA4J6gwAPck9gTJz/89jt9QwMR9Qjp+PZrs9HeAe5Wra368MX3r8ddIyIrK6t//us9nPcOACBAXh7yeXNudfiiktKvVqzCmnYAZgEFHuDvbT8Q8/my5abrvUeEdv3nK7Mwcx7gbspS5b8XvJt6PZWIbGxs3vnPh736RLMOBQAATfPycJv/9HjTXPrisrIvV6xUVlSwDgUAfwMFHuBvrNm8dclPPxuNRiLq2zvktRdnYNU6gLvlZ+W/O/+dvMw8IrJ3cHjvo896hkWwDgUAAH/FUyFfMGe8aU27MqXyi+UrikpLWYcCgL+CAg/wpwy88csVKzds3266OWxw5EvPTROLcb13gDslxd/494J/lReXE5GLq/zjxV93De7GOhQAAPw9D4Xrc8+Olzo7EFGFRvPlipU5+QWsQwHAn0KBB2haXb3uvS++2HfkqOnmpHGD5s8Zz3Ec21QAAnQq9uS7C96tqqwiIm+fNp9+9Z1/QHvWoQAA4F65yWULn53sJncmoqqq6q9XrUpOz2AdCgCahgIP0AS1pvLVDz44e/kKEYlE3LNPjZ0+CctoAzRhy6otn73xqa5eR0SBnbt8+tW3Hp6erEMBAMD9cZE5Pj93krenGxHV1dX/8NPPF68msA4FAE1AgQe4U3Ze/gvv/tv0t2drK8k/Fs0YOTSKdSgAwdHr9Us++P7n734yrRDRq3f0B59+IXWWsc4FAAAPwtHBbsGzEwL8fYhI39CwdvOW2OMnWIcCgDthLS6A/3HxasKH335XXVNDRE6O9v985cnOgf6sQwEIjkal/uKtxcmJyaab4ydPfeqZeSIR/igMAGDGbG2sn539+K9bD11PyjAajTtiYstUqhnjxuLlHUA4UOAB/rBt34HlGzaYLhfn5SF/+7XZXp5urEMBCE5mcsbitz43LVknkUgWvvyPUWPG1dfXs84FAAAPSyIWPzlt5N6Y06fPJRDR6YuXylUVz816wtbWlnU0ACDCFHoAE51O/+nSZUvXrTO19+CuAR//53m0d4C7Hdt79N35txacd5JKP/7im8fGjmcdCgAAmg3H0eOj+o4d09904D05Pf2TJUuLSstY5wIAIhyBByCi4rLy/3z1dWpmpunmsMGRz856HJeLA7iDXq9f8+XqwzsPmW629fN/5z8f+rcLYJsKAAAehejIbm4u0o1bDtbX60rKyt7/4svnnpoV3LkT61wArR2OwENrF5dw7fl33jG1d7FYNHf22PlzxqO9A9yhpKD43Xn/amzv0f0GfP7NEk8vb7apAADg0enU0e+FeRNdXZyJqKau7ptVq3cfPGRauBQAWMEReGi9DLxx/bZtG3bsNE2blzk7vbpoRpdO/qxzAQjO2SNnf/xkWU1VDRGJxeKnnpk3duJkjuNY5wIAgEfLQyFftGDyr1sPpmXk8Ty/MyY2Mztn9tTJDvb2rKMBtFIo8NBKlasqPvnhh/jrN0w3Owf6vbpopovMiW0qAKGpr61f+/WaI7sPm266yuWvv/3vrsHd2KYCAIAW42BvO3f22NgjF46dvGw0GhOTkz/6fsncGdMD2rZlHQ2gNUKBh9bobNzlxT/+qKnUEhHHcWNGRj8xdQSmzQPcIeNmxnf/+aYwp9B0MzQ84pU33sKV3gEAWhuRSDRuzIC2vp6/bI3V1etVFeovf1w5btiwYQP6YTYWQAtDgYfWpa5et3z9+t2Hbh1OdHSwXzh3YkRYV7apAITG0GDY8fP2bWu3GhoMRCSRSJ6Y88z4SVPxRg0AoNXq2b2Tp0L+0y97i4uVBoNhe0xMckbG7CmTnaWYwAjQclDgoRVJSkv79Idl+UVFpptdO7d76fmpchdntqkAhCY3I+eHD3/ITM4w3fRp4/uPf77dIRArDwMAtHYKhcui+ZP37D99Ie4GESWlpX343Xczx48PDQ5iHQ2gtUCBh1ahXqdfu2XLtn37TevVicWiaZOGjhvdX8ThQgwAf2jQN+xct+O3n7Y16BuIiOO4EaMff3reczY2NqyjAQCAIFhJJBPHDuzQvs323cdra+urqqpXbNgY2aP7tLGPY2U7gBaAAg+W71pyyhc/rsgrvHUebxtvxYvPTQnw92GbCkBokhOTV3y6PC8zz3TTXaF44ZXXe4SGsU0FAAACFBLUwc/Xc/P2wxmZBUR08WpCcmbmk+PHh3TtwjoagIVDgQdLpq2uXv3rpr1HjpoOvItEojEjo6dPHmYlwZ4P8AetRrvxhw1H9xwxXd2X47hho0bPmbvAHsdSAADgTzhLHefNHn/2QsKBQ+f1+obKSu3SdetDu3Wb9vgYnBUP8OigxoBlMhqNB0+eWrHxlwqNxrTF10fx/LMTO3bAJU8A/mDkjUd2H/5l2UatRmva4u3TZuHLrwaHdGcbDAAAhI/jKDqqe6eOflt3Hs3OKSKiK9eu3UxLGzdiWP9eUSIR1j0FaH4o8GCB0rKyvl/70/WUVNNNiUQ8YcyACWMH4sA7wO2S4m/89PXarNQs000rK6tJ02ZMnDrD2tqabTAAADAjbnLZc89MPHshIfbIxfp6XW1d3aZde85evjL98ccD/HDgBKCZoc+ARVFWqNdu2Rpz/LhpzjwRBXcNmDt7nI+XO9tgAIJSlFu44Yf1F09cbNzSMyxi3sJF3j5tGKYCAAAzZToUH9y1/a59p27czCSi3PyCxct/jOzZY/zw4S4yXPEHoNmgwIOFqK2r27J335Y9e2vr6kxbXF2ks2aM6huFmcAAf1CVqbat2Xp0zxHTBd6JyMvbZ87cBb36RLMNBgAA5s5Z6vjUjFFJqdl79p1WVWiMRuOFK/Hx164P7ddvxIB+uKAJQLNAgQezp9Pr9x89uv63HY2nu1tbScaM6jfx8QE2NpgJDHCLRqXeuX7nwe2xunqdaYuDo+OU6U+MGT9RgrNLAACgmXQN9O8Y0ObE6fjjp+L1er3pfdrpixdHDR7ULzJSIhGzDghg3vCmDcyYvqHhwLHjv+zcVVpebtrCcVzf3iEzJo9wd5OxzQYgHGqlevfGXQe3x9bX1Zu2WFlZjRz9+JSZT0qlmNYIAADNzEoiGTowIiK0S8zh81cTU3neWFlVtXn3nkOnTo0aNKhPWKhYjBoP8IBQ4MEs1dXr9h89umXP3lKlsnFjt67tn5g+or0/TuIFuKWkoHj3xt3H9h7V6/SmLWKxeNDQ4dOemOWu8GCbDQAALJuz1HHaxKF9o7rvP3g2PTOfiFQV6o3bd8QePzFiYP/eoWE4Gg/wAFDgwcxotNrdBw/viIlRV1Y2buwc6Ddt4tDgru0ZBgMQlJRrKfs27b1w7Hzjgo4ikaj/oCFTZ87y9vFhmw0AAFoPH2/3eXPGpWfmxx65kJtXTETlKtXG7Tv3HTk2OLpPv8hIO1ucGw9wH1DgwWzkFhRuP3Ag9sTJep2ucWPnQL8p44eEBHdgGAxAOPR6/dlDZ2K2HUhPSm/cKJFIBg0dPmHKdFR3AABgokNAmw4BbVJScw4dv5SXX0JEao1m+/4D+48e7RcRMbBPb7mLC+uMAOYBBR6ErsHAn42L233wUPyNG0aj0bSR47ju3TpMGDOwa+d2bOMBCERRXtHhXYeO7T2mVf8xOcXOzn7E6DGPT5gkl7sxzAYAAEBEnQL9OgX6paTnHj91JTOrgIjq6uoPnTp95MzZ4M6dB0b16tyhg0jEsY4JIGgo8CBc+UXFMcePx548qVRVNG6USMR9eoWMfayvn68Xw2wAAlFfW3/u6Llje4/cvHqz8S9cROTh5TX68fFDR4yyd3BgGA8AAOAOnTq07dShbW5+yckz8TduZvK8kef5xKSkxKQkd7k8OiI8qmdP1hkBhAsFHgRHW1194vz5QydPXU9Jvb2QyJydhg6KGD440kUmZRgPQAgMDYaEiwmnY09dPHmhvra+cTvHcT3DwkeNGRfeK4rjcBADAAAEqm0bjyenjVRWVJ6/dO3S5Zu1tfVEVKZU7oyJ3X3wEGlLWAcEECgUeBCKmtras1eunDh34dLVqzq9vnE7x3FdO7cbPjgyMqwrrlYNrZyhwXD98vVzR85ePHFBq9He/iG53G3w8BFDRzzm4enJKh4AAMB9kbtIRw+PHj6o19VraecvXc8vKCUinuerqqpNd9Df9p4QAAgFHphTV2rPX7l8+lJcXOI13W2r0xGR3NV5QN+eA/uFenng9F1o1Wqra+LPX407eenK2SvV2qrbP2RtbRPZu/fgoSN6hIWLRCJWCQEAAB6YlZUkIrRLRGiXgqLyuPibVxNSG3/V1dXVsUwGIDwo8MCA0WhMz86+EH/1QvzVm+npjZe5MrGztYkMDxrQt0dwl/aYAwytltFozE7LTjh/9er5+OTEZEOD4faPisXi7j1D+w4YHBXd197enlVIAACAZuTj5ebj1W/MyOiP3vukIId1GgBBQoGHllNYUnr1xo346zcuX7t2+1XcTeztbMJ6du4d0a1H90ArTJWH1qogpyDpyo0bV25cu5RYqb7zx8TKyiqkR2hUdN9efaKlUmcmCQEAAB4psUhka2vNOgWAQKEmwSPE88bs/PzrKSk3UlISkm6WKpV338fVRRrWo3NEaJfgoPbo7dAK6XX6zJTMtOupKYnJNxOSNSr13fdxlsl6hkVERvXpGR5uZ4fj7QAAAACtFPoSNLPisrLUzKzUzMzk9IzkzIyamtq77yMWiwI7tO0ZEtgjJNDfz4sjzJOHVqRB35CbmZudkpWRnJFxMz07LfuO6fEmEomkU5euPULDe4aFt+8YiNNJAAAAAMA8CnxpaemmTZvi4+MrKytdXFzCw8NnzJjh7HxP00fv8bEP8xStWXVNbV5hYUZObnZ+fmZuTnpWtra6usl7ikQifz/PoM4BwV0CunZuZ2tr08JRAZgw8sbSotK8rLz8zLzcjNy8zNy8rLwmGzsRiSWSjoGdgrt1DwoJ6RLUzdbWtoXTAgAAAICQmUGBz8rKevvtt2tqaohIJpOVlpbu37//3LlzixcvVigUzfLYh3mK1kOn15eUlReUFBcWl+QVFeUVFuUVFJSpVH/xEFtb6w4BbToH+gd28O0c6GeHNgIWzcgbVWXKksKSkoKSoryioryiotzCwtxCve6vLoHj4ioP7NQ5sHOXzl2DOgR2srHB37YAAAAAoGlmUOC///77mpqa8PDwl156SSaTlZWVffrpp2lpaStWrHj33Xeb5bEP8xSWp0KjUVaoy5SqMpWyVKksKy8vLisrKSsvr6i4Y7n4u9nYWPv5erZv59OunXcHfx8fH4WIw3WtwNJoVGq1SqMsVarKlMpSZXlxmbJUWVZUVl5S3qBv+NuHy+Vu7dp3COjQsX2Hjh0CA+Vu7i2QGQAAAAAsgNALfGpqanp6ulwuf+utt6ytrYnI3d39vffee+aZZ+Li4pRKpVwuf8jHPsxTmBcDb9RWVVVVV2m0VZXaqsoqbWVVlVqtUWk0Gm2lSq2pUKsrNJoGQ9OTe+8mkYg9FfI2bRRtfRS+vp7+vp4eCjnO1AXzpavXVWurqiqrqyqrqiq1Wk1VlUarVqkr1ZVadaVaqa5QqivVmj+bAN8kF1e5b9u2Pm3a+vn7+/r5+fkHODo5Pbp/AgAAAABYMKEX+MOHDxNRnz59TNXaRCqVhoWFnTt37tixY5MnT37Ixz7MUwhZRWXlmx9/UlNbV11bU1NTW1VTU1PbxHpy90giESvcXBQKVw93F08Pubenm5eXm4e7i0gkbsbMAA+sQd9QV6fT1ddVVVYTUXVVtZE3VldV8wa+trpGV6/T6/TV2mpdvU5Xr6vSVunq6utq62praqu1NTVV1TVVNTXVNfdy/PzPiEQiuZubwsPLw9PT08vLy9vHy9vHu40vLtIOAAAAAM1F6AW+oKCAiLp3737H9pCQkHPnzuXn5z/8Yx/gKZRK5YkTJ+7ePnDgQObnr2o0GtOgrqLiUmzsA38ejuOsrCQSidhKIraylkiMYmNRTUlRQQlRYjNFtUDcrf+R0cg4yaPE87zx706maPRnB6uNPH/nGRlGI2/gb7tl5H+fDGI0Ev/75+ENBqPRaPoMRt54+5YWJhKJbW3tbGzt7ewcbW3tbW3tOU5ERiop0pQUaRKuJLd8JCa4W7s9k29Ca8dxZCQifOVb3O9zzbDbM8Bxlv07VrhMu72RjC3zmlNaXNg4xpKuALcTeoFXqVREdPdq8DKZjIgqKioe/rEP8BS5ubkff/zx3duHDBni6Oj4F5FaQGOBJ72e9H+1dNZfMxLpiHTNEwrAAvG8oaamqqamqoJKWWcBAACwTBzHMX93DSAoQl9gTKlUEpHTXaeMSqVS+r17P+RjH+YpBCgkJIR1BAAAAACAZuDq6so6AoCwCP0IvMnd66KZZq01NPz9Cav3+Nj7egofH58XX3zx7u08z1f/yVXQW8yECROmTp169OhRtjFarVsTzDC9D1oTjuOwz0Orgpd6aIWY7PYuLi6nTp1i/u6aiBwcHFhHALhF6AVeLpcXFBRotVovL6/bt2u1Wvq7v8nd42Mf4CkUCsXs2bPv3q5UKmsfYqG45rJ27Vo7OzvTzAJoSTKZTCKREJFSqcQbuxbm5OQkEon+OIUEWopcLuc4jud5s5uvZAFkMllDQ0NVVRXrIK2LSCQyvTfQ6/V4zWl5crm8pqZGCG+3WhUrKyvT2aZ1dXUt/5ojhG83CjwIh9Cn0Jt+R1ZWVt6x3bTFzc3t4R/7ME8BAAAAAAAA0DKEXuC9vb2J6MaNG3dsT0pKIqI7jpk/2GMf5ikAAAAAAAAAWobQC/zQoUOJ6MyZM7dfcUqn0126dEkkEg0ePPjhH/swTwEAAAAAAADQMoRe4Dt16tS+ffvi4uJly5Y1rir3+eef19TUREZGKhSKxnvu27dvz5496enp9/vYe38KAAAAAAAAAFbMYOngzMzMt99+u7a21tHR0dfXNysrq66uztXVdfHixe7u7o13mzx5sk6nmz179qRJk+73sfd4t78lkKXL7O3tsYgdE1jEjiEsYscKFrFjCIvYMYFF7NjCInZMsF3ETgiwKhYIh9CPwBNRQEDAt99+O3jwYCsrq7S0NCcnp9GjR3/77bf3Uq3v8bEP8xQAAAAAAAAALcAMjsCbEYEcd8UReFZwBJ4hHIFnBUfgGcIReCZwBJ4tHIFnAkfgcQQehMMMjsADAAAAAAAAAAo8AAAAAAAAgBlAgQcAAAAAAAAwAyjwAAAAAAAAAGYABR4AAAAAAADADKDAAwAAAAAAAJgBFHgAAAAAAAAAM4ACDwAAAAAAAGAGUOABAAAAAAAAzAAKPAAAAAAAAIAZQIEHAAAAAAAAMAMo8AAAAAAAAABmAAUeAAAAAAAAwAygwAMAAAAAAACYARR4AAAAAAAAADOAAg8AAAAAAABgBlDgAQAAAAAAAMwACjwAAAAAAACAGUCBBwAAAAAAADADKPAAAAAAAAAAZgAFHgAAAAAAAMAMoMADAAAAAAAAmAEUeAAAAAAAAAAzgAIPAAAAAAAAYAZQ4AEAAAAAAADMAAo8AAAAAAAAgBlAgQcAAAAAAAAwAyjwAAAAAAAAAGZAwjoAND+j0cjzPOsUrVF1dbVpYDQa2SZphYxGI77sTGg0Go7jWKdopbDPM8HzvEajIXz9GcGXnQmDwWDa7fEOE4A5Dq+DAM1lzpw5169fJ6Ljx487OjqyjgPQEgYNGqTVauVyeWxsLOssAC1BpVINHz6ciEJDQ1esWME6DkBLuHLlyvz584lo4sSJ77zzDus4AK0aptADAAAAAAAAmAEUeAAAAAAAAAAzgAIPAAAAAAAAYAZQ4AEAAAAAAADMAAo8AAAAAAAAgBlAgQcAAAAAAAAwA7iMHECzSUpKqqqqIqKwsDCxWMw6DkBLuHz5ssFgkEgkoaGhrLMAtAS9Xh8fH09ETk5OXbp0YR0HoCVUVlYmJycTkbu7e7t27VjHAWjVUOABAAAAAAAAzACm0AMAAAAAAACYARR4AAAAAAAAADOAAg8AAAAAAABgBlDgAQAAAAAAAMwACjwAAAAAAACAGZCwDgBg3tLS0t58882goKAPP/zwjg+VlpZu2rQpPj6+srLSxcUlPDx8xowZzs7OTHICPAye5w8cOHD06NH8/HwbG5uAgICpU6d27dr1jrthnwdLkpOTs3Xr1oyMjPLycg8Pj8DAwGnTpnl4eNxxN+z2YAFiYmKWLl26YsUKT0/Puz96jzs5fhYAWgYuIwfw4LRa7T/+8Y/S0tLu3bvfUeCzsrLefvvtmpoaIpLJZGq1mohcXFwWL16sUCjYxAV4IHq9/v33309MTCQiJyenhoaG2tpaIlq0aNHw4cMb74Z9HizJ4cOHlyxZwvO8tbW1TCZTqVQNDQ0SieSdd94JDw9vvBt2e7AABoPhtddey8zMbLLA3+NOjp8FgBaDKfQAD8hoNH799delpaVNfvT777+vqakJDw9ft27dunXrVq9e3bFjx4qKihUrVrRwToCHtGbNmsTERE9Pz6+//nrDhg2bNm1asGABx3ErVqy4ff/HPg8Wo6ioaNmyZUajccGCBZs3b161atWmTZumT5/e0NDw5ZdfVlVVNd4Tuz2YNZ7n09LSPvnkk8zMzD+7zz3u5PhZAGgxKPAAD2jr1q1xcXH+/v53fyg1NTU9PV0ul7/11lsymYyI3N3d33vvPWtr67i4OKVS2dJZAR5UWVlZTEyMlZXVRx991L59e47jOI4bPXp03759dTrd2bNnTXfDPg+W5MqVK3q9PiIiYvTo0WKxmIisra1nzpwZEBBQXV2dlJRkuht2ezBr69atmzx58muvvXbx4sU/u8897uT4WQBoSSjwAA8iMTHxl19+CQ4OnjJlyt0fPXz4MBH16dPH2tq6caNUKg0LC+N5/tixYy0XFODhnDlzxmAwREVFubu73759wYIF3377bXR0tOkm9nmwJKZj7LfvzCamLVqt1nQTuz2YtTZt2gwePHj48OHDhw+3srJq8j73uJPjZwGgJaHAA9w3lUq1ePFiqVT6xhtvmA7O3KGgoICIunfvfsf2kJAQIsrPz2+BkADNIiUlhYgiIyPv2C6VStu1a9fY6rHPgyWJjo62tbU9f/5845FJo9F46NChlJQUqVTaeA48dnswa4MHD170Ozs7uybvc487OX4WAFoSVqEHuD8Gg+Hzzz/XarUffvihi4tLk/dRqVREdPfKq6Z5Zf/f3p3GRHm1DRy/7kFxAQoC7mi1IBqL4IpaJRXRWq0iShTrWutWqbWuVdP2SRNttWraGkRxiVutGltxB21NXEBQsVpNtUrqFjfAhUVZZhyY98NJ78wz6COMFDK8/9+nmeucuedwPJPxmrPc2dnZ/3YjgYqilj56e3sfP348OTk5PT1d07QWLVr079+/a9euejXGPKoTHx+fZcuWff3114sWLWrYsKGnp2dWVtajR498fX0XLFigj3OGPaq9Mg5yPgtAZSKBB8pn8+bNly9fHjt2bLt27V5UR+U8bm5uNvHXXntN/vmeAxyCGq67d+8+c+aMpmleXl55eXnnzp07d+5cnz59pk+frqox5lHNlJSUHHjpuQAAEM9JREFUuLm5ZWRkZGZmZmZmqqC7u7v1vXsY9qj2yjjI+SwAlYkEHiiH1NTUPXv2dO7cOTIy8qWVNU2ziaj/+ZnN5n+lccC/QM2cnDlzZtiwYZGRkXXr1rVYLMnJyTExMUeOHAkMDOzVq5demTGP6uHKlSv/+c9/zGZzVFRUaGiol5dXVlZWQkJCQkLC3LlzFy9e3LRpU70ywx7VXhkHOZ8FoHKwBx4oq8zMzBUrVtSvX3/mzJmlv6WseXl5idVBRzoV8fT0/PcaCVSsunXrikhYWNiYMWPUY03TQkJCRo8eLSKHDx9W1RjzqE7i4uKKiorGjh07atSoJk2a1KpVq1mzZlOmTBk4cGBOTs6mTZtUNYY9qr0yDnI+C0BlYgYeKKv09PSCggInJ6d58+bpwYKCAhG5cuVKdHS0iMyfP7958+aenp53797Ny8uzuYKKeHt7V2KrgVfi5eWVm5trvd1d6dChg4hcu3bNYrFomsaYR7WRn5+v7okdGhpqUxQWFrZ///6LFy+qpwx7VHtlHOR8FoDKxAw8UD5Pnjy5Y0Xt7DIajeqpyWQSkSZNmojIpUuXbF6r7h7cuHHjSm81YCf1H69nz57ZxEtKSkSkTp06ajUKYx7VhsFg0DRN07TSN9ZSEYPBoBYGM+xR7ZVxkPNZACoTCTxQViEhIftKmT9/vogEBQWpp35+fiLSp08fETl58qRKchSTyZSWlmYwGHr37l1VfwJQXiEhISKSlJRkE09NTRURNeCFMY9qpE6dOk2aNLFYLGlpaTZFKuLn56d+t2LYo9or4yDnswBUJhJ4oOK1bt3a19c3IyNj9erV+gkuS5cuLSgoCA4ObtCgQVU3ECirnj17ent7nzp1at26dYWFhSJSUlKSmJi4c+dOg8EwbNgwVY0xj+okPDxcRNasWXPy5Ek1nouLiw8dOvTTTz+JyKBBg1Q1hj2qvTIOcj4LQGXSrG+IAqC8UlJSlixZEhQUtHDhQuv49evXFyxYUFhY6Orq2qxZsxs3bhQVFXl6ei5btqx+/fpV1VrADufPn1+4cKHZbDYYDN7e3jk5OSaTycnJafz48SrPURjzqE5Wrlz566+/ioizs7Onp+fDhw/NZrOmacOHDx81apRejWGP6mH06NF5eXlr165t1KiRTVEZBzmfBaDSOH311VdV3QbAgd2+fTs5OblRo0Y2K8Tq1asXEhKSn5+flZV1//59d3f30NDQuXPnchYrHE7jxo3feuutgoKC3Nzc3Nzchg0btm/ffvbs2TYn2zHmUZ0EBwcHBAQUFBSYzebHjx83atQoMDBw5syZb7/9tnU1hj2qh/j4eKPROGjQIFdXV5uiMg5yPgtApWEGHgAAAAAAB8AeeAAAAAAAHAAJPAAAAAAADoAEHgAAAAAAB0ACDwAAAACAAyCBBwAAAADAAZDAAwAAAADgAEjgAQAAAABwACTwAAAAAAA4ABJ4AAAAAAAcAAk8AAAAAAAOgAQeAAAAAAAHQAIPAAAAAIADIIEHAAAAAMABkMADAAAAAOAASOABAP9f7N27V9M0TdNu3bpV1W0pn+LiYtXy2NjYqm4LAACoMiTwAAAAAAA4ABJ4AAAc0vHjx9W0fJcuXaq6LQAAoDKQwAMAAAAA4ABI4AEAAAAAcAA1qroBAADAHj179szJyRERJyenqm4LAACoDMzAAwAqRkREhKZpLi4uJpPJOn7x4kW1VbtGjRp5eXnWRbdu3VJFkydPto4/ffr022+/7d69u5eXl5ubW1BQ0NSpU9PT01/01uWtb+PJkyddu3bVNK1mzZp79uyxLsrLy1u4cGGvXr18fHxcXFwCAgKGDRuWmppaxiu/VHJyclRUVGBgYL169dzc3Nq2bRseHr5v376SkpKXvtbJycnHx8fDw2PFihUqMnv2bNWfv//++3NfMmTIEPVnPnjwQA++Yu8BAIDKYwEAoCKsWrVKfbMkJydbx63vfJaYmGhdtHnzZhXftWuXHjx79myTJk1Kf2EZDIZvv/229PuWvb6enN+8eVMP5ufnh4SEqPrbt2+3rp+SkuLp6fncb8+5c+e+Sl9ZLBaTyTR06NAXfTsPGDDAbDbrlc1ms4qvXLnS+iKurq4ismjRIvX09OnTqtrnn39e+h3z8vJq164tIuHh4XqwvL0NAACqEDPwAICK0a9fP/Xg6NGj1vHk5GT98YkTJ6yL1NMaNWqEhYWpyPXr13v37n3v3j0RGTZsWFxc3I4dO2bMmOHi4lJSUjJv3rzly5dbX6G89W0YjcaIiIikpCRN0zZs2DBixAi9KCsra+jQoY8fP65Tp87HH3+8bdu2PXv2fPPNN40bNxaRZcuWHTp0qNx9ZOW7776Lj48XET8/v6VLl8bHx8fHxy9evNjPz09EEhISYmJiynvN4ODgFi1aiMju3btLl+7fv7+oqEhExo0bpyKv2HsAAKCyVfUvCACA6kMln2FhYdZBHx8fEVFT2T169ChdPyQkRI/07dtXRJycnKzn5C0Wy9WrV319fUXExcXlzp079tW3mYE3mUwDBw5Ukbi4OJu/ZdOmTaro4MGD1vH09PSaNWuKyMSJE8vbP9a6desmIv7+/rm5udbxnJyc5s2bi8jgwYP1YBln4C0Wy2effaZqXr161eYdIyIi1D+E0WhUkfL2NgAAqFrMwAMAKsy7774rIikpKUajUUVu3bp1584dEfn0009FJC0trbCwUBXdv3//77//FpH+/furyKVLl3777TcR+eSTT2yWl/v7+69bt05E8vPz165da199a8XFxSNHjjxw4ICI/PDDD1OmTLGpcO7cORFxdnbWm6e0atVq6tSpERERXl5e5eoca8+ePbtx44aHh8ekSZNee+016yJ3d/fevXuLyLVr1+y48vDhw9UDm838T548UUsGRowY4ezsLK/WewAAoEqQwAMAKoxaRV9YWHjmzBkVUevn/fz81Op0k8l06tQpVXT8+HH1QKX9IrJ//371YNasWaUvHhoaGhAQICIpKSn21dcVFxePHz/+l19+EZHRo0erHxdsqPzcZDL9/PPPNkUrVqzYvXv3kiVLntsJZVGzZs2MjIzs7Ow5c+aULs3NzbX7yp06dVKT5zar6Pft26fWz48dO1ZF7O49AABQVUjgAQAVJjQ0VM3u6tvgk5KSRCQkJMTf31/tHte3wasHDRs2bN++vYqkpaWJSNOmTZs1a/bc6wcGBorIn3/+aV993fTp03/88Uf1+ODBg9ZHsusiIiLUUvmoqKiOHTvGxsbaNyVeRiaT6erVq5s3bx45cqTN5Hl5qUn406dP379/Xw+qnyH8/f27du2qInb3HgAAqCok8ACACuPi4tKjRw8ROXbsmIqoGfiePXuKyNtvvy1WCbyage/Xr5+maSqSmZkpInfv3tVeYNu2bSLy5MkT++rrDh48aDAYIiMjRSQ7O3v+/Pml/5bAwMBNmzbVq1dPRM6fPz9t2jQ/P7/GjRuPGTNmz549NrfKs09mZub3338/YMAAX1/funXrtmnT5oMPPlAn4b/KZaOiokTEYrHs3btXRfLy8tT6ef34OnmF3gMAAFWFBB4AUJHUevjU1FSj0fj48ePLly+LiLpPm0rgU1NTTSbTgwcP/vrrL7HaAC8i+fn5ZXmL/Px8dZv08tbXaZq2fv36nTt3durUSUQ2btyoL+y3NnLkyPT09OXLl4eEhDg5OYlIRkbG1q1bhwwZEhQUpGaw7bZ161Y/P79Zs2YlJiZev37dw8OjZ8+eEyZMWL9+vcrA7RYUFNS6dWuxWkW/b98+o9Goadro0aP1anb3HgAAqCok8ACAiqS2wRcVFZ06derkyZMWi6VBgwatWrUSkV69eolIYWHh2bNnk5KSLBaLwWBQB6Er7u7uItKxY8eXH8FqMNhRXxcTEzN+/HiDwRAbG6tpmsViiY6Ofm6a6u3tPXv27BMnTjx69CghIWHu3LmNGjUSkStXrkREROTk5NjXS2lpaePGjXv69Gnz5s1XrVp148aNhw8fJiUlrV+/fsKECWra/1WoVfRHjx5VLVTr50NDQ9X59ordvQcAAKoKX8kAgIoUFBSkUtyjR4+q9fNq+l1E2rRp07BhQxE5ceKEWj/fpUsX67Pc/f39ReTy5cvFxcVlea/y1tfpd4/r2rXrhx9+KCLnz5+Pi4uzrpObm5ubm6ufme/u7t6/f/+lS5fevn37888/F5F79+7p5/CV15o1a0pKSpydnU+cODF16lR183ad3b8L6FQC/+zZs4SEhLy8vMOHD8t/r5+XV+g9AABQVUjgAQAV7J133hGRY8eO2STwYrUNXu2E18+fV8LCwkSkqKhInQ9vIz8/v3379i1btly8eLF99Z9ryZIlasb7iy++sD7NrnXr1h4eHure6dZq1KihH+T+8OHD/3Hl/+H69esi0qpVq9dff92myGw26ycI2C0gIKBt27Yisnv37r179xqNRhcXF5t7xVVI7wEAgMpEAg8AqGAqLT916tTZs2flnxPsFD2Bv3jxovz3BngRGTJkiDqpfvr06VevXrUuslgss2bNunDhws2bN8PDw+2r/1ze3t5ff/21lDrNrmPHjiJy7Nix0sew6xlvcHDwyzrj+Vq2bCkit27dsjkirqSkZMKECRkZGVLmPeovoibhExMTt2zZIiKRkZGurq7WFSqk9wAAQGWqUdUNAABUN3379jUYDEajUURcXV31u8TJP9vgVWrq6enZpUsX6xc6OzuvWbNm8ODBWVlZgYGBkydPDg4Orl+//s2bNzdv3qzOmZsxY8abb75pX/0XmTJlyvr168+dO7dx48ZJkyZ169ZNRMaOHZuYmGgymXr37h0dHd2hQ4c6dercu3dv//798fHxItK9e/eXXvlF+vTps2HDhqdPn/br12/evHm+vr7Z2dkXLlyIiYlJT093dXV9+vTpjRs3vvzyy/DwcJteKqOoqKivvvoqPz//yJEjYnX7d11F9R4AAKg8Lz26BgCA8urcubP6lunbt69NUf369VXRiBEjnvvaLVu21K5du/QXlqZpH330UUlJid319fur37x50+Yiqamp6m52HTp0KC4uVsHo6OgXfXu2a9cuKyvrVbpIzZCXbnN0dPTZs2ednZ1VJCIiwmKxmM1m9XTlypXWF1GT6osWLXruW7Rr1069ysfHR/+jbJS3twEAQBViCT0AoOLpm9utN8ArahW9lNoArxszZkx6evqcOXMCAgLc3Nzc3Nw6dOgwbty4CxcurF69Wr9pvN31n6tbt26lT7OLjY1NSUkZPnx4YGBgvXr1XF1d27ZtO2jQoF27dv3xxx/6LxH22bFjx/bt23v16tW0adPatWu/8cYbEydOTEtLi42N7dSp05EjRyIjI8PDw9977z2730K/Hd2YMWNedJJ8hfQeAACoHJrFYqnqNgAAAAAAgJdgBh4AAAAAAAdAAg8AAAAAgAPgFHoAAOx34MABm3uwvVStWrWmTZv2L7UHAABUY+yBBwDAfu+///6OHTvK9RIPD4/s7Ox/qT0AAKAaI4EHAAAAAMABsAceAAAAAAAHQAIPAAAAAIADIIEHAAAAAMABkMADAAAAAOAASOABAAAAAHAAJPAAAAAAADgAEngAAAAAABwACTwAAAAAAA6ABB4AAAAAAAdAAg8AAAAAgAMggQcAAAAAwAGQwAMAAAAA4ABI4AEAAAAAcAAk8AAAAAAAOAASeAAAAAAAHAAJPAAAAAAADoAEHgAAAAAAB/B/jgC9ug6ej88AAAAASUVORK5CYII=" width="672" /></p>
+<p><img src="data:image/png;base64,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" width="672" /></p>
 <pre class="r"><code># Boxplots
-box_plot = df %&gt;% ggplot(aes(y=weeks_alive,
-                      x = dose
-                      )) + geom_boxplot()
+box_plot &lt;- ggplot(data=df, aes(y=weeks_alive,  x = dose)) + geom_boxplot()
 box_plot</code></pre>
-<p><img src="data:image/png;base64,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" width="672" /></p>
-<pre class="r"><code># My favorite - ridgeline plots
+<p><img src="data:image/png;base64,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" width="672" /></p>
+<pre class="r"><code># Jeremy's favorite - ridgeline plots
 
 # install.packages('ggridges')
-
 library(ggridges) </code></pre>
 <pre><code>## 
 ## Attaching package: 'ggridges'</code></pre>
 <pre><code>## The following object is masked from 'package:ggplot2':
 ## 
 ##     scale_discrete_manual</code></pre>
-<pre class="r"><code>ridge_plot = df %&gt;% ggplot(aes(x=weeks_alive, y = dose)) + 
-  geom_density_ridges(jittered_points = T, fill = 'orange') + theme_minimal()
+<pre class="r"><code>ridge_plot &lt;- ggplot(data=df, aes(x=weeks_alive, y = dose)) + geom_density_ridges(jittered_points = T, fill = 'orange')
 ridge_plot</code></pre>
 <pre><code>## Picking joint bandwidth of 10.5</code></pre>
-<p><img src="data:image/png;base64,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" width="672" /></p>
-<p>It’s a bit tough to tell, but the overall assumptions of normality and equal variance seem reasonable.</p>
+<p><img src="data:image/png;base64,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" width="672" /></p>
+<pre class="r"><code># add a fancy minimalist theme to make it prettier:
+ridge_plot + theme_minimal()</code></pre>
+<pre><code>## Picking joint bandwidth of 10.5</code></pre>
+<p><img src="data:image/png;base64,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" width="672" /></p>
+<p>A two sample t-test assumes independence and normality. An ANOVA assumes independence, normality, and equal variance. It’s a bit tough to tell, but the overall assumption of equal variance seems reasonable. Normality is a bit of a hard sell within groups or overall. Nevertheless, most analysts would march ahead with the analysis despite these violations of assumptions. We can discuss how you might think and talk about this in class.</p>
 <p>The global mean is</p>
 <pre class="r"><code>mean(df$weeks_alive)</code></pre>
 <pre><code>## [1] 75.55263</code></pre>
-<p>PC3. Anova</p>
+</div>
+<div id="pc3" class="section level3">
+<h3>PC3</h3>
+<p>ANOVA! Remember that we have to call <code>summary()</code> to see the ANOVA table:</p>
 <pre class="r"><code>summary(aov(weeks_alive ~ dose, data = df))</code></pre>
 <pre><code>##             Df Sum Sq Mean Sq F value Pr(&gt;F)  
 ## dose         3   4052  1350.7    3.55 0.0245 *
 ## Residuals   34  12937   380.5                 
 ## ---
 ## Signif. codes:  0 '***' 0.001 '**' 0.01 '*' 0.05 '.' 0.1 ' ' 1</code></pre>
-<p>This provides evidence that the group means are different.</p>
-<p>PC4. T-test between None and Any, and between None and High.</p>
-<pre class="r"><code>t.test(df[df$dose == 'None', 'weeks_alive'], # Samples with no dose
-       df[df$dose != 'None','weeks_alive'] # Samples with any dose
+<p>This provides evidence in support of the alternative hypothesis that the group means are not equal. It seems that the dosage of Red Dye #40 has a relationship with how many weeks the mice lived.</p>
+</div>
+<div id="pc4" class="section level3">
+<h3>PC4</h3>
+<p>T-test between None and Any, and between None and High.</p>
+<pre class="r"><code>t.test(df$weeks_alive[df$dose == 'None'], # Samples with no dose
+       df$weeks_alive[df$dose != 'None'] # Samples with any dose
        )</code></pre>
 <pre><code>## 
 ##  Welch Two Sample t-test
 ## 
-## data:  df[df$dose == &quot;None&quot;, &quot;weeks_alive&quot;] and df[df$dose != &quot;None&quot;, &quot;weeks_alive&quot;]
+## data:  df$weeks_alive[df$dose == &quot;None&quot;] and df$weeks_alive[df$dose != &quot;None&quot;]
 ## t = 4.2065, df = 33.732, p-value = 0.0001806
 ## alternative hypothesis: true difference in means is not equal to 0
 ## 95 percent confidence interval:
@@ -460,14 +376,13 @@ t.test(weeks_alive ~ dose == 'None', data = df)</code></pre>
 ## mean in group FALSE  mean in group TRUE 
 ##            69.11111            91.36364</code></pre>
 <pre class="r"><code># T-test between None and High
-
-t.test(df[df$dose == 'None', 'weeks_alive'], # Samples with no dose
-       df[df$dose == 'High','weeks_alive'] # Samples with high dose
+t.test(df$weeks_alive[df$dose == 'None'], # Samples with no dose
+       df$weeks_alive[df$dose == 'High'] # Samples with high dose
        )</code></pre>
 <pre><code>## 
 ##  Welch Two Sample t-test
 ## 
-## data:  df[df$dose == &quot;None&quot;, &quot;weeks_alive&quot;] and df[df$dose == &quot;High&quot;, &quot;weeks_alive&quot;]
+## data:  df$weeks_alive[df$dose == &quot;None&quot;] and df$weeks_alive[df$dose == &quot;High&quot;]
 ## t = 2.4958, df = 8.5799, p-value = 0.0353
 ## alternative hypothesis: true difference in means is not equal to 0
 ## 95 percent confidence interval:
@@ -475,82 +390,140 @@ t.test(df[df$dose == 'None', 'weeks_alive'], # Samples with no dose
 ## sample estimates:
 ## mean of x mean of y 
 ##  91.36364  65.25000</code></pre>
-<pre class="r"><code># Formula notation is a bit tricker. I would probably create a temprorary dataframe
-
-tmp = df %&gt;% filter(dose %in% c('None', 'High'))
-
-t.test(weeks_alive ~ dose, data = tmp)</code></pre>
+<pre class="r"><code># Formula notation is a bit tricker. I would create a temprorary dataframe
+df.tmp &lt;- subset(df, subset= dose==&quot;None&quot; | dose==&quot;High&quot;)
+t.test(weeks_alive ~ dose, data = df.tmp)</code></pre>
 <pre><code>## 
 ##  Welch Two Sample t-test
 ## 
 ## data:  weeks_alive by dose
-## t = -2.4958, df = 8.5799, p-value = 0.0353
+## t = 2.4958, df = 8.5799, p-value = 0.0353
 ## alternative hypothesis: true difference in means is not equal to 0
 ## 95 percent confidence interval:
-##  -49.960874  -2.266399
+##   2.266399 49.960874
 ## sample estimates:
-## mean in group High mean in group None 
-##           65.25000           91.36364</code></pre>
+## mean in group None mean in group High 
+##           91.36364           65.25000</code></pre>
 <p>These t-tests both support the idea that receiving a dose of RD40 reduces lifespan. However, we should not completely trust these p-values, since we are doing multiple comparisons. One option is to do a Bonferroni correction, where we only cnsider things significant if <span class="math inline">\(\alpha &lt; .05/m\)</span> where <span class="math inline">\(m\)</span> is the number of tests. In this case, we would fail to reject the null for the second test because we would set <span class="math inline">\(\alpha = .025\)</span>.</p>
-<p>The Bonferroni correction is more conservative than it needs to be, ane there are other approaches; for example, the <code>TukeyHSD</code> function takes in an anova result and does post-hoc comparisons with corrections for all of the groups.</p>
+<p>The Bonferroni correction is more conservative than it needs to be, ane there are other approaches; for example, the <code>TukeyHSD</code> function takes in an anova result and does post-hoc comparisons with corrections for all of the groups. The Reinhart book also provides a description of the Benjamini-Hochberg correction.</p>
+</div>
 </div>
 <div id="statistical-questions" class="section level2">
 <h2>Statistical Questions</h2>
-<p>Q1. a) It is a sample statistic, because it comes from a sample. b) Confidence intervals for proportions are equal to</p>
-<p><span class="math display">\[p \pm z * \sqrt{ \frac{p*(1-p)}{n}}\]</span></p>
-<p>For a 95% confidence interval, <span class="math inline">\(z = 1.96\)</span>, so we can calculate it like this:</p>
+<div id="sq1-6.12" class="section level3">
+<h3>SQ1 — 6.12</h3>
+<ol style="list-style-type: lower-alpha">
+<li><p>It is a sample statistic (the sample mean), because it comes from a sample. The population parameter is unknown in this case, but can be estimated from the sample statistic.</p></li>
+<li><p>As given in the textbook, confidence intervals for proportions are equal to:</p></li>
+</ol>
+<p><span class="math display">\[\hat{p} \pm z^* \sqrt{ \frac{\hat{p}\times(1-\hat{p})}{n}}\]</span></p>
+<p>Where we calculate <span class="math inline">\(z^*\)</span> from the Z distribution table. For a 95% confidence interval, <span class="math inline">\(z = 1.96\)</span>, so we can calculate it like this:</p>
 <pre class="r"><code>lower = .48 - 1.96 * sqrt(.48 * .52 / 1259)
 
 upper = .48 + 1.96 * sqrt(.48 * .52 / 1259)
 
 ci = c(lower, upper)
-
 print(ci)</code></pre>
 <pre><code>## [1] 0.4524028 0.5075972</code></pre>
 <p>This means that we are 95% confident that the true proportion of Americans who support legalizing marijuana is between ~45% and ~51%.</p>
 <ol start="3" style="list-style-type: lower-alpha">
-<li>We have a large enough sample, which is collected randomly, to assume that the distribution is normal.</li>
-<li>The statement isn’t justified, since our confidence interval include 50%</li>
+<li><p>We have a large enough sample (collected randomly) in which enough responses are drawn from each potential outcome to assume that the observations are independent and the distribution of the sample proportion is approximately normal.</p></li>
+<li><p>The statement is <strong>not</strong> justified, since our confidence interval includes 50%.</p></li>
 </ol>
-<p>6.20 We can use the point estimate of the poll to estimate how large a sample we would need to have a confidence interval of a given width.</p>
-<p>Basically, we want each half of the confidence interval to be 1%, i.e., <span class="math inline">\(1.96 * \sqrt{\frac{.48 * .52}{n}} = .01\)</span></p>
+</div>
+<div id="sq2-6.20" class="section level3">
+<h3>SQ2 — 6.20</h3>
+<p>We can use the point estimate of the poll to estimate how large a sample we would need to have a confidence interval of a given width.</p>
+<p>In this case, we want the margin or error to be 2%. Using the same formula from above, this translates to: <span class="math inline">\(1.96 * \sqrt{\frac{.48 * .52}{n}} = .02\)</span></p>
 <p>We can solve for <span class="math inline">\(n\)</span>:</p>
-<p><span class="math display">\[\sqrt{\frac{.48 * .52}{n}} = .01/1.96\]</span> <span class="math display">\[\frac{.48 * .52}{n} = (.01/1.96)^2\]</span> <span class="math display">\[n = (.48 * .52)/(.01/1.96)^2\]</span></p>
-<pre class="r"><code>(.48 * .52)/(.01/1.96)^2</code></pre>
-<pre><code>## [1] 9588.634</code></pre>
-<p>So, we need a sample of approximately 9,589</p>
-<p>6.38</p>
-<p>The question is whether there has been a change in the proportion over time. While the tools we have learned could allow you to answer that question, they assume that responses are independent. In this case, they are obviously not independent as they come from the same students.</p>
-<p>6.50</p>
+<p><span class="math display">\[\sqrt{\frac{.48 * .52}{n}} = .02/1.96\]</span> <span class="math display">\[\frac{.48 * .52}{n} = (.02/1.96)^2\]</span> <span class="math display">\[n = (.48 * .52)/(.02/1.96)^2\]</span> Let R solve this:</p>
+<pre class="r"><code>(.48 * .52)/(.02/1.96)^2</code></pre>
+<pre><code>## [1] 2397.158</code></pre>
+<p>So, we need a sample of at least 2,398 (since we can’t survey less than a whole person).</p>
+</div>
+<div id="sq3-6.38" class="section level3">
+<h3>SQ3 — 6.38</h3>
+<p>The question is whether there has been a change in the proportion across two groups (the students pre- and post-class) over time. While the tools we have learned could allow you to answer that question for two groups, they assume that responses are independent. In this case, the responses are not independent because they come from the same students. You could go ahead and calculate a statistical test for difference in pooled proportions (it’s just math!), but the dependence between observations violates the assumptions and means that the baseline expectations necessary to identify the hypothesis test are not met. Your test statistic will not mean what you want it to mean.</p>
+</div>
+<div id="sq4-6.50" class="section level3">
+<h3>SQ4 — 6.50</h3>
 <ol style="list-style-type: lower-alpha">
-<li>We can test this with a <span class="math inline">\(\chi^2\)</span> test.</li>
+<li>Our hypothesis focuses on the relationship of the sample and census distributions by region:<br />
+<span class="math inline">\(H_0\)</span> : The sample distribution of regions follows the census distribution.<br />
+<span class="math inline">\(H_A\)</span> : The sample distribution of regions does not follow the census distribution.</li>
 </ol>
-<pre class="r"><code>chisq.test(x = c(83,121,193,103), p = c(.18,.22,.37,.23))</code></pre>
+<p>Before we can calculate the test statistic we should check the conditions:<br />
+1. Independence: <span class="math inline">\(500 &lt; 10%\)</span> of US population, and the sample is random, hence we can assume that one individual in the sample is independent of another.<br />
+2. Sample size: The expected counts can be calculated as follows: <span class="math display">\[E N E = 500 × 0.18 = 90\]</span> <span class="math display">\[E N C = 500 × 0.22 = 110\]</span> <span class="math display">\[E S = 500 × 0.37 = 185\]</span> <span class="math display">\[E W = 500 × 0.23 = 115\]</span></p>
+<p>All expected counts are greater than 5 (which meets the rule of thumb re: empty “cells” in the table).</p>
+<p>We can test this with a <span class="math inline">\(\chi^2\)</span> test.</p>
+<pre class="r"><code># Manually using the expected counts and formula for chi-squared
+chisq = (83-90)^2/90 + (121-110)^2/110 + (193-185)^2/185 + (103-115)^2/115
+chisq</code></pre>
+<pre><code>## [1] 3.242564</code></pre>
+<pre class="r"><code># We could then look up the chi-square distribution for 3 degrees of freedom in the book or find it in R. 
+# lower.tail=FALSE ensures that R returns the proportion of the distribution at least as large as our critical value:
+pchisq(chisq, df=3, lower.tail=FALSE)</code></pre>
+<pre><code>## [1] 0.355717</code></pre>
+<pre class="r"><code># We could also use R's chisq.test function:
+chisq.test(x = c(83,121,193,103), p = c(.18,.22,.37,.23))</code></pre>
 <pre><code>## 
 ##  Chi-squared test for given probabilities
 ## 
 ## data:  c(83, 121, 193, 103)
 ## X-squared = 3.2426, df = 3, p-value = 0.3557</code></pre>
-<pre class="r"><code># Or manually
-
-# Calculate expected values
-500 * c(.18,.22,.37,.23)</code></pre>
-<pre><code>## [1]  90 110 185 115</code></pre>
-<pre class="r"><code># Use the formula for chi-squared
-chisq = (83-90)^2/90 + (121-110)^2/110 + (193-185)^2/185 + (103-115)^2/115
-
-chisq</code></pre>
-<pre><code>## [1] 3.242564</code></pre>
-<pre class="r"><code># We could then look up the chi-square distribution for 3 degrees of freedom</code></pre>
-<p>The p-value for this is large, meaning that we don’t have evidence that the sample differs from the census distribution.</p>
+<p>The p-value for this is large, meaning that we cannot reject the null hypothesis that the sample proportions do not follow the census distribution.</p>
 <ol start="2" style="list-style-type: lower-alpha">
-<li></li>
-<li>Opinion is the response and location is the explanatory variable, since it’s unlikely that people move to a region based on their opinion.</li>
+<li>Part b answers below:<br />
+</li>
 </ol>
-<ol start="2" style="list-style-type: lower-roman">
-<li>One hypothesis is that opinions differ by region. The null hypothesis is that opinion is independent of region, while the alternative hypothesis is that there is a relationship.</li>
+<ol style="list-style-type: lower-roman">
+<li>We would probably want to treat opinion as the response and region as the explanatory variable, since it’s unlikely that people move to a region based on their opinion (although that could be an empirical question!).<br />
+</li>
+<li>One hypothesis is that opinions differ by region. The null hypothesis is that opinion is independent of region, while the alternative hypothesis is that there is a relationship. More formally:<br />
+<span class="math inline">\(H_0\)</span> : Region and opinion on direction are independent.<br />
+<span class="math inline">\(H_A\)</span> : Region and opinion on direction are dependent.<br />
+</li>
 <li>We can again use a <span class="math inline">\(\chi^2\)</span> test.</li>
 </ol>
+<p>You can do this by hand calculating the expected counts and then plugging the values into the formula for <span class="math inline">\(\chi^2\)</span> as we did above. If you do that, here are the expected counts:</p>
+<p>Right direction:</p>
+<pre class="r"><code># NE:
+round(171*83/500)</code></pre>
+<pre><code>## [1] 28</code></pre>
+<pre class="r"><code># NC:  
+round(171*121/500)</code></pre>
+<pre><code>## [1] 41</code></pre>
+<pre class="r"><code># S:  
+round(171*193/500)</code></pre>
+<pre><code>## [1] 66</code></pre>
+<pre class="r"><code># W:
+round(171*103/500)</code></pre>
+<pre><code>## [1] 35</code></pre>
+<p>Wrong direction:</p>
+<pre class="r"><code>round(329*83/500)</code></pre>
+<pre><code>## [1] 55</code></pre>
+<pre class="r"><code>round(329*121/500)</code></pre>
+<pre><code>## [1] 80</code></pre>
+<pre class="r"><code>round(329*193/500)</code></pre>
+<pre><code>## [1] 127</code></pre>
+<pre class="r"><code>round(329*103/500)</code></pre>
+<pre><code>## [1] 68</code></pre>
+<p>Working through the formula for the <span class="math inline">\(\chi^2\)</span> test statistic and then calculating the p-value for 3 degrees of freedom:</p>
+<pre class="r"><code>chisq &lt;- (29-28)^2 /28 +
+  (44-41)^2 /41 +
+  (62-66)^2 /66 +
+  (36-35)^2 / 35 +
+  (54-55)^2 / 55 +
+  (77-80)^2 / 80 +
+  (131-127)^2 / 127 +
+  (67-68)^2 / 68
+
+chisq</code></pre>
+<pre><code>## [1] 0.7975941</code></pre>
+<pre class="r"><code>pchisq(chisq, df=3, lower.tail=FALSE)</code></pre>
+<pre><code>## [1] 0.8500424</code></pre>
+<p>And using R’s <code>chisq.test()</code> function here:</p>
 <pre class="r"><code>x &lt;- matrix(c(29,54,44,77,62,131,36,67), nrow = 4, # this makes a matrix with 4 rows
             byrow=T) # And this says that we've entered it row by row
 chisq.test(x)</code></pre>
@@ -559,21 +532,26 @@ chisq.test(x)</code></pre>
 ## 
 ## data:  x
 ## X-squared = 0.66724, df = 3, p-value = 0.8809</code></pre>
+<p>The answers don’t match exactly (because of the rounding I did in computing the expected counts), however the payoff/interpretation is the same: we cannot reject the null hypothesis of no relationship between region and direction opinoin in the sample.</p>
+</div>
 </div>
 <div id="empirical-questions" class="section level2">
 <h2>Empirical Questions</h2>
-<p>EQ0.</p>
+<div id="eq1" class="section level3">
+<h3>EQ1</h3>
 <ol style="list-style-type: lower-alpha">
 <li><p>The unit of analysis is the customer. The dependent variable is the type of board purchased and the independent variable is gender. Males, females, and unkown gender customers are being compared. This is a two-way test.</p></li>
-<li><p>The null hypothesis is that the board purchased is independent of the gender of the customer. The alternative hypothesis is that if we know the gender of the customer that will tell us something about the type of board they purchased.</p></li>
-<li><p>A <span class="math inline">\(\chi^2\)</span> test found statistically significant evidence that board purchase behavior differs by gender. This difference is convincing, but it does directly not tell us what we really want to know, which is the difference between men and women. It could be possible that it is simply identifying a significant difference in the number of unknown gender customers across board types. Many of these concerns are addressed in the text and with additional tests, giving increased confidence in the reality of this difference.</p></li>
-<li><p>Statistical tests help to give (or take away) confidence in a conclusion. People are not natively good at estimating how likely something is due to chance and tests help us to make these judgments. Choosing a statistical test is based on the question that you want to answer and the type of data that you have available to answer it. For example, if this were numeric data (e.g., the amount of money spent on electronics for men and women) then we could choose a t-test to compare those distributions.</p></li>
+<li><p>For this type of comparison statistical tests help to give (or take away) confidence in any observed differences across categories. Choosing a statistical test is based on the question that you want to answer and the type of data that you have available to answer it. For example, if this were numeric data (e.g., the amount of money spent on electronics for men and women) then we could choose a t-test to compare those distributions.</p></li>
+<li><p>The null hypothesis (<span class="math inline">\(H_0\)</span>) is that the board purchased is independent of the gender of the customer. The alternative hypothesis (<span class="math inline">\(H_A\)</span>) is that board purchase choice is dependent on gender.</p></li>
+<li><p>A <span class="math inline">\(\chi^2\)</span> test found statistically significant evidence that board purchase behavior differs by gender. This difference is convincing, but it does directly not tell us what the authors set out to understand, which is the difference between men and women (the test could have identified a significant difference in the number of unknown gender customers across board types). Many of these concerns are addressed in the text and with additional tests, giving increased confidence in the observed differences.</p></li>
 </ol>
-<p>EQ1</p>
+</div>
+<div id="eq2" class="section level3">
+<h3>— EQ2</h3>
 <ol style="list-style-type: lower-alpha">
-<li><p>These are counts for two categorical variables, so the procedure used was a <span class="math inline">\(\chi^2\)</span> test. The null hypothesis is that whether or not a blog is governed by one person is independent of whether it is on the left or the right.</p></li>
-<li><p>It would be surprising to see these results by chance and it make sense to believe that this difference is real. However, the main reason to be skeptical is the way that the data are grouped. The authors could have grouped them differently (e.g., 1-2 people, 3-4 people, and 5+ people); if the decision on how to group was made after seeing the data then we have good reason to be skeptical.</p></li>
-<li></li>
+<li><p>These are counts for two categorical variables, so the procedure used was a <span class="math inline">\(\chi^2\)</span> test. The null hypothesis is that whether or not a blog is governed by one person is independent of whether it is on the left or the right ideologically.</p></li>
+<li><p>Assuming that the null hypothesis of no difference across the two groups is compelling, it would be surprising to see these results in a world where ideological orientation and blog governance have no relationship. In this respect, it makes sense to believe that this difference is likely real. Perhaps the main reason to be skeptical is the way that the data are grouped. The authors could have grouped them differently (e.g., 1-2 people, 3-4 people, and 5+ people); if the decision on how to group was made after seeing the data then we have good reason to be skeptical.</p></li>
+<li><p>We can do this in R.</p></li>
 </ol>
 <pre class="r"><code># First we create the dataframe
 df = data.frame(Governance=c('Individual','Multiple', 'Individual','Multiple'),
@@ -587,7 +565,8 @@ chisq.test(matrix(df$Count, nrow=2))</code></pre>
 ## 
 ## data:  matrix(df$Count, nrow = 2)
 ## X-squared = 5.8356, df = 1, p-value = 0.01571</code></pre>
-<pre class="r"><code>percentage_data = df %&gt;% 
+<pre class="r"><code># Using Jeremy's tidyverse code:
+percentage_data &lt;- df %&gt;% 
   group_by(Ideology) %&gt;% 
   summarize(individual_ratio = sum(Count[Governance=='Individual']) / sum(Count),
             group_count = sum(Count))
@@ -599,23 +578,23 @@ shaw_benkler_plot = percentage_data %&gt;%
 
 shaw_benkler_plot</code></pre>
 <p><img src="data:image/png;base64,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" width="672" /></p>
-<pre class="r"><code># If we want to add error bars, we need to calculate them (Note that ggplot could do this for us if we had raw data - always share your data!)
-
-# I decided to use confidence intervals. The standard error is another reasonable choice
-
-# Remember that for a binomial distribution (we can consider individual/non-individual as binomial), confidence intervals are mu +- x * sqrt((p*(1-p))/n)
-
-ci_95 = 1.96 * sqrt(percentage_data$individual_ratio * (1 - percentage_data$individual_ratio)/percentage_data$group_count)
+<p>If we want to add error bars, we need to calculate them (Note that ggplot could do this for us if we had raw data - always share your data!).</p>
+<p>For our purposes here, Jeremy decided to use confidence intervals. The standard error is another reasonable choice. Either way, ggplot has a <code>geom_errorbar</code> layer that is very useful.</p>
+<p>Remember that for a binomial distribution (we can consider individual/non-individual as binomial), confidence intervals are <span class="math inline">\(\hat{p} \pm z^* \sqrt{\frac{\hat{p}~(1-\hat{p})}{n}}\)</span></p>
+<pre class="r"><code>ci_95 = 1.96 * sqrt(percentage_data$individual_ratio * (1 - percentage_data$individual_ratio)/percentage_data$group_count)
 
 shaw_benkler_plot + geom_errorbar(aes(ymin=(individual_ratio-ci_95)*100, ymax=(individual_ratio + ci_95)*100),
                                   alpha = .3, 
                                   size=1.1, 
                                   width=.4)</code></pre>
 <p><img src="data:image/png;base64,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" width="672" /></p>
-<p>The error bars do overlap in this case, but that actually doesn’t tell us whether they are significantly different.</p>
+<p>The error bars do overlap in this case, indicating that the true population proportions may not be as far apart as our point estimates suggest. Note that this is not the same as the hypothesis test.</p>
 <ol start="4" style="list-style-type: lower-alpha">
-<li>We don’t need to be very worried about the base rate fallacy because the sizes of both groups are about the same.</li>
+<li>On the one hand, we don’t need to worry about the base rate fallacy because the sizes of both groups are about the same and the paper does not abuse the evidence too egregiously. The base rate fallacy would likely come into play, however, in the ways that the results are (mis)represented. For example, you might imagine some news coverage looking at these results and claiming something (totally wrong!) like “Harvard study finds right wing blogs more than twice as likely to be solo affairs.” This is taking a relationship between the sample proportions (<span class="math inline">\(\hat{p}\)</span> in the language of our textbook) and converting that into a statement about the relationship between population proportions (<span class="math inline">\(p\)</span>). That would be a mess.</li>
 </ol>
+<p>Another way in which the base rate fallacy could play a role in this paper, however, concerns the presence of multiple comparisons. The authors conducted numerous statistical tests (indeed, one of the authors seems to recall that some of the tests were not even reported in the paper <Gasp!>) and they make no effort to address the baseline probability of false positives.</p>
+<p>In any case, the point here is that the statistical tests reported in the paper may not mean exactly what the authors said they did in the context of the publication. That may or may not change the validity of the results, but it should inspire us all to do better statistical analysis!</p>
+</div>
 </div>
 
 

Community Data Science Collective || Want to submit a patch?